You are on page 1of 780

FOURTH EDITION

PLATINUM
FOURTH ED IT ION
2 0 2 1

JAIME ALFONSO M. AHERRERA, MD


LAURO L. ABRAHAN IV, MD
MARC DENVER A TIONGSON, MD
DEONNE THADDEUS V. GAUIRAN, MD
MARC GREGORY Y. YU, MD
ENRICO PAOLO C. BANZUELA, MD

EDITOR
GERALDINE T. ZAMORA, MD
The printing of IM Platinum, Fourth Edition is financed by Top Practice
Medical Publishing Corporation, Manila, Philippines. Proceeds from the
purchase of this book will fund the development and improvement of
future editions of this book.

Please visit our Official Facebook Page, https://www.facebook.com/


IMPlatinum, for information on how to procure your IM Platinum book
at the lowest possible price. IM Platinum copies can be delivered to
your doorstep anywhere in the Philippines within 3-7 days after bank
payment. Unauthorized reproduction of this book is illegal. Beware of
counterfeits; an original IM Platinum copy contains glossy pages, not
matte or plain paper. You deserve to get the original.

Fourth Edition, 2021

Third Edition, 2018

Second Edition, 2015

First Edition, 2013

Copyright by Top Practice Medical Publishing Corporation http://www.facebook.com/IMPlatinum

Book design and layout by Manuel S. Vidal, Jr. All rights reserved. No part of this publication may
be reproduced, stored in a retrieval system, or transmitted, in any form by any means without
prior permission from Top Practice Medical Publishing Corporation.

Published by:

Top Practice Medical Publishing Corp.


No. 47 Mercury Street, Fairview, Quezon City
Contact no. +63918-807-6700

Printed in the Philippines.

-~
ISBN 978-621-95388-6-2

1" IM Platinum: The Official


Aherrera Notes
~Mf't.3it>num• Book
OBGYN Platinum
@obgy;ip!at!ntlrn• Book

II
ONE
PLATINUM Surgery Platinum
Book
I!]''

•-.-. •
@sulgef'iclilt!rlUlll

MEDICAL BOOKS
THE OFFICIAL DISTRIBUTOR Pedia Platinum
OF THESE TITLES @pcd,:,Pl;mnum Book
I!]~·:;.;

Philippine Medical
Jurisprudence
@phmeefiunsplat
• Bcok
I!] .

II
CONTRIBUTING
AUTHORS
Gene Philip Louie C. Ambrocio, MD, FAMP,FPCP,FPCCP
Marjorie Anne C. Bagnas, PTRP, MD, FPNA
Harold Henrison C. Chiu, RCh, MD
Jose Eduardo DL. Duya, MD, FPCP,FPCC
Antonio L. Faltado Jr., MD, FPCP,FPSEDM,MSc (Cand.)
Jonray R. Magallanes, MD, FPCP,FPCCP
Marion D. Patricio, MD, FPCP,FPCC
Katrina Angela Z. Reyes, MD, FPDS
Kenneth G. Samala, MD, MCMMO,FPCP,FPSMO
Aeden Bernice G. Timbol,· MD, FPCP,FPSG,FPSDE
Gelza Mae A. Zabat, MD,FPCP,FPSMID

ILLUSTRATORS
Joerelle V. Mojica, MD, FPCP,FPCC
Christian Paolo S. Vidal, RMT

CONTRIBUTING
WRITERS
FOR
BOARD
CORRELATES
Karl Phillip L. Avillo, MD

LAYOUT
EDITOR
Manuel S. Vidal, Jr.,RCh, MD

Ill
NOTICE
The information in this book has been reviewed and verified with
reliable sources, and the approaches to management have been utilized
in clinical practice. However, medicine is an ever-changing science. New
research, changes in guidelines, and human error occur. The authors,
editor, and publisher are not responsible for errors or omissions or
for any untoward outcomes from application of data in this book. The
authors, editor, publisher, and other parties who have been involved in
the preparation of this book make no warranty, expressed or implied,
with respect to the completeness, accuracy, or being up-to-date of all
the information contained in this publication. The authors, editor, and
publisher encourage the readers to confirm the information herein
with other sources, and to exercise critical thinking based on the clinical
presentation of the patient in making decisions for management.

IV
PREFACE
And it's finally here - the fourth edition of IM Platinum! Despite the ongoing
challenges of the COVID-19 pandemic, we trudged on to keep up with our
promise of providing a more enriched experience for everyone. Current
chapters were beefed up and expanded with easy-to-follow cases and the most
up-to-date guidelines. This edition features a brand new Critical Care chapter,
with detailed discussions on shock and respiratory failure; an entire section
devoted to the COVID-19 pandemic; and notable enhancements to the Labs
and IV Fluids chapters. As always, we thank you, our beloved readers, for your
unwavering support and continuous feedback, and for being with us every
step of the way. Go ahead, flip through the pages, and enjoy the ride!

ACKNOWLEDGEMENT
To the Lord Almighty, who makes all things possible;

To our families, for their unfailing love and support: Capt. Jaime Julian & Mrs.
Ma. Rosario Aherrera; Dr. Mary Anne Lim-Abrahan, Mrs. Marie Francesca
Abrahan-Benta, Mr. Edward Gabriel Abrahan; Mrs. Nancy Yu; Mrs. Maria
Cristina Tiongson, Dr. Lalaine, Mstr. Matthew Damian Tiongson, Atty. Virgilio,
Mrs. Marilyn, Mr. Patrick Jeanne & Ms. Patricia Meryl Tiongson, & Mr. Dennis,
Mrs. Maita, Ms. Maiden Kristine & Mr.Andrei Nikolai Tiongson; Engr. Deogracias
& Mrs. Thelma Gauiran; Mrs. Arabella Chiong-Banzuela, Mr. Zeus Banzuela,
Ms. Angelita Chiong, Mr. Nelson Cruz, Mrs. Epifania Cruz, Attorney Napoleon
Banzuela Jr., Dr. Rocky Lim & Ms. Arabella Aurora "Bogie" Lim Banzuela; and
Ms. Orlian Tong, Mr. Santi Litang, Mrs. Gem Zamora-Chua, Ms. Nala Racaza;

To all our mentors from the UP-PGH Department of Medicine, especially Dr.
Agnes Mejia, Dr. Rody Sy, Dr. John Afionuevo, Dr. Felix Eduardo Punzalan, Dr.
Eric Oliver Sison, Dr. Ester Penserga, Dr. Mary Anne Lim-Abrahan; Dr. Jose
Fernando Fontanilla, Dr. Maria Luz Querubin, Dr. Judy Fuentes, Dr. Rowena
Rivera, Dr. Marlyn David-Ruaro+, Dr. Ferdinand Francis Cid, Dr. Maria
Concellene Laforteza, Dr. Erlyn Sana, Dr. Alvin Mojica, Dr. Robert Arias, Dr. Rose
Ann Banal, Professor Jenny Zapf, Professor Bobbi Kurshan, Mr. Jojo Fresnedi,
Mrs. Rhodora Palomar-Fresnedi, Dr. Ramon Paterno, Dr. Rommel Duenas, Dr.
Pacifico Eric Calderon, Dr. Eric Calderon Jr., Dr. Mark Louie Mann for their
invaluable guidance and wisdom;

To our mentors from the De La Salle-HSI College of Medicine, especially Dr.


Romeo Ariniego, for inspiring us to strive for excellence for the glory of God;

To our colleagues in Internal Medicine and beyond, especially in the


Asian Hospital & Medical Center and UP Philippine General Hospital, for
enabling us to undertake this endeavor; to our students, for inspiring us to
be better educators everyday; and to our patients, our greatest teachers.

This book is lovingly dedicated to Dr. Ramon F. Abarquez, Jr., Mr. Gregorio
Yu, Jr., Dr. Lauro Abrahan, Jr., Atty. Virgilio Tiongson, and Denver's three little
angels in heaven.

Jaime, Sonny, Denver, Deonne, Greggy, Broli, and Ging


V
THE
AUTHORS
Jaime Alfonso M. Aherrera, MD, FAMP, FPCP, FPCC
Dr. Aherrera graduated with a B.S. Human Biology degree from De La Salle University-Manila and obtained
his medical degree from the De La Salle University-Health Sciences Institute, garnering awards including
the Dean's Special Award for Academic Performance and Special Citation for Academic Excellence. He
completed his residency training in Internal Medicine and fellowship in Cardiology and lnterventional
Cardiology at the University of the Philippines - Philippine General Hospital (UP-PGH), where he was Chief
Fellow and recipient of both the Dr. Ramon Abarquez Most Outstanding Fellow Award and the Dr. Clemente
Gatmaitan Most Outstanding Fellow in Research Award. He is currently completing his Masters Degree in
Clinical Epidemiology at the UP-Manila College of Medicine. He was also a board topnotcher of the Philippine
Physician Licensure Examinations in August 2009, Philippine Specialty Board of Internal Medicine given by
the Philippine College of Physicians in 2012, and the Special Board of Adult Cardiology Examination given by
the Philippine Heart Association in 2016. At a young age, he has already completed an impressive portfolio,
including four editions of IM Platinum, one of which was awarded as one of the Outstanding Books of2016 by
the National Academy of Science and Technology (NAST). He was also winner of the prestigious Dr. Francisco
Tangco Young Investigator's Award from the Philippine Heart Association for two consecutive years (2014
and 2015) and the recipient of the Most Outstanding Fellow in Cardiology Award of the Philippine Heart
Association in 2016. Recently, he was awarded as one of Ten Outstanding Young Scientists by the National
Academy of Science and Technology in 2020. He is a Fellow of both the Philippine College of Physicians
and the Philippine College of Cardiology, a Member of the Philippine Society of Cardiac Catheterization and
Interventions, and is presently practicing as an fnterventional Cardiologist at the Asian Hospital Medical
Center, UP-PGH, Manila Doctors Hospital, and Manila Med-Medical Center Manila. Despite these sterling
achievements, Jaime remains grounded, down-to-earth, and a loyal friend to his peers and colleagues.

Lauro L. Abrahan IV, MD, FPCP, FPCC


Dr. Abrahan earned his degree in B.S. Molecular Biology and Biotechnology, summa cum laude, at UP Diliman in 2005.
He obtained his medical degree in 2010 from the UP College of Medicine, finishing Class Salutatorian, cum laude.
He pursued further training at the UP-Philippine General Hospital (UP-PGH)from 2011-2019, completing residency
in Internal Medicine, followed by consecutive fellowships in Adult Cardiology and lnterventional Cardiology. Sonny
served as Chief Fellow for the senior years of both his Cardiology fellowship training programs. As a trainee, he
represented UP-PGH in numerous competitions, as well as published and presented multiple papers both in local
and international conferences. He topped his Diplomate Board Examination in 2017, eventually being awarded Most
Outstanding Cardiology Fellow in Training by the Philippine Heart Association that same year. He is currently an
active consultant at the Philippine General Hospital, St. Luke's Medical Center Global City, Manila Doctors Hospital.
and Manila Med. As the Training Officer for the Interventional Cardiology fellowship program at UP-PGH, he advocates
for prompt treatment of patients with heart attacks and other cardiovascular disorders, even during this time of
pandemic. During his limited free time, he enjoys watching Marvel movies, playing video games, swimming. and
guesting on his lovely wife's TikTok videos.

Marc Denver A. Tiongson, MD, FPCP, FPCC


Dr. Tiongson graduated with a Bachelor in Science in Biology, major in Cell and Molecular Biology, cum laude, from
the University of the Philippines-Los Banos in 2006. He pursued on to obtain his medical degree from the UP College
of Medicine in 2011 and was awarded outstanding intern in several clinical departments. Upon passing the medical
board exams, he became a faculty member of the San Beda College of Medicine and Topnotch Medical Board Prep
teaching Physiology, Biochemistry, Anatomy, Pharmacology and Pathology. Shortly after, he trained and completed his
residency in Internal Medicine at the Philippine General Hospital in 2015, during which he served as the Assistant Chief
Resident for Information Management System. He finished his Cardiology Fellowship and lnterventional Cardiology
Fellowship at the same hospital where he served as both the Chief Fellow for the Division of Cardiovascular Medicine
and Overall Chief Fellow for the Department. His interest in research garnered him awards in local and international
conferences. On top of this, he was able to publish his works in highly regarded scientific journals. He likewise co-
authored 08-Gyn Platinum, Pedia Platinum, and Surgery Platinum. Because of his extraordinary achievements he was
awarded the Dr. Ramon F.Abarquez Most Outstanding Fellow, Dr. Celemente Gatmaitan Most Outstanding Fellow in
Research, and Philippine Heart Association Most Outstanding Training Fellow in Cardiology for 2019. He is a Fellow
of both the Philippine College of Physicians and the Philippine College of Cardiology and a member of the Philippine
Society of Cardiac Catheterization and Intervention. He also held the position as the national representative of the
Philippines to the Heart Failure Association of the European Society of Cardiology - Heart Failure Specialists of
Tomorrow from 2018 to 2021. Currently, he is the Vice President of Topnotch Medical Board Prep and the chief
of clinics of Pamana Medical Center. He practices as an IM-lnterventional Cardiologist at the UP-Philippine General
Hospital. Manila Med, Medical Center Parafiaque, The Medical City South Luzon, Healthserv Los Bafios Medical Center,
Los Baiios Doctors Hospital, and Global Care Medical Center of Bay. Despite his seemingly busy schedule, Denver
remains to be a well-rounded guy and a proud family man who values time with his family especially his wife, Lalaine
and son, Matthew; and friends. His interests span video games, watching movies and binging on Nctflix.

VI
Deonne Thaddeus V. Gauiran, MD, FPCP, DPCHTM, DPSHBT
Dr. Gauiran graduated with a RS. Basic Medical Sciences degree (INTARMED), cum laude, from UP Manila in 2008. He
obtained his medical degree from the UP College of Medicine in 2011 (cum laude) and was awarded the Faculty Bronze
Medal for Academic Excellence. He was also a board topnotcher during the August 2011 Philippine Physician Licensure
Examination. He finished straight internship in Internal Medicine in 2011 and residency in Internal Medicine in 2014 from
the UP-Philippine General Hospital, where he received several awards in inter-hospital competitions. He was named Most
Outstanding Resident in Internal Medicine for three consecutive years and was also awarded by the Philippine College
of Physicians as Exemplar in Residency Training for 2015. He served as Chief Resident of the UP-PGH of Department of
Medicine for 2015. He underwent fellowship training in hematology at the UP-PGH Division of Hematology where he
also served as the Chief Fellow for 2018. He also had additional specialized training in immune-mediated hematologic
disorders in Vancouver General Hospital, University of British Columbia under the American Society of Hematology
Visitor Training Program. He is a Fellow of the Philippine College of Physicians and a Diplomate of the Philippine College
of Hematology and Transfusion Medicine and the Philippine Society of Hematology and Blood Transfusion. He is currently
practicing as a hematologist at the UP-PGH and the San Juan de Dias Educational Foundation, Inc. - Hospital. He enjoys
teaching and is also involved in organizing interhospital quiz bowls, postgraduate courses and fora, and review courses
in Internal Medicine. Despite his very busy schedule, he manages to balance his academic and professional life with his
personal interests like scuba diving, console gaming, reading, travelling, eating, and raising his cat, dogs, and fishes.

Marc Gregory Y. Yu, MD, FPCP, FPSEDM


Dr. Yu graduated with a B.S. Basic Medical Sciences degree (INTARMED) from UP Manila in 2008, finishing Class
Valedictorian, magna cum laude. He obtained his medical degree from the UP College of Medicine in 2011 where he
also finished Class Valedictorian, magna cum laude, and was further awarded Most Outstanding Medical Graduate and
Most Outstanding Intern in Internal Medicine. He finished residency training in Internal Medicine at the UP-PGH Medical
Center in 2014, where he received several awards in research and inter-hospital competitions, and was Team Captain of
the UP-PGH team that secured the national championship during the 20th PCP- Pfizer Interhospital Quiz Competition.
He completed fellowship training in endocrinology at the same institution last May 2017, where he was appointed Chief
Fellow and Chief of the Council of Chief Fellows, and was further awarded the Dr: Clemente Gatmaitan Award for Most
Outstanding Fellow in Research. Last September 2017, he was chosen by the Endocrine Society of the USAas one of22
recipients worldwide of the Early Investigator Award - becoming the first Filipino to do so. He is currently a research
fellow at the Joslin Diabetes Center in Harvard Medical School, having been awarded fellowships by the American
Diabetes Association and the Mary K. Iacocca Foundation. Dr. Yu was also Champion of the 2016 International Speaking
Competition for Young Endocrinologists, has published both fiction and nonfiction in several magazines and anthologies,
and was an occasional theatre reviewer for the Philippine Daily Inquirer. He was a recipient of the Don Carlos Palanca
Memorial Awards for Literature in 2004 and the Nick Joaquin Literary Awards for Fiction in 2010. Presently, he is a Fellow
of both the Philippine College of Physicians and the Philippine Society of Endocrinology. Diabetes, and Metabolism. In his
spare time, he dabbles in aviation, travel, new restaurants, history, classical music, and showtunes.

Enrico Paolo C. Banzuela, MD, MSEd, MHPEd, FPSP


Dr. Banzuela graduated with a B.S. Basic Medical Sciences degree (INTARMED) from UP Manila in 2002. He obtained
his medical degree from the UP College of Medicine in 2005. He finished his Masters in Educational Entrepreneurship
(MSEd) at the University of Pennsylvania in 2019. He obtained his Masters degree in Health Professions Education
(MHPEd) from the UP National Teacher Training Center for the Health Professions in 2020. He also a graduate of the
Management Development Program of the Asian Institute of Management in 2018. He also has a Postgraduate Certificate
in Teaching Evidence-Based Health Care at the University of Oxford in the United Kingdom in 2020. He is currently taking
a Masters in Health Professions Education (MHPE) at the University of Maastricht in the Netherlands. Upon passing the
medical board exams in 2005, he served as a volunteer physician for the 2005 Manila Southeast Asian Games. He also
worked as a University Researcher for the UP National Institutes of Health under the PhilHealth Research Study Group
for 3 years. He co-authored a book entitled "Survival Guide for Doctors (and Non-Doctors Too) with Dr. Willie Ong and
Dr. Liza Ong. He is also a co-author of IM Platinum, Surgery Platinum, Pedia Platinum, and Ob-Gyn Platinum. Dr. Banzuela
is the Chair of the Section of Medical Physiology (Associate Professor II) at the San Bed a University College of Medicine
and has been a faculty member since 2005. he has also served previously as lecturer for Biochemisty, Neurology, Family
and Community Medicine at the same institution. He is guest lecturer for Biochemistry at the Ateneo School of Medicine
and Public Health. He currently serves as Vice-President of the Philippine Society of Physiologists. He is the President of
Topnotch Medical Board Prep where he also teaches Physiology. Dr:Banzuela has always been involved with preparing
doctors for the medical board exams since 2005, having written guides, given orientation talks, created school-specific
board exam programs and lectured subjects at various hospitals and institutions. He enjoys fishing, driving, playing
basketball, listening to vinyl records, perfecting his aquaponic setup and going on food trips with his family.

THE
EDITOR
Geraldine T. Zamora, MD, FPCP, FPRA
Dr. Zamora graduated Class Valedictorian from both the University of Santo Tomas (B.S. Physical Therapy) and the
University of the Philippines College of Medicine. She then trained in Internal Medicine and Rheurnatology at the
Philippine General Hospital where she served as Chief Resident, and was eventually awarded Most Outstanding
Resident by the Philippine College of Physicians in 2010. She topped her four licensure examinations and was
only the third Filipino to receive the International Fellowship Grant by the Asia Pacific League of Associations for
Rheumatology, which brought her to Cedars-Sinai Medical Center in Los Angeles as a Postdoctoral Scientist in 2014.
She has co-authored, presented locally & abroad, and published several papers to date. Ging is presently with the
UP Manila COVID Study Group and heads the Philippine Vasculitis Study Group. As a trainee, she became the first
President of the UP MedRhythmics Dance Troupe and spearheaded fund raiser concerts & fashion shows for indigent
patients. She is presently the Vice President of the Hope for Lupus Foundation, and sits on the Board of the Sagip
Buhay Medical Foundation and the Asia Pacific Young Rheumatologists. She was selected as one of Philippines' The
Outstanding Young Men/Women (TOYM) in 2016 and The Outstanding Women for the Nation's Service (TOWNS) in
2019 for her contributions in Medicine as Science and Art. She has established a strong social media presence that
serves as a platform for her health tips, advocacies, and women empowerment.
Vil
CONTRIBUTING
AUTHORS
Gene Philip Louie C. Ambrocio, MD, FAMP, FPCP, FPCCP
Dr. Ambrocio graduated with a 8.S. Biology degree from UP Baguio and eventually pursued his medical degree at the
UP College of Medicine, finishing in 2009. He had his residency training in Internal Medicine, pursued his fellowship
in Adult Pulmonary Medicine, and served as the Chief Fellow of the Section of Pulmonary Medicine (2013-2014) at
the UP-PGH Medical Center. That same year, he was awarded as the Most Outstanding Pulmonary Fellow-in-Training
by the Philippine College of Chest Physicians. Despite his many responsibilities, he found time to do several research
papers that were presented in local and international conferences. He was also a recipient of the Young Investigator
Award given by the Asian Pacific Society of Respirology-European Respirology Society last 2016. He recently finished
his rotation in Lung Transplantation Medicine in Madrid, Spain to further his clinical experience in this field. Gene
is currently a Fellow of both the Philippine College of Physicians and the Philippine College of Chest Physicians. His
advocacies include smoking cessation programs and teaching.

Harold Henrison C. Chiu, RCh, MD


Dr. Harold Henrison Chiu graduated with a Bachelor of Science in Biochemistry, sum ma cum laude, from the College
of Arts and Sciences, University of the Philippines Manila in 2010 and placed 1st in the September Chemist Licensure
Examinations of the same year. He then pursued his medical education graduating as Class Valedictorian, cum laude,
Most Outstanding Medical Graduate, and overall Most Outstanding Intern from the UP College of Medicine. Following
his graduation from medicine, Harold was awarded as one of the Ten Outstanding Students of the Philippines
2016. He subsequently ranked 4th in the September 2016 Physician Licensure Examinations. He then pursued and
graduated from the residency training program of the Department of Medicine, Philippine General Hospital in 2019
where he served as Assistant Chief Resident for Postgraduate Training and was awarded the Dr. Augusto Camara
Award for Most Outstanding Resident in Research and Dr. Clemente Amante Award for Most Outstanding Resident
(Year Level 3). He placed 2nd in the Philippine Specialty Board in Internal Medicine (PSBIM) in March 2020. During
the same year, he was awarded the Philippine College of Physicians Most Outstanding Resident in Internal Medicine
(MORIM) 2020 and National Academy of Science and Technology Youth Leader 2020. He is currently the Chief Fellow
of the Division of Endocrinology, Diabetes and Metabolism, Department of Medicine, Philippine General Hospital and
is an Early Career Awardee of the US Endocrine Society. Outside his academic and research endeavors, Harold is an
active volunteer for the Sagip Buhay Medical Foundation and the Ten Outstanding Students of the Philippines Alumni
Association. In his free time, he spends time with his pet Sulcata tortoise, doing gardening, jigsaw puzzles, origami,
Chinese calligraphy and the practice of practical and workplace Feng shui. He has an active social media presence in
Twitter (@88doubledragon] and is a self-confessed fitness buff.

Jose Eduardo DL. Duya, MD, FPCP, FPCC


Dr. Joey Duya earned his Bachelor of Science in Public Health degree from the University of the Philippines Manila
in 2005, finishing Class Valedictorian, cum laude. He obtained his Doctor of Medicine degree from the UP College
of Medicine in 2010 where he finished First Honorable Mention, cum laude. He is a recipient of the multiple local
and international academic scholarships like the UPMASAscholarship and academic excellence scholarships. At a
young age, he discovered his passion for Internal Medicine and pursued a straight internship program in Internal
Medicine at the UP Philippine General Hospital. He finished his training in Internal Medicine from the same institution
in 2013 where he received numerous awards in various inter-hospital competitions and was further named Most
Outstanding Internal Medicine Resident for three consecutive years. In 2013, he was named as one of the Most
Outstanding Residents in Training by the Philippine College of Physicians. He pursued fellowship in Adult Cardiology
in UP PGH and served as a Chief Fellow and was awarded Most Outstanding Cardiology Fellow in PGH and of the
Philippine Heart Association in 2017. A multi awarded and published author, he is now the Head of the Research
Mentoring Enhancement Program of the PCP Research Committee and the Chief Research Coordinator of PCP CL.
He also dabbles as a peer reviewer of the British Medical Journal. Amidst all these, Joey is a well rounded and multi-
talented guy gifted with extra-ordinary artistic, teaching and hosting skills. He is one of the brains and brawn behind
IM PLATINUM 1,2,3 series.

Antonio L. Faltado Jr. MD, FPCP, FPSEDM, MSc (Cand.)


Dr. Faltado graduated with a B.S. Biology Major in Applied Biology degree from the De La Salle University-Dasmariflas
in 2005. He earned his Doctor of Medicine degree at the De La Salle Health Sciences Institute in 2009 where he was
awarded as one of the top 10 scholars; received both the Dean's Special Award for representing the school in interschool
competitions and the Academic Excellence Award in Internal Medicine; and was named Most Outstanding Junior Intern in
several rotations. He pursued his post-graduate internship, residency training in Internal Medicine and fellowship training
in Endocrinology, Diabetes and Metabolism at the UP-Philippine General Hospital (UP-PGH)and was awarded Outstanding
Intern in several rotations. A passionate researcher, Anton was able to present and publish a number of papers both locally
and internationally and received several research grants from the Philippine Council for Health Research and Development
and the Philippine College of Physicians. He contributed to the Philippine Covid-19 Living Clinical Practice Guidelines
as one of the evidence review experts. At present, he is completing his Master's Degree in Clinical Epidemiology at UP-
Manila and Master's degree in Healthcare Management at the Trinity University of Asia. He currently practices in Lipa City,
Batangas City and Rosario Batangas. He recently received the Most Outstanding Physician in Continuing Medical Education
2021 award given by the Lipa City Medical Society. During his spare time, he helps manage their family-owned businesses
-ANFA Royale Hotel and Palazzo Antonio Convention Center in Lipa City,Batangas.

VIII
Jonray R. Magallanes, MD, FPCP, FPCCP
Dr. JR Magallanes is currently working as a Pulmonologist-lntensivist at St. Luke's Medical Center· BGC and he is also a
COVIDconsultant volunteer at the Division of Pulmonary Medicine, Philippine General Hospital. Dr. Magallanes finished
his residency and fellowship training at UP-PGHfrom 2013-2017. Since his residency, he has been always been part of
the undergraduate academic and teaching service. He is continuing his passion for teaching as a Physiology lecturer at
the Ateneo School of Medicine and Public Health and as a medical field officer and a regular lecturer of the Respiratory
Division of AstraZeneca Philippines. He is an advocate of Inhaler Device Education and created aerosol science workshops
for pharmacists care of the Philippine Pharmacists' Association and the Mercury Drug Corporation. As a former Biology
major; he is an avid hiker and has a penchant for marine invertebrates. In his spare time, he likes to read history, evolution
and art history. He reads alot of nonfiction books with special interest in the linguistics, the austronesian culture and the
works of Johannes Vermeer. In his lengthy spare time, he plays alot ofDOTA 2, Cities Skylines and Civilization. He and his
bestfriend are planning a twitch channel on Civilization VI with a ton of history commentaries on the side.

Marion D. Patricio, MD, FPCP, FPCC


Dr. Marion Patricio is a practicing Cardiologist-lntensivist in Metro Manila and Laguna. He earned his Medicine degree
and Internal Medicine Residency training at Pamantasan ng Lungsod ng Maynila - Ospital ng Maynila. He then finished his
Fellowship in Adult Cardiology at UP- Philippine General Hospital (UP-PGH) and Critical Care Medicine at Philippine Heart
Center (PHC). He currently serves as Vice President of the Philippine Heart Association Southern Tagalog Chapter. He helps
with training in Critical Care in Manila Doctors Hospital, Medical Center Manila and PGH. He is also a member of the Heart
Transplant Team of PHC. He devotes his time to his family; his wife Joan and sons Aram and Rocco. In his free time he loves
to practice playing Classical Piano pieces by Chopin and Mozart among others; especially playing Flight of the Bumblebee
to friends for showmanship. His favorite mobile game is Summoner's War achieving Guardian status in the Arena.

Katrina Angela Z. Reyes, MD, FPDS


Dr. Reyes graduated cum laude from the Integrated Liberal Arts and Medicine (INTARMED) program of the University
of the Philippines College of Medicine in 2010. She was chief resident in her final year at the Philippine General
Hospital Section of Dermatology and was a visiting fellow at Yonsei University - Severance Hospital, Seoul. She is
currently pursuing her Executive Master in Business Administration from IE Business School, Madrid. Dr. Reyes is
the owner and dermatologist of Refinity Dermatology and Laser Clinic in Kapitolyo, Pasig and is the Laser Center
Head of the Rizal Medical Center Skin Center. In the midst of the rapidly evolving field aesthetics, she prides herself
with her strong foundation of pathologic dermatology and mastery of Asian skin at the heart of her practice. Since
the pandemic, she has been trying her luck in swimming, and of course, she never forgets to apply her sunscreen!

Kenneth G. Samala, MD, MCMMO, FPCP, FPSMO


Dr. Kenn Samala graduated from UP Manila in 2006 with a degree in BS Public Health and obtained his medical
degree from the UP College of Medicine in 2011, where he was awarded the prestigious Oreta-Dizon-Santos-Ocampo
Research Award for his work on a rapid diagnostic test for Multi-Drug Resistant Tuberculosis. He finished both
his post-graduate internship program as a straight intern and residency training in Internal Medicine at the UP
Philippine General Hospital (UP-PGH) where he received several research awards, including the Philippine College
of Physicians' Young Investigators Award in 2013 and the Dr. Antonio J.Gonzaga Research Award in 2012 and 2014.
He also completed his fellowship training in Medical Oncology and earned a Master's degree in Clinical Medicine
(Major in Medical Oncology) at UP-PGH where he served as the Chief Fellow for External Affairs. He went on to further
training and did his Thoracic Oncology Observership at Memorial Sloan Kettering Cancer Center in New York City,
USA. In his spare time, he takes delight in tending to his plants and taking care of his pet beagle. Kenn is also a self-
confessed musicphile and although the consumption of music has evolved in the past few years where streaming on
different platforms is readily available and accessible, he finds joy listening to his vinyl record collection which stirs
up nostalgia and reminds him that life is not meant to be rushed and that every moment must be savored to the fullest.

Aeden Bernice G. Timbol, MD, FPCP, FPSG, FPSDE


Dr. Timbol graduated with a B.S. Biology degree, cum laude, from the University of Santo Tomas in 2006. She obtained
her medical degree from the UST Faculty of Medicine and Surgery in 2010, finishing magna cum laude. She then pursued
residency in Internal Medicine at the UP-Philippine General Hospital (UP-PGH), continuing with fellowship training
in Gastroenterology and Hepatology where she served as Chief Fellow. As a trainee, she has produced a number of
researches and international publications for which she received several accolades including the Philippine Heart
Association Young Investigator Award. In 2017, she finished her Gastroenterology fellowship and was awarded both
the Most Outstanding Gastroenterology Fellow-in-Training by the Philippine Society of Gastroenterology/Philippine
Society of Digestive Endoscopy, as well as the Dr. Ramon F. Abarquez Most Outstanding Fellow by the UP-PGH
Department of Medicine. Consistent with her accomplishments, Aeden also topped the d!plomate board examination in
Gastroentero\ogy. In her spare time, she loves to eat out, watch movies, and play video games and table tennis.

IX
Gelza Mae A. Zabat, MD, FPCP, FPSMID
Dr. Gelza Mae A. Zabat graduated from UP Diliman with a Bachelor of Science degree in Molecular Biology and
Biotechnology. She completed her medical degree in the University of the Philippines College of Medicine. She
specialized in Internal Medicine in UP- Philippine General Hospital and proceeded to subspecialize in Infectious
Diseases and Tropical Medicine in St. Luke's Medical Center- Quezon City. She is an active infectious disease
consultant in several hospitals, as well as an assistant professor in the University of the East Ramon Magsaysay
Memorial Medical Center. She is a site Principal Investigator of the WHO COVID-19 Solidarity Treatment Trial, as well
as several trials including that of COVID-19 vaccines. Despite the workload, she is a Netflix addict (including some
K-dramas), a closet Tagalog movie fan and a proud member of BTS ARMY:-). Borahae!

Jan Melvin M. Zapanta, MD, FPCP, FPSN


Dr. Zapanta graduated from UP Manila with a degree in BS Biology, finishing cum laude, and completed both his medical
degree and internship at the UP College of Medicine - Philippine General Hospital. He specialized in Internal Medicine
and further subspecialized in Adult Nephrology in the same institution, after which he topped both the written and
oral diplomate board examinations of the Philippine Society of Nephrology (PSN) in 2016. He was recommended by
the PSN as one of the promising young nephrologists to represent the society in the Asia Pacific Society of Nephrology
Education meeting in 2017. He currently heads the PSN Committee on Patient Protection, and has been actively training
future internists and nephrologists back in his hometown province of Pampanga. He also serves as a unit head of several
hemodialysis units aside from doing his private practice in nephrology. During his free time, he loves to play and chill
with his two dogs, Chino and Max, his ultimate stress busters. Watching random Youtube videos and browsing social
media posts are his time killers. He also enjoys collecting Lego toys, reading fantasy books and comics, and traveling.

ILLUSTRATORS
Joerelle V. Mojica, MD, FPCP, FPCC
Dr. Joerelle Mojica completed her Internal Medicine residency at Manila Doctors Hospital in 2014, Cardiology
fellowship at Philippine General Hospital in 2018, and Cardiac Electrophysiology and Pacing fellowship at St Luke's
Medical Center in 2019. She is currently based in Brussels, Belgium for further Cardiac Electrophysiology and Pacing
training in Vrije Universiteit Brussel - Universitair Ziekenhuis. She focuses on new ablation technologies for atrial
fibrillation and other cardiac arrhythmias, and novel pacing modalities such as leadless pacemakers and left bundle
branch pacing. She finds balance in learning new recipes, exploring places in Europe, practicing Dutch and French
languages, and watching HIMYM and Friends. She misses her family and friends in the Philippines very much and
hopes to see them all soon.

Christian Paolo S. Vidal, RMT


Mr. Paolo Vidal is a PRC~licensed medical technologist currently undertaking his master's degree in public health
at the University of the Philippines Manila, under the DOST-ASTHRDPscholarship program. He is a recipient of the
University of the Philippines International Publication Award in 2020. At present, he works as a teaching associate
at the College of Public Health, University of the Philippines Manila. When not working. Paolo likes to listen to music,
sleep, and pester his dog, Franky. He is thankful for the invitation to contribute to the Platinum book series.

CONTRIBUTING
WRITER
FOR
BOARD
CORRELATES
Karl Phillip L. Avillo, MD
Dr. Karl Avilla graduated with a degree in B.S. Nursing. cum laude, from Southern Luzon State University in 2012
and placed 7th in the June Nurse Licensure Examination in the same year. He obtained his medical degree from West
Visayas State University - College of Medicine in 2016, finishing Class Valedictorian, cum laude. He then ranked 3rd
in the September 2017 Physician Licensure Examination. He pursued residency training in Internal Medicine at UP-
Philippine General Hospital from 2018-2020 and represented the department in several quiz bowls. He placed 1st in
the Philippine Specialty Board oflnternal Medicine (PSBIM) in March 2021. He loves an early morning run to start the
day. He enjoys teaching. When he is not working. he spends his downtime dabbling in the stock and cryptocurrency
markets, strumming his guitar and playing Call of Duty mobile or NBA 2K mobile.

LAYOUT
EDITOR
Manuel S. Vidal, Jr., RCh, MD
Dr. Mavi Vidal graduated with a BS Biochemistry degree, magna cum laude, from the De La Salle University- Manila under
the Star Scholarship Program, with several other awards: Excellence in Chemistry, Most Outstanding Thesis, and Dr. Jose
Rizal Honors Society inductee. He ranked second in the Chemistry Licensure Examination in 2014. He finished his medical
studies at the University of the Philippines Manila - College of Medicine, and is about to embark on his PhD dissertation
studies on organ-on-chip models at the UniversityofTexas Medica\ Branch, Galveston, Texas.He dreams on becoming a good
08-GYN in the future. If not busy, he enjoys the laid-back life. collecting fragrances, sipping coffee, drinking beers,
and cooking wonderful food for his family and friends. He is thankful for the opportunity to continue working for the
Platinum series.
X
TABLE
OFCONTENTS
CHAPTER
1:INTRODUCTION
TOINTERNAL
MEDICINE
Section 1: Approach to Patients in Internal Medicine
History Taking
Physical Examination 4
Section 2: Rotating in the Wards 5
Writing the Problem List 5
Making Daily Rounds s
The "S-O-A-P" Format 5
Writing the Diagnostic & Management Plan 6
Presenting the Case 6

CHAPTER
2:BASIC
DIAGNOSTICS
Section 1: Basic Terminology in Diagnostic Testing 9
Overview of Laboratory Medicine 9
Conversion Factors of Routine Laboratory Tests 10
Section 2: Normal Laboratory Values 11
Complete Blood Count 11
Coagulation Tests 13
Serum Chemistry and Related Tests 14
Hormones and Biomarkers 18
Urine Studies 20
Stool Analysis 23
Section 3: Sterile Fluids and Pathologies 25
Pleural Effusion (Pleural Fluid) 25
Ascites (Peritoneal Fluid} 27
Cerebrospinal Fluid 30
Pericardial Fluid 32
Synovial Fluid 33

CHAPTER
3:ELECTROCARDIOGRAPHY
Section 1: Basic Concepts in Electrocardiography 37
Section 2: Basic Steps in ECG Reading 40
Step 1: Determine Heart Rate 40
Step 2: Determine Rhythm 41
Step 3: Measure Intervals 46
Step 4: Determine QRS Electrical Axis 47
Step 5: Check for Chamber Enlargements 48
Step 6: Check for ST and T-wave Changes 49
Step 7: Check for Miscellaneous ECG Findings 52
Section 3: Treadmill Exercise Stress Test 54
Section 4: Pacemakers & Pacemaker Rhythms 57

CHAPTER
4:ARTERIAL
BLOOD
GAS
Section 1: Overview of Arterial Blood Gas 61
Indications and Contraindications 61
Procedure and Technique 61
Section 2: Arterial Blood Gas interpretation 62
Basic Steps in ABG Interpretation 62
Sample Cases 65
Section 3: Evaluating Oxygenation Parameters 69
Indices of Lung Function & Blood Oxygenation 69
Sample Cases 71
Section 4: Correlation with Venous Blood Gas 74

CHAPTER
5:BASIC
IMAGING
Section 1: Chest Radiograph Interpretation 77
Section 2: Plain Abdominal Radiography 88
Section 3: Basic Computerized Tomography (CT) Imaging 89

CHAPTER
6:BASIC
PROCEDURE
Nasogastric Tube Insertion 95
Intravenous Line Insertion 96
Foley Catheter Insertion 97
Endotracheal Intubation via Direct Laryngoscopy 98
Thoracentesis 99
Pericardiocentesis 100

XI
TABLE
OFCONTENTS
Arthrocentesis 102
Lumbar Tap 104
Abdominal Paracentesis 106
Central Line Insertion (lntrajugular) 107
Arterial Line Insertion (Radial) 108
Bone Marrow Aspiration 109
Needle Decompression (Needling) 110

CHAPTER
7:IVFLUIDS
6DRIPS
Section 1: Intravenous Fluids 113
Section 2: Common Intravenous Drips 115
Formulation & Computation of Basic Drips 115
Using Infusion Pumps 116
Overview of Common Drips 117
Sample Cases 121
Section 3: Blood Products & Transfusion 124
Rational Use of Blood Products 124
Blood Typing 127

CHAPTER
8:ELECTROLYTES
Section 1: Overview & General Formulas 131
Section 2: Fluids and Electrolytes 135
Water Balance 135
Sodium 136
Potassium 141
Calcium 146
Magnesium 150
Bicarbonate 150

CHAPTER
9:PERIOPERATIVE
EVALUATIDN
Section 1: Approach to Perioperative Cardiac Evaluation 153
Section 2: Approach to Perioperative Pulmonary Evaluation 160

CHAPTER
10:CARDIOLOGY
Section 1: Approach to Diseases of the Cardiovascular System 165
Approach to Common Cardiovascular Complaints 165
Common Diagnostic Tests in Cardiology 171
Section 2: Dyslipidemia 172
Section 3: Hypertension 176
Section 4: Heart Failure 184
Section 5: Chronic Coronary Syndromes 193
Section 6: Acute Coronary Syndromes 198
Non•ST•Elevation Acute Coronary Syndrome 204
ST•Elevation Myocardial Infarction 205
Section 7: Rheumatic Fever and Valvular Heart Disease 207
Section 8: Venous Thromboembolism 210
Section 9: Other Disorders of the Cardiovascular System 218
Atrial Fibrillation 218
Pericarditis 221
Cardiac Tamponade 223
Peripheral Artery Disease 224

CHAPTER
11:PULMONOLOGY
Section 1: Approach to Diseases of the Pulmonary System 229
Approach to Common Pulmonary Complaints 229
Common Diagnostics in Pulmonology 233
Section 2: Bronchial Asthma 234
Section 3: Chronic Obstructive Pulmonary Disease 246
Section 4: Pneumonia 254
Community·Acquired Pneumonia 254
Hospital-Acquired & Ventilator·Associated Pneumonia 258
Section 5: Tuberculosis 259
Section 6: Respiratory Failure and ARDS 268
Section 7: Other Disorders in Pulmonology 271
Disorders of the Pleura 271
Pulmonary Nodules 272

XII
TABLE
OFCONTENTS
CHAPTER
12:CRITICAL
CARE
MEDICINE
Section 1: Shock 277
Hemodynamics and Shock 277
Sepsis & Septic Shock 282
Cardiogenic Shock 285
Selecting Vasopressors & lnotropes 287
Section 2: Respiratory Support & Mechanical Ventilation 293
Oxygen Delivery 293
High-Flow Nasal Cannula (HFNC) 295
Non•invasive Positive Pressure Ventilation 296
Invasive Mechanical Ventilation 298
Spontaneous Breathing Trial & Weaning 305
Section 3: Advanced Cardiac Life Support 308
Adult Cardiac Arrest Algorithm 308
Adult Bradycardia Algorithm 309
Adult Tachycardia with a Pulse Algorithm 309
Advanced Therapeutic Modalities 310

13:INFECTIOUS
CHAPTER DISEASES
Section 1: Approach to Common Complaints 313
Fever 313
Fever of Unknown Origin 314
Approach to Fever and Rash 314
Section 2: Overview of Available Antimicrobials 317
Section 3: Common Bacterial Infections 325
Leptospirosis 325
Typhoid Fever 329
Tetanus 331
Infective Endocarditis 333
Urinary Tract Infection 337
Urinary Tract Infection in Pregnancy 341
Section 4: Common Viral Infections 342
Coronavirus Disease 2019 (COVID-19) 342
Dengue 348
Rabies 353
Human Immunodeficiency Virus Disease 355
Section 5: Common Parasitic and Fungal Infections 358
Malaria 358
Schistosomiasis 360
Candidiasis 361
Section 6: Sexually Transmitted Diseases 363
Section 7: Immunization 365

CHAPTER
14:GASTROENTEROLOGY
Section 1: Approach to Diseases in Gastroenterology 371
Approach to Gastrointestinal Complaints 371
Common Diagnostics in Gastroenterology 374
Section 2: Diarrhea and Constipation 376
Section 3: General Diseases of the GI Tract 380
Gastroesophageal Reflux Disease 380
Peptic Ulcer Disease 382
Diverticular Disease 387
Section 4: Gastrointestinal Bleeding 388
Section 5: Diseases of the Liver 393
Viral Hepatitis 397
Alcoholic Liver Disease 401
Non-alcoholic Fatty Liver Diseases 403
Liver Cirrhosis 405
Section 6: Diseases of the Biliary Tree and Pancreas 410
Gallstone Disease 410
Acute Pancreatitis 412
Section 7: Overview of the Basics in Nutrition 418
Section 8: Types of Nutrition 423
Enteral Nutrition 423
Parenteral Nutrition 425

XIII
TABLE
OFCONTENTS
CHAPTER
15:NEPHROLOGY
Section 1: Approach to Diseases in Nephrology 429
Approach to Complaints in Nephrology 429
Common Formulas in Nephrology 432
Clues for Diagnosis of Major Syndromes 436
Section 2: Acute Kidney Injury & Chronic Kidney Disease 438
Acute Kidney Injury 438
Chronic Kidney Disease 444
Renal Replacement Therapy 452
Section 3: Other Disorders in Nephrology 453
Glomerular Diseases 453
Nephrolithiasis 453
Renal Tubular Defects 456
Urinary Tract Obstruction 457
Section 4: Overview of Extracorporeal Therapy 459

CHAPTER
16:ENDOCRINOLOGY
Section 1: Approach to Diseases in Endocrinology 465
Approach to Common Complaints in Endocrinology 465
Physical Examination 465
Section 2: The Metabolic Syndrome and Diabetes Mellitus 467
The Metabolic Syndrome 467
Diabetes Mellitus 468
Hyperglyccmic Crises in Diabetes 477
Diabetic Foot Ulcer 483
Diabetes Mellitus and Pregnancy 485
Hypoglycemia 486
Section 3: Thyroid Disorders 487
Hyperthyroidism 487
Thyroid Storm 490
Hypothyroidism 492
Goiter and Nodular Thyroid Disease 494
Section 4: Disorders of the Adrenal Glands 495
Cushing Syndrome 495
Adrenal Insufficiency 497
Mineralocorticoid Excess 498
Section 5: Other General Endocrinologic Disorders 500
Pituita1y Diseases 500
Osteoporosis 501

CHAPTER
17:RHEUMATOLOGY
Section 1: Approach to Rheumatologic Complaints 507
History of a Patient with a Rheumatologic Complaint 507
Physical Examination 509
Diagnostics in Rheumatology 511
Section 2: Rheumatologic Disorders 513
Osteoarthritis 513
Gouty Arthritis SIS
Rheumatoid Arthritis 518
Infectious Arthritis 521
Systemic Lupus Erythematosus (SLE) 523
Antiphospholipid Syndrome (APS) 525

CHAPTER
18:IMMUNOLOGY
Section 1: Basic Concepts in Immunology 529
Section 2: Common Conditions in Allergology & Immunology 530
Urticaria & Angioedema 530
Allergic Rhinitis 531
Atopic Dermatitis 535
Anaphylaxis 536

XIV
TABLE
OFCONTENTS
CHAPTER
19:HEMATOLOGY
Section 1: Approach to Common Hematologic Complaints 541
Approach to Hematologic Complaints 541
Blood Components 541
Diagnostics in Hematology 542
Common Computations & Formulas in Hematology 546
Findings in Peripheral Blood Smear 548
Common Antiplatelets, Anticoagulants, & Fibrinolytics 550
Section 2: Anemia 551
Section 3: Bleeding 556
Thrombocytopenia 557
Coagulopathies and Related Disorders 558
Section 4: Bone Marrow Failure and Malignancies 560
Bone Marrow Failure 560
Hematologic Malignancies 561

CHAPTER
20:DERMATOLOGY
Section 1: Approach to Patients in Dermatology 565
Approach to the Patient with a Skin Disorder 565
Morphology of Skin Lesions 565
Common Diagnostic Modalities 568
Section 2: Common Cases in Dermatology 569
Acne Vulgaris 569
Contact Dermatitis 571
Psoriasis Vulgaris 573
Hansen Disease (Leprosy) 574
Stevens-Johnson Syndrome & Toxic Epidermal Necrolysis 578
Other Cases in Dermatology 580

CHAPTER
21:NEUROLOGY
Section 1: Approach to Patients in Neurology 583
Diagnostic Catechism 583
Approach to Common Complaints in Neurology 586
The Neurologic Examination 589
Section 2: Cerebrovascular Disease 595
Early Specific Management of lschemic Stroke 597
Early Specific Management of Hemorrhagic Stroke 600
Blood Pressure Management for Acute Stroke 601
Primary and Secondary Prevention of Stroke 602
Subarachnoid Hemorrhage 603
Section 3: Seizure and Epilepsy 605
Section 4: Infections of the Central Nervous System 608
Bacterial Meningitis 608
Tuberculous Meningitis 610
Brain Abscess 611

CHAPTER
22:ONCOLOGY
Section L Introduction to Oncology 615
Basic Concepts in Oncology 615
Epidemiology and Etiology of Cancer 617
Section 2: Cancer Screening 619
Cancer Screening Guidelines 619
Commonly Used Tumor Markers 620
Section 3: Overview of the Management of Cancer 622
Section 4: Oncologic Emergencies 626
Superior Vena Cava Syndrome 626
Venous Thrornboembolisrn 627
Other Oncologic Emergencies 627

629
CHAPTER
23:BOARD
CORRELATES

xv
INTRODUCTIO
TO
INTERNAL
MEDICIN
-~ APPROACH TO PATIENTS IN INTERNAL MEDICINE

1. History Taking

2. Physical Examination

0 ROTATING IN THE WARDS

1. Writing the Problem List

2. Making Daily Rounds

3. The "S-O-A-P" Format

4. Writing the Diagnostic & Management Plan

5. Presenting the Case


INTRODUCTION
SECTION
APPROACH TO PATIENTS IN INtERNAL MEDIGINE
ONE

Internal Medicine (IM) can be quite overwhelming because of the complexity of cases and

long work hours. Despite these inherent toxicities, it remains one of the most rewarding fields
in Medicine. Students and practitioners alike enjoy the intellectual stimulation and experience
of translating theoretical knowledge into direct patient care. As basic IM principles cannot be
dissociated from the cases we encounter, it is imperative for every practitioner to acquire the core
competencies and skills of an internist. The approach to patient encounter and chart writing are
discussed in the succeeding parts.

HISTORY TAKING
Good history-taking is central to good patient care
• The steps to performing a complete history are outlined below

I. COMPONENTS OF A PATIENT'S HISTORY


Chief complaint • The main reason for the patient's consult or admission

History of • Includes details of the patient's chief complaint


present illness • Problems are elicited from the patient in chronological order

Review of • Runs through all organ systems for symptoms the patient may have
systems failed to mention
• Probes further into the patient's other medical conditions, including
Past medical
present medications, other co-morbidities, past surgeries, and any
history
food/drug allergies
• Looks for the presence of diseases in the family such as hypertension
Family medical
(HPN), diabetes mellitus (DM), heart disease, early cardiac death,
history
atopy, and autoimmune disease

Personal and • Includes the patient's dietary habits, smoking history, alcohol intake,
social history illicit drug use, travel history, and if relevant, sexual history

Obstetric and • Female patients should be asked about details on menstruation and
gynecologic pregnancy

II. ATTRIBUTES OF A SYMPTOM: PQRST


• PQRST is an acronym, usually for the assessment of"pain" or "discomfort"
• Pain is one of the most commons atients to seek consult

Provocation/ • What provokes the symptom? What makes the pain worse?
palliation • What palliates or relieves the symptom?
Quality • Description of the pain (e.g., dull, aching, sharp, heavy, tingling, burning)
, Radiating pain: spreads from the source of the pain (e.g.,in sciatica, pain
Radiation or is felt shooting down the leg as far as the toes)
Referred • Referred pain: felt in a location different from the actual cause of the pain
(e.g., cholecystitis may cause right shoulder pain)
• An accurate way to measure severity is to score the pain on a scale of I to
Severity
10 (with JO points being the worst pain)
• When did the pain start? When does it happen? Is it constant or
Timing
intermittent (comes and goes)?
3
PHYSICAL EXAMINATION
History-taking is followed by a physical examination (PE). Permission should always be asked
from the patient before doing any maneuver, especially the more intrusive ones. A systematic PE
starts with a general survey of the patient followed by measurement of the patient's vital signs
and anthropometrics. Special focus is then given to certain body parts pertinent to the identified
problems of the patient. Specific details on PE findings per organ system are discussed in each of
the succeeding subspecialty chapters.

• Check the patient's level of consciousness and general state of nutrition,


General survey and look for any signs of distress
• Inspect the patient's skin and hair

Vital signs • Take patient's pulse rate, respiratory rate, blood pressure, & temperature
Anthropometrics • Take patient's weight and height, and compute for the BM!
• Check the sclerae, conjunctivae, pupillary light reflex, and extraocular
muscles (tests for visual acuity, tonometry, and fundoscopy may be
needed for patients with ophthalmologic complaints)
• Grossly examine the outer ear, nose, and oral cavity (otoscopy, runing
Head and neck fork tests, and rhinoscopy may be needed for patients with complaints
specific to these organs)
• Check for neck vein engorgement & measure jugular venous pressure
• Auscultate for bruits over the carotid artery and thyroid gland
• Palpate neck for presence of goiters, enlarged lymph nodes, & masses

• Inspect the chest for deformities, retractions, and masses


• Palpate for areas of tenderness, respiratory excursion, tactile fremiti, and
crepitations
Chest and lungs • Percuss in proper sequence & listen for differences in percussion sounds
• Auscultate both the anterior and posterior chest, noting the character of
breath sounds as well as for adventitious breath sounds; auscultate for
vocal fremiti as well

• Inspect the precordial area for bulging and identify the point of maximal
impulse (PM!)
Cardiovascular • Palpate for the apex beat as well as for any heaves and thrills
system • Auscultate using both the bell and diaphragm and note the cardiac rate
and rhythm, the character of St and S2, the presence of S3 or S4, and the
character of any murmurs present
• Inspect the abdomen's shape and contour, as well as for any visible
masses, scars, pulsations, and discolorations
• Auscultate for the character and frequency of bowel sounds, as well as
Abdomen for bruits and succussion splash
• Palpate the abdomen using both light and deep palpation, noting the
presence of tenderness, guarding, organomegaly, and masses
• Percuss for the liver span, Traube's space, and shifting dullness

• Inspect for the presence of cyanosis, clubbing, edema, and tremors


• Palpate and characterize arterial pulses on both sides
Extremities
• Check for joint swelling, erythema, warmth, tenderness, and limitation
of motion

• Neurological exam for patients with altered sensorium or those with


complaints involving the neurologic system
Others • Mental status exam for psychiatric patients
• Gynecologic exam for pregnant patients or female patients with
complaints involving the urogenital system

4
SECTION TWO
ROTATING IN liME . :ARDS
WRITING THE PROBLEM LIST
The problem list is a list that presents a current, concise picture of all of a patient's
medical problems and significant health factors
Gives the medical care team a quick yet comprehensive overview of the patient's health
care needs, thus enabling better follow up of the patient's case
Provides a readily accessible database from which epidemiologic data can be drawn
I. CHARACTERISTICS OF A GOOD PROBLEM LIST
Complete
Prioritized
Specific without being overly redundant
Dynamic (once a problem resolves it should be removed)

II. TIPS ON HOW TO MAKE A GOOD PROBLEM LIST


• First perform a thorough history & physical examination, and review all labs available
• List all problems identified & process into accurate terminologies guided by:
• Use terms that best define and encompass each problem.
• Use modifiers to better characterize each problem in terms of the following: severity (mild,
moderate, severe); duration (acute, chronic); and cause (primary, secondary)
• If unsure of a particular diagnosis or problem, labels such as "suspected" or "probable" may be
used. A "rule out" (R/0) designation is rarely needed but may denote a competing diagnosis that
is still a possibility pending the results of work-up.
• For each problem, place a corresponding date and ICD code if possible.
• Symptoms may be rarely used if it is not currently possible to describe the problem at a
higher diagnostic level.
• Group common/related problems. There could be an umbrella problem, and a list of sub-
problems under that.
• Classify problems into active or inactive (usually prior, resolved medical/surgical conditions
that are still important to be remembered).
• Arrange problem list in order of priority, with the most active problems first. The topmost line
usually reflects the problem that brought the patient in for consult (i.e., the chief complaint).
• Review the problem list regularly. As the patient's diagnosis and labs are being updated
regularly, update the problem list as well to reflect that.

MAKING DAILY ROUNDS


Given the limited time and the multitude of patients in the hospital, making daily rounds efficiently
and effectively is an important internist skill. Here are a few tips on how to do daily rounds like a pro:
Start with a quick review of the patient's past problems, which includes reviewing the
1.
previous day's chart entries.
2. Identify present problems at bedside.
3. Check the chart for entries of co-managing services, nursing staff, and pharmacists.
4. Write the day's chart entries in "S-0-A-P" format.

"S-O-A-P" Format
A short, structured note that provides clinicians a quick cognitive framework for clinical reasoning to
assess, diagnose, and treat a patient based on the presented information. Shows at a glance the essential
parts of a patient's health status, enabling quicker communication between health professionals.

"S" (Subjective) • State pertinent positives and negatives in the review of systems

"O" (Objective) • State the pertinent.physical ex<!Jlllllationfindings

"A"(Assessment) • State the present assessment in the form of a problem list

"P" (Plan) • State the diagn_e>sti~ t~erapeuti~ managem~nt plan for the day

5
WRITING THE DIAGNOSTIC AND MANAGEMENT PLAN
With the information obtained from the history and PE, a prioritized problem list is then
created, with the most urgent conditions listed first. Based on the problem list (see above), the
management plan is then outlined.
Admission • Decide where the patient will be admitted (e.g., general ward, ICU)

• Dietary preparations (e.g., general liquids, soft diet, full diet) and specific
Diet
dietary prescriptions (e.g., low-salt, low-fat, low-purine, low-protein)

Fluids& • Main IV lines (e.g., plain saline, D5-containing fluids) and side drips (e.g.,
drips vasopressors, electrolyte solutions)

• Blood pressure, heart rate, respiratory rate, temperature, peripheral oxygen


Monitoring saturation, neurologic vitals, intake and output (I/0)
• Frequency by which these parameters are checked (e.g., q4 hours, qshift)
• Prioritized list of diagnostic procedures such as imaging, blood tests and
Diagnostics
special procedures
• Medications with corresponding doses, frequency of dosing, duration and
Therapeutics side effects to watch out for
• Goals of treatment may be written as a side note
• Type of blood products, the amount to be transfused, rate of transfusion and
interval between transfusions
Transfusions • Pre-medications and side effects to watch out for
• Anticipatory measures: diuretics for possible congestion, anti-pyretics for
febrile transfusion reactions

• 0, delivery system to be used (e.g., nasal cannula, face mask) and the
Oxygen(02) amount of 0, to be delivered (in liters per minute)
support • Specify target 0, saturation for the patient (i.e., keep 0, sats >95%)
• If on mechanical ventilation, the ventilator settings are included
• Nursing care (e.g., bed turning, wound dressing, catheter care)
Others
• Referrals to different specialties and reason for referral

PRESENTING THE CASE


Goodendorsements& presentationsform an integralpart of goodphysician-physiciancommunication.This,
in turn, translatesto excellentcontinuityof carefor patients.Hereare the steps to a goodcasepresentation:
• Begin with the patient's name or initials, followed by the age, sex,
General data
chief complaint, reason for admission, & date of admission or referral

• State pertinent details in the history of present illness, review of


History
systems, past medical, family medical, and personal/social histories
• State significant diagnostic results and their interpretations, including
Diagnostic results pertinent normal findings to rule out differentials being considered by
the healthcare team

• Emphasize the developments or important events that happened to


Hospital course
the patient .,
Case summary • Summarize the case in two to three sentences
Assessment • State your assessment in the form of a prioritized problem list

• State your management plan based on the assessment; this should be


Management plan
detailed and specifically tailored to the patient
Clarifications and
• Entertain clarifications or questions from the audience
open forum

6
BASIC
DIAGNOST
__ -,....B
....
~;,c;..SI_G,_T..,.·E
.....
R....
M.....
IN_..©.-L""'O...,G.,..IE
...S..,.1..,.N..,.D...,IA_G
__
N_O_S_l_lC_t
__
E_S_T
__
IN.....
G____ _
OVERVIEW OF LABORATORY MEDICINE
Clinical science devoted to quantitative or qualitative assays of biological substances for
medical or research purposes
Raw data is translated into actionable information for patient care improvement
I. DEFINITION OF TERMS
TERM I DEFINITION
Reference • Derived from a sample of a healthy population
ranges • Reflects results of 95% of disease-free individuals
Accuracy • Measure.of how close the values are to the true value
Precision • Refers to reproducibility of a value during repeated testing of a sample
• Ability of a test to detect a true abnormality
• Very sensitive tests are helpful for screening (rules out a diagnosis or
Sensitivity disease when result is negative)
• Example: An ANA titer <I:40 has a sensitivity of 98%; arid thus a negative
result (titer <I:40) is useful for ruling out SLE
• Ability of test to detect a normal result if the abnormality is not present
• Very specific tests are useful for confirmation (rules in a diagnosis or
Specificity disease when result is positive)
• Example: HbA1c 2:6.5%has 99% specificity for diagnosis of diabetes; &
thus a positive test is useful to confirm the diagnosis
• Positive predictive value: probability that subjects with a positive
Predictive screening test truly have the disease
value • Negative predictive value: probability tl;iat subjects with a negative
screening test truly do not have the disease

II. BRANCHES OF LABORATORY MEDICINE


BRANCH I DEFINITION I EXAMPLES OF TESTS
• Complete blood count
• Microscopic and laboratory
• Coagulation parameters
Hematology examination of blood and its
• Hemoglobin electrophoresis
properties including coagulation
• Flow cytometry
• Detection and quantification of • Fasting blood sugar, lipid profile
Clinical
compounds of interest (analytes) in • Serum electrolytes
chemistry
blood and other bodily fluids • Drug testing
Clinical • Microscopic examination of bodily • Urinalysis
microscopy fluids (including feces) other than blood • Fecalysis
• Detection & identification of micro- • Gram stain
Microbiology
organisms in tissues, blo.od, other fluids • Culture and sensitivity
• Detection/quantification of compounds • Dengue duo test
Immunology of interest (analytes) in blood & other • COVID-19antigen/antibody assays
fluids that assess immune function • Antinuclear antibody (ANA)
• Blood typing
Blood • Collection, storage, and dispensing of
• Crossmat~hing
banking blood products
, Antibody screening/identification
• Real-time PCR
Molecular • Specialized field that deals with
•LAMP
pathology nucleic acid-based techniques
• FISH
9
CONVERSION FACTORS OF ROUTINE LABORATORY TESTS
General Formula
• To convert a value from conventional unit to SI unit, multiply by the conversion factor
(e.g., an albumin of2 g/dL x 10 = 20 g/L)
To convert from SI unit to conventional unit, divide by the conversion factor (e.g., an
LDL of 3.2 mmol / 0.0259 = 124mg/dL)

Convert Conventional unit to SI unit = Conventional unit x Conversion factor

SI unit
Convert SI unit to Conventional unit=
Conversion factor

Conversion Factors
I CONVENTIONAL I CONVERSION
LABORATORY
UNIT FACTOR I SI UNIT

Albumin g/dL 10 g/L


Bicarbonate mEq/L 1.0 mmol/L
Calcium (total) mg/dL 0.25 mmol/L
Calcium (ionized) mEq/L 0.50 mmol/L
Carbon dioxide mEq/L 1.0 mmol/L
Chloride mEq/L 1.0 mmol/L
Cholesterol mg/dL 0.0259 mmol/L
Creatinine mg/dL 88.4 umol/L
Glucose mg/dL 0.0555 mmol/L
Hemoglobin g/dL 10 g/L
HDL-Cholesterol mg/dL 0.0259 mmol/L
Lactic acid mg/dL 0.111 mmol/L
LDL-Cholesterol mg/dL 0.0259 mmol/L
Magnesium mg/dL 0.411 mmol/L
Phosphorus mg/dL 0.323 mmol/L
Potassium mEq/L 1.0 mmol/L
Sodium mEq/L 1.0 mmol/L
FreeT4 ng/dL 12.87 pmol/L
Free T3 pg/dL 0.0154 pmol/L
Triglycerides mg/dL 0.0113 mmol/L
Troponin I ng/mL 1.0 ug/L
Uric acid mg/dL 59.48 umol/L
VitaminD ng/mL 0.0738 pmol/L

HbA1C I PLASMA GLUCOSE


6% • 7.0 mmol/L(126mg/dl) 9% • 11.8mmol/L(212mg/dl)
7% • 8.6 mmol/L(154mg/dl) 10% • 13.4mmol/L(240mg/dl)
8% • 10.2mmol/L(183mg/dl) 11% • 14.9mmol/L(269mg/dl)
10
. . .
SECTION
NORMALLABORATORYVALUES
TWO
. . •.
The values in this chapter are only generalizationsfor most laboratoryreferences.Each laboratory .. ;
I
has its own specificreferenceranges.

COMPLETE BLOOD COUNT (CBC)


I. FISHBONE DIAGRAM OF COMPLETE BLOOD COUNT
Neutrophil Monocyte

Hemoglobin

Hematocrit
Lymphocyte Eosinophil
Basophil

Reference
rangesfor completebloodcountvaryfromlaboratoryto laboratory.
Alwayscorrelateclinically.

II. COMPONENTS OF A CBC


I
PARAMETER REFERENCE I REMARKS
• Definition of anemia (WHO):
Hemoglobin 12 0-16.0g/dl
0<I3.o g/dL in men
(Hgb) (120-160g/L)
0<12.0g/dL in women
Hematocrit • Refers to proportion of the volume of a blood sample
0.38-0.47
(Hct) that is occupied by red cells
• Automated analyzers more accurately count red cell
numbers
Red blood • Mentzer index= (MCV/RBC count) x 100
4.2-5.4x10'/mm3
cell count
(4.2-5.4x1012/L)
0 If <13:microcytic hypochromic anemia most likely
(RBC) from thalassemia
0 If >13:microcytic hypochromic anemia most likely
from iron deficiency
• Measures the average volume of red blood cells
• Used in classification ofred cell disorders:
Mean cell
0 MCV >IO0:macrocytosis
volume 80-100fl
0 MCV <80: microcytosis
(MCV)
• Falsely elevated MCV: red cell agglutination, severe
hyperglycemia causing osmotic swelling
, Measures the average Hgb content per red cell
• Reflects the hemoglobin mass
Mean cell
27-31pg • Adjunct in classification of red cell disorders
hemoglobin
0 MCH <27:hypochromia
(MCH)
° Falsely elevated MCH: hyperlipidemia
• More important in laboratory quality control
• Measures the average hemoglobin concentration in a
Mean cell given red cell volume
hemoglobin • Differentials for increased MCHC: hereditary
320-360.g/L
concentration spherocytosis,,homozygous sickle cell disease,
(MCHC) hemoglobin C disease
• More important in laboratory quaiity control
11
PARAMETER I REFERENCE I REMARKS
• Quantitates cellular volume heterogeneity
(reflects range of red cell sizes within a sample)
Red cell • Very useful in classification of anemia (especially
distribution 11-16% in microcytic anemia)
width
(ROW) • Also increasingly being used as a prognostic tool
in various inflammatory, cardiac, autoimmune,
and malignant conditions
White blood 4,500-11,000/mm 3
cell (WBC) • Number of white blood cells per mm' of blood
(4.5-11x109/L)
count
• Relative percentage or absolute number of
neutrophils in peripheral blood
50-70% • Neutropenia defined as absolute neutrophil count
Neutrophil
ANC':1,500-8,000/mm' (ANC) <2 SD below mean of normal population
• Neutrophilia defined as increase in ANC >2 SD
above normal population mean
1l • Relative percentage or absolute number of
lymphocytes in peripheral blood
20-50% • Lymphopenia defined as absolute lymphocyte
Lymphocyte
ALC*:1,000-4,000/mm' count (ALC) <!,ooo/mm 3 (typically reflects
depletion ofT-cells)
• Lymphocytosis defined as ALC >4,000/mm'
0-6% • Relative percentage or absolute number of
Eosinophil
AEC': 50-500/mm' eosinophils in peripheral blood
0-2% • Relative percentage or absolute number of
Basophil ABC': 25-100/mm' basophils in peripheral blood
• Relative percentage or absolute number of
2-9% monocytes in peripheral blood
Monocyte
AMC': 100-700/mm • Monocytosis defined as increase in absolute
3

monocyte count (AMC) >800/mm'


• Number of platelets per mm 3 ofblood
Platelet 150,000-450,000/mm'• Falsely low levels: platelet activation in
count (150-450x109/L) disseminated intravascular coagulation (DIC)
and acute leukemia
• Reticulocytes are immature, non-nucleated RBCs
that still retain ribosomal RNA
Reticulocyte 0.005-0.015
count 0.5%to 1.5% • Used to calculate for reticulocyte production
index which is helpful in pathophysiologic
classification of red cell disorders
•Absolutecounts(AC)arecalculated
by multiplying
theWBCcountby 1000and% of cellof interest

12
COAGULATION TESTS
I. PROTHROMBIN,
I
PARAMETER REFERENCE
Prothrombin
time(PT)
PARTIAL THROMBOPLASTIN,

9-12 sec
I
AND THROMBIN TIME
USES
• Assess deficiencies/inhibitors of extrinsic & common pathways
• PT percent activity has no proven clinical utility
I
International
• Developed to standardize PT to allow for monitoring of
normalized 0.8-1.1
oral vitamin K antagonist (e.g., warfarin) therapy
ratio (INR)
Partial • Assess deficiencies/inhibitors of extrinsic & common pathways
thromboplastin 25-40 sec • Used to monitor response to anticoagulation (e.g.,
time(PTT) unfractionated heparin)
• Assesses deficiencies/dysfunction of fibrinogen or the
Thrombin
14-16sec presence of an inhibitor of thrombin (i.e., factor Ila)
time(TT)
• Most common cause of prolonged TT: anticoagulants

MixingStudies:
• Used to determine if prolonged PT or PTT is due to a factor deficiencyor an inhibitor
• ProlongedPT/PTTbecomes normalafter initialmix& stays normalafter 2 hrs: suggests factordeficiency
• ProlongedPTTbecomesnormalafterinitialmixbutprolongsafter2 hrs:indicatesfactorVIII(orfactorV) inhibitor
• Prolonged PT or PTT remains prolonged after mixing:suggests presence of inhibitor(e.g., lupus
anticoagulant or anticoagulation)

II. INTERPRETING ABNORMAL PT AND PTT

PT/INR
I PTT

• Factor VII deficiency (congenital & acquired from early warfarin


Prolonged Normal therapy/vitamin K deficiency, early liver disease)
• Factor VII inhibitors
• Deficiency of factors VIII, IX, • Non-specific inhibitors:
XI or contact factors (usually heparin, lupus anticoagulants,
Normal Prolonged single factor deficiency only) direct thrombin inhibitors
• Contact factor deficiencieselevate • Specific inhibitors: usually
PTT, but do not cause bleeding factor VIII inhibitors
• Non-specific inhibitors:
0 Drugs (direct Xa inhibitors, direct

thrombin inhibitors, heparin)


0 Lupus anticoagulants

• Congenital deficiency of • Specific factor inhibitors


factors X, V, II, or fibrinogen • Severe vitamin K deficiency
Prolonged Prolonged
(usually single factor • Severe liver disease
deficiency) •DIC
• Isolated factor X deficiency
(usually with amyloidosis)
• Severe fibrinogen depletion
• Hemodilution
• Drugs (e.g., heparin, direct
thrombin inhibitors)
• Inhibitors to factor II or fibrinogen
Normal • Congenital deficiency of
•DIC
(but prolonged TT) fibrinogen
• Acquired hypofibrinogenemia:
liver disease, systemic
fibrinolysis, massive hemorrhage
13
III. OTHER COAGULATION TESTS
I
PARAMETER REFERENCE I REMARKS
• Detects primary hemostatic disorders
Bleeding <5 mins
time(BT) • Prolonged BT in asymptomatic patients do not predict
risk of abnormal bleeding during invasive procedures
• Plasma protein broken down by thrombin into fibrin
175-400 mg/dl • Assesses fibrinogen activity (critical value is <IOOmg/dL)
Fibrinogen
(5.1-11.Bumol/L) • Used in evaluating DIC and assessment of various
bleeding conditions
• Terminal product of fibrinolysis generated by action of
plasmin on cross-linked fibrin
D-dimer <500 ng/ml
• Used in excluding diagnosis of venous
thromboembolism (VTE) and in the diagnosis of DIC
• Designed to measure plasma levels of anticoagulants
Anti-Xa Laboratory- • Usually used to monitor response to therapy with
levels dependent LMWH (and other newer anticoagulants) or when the
P1T cannot be reliably used for UFH

SERUM CHEMISTRY AND RELATED TESTS


I. FISHBONE DIAGRAM FOR SERUM CHEMISTRY AND RELATED TESTS

Na Cl BUN Ca TP AST LDH

CO2
K Crea Alb ALT AP

II. COMPONENTS OF SERUM CHEMISTRY AND DESCRIPTION


PARAMETER I
SI UNITI CONV I REMARKS
• Possible critical value: >3x ULN
25-125 25-125
Amylase IU/L U/L • Sensitive (but not specific) test for pancreatic
disorders
• Found in the liver (predominantly), kidneys,
Alanine heart, and skeletal muscle
5-35 5-35
Aminotransferase IU/L mU/L • Specific & sensitive for hepatocellular disease
(ALT/SGPT) • Liver parenchymal injury releases this enzyme
to the bloodstream
• Makes up -60% of total body protein
35-50 3.5-5.0
Albumin g/L • Synthesized in liver (thus a measure ofliver function)
g/dl
• Also a prognostic tool in many conditions

Alkaline • Highest concentrations in liver, biliary tract


0.5-2.0 30-120
Phosphatase uKat/L U/L epithelium, and bone
(ALP) • Used in evaluation ofliver and bone disorders
• Used to evaluate hepatocellular disease, but not
Aspartate very specific
7-40 7-40
Aminotransferase IU/L • AST/ALT ratio usually >LOin alcoholic cirrhosis,
mU/L
(AST/SGOT) chronic passive congestion, or metastatic tumors
to the liver
SI units: International System of Units Conv: Conventional Units
14
ISI UNIT I CONV I
I
PARAMETER REMARKS

, Byproduct of protein catabolism


6-47 10-80
Ammonia umol/L mcg/dL , Used to support diagnosis of severe liver
disease & follow-up of hepatic encephalopathy
• Possible critical value: >12mg/dL
Total Bilirubin 5.1-17 0.3-1.0
(TB) umol/L mg/dL • Sum of direct and indirect bilirubin
• JaundiGe presents with total bilirubin >2.5mg/dL'
• Conjugated form ofbilirubin
• Elevation usually associated with extrahepatic
biliary obstruction
Direct 1.7-5.1 0.1-0.3
• Conjugated hyperbilirubinemia (>50%
Bilirubin (DB) umol/L mg/dL
ofbilirubin is conjugated) usually from
gallstones, tumors, inflammation, scarring, or
obstruction of extrahepatic ducts
• Unconjugated form ofbilirubin
Indirect 3.4-12.0 0.2-0.8 • Unconjugated hyperbilirubinemia (when <15-
Bilirubin (IB) umol/L mg/dL 20% of total bilirubin is conjugated): usually
·from RBC hemolysis, hepatitis, or sepsis
• Possible critical value: >IOOmg/dL
Blood Urea 2.9-8.2 8-23 • Measures amount of urea nitrogen in blood
Nitrogen (BUN) mmol/L mg/dL • Related to hepatic metabolic function and
renal excretory function
• Possible critical values: <6 or >13mg/dL
2.10-2.50 8.4-10.6 • Directly measures calcium in blood
Calcium .mmol/L mg/dL • To evaluate parathyroid disease, renal disease,
malignancies, & other systemic conditions
• Possible critical values: <2.2or >7mg/dL
Calcium, 1.15-1.35 4.6-5.1
• Roughly 50% of total calcium in blood exists
Ionized mmol/L mg/dL
in its free (ionized) form
• Non-specific acute-phase reactant to evaluate
bacterial infections and inflammatory conditions
• Produced by the liver during acute inflammation
C-Reactive <10.0 <1.0 • More sensitive and rapidly responding indicator
Protein mg/L mg/dL than erythrocyte sedimentation rate (ESR)
• High-sensitivity CRP (hs-CRP):enables accurate
assays even at low levels (independent prognostic
marker for cardiovascular conditions)
• Measurement of the rate at which RBCs settle in
Erythrocyte 0-15 0-15 saline or plasma over a specified time period
sedimentation mm/h mm/h
rate (ESR) • Non-specific marker of inflammation to monitor
treatment of inflammatory/autoimmune conditions

• Possible critical values: <80 or >ll5 mEq/L


96-106 96-106 , Measurement can give an indication of the
Chloride mmol/L mEq/L acid-base balance and hydration status of
patients (tested alongside other electrolytes)
, Excreted entirely by the kidneys & is therefore
50-110 0.6-1.2 directly proportional to renal function
Creatinine umol/L mg/dL • Used to estimate renal function
, Affected by protein intake and muscle mass

15
I
PARAMETER SI UNIT I CONV I REMARKS
• Used to assess folate availability in the
work up of anemia and pregnancy
11-57 5-12 , If folic acid is low, tissue folate is measured
FolicAcid nmol/L ng/ml by determining content of folate in RBCs
(RBC folate: 360-1400 nmol/L)
• Done in conjunction with vitamin B12assays
" • Globulin represents all non-albumin protein
• May be used as measure of nutrition
• Normal albumin/globulin ratio: >I.0
23-34 2.3-3.4
Globulin g/L g/dl • Lesser ratios seen in liver disease,
autoimmune conditions, paraproteinemia
• Very high globulin levels should make
you suspect paraproteinemia
Glucose 3.9-6.1 70-110 • Possible critical values: <40-50 & >400 mg/dL
(fasting) mmol/L mg/dl • Should be evaluated in relation to meal-time
• Good indicator of body iron stores
• Assess iron deficiency & overload states
20-200 20-200
Ferritin ug/L ng/ml • Acute-phase reactant & may rise in
conditions not reflecting iron stores (e.g.,
inflammation, infection, cancer)
• -70% of iron is bound to hemoglobin
• Iron is bound to transferrin: when iron is
Iron low, transferrin levels increase (& vice versa)
Male 13-31umol/L 75-175mcg/dl
Female 5-29umol/L 28-162mcg/dl • Serum iron determination is a
measurement of the quantity of iron bound
to transferrin
• Measurement of all proteins available for
:, binding mobile iron (transferrin accounts
for a majority of these)
• Increased in iron deficiency (mostly)
,, • Since transferrin is a negative acute
' phase reactant, TIBC will be decreased
Total Iron- in inflammatory states (e.g., malignancy,
Binding 45-73 250-410 liver disease, or connective tissue disease)
Capacity umol/L mcg/dl • TIBC is more reflective of hepatic
(TIBC) function rather than iron metabolism
• Serum iron & TIBC used to compute for
transferrin saturation (TSAT): useful to
assess iron status (reference: 20-50%)
0 TSA T = [Serum iron/TIBC] xroo%

0 TSA T <20%: suggests iron deficiency

0 TSAT >50%: suggests iron overload

Lactic Acid • Used to document presence of tissue


Arterialblood 0.3-0.8mmol/L 3-7 mg/dl hypoxia, determine its severity, and
Venousblood 0.6-2.2mmol/L 5-20 mg/dl monitor effect of therapy
Lactate • Used in a multitude of inflammatory
Dehydrogenase
states, myocardial injury, red cell
(LOH)
Adult 45-90 IU/L 45-90 U/L hemolysis, infections, and malignancy as
>60yrsold 55-100IU/L 55-100U/L marker of tumor burden

16
PARAMETER I SI UNIT I CONV I REMARKS
• Enzyme secreted by pancreas to break
down triglycerides into fatty acids
0-160 0-160
Lipase units/L units/L • May rise in renal failure, intestinal
obstruction, or imestinal perforation
• Values >3x: seen in acute pancreatitis

• Possible critical values: <0.5 or >3 mg/dL


0.65-1.05 1.3-2.1
Magnesium mmol/L mg/dl • Intracellular cation essential in most organs
(especially in cardiovascular system)

• Critical values: <120 or >160 mEq/L


135-145 135-145
Sodium mmol/L • Major cation in extracellular space
mEq/L
• Major determinant of extracellular osmolality
• Critical values: <I mg/dL
1.0-1.5 3.0-4.5 • Phosphorus in the body exists as phosphate
Phosphate mmol/L mg/dl • Measures inorganic phosphate, most of
which is intracellular
3.5-5.1 3.5-5.1 • Critical values: <2.5 or >6.5 mEq/L
Potassium mEq/L
mmol/L • Major intracellular cation
Total protein 60-80 g/L 6.0-8.0 g/dl • Sum of measured serum albumin & globulin

• Critical value: >12 mg/dL


120-420 2 0-7 0 • Byproduct of purine catabolism
Uric Acid umol/L mg/dl • Majority is excreted by kidney (smaller
proportion excreted by GI tract)

Total <5.2 <200 • Lipid associated with atherosclerotic disease


Cholesterol mmol/L mg/dl • Done as part of lipid profile testing
0.45-1.71 40-150 • Possible critical value: >400 mg/dL
Triglycerides mmol/L mg/dl • Act as energy storage source
• Associated with decreased cardiovascular risk
>0.91 >35
HDL-C mmol/L mg/dl
• HDL removes cholesterol from peripheral
tissues & transports them to liver for excretion
• Associated with increased cardiovascular risk
<3.4 <130
LDL-C • Carries cholesterol that can be deposited
mmol/L mg/dl
into linings of blood vessels
• Major form circulating in the blood (it
is the inactive precursor to the active
hormone 1,25-dihydroxyvitamin D)
25-hydroxy- 62.5-200 25-80 • Used to assess vitamin D deficiency and
vitamin-D nmol/L ng/ml monitor treatment response
• Also tested at baseline prior to starting
osteoporosis treatment
• Vitamin D deficiency: defined as <20 ng/mL

• Active hormone tested in some conditions:


0 Renal disease
1,25-dihydroxy- 0 Enzyme abnormalities (that
vitamin-D
45-159 pmol/L 18-64 pg/ml convert 25-hydroxyvitamin D to
Male
45-194 pmol/L 18-78 pg/ml 1,25-dihydroxyvitamin D)
Female
0 Hypercalcemic states from
hyperproduction of1,25-dihydroxyviramin
D (e.g.1 sarcoidosis, some lymphomas)

SI units: International System of Units Conven: Conventional Units


17
HORMONES AND BIOMARKERS
PARAMETER I SI UNIT I CONV I REMARKS
• Screening serum marker for fetal body
Alpha wall defects (e.g., neural tube defects)
<40 <40
Fetoprotein • Also used in the evaluation and follow-
mcg/l ng/ml
(AFP) up of hepatocellular carcinoma, germ cell
tumors/yolk sac tumors and other cancers

Adreno- • Assesses anterior pituitary gland


corticotropin function
(ACTH)
7-69 pg/ml 7-69 pg/ml • Used in the evaluation of
Male
Female 6-58 pg/ml 6-58 pg/ml hypercortisolism and hypocortisolism

Cancer Antigen 0-35 0-35 • Used in the evaluation and follow-up


(CA) 125 kU/l U/ml of ovarian cancer (CA)

0-37 0-37 • Used in evaluation & follow-up of


CA19-9
kU/l U/ml pancreatic, biliary, & stomach cancers

Calcitonin • Used in evaluation and follow-up of


Male <13.8 ng/l <13.8pg/ml medullary thyroid cancer
Female <6.4 ng/l <6.4 ng/l • Also high in lung, breast, pancreatic CA

Carcinoembryonic
• Used in the evaluation and follow-up
antigen (CEA)
of colon (and other gastrointestinal
Nonsmoker 0-3 mcg/l 0-3 ng/ml
0-5 mcg/l 0-5 ng/ml tumors) and breast cancers
Smoker

Cortisol • Important in assessing adrenal function


AM 170-635 nmol/L 6-23 mcgldl • Demonstrates diurnal variation: peak in
PM 82-413 nmol/L 3-15 mcg/dl the morning and nadir around midnight

Creatine Kinase • Elevated in conditions with injury to


(CK)-Total
cardiac muscle, skeletal muscle, brain
Male 20-215 lU/L 20-215 U/l
Female 20-160 lU/L 20-160 U/L • Isoenzymes: CK-MM, CK-MB, CK-BB

CK-MB 0-16 U/L 0-16 U/L • CK isoenzyme specific for myocardium

• Makes up almost all of circulating CK


CK-MM 8-97 Ull 8-97 U/L
• Elevations in skeletal muscle injury
Follicle-stimulating
• Produced in anterior pituitary gland
hormone (FSH)
• For evaluation of infertility
Male 1-10 lU/l 1-10 mU/ml
• For assessment of anterior pituitary
F (premenopausal) 20-50 IU/L 20-50mU/mL
F (postmenopausal) 40-250 lU/l 40-250mUml gland function

Luteinizing
hormone (LH) • Produced in anterior pituitary
Males 1-9 IU/l 1-9 lU/l • Evaluation of infertility & assessment
Female (follicular) 2-10 lU/l 2-10 lU/l of anterior pituitary gland function
Female(mid-cycle) 15-65 lU/l 15-65 lU/l • Also used to determine whether
Female (luteal) 1-12 IU/l 1-12 lU/l ovulation has occurred
F (postmenopausal) 12-65 IU/l 12-65 IU/l

• Produced in anterior pituitary


Growth
• Evaluation of short stature, delayed
hormone 0-10 mcg/l 0-10 ng/ml
sexual maturity, growth deficiencies,
(hGH), fasting
or ectopic production from neoplasms

18
PARAMETER I SIUNIT I CONV I REMARKS
• Used to document wherher ovulation
has occurred (ro evaluare inferrility)
Progesterone
• To moniror placental srarus in high-
Male 0-1.3nmol/L 0-0.4 ng/ml
risk pregnancies
Female (follicular) 0.3-4.8 nmol/L 0.1-1.5ng/ml
• To monitor progesterone supplementation
Female (luteal) 8.0-89 nmol/L 2.5-28ng/ml
in inadeguare !urea! phase (ro
mainrain an early pregnancy)
• Secreted in anterior pituitary gland
Prolactin
• To monitor activity of prolactinomas
Male 1-20 mcg/l 1-20 ng/ml
• Also elevated in some paraneoplastic
Female 1-25 mcg/l 1-25 ng/ml
syndromes (e.g., lung cancer)
• To evaluate ambiguous sex characteristics,
Testosterone
precocious puberty, virilizing syndromes
Male 9.5-30 nmol/l 275-875ng/dl
among females, and male infertiliry
Female 0.8-2.6 nmol/l 23-75 ng/dl
• Tun1or n1arker for rare ovarian/
Pregnant 1.3-6.6 nmol/l 38-190ng/dl
resticular rumors
Prostate Specific • Elevated in prostate diseases,
Antigen (PSA) especially in prostate cancer
<40years old 0-2.0 mcg/l 0-2.0 ng/ml • Free (unbound) PSA: more accurate
~40 years old 0-4.0 mcg/l 0-4.0 ng/ml for screening
• To evaluare hyperparathyroidism & in
Parathyroid 1.4-6.8 13.2-64
disringuishing non-pararhyroid from
Hormone pmol/l pg/ml
pararhyroid causes of hypercalcemia
• Peptide precursor of calcitonin
• Biomarker which is specific in
identifying sepsis and can be used in
the diagnosis of bacterial infections
• Mildly elevated (0.15-2 ng/mL) in:
0 Localized mild-to-moderate infection
<0.15 <0.15 0 Noninfectious systemic
Procalcitonin
ng/ml ng/ml inflammatory response
0 Untreated end-stage renal disease
• Significantly elevated (>2 ng/mL) in:
0 Bacterial sepsis
0 Severe localized infection
0 Severe noninfectious inflammation
0 Medullary thyroid carcinoma
• Identifies lgM directed against the Fe
fragment oflgG
• Marker for rheumatoid arthriris
Rheumatoid <60 <60
• May be posirive in other
Factor IU/ml U/ml
autoimmune disorders (e.g., SLE,
Sjogren's syndrome) and orher
diseases (e.g., TB, chronic hepatitis)
13-27 1.0-2.1 • Measures unbound active thyroxine
FreeT4
pmol/l ng/dl (more accurate than total T 4)
3.5-6.5 2.4-5.0 • Less stable than T 4; comprise -7-10%
Free T3 pg/ml
pmol/l of thyroid hormones
Thyroid 0.4-4.8 0.4-4.8 • Used to differentiate primary from
Stimulating
mlU/l ulU/ml secondary thyroid disorders
Hormone (TSH)
19
I
PARAMETERSI UNIT CONV I I REMARKS
0-0 09 0-0.09 • To evaluate suspected acute coronary syndromes
Troponin I • In cardiac injury, troponin becomes elevated
ng/ml mcg/L
sooner & remain elevated longer than CK-MB
• Measures cardiac troponin 5- to 100-fold lower
High
<14.4 <14.4 than early conventional assays
Sensitivity • Allows lower limits of detection, earlier detection
ng/L ng/L
Troponin I of myocardial injury, detection of smaller areas of
injury, & reduced time to diagnosis
SI units: InternationalSystem of Units Conv: ConventionalUnits

URINE STUDIES
I. ROUTINE URINALYSIS
Provides significant information about the urinary system
Ideally a midstream catch specimen after cleaning external genitalia is preferred especially if
primary indication for doing urinalysis is suspected urinary tract infection (UT!)
If with indwelling catheter, fresh specimen should be submitted (avoid samples that
have been stagnant in the catheter tubing or bag)
Sample must be analyzed within 2-4 hours from collection to prevent cellular lysis and
precipitation of solutes
Yellowishcolor from urochrome: intensity depends on urine concentration & specific gravity
PARAMETER
NORMAL I I REMARKS
• Determined by chemical content, concentration and pH
• Color changes can indicate presence of a disease process,
metabolic abnormality, or ingested food/drug
Yellow
° Colorless: if high output and with low osmolality
Urine color
0 Dark-yellow: concentrated urine during limited fluid
intake or excess bilirubin
0 Red: hemoglobin from RBCs
0 Brown: myoglobin from rhabdomyolysis or methemoglobin
• Precipitation of solutes dissolved in urine (amorphous urates
& phosphates) can cause normal urine to appear cloudy
Urine clarity Clear ° Cloudy: leukocytes, bacteria, crystals, lipids, contaminants
0 Turbid: mucus, pus, semen or prostatic fluid,

radiocontrast media (RCM) or contaminants


• Relative density of urine compared to density of pure water
• Most accurate when urine pH is 7.0-7.5 (acidic urine causes
falsely high SG, while alkaline urine causes falsely low SG)
Specific 1.010to • Indirectly measured when using reagent strips
gravity (SG) 1.025 0 High SG (hypersthenuria) or concentrated urine: dehydration,

AKI with oliguria, CHF, SIADH, glucosuria, proteinuria


0 Low SG (hyposthenuria) or dilute urine: diabetes
insipidus, CKD, ingestion of large amounts of water, aging
• Useful in diagnosing systemic acid-base disorders:
0 Alkaline urine (pH >7.0): infection with urea-splitting

organisms (Proteus sp.), prolonged storage of urine,


vomiting, gastric suction, diuretic & alkali therapy,
4.5 to 8 metabolic & respiratory alkalosis, and in those on
pH
Average: 6 vegetarian, citrus fruit, and low carbohydrate diets
0 Acidic urine (pH <5.0): metabolic & respiratory acidosis,
uremia, & infection with acid-producing bacteria(£. coli)
• Urine pH >5.0 in the setting of metabolic acidosis may
indicate possible renal tubular acidosis
20
I
PARAMETER NORMAL I REMARKS

• Glucosuria from hyperglycemia may occur at blood glucose


levels >IO mmol/L (>180 mg/dL)
Glucose Negative • Glucosuria can also occur in cases of proximal tubular
disorders (Fanconi syndrome), pregnancy, hormonal
disorders, liver disease, & drug intake (e.g., thiazides, steroids)

• Screening measure using a reagent strip for presence of


proteins in urine (most especially sensitive to albumin)
• Insensitive to other proteins (globulins, hemoglobin, light chains)
• Scored from trace to 4+ based on concentration of protein
Protein 0-8 mg/dl Negative: <15mg/dL 2+: wo-300 mg/dL
Trace: 15-30 mg/dL 3+: 300-IO00 mg/dL
1+: 30-IOo mg/dL 4+: >IOoo mg/dL
• If proteinuria is detected by dipstick, it is best quantified
either by 24-hour urine testing, or spot urine PCR or ACR
• Leukocyte esterase is released from lysed urine leukocytes
Leukocyte
Negative • Positive leukocyte est erase: reflect presence of pyuria, even in
esterase
the absence ofleukocytes (indicates infection or inflammation)

• Positive nitrite test: presence of nitrate-reducing bacteria


Nitrites Negative
(e.g., E.coli, Proteussp., Enterobactersp., Klebsiellasp.)

• Detected if in the form of acetoacetate & acetone


• May appear in urine during prolonged starvation or
Ketones Negative fasting, alcoholic or diabetic ketoacidosis
• Note that beta-hydroxy butyrate (80% of total ketones) is
not normally detected

• Various crystals can be seen, most commonly calcium


Crystals Negative oxalate, uric acid, calcium phosphate, triple phosphate
(struvite), cholesterol and cystine crystals

• Cylindrical bodies unique to the kidneys, formed from


aggregation ofTamm-Horsfall glycoprotein (uromodulin)
Casts Negative
• May be hyaline, granular, waxy, fatty, RBC, WBC, or
tubular epithelial cell/muddy brown cast

• Hematuria is defined as ~3 RBCs/hpf


• Transient hematuria may be from strenuous exercise
RBC 0-2/hpf • Persistent microscopic hematuria on 3 repeated samples warrants
investigation of glomerular lesions or non-glomerular hematuria
• Macroscopic hemaniria: malignancy, bleeding diathesis, calculi
• Most commonly in the form ofneutrophils (indicates
infection or contamination)
WBC 0-4/hpf • Eosinophils detected by Hansel or Wright stain can
indicate drug-induced hypersensitivity, renal cholesterol
embolism, RPGN, and prostatitis

• Can be squamous, transitional, or renal tubular epithelial


None
Epithelial • Squamous epithelial cells rarely have diagnostic value
to very
cells and their presence indicates that urine specimen was not
minimal
midstream clean catch (contaminated)

Bacteria Negative • Presence indicates contamination or infection

Mucus Small • Commonly in urine sediments & has no clinical


thread amounts significance
21
II. 24-HOUR URINE CHEMISTRY
Timed urine collection used in the metabolic evaluation of renal stone disease, proteinuria,
estimation of renal function using creatinine clearance, or estimation of residual kidney
function in ESRD using urea and creatinine clearance
Requires collection of all urine within a 24-hour period (unlike random urine sampling)
The completeness of a 24-hour urine collection is important, especially in evaluating
proreinuria (measured in g/day), and completeness is assessed by urinary creatinine excretion
(not by urine volume)
The 24-hour urinary creatinine excretion reflects muscle mass and excretion is relatively
constant over time in each person
Unlike crearinine excretion, protein excretion varies throughout the day, hence a random urine
sample will be inaccurate in measuring proteinuria, unless corrected for creatinine excretion

PARAMETER I NORMAL
I REMARKS

• Indicative of daily fluid intake (minus insensible losses);


diminishes with low fluid intake, sweating, and diarrhea
Total 1000-2000 • Target >2500 ml/day
Volume ml/day • Decreased urine volume is a major risk factor for stone
disease as concentrated urine raises the supersaturation
of all stone-forming salts

Creatinine 20-25 mg/kg


Male (0.15-0.22mmol/kg) • Used for assessment of completeness of collection
• Valid only in steady state of constant serum creatinine
Female 15-20 mg/kg concentration ,vith time
(0.13-0.15mmol/kg)

Total Protein • Normal or mildly increased: <150 mg/day


At rest 50-80 mg/day • Moderately increased: 150-500 mg/day
Exercise <250 mg/day • Severely increased: >500 mg/day
• Nephrotic range: >3500 mg/day

• Ar <30 mg/day, nonnoalbuminuria


• A2 30-300 mg/day, microalbuminuria
Albumin <30 mg/day
• A3 >300 mg/day, macroalbuminuria
• Nephrotic range >2200 mg/day

Glucose 50-300 mg/day • See discussion above


• <5.5:increases risk of uric acid precipitation
pH 4.5 to 8.0 • >6.7: increases risk of calcium phosphate precipitation
• >7.0-7.5:indicates UT! from urease-producing bacteria

• Reflects dietary sodium intake (minus extrarenal loss)


100 mEq/day • Much lower than dietary intake in diarrhea and with
Sodium
(100 mmol/day) excessive sweating
• High sodium intake is a major cause of hypercalciuria

• Reflects dietary potassium intake (minus extrarenal loss)


• Most useful in monitoring compliance of treatments
40-60 mEq/day such as potassium citrate
Potassium (40-60 mmol/day) • Much lower than dietary intake in diarrhea states
• Measure of dietary alkali intake because most dietary
potassium is accompanied by organic anions
100 mEq/day
Chloride • Chloride varies with sodium intake
(100 mmol/day)

• Hyperuricosuria: encountered with overproduction of


600-800 mg/day endogenous uric acid or overindulgence of purine-rich food
Uric acid
(3.57-4.76mrrdJday) • Risk factor for Ca" oxalate stones when urine pH is >5.5
• Risk factor for uric acid stone when urine pH <5-5

22
I I


PARAMETER NORMAL REMARKS I
, A higher value is expected in males
• Urine calcium is dependent on dietary calcium,
$250-300mg/day
Calcium
mmol/day) sodium intake, and protein intake
(6.24-7.49
• Rule out secondary causes before making the
diagnosis of idiopathic hypercalciuria I
• Low urinary magnesium detected with low
30-120mg/day magnesium intake, intestinal malabsorption (small
Magnesium
mmoVday) bowel disease), and following bariatric surgery
(1.23-4.94
• Low magnesium may increase risk of calcium stones
• Indicative of dietary organic and inorganic
$1100mg/day phosphorus intake and absorption
Phosphorus
($35.5mmol/day) • Higher excretion may increase the risk of calcium
phosphate stone formation
• Commonly encountered in intestinal disease with
$45 mg/day fat malabsorption (e.g., inflammatory b_oweldisease)
Oxalate
($0.51mmol/day) and following bariatric surgery
• Values >IOO mg/day (1.14 mmol/day) suggest
primary hypernxaluria (PH)
• Potent inhibitor of calcium salt crystallization
• Hypocitraturia: risk factor for stone disease (found
;;:320mg/day
Citrate
(<!1.67mmol/day) in up to a third of calcium stone formers)
• Low urinary citrate: can be idiopathic or from other
factors (e.g., diet, metabolic acidosis, hypokalemia)

STOOL ANALYSIS
I. TESTS FOR OCCULT GI BLEEDING
SPECIMEN &
TEST
I INDICATIONS I REMARKS

• Detects heme part of Hgb from blood in stool


• Fresh stool (heme, however, is common to blood from all
Fecal sample in sources & thus is not specific for GI bleeding)
occult a sterile • Specific restrictions apply prior to testing:
blood test container avoidance of intake of red meat, avoidance of dental
(FOBT) • Used to detect procedures, avoidance of vitamin C
conditions • Test relies on a chemical reaction to produce a color
that can cause change
bleeding
within the GI • Specifically detects globin part of human
tract hemoglobin (will not detect globin from non-human
Fecal blood) from blood in stool
immuno- • Used as a
screening test • Less sensitive for upper GI bleeds as hemoglobin is
chemical
for colorectal broken down before it reaches the lower GI tract
test (FIT)
cancer • No specific restriction prior to testing
• Test uses antibodies to detect human globin

23
II. ROUTINE FECALYSIS
Test done on a stool sample for differential diagnosis of certain diseases of the GIT
• It can be divided into: physical+ chemical+ microscopic examination
PARAMETER I NORMAL
I REMARKS

Ph,]ISICQ 1Ex_qminat1'1!'
'
• Black: bleeding in GIT, intake of iron, bismuth, charcoal
Yellow,
• Red: bleeding, usually in lower GIT
Color/ brown,green
• Green: biliverdin/oral antibiotics, green vegetables
appearance (depends on
• Clay/acholic: biliary obstruction or residual barium sulfate
food intake)
• Mucus: colitis (infectious/inflammatory), neoplasm
Well-formed • Semi-solid or watery: dysentery & other gastroenteritis
Consistency (solidto semi- , Fatty: maldigestion, vitamin deficiency, pancreatic disorders
solid) • Frothy: pancreatic disorders
. .
Cb~mical Ex".imi~ai1,en ;;;_
"''
Neutralto • Acidic stools (<5.5pH): may be used to determine
pH
alkaline(7.0-7.5) lactose intolerance
• Detects hemoglobin in stool
Occult
Negative • Associated with any bleeding (e.g., hemorrhoids,
blood
neoplasm, ulcers, inflammatory bowel disease, infections)
• 2:60stained droplets of neutral fats/HPF: steatorrhea
<6 gramsof
Fat (pancreatitis or exocrine pancreatic insufficiency), celiac
fat/24hrs
disease, intake of castor oil or mineral oil, cystic fibrosis
Nitrogen <2.5g/24hrs , >2.5 g/24 hrs may indicate chronic progressive pancreatitis
.. fr

Micros~opic ExamiJStion "


• >50 leukocytes/HPF: shigellosis
Leukocytes 0-1/HPF • <20 leukocytes/HPF: salmonellosis, invasive E.coli
, <5 leukocytes/HPF: cholera, EPEC, ETEC, viral diarrhea
• Detects intact RBCs in stool (unlike occult blood)
RBCs None • Suggests infection (parasitic, bacterial, viral), inflammatory
bowel disease, neoplasm, ulcers, hemorrhoids
Bacteria Normalfiora • Pathogenic bacteria: Campylobacter,Salmonella,Shigella
Amoebas, • Pathogenic cysts & trophozoites: Entamoeba histolytica,
None
flagellates Giardialamblia, Trichomonas vagina/is
Eggs of None • Pathogenic eggs: roundworm, hookworm, Taenia
parasite
• May be present in small quantities in skin, mouth, intestine
Yeasts Noneto rare • Presence of moderate/heavy yeast seen in potential
yeast overgrowth or non-viable, dietary-derived yeast
• Charcot-Leyden crystals:parasitic infections (e.g.,amoebiasis)
Crystals None • Triple phosphate and calcium oxalate crystals: ingestion of
certain food (spinach, tomatoes, berries)
Meat fibers,
None • Presence may suggest maldigestion
starch,trypsin
Undigested
None • Large amounts may suggest maldigestion
food

24
SECTION THREE
I
~~~~==-------
STE-Rll.:E.
FLWIDSAND PATHOLOGIES
PLEURAL EFFUSION (PLEURAL FLUID)
<
I. ETIOPATHOGENESIS AND MANIFESTATIONS
Collection of fluid abnormally present in the pleural space due to either excess fluid production
and/or decreased lymphatic absorption (normal pleural space contains only ~IO mL off!uid)
Patients may present with pleuritic pain, cough and dyspnea
PE findings: decreased breath sounds, decreased or absent tactile fremiti, dullness on percussion

• Most common cause of pleural effusion


• Diagnostic thoracentesis should be done to verify "transudative" effusion if:
0Effusions are not bilateral & comparable in size
Heart failure 0Patient is febrile
0 Patient has pleuritic chest pain

• Pleural fluid NT-proBNP ,1500 pg/mL is virtually diagnostic of heart failure


• Due to passage of peritoneal fluid from abdomen through the diaphragm
Cirrhosis
• Usually right-sided (hepatic hydrothorax)
• Nephrotic syndrome due to hypoalbuminemia
• SVC obstruction
Otliers • Myxedema
• U rinothorax
• Pulmona embolism may also be exudative)

• Most common cause of exudative pleural effusion


• Acute febrile illness with chest pain, sputum production, & leukocytosis
Parapneumonic • If free fluid separates the lung from the chest wall by >IO mm, a therapeutic
(bacterial thoracentesis should be performed
pneumonia, • The following factors indicate need for a chest tube thoracostomy (CTI)
lung abscess, 0Loculated pleural fluid
bro'nchiectasis, 0Pleural fluid pH <7-20
empyema) 0 Pleural fluid glucose <3-3mmol/L (<60 mg/dL)

0 Positive gram stain or culture of the pleural fluid

0 Presence of gross pus in the pleural space

• Three most common causes: lung CA, breast CA, and lymphoma
• Diagnosis usually by cytologic exam (thoracoscopy is next best procedure if
Malignancy cytology is negative)
• Glucose levels may be low if tumor burden is high
• Pleurodesis or insertion of a small indwelling catheter may be considered
• Diagnosis most commonly overlooked in differentials of undiagnosed effusion
Pulmonary
• If effusion increases in size after anticoagulation, consider recurrent emboli,
·embQlism
hemothorax or a pleural infection
• Most common cause of an exudative pleural effusion in developing countries
• Usually associated with primary TB (hypersensitivity reaction to TB protein)
Tuberculous • Qualitative analysis shows predominantly small lymphocytes
'pleuritis • Diagnosis is established by (in pleural fluid):
0 High levels of adenosine deaminase (>40 IU/L), OR
0Interferon gamma (>140 pg/mL)
• Pleural fluid hematocrit should be obtained if initial tap is bloody
• If pleural fluid hematocrit is,V,
of that in peripheral blood, hemothorax
Hemothorax should be considered and tube thoracostomy should be done
• If pleural hemorrhage ,200 mL/hr, consider angiographic coil
embolization, thoracoscopy or thoracotomy
25
II. CHARACTERISTICS OF NORMAL PLEURAL FLUID
PARAMETER I FINDINGS
Color & appearance • Clear or yellow fluid

pH • 7.60to 7.64
Protein content • 1-2g/dl (<2%)

Cell count • <1,000WBC/mm3

Glucose • Similar to plasma

LDH • <50%of plasma

JIJ. APPROACH TO PLEURAL FLUID ANALYSIS


First step: determine if effusion is exudative vs transudative (use the Light's criteria by
obtaining LOH and protein level from serum and pleural fluid)
Other diagnostics for exudative pleural effusions:
Description of the appearance of the fluid
0

Glucose & protein level


0

Differential cell count


0

Microbiologic studies and cytology


0

Work up for tuberculosis


0

Liaht's Criteria
Exudative pleural effusions meet at least one of the following criteria:
• Pleural fluid protein/serum protein >0.5
• Pleural fluid LDH/serum LDH >0.6
• Pleural fluid LOH >213normal upper limit for serum
These criteria may misidentify-25% of transudates as exudates. If 2:1 of the exudative criteria are
met & patient is clinicallythought to have a transudative effusion, the difference between the protein
levels in the serum and the pleural fluidshould be measured. Ifthis gradient is >31 g/L, the exudative
categorization by these criteria can be ignored because almost all such patients have transudalive
pleural effusion.
Source:LightRW,et al.AnnInternMed;1972
JamesonJL,et al. Harrison's
Principles
of InternalMedicine20thedition,2018

Case 1. A 50-year-old male presented with a few days' history of fever, cough, and
progressive dyspnea. Physical examination revealed right-sided chest lag with dullness
on percussion on right lower lung fields and decreased vocal and tactile fremiti. Chest
x-ray revealed pleural effusion on the right. Thoracentesis drained -600 mL of free-
flowing serosanguineous fluid. Pleural fluid studies revealed glucose of 65 mg/dL,
pH of 7.23, LOH of 500 IU/L, total protein of 50 g/L. Bacterial cultures were negative.
Adenosine deaminase was also normal. Peripheral blood LOH was 350 IU/L and serum
total protein was 60 g/L.

Diagnosis: Parapneumonic effusion


• Note the history signifying progressive pneumonia. This patient is in the early
spectrum of parapneumonic effusion which is due to increased pulmonary interstitial
fluid traversing the pleura to enter the pleural space The pleural fluid in this stage usually
presents with yet negative bacterial cuJcures, fluid glucose still >60 mg/dL, pH >7.20and
LOH <3XULN of serum.
• Nevertheless, the effusion is exudative since patient already met two of Light's criteria:
Pleural fluid protein/serum protein = 50/60 = 0.83 (fulfilling the cutoff of >0.5)
0

Pleural fluid LOH/serum LOH= 500/350 = 1.43 (fulfilling the cutoff of >0.6)
0

26
ASCITES (PERITONEAL FLUID)
I. ETIOPATHOGENESIS
Ascites is the condition of pathologic fluid collection within the abdominal cavity
Occurs if there is a disruption in the pressure forces between abdominal intravascular
& extravascular fluid spaces, which allows extravascular fluid to accumulate
I
; .
I PROTEIN I SAAG
CONDITION
I FINDINGS
(g/L) (g/dl)

Cirrhosis <25
• Straw-colored ;;:1.1
CHF Variable

Neoplasm • Straw-colored, hemorrhagic, mucinous, chylous >25


TB peritonitis • Clear, turbid, hemorrhagic, chylous >25
Pyogenic <1.1
• Turbid or purulent >25
peritonitis
Nephrosis • Straw-colored or chylous <25

LABORATORY FINDINGS

• Absolute PMN count ;,:250/mm'


• Spontaneous/primary
Infectious • Positive gram stain or culture
bacterial peritonitis
•pH<7-o
• Fulfillment of at least 2 (with PMN
Surgical/ count ;,:250/mm'):
perforation • Secondary peritonitis 0Total protein >I g/dL
peritonitis 0Glucose <50 mg/dL
0LOH >225mU/mL
Hemorrhagic • Malignancy, TB, trauma,
• RBC count >50,000
ascites ruptured omental varix
Pancreatic • Pancreatitis • Increased amylase >2000 U/L or
ascites • Intestinal perforation -5-fold greater than serum values
Malignancy • Peritoneal carcinomatosis • Positive malignant cells in cytology
• Biliary or proximal small
• Increased bilirubin >6 mg/dL
Others intestinal perforation
• Chylous ascites • Increased triglycerides >200 mg/dL
Source:JamesonJL, et al. Harrison'sPrinciplesof InternalMedicine20th edition,2018
FeldmanM, et al. Sleisengerand Fordtran'sGastrointestinaland LiverDisease10th Edition;2016

27
II. CHARACTERISTICS OF NORMAL PERITONEAL FLUID
PARAMETER I NORMAL I REMARKS & ABNORMAL FINDINGS
• Opacity of cloudy fluid is caused by neutrophils
0 Nearly clear: ANC <IOoo/mm'
° Cloudy: ANC >5000/mm'
0 Mayonnaise-like: >50,000/mm'
• Pink or bloody fluid
0 Pink: RBC count >IO,ooo/mm'
0 Red: RBC count >20,000/mm'
Clear or 0 Traumatic tap: blood streaked, frequently clots
Color&
transparent to 0 Nontraumatic tap: homogenous, does not clot
appearance
yellow in color • Bloody, non traumatic fluid: seen in portal
hypertension (bloody hepatic lymph from lymphatic
rupture), hepatocellular carcinoma (HCCA), or TB
• Bile-stained ascitic fluid: seen in deeply jaundiced
patients
• Dark brown fluid: seen in biliary perforation
• Tea-colored to jet black fluid: seen in pancreatic
necrosis, malignant melanoma
• Men normally have no or very little intraperitoneal fluid
Amount ~50-75 ml
• Women may have more, depending on menstrual cyclephase

• Elevated counts may be seen in any inflammatory


process, with SBP being the most common cause
<500 WBC/mm3 • Also elevated (with lymphocytic predominance) in:
0 Tuberculous peritonitis
Cell count 0 Peritoneal carcinomatosis
• Elevated absolute PMN count (;:,250/mm') may be seen
in any inflammatory process (SBP is the most common)
<250 PMN/mm3
• Presence of neutrophils leads to a shimmering effect
when glass tube is rocked back & forth in front of a light
Determined by • >2.5g/dL: heart failure, early Budd-Chiari syndrome,
Total serum protein IVC obstruction, sinusoidal obstruction syndrome
protein concentration & • <2.5g/dL: malignancy-related ascites, massive liver
portal pressure metastases, HCCA, late Budd-Chiari syndrome
Best interpreted
Albumin
using SAAG
• See next section(SAAG)

Concentration in • May be low (up too mg/dL) in SBP or intestinal


Glucose fiuid is similar to perforation because glucose is consumed by ascitic
serum fluid-stimulated neutrophils and/or bacteria
Concentration in
cirrhotic
ascites • Elevated ascitic fluid LDH may be seen in SBP (due to
LDH
usually less than release ofLDH from neutrophils)
half of the serum
Concentration
• Elevated in acute pancreatitis & intestinal perforation
in ascitic fiuid
due to release ofluminal amylase into the ascitic
Amylase usually half of
the serum value fluid (usually >2000 U/L & -5x higher than the
simultaneous serum values)
(~50 U/L)

• Biliaryperforation or proximal small intestinal


Bilirubin 0.7-0.8 mg/dl perforation: ascitic fluid bilirubin concentration
greater than serum bilirubin (and >6 mg/dL)

28
I NORMAL I
I
PARAMETER REMARKS & ABNORMAL FINDINGS

• Chylous ascites: ascitic fluid TG greater than the serum


Concentration
Triglyceride TG and >200 mg/dL (usually >1000 mg/dL)
in fluidsimilar
(TG) • Cirrhosis: neither transparent nor frankly milky (TG
to serum
ranging from 50 mg/dL to 200 mg/dL)
• Gram stain demonstrates bacteria only when >10,000
bacteria/ml are present
Gram stain Absent • Helpful in diagnosing intestinal perforation and
overwhelming infection (advanced SBP or asplenic
pneumococcal sepsis)
• Malignancy is detected when tumor cells line the
Absenttumor
Cytology peritoneal cavity & exfoliate into the ascitic fluid
cells
(peritoneal carcinomatosis)

III. APPROACH TO PERITONEAL FLUID ANALYSIS

Serum Ascites-Albumin Gradient (SAAG)


Replaced the description of"exudative" or "transudative" ascitic fluid
The gradient correlates directly with portal pressures
Specimens (serum & ascitic fluid) should be obtained nearly simultaneously on the same day
Unless a laboratory error has been made, the serum albumin level is always the larger value
SAAG = serum albumin - ascitic fluid albumin

• Portal hypertension • Peritoneal carcinomatosis


• Cirrhosis • Infection (peritonitis, TB)
• Cardiac ascites • Nephrotic syndrome
• Budd-Chiari Syndrome • Pancreatic or biliary ascites
• Portal vein thrombosis • Serositis in connective tissue disease
• Veno-occlusive disease • Bowel obstruction or infarction
• Fatty liver of pregnancy • Postoperative lymphatic leak
• Fulminant hepatic failure
• "Mixed" ascites
• Myxedema
Source:JamesonJL,et al. Harrison's
Principles
of Internal
Medicine
20thedition,2018
Feldman
M,et al.Sleisenger
andFordtran's Gastrointestinal
andLiverDisease10thEdition;2016

Case 2. A 62/M presented at the ER for generalized weakness and a progressively


enlarging abdomen. He admits to heavy alcohol intake and binge drinking almost every
other day. PE showed spider angiomata on the chest, globular abdomen with fluid wave,
and an obliterated Traube's space. Paracentesis was done and two liters of straw-colored
fluid were drained. Ascitic fluid studies revealed albumin of 18 g/L, WBC of 350/mm',
and LDH ofuo IU/L. Serum albumin was 40 g/L with serum LDH of280 IU/L.

Diagnosis: Ascites from alcoholic cirrhosis with portal hypertension


• This is a patient with heavy alcohol intake, now presenting with signs of chronic liver
disease and portal hypertension.
• The diagnosis of ascites from alcoholic cirrhosis is supported by a high-gradient SAAG
>II g/L (serum albumin - ascitic albumin= 40 g/L- 18g/L = 22 g/L). Furthermore, the
ascitic LDH (uo IU/L) is less than half of the serum LDH (280 IU/L). The WBC
(350/mm') is also <500/mm' and hence is not indicative of inflammation or infection.

29
CEREBROSPINAL FLUID (CSF)
I. ETIOPATHOGENESIS AND MANIFESTATIONS
CSF is a clear fluid circulating in the intracranial and spinal compartments formed as
an ultrafiltrate of plasma
Usual indications for lumbar puncture
0 Diagnostic: CNS infections, autoimmune diseases, CNS vasculitis, subarachnoid
hemorrhage, malignancy
0 Therapeutic: benign intracranial hypertension, acute communicating hydrocephalus
0 Delivery of intrathecal drugs: chemotherapy, antibiotics

II. CHARACTERISTICS OF NORMAL CSF FLUID


CSF composition usually maintained constant, but various conditions cause distinct
alterations in CSF findings
Total volume is ~140 mL in adults
PARAMETER I NORMAL VALUES
Color & appearance • Clear and colorless

Glucose • 50-75 mg/dL(2.8-4.2 mmol/L)

Protein • 15-60 mg/dL(150-600 mg/L)

Cell Count • 0-5 cells/uL (0-0.5 x 106 cells/L)


• 0-5 cells/uL
0 Neutrophils:0-6%
WBCs 0 Lymphocytes:40-80%
0 Monocytes: 15-45%
RBCs • None

LDH • <40 units/L

Opening pressure • 100-200 mmHp

Cytology • No malignant cells


Culture & sensitivity • No organisms present

III. APPROACH TO CSF FLUID ANALYSIS


CSF analysis usually consists of opening pressure assessment, biochemical analysis,
cytology, micro biologic evaluation, and biomarker assay
• Essential in assessment of various neurologic conditions in conjunction with other
diagnostics (e.g., imaging)
Assessment of meningitis and encephalitis are major indications for CSF analysis
0 Meningitis: classic triad of altered consciousness, fever, and neck stiffness; as well as
other signs and symptoms such as nausea, vomiting, headache, and photophobia
0 Meningoencephalitis: may additionally present with altered sensorium, confusion,
behavioral changes, seizures and focal neurologic deficits

30
A. CSF Findings in Common Meningitic Conditions
I
BACTERIAL
PARAMETER MENINGITIS
I MENINGITIS
TB I MENINGITIS
VIRAL I MENINGITIS
FUNGAL I ASEPTIC
MENINGms
Normal
Normal to
Pressure Increased Increased to mildly Increased
elevated
elevated
Clear to
Color Turbid Turbid Clear Clear
turbid
Normal to Normal to
Glucose <40 mg/dL Low Low
slightly low slightly low
Normal Normal
Markedly
Proteins Elevated to slightly to slightly Elevated
elevated
elevated elevated
RBCs Elevated Elevated Normal Normal Elevated
>100/mm' but
Elevated but Mildly
WBCs 10-2000/mm' not markedly 10-50/mm'
<500/mm3 elevated
elevated
Predominant
PMNs Lymphocytes Lymphocytes Lymphocytes PMNs
WBC
Acid fast India ink for
Gram stain Positive Negative Negative
bacilli cryptococcus

Microbial Positive Positive Negative Positive Negative


culture

B. CSF Findin sin Common Acute Demyelinatin /Inflammatory Disorders

• Pleocytosis, high protein concentration, oligoclonal bands, & lgG index


Transverse
• CSF sugar and pressure usually normal
myelitis
• Elevated CSF IL-6

Multiple • Pleocytosjs (5-50 cells/mm', lymphocytic), elevated protein


sclerosis concentration, elevated IgG index

Guillain Barre • Normal CSF counts, elevated protein concentration


syndrome • Cytoalbuminologic dissociation

• Low cell count with mononuclear predominance


Encephalitis • Normal CSF glucose
• Normal to increased CSF protein

Subarachnoid • Erythrochromic or xanthochromic color


hemorrhage • Predominantly RBC's

Case 3. A 58/M with chronic cough, weight and generalized weakness presented with fever,
progressively worsening headache, nausea, vomiting and increased sleeping time. Cranial CT
scan showed prominent meningeal & basal cistern enhancement. Lumbar tap showed elevated
opening pressures with yellowish turbid CSF. CSF glucose was noted to be low at 50 mg/dL &
total protein was elevated. WBC count was 400/mm' with lymphocytic predominance.

Diagnosis: Tuberculous (TB) meningitis


• This is a typical case of TB meningitis in a patient with concomitant pulmonary
TB. CT findings in this patient are quite typical ofTB meningitis. Note the low CSF
glucose, elevated total protein, & moderately high WBC <500/mm') with lymphocytic
predominance. An AFB smear, if done, would probably show acid fast bacilli as well.

31
PERICARDIAL FLUID
I. ETIOPATHOGENESIS
Pericardia! fluid is the ultrafiltrate of plasma that resides within the pericardia I sac and acts as
a lubricant between the visceral and parietal layer of the pericardium
The composition of pericardia! fluid is believed to be a result of Starling forces and the
gradients between hydrostatic and osmotic pressure of the pericardia! fluid & plasma
Pericardia! drainage and analysis of pericardia! fluid is indicated in the following conditions:
0 Purulent or tuberculous pericarditis
0 Neoplastic pericardia! involvement
0 Pericardia! effusion of unknown origin
0 Massive idiopathic chronic pericardia! effusion
0 Tamponade caused by uncontrolled pericardia! effusion with hemodynamic instability

II. CHARACTERISTICS OF NORMAL PERICARDIAL FLUID


Physiological pericardia! fluid is rich in protein, albumin, LOH and nucleated cells with low
levels of glucose and cholesterol
PARAMETER I NORMAL I REMARKS & ABNORMAL FINDINGS

• Bloody: iatrogenic (e.g., trauma, anticoagulation, post-


invasive cardiac procedures), malignancy, tuberculosis
Color and Clear & straw-
• Milky: lymphatic involvement (e.g., chylopericardium)
appearance colored
• Cloudy & turbulent: increased capillary leakage (if there
is leukocytosis, highly suggests infection)
• Pericardia! sac can hold 80-200 mL of fluid acutely but
Amount 15-50 ml
can handle as much as 2L if fluid accumulates slowly
Total protein 1.7-4.6g/dl • No standardized biochemical test meets statistical
relationship between specific causes of effusion
LOH 141-2613U/L • Light's criteria may be applied to pericardia! effusion,
but should not be used as sole basis for diagnostic approach
Albumin 1.19-3.06g/dl • Low protein levels: suggest transudative causes
• Elevated protein levels: seen in tuberculosis or myeloma

Glucose 80-134 mg/dl • Low glucose: infection, malignancy, lupus, tuberculosis

RBCs None • No standardized criteria to determine cause of effusion, but


elevated WBC counts are highly suggestive of inflammation
WBCs x 106 cells/L
19-2210
• WBC >IO,ooo with neutrophilic predominance: suggests
PMNs 0-116x 106 cells/L bacterial or rheumatic cause
• Lymphocytic predominance: secondary to tuberculosis,
Lymphocytes 19-1634x10' cells/L lymphoma, sarcoidosis, malignancy

Microbial • If bacterial infection is suspected, at least 3 cultures of


Negative
cultures pericardia! fluid for aero bes and anaerobes are done

Malignant Absent
• Cytology aids in identifying malignant causes but has
cells variable sensitivity depending on type of malignancy
Source:FenderEA,et al. Heart;2021
III. APPROACH TO PERICARDIAL FLUID ANALYSIS
Extent to which effusions should be evaluated with fluid analysis is still controversial:
0 Patients with new pericardia] effusion need to be assessed for myocarditis or pericarditis
0 Tamponade, possible purulent effusion, or pericarditis with poor prognostic indications
0 Patients with recurrent or large effusions that do not resolve with treatment
Echocardiography is the imaging modality of choice for the diagnosis of pericardia! effusion
Routine analysis: cell count with differential, glucose, total protein, LOH, gram stain/culture
Special testing may be done depending on the condition being evaluated
0 Malignancy: cytology, tumor markers
0 Tuberculosis: adenosine deaminase, PCR, interferon-gamma
0 Viral cultures: viral infections
0 Molecular analysis for specific infectious processes
32
Case 4. A 23/M with weight loss, night sweats, afternoon fevers, & enlarging neck masses came
in for dyspnea. PE showed bilateral non-tender matted fixed masses in the supraclavicular
area, distended neck veins, axillary/inguinal lymphadenopathy. Chest x-ray revealed a water- :. •.·
bottle cardiac configuration. Echocardiography showed massive pericardia! effusion but with
no overt signs of tamponade. Pericardiocentesis was done which drained serosanguineous
fluid with presence of atypical round cells on cytology. Pericardia! fluid glucose is 55 mg/dL &
WBC count showed predominance oflymphocytes. Microbial cultures are negative.
Diagnosis: Pericardia! effusion likely from lymphoma
• This is a patient with lymphoma presenting with massive pericardia! effusion. The atypical
round cells seen in cytology points to a malignant etiology. Other pericardia! fluid findings
supportive of a lymphoma include the low pericardia! fluid glucose and the lymphocytic
predominance. The negative microbial culture result makes an infectious cause unlikely.

SYNOVIAL FLUID
I. ETIOPATHOGENESIS
Synovial fluid is an ultrafiltrate of plasma across the synovial membrane enriched with
various compounds produced by synoviocytes
Arthrocentesis with synovial fluid analysis should be attempted in all patients with joint
effusion or signs suggestive of inflammation without a known cause
Important indications include:
Evaluation of septic arthritis in those with acute swollen joint with warmth & tenderness
0Differentiating gout from pseudogout

II. CHARACTERISTICS OF NORMAL SYNOVIAL FLUID


Normally, the biochemical composition of synovial fluid is almost like that of plasma
• Certain conditions alter composition of synovial fluid
PARAMETER
I NORMAL FINDINGS

Color and appearance • Clear and transparent


Viscosity • High due to the presence of hyaluronic acid
Amount (knee) • <3.5 ml
WBC • <200/mm3
PMNs • <25%
Glucose • Similar to plasma
Microbial culture • Negative

III. APPROACH TO SYNOVIAL FLUID ANALYSIS


Laboratory evaluation of synovial fluid usually involves:
0 Physical examination of the fluid: appearance, color, clarity and viscosity
° Chemical analysis: glucose, total protein
0 Microscopic evaluation: cell count and differential, crystal identification
A. Categories ofSynovial Fluid based on Clinical & Laboratory Findings

PARAMETER I ,N~~TORY I INFLAMMATORYI SEPTIC I HEMORRHAGIC


Volume (knee) Often >3.5 Often >3.5 Often >3.5 Usually >3.5

Clarity Transparent Translucent Opaque Bloody

Color Yellow Yellow Yellow Red

Viscosity High Low Variable Variable

WBC 0-2,000/mm 3 >2,000/mm 3 >20,000/mm 3 Variable

PMNs <25% 2:50% 2:75% 50-75%

Culture Negative Negative Often positive Negative

33
B. Microscopic Evaluation of Crystals in Synovial Fluid

CRYSTAL
I ASSOCIATED
CONDITION/$
• Gout and conditions
DESCRIPTION

Monosodium • Needle-shaped with negative/yellow


of impaired uric acid
urate birefringence in compensated polarized light
metabolism

Calcium • Rhomboid-shaped with positive/blue


• Pseudogout
pyrophosphate birefringence in compensated polarized light

Calcium
• Coffin-lid-shaped with no birefringence in
phosphate • Osteoarthritis
compensated polarized light
(apatite)

• Envelope- or pyramid-shaped with


Calcium • Renal diseases on
negative/yellow birefringence in
oxalate dialysis
compensated polarized light
• Hyperlipoproceinemia • Rhombic-shaped with negative/yellow
Cholesterol
& rheumatoid anhritis birefringence in compensated polarized light

• Post-intraarticular • Flat, small, polymorphic with variable


Corticosteroid
steroid injections birefringence in compensated polarized light

Case 5. A45/M presented at the ER for acute swelling of his left knee and left big toe. PE showed
swollen left knee and 1st metatarsophalangeal joint with warmth and tenderness. He reports
attending a party the night before with intake of beer and offal. Arthrocentesis was performed
which drained ~IO mL of yellowish, slightly opaque, non-viscous fluid. Synovial fluid WBC
was 3,000 with PMN predominance. Gram stain was negative.
Diagnosis: Gouty arthritis
• This is a typical case of a patient with gouty arthritis presenting with acute flare after intake of
beer & dietary indiscretion. Synovial fluid analysis is compatible with inflammatory arthritis
due to the high synovial fluid WBC with PMN predominance, plus a negative string sign.
Polarized light microscopy will likely reveal needle-shaped crystals with yellow birefringence.

REFERENCES
1. Brunzel,N. (2018).Fundamentalsof Urine and Body Fluid Analysis,4th Edition. St. Louis, Missouri: Elsevier.
2. Clinical L.1.boratoryTests - Reference Values. Available at hnps://www.royalcollege.ca/rcsite/documems/creden1ial-exams/
clinical-lab•tests•reference•values-e.pdf. Accessed December 5, 2020.
J. Dandona P,Nix D. Wilson 1'.·tF, et al Procalcitoninincreaseafterendotoxin injection in nom1alsubjects.The Journalof Clinical
Endocrinology& Metabolism.1994;79(6):1605-1608.
4- DickersonRN.Nitrogenbalanceandproteinrequirementsforcriticallyillolderpatients.Nutrients.2016;8(4):226.
doi:10.3390/nu8o40226
5. FenderEA and ZackCJ,Shining a new light on pericardia!fluid.Hean. 2021.107(19);1528-1529.
6.Gn.">Cr
JP,ArberDA GladerBE,et al.(2018).
Wintrobe's dinica1hematolomi14thedi.WoltersKluwerHealthPharmaSolutions(Europe)Ud.
7. Hrishi AP, Se1huramanM. Cerebrospinalfluid (CSF)analysis and interpretationin neurocricicalcare for acute neurological
conditions. IndianJ CritCareMed. 2019;23(Suppl 2):S115-S119,
8.JamesonJL,KasperDL,Longo DL,FauciAS, HauserSL,LoscalzoJ.Harrison'sPrinciplesof InternalMedicine.20th Edi1ion.New
York:McGrawHill Education,2018,
9.Kasi,gaE.The imponanceofstooltestsin diagnosisand follow-upofgastrointestinal
disorder.;in children.TurkPediatriAr.; 20(!);;4(3,-141-8
to. Kidney Disease: Improving Global Outcomes (KDIGO) CKD Work Group. KDIGO Clinical Practice Guideline for the
Evaluationand Managementof Chronic KidneyDisease. KidneyInter.,Suppl. 2013;3:1-150
n. LightRW,MacgregorMl, LuchsingerPC, BallWC Jr.Pleuraleffusions:the diagnosticseparationof transudatesand exudates.
Ann Intern Med. 1972Oct:77(4),507-13.
12. Lippi G. Plebani M. A modem and pragmatic definition of Laboratory Medicine.Clin Chem Lab Med. 2020:58(8),1171.
13. McPhersonRAand PincusMR.(2017),Henry'sClinical Diagnosis and Managementby LaboratoryMethods,23rded.St Louis,
Missouri:Elsevier.
14. PaganaKD,PaganaTJ,PaganaTN. (2019).Mosby'sDiagnostic& LaboratoryTest Reference.14thed. St Louis,Mo:Elsevier.
15. Poner,RS.The MerckManualof Diagnosis and Therapy,20th Edition.Hoboken,NJ:Wiley,2011.
16. Shelter DE, Russell As. (2021),Synovial fluid analysis.In Shmerling RH (Ed.),UpToDate.RetrievedJune 12,2021,from https://
www.uptodate.com/contents/synovial-fluid-analysis
17. Skoreck.iK, Chertow GM, Marsden PA, Taal MW, and Yu ASL (editors).Brenner and Rector'sThe Kidney,10th Edition.
Philadelphia,PA:Elsevier.2016.
18. Strasinger SK and Di LorenzoMS. (2014).Urinalysisand Body Fluids, 6th Edition. FADavisCompany.Thailand

34
ELECTROCARD
BASIC CONCEPTS IN ELECTROCARDIOGRAPHY
1. Electrocardiography
2. Standardization

0 BASIC STEPS IN ECG READING


1. Step 1: Determine Heart Rate
2. Step 2: Determine Rhythm
3. Step 3: Measure Intervals
4. Step 4: Determine QRS Electrical Axis
5. Step 5: Check for Chamber Enlargements
6. Step 6: Check for ST and T-wave Changes
7. Step 7: Check for Miscellaneous ECG Findings

0 TREADMILL EXERCISE STRESS TEST

0 PACEMAKERS & PACEMAKER RHYTHMS


1. Overview of Pacemakers
2. Pacemaker Rhythms
'--E-"-LE-"-C"-T'"""R-'-O'-C'"'"A""'"'R""""D_l-'-0-"-G"'""'RA'-"--P'""'H-'-Y
_______________ _
An electrocardiogram (ECG) is requested to determine cardiac rate & rhythm, and to
identify conduction disturbances, chamber enlargement, & other cardiac abnormalities

I. ELECTROCARDIOGRAM (ECG) LEADS


A. Limb Leads
0 Bipolar leads: I, II, III
0 Unipolar leads: aVF, aVR, aVL

B. Standard Chest/ Precordial Leads

Placement of Standard Chest Leads


V1: 4th ICS,rightparasternalborder
V2: 4th ICS,left parasternalborder
• V3: betweenV2 & V4
V4: 5th ICS,left midclavicularline
VS:5th ICS,left anterioraxillaryline
V6: 5th ICS,left midaxillaryline

V4R-V6R:for suspectedRV infarct V7-V9:for LV posteriorwall infarct


• V4R:5th ICS,rightmidclavicular line V7: left posterioraxillaryline, same
• V5R:5th ICS,rightanterioraxillaryline horizontalplaneas V4 to V6
• V6R:5th ICS,rightmidaxillaryline VB:midscapular,samehorizontalplane
as V7 andV9
V9: left paraspinalborder,same
horizontalplaneas V4 to V6
Source:KligfieldP,et al. Recommendations
for theStandardization
andInterpretation
of the ECG.Gire2007

37
II. THE P-QRS-T WAVES, SEGMENTS, AND INTERVALS
• The normal ECG is composed of different waveforms which represent electrical events
during each cardiac cycle
Waves are labeled starting with the P wave, followed by the QRS complex, and then the
ST-T-U complex
ORS Complex

~-P-R-,n-te_rv_a_l_~I Q

s
QT Interval

BasicComponentsof an ECGTracing.The P-waverefiectsatrial depolarization(activation).The


PR-interval(distancebetweenonsetof the P-waveto the onsetof the QRScomplex)refiectsthe
conductionfrom the atria to the ventricles.The PR-segment(betweenthe end of P-waveand
onsetof the QRScomplex)servesas the referenceisoelectricline of the ECG.The QRScomplex
representsventriculardepolarization.The first negativewave is the Q-wave.If the first wave is
not negative,then the QRS complexdoes not possessa Q-wave.The first positivewave is the
R-wave,usuallyfollowedby a negativewave (the S-wave).The ST-segmentcorrespondsto the
plateauphase(phase2) of the cardiomyocyteaction potential.The J-point is the pointwherethe
ST-segmentstarts. The T-waverefiectsrapid repolarizationof contractilecells. The QT-interval
(onsetof QRS complexto the end of the T-wave)refiectsthe durationof both ventriculardepo-
larizationand repolarization.The term interval implies that a wave is included,while the term
segment describesa normallyisoelectricportionof the ECG.

38
STANDARDIZATION
The ECG is a plot of voltage on the vertical axis versus time on the horizontal axis
The standard recording speed of an ECG tracing is 25 mm/sec, which makes each small
square equivalent to 0.04 secs (40 msec) on the horizontal axis
In terms of voltage on the vertical axis (amplitude), a standard marker is usually inscribed on -
the left of the tracing as an upright rectangle, with a height of IO mm (IO small squares= I mV)

I. STANDARD ECG MEASUREMENTS AT 25 mm/sec AND IO mm/mV


0.20 sec

: L-
7
J
5mmor
0.5mV

LJ
0.04 sec

Horizontal Axis
• I small square= I mm= 0.04 second (or 40 msec)
• 1 large square= 5 small squares= 5 mm= 0.20 sec (or 200 msec)

Vertical Axis
• IO small squares= IO mm= 1 mV
• I small square= I mm= 0.1 mV

II. ADJUSTMENTS IN VOLTAGE AMPLITUDE


Check if standard marker at the left side of the tracing follows rypical calibration of
IO mm (IO small squares)= I m V
For patients with large voltages (e.g., ventricular hypertrophy, thin patients),
amplitudes may be halved (i.e., 5 mm= 1 mV) so that the waves/complexes can fit on the
tracing paper and do not overlap with waves from other leads
For patients with small voltages (e.g., severe LV dysfunction, significant pericardia!
effusion, COPD, obese), amplitudes may be doubled (i.e., 20 mm= 1 m V) so that the
waveforms are easier to see and interpret
A B • '' I
: r: :. •I'
'; '
! •.•

Onvn

.. I·
.. r >.. • J':
C ,

Adjustmentsin ECGVoltageAmplitude.A. This is an exampleof a 10-mmstandardmarker.This


uprightrectangledenotesthat 10 mm= 1 mV,whichis the defaultsettingon ECGmachines.B.
In this tracingof a patientwith largevoltageamplitudes,the settingwas adjustedso that 5 mm
= 1 mV.To obtainthe actualamplitudeof the R wave,the measuredheightof 4.5 mmshouldbe
multipliedby a factorof two (4.5smallsquaresx 2 = 9 mm= 0.9 mV).
39
SECTION TWO
I IIBASIC STEPS IN ECG READING
Step 1: Determine rate
Step 2: Determine rhythm
Step 3: Measure intervals (PR-interval, QRS-interval, QT-interval)
Step 4: Determine QRS electrical axis
Step 5: Check for chamber enlargements
Step 6: Check for ST and T-wave change
Step 7: Check for miscellaneous ECG findings

STEP 1: DETERMINE HEART RATE


I. FOR REGULAR RHYTHM
The normal heart rate (HR) is from 60 to wo bpm
Tachycardia: HR> JOO beats per minute (bpm)
0

Bradycardia: HR< 60 beats per minute (bpm)


0

The most accurate way to determine the HR is by counting the number of squares
between two R waves (i.e., using the R-R interval)

HR= 1500 HR= 300


(No. of small boxes between R-R) (No. of big boxes between R-R)

HR= = 83 bpm
18
Countthe# of smallsquaresbetween2 successive
QRS complexes.The numerator(1500) is a
constant,& whendividedby 18(# of smallsquares
betweenQRScomplexes), yieldsa HRof 83 bpm.

II. FOR IRREGULAR RHYTHM


For irregular rhythms (R-R intervals are not equal, as in atrial fibrillation), count the
number of QRS complexes within a 6 second strip (30 large squares) & multiply by 10

HR= [Number ofQRS complexes within 30 large boxes] x IO

Exam le:
1 2 l 4 5 6 1 8 9 10 11 12 IJ 14 15 1& 17 18 19 20 21 22 23 24 25 26 27 28 29 30

HR 10 complexes x 10 A 6-secondstrip can be identifiedby counting30 largesquares(150


100 bpm small squares).Countthe # of QRS complexeswithin the 6-second
strip.Thisvalueis multipliedby 10 in orderto yieldthe HRin 1 minute.

III. EYEBALLING METHOD


• The sequence "300-150-100-75-60-50-43-37'' is used
Start counting from the first QRS complex that falls on a thick line
The next thick line is 300 bpm, the next thick line 150bpm, then 75 bpm, then so on & so forth
Stop the sequence upon encountering the next QRS complex

In the tracing on the left, the next QRS


,---,--~-,---,--,, complexwas encounteredat the 4th thick line
(countingfrom the QRS complexmarkedby
the "star"),which represents75 bpm. This is
the approximateHR of this strip.
40
STEP 2: DETERMINE THE RHYTHM
The first step is to check for P waves: shape, morphology, and axis
The QRS morphology is then examined:
0 R-R intervals are normally equal (i.e.,peaks ofR waves are equidistant from one another)
0 A normal QRS duration <0.12 secs indicates that the rhythm is supraventricular (i.e., -
originating at or above the AV node)
0 Wide QRS complexes (~0.12 secs) mean that the rhythm is ventricular in origin, or
supraventricular but with aberrant conduction (e.g., bundle branch blocks, !VCD)
The basic underlying rhythm should be included in the official reading (e.g., RSR, AF)
Include premature beats (e.g., PAC, PVC) or nonsustained arrhythmias (e.g., runs ofVT)
I. SINUS RHYTHM
' .........
.
. ' ......
.. .. .... .. ...
. .. .. ..
..
.............
.......... ..
. ......................
.

• Rhythm is normally determined by the sinoatrial node, which fires at 60-100 bpm
• P-wave is normally upright in lead II (and usually in leads I, aVL, and a VF)
• Each p-wave is followed by a QRS complex, & each QRS complex is preceded by a p-wave
• The distance between the R-R intervals should be equal
• May be fast (sinus tachycardia) or slow (sinus bradycardia)
B. Sinus Arrh thmia
.. ...: . ....
.. .. :........ : .

• Sinoatrial node discharges irregularly (sinus node rate varies with the respiratory cycle)
• Rate: normal (still within 60-100 bpm)
• Rhythm: variation in the P-P interval or R-R interval ~120 msecs (or the shortest and
longest of these intervals differ by >IO%)
• P-waves, PR Interval and QRS: normal

II. ESCAPE RHYTHMS


• When there is severe sinus bradycardia, a significant sinus pause (clinically >3 seconds),
or sinus arrest, an escape rhythm may take over
unctional (Atrioventricular) Rh thm

• Pacemaker: AV junction with a ventricular rate of 40 to 60 bpm


• P wave: may appear before, after or buried within the QRS complex
• Rhythm (R-R interval): regular
• QRS complex: narrow (<0.12 sec)

, Pacemaker: His-Purkinje system (HPS) with a ventricular rate between 20 to 40 bpm


• P wave: absent
• Rhythm (R-R interval): regular
• QRS complex: wide (~0.12 sec)
41
III. SINOATRIAL (SA) NODAL DISORDERS
A. Sinus Pause

• Temporary cessation of sinus node activity


• May be synonymous with sinus "arrest" - which pertains to a prolonged sinus pause
(definition of"prolonged" is arbitrary)
• Difference from sinus exit block: the sinus pause is not a multiple of the normal P-P interval
• In the image above, the arrows denote the supposedly regular P-P intervals. Note the
absence of expected p waves at the positions of the 4th and 5th arrows, consistent with
sinus pause. This pause is not a multiple of the normal P-P interval.

• Failure of impulse transmission


• No visible P-QRS-T complex for;, 1cycle, wherein the P-P interval of the pause is an
exact multiple of the normal P-P interval (differentiating it from sinus pause)
• In the image above, note the atrial rate (p wave) is 75 bpm. The 4'h arrow indicates the expected
appearance of a p wave, but atrial depolarization does not occur (due to failure of impulse
transmission). Note that the pause is a multiple (2x in this case) of the normal P-P interval

IV. ATRIOVENTRICULAR BLOCKS


• Atrial impulse is conducted to the ventricles with a delay or is not conducted at all
• By increasing severity, the block can be classified into first, second, or third degree AV block

• Prolonged PR interval (>0.20 sec or >5 small squares)


• P-wave is always followed by a QRS complex, but with a slight delay
• In the image above, the PR intervals are consistently 0.24 seconds (6 small squares)

--:i::::;::::i:::r:r::1:<::
1:~: iTT/1':
::::~:: i !:::::::::~:: !i:X::::i:::::::~::
:::::j:::: i::::
!:::
• PR interval progressively lengthens, then the impulse is blocked (P is not followed by
QRS, resulting in a dropped beat)
• In this example, the 1st PR interval is 0.16 secs, 2nd PR interval is 0.20 secs, and 3rd PR
interval is 0.28 secs. The PR interval is progressively lengthening followed by a P wave
without a QRS (marked X). After the dropped beat, the PR interval is reset to 0.16 secs

• PR intervals of conducted beats are constant in length; beats are dropped with no warning
• PR intervals may be normal or prolonged
• Contrary to Mobitz Type 1, the PR interval in this example is consistently 0.16 secs prior
to the dropped/non-conducted beat (marked X)
42
• Ratio of P waves to successfully conducted QRS complexes is at least 3:1or higher
• Impulses from atria are blocked at the level of the AVnode & only reach ventricles occasionally
• In contrast to complete heart block, the PR intervals are constant (0.16secs in this example).
Every 3rd p wave is successfully conducted to the ventricles (3:1conduction ratio).
-
• P and QRS waves occur regularly but are independent of each other
• No consistent relationship between atrial and ventricular activity
• P-P intervals and R-R intervals are constant
• Escape rhythm is either junctional (narrow QRS) or ventricular (wide QRS complex)
• In the image, the atrial rate (P wave) is 75 bpm while the ventricular rate (QRS
complex) is 40 bpm, both of which are constant but with no consistent relationship
between atrial & ventricular activity

V. TACHYCARDIAS ORIGINATING FROM THE ATRIA


A. Atrial Fibrillation (AF)

• Description: rapid, erratic electrical discharge from multiple atrial ectopic foci
• Rate: atrial rate is " 350 bpm; ventricular rate varies
• Rhythm: irregularly irregular
• P-waves: no discernable P-waves
• QRS: usually normal (narrow complex)

• Description: organized but rapid atrial activity


• Rate: atrial rate is 250-350 bpm; ventricular rate varies
• Rhythm: variable (depending on AV conduction); usually irregular but can be regular
if with some degree of AV block
• P-waves: saw-tooth appearance
• QRS: usually normal (narrow complex)
C. Focal Atrial Tach cardia (AT)

• Description: Single focus (distinct from the SA node) in the atria that is exhibiting
automaticity & discharging at a faster rate than the SA node, effectively overriding it
• Rate: Fast(> 100 bpm)
• Rhythm: Regular
• P-waves: discrete; with morphology different from that of sinus P waves
• PR Interval: variable
• QRS: normal
43
• Description: similar to atrial tachycardia (see above) but with multiple discharging
foci; often seen in patients with pulmonary disease
• Rate: fast; irregular atrial rate (> 100 bpm)
• Rhythm: Irregular
• P-waves: ~3 different forms/morphologies
• PR Interval: variable
• QRS: normal

VI. TACHYCARDIAS INVOLVING THE AV NODE


This subset of supraventricular tachycardias (SVTs) has ventricular rates so fast that the
P waves may not be visible
AVnode is involved as pan of the reentrant circuit that initiates &/or propagates the arrhythmia

General Description of Paroxl,/smal Sur,raventricular Tachl,/cardias


Rate . 150-250bpm
Rhythm . Regular
PWave Frequently buried in preceding T wave
PR Interval . Usually cannot be measured
QRS . Normal, but may be wide if with aberrant ventricular conduction

• Most common paroxysmal sustained tachycardia in healthy young adults


• More common in women
• Reentrant circuit involves fast and slow pathways within the AV node
• Regular tachycardia with retrogradely conducted P waves visible at the end of the
QRS complex or not visible at all (buried in QRS):
"Pseudo-S wave" in inferior leads (II, III, a VF) (see arrows on image on the left)
0

"Pseudo-r' wave" in lead V1 (see arrows on image on the right)


0

• Narrow QRS tachycardia with a short RP interval(< 70 msec)

I 1---1,.
RP> 70 msec

• Paroxysmal SVT with its reentrant circuit involving the AV node & an accessory
pathway connecting the atria & ventricles (basically acts as a "shortcut" that bypasses
the AV node)
• Regular narrow-QRS tachycardia with a short RP interval (but in contrast to
AVNRT, RP interval is >70 msec)
• During sinus rhythm (when patient is not having an episode of tachycardia), the
Wolff-Parkinson-White or WPW pattern may be observed in the resting 12-lead ECG

44
VII. VENTRICULAR TACHYCARDIAS
Wide QRS tachycardias (~120 ms or 3 small squares) are usually ventricular in origin
Differentials for wide-QRS tachycardia:
Ventricular tachycardia (VT): more common
0

Supraventricular tachycardia (SVT) with aberrancy


0 -
0 Pacemaker rhythm
When faced with a wide-complex tachycardia and the morphology is in question (VT vs. SVT
with aberrancy), it is safer to treat as VT (more common & more life-threatening differential)

A. Ventricular Tachycardia (VT)


0~3 consecutive wide QRS beats at a rate >IOO bpm
0P waves not usually visible (buried in QRS complexes)
0P waves can occasionally be seen in some cases of VT, but there is no consistent
relationship between P waves and QRS complexes (i.e., AV dissociation)
0May be broadly classified based on morphology, duration, and presentation

• Ventricular focus with automaticity Beat-to-beat variations in appearance


produces a rapid sequence of bizarre (changing QRS morphology)
PVC-like wide QRS complexes (all Torsades de Pointes (TdP):
beats have the same appearance) 0 Specific type of polymorphic VT
• Very fast rate of -250 bpm & originates that occurs in the context of a
from a single focus within the ventricles prolonged QT-interval (not due to
• Wide QRS complexes resembling ischemia)
successive PVCs of similar appearance 0 Presents with an oscillating pattern
or morphology mimicking a "turning (twisting) of
• There are also no discernible p waves the points" stitching pattern (waxing
in this example and waning QRS amplitude)

2. VT According to Duration
• Sustained: VT that lasts ~30 seconds, or requiring immediate termination due to
hemodynamic instability
• Non-sustained: VT that self-terminates within 30 seconds (presence of at least 3
successive PVCs is already considered a run of VT)

3. VT According to Presentation
• Pulseless VT: no effective cardiac output (no pulse, no BP)--->defibrillate (treat as VF)
• Unstable VT: with pulse, but hypotensive---> electrical cardioversion
• Stable VT: with pulse and normal BP--->medical/pharmacologic cardioversion

·············· ............ .

···:····!····:····:·
• Continuous irregular activation with no discrete QRS complexes
• Associated with coarse or fine chaotic undulations
• No identifiable or distinct P waves, QRS complexes, or Twaves
• Rate is very fast, but there is no effective cardiac output
45
STEP3:MEASUREINTERVALS
I. NORMAL ECG INTERVALS AND DURATIONS

PARAMETER

Heart Rate
I
NORMAL INTERVAL &
DURATION
60 - 100bpm
I IN SMALL SQUARES

15-25 small squares


Bradycardia <60 bpm >25 small squares
Tachycardia >100bpm <15 small squares
Duration <0.12secs <3 small squares
Pwave
Amplitude<0.25mV <2.5 small squares
PR Interval 0.12- 0.20 secs 3-5 small squares

QRS Duration <0.12secs <3 small squares

QT Interval (corrected) 0.39- 0.46 secs


Male :S0.45secs
Female :S0.46secs
Sources:
Rautaha~uPM,etal.AHNACCF/HRS. Ore2009.
Goldberger
AL,et al. Goldberger's
ClinicalElectrocardiography:
8th Ed.2012.

II. CORRECTED QT INTERVAL (QTc) USING BAZETT'S FORMULA


Done to adjust for abnormal heart rates (HR <60 or >IOO 6pm)
Normal values (depending on sex):
Qn=--'(~Q~T~a~c~t~u~al~i~n~s~e~c)'----- • :S450msec in men
interval in sec)
• :S460msec in women
Source:LuoS, et al.A comparison
of commonlyusedQTcorrectionformulae.J Electrocardiol
2004.

Example:
Computation ofQTc:

Qn=---(Q~T_ac_t_u_al_i_n_se_c_)
__ _
interval in sec)

QT actual = 7 small boxes x 0.04 soc/small box = 0.28 sec QTc = = 0.40 sec
R~R Interval= 12 small boxes x 0.04 sec/small box= 0.48 sec

III. CORRECTED QT INTERVAL (QTc) IN ATRIAL FIBRILLATION (AF)


In patients with AF with slow or rapid ventricular response, follow these steps:
0 Determine the longest and shortest RR intervals in the AF strip
0 Measure the QT intervals that immediately follow these RR intervals
0 Apply Bazett's formula for each pair of RR and QT intervals
• Average the two values to determine the QTc for the entire strip
LongeJtRR,

QTc1 + QTc2
QTc=
2

Source:At-KhatibSM,et al. WhatcliniciansshouldknowabouttheQTinterval.JAMA.2003


46
STEP 4: DETERMINE QRS ELECTRICAL AXIS
Axis refers to the direction of the mean QRS vector in the frontal plane or the hexaxial system
In adults, the normal QRS electrical axis in the frontal plane is between -30 & 90°
(directed downward or inferior and to the left)
~•-V_a_lu_e_s_t_h_a_t_d_o_n_o_t_(_a_ll_b_e_t_w_e_e_n_t_h_e_se_n,.u_m_b_e_r_s_a_re_a_b_n_o_rm_a_l
_________ --, -
....
aVF•

-90°

,.,.

ono
....
aVF+

Interpretation:
• Normal Axis: -30° to +90°
• Right Axis Deviation (RAD): +90° to +180°
• Left Axis Deviation (LAD): -30° to -90°
• Extreme Axis Deviation: -90° to +180°
Source:Goldberger
AL, et al. Goldberger's
ClinicalElectrocardiography:
A Simplified
Approach8thEd.2012.

I. QUADRANT METHOD (LEADS I & aVF)


• Simplest way to determine the axis is to check leads I and a VF
A positive QRS complex in a lead has a ventricular axis that is in the same direction as
that lead, while a negative QRS complex in a lead has an axis that is in the direction
opposite to that lead
INTERPRETATION I LEAD I I LEADaVF
Normal Axis Positive Positive

Left Axis Deviation Positive Negative

Right Axis Deviation Negative Positive

Extreme Axis Deviation Negative Negative

II. THREE-LEAD ANALYSIS (LEADS I, aVF, & II)'


• More accurate than the quadrant approach
• Takes into account leads I, a VF and II
INTERPRETATION I LEAD I I LEAD II I LEAD aVF
Normal Axis Positive Positive Positive I Negative

Left Axis Deviation Positive Negative Negative


Right Axis Deviation Negative Positive Positive

Extreme Axis Deviation - Negative Negative Negative


*Three-lead analysis is a refinement of the Quadrant Method & highlights the utilityof lead II in distin-
guishing between normal axis and LAD.
47
III. VECTOR ANALYSIS
Compute the net voltage amplitudes of the QRS complexes in leads I and aVF
• Plot the computed values on the hexaxial system along leads 1(x axis) and aVF (y axis)
Draw lines perpendicular to each lead axis
From the origin (zero point), draw a third line towards the point of intersection of the
first two perpendicular lines
The angle formed by this third line represents the frontal axis for the ECG tracing

_____J

.90
,v,
VectorAnalysisforDetermining FrontalAxis.Inthisexample,the net amplitudesof I and aVFare +10and +12
respectively.Plottingthese amplitudesas vectorson the grid,perpendicularlinesare droppedfromeach axis
(brokenlinesinthe diagram)and intersectat a commonpointinthe southeastquadrant,whichis withinnormal
axis territory.Ifwe drawa linefromthe originto the pointof intersection,thisformsan angleof approximately
+50degrees,and thisthereforecorrespondsto the frontalaxis forthe ECG.

STEP 5: CHECK FOR CHAMBER ENLARGEMENTS


I. ATRIAL ENLARGEMENT

CHAMBER DESCRIPTION

"Tall P waves" • Peaked P waves with amplitudes in

. . lead II of> 0.25 mV ("P pulmonale")

..::K·
... . :
Right Atrial • Prominent initial positivity in lead V1
=
Enlargement/
Abnormality . .. ' or V2 > 0.15mV (1.5mm at usual gain)
• Increased area under initial positive
. . portion of the P wave in lead V1to
>0.06 mm-sec

• Prolonged P wave duration (~ 120 msec)


• Prominent notching of P wave, usually
most obvious in lead II, with the

Left Atrial .. . . . . .... . . . . ..


••• ••••
"Wide P waves"
• •••• e1a111
interval between notches ,0.04 msec
("P mitrale")
Enlargement/
r\J

. . • • • 4 • • • •
• Ratio between duration of P wave in lead

.. . . . .
Abnormality II & duration of the PR segment > 1.6
: :·~····:

. • ,t I I I I~ • • •
• Increased duration & depth of
terminal-negative portion of P wave in
lead V1(P terminal force) so that area
subtended by it is > 0.04 mm-sec
'Figuresabovecomparefeaturesof rightatrialenlargementversusleftatrialenlargement.The P waveis best
examinedinthe inferiorleads (II,111,
and AVF),as wellas V1and V2.In RAA,the P waveis characteristically
tall(or has a prominentlypositiveinitialportionin lead V1). In LAA,the P waveis wideand notched/bifid
in
lead II,and is biphasicwitha prominentlynegativeterminalportionin leadV1.
48
II. LEFT VENTRICULAR HYPERTROPHY (LVH)
V1 V6 . ...

-
Sokolow-Lyon Criteria:
• [S in V1] + [R in V5 or V6] >35 mm, OR
• R in aVL >11 mm

:20mm 24mm. Cornell Criteria:


• [S in V3] + [R in aVL]
2:20 mm in females
0

2:28mm in males
0

In thisexample,theS in V1 is 20 mmandtheR in V6 is 24 mm.UsingtheSokolow-Lyon


criteria,thissatis-
fiesthecriteriafor leftventricular (20mm+ 24 mm= 44 mm).
hypertrophy

III. RIGHTVENTRICULARHYPERTROPHY(RVH)
V1 . : ·: ::, :··, V6 Some Criteria for RVH
. • R in V1 2:0.7mV
.
. • QR inV1

.
. I
. 6mm
• R/S in V1 >1 with R >0.5 mV
• R/S in V5 or V6 <1
• Sin V5 or V6 >0.7 mV
• R in V5 or V6 2:0.4mV with S in V1 $0.2 mV
• Right axis deviation (>+90°)
• S1-Q3 pattern
• S1-S2-S3 pattern
• P pulmonale
Assumingthatthetracinghasrightaxisdeviation,leadV1 is suggestive
of rightventricular
hypertrophy
becauseR/S>1 & R >0.7mV.Lookingat leadV6,R/Sratiois <1,againconsistent withRVH.
Source:
Murphy forleft,right& combined
ML,etal.ECGcriteria cardiac
ventricular AmJ Cardiol
hypertrophy. 1984
Hancock
EW,etal.AHNACCF/HRS Standardization
& Interpretation
oftheECG.Circ2009

STEP 6: CHECK FOR ST AND T-WAVE CHANGES


I. THE CONTIGUOUS LEADS
Refer to leads that are grouped together to represent specific walls of the heart
At least two consecutive leads demonstrating ST-T wave changes are required to satisfy
criteria form ocardial ischemia/infarction

II, III, aVF • Inferior wall


I,aVL • High lateral wall
• Septa! wall
• Anterior wall
• Lateral wall
• Anteroseptal wall
V3-V6,I,aVL • Anterolateral wall
V5, V6, II, III, aVF • Inferolateral wall
• Posterior wall
Almost All Leads • Diffuse, massive
V3R, V4R (right-sided leads) • Right ventricular wall
Forexample,if thereareSTsegmentdepressions>1 mmin leadsV1to V3,thenwelabelit as anteroseptal
wallischemia.If STsegmentdepressions
occurin V3 to V6,thenwe labelit as anterolateral
wallischemia
Sourte: Wagner
GS,etal.AHNACCF/HRS. Circ2009
49
II. VARIABILITY OF ECG PATTERNS WITH ACUTE MYOCARDIAL ISCHEMIA
PATHOLOGY
I POSSIBLE ECG FINDINGS
Subendocardial Ischemia
• Transient ST-depressions
(Non-infarcted)
• Transient ST-elevations or paradoxical T-wave
Transmural Ischemia
normalization
(Non-infarcted)
• Sometimes followed byT-wave inversions
• ST-depressions
Non-ST Elevation Myocardial
• T-wave inversions
Infarction (NSTEMI)
• NoQ-waves
• Hyperacute T-waves and persistent ST-elevations
ST-Elevation Myocardial
• T-wave inversions
Infarction (STEMI)
• Development ofQ-waves
Source:Goldberger
AL,et al. Goldberger's
ClinicalElectrocardiography
8th Ed.2012.

III. CRITERIA FOR ECG MANIFESTATIONS OF MYOCARDIAL INFARCTION


MANIFESTATION I CRITERIA
• New horizontal or downsloping ST depression;,, 0.05 mV
(0.5 mm) in two contiguous leads (e.g., ST-depression in
ST-Depression leads V3-V6 suggests anterolateral ischemia)
• ST depression ;,,o.rm V (I mm) in ;,,8surface leads+ ST-elevation
in aVR &/or Vr suggests left main (or equivalent) obstruction
• T-wave inversion;,, 0.1 mV (1mm) in two contiguous leads
T-Wave Inversion
with a prominent R wave or R/S ratio >I
• New ST elevation at the J point ;,,a.I mV (1 mm) in two
contiguous leads (applies to all leads EXCEPT V2-V3)
ST-Elevation (STE) • In leads V2-V3, the following cut-off points apply:
(in absence ofCLBBB) 0 ;,,0.20 m V in men ;,,40years

0 ;,,0.25mV in men <40 years

0 ;,,0.15mV in women
ST-Elevation (STE) • The Sgarbossa criteria is used to identify AMI in a patient
(in CLBBB or RV with complete LBBB or RV pacing (see next section/table)
Pacing) • A score;,, 3 points has 90% specificity for STEM I
• > 0.04 sec (1 mm) wide
Pathologic Q Waves • > 0.2 mV (2 mm) deep
• > 25% ofQRS complex amplitude

• T-wave inversion, ST segment depression/elevation not


Non-specific fulfilling criteria for ischemia or infarction (as outlined above)
ST-T Wave Changes • Flattened or slightly inverted T-waves
• ST segments slightly above or below the isoelectric line
Recall:
0.1mV= 1 mm= 1 smallsquare
0.5mV= 5 mm= 5 smallsquares
1.0mV= 10mm= 10smallsquares
Sources:O'GaraPT,et al. 2013ACCF/AHA guidelinefor themanagementof STEMI.Circ2013.
IbanezB, et al. 2017ESCGuidelines
for themanagement of STEMI.EurHeartJ 2018.

so
IV. SGARBOSSA CRITERIA
Used to identify AMI in a patient with complete LBBB or RV pacing

-
• A score;, 3 oints has 90% s ecificit for STEM!
ST-elevation;, 0.1 mV ST-elevation;, 0.5 mV
ST-depression;, 0.1 mV (5 mm) and discordant
Criterion (I mm) and concordant
(1 mm) in V1, V2, or V3
with QRS complex with QRS complex

Illustration

Points 5 points 3 points 2 points


EB,etal. NEJM1996.
Source:Sgarbossa
V. CLASSIFICATION AS TO TIMING OF STEM!
Hyperacute I Acute I Hours
............
. . . . . .. . . . .

• Typical STE appear • Persistence of STE


• Peaked Twaves
• T waves may still be upright • Q waves start to appear

Days 1-2 I Days Later I Weeks Later

• STE may still be present


• ST segments normalize • ST segments and T waves
• R wave forces diminish
• T waves are still inverted normalize
• T wave inversions appear
• Significant Q waves present • Q waves are persistent
• Q waves become deeper

VI. POSTERIORLVWALLINVOLVEMENT
Posterior LV wall infarction, which is usually associated with lateral or inferior involvement,
may be indirectly recognized by reciprocal or "mirror-image" ST depressions in leads V1 to V3
For ST-depression in V1-V3(especially with positive terminal T-wave): ST-elevation(;, 0.5 mm)
in leads V7-V9 signifies a posterior wall MI

v,~-
_v_1~ ... . .. . . .. ..
. .
..
.
Al . .. .. ... . .. . ' ..
. .. . .
. . ' .·
. '!
. . :
.
..
. .. .
. ... . :. .. AS. . .. . . ..
ThefirstimageshowsSTsegment depression
in V1andV2,whichwhenrotated180°andviewedagainstthe
lightfrombehindthetracingpaper(imageontheright)is similarto a septalwallinfarct.
51
VII. RECIPROCAL CHANGES
Pertains to ST-depression in leads opposite those demonstrating ST-elevation
Some examples:
0 Anterior MI (V3-V4):reciprocal changes in inferior wall (II, III, aVF)
0 Inferior MI (II,III, aVF):reciprocal changes in high lateral (I, aVL)or anterior wall (V3-V4)
Lateral Ml (I, aVL, V5-V6):reciprocal changes in V1 or inferior wall (II, III, aVF)
0 Posterior MI: reciprocal changes in anterior/anteroseptal wall (V1-V3)

Inferiorwall STEM!.Note the ST segmentelevationin leads 11,111,


aVF (inferiorwall STEM!)and
concomitant ST segmentdepressionin leadsI, aVLandV3-V4(reciprocalchanges).

STEP 7: CHECK FOR MISCELLANEOUS ECG FINDINGS


I. PREMATURE BEATS
A. Premature Atrial Complex (PAC)
, :·:·:·,·:
:·:::-:·::·:-;-:·:
....
,.........
,.........
,........ ?F:?!
·:·:·:rTr!Fl'!?:r
:.:.:.:.i.:.:.:.:.l.:.:.:.
.................................. :.1.:. i.:.:.:.:.1.~}.l.l.ti.;
:.:.:J.:.:.:.:. -~
-~-
j .i.:.!.i.
l.i.i.
~.:.~
• Premature P-waves (earlier than the next expected sinus P-wave)
• PAC P-wave has a different morphology compared to the sinus P-wave
• QRS is usually narrow (unless with associated bundle branch block)
• In image above, the 4th beat (arrow) appeared earlier than the expected sinus beat
• The PAC P-wave is narrower than the P-waves of the other sinus beats

• Prematurely occurring QRS complex which is wide and bizarre-looking


• Usually no preceding P wave (but may occasionally have a retrograde P wave
occurring after the PVC)
• T wave is opposite in deflection compared to the bizarre QRS complex
• In image above, the 4th QRS complex is broad (2: 120 msec) with an abnormal
morphology. There is no preceding P wave, and discordant ST segment and T wave
changes can be seen.

52
Some Variations in PVCs
VARIATION I DESCRIPTION I SAMPLE TRACING

Bigeminy

Trigeminy
• PVCs alternate
with sinus beats

• PVC occurs after


every 2 sinus
beats
-
• 2 successive PVCs
• 3 successive
Couplets PVCs are already
considered non-
sustained VT

II. BUNDLE BRANCH BLOCKS (BBB)


Delayed conduction in the ventricle supplied by the diseased bundle branch, resulting
in a widened QRS complex and other characteristic morphologic features
When identifying BBBs, examine the morphology ofQRS complexes in leads V1 & V6
BBB I ILLUSTRATION I CRITERIA*

&·····
• QRS duration ~120 ms
Complete " .. ; .. · ..
.
• rsr', rsR', or rSR' in V1 or V2
• Slurred S-wave of greater
Right Bundle ' . . ... . duration than R wave, OR
Branch Block
>40 ms in I & V6
(CRBBB)
• Normal R peak time in V5 &

\(?~¾
V6 but >50 ms in V1
• QRS duration ~120 ms
Complete . . . • Broad notched or slurred R
wave in I,aVL, V5 & V6
Left Bundle
• OccasionalRS pattern in V5& V6
Branch Block
: ... • Absent Q waves in I, V5, V6
::· . ·.
:
(CLBBB) : '
.: ·. ' • ST and T waves usually
opposite in direction to QRS
*IncompleteRBBBor LBBBwill havethe samecriteria,butthe QRSdurationis <120 msec
Sources:
Surawicz
B,et al.AHA/ACCF/HRS.
Gire2009& KusumotoFM,et al. 2018ACC/AHA/HRS.
Gire2019

III. OTHER FINDINGS


Poor R wave • R-wave in leads V1-V3is< 0.3 mV (3 small squares)
progression • Normal R-wave in leads V4-V6
• QRS complexes < 5 small squares in limb leads or< IO small squares in
Low voltage precordial leads
complexes • Clinical scenarios: COPD, anasarca, obesity, myocarditis, moderate-
sized to massive pericardia! effusions
• Hypokalemia: prominent U waves+ flattened T waves
• Hyperkalemia: peaked T-waves IO mm, widened QRS, prolonged PR
Electrolyte
interval, loss of P waves, eventually sine wave pattern
abnormalities
• Hypocalcemia: prolonged QT interval
• Hypercalcemia: shortened QT interval
Source:Prutkin,JM.ECGTutorial:Miscellaneous
Diagnoses.
UpToDate
53
SECTION THREE
TREADMILL EXERCISE STRESS TEST

TREADMILL EXERCISE TEST (TET)


I. PURPOSE OF EXERCISE STRESS TESTING
• Diagnosis & evaluation of coronary artery disease (CAD) or ischemic heart disease (!HD)*
• Assessment of cardiovascular risk & prognosis of CAD'
• Assessment of therapeutic response
• Assessment of perioperative risk for noncardiac surgery
• Exercise prescription
*Asidefromthe diagnosisof CAD,TETalsoprovidesprognosticinformationsuchas heartrateand
bloodpressureresponseto exercise,exerciseduration,and maximumworkloadachieved(usually
measuredin METSor metabolicequivalents)

II. SCREENING AND PREPARING FOR TREADMILL EXERCISE TESTING


A. An Exercise ECG (TET) is recommended as the initial diagnostic/prognostic test if:
0 Resources and local expertise for a stress imaging study are not available
0 Patient has an intermediate pre-test probability
0 Patient's resting ECG is normal
0 Patient is able to exercise adequately to reach target heart rate

B. Resting ECG Findings That Can Affect the Interpretation ofTET Results•
• Left ventricular hypertrophy
• Conduction disturbances (LBBB, RBBB, left anterior fascicular block, IVCD)
• Arrhythmias (atrial fibrillation, frequent PVCs)
• Wolff Parkinson White pattern
• Paced ventricular rhythm
• Electrolyte imbalance
• Digoxin-related ST-T wave changes
*If anyof theserestingECGfindingsare present,anotherdiagnostictest suchas stressimaging
maybe moreappropriatefor diagnosisof CAD
Sources:SIHDTaskForce,CPGforDiagnosisand Management
of PatientswithCAD.PHA2014

III. CONTRAINDICATIONS TO EXERCISE STRESS TESTING


ABSOLUTE I RELATIVE
CONTRAINDICATIONS CONTRAINDICATIONS
• Acute Ml (within 2 days) • Known left main coronary artery stenosis
• High-risk unstable angina • Moderate to severe aortic stenosis with
• Uncontrolled cardiac arrhythmia uncertain relationship to symptoms
with hemodynamic compromise • Tachyarrhythmias with uncontrolled ventricular rates
• Active endocarditis • Acquired advanced or complete heart block
• Symptomatic severe aortic stenosis • Hypertrophic cardiomyopathy with a severe
• Decompensated heart failure resting gradient
• Acute pulmonary embolism or • Recent stroke or transient ischemic attack
pulmonary infarction • Mental impairment & limited ability to cooperate
• Acute myocarditis or pericarditis • Resting hypertension (>200/110 mmHg)
• Physical disability precluding safe • Uncorrected medical conditions (e.g.,
and adequate testing electrolyte imbalance, hyperthyroidism,
significant anemia)
Source:FletcherGF,et al. Exercisestandardsfortestingand training.Circ2013

54
IV. PERFORMING THE STRESS TEST
A. Im ortant Formulas in Stress Testin
• Highest HR an individual can achieve without
Maximum HR= 220 • Age
severe problems through exercise-induced stress
• Specific age-based HR maintained during aerobic
Target HR= Maximum HR x 0.85 exercise to ensure optimal cardiovascular function
• It is computed as 85% of the maximum heart rate

• Commonly used in apparently healthy individuals


Standard
• Has 3-minute periods to allow achievement of a steady state before
Bruce Protocol
workload is increased for next stage
Modified • Commonly used in elderly patients and in patients whose functional
Bruce Protocol capacity is limited by a possible cardiac disease
• Commonly used for patients with limited exercise tolerance
NIH Protocol
• Protocol commonly used for functional capacity testing after an ACS

C. Main Phases of a Stress Test Protocol


PHASE I DESCRIPTION
• Consent is secured & patient is instructed regarding what to expect
Patient • Leads are attached
preparation • Removing the influence of anti-ischemic drugs improves sensitivity
for diagnosing CAD significantly (e.g., beta-blockers)
• Patient begins running on the treadmill
• Physician monitors patient's symptoms & ECG for signs of ischemia
Testing • Ends when patient expresses desire to stop or physician orders for
proper termination
• For results to be considered conclusive, patient should have
achieved at least 85% of the maximum predicted HR (seeformula above)
• Patient stops running and is allowed to cool down
Recovery
• Physician should still be present to monitor the patient, since
phase
diagnostic ECG findings may appear only during the recovery phase

• Patient's request to stop • Marked ST displacement(> 2 mm horizontal


• ST elevation (> 1.0 mm) in leads or downsloping) and suspected ischemia
without Q waves from previous MI • Drop in SBP >IO mm Hg, despite an increase
(other than a VR, aVL, or V1) in workload, in absence of other evidence
• Drop in SBP of >IO mm Hg, despite an ofischemia
increase in workload, when accompanied • Increasing chest pain
by any other evidence of ischemia • Fatigue, shortness of breath, wheezing, leg
• Moderate to severe angina cramps, or claudication
• Central nervous system symptoms (e.g., • Arrhythmias other than sustained VT,
ataxia, dizziness, or near-syncope) including multifocal ectopy (PVCs with
• Signs of poor perfusion (cyanosis or pallor) different morphologies), ventricular triplets,
• Sustained VT or arrhythmia that SVT, AV heart block, or bradyarrhythmias
interferes with normal maintenance of • Exaggerated hypertensive response (systolic
cardiac output during exercise BP >250 mmHg &Jor diastolic BP >115mm Hg)
• Technical difficulties monitoring ECG • Development of a bundle branch block that
orSBP cannot be distinguished from VT
Source:FletcherGF,et al. Exercisestandards
for testingandtraining.Gire2013
55
VI. INTERPRETATION OF ECG CHANGES DURING EXERCISE
The PQ or PR segment (segment between the end of the P wave and the onset of the
QRS) is used as the isoelectric reference line (or baseline)
The point of ST-segment measurement is 80 milliseconds after the Jpoint (junction of
QRS complex and ST segment), termed as ST8o
ECG abnormalities should be present in at least three consecutive beats to be
considered diagnostic/significant
PATTERN I TRACING I INTERPRETATION & DESCRIPTION

No ST
• Negative stress test
Depression

• May be a typical response to exercise,


especially in older, apparently healthy
individuals
Rapid
• Slow-upsloping ST-depressions (ST8o
Upsloping ST
depression ;,0,15 m V with ST-segment
Depression
slope more positive than +1.0 mV/sec)
may suggest ischemia in those with a
high pretest probability for CAD

• Consistent with myocardial ischemia


Horizontal ST • ST8o depression;, 0.1 mV (1 mm)
Depression • ST-segment slope from -1.0 mV/sec to
+1.0 mV/sec

• Consistent with myocardial ischemia


Downsloping • ST8o depression ;, 0.1 m V (r mm)
ST Depression • ST-segment slope more negative than
-1.omV/sec

... . ., ..................................
..... ' . . .. .. ' ..........
.
.
.. ..
·····:····:····:····:
....................
ST-Segment
... . . . . ... . . . ... . . . . ... . . . . ...
Elevation in a
. . . ....... ..
. . . . . ..... .····················•····
.···········
. .. • Represents a severe ischemic response
.. . . .. ... . . .. ........
~
Non-Q-Wave ... .. .. . . .. ..... . . ...' .........
. . .... .. . . .. • ST8o elevation;, 0.1 mV (I mm)
Lead .. .., ....... ;,..... .
: .. '.: . ... ': . ... ·: . .... .:
.

*For ST-depressions, positivestandardtest responsesinclude horizontalor downsloping


depression, whereasupslopingSTdepression is consideredequivocal.All ST-depressions
< 1.0
mmadditionalfrombaselineis definedas negative.ST-segment elevationin thosewithoutprior
infarction(i.e.,non-Q-wave
lead)localizesthesiteof severetransientischemia.
Sources:LuongMW,et al.Stresstesting.BCMJ2016
FletcherGF,et al. Exercisestandardsforteslingand training.Circ2013

56
SECTION FOUR
PACEMAKERS·&PACEMAKERRHYlHMS
.;;;0-'-V=E.;..;R;..;.V=IE;;.;.W.;;_..;;;;O..;;..F"""P..;;..A.;_;;C""E""'M=A""'K"""E"-'R""'S
_____________ _
Pacemakers are indicated for patients with symptomatic bradycardia and no identified
reversible causes (e.g., ischemia, electrolyte derangements, thyroid dysfunction)
Single-chamber pacemakers have a pacing lead threaded through the subclavian vein
and superior vena cava, with its tip implanted in the endocardium of the RV
Dual-chamber pacemakers have both an RA and RV lead

PACEMAKER RHYTHMS
I. VENTRICULAR PACED RHYTHM
Pacing stimulus is initiated by a lead in the RV, resulting in ventricular depolarization
• RV & LV depolarization are not simultaneous, hence QRS complex is wide (similar to LBBB)
Good capture: each pacemaker spike ("blip") is followed by a QRS complex (pacing
stimulus successfully captures the ventricles)
Poor capture: some pacemaker spikes are not followed by a QRS complex

VentricularPacedRhythmwith lntenmittent Lossof Capture.Thepacingrateis still setat 75 bpm.However,unlike


the previousfigure,not all spikesare followedby an LBBB-likeORScomplex,whichis indicativeof pacemaker
malfunction(lossof capture).Out of the 6 ORScomplexesin this example,only the 1st,3rd, 5th, and 6th ORS
depictgoodventricularcaptureby the RV pacinglead.The 2nd beatis actuallyan intrinsicnarrowORScomplex
probablyconductedfromthe precedingsinusp wave,albeitwith somedegreeof AV delay(PR intervalis 0.24
secs).The4th ORScomplex,whichis alsonarrow,is likelya junctionalescapebeat,withnodaltissueservingas a
subsidiarypacemaker dueto the prolonged pausebetweenthe3rdand4th beats.Poorcapturecanbe explainedby
deviceproblems(e.g.,displaced or fracturedlead,pacemakergeneratornearingend-of-life)or patientproblems(e.g.,
myocardialischemialinfarction,electrolyteimbalance,uremia).

II. ATRIALPACEDRHYTHM
Pacing stimulus is initiated by a lead in the RA, resulting in atrial depolarization that is
then conducted down the AV node and to the ventricles
Pacemaker spike ("blip") is followed by a P wave and a narrow QRS complex
Cannot be used for patients with AV blocks (device assumes that AV nodal conduction
is normal and that an impulse that starts in the atria will effectively reach the ventricles)

Atrial PacedRhythmwith GoodCapture.Notethe pacemakerspikeprecedesonly the P wave.SinceAV nodal


conductionis preserved,
theatrialimpulsecanthenbeconducted
to theventricles,
producing
a narrowORScomplex.

57
III. ATRIOVENTRICULAR (AV) SEQUENTIAL PACED RHYTHM
Pacing stimuli are delivered in both the RA and RV, with a delay programmed into the
device to simulate the normal physiologic delay at the AV node
Pacemaker spikes preceding both the P wave and the QRS complex

AVSequentialPacing.Pacemaker
spikesarepresentbeforeboththe P waves& theQRScomplexes.

REFERENCES
1. Al-Khatib SM, LaPointe NM, Kramer JM. Califf RM. What clinicians should know about the QT interval. JAMA 2003 Apr 23-
30;289(16):2120-7.
2. Bernstein AD, Daubert JC, Fletcher RD, et al. The revised NASPE/BPEG generic code for antibradycardia, adaptive-rate, and
multisite pacing. Pacing Clin Electrophysiol 2002;25:260.
3. Fletcher GF, Ades PA, Kligfield P,et al. Exercise standards for testing and training: a scienrific statement from 1he American Hean
Association. Circulation. 2013 Aug 20;128(8):873-934.
4.Gibbons RJ, Balady GJ, Bricker JT, et al. ACC/AHA 2002 guideline update for e.xercise testing: summary article: a report of the
American College of Cardiology/American Hean Association Task Force on Practice Guidelines (Committee to Update the
1997Exercise Testing Guidelines). Circulation. 2002 Oct 1;106(14):1883-92.
5.Goldberger AL,Goldberger ZD, and Shvilkin A. Goldberger's Clinical Electrocardiography:A Simplified Approach, 8th Edition.
Philadelphia, USA: Saunders, 2012.
6. Hancock EW, Deal BJ, Mirvis DM, et al. AHNACCF/HRS recommendations for the standardization and interpretation of the
electrocardiogram: pan V: electrocardiogram changes associated with cardiac chamber hypenrophy: a scientific statement
from 1he American Hean Association Electrocardiography and Arrhythmias Commitree, Council on Clinical Cardiology; the
American College of Cardiology Foundation; and the Hean Rhythm Society. Circulation. 2009 Mar 17;119(10):e251-61.
7. Ibanez B, James S, Agewall S, et al. 2017ESC Guidelines for the management of acute myocardial infarction in patients presenting
with ST-segment elevation: The Task Force for the management of acute myocardial infarction in patients presenting with ST-
segment elevation of the European Society of Cardiology (ESC). Eur Heart J.2018 Jan 7:39(2):119-1n-
8. Kligfield P,Genes LS, Bailey JJ,et al. Recommendations for the standardization and interpretation of the electrocardiogram: pan
I: The electrocardiogram and its technology: a scientific starement from the American Heart Association Electrocardiography
and Arrhythmias Commiuee, Council on Clinical Cardiology; the American College of Cardiology Foundation; and 1he Heart
Rhythm Society. Circulation. 2007 Mar 13;115(10):1306-24.
9.Luo S, Michler K. Johnston P, and Macfarlane PW. A comparison of commonl>' used QT correction fonnulae: The effect of
hean rate on 1heQTc of nonnal ECGs.J Electrocardiol.2004; 37Suppl, 81-90.
10. Luong MW, Ignaszewski M, Taylor CM. Stress testing. BCMJ, 2016;58(2):70•76.
11. Murphy ML. Thenabadu PN, de Soyza N, et al. Reevaluation of electrocardiographic criteria for left, right and combined
cardiac ventricular hypertrophy. Am J Cardiel. 1984;53(8):1140•47.
12. O'Gara PT, Kushner FG, Ascheim DD, et al. ACCF/AHAguideline for the management of ST-elevation myocardial infarction:
A repon of the American College of Cardiology Foundation I American Hean Association Task Force on Practice Guidelines.
Circulation. 2013;127(4):e362-425.
13. Prutk..in, JM. ECG tutorial: Miscellaneous diagnoses. UpToDate, www.uptodate.com/contents/ecg·tutorial·miscellaneous·
di:.lgnoses. Accessed 11 Sep,embcr 2021.
14. Rautaharju PM, Surawicz H, Genes LS, er al. AHNACCF/HRS recommendations for the standardization and interpretation of
the electrocardiogram: part IV: the ST segment, T and U waves: a scientific statement from the American Heart Association
Electrocardiography and Arrhythmias Committee, Council on Clinical Cardiology; the American College of Cardiology
Foundation; and the Heart Rhythm Society. Circulation. 2009; 119:e241•250.
15. Sgarbossa EB, Pinski SL. Barbagelata A, et al. Electrocardiographic diagnosis of evolving acute myocardial infarction in the
presence of left bundle-branch block N Engl J Mcd.1996Feb 22;334(8)<48t-7.
16. Wagner GS, Macfarlane P, Wellens H, et al. AHNACCF/HRS recommendations for the standardization and interpretation
of the electrocardiogram: pan VI: acute ischemia/infarction: a sciemific statement from the American Heart Association
Electrocardiography and Arrhythmias Committee, Council on Clinical Cardiology; the American College of Cardiology
Foundation; and the Hco.rt Rhythm Society. Circulation. 2009 Mar 17;119(10):e262•70.
17. Yaneza LO, Dolor Torres MC, Chua PU, et al. Philippine Heart Association Stable lschemic Hean Disease (SIHD) Task Force
from the Philippine Heart Association CAD Guidelines Writing Committee: Clinical Practice Guidelines for Diagnosis and
Management of Patients with Coronary Artery Disease. Philippine Heart Association 2014.
18. Zipes DP, Libby P, Bonow R, Mann DL, Tomaselli CF. Braunwald's Heart Disease: A Textbook of Cardiovascular Medicine. mh
Edition. Else\•ier/Saunders, 2019.

58
ARTERIAL
BLOOD
GAS
QJ OVERVIEW OF ARTERIAL BLOOD GAS
1. Indications and Contraindications
2. Procedure and Technique

0 ARTERIAL BLOOD GAS INTERPRETATION


1. Basic Steps in ABG Interpretation
2. Sample Cases

0 EVALUATING OXYGENATION PARAMETERS


1. Indices of Lung Function & Blood Oxygenation
2. Sample Cases

0 CORRELATION WITH VENOUS BLOOD GAS


SECTION ONE
OVERVIEW OF ARTERIAL BLOOD GAS (ABG)
INDICATIONS & CONTRAINDICATIONS
, Diagnostic exam that determines arterial oxygen (PaO2), carbon dioxide (pCO2), acidity
(pH), oxygen saturation (SaO2) and bicarbonate (HCO3)
• Important in the management of critical illness, respiratory, metabolic, or renal disease
I
::
•. '.

I. INDICATIONS
Management of acid-base disorders
Determination of oxygen and carbon dioxide partial pressures
Assessment and monitoring of response to therapeutic interventions (e.g., adjustment
of mechanical ventilator settings)

II. CONTRAINDICATIONS (if present, may consider alternative site)


Abnormal Allen's test
Local infection at the puncture site
Severe vascular diseases
Raynaud's syndrome

Modified Allen's Test


1. Instruct the patient to make a fist. This will empty the blood from the hand and fingers.
2. Press down on the radial and ulnar arteries.
3. Instruct the patient to open the hand.
4. Release the pressure on the ulnar artery while still keeping the radial artery compressed.
a. If the hand does not fill with blood, it indicates that there is occlusion of the ulnar
artery distal to the wrist (abnormal test result)
b.lf there is prompt return of color to the hand, it indicates a normal test result.
5. Repeat the test, except that the pressure on the radial artery is released while the ulnar
artery remains compressed.

III. COMPLICATIONS
Post-procedural pain and paresthesia
Hematoma
Minor bleeding
Infection
Vessel laceration
Pseudoaneurysm formation

PROCEDURE AND TECHNIQUE


MATERIALS METHODS
• Clean gloves 1. Prepare materials & explain nature of procedure to the patient
• Antiseptic (e.g., 2. Hand hygiene then wear clean gloves
povidone-iodine, 3. Locate a palpable artery, with the usual site being the radial artery
chlorhexidine) 4. Ensure adequate collateral circulation by performing the Allen's test
• Pre-heparinized 5. Optional: administer 0.5 to I mL anesthesia using 1%or 2%
lidocaine (just enough to make a dermal wheal)
3 cc syringe with
6. Using one or two fingers, palpate for the artery while holding the
a gauge 22 to 25
needle in the other hand
needle
7. Fingers should be proximal to the desired puncture site
• Sterile gauze 8. The artery should be punctured at a 30-45°angle
• Adhesive bandage 9. Because of the arterial pressure, the syringe will fill on its own
• Lidocaine 10.After obtaining 2-3 mL of blood, remove the needle while
• Container with ice simultaneously applying pressure at the puncture site
11.Maintain digital pressure until hemostasis is achieved (-5 mins)
12.Remove any air in the syringe by holding it upright (needle up) and
gently tapping the syringe to let the air loose
13.Place the specimen in the container with ice and send for analysis
61
BASIC STEPS IN ABG INTERPRETATION
I.Determine the primary acid-base disorder
2. Predict if compensation is appropriate
3. Check for secondary acid-base disorders (or mixed acid-base disorders)
I::,.when needed
4. Compute for anion gap and 1::,./
5. Check oxygenation status (discussedin Section 3)

STEP 1: DETERMINE THE PRIMARY ACID-BASE DISORDER

Arterial pH 7.4 7.35- 7.45 Acidosis Alkalosis


pC02 40 mmHg 35-45 mmHg Alkalosis Acidosis

HC03 24 meq/L 22- 26 meq/L Acidosis Alkalosis

Anion gap 10 8-12

1. Look at the pH
• If pH <7.35:it is "acidic"
• If pH >7.45:it is "alkalotic"
• If value falls within normal range (7.35-7.45),acidosis (<7.4)or alkalosis (>7.4)may still
be present but compensated; so the next step is to look at pC02, HC03, & anion gap

2. Look at the Respiratory (pC02) and Metabolic (HC03) Components


• For pC02: if value is >45 mmHg, it is "acidic"; if value is <35mmHg, it is "alkalotic"
• For HC03: if value is <22 meq/L, it is "acidic"; if value is >26 meq/L, it is "alkalotic"

B. Determine Primary Disorder by following either Step:


1. Compare changes ofHC03 and PC02 from normal values
• One way to assess whether primary problem is respiratory or metabolic in origin is
to compare changes of HC03 and PC02 from baseline
• If the change in HC03 (Li.HC03)from baseline is larger, then the problem is
primarily metabolic (and vice versa)
CHECK pH
I CHECK 1\HCO, & 1\pC0 2 PRIMARY DISORDER

Li.HC03 > Li.pC02 Metabolic acidosis


pH <7.4
Li.pC02 > Li.HC03 Respiratory acidosis

Li.HC03 > Li.pC02 Metabolic alkalosis


pH>7.4
Li.pC02 > Li.HC03 Respiratory alkalosis
Sources:SkoreckiK.et al. BrennerandRector'sTheKidney,10thEdition.Elsevier.2016
RoseBO.Clinicalphysiology
of acid-baseandelectrolyte
disorders,5thed, McGraw-Hill,
2001

62
2. Match the pC02 or HC03 with the pH
• If pH is a·:mormal, determine whether the respiratory (pC02) or metabolic (HC03)
value is consistent with the pH value
• Example.;:
• If pH is low (acidotic) + pC02 is high (acidotic) then disorder is "respiratory" acidosis
• If pH is high (alkalotic) + HC03 is high (alkalotic) then disorder is "metabolic" alkalosis

pH PRIMARY DISORDER .
Low pH <7.4 Low (Acidosis) Low or normal Metabolicacidosis
(Acidosis) High or normal High (Acidosis) Respiratoryacidosis

HighpH>7.4 High (Alkalosis) High or normal Metabolicalkalosis


(Alkalosi!:) Low or normal Low (Alkalosis) Respiratoryalkalosis

STEP 2: PREDICT APPROPRIATENESS OF COMPENSATION


Assess for appropriateness of compensation using the following formulas in the table
Tablebelowdemonstratesthe expectedcompensatoryresponsefor simpleacid-basedisorders

y
R
I PHYSIOLOGIC
COMPENSATION*
I EXPECTED
COMPENSATORY
I FORMULA
VALUES
Forevery 1 meq/LFALLin
Metabolic Acidosis•• pCO, should HC03, l t.pCO,'"
(Low pH+ Low HCO,) decrease pC02 shouldDECREASE 1.25X t.[HC03]
by 1.25mmHg
Forevery 1 meq/LRISEin
Metabolic Alkalosis pC02 should HC03, i t.pCO,'" .
(High pH+ High HCO 3) increase pC02 shouldINCREASEby 0.75x t.[HC0 3]
0.75mmHg
Acuteacidosis:for every
1 mmHgRISEin pCO,, i t.[HC0 3]'"
HC03 shouldINCREASEby 0.1 x t.pC0 2
Respiratory Acidosis HC03 should 0.1 meq/L
(Low pH+ Hig;h pCO,) increase Chronicacidosis:for every
1 mmHgRISEin pC02 , i t.[HC0 3] '"
HC03shouldINCREASEby 0.4 x t.pCO,
0.4 meq/L
Acutealkalosis:for every
1 mmHgFALLin pC02, l t.[H00 3] ==
HC03shouldDECREASE 0.2 x t.pCO,
Respiratory 1\lkalosis HCO3 should by 0.2 meq/L
(High pH+ Low pCO ,) decrease Chronicalkalosis:for every
1 mmHgFAl:.L in pC02 , t t.[HC0 3]"'
HC03 shouldDECREASE 0.4 x t.pC0 2
by 0.4 meq/L
'In general,compensatory
responses oftenreturnthe pH toward,but not to, the normalvalue(EXCEPT
chronicrespiratory
alkalosis
- it oftenreturnsthepHto a normalvaluewhenprolonged).
"Example:A patientwithmetabolic acidosis
andHC03of 12meq/L(froma normalHC03of 24meq/L)would
beexpected to havea pC02 of -25 mmHg(froma normalpC02 of 40 mmHg).
Valuesof pC02 thatarehigher
or lowerthan-2E,mmHgindicatea mixeddisorder(seeStep3).
Source:Jameson
JL,etal.Harrison's
Principles
ofInternal
Medicine.
20thEdition.
NewYork:
McGraw
HillEducation,
2018

63
STEP 3: CHECK FOR SECONDARY ACID-BASE (OR MIXED ACID-BASE) DISORDERS
Mixed acid-base disorders are defined as independently coexisting disorders (not
merely compensatory responses)
Example: A patient with metabolic acidosis from diabetic ketoacidosis may develop an
independent respiratory disturbance (acidosis or alkalosis) from pneumonia
PRIMARY
DISORDER
I COMPENSATION I SECONDARY ACID-BASE
DISORDER
Actualreductionof pC02 frombaselineis GREATER Secondary RESPIRATORY
Metabolic than that of predicted/calculated
compensation ALKALOSIS is present
Acidosis Actualreductionof pCO,frombaselineis LESSthan Secondary RESPIRATORY
that of predicted/calculated
compensation ACIDOSISis present
Actualincreaseof pC02 frombaselineis GREATER Secondary RESPIRATORY
Metabolic than that of predicted/calculated
compensation ACIDOSISis present
Alkalosis Actualincreaseof pC02 frombaselineis LESSthan Secondary RESPIRATORY
that of predicted/calculatedcompensation ALKALOSIS is present
Actualincreaseof HCO,frombaselineis GREATER Secondary METABOLIC
than that of predicted/calculated
compensation ALKALOSIS is present
Respiratory
Acidosis Actualincreaseof HC03 frombaselineis LESSthan Secondary METABOLIC
that of predicted/calculated
compensation ACIDOSISis present
Actualdecrease of HC03 frombaselineis GREATER Secondary METABOLIC
Respiratory than that of predicted/calculated
compensation ACIDOSISis present
Alkalosis Actualdecrease of HC03 frombaselineis LESS Secondary METABOLIC
than that of predicted/calculatedcompensation ALKALOSIS is present

STEP 4: COMPUTE FOR ANION GAP AND I'!./ I'!.


A. Formula for Anion Ga
Normal anion gap is 8-12
Anion gap= Na - (HC03 + Cl)
High anion gap is >12
B. Usual Causes of Metabolic Acidosis
HIGH-ANION GAP METABOLIC
ACIDOSIS (HAGMA)
I NORMAL-ANION GAP METABOLIC
ACIDOSIS (NAGMA)
• M: Methanol • H: Hyperalimentation
• U: Uremia • A: Acetazolamide
• D: Diabetic ketoacidosis • R: Renal tubular acidosis
• P: Paraldehyde • D: Diarrhea
• I: Isoniazid, iron • U: Uretero-pelvic shunt
• L: Lactic acidosis
• P: Post-hypocapnia
• E: Ethylene glycol, ethanol
• S: Salicylates
C. Check for Ml'!.
ForHigh-AnionGa~ MetabolicAcidosis(HAGMA)
=I'!.Anion gap
l'!./1'!. If =I, there is pure HAGMA
If <I, there is HAGMA+ NAGMA
l'!.HC03 If >I, there is HAGMA+ metabolic alkalosis
,,
ForNormal-AnionGap MetabolicAcidosis(NAGMAJ
!'!.II'!.=I'!.Chloride If =1, there is pure NAGMA
If <I, there is NAGMA+ HAGMA
l'!.HC03 If >I, there is NAGMA+ metabolic alkalosis

D. Computing for Bicarbonate Deficit

HC03 deficit= (Desired HC03-Actual HC03) x weight (in kg) x 0,4


64
SAMPLE CASES FOR ARTERIAL BLOOD GAS INTERPRETATION
Case I

20/M with diarrhea of >5x/day. He has poor skin turgor and cold and clammy skin.
ABG Results: pH= 7.32; pCO2 = 28 mmHg; HCO3 = 14 meq/L

• Check the pH, HCO3, and pCO2.


0 Since pH is <7-4,it is ACIDOTIC.
0 Since PCO2 is <40 mmHg, it is ALKALOTIC. '
0 Since HCO3 is <24 meq/L, it is ACIDOTIC.
Step 1:Primary
0 In this case, pH (acidotic) and HCO3 (acidotic) match, suggesting that
Disorder
the primary disorder is "metabolic acidosis."
• Also, the l'IHCO3 > l'lpCO2. Therefore, the acidosis is METABOLIC in origin.
0 l'IHCO3 = (24 - 14)/24= 0.42 or 42% (24 is the normal HCO3 level)
0 l'lpCO2 = (40 - 28)/40 = 0.30 or 30% (40 is the normal pCO2 level)

• In metabolic acidosis, for every I meq/L FALL in HCO3, pCO2 should


DECREASE by 1.25mmHg to compensate.
• Appropriate compensation = (24 - 14)x 1.25= IOx 1.25= 12.5reduction in pCO2
• Since there is a IO meq/L FALL in HCO3 from normal (24 -14), we expect
Step 2: Predict
that pCO2 should DECREASE by 12.5mmHg to compensate.
Compensation
• Our patient has a·pCO2 of 28 mmHg, which means that he had a 12
I mmHg decrease in pCO2 (from a normal pCO2 of 40 mmHg). This means
that there is physiologic compensation since the actual decrease in pCO2
is the same as the expected compensation.

Final • Our patient has COM PENSA TED METABOLIC ACIDOSIS (consistent
Interpretation with the scenario since diarrhea primarily causes metabolic acidosis)

Case2

45/M undergoing elective surgery under general anesthesia (intubated on ventilator support)
ABG Results: pH= 7.48; pCO2 = 32 mmHg; HCO3 = 22 meq/L
Serum chemiury: Na= 136 meq/L; Cl= 101 meq/L; K = 3.6 meq/L

• Since pH is >7-4,the primary disorder is ALKALOSIS.


• In this case, pH (alkalotic) and pCO2 (alkalotic) match, suggesting that
Step 1: the primary disorder is "respiratory alkalosis."
Primary , Also, the l'lpCO2 > l'IHCO3. Therefore, the alkalosis is RESPIRATORY
Disorder in origin
l'lpCO2 = (40 - 32)/40 = 0.20 or 20% (40 is the normal pCO2 level)
0

l'IHCO3 = (24 - 22)/24 = 0.08 or 8% (24 is the normal HCO3 level)


0

• .In acute respiratory alkalosis, for every I mmHg FALL in pCO2, HCO3
should DECREASE by 0.2 meq/L to compensate.
Step 2: Predict
• Since there is an 8 mmHg FALL in PCO2 from normal (40 - 32 mmHg),
<;;ompensatior,
we expeGt that HCO3 will DECREASE by -2 meq/L to compensate.
• Appropriate compensation: 8 mmHg x 0.2 meq/I:; = 1.6meq/L
• The HCO3 of our patient in the ABG is 22 meq/L. We see that the
Step 3: HCO3 actually decreased by 2 meq/L from a normal level of 24 meq/L,
Secondary therefore there is appropriate compensation in this case.
Disorder • There is no secondary disorder. The decrease in HCO3 is a physiologic
compensation.
.
Final
ii • G>urpatient has COMPENSATED RESPIRATORY ALKALOSIS
(probably due to the mechanical veruilation dyssynchrony causing.
Interpretatiort
I .tachypneii.J - '
65
Case 3
72/F with COPD was brought in for few days lethargy, dyspnea, cough, vomiting, & fever.
ABG Results: pH= 7.25; pCO2 = 68 mmHg; HCO3 = 34 meq/L; PaO2 = 48 mmHg
• Since pH is <7-4,the primary disorder is ACIDOSIS.
• In this case, pH (acidotic) and pCO2 (acidotic) match, suggesting that the
Step 1:Primary primary disorder is "respiratory acidosis."
Disorder • Also, the 6pCO2 > 6HCO3. Therefore, acidosis is RESPIRATORY in origin
0 6pCO2 = (68 - 40)/40 = 0.70 or 70% (40 is the normal pCO2 level)
0 6HCO3 = (34 - 24)/24 = 0.42 or 42% (24 is the normal HCO3 level)
• In acute respiratory acidosis, for every I mmHg RISE in pCO2, HCO3
should INCREASE by 0.1 meq/L. This is considered "acute" since
Step 2: Predict patient's COPD was previously well-controlled.
Compensation • Since there is a 28 mmHg increase in pCO2 (68 - 40 mmHg), HCO3
should increase by -2.8 meq/L as compensation.
• Appropriate compensation= (68 - 40) x 0.1 = 2.8 meq/L increase in HCO3
• The HCO3 ofour patient is 34 meq/L (an actual increase of 10 meq/L from
Step 3: a normal HCO3 of24 meq/L). The actual increase in HCO3 (IO meq/L) is
Secondary GREATER than the calculated appropriate compensation of 2.8 meq/L.
Disorder • Therefore, patient has CONCOMITANT (SECONDARY) METABOLIC
ALKALOSIS.
• Our patient has PRIMARY RESPIRATORY ACIDOSIS WITH
CONCOMITANT METABOLIC ALKALOSIS.
Final • This suggests that patient has an acute exacerbation of her COPD,
Interpretation leading to CO2 retention. Because of the sudden change, the HCO3 has
not had time to adjust yet, & the pH decreases due to respiratory acidosis.
The vomiting could account for the secondary metabolic alkalosis.
Case4
A 55/M diagnosed with COPD and heart failure is on high-dose furosemide (diuretic).
ABG Results: pH= 7.42; pCO2 = 67 mm Hg; HCO3 = 42 meq/L
Blood Chemistry: Na= 140 meq/L; Cl= 88 meq/L; K = 3.5 meq/L
• Since pH is >7.4, the primary disorder is ALKALOSIS.
• In this case, pH (alkalotic) and HCO3 (alkalotic) match, suggesting that
Step 1:Primary the primary disorder is "metabolic alkalosis."
Disorder • Also, the 6HCO3 > 6pCO2. Therefore, the alkalosis is METABOLIC in origin.
0 6HCO3 = (42 - 24)/24 = 0.75 or 75% (24 is the normal HCO3 level)
0 6PCO2 = (67 - 40)/40 = o.68 or 68% (40 is the normal pCO2 level)
• In metabolic alkalosis, for every I meq/L RISE in HCO3, PCO2 should
INCREASE by 0.75 mm Hg.
Step 2: Predict • Since there is an 18 meq/L increase in HCO3 (from normal of 24 meq/L),
Compensation we expect that pCO2 will increase by -13.5 mm Hg (or expected pCO2
should be -53.5 mmHg).
• Appropriate compensation= (42 - 24) x 0.75 = 13.5mmHg increase in PCO2
• The pCO2 of our patient in the ABG is 67 mm Hg (actual increase of 27
mmHg from a normal pCO2 of 40 mmHg). This actual increase in pCO2
Step 3:
is GREATER than the calculated appropriate compensation of -13.5
Secondary
Disorder mm Hg, which suggests that there is a CO NCO MIT ANT RESP IRATORY
ACIDOSIS.
• This is because pCO2 is higher than predicted and pH is already normal.

1-... • Our patient has PRIMARY METABOLIC ALKALOSIS with


Final CON CO MIT ANT RESPIRATORY ACIDOSIS.
Interpretation • Metabolic alkalosis is probably from the diuretic use (contraction alkalosis),
& the secondary respiratory acidosis is probably chronic from COPD.

66
Case5
50/M with vomiting was diagnosed with acute pancreatitis.
ABG Results: pH= 7-28; pCOz = 31 mmHg; HC03 = 16 meq/L
Blood Chemistry: Na= 132meq/L; Cl= 91 meq/L

• Since pH is <7.4,the primary disorder is ACIDOSIS.


• In this case, AHCO3 > ApCO2. Therefore, acidosis is METABOLIC in origin.
Step 1: Primary
0 AHCO3 = (24 - 16)/24= 0.33 or 33% (24 is the normal HCO3 level)
Disorder
0 ApCO2 = (40 - 31)/40= 0.23 or 23% (40 is the normal pCO2 level).
.
.
• Therefore, the primary disorder is metabolic acidosis.
• In metabolic acidosis, for every I meq/L FALL in HCO3, pCO2 should
DECREASE by 1.25mmHg to compensate.
Step 2: Predict
• Since there is an 8 meq/LFALLin HCO3 (from normal), we expect that pCO2
Compensation
will DECREASE by -10 mmHg (or expected pCO2 should be -30 mmHg).
• Appropriate compensation= (24- 16)x 1.25= IO mmHg REDUCTION in pCO2

Step 3: • The pCO2 of our patient in the ABG is 31mm Hg (actual reduction of 9
Secondary mmHg). The actual decrease in pCO2 is more or less equal to the calculated
Disorder appropriate compensation of IO.
• Anion gap= Na - (HCO3 +Cl)= 132- (16+ 91)= 25
• Since AG is >12,our patient has high anion-gap metabolic acidosis (HAGMA).
Step 4: Anion
• For HAGMA, compute for (AAnion Gap)/AHCO3
Gap and MA
0 A/A= (25-12)/(24- 16)= 13/8= 1.6
0 Since the A/A is >I, there is also metabolic alkalosis.
• Our patient has PRIMARY HIGH-ANION GAP METABOLIC ACIDOSIS
with CONCURRENT METABOLIC ALKALOSIS.
Final
• Due to pancreatitis, patient developed high-anion gap metabolic acidosis.
Interpretation
The concurrent metabolic alkalosis is likely due to vomiting, which leads to
hydrogen ion loss in the GI tract.

Case6
70/M with liver disease and on high-dose diuretics because of ascites
ABG Results: pH= 7.55; pCOz = 38 mm Hg; HC03 = 33 meq/L
Blood Chemistry: Na= 140 meq/L; K = 4.0 meq/L; Cl= 91 meq/L
• Since pH is >7.4, the primary disorder is ALKALOSIS.
• In this case AHCO3 > ApCO2.Therefore, the acidosis is METABOLIC in origin.
Step i: Primary
0 AHCO3 = (33 - 24)/24 = 0.38 or 38% (24 is the normal HCO3 level)
Disorder
0 ApCO2 = (40 - 38)/40 = 0.05 or 5% (40 is the normal pCO2 level)

Therefore, the primary disorder is metabolic alkalosis.


• In metabolic alkalosis, for every I meq/L RISE in HCO3, pCO2 should
INCREASE by 0.75 mmHg.
Step 2: Predict
• Since there is a 9 meq/L increase in HCO3 (from normal of 24 meq/L), we expect
Compensation
that pCO2 will increase by -6.75 mmHg (or pCO2 should be -46.75 mmHg)
• Appropriate compensation= (33- 24) x 0.75 = 6.75mmHg increase in pCO2
• The pCO2 of our patient in the ABG is 38 mm Hg (it decreased by 2 mm Hg
from a normal of 40 mm Hg). This actual change in pCO2 is much less than
Step 3:
the calculated appropriate increase in pCO2 by 6.75mmHg, which suggests
Secondary
that there is a CONCOMITANT RES Pl RATORY ALKALOSIS.
Disorder
• This is because the pCO2 did not increase as much as predicted, leading to
a pH that is higher than expected.
• Our patient has PRIMARY METABOLIC ALKALOSIS AND
CONCOMITANT RESPIRATORY ALKALOSIS.
Final
• The diuretic use may have accounted for the metabolic alkalosis while the
Interpretation
respiratory alkalosis may have been brought about by tachypnea from a
I distended abdomen (improper diaphragmatic excursion).
67
Case7
26/M with T1DM, not taking insulin recently, presented with 3-day history of vomiting
and increased sleeping time.
ABG Results: pH= 7.IO; pCOz = 30 mmHg; HCO3 = 6 meq/L
Blood chemistry: Corrected Na= 155meq/L; Cl= 110meq/L; K = 2.8 meq/L
• Since pH is <7,4, the primary disorder is ACIDOSIS.
• In this case, pH (acidotic) and HCO3 (acidotic) match, suggesting that the
Step 1:
primary disorder is "metabolic acidosis."
Primary
• Also, the t,.HCO3 > t,.pCOz. Therefore, the acidosis is METABOLIC in origin.
Disorder
0 t,.HCO3 = (24 - 6)/24 = 0.75 or 75% (24 is the normal HCO3 level)

0 t,.pCOz = (40 - 30)/40 = 0.25 or 25% (40 is the normal pCOz level)

• In metabolic acidosis, for every I meq/L FALL in HCO3, pCO2 should


DECREASE by 1.25mmHg to compensate.
Step 2: Predict • Since there is an 18 meq/L FALL in HCO3 (from a normal value of 24), we
Compensation expect that pCO2 will DECREASE by -22.5 mmHg (from a normal value
of4ommHg).
• Appropriate compensation= 18x 1.25= 22.5 mmHg REDUCTION in pCO2
• The pCOz of our patient in the ABG is 30 mm Hg (actual reduction of
Step 3: IO mmHg). The actual decrease in pCOz (IOmmHg) is LESS than the
Secondary calculated appropriate compensation of 22.5 mmHg.
Disorder , Therefore, our patient has CONCOMITANT (SECONDARY)
RESPIRATORY ACIDOSIS.

• Anion gap = 155- (no + 6) = 39


• Since AG is > 12,our patient has HAGMA.
Step 4: Anion
• For HAGMA, compute for (t,.Anion Gap)/t,.HCO3
Gapand/1,./ /I,
0 t,./t,. = (39 - 12)/(24- 6) = 27/18= 1.5

0 Since the t,./t,. >I, there is also metabolic alkalosis.

, Our patient has PRIMARY HIGH ANION GAP METABOLIC ACIDOSIS


with CONCOMITANT RESPIRATORY ACIDOSIS and METABOLIC
Final ALKALOSIS.
Interpretation • Our patient is likely in DKA (causing the metabolic acidosis) and his
comatose state is causing CO2 retention, leading to respiratory acidosis.
The metabolic alkalosis is probably due to vomiting (alkaline tide).

68
SECTION THREE
EVALUATING OXYGENATION PARAMEl;ERS
INDICES OF LUNG FUNCTION AND BLOOD OXYGENATION

I. MEASURES OF OXYGENATION
The partial pressure of oxygen in arterial blood or PaO2 is a measure of dissolved
'
oxygen in arterial blood plasma
Hypoxemia is defined as a low PaO2, which can be obtained by blood gas analysis
A reduced PaO2 is a non-specific finding (i.e.,it only signifies a disturbance of gas exchange)
PaO2 is different from Sa 02, which describes the amount of oxygen bound to hemoglobin
SaO2 can also be measured via blood gas analysis
When SaO2 is measured using pulse oximetry, the more appropriate term then becomes SpO2

AS 't fH . . L tR A' (F'O o/c)


SEVERlTYOF
HYPOXEMlA I PaO2 VALUE
I ESTIMATED SpO2

Mild Hypoxemia 60-79mmHg 90-94%


Moderate Hypoxemia 40-59 mmHg 75-89%
Severe Hypoxemia <40 mmHg <75%
Source:ShelledyDC;et al. Jones& BartlettPublishers;2014

B. Determination ofFiO2 in Patients with Nasal Cannula or a Simple Face Mask


° Fraction of inspired oxygen (FiO2) is the concentration of oxygen (expressed in
percent) that a person inhales
0Ambient or room air is made up of21% oxygen, hence FiO2 of room air= 21%
0When you give supplemental 02, you are raising the patient's FiO2 to a level over 21%
(ranges from 22-100%)
° For every I LPM, there is an estimated increase of 4% in FiO2

Formula for Fi02 Determination:


FiO2 = 4% x liters per minute (LPM) + 20%

Example: A patient is on nasal cannula at 5 LPM. Compute for the FiO2:


FiO2 = (4x 5) + 20
=40%

OXYGEN DELIVERY
SYSTEM
I OXYGEN(inFLOW
1pm)
RATES I ESTIMATED FiO2

1 24
2 28
3 32
Nasal Cannula (NC) 4 36
5 40
6 44
5-6 40
Simple Face Mask• 6-7 50
7-8 60
'Facemaskscannotdeliver100%oxygen,unlessthereis a tightseal.Non-rebreather
facemasks
can deliveran FiO2up to -80%.An FiO2of 100%canonly be deliveredwith a ventilatoror a
tight-fitting
facemask.

69
C. Related Formulas
VARIABLE I FORMULA I REMARKS
• PaO2 <4-5 times the
• Measured PaO2 on ABG should be equal FiO2 suggests poor
Expected Pa02 to 5 x FiO2 lung function or
based on • A normal PaO2 on room air (21% FiO2) hypoventilation
current FiO2 is !05 mmHg. A patient being given 50% • This is as a rough
FiO2 should have a PaO2 of -250 mmHg. gauge for adequate
oxygenation
If age <60 years old:
Desired Pa02 = 104 - (0.43 x age) • Used to adjust
Desired Pa02 ventilator FiO2
If age 60 years and above: settings
Desired Pa02 = 80 - (age - 60)
• Increasing the FiO2
can increase PaO2
& SpO2 to desired
Desired Fi02 levels
Current FiO2 x Desired PaO2
to reach Desired FiO2 = • Current PaO2:
desired Pa02 Current PaO2 obtained from ABG
• Desired PaO2:
obtained using above
formula

II. ALVEOLAR-ARTERIAL OXYGEN GRADIENT (PAO2-PaO2 GRADIENT)


Venous admixture (most common cause ofhypoxemia) can be evaluated using the A-a gradient
An increased A-a gradient suggests defect in diffusion, V/Q mismatch, or right-to-left shunting
Hypoxemia in the setting of a normal A-a gradient suggests:
0 Hypoventilation (e.g., CNS disorder, neuromuscular disease, COPD)
0 Patient being at high altitudes (resulting in a lower atmospheric pressure than the
usual 760 mmHg at sea level, and consequently, a lower PiO2 [partial pressure of
inspired oxygen] than the usual 160 mmHg)
VARIABLE I FORMULA I REMARKS

A-a gradient= PAO2 - PaO2 • PaO2:arterial oxygen


Formula for partial pressure; obtained
A-a Gradient Where: fromABG
PAO2 = FiO2 (P,.m- P.,,0 ) - (PaCO2/o.8) • PAO2:alveolar oxygen
partial pressure
At room air (FiO2 21%)& • PaCO2:arterial carbon
Simplified sea level (Pum 760 mmHg): dioxide partial pressure;
Formula for
also obtained from ABG
A-a Gradient
A-a gradient= [150- 1.25(PaC02 )]- Pa02 • FiO2:fraction of inspired 0,

• Normal (or expected) A-a


gradient for a young person
is 5-10mm Hg, & it normally
increases with age
Normal
Age • For every decade a person
Value for
Normal A-a gradient= -- +4 has lived, the A-a gradient
A-a Gradient 4
(dependson age) is expected to increase by
-1mmHg
• Ex. A 60-year-old person
should have an A-a
gradient <19mmHg
70
III. Pa02-Fi02 RATIO (PFR)
Ratio of arterial oxygen partial pressure (Pa Oz) to fraction of inspired oxygen (FiO2)
It is also called the Carrico index or the PF ratio
Serves as a clinical indicator of hypoxemia
• PaO2: arterial oxygen partial pressure; obtained from ABG
• Fi 02: fraction of inspired 02 (in decimal) at the time the ABG was drawn
• Normal PFR value (at sea level)= 100 mm Hg/ 0.21= 4i6 mm Hg (or more)
PaO2
PFR= FiO2 • Severity of Acute Respiratory Distress Syndrome or ARDS by PaO2/FiO2: .
• Mild: 200 to 300 mmHg
• Moderate: 100 to 199mmHg
• Severe: < 100 mmHg

SAMPLE CASES FOR EVALUATING OXYGENATION PARAMETERS


Case I
65/M, 20-pack-year smoker, came in for cough & fever. He was dyspneic/rachypneic on admission.
ABG Results: pH= 7.48; pCO2 = 30 mmHg; HCO3 = 19meq/L; PaO2 = 49 mm Hg; SaO2 = 78%
ABG taken at room air.
• Since pH is >7.4,the primary disorder is ALKALOSIS.
• In this case liHCO3 < lipCO2. Therefore, alkalosis is RESPIRATORYin origin
• lipCO2 = (40 - 30)/40 = 0.25 (40 is the normal pCO2 level)
• liHCO3 = (24 - 19)/24= 0.21 (24 is the normal HCO3 level)
• Since there is a IO meq/L FALL in pCO2 (from normal), we expect that
Interpretation
HCO3 will DECREASE by 2 mmHg
ofABG
• Appropriate compensation= 10 x 0.2 = 2 mmHg REDUCTION in HCO3
• The HCO3 of our patient in the ABG is 19 meq/L (actual decrease of
5 meq/L which is more than the calculated compensation)
• Interpretation: Primary Respiratory Alkalosis with Secondary Metabolic
Acidosis
Estimated
• The estimated FiO2 at room air is 21%
Fi02given

A. Evaluate Oxygenation using the Described Indices


• Compute for expected PaO2 based on the estimated Fi 02.
• Expected PaO2 = 5 x FiO2 = 5 x 21= 105mmHg
UsingPaO2 • A PaO2 less than 4-5 times the FiO2 suggests poor lung function.
• The patient's PaO2 is 49 mmHg
• Interpretation: suggestive of moderate hypoxemia
Using A-a • A-a Gradient= (150- 1.25(30)] - 49 = 63.5 mmHg
Gradient • An elevated A-a gradient suggests defects in diffusion or V/Q mismatch
UsingPaO2- • PFR = PaO2 / FiO2 = 49 / 0.21 = 233
FiO2 Ratio • Normal value is >500 mmHg. Therefore, the patient's low PFR indicates
(PFR) lung injury or ARDS (mild severity)

B. Adjust the FiO2 Being Given to Correct Hypoxemia


Compute for • Desired PaO2 (for age >60 years)= 80 - (age - 60) = 80 - (65- 60) = 75 mm Hg
Desired PaO2 • Therefore, adjust delivered FiO2 to target a desired PaO2 of 75 mmHg (seenext)
• Desired FiO2 = (current FiO2 x desired PaO2) / current PaO2
Compute for = (0.21x 75) / 49 = 0.32 or 32%
the Desired • To increase the PaO2 to the desired levels, you may hook the patient to
FiO2 supplemental 02 via nasal cannula at 3 LPM (estimated FiO2 of 32%),
and then reassess clinically and with a repeat ABG after I hour.

71
Case2
A 40/M (70 kg) suffered 2nd degree burns affecting 80% TBSA with involvement of the
face due to an accidental explosion. He was referred for bronchoscopy to investigate the
possibility of inhalational injury. He was tachypneic at 28 breaths per minute with SpO2
of 94% at 4 LPM 02 support. Video-assisted bronchoscopy did not reveal any significant
findings. Initial chest radiograph revealed no significant infiltrates. On the third hospital
day, he complained of increasing difficulty of breathing. Chest physical examination
revealed crackles in bilateral mid to basal lung fields. Oxygen saturation was persistently at
74% despite 02 at IO LPM via face mask. Patient was eventually intubated due to hypoxemic
respiratory failure. Empiric broad-spectrum antibiotics were started.

ABG (pre-intubation): pH 7.20, pCO2 57 mmHg, HCO3 28 meq/L, PaO2 65 mmHg, SaO2 70%
(Respiratory acidosis with metabolic alkalosis; mild hypoxemia)

MV settings: AC mode
TV 420 (6 mL/kg) Trigger 2
BUR16 PEEP 6
IFR6o FiO2rno%

Initially, his oxygen saturation improved to 95% with FiO2 at 50%. However, in the
succeeding hour, FiO2 was increased to 70% due to a decrease in SpO2 to 88%. The patient
was persistently hypoxemic despite increasing oxygen support. The PEEP was maintained
at6cm H2O.

Repeat ABG: pH 7.35, pCO2 38 mmHg, PaO2 60 mmHg, HCO3 22 meq/L with an FiO2
increased to So% (compensated metabolic acidosis). The patient's repeat chest radiograph
revealed diffuse bilateral infiltrates.

A. What is the computed A-a gradient using data from the repeat ABG?
• A-a gradient= PAO2 - PaO2
= [FiO2 (Patm - PH2O) - (PaCO2/o.8)] - PaO2
= [o.8 (760 - 47) - 38/0.8] - 60
= [o.8 (713)- 47.5]- 60
A-a Gradient
= 522.9- 60
=462.9 mmHg
• Full formula (not simplified) for A-a gradient was used because
FiO2 on repeat ABG (80%) was no longer 21%(room air)
What is the
• Expected A-a gradient= (age/ 4) +4
expected A-a
• = (40/4) +4
gradient adjusted
•=14mmHg
forage?

• The expected A-a gradient for this patient is 14mmHg based on


his age adjustment. An increased A-a gradient suggests defects in
Interpretation diffusion, V/Q mismatch or right-left shunting.
• In this case, we are considering acute lung injury (ARDS) from
inhalational injury resulting in hypoxic respiratory failure.

B What is his PaO2-FiO2 Ratio 1

PFR =---PaO2
PaO2-FiO2 Ratio FiO2
(PFR)
=60 I o.8
=75
Interpretation • The PFR of the patient is consistent with severe ARDS (<IOO)

72
C. What is the Desired PaO2?
Desired PaO2 = ro4 - (0-43 x age)
= 104 - (0.43 X 40)
Desired PaO2
= ro4- 17.2
= 86.8 mm Hg

• Desired PaO2 computation based on the age of the patient is -87 mmHg.
Therefore, it is prudent to increase the FiO2 settings of the mechanical
Interpretation
ventilator, as well as follow the recommendations for increasing PEEP
.
in the setting of ARDS.

D. What is the Desired FiO2?

Desired FiO2 = Current FiO2 x Comeuted desired PaO2


Desired FiO2 Current PaO2
= 0.8 X 87 / 60
=I.16

• Desired FiO2 computation for this case approximates 100% (-1),


hence, the Fi02 setting should be maintained at 100%. Other
:ventilator parameters should also be adjusted based on the
Interpretation
recommendations for ARDS management.
• Once the patient's lung condition improves and oxygen
requirement goes down, we can start downtitrating the Fi 02.

73
--!l!L~T!::,~~~=Q~~
VENOUS BLOOD GAS (VBG)
VBG is similar to an ABG, but drawn/extracted from the venous circulation:
0Via venipuncture (peripheral sample)
0Via a pre-existing central line (central sample)
0Via a pulmonary arterial catheter (mixed venous sample)
Less painful and allows for convenient sampling in critically ill patients who require
serial monitoring of pH or mixed venous 02 saturation (SvO2) after intervention has
been initiated
The letter 'v' is used to designate a VBG sample rather than an ABG sample (e.g.,
PvCO2, PvO2, SvO2)

I. CONVERTING VBG VALUES TO ABG EQUIVALENTS


The advantage of a VBG being a surrogate for an ABG lies in a patient with suspected
acid-base disturbance but with stable oxygenation status
Values for pH & HCO3 in VBG samples correlate well with ABG samples
0 VBG pH values are 0.03-0.4 units lower than ABG pH values
VBG HCO3 values are 1-2mEq higher than ABG HCO3 values
0

There is marked variability between VBG & ABG values of pCO2 & pO2, especially in
shock and/or heart failure, so an ABG is still recommended for these determinations
Since pvCO2 values are generally higher than paCO2 values, a normal pvCO2 has a
high negative predictive value for hypercapnic respiratory failure (CO2 retention),
making it an acceptable "rule out" parameter for type II respiratory failure

II. MIXED VENOUS OXYGEN SATURATION (Sv02)


Percentage ofO2 bound to hemoglobin in blood returning to the right side of the heart
Reflects amount of oxygen "left over" after the perfused tissues remove what they need
Provides indirect evidence of adequate resuscitation in shock; and when simultaneously
taken with an ABG, provides data on hemodynamics
A. Extracting Sv02
0 SvO2 sample should ideally be blood extracted from a pulmonary arterial catheter
(PAC), since all venous blood from the SVC and IVC have already "mixed" by the time
it reaches the pulmonary artery (more reflective of the amount ofO2 remaining after
tissue extraction)
0 In the absence of PAC monitoring, a blood sample extracted from a central line in the
internal jugular or subclavian veins (ScvO2) can be a surrogate for the true SvO2
B. Interpreting the Sv02 (See CriticalCarechapterfor further details)
0 SvO2 (or ScvO2) is primarily affected by oxygen delivery (DO2, supply) and 02
consumption (VO2, demand)
0 Normal SVO2 = 65-70%

REFERENCES
1. Byrne AL, Bennett M, Chauerji R,Symons R, Pace NL Thomas PS. Peripheralvenous and arterialblood gas analysis in adults:
are they comparable?A sys1cmatic reviewand meta-analysis.Respirology.2014 Feb;19(2):168-175.
2.Chetana Shanmukhappa S, LokeshwaranS. Venous Oxygen Saturation.[Updated2020 Nov 1]. In:StatPearls(lntemetJ.Treasure
Island(FL):S1atPearlsPublishing;2021 Jan.Availablefrom:www.ncbi.nlm.nih.gov/books/NBK564395/
J. Hafen BB, Sharma S. Oxygen Saturation. !Updated 2021 Aug 12). In: StatPearls [Iruemet]. Treasure Island (FL):Sta1Pearls
Publishing;2021Jan.[Figure,Mixed venous oxygen saturation.Comributed by Chan Lee].Availablefrom:www.ncbi.nlm.nih.
gov/books/NBK525974ifigurelanicle-26491.image.fJ/
4.JamesonJL,KasperDL, Longo DL,FauciAS, Hauser SL,LoscalzoJ.Harrison'sPrinciplesoflntemal Medicine.20th Edition.New
York:McGr:1.w Hill Education,2018.
5. KarbingOS, KjaergaardS, Smith SW, Espersen K, Allered C, Andreassen S, et al. Variationin the PaO2/FiO2ratio with FiO2:
mathematicaland experimental description, and clinical relevance.CritCare.2007;11(6):R118.
6. Leong B. and Lee K.W. In:Ooi S. and Manning P.,eds. Guide To The Essentials In EmergencyMedicine. Singapore:McGraw•
Hill, 2014.pp.158-68.
7. Rose BO. Post T. (zooo) Clinical physiology of acid-base and electrolyte disorders, 5th ed, McGraw-Hill,2001. McGraw-Hill
ProfessionalPublishing.
8.Skoreck.iK, Chcnow G, Marsden P, Taal M, Yu A, and Brenner B. (editors). Brenner and Rector'sThe Kidney, 10th Edition.
Philadelphia,PA:Elsevier.2016.
74
BASIC
IMAGIN
CHEST RADIOGRAPH INTERPRETATION
1. Identify General Data of the Patient
2. Determine View
3. Assess Quality of Film
4. Assess Anatomy
5. Determine the Presence of Abnormalities

0 PLAIN ABDOMINAL RADIOGRAPHY


1. Indications for Plain Abdominal Radiography
2. Normal Anatomy

0 BASIC COMPUTERIZED TOMOGRAPHY (CT) IMAGING


1. Normal Chest CT Anatomy in Axial View
2. Normal Abdominal CT Anatomy in Axial View
3. Cranial CT Scan
SECTION ONE
CHEST RADIOGRAPH INTERPRE'FATION

BASIC STEPS IN READING CHEST RADIOGRAPHS


Step I: Identify general data of the patient
Step 2: Determine view (PA, AP, lateral, decubitus)
Step 3: Assess quality of film
Step 4: Assess anatomy
Step 5: Determine the presence of abnormalities
STEP 1: IDENTIFYING GENERAL DATA OF THE PATIENT
Patient's name and age
Date/time chest x-ray (CXR) was taken
• Diagnosis of patient
Indication for CXR

STEP 2: DETERMINING THE VIEW


A. Antero-Posterior (AP) versus Postero-Anterior (PA) View
ANTERO-POSTERIOR (AP) I POSTERO-ANTERIOR (PA)

• Not appreciated • Present


Other Aspects

• Arms are parallel to body • Arms at an angle with the body, with
• Heart & other structures seem hands at the waist
magnified • Heart not magnified

B. Lateral Film
0 Usually requested to further assess and clarify lesions seen on frontal x-ray views
0 Left lateral view: preferred position as it provides better anatomic detail of the heart
77
C. Lateral Decubitus Film

• Lateral decubitus film is a special


projection used to demonstrate pleural
effusions and pneumothorax
0 When investigating pleural effusions,
the side of interest should be down
0 When investigating pneumothorax,
the side of interest should be up
• Image on the left shows pooling of
pleural fluid (white interface) at the
bottom due to the effects of gravity (e.g.,
pleural effusion)

STEP 3: ASSESSING THE QUALITY OF THE FILM


ASPECT I QUALITY INDICATOR
• Good visualization is characterized by inclusion of entire thoracic cage:
0 Lung apices and first ribs cranially
Visualization ° Costophrenic angles caudally
0 Edges of ribs laterally
• Reported as: "good visualization"
• Chest x-rays are conventionally taken while patients are in the
inspiratory phase of respiratory cycle
Inspiratory • One should count >8intercostal spaces, 6-8 anterior ribs, 9-II posterior ribs
effort • Diaphragm should be intersected by the 6th rib in the midclavicular line
• Inadequate inspiration leads to crowding of structures
• Reported as: "good inspiratory effort"
-
• Pertains to the degree to which x-rays have passed through the body
• A well-penetrated/exposed chest x-ray is one where vertebrae are just
Exposure/ visible behind the heart and upper four thoracic vertebrae are visible
penetration • Newer technologies use digital correction to compensate for incorrect
penetration
• Reported as: "good exposure/penetration"
• The spinous processes should be in the midline and should be
Obliquity/ equidistant from medial ends of the clavicles
rotation • Rotation may lead to misinterpretation of structures
• Reported as: "no obliquity or significant rotation"
• Artifacts may be due to radiographic technique (seeabove), patient
factors (poor cooperation with positioning or unnecessary movement,
Presence of
obesity) or non-anatomical objects (medical devices)
artifacts
• Some may be unavoidable (allowable if it doesn't impair evaluation of
important structures)

Example of a Good Quality Radiograph (see next page)


• Inclusion oflung apices, costophrenic angles and rib edges within the plate (good visualization)
• Presence of 10 intercostal spaces, 11 posterior ribs, 8 anterior ribs with the 6th anterior rib
intersecting the right hemidiaphragm in the midclavicular line (good inspiratory effort)
• Visibility of vertebrae behind the heart and upper four thoracic vertebrae (good
exposure) & equal spacing between visible spinous processes & the medial ends of the
clavicles (no obliquity)

78
1st Posterior Rib

3rd Anterior Rib

5th Anterior RI

6th Anterior Rib

Posterior Rib

79
STEP 4: ASSESSING ANATOMY
AN IL
RIGHT LUNG* I LEFT LUNG**
• Divided into three lobes • Divided into two lobes:
0 Right upper lobe (RUL) 0 Left upper lobe (LUL)
0 Right middle lobe (RML) 0 Left lower lobe (LLL)
0 Right lower lobe (RLL) , Dividing the two is the oblique fissure
• Horizontal & oblique fissuresdivide the lobes

I RighUleft
upperlobe -o-Overlap:Rightmiddle,lowerlobe
- RighUleft
lowerlobe + Overlap:Rightupper,lowerlobe
o Rightmiddlelobe
'The rightlungis separatedintothreelobes:upper,middleandlowerlobes.Theobliquefissuredividestheupper
andmiddlelobefromthelowerlobewhilethehorizontal fissuredividestheupperlobefromthe middlelobe.
.. Theleftlung,on theotherhand,is dividedintotwolobes:the upperandlowerlobe,byan obliquefissure.

B. Normal Heart and Vessels


1. Cardiothoracic Ratio (CTR) of the Heart
• Size of heart is assessed by CTR
• Calculated by ettin° the ratio between the cardiac size and the thoracic width
• Cardiac size is measured by drawing
vertical parallel lines down the most
lateral points on each side of the heart,
and measuring between them
• Thoracic width is measured by drawing
vertical parallel lines down the inner
aspect of the widest points of the ribs
cage and measuring between them
• Normal: CTR <50% measured in
the PA view (the AP view will cause
magnification of the heart size)
Intheexampleabove,thecardiacsizeis measured
at 15.2cmwhilethethoracicwidthis measured
at
32.8cmgivinga normalCTRof 0.46
80
2. Cardiac Chambers and Great Vessels
• Left heart border corresponds to the lateral border of the left ventricle
• Right heart border corresponds to the lateral border of the right atrium
• Upper zone vessels are normally smaller than lower zone vessels

Theseare chestx-raysin the PA ginsof the heartandgreatvesselsandt


respectivepositions.The importa ing the rightborderof the heartincludethe
atrium(3) and the superiorven tructuresprominenton the left heartborder
composedof the aorta(9),the m ry (5),theindentationformedby the leftatriu
andtheleftventricle(8).

1 Superior venacava(SVC) 4 Rightventricle


(RV) 7 Leftatrium(LA)
2 Inferiorvenacava(IVC) 5 Pulmonaryartery(PA) 8 Leftventricle
(LV)
3 Rightatrium(RA) 6 Pulmonaryveins(PV) 9Aorta(Ao)
Theseare chestx-raysin the lateralview showingthe marginsof the heartand the greatvessels.
Therightventricle(4) occupiesthe mostanteriorarea.Thespacebetweenthe rightventricleandthe
chestwall is the retrosternal
spaceandis obliteratedin casesof rightventricular
enlargement.
Theleft
ventricle(8)occupiesthemostposteriorareaof theheart.Behindit andanteriorto thevertebraeis the
retrocardiacspace,whichis obliteratedin casesof leftventricularenlargement.
81
• Normal vascular markings are more prominent
in the lower lung zones, show gradual tapering
towards the periphery of the lungs, and are
usually symmetric
0 Larger caliber vessels are seen near the hilum,
with uniform tapering towards the periphery
0 Upper lung zones have smaller caliber vessels
(blood pooling in lower lung due to gravity)

4. Normal Mediastinum
• The mediastinum contains the heart, great vessels (middle mediastinum) and
potential spaces: anterior to the heart (anterior mediastinum), posterior to the heart
(posterior mediastinum) and above the heart (superior mediastinum)
• The aortic knob corresponds to the left lateral extent of the aorta as it makes its way
posteriorly over the left main bronchus and pulmonary vessels

• The bony structures


are the densest (and
most radiopaque)
structures seen on a
normal chest x-ray.
• The bones visible
on the chest x-ray
are clavicles, ribs,
scapulae, parts of the
spine and proximal
humeri.
• The bony structures
are often used as
references by which
obliquity, adequacy
of inspiration and
exposure are assessed.

This is a chestx-rayin the PAview showingimportantbonystructures.The bonesare the most


radiopaque (whitest)structuresin a chestradiograph.Notethe prominenceof spinousprocesses
whichare equidistantfromthe bilateralclavicularheads.Alsonotethat the scapulais wingedout
in this image,a markerof the PAviewradiograph, andpromotesbettervisualization
of upperlung
fields.
82
D. Other Structures
I. Other Structures in PA View

1Aorticknob 5 Righthilum 9 Cardiophrenic


angle 13Stomach
2 Trachea 6 Lefthilum 10Costophrenic
angle 14Gastricbubble
3 Carina 7 Righthemidiaphragm 11Liver
4 Carinalangle 8 Lefthemidiaphragm 12Aorta
Theseare chestx-raysin the PAview.Thetrachea(2) is an areaof lucencylocatedin themiddle
andslightlypushedto the rightby the aorta(1) as it proceedscaudallyandeventuallybifurcates
intothetwomainbronchi.Notethattheright(5) andleft(6) hilaare roughlyof thesameleveland
density.Also,notethetwohemidiaphragms withtheright(7)slightlyhigherdueto theliverpushing
it cranially.The gastricbubble(14)is usuallyvisiblebelowthe left hemidiaphragm(8).Thesharp
costrophrenic angles(10)signifyabsenceof fluidin thosecompartments.

2. Other Structures in Lateral View

1 Trachea 4 Spine 7 Costophrenic


angle 9 Cardiophrenic
angle
2 Rightbronchus 5 Manubrium 8 Hemidiaphragm 10Gastricbubble
3 Leftbronchus 6 Sternum

Theseare chestx-raysin the lateralviewshowingotherimportantradiographic landmarks. The


trachea(1) is still visibleas an areaof lucencyin the middle.Notethatthe heartis cradledin
the spaceformedby the anteriorchestwall and the diaphragm(8). The costophrenic (7) and
cardiophrenic(9)anglesaredistinctlynoted.

83
• The trachea is noted at the center (or slightly to the right) as an
Airways area of radiolucency. It eventually branches at the carina into the
two main bronchi.
• The lung roots or the hila are occupied by the major bronchi and
the pulmonary vessels
Hilar structures
• The left hilum is often higher than the right
• Note that both should be of similar density and overall size
• Created by the dome of each hemidiaphragm & the lateral chest walls
Costophrenic
angles • On the frontal chest x-ray, the costophrenic angles should form
sharp acute angles
• Well-defined domed structures occupying the caudal end of the
chest x-ray film
• Right hemidiaphragm slightly higher than the left as the liver
Hemidiaphragms
below it pushes it cranially
• Below the left hemidiaphragm, a round area oflucency is
occasionally found (stomach bubble)
• Curved densities overlying both hemithoraces (which may be
Breast tissue asymmetric) may be noted
• Do not mistake these as underlying lung disease

STEP 5: DETERMINING THE PRESENCE OF ABNORMALITIES


A. Abnormalities in the Lung and Pleura
0Increased density in lung zones appears as areas which are whiter than surrounding
structures (e.g., consolidation, infiltrates, masses)
0Decreased density in lung zones appears as areas which are darker than surrounding
structures (area may be filled with air)
0Displacement (pulling/pushing) oflung fissures may be due to volume changes within
a certain lung lobe/zone
0Lung hyperexpansion causes bilateral blunting of costophrenic angles with flattening
of bilateral hemidiaphragms and distortion of lung markings

Reticular Opacities I Reticulo~t?dular


opac, ,es
I Consolidation

• May be fine (pulmonary • May be seen in • lnhomogenous opacities


edema), medium silicosis,sarcoidosis • Prominent air
(pulmonary fibrosis)or or lymphangitis bronchogram sign
coarse (usual interstitial carcinomatosis • In this image, the
pneumonia, sarcoidosis, ovoid radiopaque
Langerhans cell structure corresponds to
histiocytosis) consolidation
84
Atelectasis I Bronchiectasis I COPD/Emphysema

• Density in the area of the • Appears as "bunches of • Hyperaerated lungs


collapsed lung grapes" (ring shadows) • Flattened hemidiaphragms
• Displacement of inter- • Tram-track lines • Tubular heart
lobular fissures (direct sign) • In this image, the grape- • Occasionally, bullae
• Surrounding structures like structures in the formation
deviated to the side of the right lung represent • In this image, note that
collapsed lung (ipsilateral bronchiectasis the lung fields are darker
mediastinal shift) than usual, the mediastinal
• Ipsilateral diaphragmatic structures are compressed
elevation medially (tubular heart)
• In this image, atelectasis in and the hemidiaphragms
the RUL (appears white) are flattened
causes traction, pulling
structures ipsilateral to it
(pointed arrows)

Pleural Effusion I Pneumothorax I Pulmonary Metastasis

• Blunting of costophrenic • Hyperlucent pulmonary • Rounded, opaque


angles area nodules of variable size
• Meniscus sign • Loss of vascular markings ("cannonball lesions")
• Note the disappearance beyond visceral pleural line scattered throughout both
of the left costophrenic • Mediastinal structures lungs
angle in the image. The deviated to contralateral • Common primary sources
left lower lung field is side (tension of spread include breast
occupied by a radiopaque pneumothorax) and colorectal CA
substance (fluid, which is • In this image, air (black)
white, in this case), which occupies most of the right
exhibits a meniscus sign lung field, compressing it
(due to surface tension) & pushing structures to
contralateral side

85
B. Abnormalities in the Cardiac Size
° Cardiomegaly is defined as a CTR >50% in PA view (see computation of CT ratio above)
0 In the presence of cardiomegaly, it is best to identify which chambers are enlarged
Left Ventricular I Right Ventricular I Left Atrial Right Atrial
Enlargement Enlargement Enlargement I Enlargement
' ,:;;.-
#-s ,F's
e~ ::-=s 8
=
f:::
F· ' l r:
r_~
l~
'~

~··
'~
-
• Apex displaced • Apex displaced • Prominence ofleft • Bulging right heart
inferiorly and superiorly and atrial appendage border (height >I/2
laterally (drooping laterally (uplifted • Loss of cardiac of right cardiac
apex) apex) waistline silhouette and
• On lateral view: • On lateral view: • Widening of width >I/3 of right
obliteration of obliteration of carinal angle (>70°) hemithorax)
retrocardiac space retrosternal space • Double density
sign on right
cardiac border

C. Other Cardiac Abnormalities


Pericardia! Effusion I Aortic Aneurysm I Pneumomediastinum

• Generalized enlargement • Widened mediastinum • Presence of gas between


of the cardiac shadow >30% of thoracic diameter, the mediastinal structures
("water bottle sign") with or mediastinum >8-10 cm • In this image, the dark
normal vascular markings • In this image, the dark area surrounding the heart
• In this image, note the horizontal line marks the is air trapped within the
widened cardiac shadow widened mediastinum mediastinal compartment
which looks like a water • Other differentials for
bottle mediastinal widening
include technical factors
(e.g., improper positioning)
or masses

86
D Ab I'. . th P 1
I I
1t
Pulmonary Arterial Pulmonary Venous Decreased Pulmonary
Congestion Congestion Vascularity (Oligemia)

• Uniformly enlarged • Prominent hilar vessels • Decreased blood flow


vessels at the hila and (hilar fullness) in a bat-wing into the pulmonary
within the lungs distribution vasculature
• Loss of vessel tapering • Cephalization of pulmonary • Uniformly small size of
at the periphery vasculature pulmonary vessels
• Prominent convex- • Kerley A lines: thickened oblique • Concave-shaped (less
shaped main interlobular septae coursing prominent) main
pulmonary artery (PA) towards the hila pulmonary artery
segment in the left • Kerley B lines: thickened segment in left cardiac
cardiac silhouette subpleural interlobular septae at silhouette
• Diameter of the lung periphery • Lungs appear
descending branch of hyperlucent
right PA is larger than
diameter of the trachea
• Seen in acyanotic • Seen in left-sided heart diseases • Seen in congenital
congenital heart such as left ventricular failure heart diseases with RV
disease with increased and mitral or aortic valve disease outflow obstruction
pulmonary blood flow (e.g.,Tetralogy ofFallot,
due to left-to-right pulmonary stenosis), &
shunting pulmonary embolism

E. Other Structures
FINDINGS I DESCRIPTION AND DIFFERENTIALS
• Any phenomenon that increases volume or pressure in one
Tracheal hemithorax will push the trachea (and mediastinum) towards
displacement the contralateral side while any phenomenon which causes
volume loss will pull the trachea towards the ipsilateral side

• Enlargement in the area may point to space-occupying lesions


Hilar enlargement
(enlarged lymph nodes) or pulmonary hypertension
Abnormal hilar • As with the trachea, volume changes within a certain
position hemithorax may lead to pushing or pulling of the hilum

• Obscured hemidiaphragm may be due to adjacent lung


Diaphragmatic
disease; displacement may be due to presence of air or viscus
abnormalities
underneath or phrenic nerve palsy

Soft tissue • Irregular low densities within soft tissues may be due to air
densities tracking (subcutaneous emphysema)

87
SECTION TWO
P.LAIN ABDOMINAL RADIOGRAPHY

INDICATIONS FOR PLAIN ABDOMINAL RADIOGRAPHY


Emergent evaluation of bowel gas in suspected perforation or bowel obstruction
Evaluation of post-procedural and post-operative free gas
Evaluation of calculi and radiopaque lines, tubes, and foreign bodies
Evaluation of other abdominal conditions (e.g., acute inflammatory bowel disease or
intussusception in pediatric patients)

NORMAL ANATOMY SEEN IN PLAIN ABDOMINAL RADIOGRAPHY

·...
:Ascending; : · ·.. Descending":
\ Colon / \ ·· .. Colon

·. :

I. SOME TISSUES SEEN IN PLAIN ABDOMEN


STRUCTURE I REMARKS

• Seen as a gray area in the right upper quadrant


Liver
• Superior edge forms the right hemidiaphragm contour
• Lateral edges arise inferolaterally in a near-straight line from the
Psoas muscles
transverse processes of the lumbar vertebrae towards the hip muscles
• Lie at the T12-L3level, lateral to the psoas muscles
Kidneys • Right kidney usually slightly lower than the left kidney due to the position
of the liver, and also appears slightly smaller
Spleen • Lies in the left upper quadrant immediately superior to the left kidney

• May be visible in the left upper quadrant of the abdomen, below the left
Stomach hemi-diaphragm
• Lowest part of the stomach crosses the mid line
Small • Occupies a central position in the abdomen
intestine • May appear as pockets of polygonally-shaped gas
Large • Occupy peripheral positions in the abdomen
intestine • Feces can be identified by a mottled appearance due to its partly gaseous content
88
r SECTION THREE
BASIC COMPUTERIZED TOMOGRAPHY (CT) IMAGING

NORMAL CHEST CT ANATOMY IN AXIAL VIEW

Main Bronchi

Esophagus ---

The pulmonary trunk divides into the right and left pulmonary arteries. The ascending aorta
passes dorsally to the pulmonary trunk but ventrally to the right pulmonary artery, with the
superior vena cava to its right. The right pulmonary artery courses posterior to the ascending
aorta and anterior to the right main bronchus. The left pulmonary artery passes anterior to the
descending aorta and lies superior to the left main bronchus. The esophagus lies posterior to the
left main bronchus and passes to the right of the descending aorta. The appearance of air in the
lungs gives them a hypodense appearance.

NORMAL ABDOMINAL CT ANATOMY IN AXIAL VIEW


Colon

Right Portal Vein

Inferior Vena Cava


Spleen

Right Adrenal Gland

Aorta

The liver lies to the right and is usually the most prominent structure on abdominal CT.
Its normal density is usually brighter than the spleen and surrounding muscles, and the
parenchyma should appear solid and homogenous. The stomach lies to the left of the liver, with
the body of the pancreas posterior to the distal portion of the stomach. The spleen lies on the
leftmost side and should possess a concave visceral surface. The adrenal glands lie superior to
and slightly medial to the kidneys, which should appear elliptical in shape. The abdominal aorta
runs retroperitoneally, anterior to the vertebral bodies, with the inferior vena cava to its right.
89
CRANIAL CT SCAN
• CT images of the brain are conventionally viewed from below, as if looking up into the
top of the head
Thus, the right side of the brain is on the left side of the viewer, and the anterior part of
the head is at the top of the image
White matter is located more centrally and appears darker than grey matter, and CSF
appears darker than either grey or white matter due to its lower density

I. NORMAL STRUCTURES
The diagrams below represent a normal cranial CT in axial view
• Lobes should generally appear symmetric
Frontal lobes generally appear larger than the other lobes
Ventricles appear darker than lobes as they contain CSF

Frontal Bone

Choroid plexus

Occipital Bone

Frontal bone

Temporal Lobe
Suprasellar Cistern

Temproal Bone

Mastoid Air Cells

Occupltal Bone

90
II. COMMON CRANIAL PATHOLOGIES

II

• Hyperdensity (bright) on the right


• Hypodensity (dark) on the right capsuloganglionic area consistent with a
frontotemporoparietal area consistent hemorrhage (bleed is BRIGHT on CT)
with an infarct in the right !CA territory • There may be shift of midline structures
(infarct is DARK on CT) to the contralateral side if amount of
hemorrhage is significant

Estimatin Bleed Volume in CT scan (Kothari Method):

AxBxC
Volume of Blood (in cc)=
2

Normal

• A: largest hematoma diameter (in cm)


• B: diameter perpendicular to A (in cm)
• C: number of CT scan slices with hemorrhage x slice thickness (in cm)

•Jn counting CT slices with hemorrhage (C):


• If >75% oflargest hematoma size: count as I slice
• If 25-75%:count as 0.5 slice
• If <25%:disregard that CT slice
Source:KothariRU,et al. Stroke;1996

91
Subarachnoid Hemorrhage I Extradural Hematoma

LocaUon of
Coronal Suture

• Characterized by blood in subarachnoid • Lens-shaped collection of blood in


space (white in image above) extradural area following head trauma
• Hyperdensities (blood) may be seen in the • The collection of blood is usually limited to
basal cisterns, fissures, and ventricles the points of the sutures

Hydrocephalus I lntracerebral Mass

Edema

• lntracerebral masses may appear as focal


• An expansion of the ventricular system areas of high- or mixed-density, with
due to increased CSF volume contained surrounding edema
within it • Mass effect may be seen with effacement
• Ventricles appear disproportionally dilated of ventricles and sulci, and shift of midline
structures to the contralateral side

REFERENCES
1.Coche EE, et al., Comparative Interpretation of CT and Standard Radiography of the Chest, 27 Medical Radiology, DOI:
10.1007/978-3-540-79942-9_2,© Springer-Verlag Berlin Heidelberg; c2011.
2.Chu DG, MillerW.too Norma.IChest X-rays.Universityof Pennsylvania Accessibleonline. h1rp-J/ww·w.med.upenn.edu/normal0<.r/
J. Gay SB, Ola1..agastiJ,HigginbothamJW, Gupta A, Wurm A, Nguyen J.Introduction to Chest Radiology. University of Virginia.
2013.Accessible online. lmps://\\T\V\Y.med-ed.virginia.edu/courses/rad/cx.r/
4. Heuer AJ, Scanlan CL Chest Imaging. Wilkins' Clinical Assessment in Respiratory Care, 7th edition. Missouri: Elsevier Health
Science; c2014.
5. Kothari RU, Brott T, Broderick JP, Barsan WG, Sauerbeck LR, Zuccarello M, et al. The ABCs or measuring intracerebral
hemorrhage volumes. Stroke. 1996;27(8):1305•5
6. Radiology Care. Accessible on line. http://www.radiologycare.com/
7. Radiology Masterclass 2017.Accessible on line. htcp:l/www.radiologymasterclass.co.uk/tu1orials
8. Radiopaedia. Accessible online. hnps://radiopaedia.org/anicles/
9.Smith MB. Interpreting Chest X-rays.American Journal orRoen1genology. 2011;196: W352-W352
10. Sys1ernic Analysis or an Abdominal X•ray. The University or Nottingham. Accessible online. hnps://nle.nottingham.ac.uk/
websites/abdominal_radiology/viewingapproach.html
11. Undergraduate Diagnostic Imaging Fundamentals. Accessibleonline. https:1/undergradimaging.pressbooks.com/
12. Voigt S. How <oRead a Chest X·ra)'•A Step by Step Approach. SSMJ. 2008. <(2)

92
BASIC
PROCEDU
1. Nasogastric Tube Insertion
2. Intravenous Line Insertion
3. Foley Catheter Insertion
4. Endotracheal Intubation via Direct Laryngoscopy
5. Thoracentesis
6. Pericardiocentesis
7. Arthrocentesis
8. Lumbar Tap
9. Abdominal Paracentesis
10. Central Line Insertion (lntrajugular)
11. Arterial Line Insertion (Radial)
12. Bone Marrow Aspiration
13. Needle Decompression (Needling)
Physicianspeiform a wide range of medicalproceduresfor diagnosticand therapeuticpurposes.Internal
Medicine specialists peiform venipuncture, intravenous line insertion, arterial puncture, intubation,
nasogastrictube insertionand urinary catheterization.Other internalmedicinesubspecialistsalsopeiform
specialproceduressuch as centrallineinsertions,arterialline insertions,bonemarrowaspirationbiopsyand
lumbarpuncture.Forstudents rotatingin internalmedicine,at times, they are askedto peiform or assist in
peiformingprocedures.Hence,knowledgeon the indicationsand contraindications as well as materialsand
steps in peiformingthe procedurewould be helpful. Note that all proceduresshouldbeginwith the operator
first identifyingthe patient by name, introducinghim/herselfto patient,explainingthe nature of procedure,
and obtaininginformed consent. Properhand hygiene should be done beforeand after each procedure.
Materials(includingsharps) should be discardedproperly.Properdocumentationshould be made, which '.
shouldincludethe patient'sresponse,any adverseevents/complications, and laboratorywork-upsent. ·· i
NASOGASTRIC TUBE (NGT) INSERTION
INDICATIONS AND CONTRAINDICATIONS I MATERIALS NEEDED
Indications: • Nasogastric tube
• Management of ileus or gut obstruction • Asepto syringe
• Administration of medications • Sterile lubricating gel
• Nutritional support • Clean (non-sterile) gloves
• Lavage
• Zinc oxide tape
Contraindications: • Stethoscope
• Esophageal stricture
• Basilar fracture
• Facial fracture
• Caustic ingestion
STEPS
r. Wear clean gloves & approximate length using the distance from the tip of the nose to the top
of the ear to the tip of the xiphoid (seefigure below)
2. Lubricate the nasogastric tube with sterile gel
3. Gently insert tube in one nostril while instructing the patient to swallow
4. Confirm placement by injecting air using Asepto syringe while auscultating the epigastric area
5. Repeat process by auscultating the lungs
6. Once placement of tube is confirmed, secure the tube using zinc oxide tape
7. Loop the tube and cover the tip by partially inserting a part of the tube into the opening (or
keep open if used for decompression)
8. lnstruct the patient and relatives on care ofNGT

MarkedNGTlocation
at edge of nostril

Figureaboveshowshowto approximatethe lengthto be inserted.


95
INTRAVENOUS LINE INSERTION

INDICATIONS AND CONTRAINDICATIONS I MATERIALS NEEDED

Indications: • IV cathula
• Administration of therapies that are better given • Macroset/IV tubing
intravenously (e.g., antibiotics, chemotherapeutic drugs) • Clean (non-sterile)
• Administration of blood and blood products gloves
• Clinical situations requiring emergency treatment using • Tourniquet
IV medications such as cardiac arrest • Cotton
• Nutritional support for those who cannot tolerate oral • Rubbing alcohol
intake (e.g., patients who underwent abdominal surgery) • Micropore
• Splint (optional)
Contraindications:
• Appropriate therapy can be given via a less invasive route
(e.g., enterally or per orem)
• Insertion in an infected, injured or burned extremity
should be avoided (if possible)

STEPS
1. Wear clean gloves and select position/site ofvenipuncture
2. Swab puncture site with alcohol in concentric circles inside to out
3. Apply tourniquet & stabilize the selected vein, then insert needle bevel up (see images below)
4. Observe for blood backflow
5. Remove needle while pushing cathula further into the vein (see images below)
6. Attach infusion set quickly while pressing on vein to prevent excessive escape of blood
7. Release tourniquet
8. Try running the IV line to check that fluid infuses continuously and there is no bulging at the
insertion site; then cover the insertion site with a 1x1 sterile gauze and tape securely
9. Loop tubing, secure with tape and apply a splint (optional)
10. Instruct the patient on care of IV site

(A) Insertthecannulaat a -45 degreeangleandobservefor backflowof blood.


(B)Slowlyinsertthecatheterintotheveinwiththeneedleretractedto avoidthroughandthroughpuncture.
(C)Completely removetheneedleandsecurethecatheter.

Various IV Catheters Available


SIZE I RECOMMENDED USE

14G • Trauma, rapid blood transfusion, surgery

16-18G • Trauma, rapid fluid replacement, rapid blood transfusion

20G • Most infusions, trauma, rapid fluid replacement, routine blood transfusion

22G • Most infusions (neonate, pediatric, older adults), routine blood transfusion

24G • Routine blood transfusion, neonate or pediatric blood transfusion

26G • Pediatrics, neonates

96
FOLEY CATHETER INSERTION
INDICATIONS AND CONTRAINDICATIONS I MATERIALS NEEDED

Indications • Foley catheter with urine bag


• Management of urinary retention ° Fr Size Men: 14-16
• Accurate monitoring of urine output in the ° Fr Size Women: 10-12
following situations: • Drapes
° Critically ill patients • IO cc syringe
0 During prolonged surgery • 1 vial sterile water
0 Patients requiring fluid monitoring during surgery • Clean (non-sterile) gloves
0 Urinary incontinence • Sterile gloves
• Daily urine output used for diagnostic purposes • Cotton
• Post-prostate surgery • Rubbing alcohol "
• Hematuria with clots • Povidone-iodine .
• Prolonged immobilization • Lubricant (KY jelly)
• Neurogenic bladder • Micropore
• Improve comfort for end-of-life-care • Sterile specimen bottles (if for
Contraindications: urine collection)
• Presence of urethral injury
• Gross hematuria
• Urethral infection
STEPS
1. Provide as much privacy as possible
2. Position patient properly
3. Wash and rinse urethral area
4. Open Foley catheter package, put aside but maintain sterile zone around Foley catheter
5. Wear clean gloves; clean urethral opening aseptically:
a. For male: in circular motions inside to out
b. For female: follow a "7''figure then drop
6. Attach drainage end of Foley catheter to urine bag
7. Change to sterile gloves
8. Lubricate tip of catheter liberally
9. Insert lubricated end of catheter into urinary meatus gently then push in until sure that
catheter tip is well inside the bladder (usually up to the port where you inject water and there
is urine backflow)
10. Observe for urine flow
11. Infuse 5-10 mL of sterile water to inflate balloon
12. Pull Foley catheter slowly until with some resistance
13.Secure Foley catheter with tape, dry patient's perineum, & instruct patient on catheter care

Figuresaboveshowthe usualset-upof indwellingFoleycatheterin a female(left)andmale(right)patient

97
ENDOTRACHEAL INTUBATION VIA DIRECT LARYNGOSCOPY
INDICATIONS AND CONTRAINDICATIONS I MATERIALS NEEDED
Indications: • Endotracheal tube
• Acute respiratory failure • Styler
• Airway protection • Sterile lubricant
• Preoperative setting in patients requiring general • Laryngoscope
anesthesia • Zinc oxide tape
• Advanced airway in patients suffering from cardiac • Sterile gloves
arrest • Oxygen tubings
• Bag valve mask
Contraindications: • Suction device
• Trauma to upper airway • Lidocaine spray
• Pulse oximeter
• Cardiac monitors
• Sedative medication
• IO cc syringe
• Stethoscope
STEPS
I. Obtain assistance, assess, preoxygenate and position patient
2. Open the patient's mouth and carefully position the laryngoscope (see image below)
3. Deflect the tongue and soft tissue to the left side of the mouth with the flange
4. Identify and locate the epiglottis
5. Improve view by using bimanual laryngoscopy, head elevation, and lower neck flexion (not
done when there is suspected neck trauma/injury)
6. Insert the endotracheal tube through the vocal cords into the trachea under direct
visualization (see image below)
7. Remove the sty let and inflate balloon
8. Confirm positioning of tube within trachea (e.g., visualization, auscultation, CO2 detection)
9. Check for any secretions/output from the ET tube
to. Secure the tracheal tube and hook to mechanical ventilator; provide sedation and analgesia
11.Send for post-intubation AP films to confirm tube placement and adjust if needed
12.Send for post-intubation blood gases to check oxygenation and gas exchange as well as to
guide adjustment of mechanical ventilator settings

Rapid Sequence Intubation (RSI):


• Technique that induces immediate unresponsiveness (induction agent) and muscular
relaxation (neuromuscular blocking agent)
0 Induction medications: propofol (2 mg/kg IV), midazolam (0.3 mg/kg IV)
0 Paralyzing agents: succinylcholine (2 mg/kg IV), rocuronium (1-1.2mg/kg IV)
• Fastest and most effective means of controlling the emergency airway

98
THORACENTESIS
INDICATIONS AND CONTRAINDICATIONS I MATERIALS NEEDED

Indications: • Thoracentesis set


• Bilateral pleural effusions that are markedly • Abbocath catheter gauge #16
different in volume • 3-way stopcock
• Pleurisy • Macroset/IV tubing
• Features suggestive of infection such as fever • Drapes
• Imaging features suggestive of a cause of the • 50 cc syringe
effusion other than heart failure (HF) • 10 cc syringe
• Non-resolution of effusion despite HF therapy • Lidocaine 2% ampules
• Clean (non-sterile) gloves
Relative contraindications: • Sterile gloves
• Bleeding diathesis • Rubbing alcohol
• PT/PTT twice the midpoint of normal range • Povidone-iodine
• Platelet count <50,000/mm' • Sterile gauze & cotton
• Serum Crea >6 mg/dL • Micropore
• Active infection at the site of puncture • Sterile specimen vials/bottles
STEPS
1. Examine the patient and review available labs (CXR, CBC, blood chemistry, bleeding parameters)

2. Extract simultaneous serum specimen for LDH, albumin, total protein and glucose
3. Position patient in sitting position with the posterior axillary line accessible for needle insertion
4. Confirm & mark topmost site of insertion by counting ribs based on CXR & percussing fluid level
(usual site of insertion is at the 8th JCS posterior axillary line; chest UTZ with markings can be done)
5. Observe sterile technique (e.g., use of sterile gloves, Povidone-iodine prep, and drapes)
6. Anesthetize skin over insertion site with 2% lidocaine, including the superior surface of the rib and pleura
7. Insert thoracentesis needle perpendicularly through the anesthetized area to the same depth as the
first needle and observe for backflow of pleural fluid
8. Once with backflow, leave the catheter in place, secure it with micropore, remove needle and attach
3-way stopcock and tu bing
9. Aspirate needed amount, then turn the stopcock and evacuate fluid through the tubing (do not
remove more than 1.5L to avoid increased risk of re-expansion pulmonary edema or hypotension)
10. Fill specimen tubes/containers and label properly
11. When draining offluid is completed, have patient take a deep breath or ask patient to cough and
gently remove needle
12. Cover insertion site with sterile occlusive dressing
13.Send specimens for qualitative studies (pH, specific gravity, cell count and differentials, protein,
LOH, albumin, glucose), gram stain and culture, AFB smear and additional studies as necessary
(e.g., cytology for malignancy, amylase for pancreatitis, triglycerides for chylothorax)
14. Monitor patient closely and watch out for untoward reactions (chest pain, dyspnea, cough, infection)
15.Obtain upright CXR to evaluate lung expansion/fluid level and rule out pneumothorax
16.Provide post-procedural analgesics as necessary

Internallntercostal
muscle
Innermost lntercostal
muscle

Costalgroove

Neurovascular
Bundle
@ lntercostal
vein
• lntercostal
artery
• lntercostal
nerve

99
PERICARDIOCENTESIS
INDICATIONS AND CONTRAINDICATIONS I MATERIALS NEEDED
Indications: • Local anesthetic (e.g., lidocaine 1%)
• Presence of life-threatening hemodynamic • Chlorhexidine-based skin preparation
compromise in a patient with suspected solution or Povidone-iodine
cardiac tamponade • Plastic drainage tubing
• To identify the cause of pericardia! effusion • Spinal needle gauge 18
through microscopic, biochemical and • Sterile drapes, gown, and mask
culture studies • Syringes, IO mL and 60 mL
• Ultrasound (or handheld echo)
Absolute contraindication:
• Three-way stopcock
• No absolute contraindication in a
hemodynamically unstable patient
Relative contraindications:
• Uncorrected bleeding disorder
• Traumatic cardiac tamponade
STEPS
1.Examine the patient and review available labs (echo, CBC, blood chemistry, bleeding parameters)
2. Extract simultaneous serum specimen for LDH, albumin, total protein and glucose
3. The patient should be placed in a supine or 30 to 45 degree semirecumbent position
4. Identify landmarks - xiphoid process, 5th and 6th ribs and the left sternocostal margin (puncture site)
5. Observe sterile technique (e.g., sterile gloves, Povidone-iodine prep, and drapes)
6. Anesthesize the area by making a skin wheal first and penetrating deeper tissues
7. Introduce the needle substernally r cm inferior to the left xiphocostal angle
8. Once below the cartilage cage, adjust the angulation of the needle to make it approximately 30
degrees with respect to the chest wall
9. Target the left mid-clavicle and advance needle slowly while applying negative pressure. If no
fluid comes out to the syringe, needle should be withdrawn and re-directed. lf available, use the
ultrasound to help target the largest pocket offluid. In there is no UTZ, withdraw needle and
redirect it along a deeper and posterior trajectory.
10. If there is no fluid withdrawn at the second attempt, pull back the needle and redirect 10 degrees to
the patient's left. Perform systematic redirection until the needle is aimed at the left shoulder.
11.Once fluid aspirated, withdraw as much fluid as possible. Stabilize the needle against the patient's
torso. Use a 3-way stopcock and attach intravenous tubing to allow aspiration of pericardia! fluid
into the syringe and eject the fluid into a basin.
12.Remove the needle when fluid can no longer be aspirated.
13.Cover insertion site with sterile occlusive dressing
14. Send specimens for qualirative studies (pH, specific gravity, cell count and differentials, protein,
LOH, albumin, glucose), gram stain and culture, AFB smear and additional studies as necessary
(e.g., cytology for malignancy)
15.Monitor patient closely and watch out for untoward reactions (chest pain, dyspnea, cough, infection)
16. Obtain upright CXR to rule out pneumothorax
17.Provide post-procedural analgesics as necessary

100
Laboratory Work-up to Request for Aspirated Fluids (Pleural and Pericardia/ Effusion)
TEST TO
REQUEST I REMARKS

· Chemieal-q,ndBiochemicalTests ... ·IC '

• Low pH (<7.3)can be secondary to infection, pleuritis, tumor or


pH
fibrosis
• Transudative fluid: < 3.0 g/dL (30 g/L)
Protein • Tuberculous: >4.0 g/dL (40 g/L)

I
• Multiple myeloma: 7 to 8 g/dL (70-80 g/L)
Lactate
• Done to help determine the etiology of the pleural fluid
dehydrogenase
(LOH) • Key in differentiating transudates and exudates '

• Low glucose concentration (<60 mg/dL) narrows differential to


Glucose
possible infection, malignancy, lupus, tuberculosis
Cytology • Can establish malignant effusion but with low sensitivity (<60%)
'
Cell couritand,diffe1:enti-al - .Sc "" :,

• Complicated parapneumonic pleural effusions: >50,000/µL


Cell count • Exudative effusions: >IO,ooo/µL
• Chronic exudates: <5,000/µL
• If predominant (85-95%)effusion can be secondary to
0 Tuberculosis
Lymphocytosis 0 Lymphoma
0 Sarcoidosis
0 Malignancy (half of the cases)
• Presence of >IO%of the differential count
° Chronic eosinophilic pneumonia
0 Drugs
° Fungal infection
0 Hemothorax
Eosinophilia 0 Malignancy
0 Parapneumonic effusion
0 Parasitic disease
0 Pneumothorax
0 Pulmonary infarction
0 Tuberculosis
Microbiology·
Gram stain • To identify bacteria present
Culture
• To determine sensitivity of bacteria to antibiotics
studies
AFB smear • To detect presence of Myocbacterium

101
ARTHROCENTESIS
INDICATIONS AND CONTRAINDICATIONS I MATERIALS NEEDED

Indications: • Sterile gloves


• For diagnosis and therapeutic relief of • Sterile gauze, dressing
synovial effusion • Sterile drapes, gown, and mask
• For evaluation of septic arthritis in the • Sterile specimen vials/bottles
setting of acute monoarthritis • Adhesive tape
• For initial confirmation of gouty arthritis • Syringes: 5 mL, JO mL, or 20 mL,
• For prevention of adhesions in cases of depending on joint & size of effusion
hemarthrosis • Hemostat
• For suspected acute prosthetic joint infection , Antiseptic solution with skin swabs
• Local anesthetic (e.g., lidocaine 1%)
Contraindications: • For large joints (e.g., shoulder, knee,
• Uncorrected bleeding disorder ankle): 1.5-inch 21-to 18-gauge needle
• Overlying skin infection/cellulitis • For medium joints (e.g., wrist): 1-in.
(2.5-cm) 21-gauge needle
• For small joints (e.g., MCP, MTP): I-in.
(2.5-cm) 25-gauge needle
STEPS
1. Examine the patient and review available labs (e.g., CBC, blood chemistry, bleeding parameters)
2. Extract simultaneous serum specimen for uric acid analysis
3. The patient should be placed in a supine or sitting position, depending on the joint to be aspirated
4. Identify landmarks - coracoid (shoulder); olecranon process, lateral epicondyle, radial head (elbow);
radiocarpal joint (wrist); patella (knee); tibialis anterior (ankle); extensor tendon (MCP, MTP)
5. Observe sterile technique (e.g., sterile gloves, Povidone-iodine prep, and drapes)
6. Anesthetize the area by making a skin wheal first and penetrating deeper tissues
7. Introduce the needle 4 cm lateral and inferior co the coracoid for shoulders; laterally into the
triangle formed by the olecranon process, lateral epicondyle, and radial head for elbows; dorsal
into the dimple over the radiocarpal joint for wrists; posteriorly and medial to the patella for knees;
anteriorly into the divot medial to the tibialis anterior for ankles; dorsally and medial or lateral to
the extensor tendons for MCP and MTP joints
8. Maintain a horizontal needle trajectory and aspirate as the needle is advanced. If no fluid comes out
to the syringe, needle should be withdrawn and re-directed
9. 0nce fluid aspirated, withdraw as much fluid as possible. If the first syringe is full and there is still
fluid in the joint, firmly grip the distal half of the syringe using a hemostat and switch to a new
syringe without removal of the needle from the joint
10. Remove the needle when fluid can no longer be aspirated
tt. Immediately apply pressure on insertion site and cover with sterile occlusive dressing
12.Send specimens for qualitative studies (viscosity, cell count and differentials, protein, glucose),
Gram stain and culture, crystal analysis
13.Monitor patient closely and watch out for untoward reactions (hemorrhage)
14.Provide post-procedural analgesics as necessary

102
Laboratory Work-up to Request for Synovial Fluid
TESTTO
REQUEST I
Chemicaland BiochemfoalTests
REMARKS
.
s: ·- ;;;:
Glucose • Decreased in inflammatory or septic arthritis

Protein • Increased (>3 g/dL) in inflammatory and hemorrhagic arthritis

Uric Acid
• Increased (>8 g/dL) in crystal-induced arthritis
(serum)
• LE Cell: lupus erythematosus
• Reiter Cell: Reiter syndrome, non-specific inflammation
• RA Cell: rheumatoid arthritis, immunologic inflammation
Cytology • Rice bodies: tuberculosis, septic arthritis, rheumatoid arthritis
• Cartilage cells: osteoarthritis
• Fat droplets: traumatic injury, chronic inflammation
• Hemosiderin: pigmented villonodular synovitis
·" . ~, 'K ", ., .,
Cell'Countand Differential .,,. l ~- •J, .
"' "'"
• Leukocyte count >IO,ooo/µL: crystal-induced arthritis (e.g., gout,
pseudogout), chronic inflammatory arthritis (e.g., rheumatoid
Cell count arthritis, SLE, ankylosing spondylitis), septic arthritis
• Leukocyte count <IO,ooo/µL: osteoarthritis, osteochondritis
dissicans, trauma, and synovioma
• If >50%, the synovial effusion can be secondary to:
0 Urate gout
0 Pseudogout
Neutrophilia 0 Rheumatoid arthritis
• If >75%, the synovial effusion can be secondary to:
0 Acute bacterial arthritis

Lymphocytosis • If >15%,could indicate early rheumatoid arthritis


,., o''•"!i'i ;~}°if
M_jcrobiology~ :"·f·" .... f·
h'
:,, .,.., .~,- ~l

Gram stain • To identify bacteria present


Culture
• To determine sensitivity of bacteria to antibiotics
studies
AFB smear • To detect presence of M ycobacterium
.,
Crystal Anal1fsis:
.,,
?!.'
.,
11!1 -i''
..,,
,,,: 0
1¢,ii §; -~

• Monosodium urate: gouty arthritis


• Calcium pyrophosphate: pseudogout
• Cholesterol: hyperlipoproteinemia and RA
• Corticosteroid: after intra-articular steroid injections
• Calcium oxalate: renal dialysis

103
LUMBAR TAP
INDICATIONS AND CONTRAINDICATIONS I MATERIALS NEEDED

Indications: • Sterile gloves


• For suspected CNS disease (e.g., meningitis, • Sterile drapes, gown, mask
Guillain-Barre syndrome, malignancy), • Sterile gauze, dressing
particularly infection • Antiseptic solution with skin swabs
• For suspected subarachnoid hemorrhage • Local anesthetic (e.g., lidocaine 1%
with an unremarkable CT scan without epinephrine)
• For therapeutic relief of idiopathic • Syringe, 3 mL
intracranial hypertension (pseudotumor • Needles, 20- and 25-gauge
cerebri) • Spinal needles, 20- and 22-gauge
• For therapeutic maneuvers (e.g., spinal • Three-way stopcock
anesthesia, intrathecal administration of • Manometer
medications) • Four plastic test tubes, numbered 1-4,
with caps
Absolute contraindications:
• Presence of skin infection over puncture site
• Unequal pressure between the supratentorial
and infratentorial regions

Relative contraindications:
• Increased intracranial pressure
• Uncorrected bleeding disorder
• Suspected spinal epidural or brain abscess
STEPS
I. Examine the patient and review available labs (e.g., CT scan, CBC, blood chemistry, bleeding
parameters)
2.The patient should be placed in a lateral decubitus position with the hips, knees, and chin
flexed toward the chest
3. Identify landmarks - right and left posterior superior iliac crests - to locate the L3-Li
interspace (seeimage below)
4. Observe sterile technique (e.g., sterile gloves, Povidone-iodine prep, and drapes)
5. Anesthetize the area by making a skin wheal first and penetrating deeper tissues
6. Introduce the needle at a slight cephalad angle with the bevel of the needle facing up
(parallel to longitudinal dural fibers)
7. Slowly advance the needle until a characteristic "pop" is felt. Remove the styler and observe
for CSF return. If no fluid comes out, sty let should be replaced and the needle should be re-
directed, then check again for CSF return
8. Once there is CSF return, instruct the patient to slowly extend their legs. Attach the sterile
manometer through the 3-way stopcock to measure the opening pressure. Empty the
manometer into Tube# I and then at least IO drops ofCSF into Tubes# 2-4
9. Replace the styler and remove the needle
IO. Immediately apply pressure on insertion site and cover with sterile occlusive dressing
11. Sendspecimens for analysis: Tubes #I and 4 for cell count and differential, Tube #2 for
glucose and protein, Tube #3 for Gram stain and culture sensitivity
12.Monitor patient closely and watch out for untoward reactions (headache, hemorrhage,
hypertension, loss of consciousness, seizure)
13.Provide post-procedural analgesics as necessary

104
I

sac surrounding the


spinal cord
1s·

Laboratory Work-up to Request for Cerebrospinal Fluid

TESTTO
REQUEST
I REMARKS
· Chemicalanii BiochemicalTests
• CSF glucose level normally approximates 60% of peripheral blood
Glucose
• Low glucose suggests bacterial infection

Protein • High in demyelinating polyneuropathies, or postinfectious states

Cell count and • Increased white blood cells (WBCs) suggests an infection or
differential leukemic infiltration
count • Increased WBC and RBC suggests a traumatic tap

Gram stain • To identify bacteria present

Culture studies • To determine sensitivity of bacteria to antibiotics

AFBsmear • To detect presence of Mycobacterium

105
ABDOMINAL PARACENTESIS
INDICATIONS AND CONTRAINDICATIONS I MATERIALS NEEDED

Indications: • Abbocath catheter gauge # 16


• New-onset ascites or hospitalization due to ascites • Macroset/lV tubing
• Management of diuretic-resistant ascites • IO cc and 50 cc syringe
• Ascites with fever, abdominal pain, leukocytosis, • Lidocaine 2% ampules
acidosis or azotemia, encephalopathy • Clean (non-sterile) gloves
• Sterile gloves
Relative contraindications: • Alcohol & povidone-iodine
• Bleeding diathesis • Sterile gauze & cotton
• Massive ileus/bowel distention • Micropore
• Infection at the site of puncture • Sterile specimen vials/bottles

STEPS
1. Examine the patient and review available labs (ultrasound, CBC, blood chemistry, PT/PTT)
2. Have patient empty bladder prior to procedure
3. Extract simultaneous serum specimen for LOH, albumin, total protein and glucose
4. Assemble materials and prepare sterile field
5. Place patient in supine position with the trunk elevated at 45 degrees
6. Confirm and mark the site of access (usually midline, 3-4 cm below umbilicus, halfway
between symphysis pubis and umbilicus)
7. Anesthetize skin over insertion site with 2% lidocaine, down to and including the peritoneum
8. Insert needle perpendicularly through the anesthetized area to the same depth as the first
needle and observe for backflow of fluid
9. 0nce with backflow, leave the catheter in place, remove needle and attach tubing draining
into specimen bottles
IO. Remove the necessary amount of ascitic fluid & monitor for hypotension
11. When draining is completed, remove needle & cover insertion site with occlusive dressing
12. Fill specimen tubes/containers and label properly & send specimens for qualitative studies
0Routine: cell count & differential, albumin, total protein
0Optional (usually when infection suspected): GS/CS (in blood CS bottles), glucose, LOH
0Unusual: AFB smear & culture, cytology for malignancy, triglyceride, bilirubin
13. Provide post-procedural analgesics as necessary

Umbillcus"'-
Puncturesite:
3. 4 cmbelowumbilicus,
orhalfwaybetweenumbllicus
X····y·····)(
andsymphysls pubis

45°(

Thefigureon the rightshowsthe threeusualsitesfor paracentesis


(markedwithan "X"). The left lower
quadrantis the preferredlocationby some(2 fingerbreadths
medialto anteriorsuperioriliacspine)
Source:RunyonBA.TheAmerican
Association
fortheStudyof LiverDiseases.
Practice;
2012
106
CENTRAL LINE INSERTION (INTRAJUGULAR)
INDICATIONS AND CONTRAINDICATIONS I MATERIALS NEEDED

Indication: • Sterile gloves, drapes, gown, mask


• For hemodynamic monitoring of critically ill • Antiseptic solution with skin
patients swab (e.g., 2% chlorhexidine)
• For fluid resuscitation • Sterile saline flush, 30 mL
• For emergency venous access • Local anesthetic (e.g., lidocaine 1%)
• For long-term access for parenteral nutrition • Scalpel, blade number 11
• For drug infusions with narrow therapeutic index • Central line kit
• Fr 6 to 8 catheter & dilator :. .
Relative contraindications: • )-tipped flexible guidewire
• Distorted local anatomy (e.g., trauma)
• Needles, 18-and 22-gauge
• Presence of skin infection over insertion site
• Syringe, 5 mL
• Thrombosed veins or proximal vascular injury • Polypropylene suture 4-0
• Severe coagulopathy & hemorrhage
STEPS
J. Examine patient & review available labs (e.g., UTZ, CBC, bleeding parameters, liver function)
2. The patient should be placed in a Trendelenburg position (I0-15° head-down) with the head
turned to the contra lateral side of the insertion site
3. Use an ultrasound to identify the target vein or use landmarks: palpate the carotid artery & locate
the clavicular & sternal heads of the sternocleidomastoid (SCM).Expose neck from the angle of the
jaw superiorly, to the nipples inferiorly, to the midaxillary line laterally, & to the sternum medially
4. Observe sterile technique (e.g., sterile gloves, Povidone-iodine prep, and drapes)
5. Anesthetize the area by making a skin wheal first and penetrating deeper tissues
6. Insert 18-gauge introducer needle at a 45° angle at the medial edge of the sternal head of SCM
7. Advance the needle directed lateral to the carotid pulsation, aiming for the ipsilateral nipple
while aspirating. Once venous blood is aspirated, remove syringe from the needle & advance
guidewire into the vessel, with the tip being positioned in the superior portion of the IVC
8.Once ~15cm of the guidewire has been advanced with a slight rotating motion, remove the
introducer needle while holding the guidewire in place
9. Make a 3-mm incision using an 11-bladescalpel around the guidewire and advance dilator
over the guidewire, making sure that the guidewire is not being advanced or pulled out
IO. Once ~3-4 cm of the dilator has been advanced, withdraw the dilator while maintaining the
guidewire position then apply immediate pressure on the exit site
11.Advance the catheter over the wire into the vessel. If there is any resistance while advancing
the catheter, withdraw the catheter & re-dilate the site. Once ~16-18cm (right-sided) or ~20
cm (left-sided) of the catheter has been advanced, carefully remove the guidewire
12.Flush the line with sterile saline & secure catheter with sterile adhesive bandage or dressing
13.Watch out for untoward reactions (e.g.,hemorrhage, hematoma, air embolism, pneumothorax)
'-.....)

\ Markerfor
Carotidartery
Sternalhead
Internal of the SCM
JugularVein

Imageon the leftshowsthelandmarks thatmustbe identified.


Thecentralapproach
is the mostcommonly
used(imageon the right).Findthe bifurcation
of the SCMmuscleandinsertyourneedlejust caudalto its
apexat a 30° angle,aimingtowardthe ipsilateralnipple.
107
ARTERIAL LINE INSERTION (RADIAL)
INDICATIONS AND CONTRAINDICATIONS MATERIALS NEEDED
Indication: • Sterile gloves
• Continuous intraarterial BP monitoring • Sterile drapes, gown, mask
• Frequent (arterial) blood drawing • Sterile gauze
• Detection of abnormal arterial waveform patterns • Transparent adhesive dressing
• Antiseptic solution
Absolute contraindications:
• Local anesthetic (e.g., lidocaine 1%)
• Distorted anatomy, skin infection, or thrombus at
• lntravascular catheter, 20-gauge
the insertion site
• Needle, 22-gauge
• Severe peripheral vascular disease
• Syringe
• Absent pulse or poor circulation
• Guidewire
Relative contraindications: • Scalpel, blade number II
• Coagulopathy • Adhesive tape or bandage
• Partial or full thickness burn • Splint board
• Active Raynaud's syndrome at insertion site • Ultrasound
• Vascular graft in upper extremity
• Thromboangiitis obliterans

STEPS
1. Examine the patient and review available labs (CBC, blood chemistry, bleeding parameters)
2. Patient is placed in supine position with the arm palm up on a flat surface and wrist
dorsiflexed to 30-45°,supported by a rolled gauze or towel beneath
3. Palpate the radial artery (~1-2cm proximal to the wrist)
4. Observe sterile technique (e.g., sterile gloves, Povidone-iodine prep, and drapes)
5. Aneslhetize the area by making a skin wheal first and penetrating deeper tissues
6. Insen introducer needle at a 30-45°angle toward the palpated pulsation
7. Once a return of blood is seen in the syringe, remove syringe from the introducer needle and
advance a guidewire into the artery. If there is resistance while inserting the guidewire, stop
advancing the guidewire and reposition the needle until the return of pulsatile blood
8. Make a 3-mm skin incision using an 11-bladescalpel at the site of insertion
9. Remove the introducer needle while holding the guidewire in place
10. Advance the catheter over the guidewire into the artery in a clockwise and counterclockwise
manner until a pulsatile blood return is observed
11.Carefully remove the guidewire and attach the appropriate arterial line tubing to the catheter
12.Secure catheter with sterile adhesive bandage or dressing
13.Watch out for untoward reactions (hematoma, hemorrhage, air embolism)

Arterial
line
Pressure transducer and
automatic flushingsystem

Imageontheleftshowsproperpositioning of thehand& arteriallineinsertion.Imageon


therightshowsa simplified
diagramof arterialpressuremonitoringusingtheradialartery
108
BONE MARROW ASPIRATION
INDICATIONS AND CONTRAINDICATIONS I MATERIALS NEEDED

Indications: • Sterile gloves


• For evaluation (i.e., diagnosis, staging, • Sterile drapes, gown, mask
therapeutic monitoring) of hematologic and • Sterile gauze, dressing
lymphoproliferative disorders • Antiseptic solution (e.g., Povidone-
• For assessment of bone marrow cellularity, iodine)
cellular morphology, and maturation • Local anesthetic (e.g., lidocaine 1%)
• Scalpel, blade number 11
Absolute contraindications: • Syringe, 2 mL
• Severe bleeding disorder (e.g., severe • Needle with stylet, 16-or 18-gauge
hemophilia, severe DIC)

Relative contraindication:
• Severe thrombocytopenia (<20,000/µL)
STEPS
1. Examine the patient and review available labs (CBC, blood chemistry, blood smear,
bleeding parameters)
2.The patient should be placed in a lateral decubitus or prone position
3. Identify landmarks - posterior iliac crest and posterior superior iliac spine
4.Observe sterile technique (e.g., sterile gloves, Povidone-iodine prep, and drapes)
5. Anesthetize the area by making a skin wheal first and penetrating deeper tissues
6.Make a 3-mm skin incision using a scalpel approximately three finger-widths from the
midline and two finger-widths inferior to the iliac crest
7. Slowly advance the needle with stylet perpendicularly. Once needle is in contact with the
periosteum, point it laterally in the direction of the anterior superior iliac spine & steadily
rotate in a clockwise and counterclockwise manner until the cortical bone is penetrated
8. Once within the marrow cavity, remove the sty let & attach a 2-mL syringe to the aspiration
needle. Aspirate -0.3 mL of marrow contents. If no fluid comes out, reinsert the sty let &
advance the needle a short distance, then repeat aspiration attempt until successful
9. Reinsert sty let and remove the needle, again in a clockwise and counterclockwise manner
10. Immediately apply pressure on insertion site and cover with sterile occlusive dressing
11.Send specimen for smear analysis (cell count, cell differential, cell morphology)
12.Monitor patient closely and watch out for untoward reactions (hemorrhage)
13.Provide post-procedural analgesics as necessary

Cortical
bone

109
NEEDLE DECOMPRESSION (NEEDLING)
INDICATIONS AND CONTRAINDICATIONS I MATERIALS NEEDED
Indication: • Large-bore needle (Gauge 14 or 16)
• Urgent management of tension pneumothorax • Sterile gloves
• Povidone-iodine
Relative contraindications: • Cotton
• Anticoagulation, coagulopathy and bleeding • Micropore
diathesis

STEPS
1. Wear gloves and prep the site with povidone-iodine
2. Locate the site at the second JCS superior to the third rib at the midclavicular line
3. Hold the needle perpendicular to the chest wall
4. Place the needle just above the cephalad border of the third rib
5. Puncture the site in the middle third to minimize the risk of injury co the internal mammary
artery
6. Listen for a hissing sound once the needle is in the pleural space
7. Remove the needle leaving the cannula in place
8. Cannula is left open to air
9. Secure the catheter in place
IO.Send patient for tube thoracostomy

"X"marks the appropriate site of


puncture during needle decompression

REFERENCES
I. Adams, J.lntravenous and vascular access therapy. In: Clinical Nursing Skills and Techniques. Peny AG and Potter PA (editors).
SL Louis, USA: Mosby, 2006. p.898.
2. American College of Surgeons Committee on Trauma. Advanced Trauma LifeSupport (ATLS) Student Course Manual, 9th ed.
Chicago, USA: American College of Surgeons, 2012.
J. Androes, Mark, and Alan Heffner. "Placement of Jugular Venous Catheters." UpToDate, 28 Aug. 2021, WW\.v.uptodate.com/
4.Ellett, ML What is known about methods of correctly placing gastric tubes in aduhs and children. Gastroenterol Nurs.
2004;27(6),253.
5. Feldman M, Friedman L, and Brandt L (editors). Sleisenger and FordtranS Gastrointestinal and Liver Disease 10th Edition.
Philadelphia: Saunders; 2016.
6. Frank, RL Peripheral venous access in adults. \'/\...W.uptodate.com. Accessed August 8, 2017.
7. Hanson, RL Predictive criteria for length of nasogastric tube insertion for tube feeding. JPEN J Parenter Enteral Nutr. 1979;3(3):16o.
8.Johnson, Kimberly, and Daniel Sexton. "Lumbar Puncture: Technique, Indications, Contraindications, and Complications in
Adults." UpToDate, 2 July 2018, \w,w.uptodate.com/.
9.Jameson JL, Kasper DL. Fauci AS, Hauser SL, Longo DL, and Loscalzo J (editors). Harrison's Principles oflntemaJ Medicine, 20th
ed. McGraw Hill Education, 2018.
10. Lafferty, Keith A, MD. "Rapid Sequence Intubation: Background, Indications, Contraindications." Medscape, 7 Apr. 2020,
http://\'/\'/\v.emedicine.medscape.com/article/8o222.
11. Levitan RM, Mechem CC, Ochroch EA, Shafer FS, and Hollander JE. Head-elevated laryngoscopy position: improving
laryngeal e.xposure during laryngoscopy by increasing head elevation. Ann Emerg Med. 2003~1(3):322.
12. Light RW, Macgregor MI, Luchsinger PC, Ball WC. Pleural effusions: the diagnostic separation oftransudates and exudates.
Ann Intern Med. t972;n(4):507-13.
13. Meddings J, Saint S, Fowler KE, et al. The Ann Arbor Criteria for Appropriate Urinary Catheter Use in Hospitalized Medical
Patients: RAND/UCLAAppropriateness Method. Ann Intern Med. 2ot5 May:162(9Suppl):S1-S34.
14. Roberts, W. Neal. "Joint Aspiration or Injection in Adults: Technique and Indications." UpToDate, 16Apr. 2021,\'/\'/\\'.uptodate.
com/contents/joint-aspiration-or-injection-in-adults-technique-and-indications
15. Runyon BA. The American Association for the Study of Liver Diseases. Practice Guideline: Management of adult patients with
ascites due to cirrhosis: update 2012. Available from: http://w·w,v.aasld.org/sites/default/files/guideline_documents/
16. Schaeffer, A. Placement and management or urinary bladder catheters in adults. www.uptodate.com. Accessed August 8, 2017.
17. Theodore, Anhur, et al. "Intra-Arterial Catheterization for Invasive Monitoring: Indications, Insertion Techniques, and
Interpretation." UpToDate, 15Jan. 2021,\'/\'/\V.uptodate.com/
18. Thomsen 1W, Del.aPena J,and Setnik GS. Videos in clinical medicine. Thoracemesis. N Engl J Med. 2006;355(15):e16.
19. Zehnder, James. "Bone Marrow Aspiration and Biopsy: Indications and Technique." UpToDate, 31Jan. 2020
110
IVFLUIDS
&DRIP
INTRAVENOUS FLUIDS

0 COMMON INTRAVENOUS DRIPS

1. Formulation & Computation of Basic Drips


2. Using Infusion Pumps
3. Overview of Common Drips
4. Sample Cases

0 BLOOD PRODUCTS & TRANSFUSION


1. Rational Use of Blood Products
2. Blood Typing
SECTION ONE
INTRAVENOUS FLUIDS
INTRAVENOUS FLUIDS AND COMMON INDICATIONS
I. BASIC TYPES OF INTRAVENOUS FLUIDS (!VF)
IV FLUIDS I DESCRIPTIONS I EXAMPLES
• Normal saline (0.9% NaCl)
• Balanced salt/electrolyte solutions
• Lactated Ringer's
Crystalloids which may be isotonic, hypertonic
• Hypertonic saline (3%, 5%, 7.5%)
or hypotonic
• Ringer's solution
• High-molecular weight solutions •Albumin
which draw fluid into intravascular • Hetastarch, pentastarch
Colloids
compartment via oncotic pressure • Plasma
• Effective plasma expanders • Dextran
• Provide water that is not bound by • D5W (5% dextrose in water),
Free H20
macromolecules or organelles, thus DwW, D20W, D50W, D50-50,
solutions
is free to pass through membranes dextrose/crystalloid mixes
• Whole blood, pRBC,
Blood
• Essentially considered colloids FFP, platelet concentrate,
products
cryoprecipitate

II. COMPOSITION OF INTRAVENOUS FLUIDS

• Useful in dehydrated states & hypernatremia


D5W 50
• Can be used as diluent
D10W 100 • Used to correct hypoglycemia
0.9 • Fluid of choice for initial resuscitation
154 154
NSS • Can be used as diluent
D50.9 • Same as 0.9 NSS but with glucose
50 154 154
NSS • Useful for patients on NPO
LR 130 109 4 3 28
• Useful for traun1a, surgery, & burn patients
NR 140 98 5
• Routine fluid & electrolyte maintenance
D5NM 50 40 40 13
with minimal carbohydrate calories
D5NMK 50 40 40 30 • Same as D5NM, but with additional K•
Ionsarein milliequivalents
or mEqs Glc:Glucose(ing/L)
Source:SunJoanBai,et al. FluidTherapy.
J KoreanMedAssoc.2010
lngelfinger,
J, et al. Maintenance
IntravenousFluidsinAcutelyIll Patients.
NEJM;2015

III. COMMON INDICATIONS FOR INTRAVENOUS FLUID ADMINISTRATION


Maintain normal BP: normal or isotonic saline is the initial fluid of choice (0.9 NSS)
Return the intracellular fluid (!CF) volume to normal
0 In acute hyponatremia, the !CF volume in the brain rises (i.e., cerebral edema) &
may become dangerously high, so hypertonic saline is given to raise plasma Na·
0 When there is a large water deficit in !CF compartment (e.g., severe hypernatremia
leading to cellular shrinkage/dehydration), electrolyte-free water (D5W) is given
Replace ongoing renal losses
Give maintenance fluids to match insensible losses
Provide glucose as fuel substrate for the brain: dextrose-containing fluids for patients on NPO
113
IV. COMMONLY USED INTRA VENOUS FLUIDS
A. Normal Saline (0.9% NaCl, pNSS)
0 Least expensive and most used resuscitative crystalloid
0The only solution that may be administered with blood products
0Does not provide free water or calories
B. Lactated Ringer's Solution (LR)
0Lactate is converted readily to bicarbonate in the liver
0Minimal effects on normal body fluid composition and pH
0Most closely resembles the electrolyte composition of normal blood serum
0Does not provide calories
C. D5W or ¼ Normal Saline
0Provides 170calories/L from 5% dextrose
0Provides free water for insensible losses & some Na· to promote renal function & excretion
With added K·, this is an excellent maintenance fluid in the immediate postoperative period
0

0Prevents excess catabolism and limits proteolysis


D. Hypertonic Saline (3% NaCl)
0!026 mOsm/L, 513mEq/L Na•
0Increases plasma osmolality & thereby acts as a plasma expander, increasing circulatory
volume via movement of intracellular & interstitial water into intravascular space
0Due to high sodium content, poses risk for hypernatremia

V. COMMONLY USED TERMS IN FLUID THERAPY


TERM I DEFINITION

Fluid • Daily sum of all fluid intake and output


balance • Cumulative fluid balance: summation of all daily balances over a specified time

• >IO%of fluid accumulation


Fluid
overload • Fluid Accumulation = Cumulative fluid balance in liters XIOO
Baseline body weight in kg

• A rapid fluid infusion given to correct hypotension and hypovolemia


Fluid
• Typically, at least 4 mL/kg infusion given over a maximum of 10·15minutes
bolus
• For hypotensive& septic patients, 30 mUkg infusion over 1·2hours is recommended
Fluid • A bolus ofmo-200 mL given over 10-15minutes with reassessment of
challenge hemodynamic status to optimize tissue perfusion

VI. GUIDE FOR FLUID THERAPY


Selection ofIV fluids is guided by physiologic principles
• Rational fluid use must be viewed through the 4 D's: Drug, Dosing, Duration, & De-escalation
TERM I DEFINITION

• Fluids must be considered as drugs, with indications, contraindications,


Drug
and potential side effects
• Requirements for fluid resuscitation and the patient's response may vary
greatly during an acute illness, not only in terms of the dose, but also the
Dosing
timing and route of administration
• Pharmacokinetic/pharmacodynamic principles should be considered
• Fluids must be tapered when the initial acute problem resolves
Duration • Potential for fluid overload with adverse effects can ensue when the
clinician is not aware of the need to stop
De-
• Consider withholding/withdrawing fluids when they are no longer required
escalation
114
mL/hr ugtt/min divided by 4 is equal to gtt/min
(drops/min)

Overview of Steps to Compute for JV Drip Rate (when desired dose and drip preparation are given):

Determine the drug concentration of the medication (as prepared in the drip).
I This is computed by dividing the dose of the drug by the volume of the diluent
(preferably in mL or cc).

Convert the computed concentration, as necessary (e.g., mg to mcg if the given


2 desired dose is in mcg). This is done to simplify the computation of the drip rate (see
formula and examples below).

Compute the drip rate using the formula below. Before this, first identify four
components of the equation:
Desired dose of the drug
0

Patient's weight
0

Number of minutes in an hour


0

0 Drug concentration

General Formula:
Desired Dose x Body weight (kg) x 60 (mins/hr)
Drip Rate (mL/hr) =
Drug Concentration

Formula if the desired dose of the drug is given in mcglkglmin (e.g., dopamine drip)
Drip Rate (mL/hr) = Desired Dose (mcg/kg/min)x Body weight (kg) x 60 (mins/hr)
3 Drug Concentration (mcg/mL)

Formula if the desired dose of the drug is given in mcg/min (e.g., nitroglycerin drip). Since
the drug is not weight-based, the "body weight" may be omitted from the equation.
Desired Dose (mcg/min) x 60 (mins/hr)
Drip Rate (mL/hr) =
Drug Concentration (mcg/mL)

Formula if the desired dose of the drug is given in mg/hr (e.g., nicardipine drip). Since the
dose is given in mg/hr, we do not need to multiply by 60 minutes. Note also that the drug
concentration is in mg/mL (i.e., no need to perform step #2 above).

Drip Rate (mL/hr) = Desired Dose (mg/hr)


Drug Concentration (mg/mL)

To Compute for tile Dose being given to a Patient (w/1en drip rate and preparation are given):

• When reporting/endorsing a case, it is better to state the dose of the drug that the
patient is being given and not the drip rate.
• To compute for the specific dose being delivered, use the following formula:

Dose of Drug Being Given (mcg/kg/min) =


Present drip rate (mL/hr) x Drug Concentration (mcg/mL)
Body weight (kg) x 60 (mins/hr)

115
USING INFUSION PUMPS
Intravenous (IV) "smart" infusion pump systems have received widespread use due to
improved safety from medication errors
Particularly important in the ICU where multiple JV drugs are part of usual care

I. GENERAL CONCEPTS WITH INFUSION PUMPS


Infusion pumps ensure that accurate drip rates with the appropriate dose of
medications or fluids are delivered to the patient
Some infusion pumps offer drip rate settings only: you need to compute for the drip rate
based on the concentration of the drug, weight of patient & desired dose (seepreviousdiscussion)
Smart infusion pumps: programmable & can calculate drip rate by encoding patient's
weight, concentration of the medication in the diluent & the desired dose to be delivered
In adjusting the dose, you can directly adjust drip rates more accurately based on desired dose

General Elements in Infusion Pumv Use


• Selecting the drug
• Selecting a dose
• Selecting an infusion rate
• Calculating and ordering the infusion
• Programming the infusion pump
• Delivering the infusion

II. GENERAL TYPES OF INFUSION PUMPS


LARGE VOLUMETRIC PUMPS I SYRINGE PUMP
Syringe plunger driver
Moves forward in line with the chosen flow rate,
but only pushes the plunger if patient is higher
than the pump or if there is a flow restriction

Roller clamp
Shut when loading
the set into the
pump, otherwise
mostly left open

CJ
Syringe gripper
Holds the syringe plunger
LVP back to control flow
Controls the
Syringe clamp
infusion flow rate
Holds the syringe body in
by slowing the flow
place

• Pumps that infuse large volumes of • Infusion pumps that deliver low
nourishment or medication fluid (using medication volumes for medication
large volume pumps or LVP) required in small quantities
• Positioned above the patient's head • Holds the syringe plunger in place, not
• Free flow can be controlled by allowing it to siphon
mechanisms such as 'peristaltic fingers'
Source:OhashiK, et al. DrugSat;2014
ApkonM,et al. QualSatHealthCare;2004

116
OVERVIEW OF COMMON DRIPS
The following tabulated drugs are the commonly used drips in clinical practice. These doses
are not all-encompassing. Please refer to the drug inserts/pharmacology textbooks for full
prescription details. Some of the drips will be discussed in detail in the next section.

I. VASOPRESSORS AND INOTROPES


DOSE I PREPARATION'I TITRATION/NOTES
Dopamine
• <2 mcg/kg/min: selective dilation of renal,
• Usually started at
splanchnic, and cerebral arteries
2-5 mcg/kg/min
• 2-10 mcg/kg/min: inotropic dose • 400 mg+
• Dose is increased
• >IO mcg/kg/min: increase in systemic 25omLD5Wb
every 2-5 minutes
vascular resistance (SVR)
• Maximum dose: 20-50 mcg/kg/min

Dobutamine
• 2.5 to 20 mcg/kg/min • Increases stroke
• Minimal positive chronotropic activity at 0 5oomg+ volume & decreases
low doses (2.5 mcg/kg/min) & moderate 25omL SVR with net BP effect
positive chronotropic activity at high doses D5W' being unchanged or
• Maximum dose: 20 mcg/kg/min slightly increased

Norepinephrine

• 2 to 4 mcg/min (or 0.05 too.IO mcg/kg/min) • IfSBP cannot be


• Non-weight-based dosing (based on an maintained at >90
~80-kg patient): •4mg+ mm Hg with a dose
0 Initial: 5 to 15mcg/min; titrate to goal MAP 25omL of 15 mcg/min, it is
0 Usual range: 2 to 80 mcg/min D5Wd unlikely that further
0 Maximum dose: 80-250 mcg/min (in increases in dose will
refractory shock) be of benefit

Vasopressin
• Used to reduce dose
of catecholamines to
• 0.01 to 0.1 units/min • IOOunits+
minimize side effects
• Maximum dose: 25omLD5W
• Adverse events
0 0.06 units/min for septic shock (max of 20
from excessive
0 0.1 units/min for post-cardiotomy units in 50
vasoconstriction are
vasoplegia mLdiluent)
uncommon at a dose
of <0.04 units/min
'More concentrated dose chosen for patients who cannot tolerate higher fluidvolumes(e.g., CHF,CKD)
'Dopamine: commonpreparationis 200 mg per ampule.Some are already premixedas 400 mg in 250cc 05W
'Dobutamine: common preparationis 250 mg per ampule. Some already premixedas 500 mg in 250cc 05W
'Norepinephrine:common preparation is 4 mg per ampule. Some preparations contain 10 mg per ampule
Source:Manno,PL,et al. WoltersKluwerHealth/Lippincott Williams& Wilkins;2014
JamesonJ, et al. Harrison'sPrinciplesof InternalMedicine,20thEdition;2018

II. LOOP DIURETICS (titration depends on diuretic response & fluid status)
DRUG I DOSE I PREPARATION
• Bolus: 20-40 mg SIVP over 1-2 min • Drip: IOOmg+
Furosemide
• Continuous infusion: I-IO mg/hr IOOmL diluent

• Bolus: 0.5-1 mg SIVP over 1-2 min • Drip: 2.5 mg+


Bumetanide
• Continuous infusion: 0.5-2 mg/hr (start at I mg/hr) 90 mL diluent

117
III. VASODILATORS OR ANTI-HYPERTENSIVES
DRUG I DOSE I PREPARATION I TITRATION
• Initial: 5 mg/hr • IO mg+ enough • Titrate by 2.5 mg/hr
Nicardipine (e.g., HPN emergency) diluent to 915minutes until target
• Maximum: 15mg/hr make 100 mL BP is reached
Nitroglycerin • Titrate by 5 mcg/min
(NTG)or • IO mg+ 95 mins until pain
• Initial: 5 mcg/min
glyceryl 9omLD5W relief achieved or BP is
trinitrate controlled
• Infusion: 2 mg/min • For HPN emergency,
• Repeated IV bolus: 20 • 200 mg+ initial goal is to reduce
mg over 2 min, then enough PNSS MAP by no more than
Labetalol
40-80 mg at IO-min to make 25% within minutes to
intervals up to 300 mg 2oomL 2 hours or to a range of
total (max dose) 160/100-110mmHg
Isosorbide • Titrate by I mg/hr every
• IO mg+
dinitrate • Infusion: 1-5mg/hr 15-30minutes until chest
90 mL diluent
(ISDN) pain-free (ifBP is normal)

IV. ANTI-ARRHYTHMICS
DRUG I DOSE I PREPARATION I TITRATION/NOTES

• Bolus: 150-300 mg
• Sample order for VT:
slow IV push (SIVP),
give 150mg SIVP x IO
then
• 150-600 mg+ mins, then start drip -
• Continuous infusion:
250-500 mL 900 mg+ 500 mL D5W
Amiodarone I mg/min for 6 hrs,
D5Wx x 33 cc/hr (1mg/min) x
then 0.5 mg/min
16-24hours 6 hours, then decrease
infusion
infusion rate to 17cc/hr
• ~I g of amiodarone is
(0.5 mg/min) x 18hours
given within 24 hours
• Rate of infusion should
be reassessed as soon as
the patient's basic cardiac
• Bolus: 1-1.5mg/kg
• 1-2grams+ rhythm appears to be
SIVP, then
Lidocaine 500 mL diluent stable, or at the earliest
• Infusion: 1-4 mg/min
(pNSS or D5W) signs of toxicity
• It is rarely necessary to
'
continue IV lidocaine for
prolonged periods
• Bolus: 0.5 mg/kg SIVP • Titrate by 50 mcg/kg/
over I min, then • 2,500 mg+ min increments every 4
Esmolol
• Infusion: 50-200 mcg/ 250 mL diluent mins until desired HR
kg/min achieved
Source:JamesonJ, etal. Harrison's
Principles
of InternalMedicine,
20thEdition;2018
ZipesDP,et al. Braunwald's HeartDisease,11thEdition;2019

118
V. ANTITHROMBOTICS
DRUG I DOSE I PREPARATION I TITRATION

• PTT monitored
every 6 hrs (after
• For ACS: 60 units/kg IV
• 10,000 units in a dose change)
push as loading dose (LD),
Unfractionated enough pNSS & drip adjusted
then drip at 12units/kg/hr
heparin to make 100 mL accordingly to
, For VTE: 80 units/kg IV
(UFH) (concentration of reach target PTT
push as LD, then drip at 18
100 units/mL) of 1.5-2.3times
units/kg/hr
the control (or
46-70 sec)
• i.5 million units+

I
• For STEMI: 1.5million
Streptokinase 5occpNSSx
units IV for I hour
1hour

• For acute ischemic stroke:


0.9 mg/kg (not to exceed
90 mg), 10% of total • Review
treatment dose as bolus for • Alteplase indications &
I minute, the rest for I hour 50 mg+ IOOmL connraindications
Alteplase • For STEM!: bolus of15 mg, preservative-free to fibrinolysis
then infusion ofo.75 mg/ sterile water for
kg for 30 mins (max 50 mg), injection
then 0.5 mg/kg (max 35 mg)
over the next 60 mins (total
dose not to exceed 100 mg)
-
•Fora 60-kg
• Tirofiban patient: give 1500
• Adjunctive therapy for
12.;;mg/50 mL mcg (30 mL) iV
repetfusion with primary
vial (250 mcg/ bolus (25 mcg/
percutaneous coronary
mL) reconstitute kg), then.start
Tirofiban intervention (PCI) for
with5omL drip to run at II
STEMI: 25-mcg/kg IV
sterile water cc/hr (0.15mcg/
bolus, then 0.15 mcg/kg/
then add to 200 kg/min) x 18 hrs
min for up to 18 hours
mLD5W I'-• Reduce dose by
50% for CrCl <60
Source:JamesonJ, et al. Harrison's
Principles
of InternalMedicine,
20thEdition;2018

VI. DRIPS FOR GASTROINTESTINAL BLEEDING


DRUG I DOSE I PREPARATION I TITRATION
• In emergency
• Bolus: 50 mcg IV, then • 600 mcg+ situations, may
Octreotide
• Infusion: 25 to 50 mcg/hr 25omLD5Wx be given as IV
(up to 5 days) 25 mcg/hr bolus (undiluted)
over 3 to 5 mins

• Bolus: 250 mcg IV, then • 3mg+25omL


Somatostatin • Maintain the drip
• Infusion: 250 mc~/hr D5Wx 12hours
rate continuously
Omeprazole • Bolus: 80 mg IV, then • 4omg+9omL for72 hours
Pantoprazole • Infusion: 8 mg/hr pNSS x 5 hours

119
VII. SEDATION
DRUG I DOSE IPREPARATION I TITRATION/NOTES
• LD: 1 to 2 mcg/kg
• Less hypotension than other
(25 to 100 mcg) • 1,ooomcg+
opioids due to relative lack of
• Maintenance dose 1oomLpNSS,
histamine release
Fentanyl (MD): 0.7 to 10 or
• Hemodynamic effect: decreases
mcg/kg/hr infusion • 2,500 mcg +
HR, decreases SVR, may decrease
(50-700 mcg/hr) 25omL PNSS
BP, no effect on contractility
• Initial: I mcg/kg/hr
• Titrate every 5-10 minutes in
• 2oomg+ increments of 5 to IO mcg/kg/min
enoughPNSS • Dose >70 mcg/kg/min increases
Propofol • 5 to 50 mcg/kg/min
to make risk of propofol infusion syndrome
IOomL • Hemodynamic effect: decreases
HR, SVR, BP, & contractility
• 4oomcg • Titrate in 0.1 mcg/kg/hr increments
• LD: 1mcg/kg over (2 vials of 2 mL to desired level of sedation
Dexme- IO minutes, then of IOO mcg/mL (max dose: 0.7 mcg/kg/hr)
detomidine • MD: 0.2-0.7 mcg/ solution)+ • LD is optional for stable patients
kg/hr enough PNSS • Hemodynamic effect: decreases HR,
to make 100mL decreases SVR, may decrease BP
• Titrate by I mg/hr based on sedation
(titrate to lowest dose possible)
• LD: 0.01 to 0.05 • Half-life may be prolonged with
mg/kg (0.5-4 mg) • 5omg+ liver & kidney injury
Midazolam • MD: 0.02 to 0.1 enoughPNSS • Good choice for short-term
mg/kg/hr infusion to make 100mL treatment of agitation
(2 to 8 mg/hr) • Hemodynamic effects: increases
HR, decreases SVR, may decrease
BP, no effect on contractility
• Bolus: 2-4 mg SIVP • 16mg+ • Depends on type of sedation desired
Morphine
• Infusion: 1-2mg/hr 50 mLpNSS • IV boluses may be given for chest pain

VIII. OTHER ESSENTIAL DRIPS


DRUG I DOSE I PREPARATION I TITRATION/NOTES
• Caution with high-dose steroids
• High-dose (e.g., • 500-1000 mg+
Methyl- • Monitor vital signs every 15
severe SLE, such as 250 mL pNSS
prednisolone mins for the first 2 hours on the
myocarditis, nephritis): x 5 hours (50
drip (monitor for arrhythmias,
15-30mg/kg/day cc/hr) x 3 days
hypertensive emergency)
• Infusion: 60-90 mg • 60-9omgin
• Single dose usually given every
Pamidronate single dose IV infusion 25omLpNSS
3-4 weeks
x 2-24 hrs x 2-24 hours
• For osteoporosis: 5 mg
IV infusion over no less
than 15min, once a year
• For osteoporosis • For osteoporosis: an average
prevention: 5 mg IV • 5 mg in 100 mL of at least 1200 mg calcium &
Zoledronic
infusion over no less D5Wx15-min 800-1000 IU Vitamin D daily
acid
than 15min, every 2 yrs infusion is recommended together with
• Hypercalcemia of zoledronic acid
malignancy: single dose
of 4 mg IV infusion
over no less than 15min
120
SAMPLE CASES
Case 1. You want to start dopamine drip at a desired dose of5 mcg/kg/min in a 65-kg patient.
The dopamine drip was formulated as 400 mg dopamine in 250 mL of D5W. What will be the
IV drip rate?
Determine the drug concentration (as prepared in the drip). This is computed by
dividing the dose of the drug by the volume of the diluent (preferably in mL or cc).
400 mg dopamine . .
Drug Concentration (mg/mL) = L f fl .d = 1.6 mg dopamme per mL of fluid
250 m O Ul

Convert the computed concentration to simplify the equation. Since the desired dose is
2
given in mcg/kg/min, convert the concentration (mg/mL) to mcg/mL to simplify.
1.6 mg dopamine permL of fluid= 1600 mcg dopamine per mL of fluid

Compute drip rate using the formula below (sincethe desireddose is given in mcg!kg!min)

Desired Dose (mcg/kg/min) x Body weight (kg) x 60 (mins/hr)


Drip Rate (mL/hr) =
Drug Concentration (mcg/mL)

5 (mcg/kg/min) x 65 (kg) x 60 (min/hr) = 12 mL/hr


16oomcg/mL

Case 2. You want to start amiodarone drip at a desired dose of 1.0mg/min in a patient with acute atrial
fibrillation. The amiodarone drip was formulated at 300 mg+ 250 mL D5W. What will be the JV rate?
Determine the drug concentration of the medication (as prepared in the drip).
I 300 mg amiodarone
Drug Concentration (mg/mL) 1.2mg amiodarone per mL of fluid
250 mL offluid

Since the desired dose is given in "mg/min" and the computed drug concentration in
2
step #I is already in "mg/mL," there is no need to convert into "mcg."

Compute the drip rate using the formula below since the desired dose is given in
mg/min. Since the drug dosing is not weight-based, the "body weight" may be omitted
from the equation.
Desired Dose (mg/min) x 60 (mins/hr)
3 Drip Rate (mL/hr) =
Drug Concentration (mg/mL)
1.0 (mg/min) x 60 (mins/hr)
= =50 mL/hr
1.2mg/mL

Case 3. You want to start omeprazole drip at a desired dose of 8 mg/hr. Your formulation of
omeprazole drip is 40 mg in JOO mL fluid. What will be the IV drip rate?
Determine the drug concentration of the medication (as prepared in the drip).
I 40 mg omeprazole
Drug Concentration (mg/mL) = =0.4mg/mL
JOO mL of fluid

Since the desired dose is given in "mg/hr" and the computed drug concentration in ~tep
2
#I is already in "mg/mL," there is no need to make unit conversions.

Compute the drip rate using the formula below since the desired dose is given in mg/hr.
Since the dose is given in mg/hr, we do not need to multiply by 60 minutes.

3 Desired Dose (mg/hr) 8 (mg/hr)


Drip Rate (mL/hr) =
o-4(mg/mL)
= 20 mL/hr
Drug Concentration (mg/mL)

121
Case 4. You want to start vasopressin drip at a desired dose of 0.03 unit/min. Your formulation
ofvasopressin drip is 100 units in 250 mL fluid. What will be the IV drip rate?
Determine the drug concentration of the medication (as prepared in the drip).
I
Drug Concentration (units/mL) = 100 units vasopressin = 0.4 units/mL
250mL offluid

Since the desired dose is given in "units/min" and the computed drug concentration in
2 step #I is in "units/mL," there is no need to make conversions.

Compute the drip rate using the formula below since the desired dose is given in units/min.
Since the drug is not weight-based, the "body weight" may be omitted from the equation.
Desired Dose (units/min) x 60 (mins/hr)
Drip Rate (mL/hr) =
3 Drug Concentration (units/mL)
0.03 (units/min) x 60 (mins/hr)
= 4.5 mL/hr
0.4 (units/mL)

Case 5. You want to start lidocaine drip at a desired dose of I mg/min. Your formulation of
lidocaine drip is 2 grams in 500 mL fluid. What will be the IV drip rate?
Determine the drug concentration of the medication (as prepared in the drip).
I 2 g lidocaine
Drug Concentration (grams/mL) = = 0.004 grams/mL
500 mL of fluid

Convert the computed concentration, as necessary. Since the desired dose is given in
"mg/min" and the computed drug concentration in step #I is in "grams/mL," there is a
2 need to convert to simplify.
0.004 grams oflidocaine per mL offluid = 4 mg per mL offluid

Compute the drip rate using the formula below since the desired dose is given in mg/min.
Since the drug is not weight-based, the "body weight" may be omitted from the equation
Desired Dose (mg/min) x 60 (mins/hr)
Drip Rate (mL/hr) =
3 Drug Concentration (mg/mL)

1(mg/min) x 60 (mins/hr)
= = 15mL/hr
4(mg/mL)

Case 6. Your 65-kg patient intraoperatively is on esmolol drip (formulated at 2,500 mg+ 250
mL fluid) at 15mL/hr. What is the dose of esmolol being given to the patient in "mcg/kg/min?"

Determine the drug concentration of the medication (as prepared in the drip).
I 2500 mg esmolol
Drug Concentration (mg/mL) = IO mg/mL
250 mL of fluid

Convert the computed concentration, as necessary. Since the question is to get the dose at
"mcg/kg/min" and the computed drug concentration in step #I is in "mg/mL," we must convert
2
IO mg/mL of esmolol is equal to 10,000 mcg/mL

To compute for the specific dose being delivered, use the following formula:

Dose being given= Present drip rate (mL/hr) x Drug Concentration (mcg/mL)
3 (mcg/kg/min) Body weight (kg) x 60 (mins/hr)

= 15(mL/hr) x 10,000 (mcg/mL) = 38.5 mcg/kg/min


65 (kg) x 60 (mins/hr)

122
Case 7. Your 60-kg intubated patient is still on light sedation despite midazolam drip, hence
the ICU team decided to start on fentanyl drip to attain moderate sedation and wants to
start at I mcg/kg/hour. What will be the IV drip rate?
Determine the drug concentration of the medication (as prepared in the drip). If your
formulation of fentanyl drip is I mg in 100 mL of fluid:
I
1 mgfentanyl
Drug Concentration (mg/mL) = = 0.01 mg fentanyl per mL of fluid
100 mL of fluid

Convert the computed concentration to simplify the equation. Since the desired dose is
given in mcg/kg/hour, convert the concentration (mg/mL) to mcg/mL to simplify.
2
0.01 mg fentanyl per mL of fluid= 10 mcg fentanyl per mL of fluid

Compute the drip rate using the formula below, since the desired dose is given in mcg/kg/hr
Desired Dose (mcg/kg/hour) x Body weight (kg)
Drip Rate (mL/hr) =
Drug Concentration (mcg/mL)
3
I (mcg/kg/hour) x 60 (kg)
= =6 mL/hr
10(mcg/mL)

Case 8. Your 70-kg post-bypass patient is for delayed extubation and the ICU team decided
to continue se d ation wit h propo io I at 5 mcg,/k:g/minute. W h atwi II b et h e IV d rip rate?
Determine the drug concentration of the medication (as prepared in the drip). If your
formulation of propofol drip is 200 mg in 100 mL of fluid:
I
200 mg propofol
Drug Concentration (mg/mL) = = 2 mg propofol per mL of fluid
100 mL of fluid

Convert the computed drug concentration to simplify the equation. Since the desired
dose is given in mcg/kg/min, convert the concentration (mg/mL) to mcg/mL to simplify.
2
2 mg propofol per mL of fluid = 2000 mcg propofol per mL of fluid
Compute drip rate using the formula below, since the desired dose is given in mcg/kg/min
Desired Dose (mcg/kg/min) x Body weight (kg) x 60 (mins/hr)
Drip Rate (mL/hr) =
3 Drug Concentration (mcg/mL)
5 (mcg/kg/min) x 70 (kg) x 60 (mins/hr)
= =I0.5 mL/hr
2000 (mcg/mL)

Case 9: Your 70-kg recently intubated patient just woke up and remains agitated despite
midazolam bolus. The ICU team decided to start sedation with midazolam drip at
0.02 mg/kg/hour. What will be the IV drip rate?
Determine the drug concentration of the medication (as prepared in the drip). If your
formulation of midazolam drip is 50 mg in 100 mL fluid:
I
Drug Concentration (mg/mL) = 50 mg midazolam = 0.5 mg midazolam per mL of fluid
100 mL offluid

2 No need to convert computed concentration.

Compute drip rate using the formula below, since the desired dose is given in mg/kg/hr
Desired Dose (mg/kg/hour) x Body weight (kg)
Drip Rate (mL/hr) =
3 Drug Concentration (mg/mL)
0.02 (mg/kg/hour) x 70 (kg)
= =2,8mL/hr
o.5(mg/mL)
123
SECTION THREE
BLOOD PRODUCTS AND TRANSFUSION
RATIONAL USE OF BLOOD PRODUCTS
WholeBlood
I

PackedRedBlood Cell Platelet-richPlasma


I

FreshFrozenPlasma Platelets
I
't 't
Cryoprecipitate Cryosupernate
• Fibrinogen • FactorII
• FactorVIII • FactorVII
• FactorXIII • FactorIX
• VonWillebrandFactor • FactorX
• SomeFactorXIII
• SomeVonWillebrandFactor

I. PACKED RED BLOOD CELL (pRBC) TRANSFUSION


• Increases O2-carrying capacity in the anemic patient
Initially transfused slowly for the first 15mins, then is completed within 4 hours
ABO group ofRBC products must be compatible with the ABO group of recipient & RBC
product must be compatible with the recipient (exceptions are made in emergencies)
In average-sized adults, one unit pRBC increases hemoglobin by I g/dL or hematocrit by 3%
The criteria for pRBC transfusion should be based on hemoglobin level, the patient's
clinical condition and the risk of inadequate oxygenation
A. Transfusion Thresholds for Different Clinical Situations
CLINICAL SITUATION I POTENTIAL TRANSFUSION THRESHOLD
Adult inpatients
(hemodynamically stable)
• Hemoglobin (Hgb) s.7g/dL
ICU patients
(hemodynamically stable)
Postoperative orthopedic • Hgb <8 g/dL or for symptoms (e.g., chest pain,
or cardiac surgery patients• orthostatic hypotension, CHF, tachycardia
Cardiovascular disease unresponsive to fluids)
• The AABB does not recommend for or against a liberal
Acute coronary syndrome
or restrictive RBC transfusion strategy
All patients • Guided by symptoms as well as by hemoglobin level"
'Not generalizableto preoperativesetting
'*Weakstrength of recommendation

B. Preparation and Monitoring


0PRBC is available in bags of150-200 mL from which most of the plasma has been removed
0Each bag contains -20 g/100 mL hemoglobin and 55-75%hematocrit
0Each bag is usually transfused in 2-3 hours, but a slower rate of 10-15drops/min should
be done for the first 10 minutes
0In those who are not actively bleeding, repeat hemoglobin determination may be done
15minutes after transfusion

Source:
Philippine
CPGfortheRational Useof BloodandBloodProducts
andStrategies
forImplementation;
2009
CarsonJL,et al. CPGfromtheAmerican Association
of BloodBanks(AABB).JAMA.2016
124
C. Blood Transfusion Reactions•
TYPE I REMARKS
• Mild: mild allergic or urticaria! reactions
• Moderately-severe: moderate-severe urticaria!, febrile non-hemolytic
Acute
reactions, possible bacterial contamination
reaction
• Life-threatening: acute intravascular hemolysis, septic shock, fluid overload,
anaphylactic reaction, transfusion-associated lung injury (TRALI)
• Transfusion-transmitted infections
Delayed
• Others: delayed hemolytic reactions, post-transfusion purpura, graft-
reaction
versus-host disease, iron overload in repeated transfusion
*Routineuseof paracetamol or diphenhydramineas pre-medications
for bloodtransfusion
is not
recommended. Instead,leukocyte-depleted
productsmaybe usedfor thosewithprevioushistory
of febrilenon-hemolytic
Source:
Clinical
transfusionreactions.
UseofBlood1nMedicine,
Obstetrics,
Ped.itrics,
Surgery
&Anesthesia,
Trauma
CPGforthe RationalUseof Blood& BloodProducts& Strategies
Philippine
andBums;
WHO
for Implementation;
2009
200
Leukocyte • Decreases risk of recurrent febrile, non-hemolytic transfusion reactions
reduction/ • Decreases risk of CMV transmission
depletion • Decreases risk of HLA-alloimmunization
• Removal of residual plasma
• Decreasesrisk of anaphylaxisin IgA-delicientpatients with anti-JgAantibodies
Washing
• Decreases reactions in patients with history of recurrent, severe
allergic, or anaphylactoid reactions to blood product transfusion
• Prevention of transfusion-associated graft versus host disease (TA-GVHD)
Irradiation • Used for those with history of use of several medications (e.g.,fludarabine,
cladribine, clofarabine, bendamustine, alemtuzumab, etc.)or transplantation

II. PLATELETTRANSFUSION
• Cross-matching not required but ideally should be ABO type-specific
• Premedication is not necessary
A. General Indications•
Based on Philippine CPG on Rational Use of Blood and Blood Products

• Ongoing massive bleeding to maintain platelet count >50,ooo/mm', if with CNS


trauma or bleeding maintain platelet count >IOO,ooo/mm'
• Massive blood transfusion and with platelet count <20,ooo/mm'
• Episodes of hemorrhage or during times of active treatment in chronic, stable, severe
thrombocytopenia such as aplastic anemia and myelodysplasia
• Persistent mucosa! bleeding in patients with hemolytic disorders
• Adults receiving therapy for acute leukemia at threshold of10,ooo/mm'
• Those with qualitative platelet dysfunction with bleeding or who will be undergoing surgery
Additional Indications from the American Association of Blood Banks (AABB)

• Reduction of risk for spontaneous bleeding in hospitalized patients with therapy-induced


hypoproliferative thrombocytopenia (i.e., transfuse for platelet counts <IO,ooo/mm')
• Prophylaxis for elective central venous catheter placement with platelet <20,000/mm'
• Prophylaxis for elective diagnostic lumbar puncture with a platelet <50,000/mm'
• Prophylaxis for major elective non-neuraxial surgery with a platelet <50,ooo/mm'
• For patients having bypass who exhibit perioperative bleeding with
thrombocytopenia and/or evidence of platelet dysfunction
*Plateletsaregenerallynotoseful'inthefollowing(thrombocytopeniais dueto increasedplatelet
destruction):
drug-induced
thrombocytopenia (e.g.,heparin-induced
thrombocytopenia),
TTP,HUS,ITP
125
B. Preparation and Monitoring
I SINGLE DONOR PLATELET (SDP)
I
RANDOM DONOR
PLATELETS (RDP) OR PLATELET PHERESIS

• Obtained using automated


• Platelets derived from whole instrumentation
blood within 8 hours of blood • Contains 2c3.ox10"per bag in
donation -250 mL of plasma
Description • Contains 2'5.5x 10'0 platelets per • Generally preferred, especially those
bag in -50 mL of plasma on repeated platelet transfusions
• Indications for RDP are the • Advantage: may reduce risk of
same as for SOP infectious disease transmission by
reducing number of donor exposures
• I unit (-50 mL) per 10 kg body • 1 pack(-200-600 cc, equivalent to
Dose
weight 4-8 units ofRDP)
• Expect an adult platelet count
• 1unit increases platelets by
Response increment of -30,000-60,000/mm'
5,000-10,000 cells/uL
for each SDP given
Source:Philippine
CPGfortheRational
Useof Blood& BloodProducts
andStrategies
for Implementation;
2009

III. FRESH FROZEN PLASMA (FFP) TRANSFUSION


Contains coagulation factors and plasma proteins including fibrinogen, antithrombin,
albumin, and proteins C and S
Must be ABO-compatible with patient

A. General Indications:
Based 011 Philippine CPG on Rational Use of Blood and Blood Products•

• Multiple coagulation factor deficiencies with severe bleeding or disseminated


intravascular coagulation with bleeding
• Single coagulation factor deficiencies when no virus-safe fractionated product is available
• Severe bleeding due to warfarin or patients taking warfarin who will undergo
emergency surgical procedure (should not be given for reversal ofwarfarin
anticoagulation in the absence of severe bleeding)
• Trauma casualties with 2,30%blood loss & who will be requiring massive transfusion
*Thefollowingcanbe usedasthresholds atwhichtherapeuticor prophylactic
replacement
maybe
indicatedin an appropriate clinicalsetting(mustcorrelate
clinically):
0Prothrombin time(PT)>1.5timesthemid-range of normal
0Activated partialthromboplastin time(aPTT)>1.5timesthetopof thenormalrange
° Factorassay<25%
Source:Ph1hpp1ne
CPGfortheRationalUseof BloodandBloodProducts
andStrategies
for Implementation;
2009

B. Preparation and Monitoring


0 Volume of each unit is -250 mL (may vary)
0 Usually given at 15-20mL/kg or 4-7 units for an average-sized adult (increases
coagulation factors by -20-25%)

126
JV.OTHERS
PRODUCT I REMARKS I INDICATIONS

• Contains fibrinogen, factor VIII, • Fibrinogen deficiency when


factor XIII, and von Willebrand there is clinical bleeding, an
factor (labile clotting factors) invasive procedure, or trauma
Cryoprecipitate • Given in pools of 6-IO units: • DIC with bleeding
increases fibrinogen by • Von Willebrand disease•
30-60 mg/dL (1 unit/5 kg raises • Factor XIII deficiency•
fibrinogen by 70 mg/dL) • Hemophilia A with bleeding•

• Plasma exchange in TIP


• Alternative to FFP for coagulopathy
• Rapid temporary warfarin
• Contains plasma proteins and reversal in those needing
Cryosupernate factors II, VII, IX and X (stable emergency surgery
clotting factors) • Warfarin overdose with life-
threatening bleeding in addition to
prothrombin complex concentrates
• Hemophilia B with bleeding

• Patients with markedly decreased


• Mostly replaced with
Granulocyte ANC or those with gram-negative
granulocyte colony-stimulating
concentrate sepsis with ANC <500 and not
factor (G-CSF) injections
responding to antibiotics
• Provides both O2-carrying • Massive blood loss .. &
capacity and volume expansion coagulopathy
Fresh whole (RBC, plasma, and platelets) • Trauma or hemorrhagic shock
blood • Dose: 20 mL/kg x 4 hours when specific blood product
• Should be of the same ABO and therapy is not available
Rh(D) type • Neonatal exchange transfusion
'Use of cryoprecipitate is not generally considered appropriate for hemophilia,von Willebrand's
disease, or deficiency in factor XIIIor fibronectin,unless alternative therapies are unavailable
*'Massivebloodloss: loss of 1-1.5xthe whole bloodvolumewithin24 hours; or rate of loss is 150mUmin
Source:PhilippineCPGfor the RationalUseof Bloodand BloodProductsand Strategiesfor lmplemenlal1on;
2009

BLOOD TYPING
The ABO bloodgroup system is the most clinicallysignificantin transfusion medicine& for this reason,all
donor bloodfor transfusionis tested & labeledwith the ABO group. The 4 main phenotypes are A, B, AB, &
0: they are determinedbasedon the presenceof theA and B antigens,with the O groupsignifying lackof both
antigens.An adult lacking the correspondingantigen will possessanti-A &/or anti-B antibodies.Anti-A or
anti-B immunoglobulinscan causeintravascularhemolysis when ABO-incompatibleRBCs are transfused.

A
BLOODTYPE
I ANTIGEN
(Present on RBC)
A-Antigen
I ANTIBODY
(Present in Serum)
Anti-B
B
. B-Antigen Anti-A

AB (universal recipient) A and B Antigen None

0 (universal donor) None Anti-A, Anti-B, Anti-AB


Interpretation
• Donor with type-0 blood can give pRBCs to recipient of any blood type
• Donor with type-Ablood can give pRBCs to recipient with type A or AB
• Donor with type-B blood can give pRBCs to recipient with type B or AB
• Donor with type-ABblood can give pRBCs only to recipient with type AB
127
REFERENCES (tor Chapter 7 & BJ
1.Apkon M, Leonard J, Probst L. Design of a safer approach to IV drug infusions: failure mode effect
analysis. Qua! Saf Health Care 2004;13:265-71.
2. Adrogue 1-lJand Madias NE. Hyponatremia. N Engl J Med. 2000;342(21):1581-1589
3.Adrogue HJ and Madias NE. Hypematremia. N En_glJ Med. 2000;341(to):1493-1499
4. Bai SJ, Lee JW, and Lee KY.Fluid Therapy: Classification and Characteristics of IV Fluids. J Korean Med
Assoc. 2010 Dec; 53(12):1103-1112.
5.Barr J, Fraser GL, Puntillo K, et al Clinical Practice Guidelines for the Management of Pain, Agitation,
and Delirium in Adult Patients in the Intensive Care Unit. Crir Care Med, 2m3; 41:263.
6. Bojar, Robert. Manual of Perioperative Care in Adult Cardiac Su:gery. 6th ed., Wilex-Blackwell, 2021.
7.Carson )L, Guyan G, Heddie NM, Grossman BJ, Cohn CS, Fung MK, et al. Clinical Practice Guidelines
From the AABB Red Blood Cell Transfusion ihresholds and Storage. JAMA. 2016;316(19):2025-2035.
8. Devlin JW, Skrobik Y, Gelinas C, et al. Clinical Practice Guidelines for the Prevention and Management
of Pain, Agitation/Sedation, Delirium, Immobility, and Sleep Disruption in Adult Patients in tlie ICU.
Crit Care Med 2018;46:e825.
9. "'Dosing &amp; Administration Guidelines for Activase® (Alteplase)." Activase, www.activase.com/ais/
dosing-and-administration/dosing.html.
IO. "Dosing and Administration: Dosing Forumulas and Calculator." Aggrastat® (Tirofiban Hydrochloride)
Injection, ww,v.aggrastathdb.com/dosing-and-administration.
11. Global RxPh. Tlie Clinicians Ultimate Reference. 2017. Intravenous Dilution Guides. Retrieved from
http://www.globalrph.com/methylprednisolone_dilution.htm
12. Goltzman, D. Treatment ofhypocalcemia. In: UpToDate, 2m6.
13. Harby, Peter W, et al. "Tocilizumab in Patients Admitted to Hospital with COVID-19 (RECOVERY):
Preliminary Results of a Randomised, Controlled, Open-Label, Platform Trial." 2021, doi:10.1101/2021.
02.11.21249258.
14. lngelfinger J, Moritz M, and Ayus J. Maintenance Intravenous Fluids In Acutely Ill Patients. New Engl
Med 2015;373(14):1350-1360.
15. ~ameson, J. Larry, et al. Harrison's Principles of Internal Medicine, 2och Edition. McGraw-Hill
ducation, 2018.
16. Jayaram S. Intravenous Fluids: Compositions and Uses. Online lecture. 2017.Retrieved from: https://
www.med.mun.ca/getdoc/6f26f87o-6C78-4a73-8ofo-98200858aafd/lntravenousFluids.aspx
17. Karen, Tietze, and- Barry Fuchs. '"Sedative-Analgesic Medications in Critically Ill Adults: Properties,
Dosage Regimens, and Adverse Effects." Uptodare, 13 Mar. 2021, www.uptodate.com/contents/
sedat1ve-ancllgesic-medications-in-critically-ill-adults-properries-dosage- regimens-and-adverse-
effects?searcli=PADIS%2oguidelines&topicRef=2888&source=see_link.
18. Kasper DL, Fauci AS, Hauser SL, Longo DL, Jameson JL, and Loscalzo J (editors). Harrison's Principles
of Internal Medicine, 19th ed. USA: McGraw Hill Education, 2015.
19. Kaufman RM, D\"ulbegovic B, Gernsheimer T, Kleinman S, Tinmouth AT, Capocelli KE, et al. Platelet
Transfusion: AC inical Practice Guideline from the AABB. Ann Intern Med. 2015;162:205-213.
20.Koda-Kimble M, Young LY,et al. Handbook of Applied Therapeutics. Lippincott Williams & Wilkins,
2006
21. Kroll MH and Ellin RJ. Relationships between magnesium and protein concentrations in serum. Clin
Chem. 1985;31(2):244-6.
22. Mal brain M, Regenmorrel N, and Owczuk R. It is rime to consider the four D's of fluid managemenL
Anaesthesiol lntensive Ther. 2015;47 Spec No: s1-s5.
23. Marino, P. L. (2014).Section XV Critical Care Drug Therapy. In Marino"s the ICU book (4th ed.). essay,
Wolters Kluwer Health/Lippincott Williams &amp; Wilkins.
24. Niluk Leon Senewiratne NL, Woodall A and Can AS. Sodium Bicarbonate. In: StatPearls[Internet].
Treasure Island, FL: StatPearls Publishing. 2021.
25. Ohashi, K., Dalleur, 0., Dykes, P.C. et al. Benefits and Risks of Using Smart Pumps to Reduce Medication
Error Rates: A Systematic Review. Drug Saf '}_7, 1011-1020(2014).https://doi.org/10.1007/s40264-014-0232-1
26. Payne RB, Little AJ, Williams, RB, and Milner JR. Interpretation of serum calcium in patient with
abnormal serum proteins. Br Med J. 1973;4(589~):643-6.
27. Pinskx, Michael, et al. Hemodynamic Monitoring (Lessons from the ICU). 1st ed. 2019, Springer, 2019.
28. Rasdike RA, Reilly BM, Guidry JR, Fontana JR, and Srinivas S. The weight-based fieparin dosing
nomogram compared with a "standard care" nomogram: a randomized controlled trial. Ann Intern
Med 1993;u9:874-81.
29. Roback JD, Caldwell S, Carson J, Davenport R, Drew MJ, Eder A, et al. Evidence-based practice
guidelines for plasma transfusion. Transfusion. 2010;50:227-1239.
30. Rondon-Berrios H and Tomas Berl. Mild Chronic Hyponatremia in the Ambulatory Setting:
Si_gnificanceand Management. CJASN December 2015; 10 (12)2268-2278.Rose BD and Post T (editors).
Clinical physiology of acid-base and electrolyte disorders, 5th ed. USA: McGraw-Hill, 2001.
31. ScyMed Medical Calculator System. http://www.scymed.com/.
32. Shane E and Berenson JR. Treatment ofhypercalcemia. In: UpToDate, 2016.
33. Skorccki K, Chcrtow GM, Marsden PA, Taal MW, and Yu ASL (edilors). Brenner and Rector's The
Kidney, 10th Edition. Philadelphia, PA: Elsevier. 2016.
34. Sterns RH. Diagnostic evaluation of adults with hyponatremia. Retrieved fromUptodate. Accessed
onlme August 26, 2021
35. Shafiee MAS, Bohn D, Hoorn EJ, and Halperin ML How to select optimal maintenance intravenous
fluid therapy. QJM Med. 2003; Aug 96:601-610.
36. Stone, /ohn H., et al. "Efficacy ofTocilizumab in Patients Hospitalized with Covid-19." New England
Journa of Medicine, vol. 383, no. 24, 2020, pp. 2333-2344.,doi:10.1056/nejmoa2028836.
37. 'rhomas C. 2007. Syndrome of Inappropriate Antidiuretic Hormone Secretion. Retrieved from: https://
emedicine.medscape.com/anicle/246650-differential
38. Technical Working Group. Philippine Clinical Practice Guidelines for the Rational Use of Blood and
Blood Products and Strategies for Implementation. 2009. Available from: http://www.pshbt.org/images/
docs/Blood_Safety_CPG_and_Policy Final_10282009.pdf
39. Triulzi DJ (ed). Blood transfusion therapy, a physician's handbook, 7th ed. Bethesda, MD: American
Association of Blood Banks, 2002.
40. World Health Organization. The Clinical Use ofBlood in Medicine, Obstetrics, Pediatrics, Surgery and
Anesthesia, Trauma and Burns. 1-334;WHO. 2002.
41. Zipes DP, Libby P, Bonow R, Mann DL, Tomaselli GF. Braunwald's Heart Disease: A Textbook of
Cardiovascular Medicine. lllh Edition. Elsevier/Saunders, 2019.

128
ELECTRO
OVERVIEW
1. General Formulas
2. Sample Cases

0 FLUIDS AND ELECTROLYTES


1. Water Balance
2. Sodium
3. Potassium
4. Calcium
5. Magnesium
6. Bicarbonate
SECTION ONE
OVERVIEWQF FLUID AND ELECTR'Ol:YlES
GENERAL FORMULAS
I. ACID-BASE BALANCE EQUATIONS
A. Henderson-Hasselbach Equation
0 This equation is used to understand the acid-base balance in a bicarbonate buffer
system and to verify the values of an arterial blood gas (ABG) result
0 Before interpreting an ABG, the equation can be used to check the consistency of the
measured values and if there are errors in measurement (arterial versus venous blood)
0 By using the measured HCO3 and PCO2 values from the ABG, compute for the
expected pH and compare it with measured pH

LEGEND I NORMAL

pH= 6.10 + log


[HCO3]
o.03xPCO2
t--pH_:_s_e_ru_m_pH
________
[HCO,]:serum bicarbonate(mmol/L)
RANGE

I:
---tr-7_.3_5_-_7._45_---t
24 ± 2 mmol/L ..
PCO,:partial pressureof CO,(mmHg) 40 ± 5 mmHg

B. Expected Compensatory Response for Simple Acid-Base Disturbances


METABOLIC DISTURBANCE &
EXPECTED COMPENSATION
I RESPIRATORY DISTURBANCE &
EXPECTED COMPENSATION
• Respiratory acidosis:
• Metabolic acidosis:
Acute acidosis j /':,.[HCO3] = 0.1 x /':,.pCO2
L /':,.
pCO2 = 1.25x /':,.[HCO3]
Chronic acidosis j !':,.[HCO3] = 0.4 x /':,.pCO2
• Respiratory Alkalosis:
• Metabolic alkalosis:
Acute alkalosis LI':,.
[HCO3] = 0.2 x /':,.pCO2
j /':,.pCO2 = 0.75 x /':,.[HCO3]
Chronic alkalosis Lt:,.[HCO3] = 0.4 x t,. pCO2
Source:RoseBD.Clinicalphysiology
of acid-baseandelectrolytedisorders,5thed, McGraw-Hill,
2001

C. Serum Anion Gap (AG)


0 The AG represents the unmeasured anions normally present in plasma & unaccounted
for by the serum electrolytes exclusive ofK• that are measured on the electrolyte panel
0 The interpretation of the AG requires either a normal serum albumin, or correction of
the AG to a normal plasma albumin
0 In normoalbuminemia, elevation of AG is usually due to addition of non-chloride
containing acid anions, most commonly:
• Inorganic anions (PO4J-,S04')
• Organic anions (ketoacids, lactate, uremic organic anions)
• Exogenous anions (salicylate or ingested toxins with organic acid production)
• Unidentified anions

Serum AG= Na• - (Cl+ HC03)

Corrected AG= Actual AG+ [0.25 x (45 - Alb)]

• Na:serumsodiumin mmol/Lor mEq/L


Interpretation:
• Cl: serumchloridein mmol/Lor mEq/L
• NormalAG: 10± 2 mmol/Lor mEq/L
• HCO3: serumbicarbonatein mmol/Lor mEq/L
• HighAG:>12mmol/Lor mEq/L
• Alb:serumalbuminin g/dL
SourceRoseBD.Cl1mcal
physiology of acid-baseandelectrolytedisorders,5thed, McGraw-Hill,
2001
SkoreckiK, et al. BrennerandRector'sTheKidney,10thEdition.Elsevier.2016

131
D. Urine Anion Gap
0Indirect measure of ammonia in urine
0Helps in differentiating the causes ofNAGMA (Normal Anion Gap Metabolic Acidosis)

Urine AG= Na•+ K• - Cl

Interpretation: • Na:urinesodiumin mmol/Lor mEq/L


• UrineAG <0:GI lossesor diarrhea(Ne"GUT"ive) • K: urinepotassiumin mmol/Lor mEq/L
• UrineAG >0:renaltubularacidosis(TypeI or IVaa) • Cl: urinechloridein mmol/Lor mEq/L

E. Bicarbonate/Base Deficit
0 Amount of bicarbonate required to correct metabolic acidosis
0 Use lean body weight (BW) in the computation

Bicarbonate deficit = (Desired HC03 - Actual HC03 ) x BW in kg x 0.4

II. OSMOLALITY
A. Serum Osmolality
0 Balance between the water and the chemicals dissolved in blood
0 Helps in assessing presence of severe dehydration or over hydration

Serum Osmolality = 2 (Na•+ K•) +RBS+ BUN

Interpretation: • Na:serumsodium
• Normalosmolality:
280-295mosmol/kg • K: serumpotassium
• Increased
osmolality:
dehydration,
poorlycontrolled • RBS:randombloodsugarin mmol/L
OM(OKA,HHS),diabetesinsipidus • BUN:serumBUNin mmol/L
• Decreasedosmolality:
overhydration,
diuretics,SIADH

BS erum 0 smo 1a JG ap

Osmolal gap = measured osmolality - calculated osmolality

Interpretation:
• Measuredosmolality: donein a
• Normalgapis S10mOsm/kg
laboratory(inmOsm/kg)
• If >10:indicatesunmeasured osmotically
active
• Calculated osmolality:
usingabove
substances (e.g.,methanol,ethyleneglycol,sorbitoland
formula
mannitol)andin pseudohyponatremia

C. Urine Osmolality (Estimate)


0 Measure of urine concentration
0 Not reliable in the presence of glucosuria

Urine Osmolality = (Urine SG - 1) x 35,000

Interpretation:
• Normal24-hoururineosmolality= 50-800mOsm/kg
• UrineSG:urinespecificgravity
• Largevaluesindicateconcentrated urine(e.g.,heart
• Each30-35mosmol/kg raisesurine
failure,dehydration,
shock,SIADH)
SG by ~0.001
• Smallvaluesindicate dilutedurine(e.g,.diabetesinsipidus
or DI,renaltubularnecrosis,renalfailure,pyelonephritis)
Source:RoseBO.Clinicalphysiology
of acid-baseandelectrolyte
disorders1,5thed,McGraw-Hill,2001
JamesonJl, et al. Harrison's
PrinciRles
of InternalMedicine20thedition,2018
PayneRB,et al. BrMedJ. 1973& KrollMHhetal. ClinChem.1985
CalviLMandBus inskyDA.JASN.2008
ThomasC. Syndrome of Inappropriate
Antidiuretic
Hormone Secretion

132
III. CORRECTION OF ELECTROLYTES
A. Corrected Sodium (Na')
In marked hyperglycemia, ECF osmolality increases
0

Na· falls in proportion to ECF dilution, declining 1.6mEq/L per IOO mg/dL increase in RBS
0

In marked hyperlipidemia, serum Na· also falsely decreases, leading to pseudohyponatremia


0

• ActualNa·:serumsodium
Corrected Na•= Actual Na•+ 0.016 (RBS - roo)
• RBS: randombloodsugarin mg/dl
= Actual Na'+ 0.2 (TG) • TG:serumtriglycerides
in g/L

B. Corrected Calcium (Ca")


° Calcium must be "corrected" when the albumin is abnormal
This is to correct for the change in total calcium due to the change in albumin-bound Ca"
0

Measures total serum Ca" composed of albumin-bound Ca'\ ionized Ca'· and
0

Ca" complexed with various anions


Measuring ionized or free form of Ca'· instead of total serum Ca" is recommended in criticalcare
0

settings, patients on hemodialysis, & other ambulatory settings (e.g.,CKD,hyperparathyroidism)

.____ __
Corrected Ca"= Actual Ca''+ ((40 - alb) x 0.02]
_
• ActualCa2': serumCa2• in mmol/L
• Alb:serumalbuminin g/L ___,
__.____ ·

C. Corrected Magnesium (Mg'')


In hypoalbuminemic states, total serum magnesium must be corrected since 30% of it
0

is bound to albumin
• ActualMg2': serumMg2' in mmol/L
Corrected Mg"= Actual Mg"+ ((40 - alb) x 0.005]
• Alb:serumalbuminin g/L

SAMPLE CASES
Case r
A 32-year-old male with type I diabetes was admitted for drowsiness, fever, cough, diffuse
abdominal pain, and vomiting. He tested positive for ketone bodies in the urine, hence
diabetic ketoacidosis was considered. Laboratory results showed:
Chemistry RBS 450 mg/dl (25.0mmol/L),urea60 mg/dl (10.2mmol/L),
Creatinine1.4mg/dl (123.7µ mol/L),Albumin26 g/L
Electrolytes Na 152mEq/L,K 5.3mEq/L,Ca 2.2 mmol/L,Mg0.7 mmol/L,
PO,2.3 mEq/L(0.74mmol/L),Cl 110mmol/L
ABG pH 6.9,PO2 95 mmHg,PCO2 28 mmHg,HCO3 9 mEq/L,and0 2 sat98%

r. Corrected Sodium
Corrected Sodium = Actual Na+ 0.016 (RBS - IOO)
= 152mEqs/L + 0.016 (450 mg/dL- wo)
= 157.6mEqs/L
In marked hyperglycemia, ECF osmolality increases. Serum Na· falls in proportion to ECF
dilution, decliningr.6 mEq/ L per 100 mg/dL increase in RBS. In this example, the true level
of sodium is 157.6mEqs!L.

2. Corrected Calcium
Corrected Calcium = Actual Ca'·+ (40 - Albumin) x 0.02
= 2.2 mmol/L + (40 - 26 g/L) x 0.02
= 2.48 mmol/L

3. Corrected Magnesium
Corrected Magnesium= Actual Mg+ (40 - Albumin) x 0.005
= 0.7 mmol/L + (40 - 26 g/L) x 0.005
= 0.77 mmol/L
133
Case2
A 24-year-old female came in for diarrhea after binge-eating in a sushi restaurant. At the
ER, she was noted to be weak, with BP of 90/60 mm Hg, HR of 105 bpm, RR of 26 cpm,
temperature of 37.2°C, and weight of 50 kg. She had dry oral mucosa and axillae, clear
breath sounds, and flat and soft abdomen with no tympany. No peripheral edema.

Some of her initial laboratory tests were as follows:

CBC Hgb142g/dl, Hct0.46,WBC13.5(Neut78%Lym21%),Pit 256

ABG pH 7.32,pCO,33 mmHg,HC03 18mmol/L,p02 97,0 2 sat 99%

Serum electrolytes Na' 138mmol/L,K' 3.1mmol/L,Cl 110mmol/L,albumin38 g/L

Random urine electrolytes Na' 100mmol/L,K' 22 mmol/L,Cl 126mmol/L

Urinalysis Yellow,pH6.0,s.g. 1.025,negativefor protein& sugar,


0-1WBC,0-1 RBC

I. What is the primary acid-base disturbance?


Given a low arterial pH (acidotic), low serum HCO3 (acidotic) and low pCO2 (alkalotic),
primary metabolic acidosis is considered.

2. What kind of metabolic acidosis is present? And is there compensation?

Computing for the serum anion gap,


Serum AG = Na' - (Cl+ HCO3)
=138- (110+ 18)
= IO

Corrected AG = Actual AG+ [0.25 x (45-albumin)]


=IO+ [0.25 X (45 - 38}
= 11.75(which indicates NAGMA)

Expected compensation
t:,.pCO2 = 1.2x t:,.HCO3
= 1.2 X (24 - 18)
= 7.2
Expected pCO2 = 40 - 7.2 = 32.8 (which indicates compensated NAG MA)

3. What is the cause of the acid-base disturbance?


Based on the history, the most likely cause ofNAGMA is gastrointestinal loss ofHCO3
from diarrhea. To verify, we check urine AG,

Urine AG =(Na'+ K') - Cl


= (100 + 22) - 126
= -4 (which is <O or negative, hence GI losses explain the NAG MA)

4. What is the bicarbonate deficit? And how would we correct the underlying disturbance?

Bicarbonate deficit = (Desired HCO3 - Actual HCO3) X weight x 0.4


= (24 - 18)X 50 X 0.4
= 120 mEqs of bicarbonate

In this case, there is no urgent indication to give bicarbonate as the underlying


gastrointestinal problem can be addressed by rehydration with JV isotonic crystalloids,
volume/volume replacement with oral rehydration salts, and potassium supplements.
In these cases, we first expect respiratory compensation by hyperventilation and then
compensation by the kidneys through generation of bicarbonate.

134
SECTION TWO
FLUIDS AND ELECtROL¥l'ES

ELECTROLYTE
SI UNITS
I To Convert St Units to
Conventional Units,
I CONVENTIONAL
(Normal Values)
I Multiply St unit by:
UNITS

Sodium 137-145mmol/L 1 137-145mEq/L


Potassium 3.5-5.1mmol/L 1 3.5-5.1mEq/L
Calcium 2.10-2.55mmol/L 4 8.4-10.2mg/dl
Calcium, Ionized 1.05-1.30mmol/L 4 4.2-5.2mg/dl
Magnesium 0.70-1.00mmol/L 2.43 1.6-2.3mgldl

WATER BALANCE
Water is the most abundant constituent (50% of body weight in women & 60% in men)
Total body water (TBW): 55-75%intracellular fluid (!CF) & 25-45%extracellular fluid (ECF)
Osmolality: solute or particle concentration of a fluid (mosmol/kg of water)

I. HYPOVOLEMIA
RENALCAUSES I EXTRARENALCAUSES
• Osmotic diuresis (e.g., mannitol) • GI tract losses (e.g., impaired GI
• Diuretics (e.g., furosemide) reabsorption, enhanced fluid secretion)
• Defects in renal transport proteins, • Insensible losses (evaporation of water
mineralocorticoid defects (e.g., deficiency) from skin and respiratory tract)
• Tubulointerstitial injury (e.g., interstitial • Accumulation of fluid within specific
nephritis, acute tubular injury) tissue compartments (e.g., interstitium,
• Excessive renal water excretion (e.g., DI) peritoneum, GI tract)

Some/indinqson examinationinclude:
• Decreased jugular venous pressure (JVP)"
• Orthostatic tachycardia (increase of >l5-20 bpm on standing)*
• Orthostatic hypotension (drop of >l0·20 mm Hg blood pressure on standing)*
• In severe cases: peripheral cyanosis, cold extremities, oliguria, altered mentation•
• Diminished skin turgor; dry oral mucous membranes
*Thesearethe morereliablefindingson e,xamination

Normonatremic or • Normal saline (0.9% NaCl) is the most appropriate


hyponatremic patients resuscitation fluid
• Hypotonic solutions (see below)
Hypernatremic 05% dextrose if water loss only (e.g., diabetes insipid us)
patients 0 Hypotonic saline if both water and sodium loss

Patients with HCO3 loss


• IV bicarbonate (150 mEq ofNaHCO3 in 5% dextrose)
& metabolic acidosis

Patients with severe • pRBC transfusions without increasing the hematocrit


hemorrhage or anemia beyond 35%
*Mildhypovolemiacangenerallybe managedwithoralhydrationandresumption
of normal
maintenance
diet. Severehypovolemiais managedwithIVhydration.
135
II. HYPERVOLEMIA
Occurs when volume of the ECF compartment is expanded relative to its capacitance
This happens when additional sodium loading is not matched by an additional sodium
excretion, hence sodium retention, which can be primary or secondary
PRIMARY CAUSES I SECONDARY CAUSES
• Oliguric acute kidney injury • Heart failure
• Chronic kidney disease • Cirrhosis
• Glomerular disease • Idiopathic edema
• Severe bilateral renal artery stenosis
• Sodium-retaining tubulopathies (genetic)
• Mineralocorticoid excess

• Reasonable goal of restricting to 50-80 mmol (~3-5g) of salt/day


Sodum (Na·)
• Avoid giving unnecessary IV saline (combination of saline & diuretics in
restriction &
HF is associated with adverse outcomes)
fluid balance
• Advise water restriction only in presence ofhyponatremia (Na·<135)
• Loop diuretics in severe hypervolemia & HPN, especially in advanced CKD
• Thlazides/thiazide-like diuretics (e.g.,metolazone) as adjunct to loop diuretics
• Aldosterone antagonists (e.g., spironolactone, eplerenone) for cirrhosis
with ascites, cardioprotection and renoprotection
Diuretics
• Diuretic resistance may be treated by:
° Combining loop diuretics with other diuretics {thiazides, metolazone)
0 Giving a bolus dose ofloop diuretic followed by high-dose continuous
infusion or intermittently giving high-dose loop diuretics
Extracorporeal • Reserved for extreme resistance to diuretics (often with renal impairment)
ultrafiltration • Includes hemodialysis, isolated ulcrafiltration, peritoneal dialysis

SODIUM
• Disorders are due to abnormalities in water homeostasis that lead to changes in the relative
ratio of Na· to body water (plasma Na· concentration tells nothing about the volume status)
I. HYPONATREMIA
Defined as plasma sodium <135mmol/L
• Almost always due to increased circulating AVP &/or increased renal sensitivity to AVP
combined with any intake of free water (exception is hyponatremia due to low solute intake)
Hyponatremia causes generalized cellular swelling
A. Approach and Causes ofHyponatremia
0Hyponatremia is classified into three groups, depending on the volume status of the patient
Initial approach consists of a thorough history, physical exam, and selected laboratory tests
. •
0


Subclinically
volume- Increasein TBWpreaterthan
expandedpatients increasein totabodyNa'
UN, >20 UN, <20 UN, >20 UN, >20 UN, <20
• Renal losses • Extrarenal • Glucocorticoid • Acute or • Nephrotic
• Diuretic excess losses deficiency chronic syndrome
• Mineralocorticoid • Vomiting • Hypothyroidism renal • Cirrhosis
deficiency
• Salt-losingdeficiency • Diarrhea • Stress failure • Cardiac
• Bicarbonaturia with • Third spacing • Drugs failure
RTA & metabolic offluids • Syndrome of
alkalosis • Burns inappropriate
• Ketonuria • Pancreatitis antidiuretic
• Osmotic diuresis • Trauma hormone (SIADH)
• Cerebral salt
secretion
wastin syndrome
UN,= Urinesodiumin mmol/L
(amount
of sodiumin a urinesample,
usedto classifyhyponatremia)

136
B. Manifestations (Primarily Neurologic)
• Early symptoms include nausea, headache, and vomiting
• Severe cases may present with seizures, brainstem herniation, coma, and death
• Key complication is normocapnic or hypercapnic respiratory failure (associated
hypoxemia may amplify neurologic injury)
• Clinical assessment should focus on the underlying cause:
• Check for intake of drugs and supplements
• Assessment of volume status (see above) is central to the diagnostic approach
• Consider all possible causes of excessive circulating AVP

C. Major Considerations in Management


0 Presence/severity of symptoms determine the urgency and goals of therapy
• Those with chronic hyponatremia are at risk for osmotic demyelination syndrome (ODS) if Na·
is corrected >8-10mmol/L within the first 24 hours &/or by >18mmol/L within the first48 hours
• Response to hypertonic saline, isotonic saline and vasopressin antagonists are
unpredictable; ergo, frequent Na· monitoring is needed
0 Once therapy instituted, focus on treatment or withdrawal of underlying cause
OPTIONS I Na• CONTENT I HC0 3·CONTENT
NaHCO3 8.4% solution 50 mL 50 mEqsNa• 50 mEqsHC03·
NaHC03 Grain X tab (650 mg) 7.7mEqsNa•pertab 7.7mEqsHC03 pertab
NaCl Tab (1 g) 17mEqsNa· pertab ---
Normal saline solution (1 L) 154mEqsNa• ---
Sources:JamesonJL,et al. Harrison'sPrinciplesof InternalMedicine20thedition,2018
KumarS, et al: Diseasesofwatermetabolism, inAtlasof Diseasesof the Kidney-;1999
SternsRH.Diagnostic evaluationof adultswithhyponatremia. UpToDate
Rondon-Berrios et al. ClinJ AmSoc Nephrol.2015
Correcting Sodium Deficit and Hyponatremia
Hyponatremia is almost always caused by impaired renal water excretion, hence
assessment and correction of volume status is important in correcting hyponatremia
STEPS IN CORRECTING HYPONATREMIA I SAMPLE CASE
Case 1: A 30/M, 60 kg (TBW = 36), complaining of a 3-day history of diarrhea.
BP 90/50, tachycardic, with serum Na·= 120 mmol/L, K· = 3.2 mmol/L. He has ongoing losses of
200 cc/LBM, ~8 times/day [note that Na· losses in LBM = 25 - 50 mmol/L]
1.Identify by history & examination if the hyponatremia
Type ofhyponatremia:
is hypovolemic, euvolemic or hypervolemic. Correct
hypovolemic hyponatremia
hemodynamic instability first before correcting hyponatremia
2. Determine the rate of correction
Rate of correction desired
• Acute (<48h) and severely symptomatic (e.g. seizures)= 4-6
in this case:
mmol/L within the first 2-4 hours, with frequent Na· monitoring
• Chronic or those who are asymptomatic = not more than 8-10
7 mmol/L in 24h
mmol/L in any 24h period

3. Determine the type of fluid infusate desired Given the hemodynamic


lnfusate Na" (mmol/L) K· (mmol/L) ECF d1strobut1on status, consider giving
fluid challenge first of1L
5%NaClfnwater 855 0 100% pNSS/h for the next 2-3
3%NaClInwater 513 0 100% hours and recheck the
electrolytes. Subsequently,
0.9%NaclInwater 154 0 100%
--+-----+--------11 after hemodynamic status
Ringer's
lactate 130 4 97% stabilized, may start fluid
0.45%N8CIln•water 77 0 73% infusate desired:

Inwater
5%Dextrose 0
__ ....._____ 0 .._ _____ 40% _. 1 pNSS 1L+ 20 mEqs KC!

137
STEPS I SAMPLE CASE
4. Estimate the effect of I liter of any infusate 1 liter of pNSS + 20 mEqs KC! will change the
containing Na· and K· on serum Na· by:
serum Na•
(154+ 20) - 120
Change in serum Na•= Change in serum Na·=
36 + I
(lnfusate Na·+ Infusate K·) - serum Na·
TBW+1 = 1.46 mmol/L

7mmol
5. Calculate the drip rate Amount of fluid = =4.8L
1-46mmol/L
Amount of fluid= Drip rate 4800ml =20omL/hr
=
Change in serum Na• desired 24h
Estimated effect of I liter infusate
' Sample order: Start !VF with PNSS 1L+ 20 mEqs.
Drip rate= KC] at 200 cc/h, recheck serum Na' & K' every
4-6h. One must also take into account any ongoing
Amount of fluid
GI fluid losses, hence may give volume/volume
target number of hours replacement via oral route with oral rehydration
salts or may increase !VF rate as needed

STEPS I SAMPLE CASE


Case 2: A 50/F, 48 kg (TBW = 24), admitted due to increased sleeping time the past week.
She has had persistent cough and weight loss over the last 3 months. BP 120/70, normal
HR, RR and temperature, no orthostatic hypotension, unsustained wakefulness to name
calling, with note of decreased breath sounds on her left mid to lower lung field. No noted
ascites or peripheral edema. She has pleural effusion on CXR; serum Na· = 125 mmol/L;
K· = 3.7 mmol/L.

Type ofhyponatremia:
I. Identify type ofhyponatremia
euvolemic hyponatremia
Rate of correction desired in this case:
2. Determine the rate of correction
8 mmol/L in 24h
Given the status, there is no need to stabilize
3. Determine the type of fluid infusate
the hemodynamics. We can give this patient
desired
hypertonic saline solution (3%)
4. Estimate the effect of I liter of any infusate 1liter of3% NaCl will change the serum Na by:
containing Na· and K· on serum Na·
513- 125
Change in serum Na·= Change in serum Na·=
24 + I
(lnfusate Na·+ Infusate K·) - serum Na·
TBW+1 = 15.5mmol/L

5. Calculate the drip rate 8mmol


Amount offluid = =0.52 L
15.5mmoUL
Amount of fluid=
52omL
Change in serum Na• desired Drip rate = =22mL/hr
24h
Estimated effect of I liter infusate

Drip rate= Sample chart order: Start !VF with 3% NaCl rL


Amount of fluid at 22 cc/h. Monitor serum Na' every 6-8 hours.
Monitor urine output, VS & Neuro vital signs.
target number of hours

Source:AdrogueHJ,et al. Hyponatremia.


N EnglJ Med2000
138
II. HYPERNATREMIA
• Defined as an increase in plasma sodium concentration to >145mmol/L
Usually the result of a combined water and electrolyte deficit, with losses of water in
excess of those of sodium (net water loss), or a hypertonic sodium gain

A Ef ' th .
DUE ~o NET WATER Loss I DUE TO HYPERTONIC SODIUM GAIN

• GI water loss/diarrhea: most common • Hypertonic feeding preparation, ingestion


gastrointestinal cause of sea water, sodium chloride
• Insensible water loss (fever, exercise, heat, • Hypertonic sodium bicarbonate infusion,
burns, mechanical ventilation) hypertonic saline enemas, hypertonic
• Renal water loss: osmotic diuresis from sodium chloride infusion, hypertonic
hyperglycemia, excess urea, post- dialysis
obstructive diuresis, mannitol • Primary hyperaldosteronism, Cushing
• Diabetes insipidus (central, nephrogenic) syndrome

B. Manifestations
0 Symptoms are explained by cellular shrinkage due to efflux of intracellular water
0 Primarily neurologic: change in sensorium is the most common manifestation
0 Hypernatremic rhabdomyolysis secondary to osmotic damage to muscle membranes

C. Management
° Central to the management is correction of the underlying cause:
• Chronic hypernatremia (>48 hours): correction must be carried out slowly to avoid
cerebral edema (e.g., correct deficit over 48 hours); plasma sodium should not be
corrected >IO mmol/day
• Acute hypematremia due to Na· loading can be safely corrected at the rate of1 mmol/h
0 Water, as much as possible, must be administered by mouth or by NGT
0 Alternatively, D5W can be used with corresponding CBG monitoring

Sample Case 1: A 70/F, 50 kg (TBW = 25), chronically bedridden, presented with decrease
in sensorium at the ER. She was noted to be febrile, tachypneic with BP 90/60, dry mucous
membranes and no axillary sweat. Serum Na· = 158mmol/L and K· = 3.8 mmol/L. How do you
manage her?

1.Identify in history and physical examination if Cause ofhypernatremia:


hypernatremia is from net water loss or hypertonic Na· gain Net water loss

2. Determine the rate of correction


• Acute (<48h):1mmol/Uh for severalhours with frequent monitoring Desired rate of sodium
• Chronic or those with unknown duration: <IO mmol/L correction: 8 mmol in 24h
decrease in serum Na· in 24h
3. Determine the type of fluid infusate desired

40%
Fluid infusate desired:
73% o.45%NaCl
97%

0.9%NaClinwater 154 100%

1Lofo.45% NaCl will


4. Estimate the effect of I liter of any infusate containing Na·
change serum Na· by:
and K• on serum Na· ·
. N (lnfusate Na·+ Infusate K•) - serum Na• = 77-158
Ch ange m serum a·=---------------
TBW + 1 25 + I
=3.nmmol/L
139
5.Calculate the drip rate

Amount of fluid= 8mmol


Amount of fluid = = 2.6 L
Change in serum Na• desired 3.11mmol/L
Estimated effect of I liter infusate

Drip rate= Drip rate = ----


2600ml
24h
=Io8mL/hr
Amount offluid
target number ofhours

6. Add insensible losses


Insensible losses= IO x 50 = 0.5 Lor -20 ml/h
6. Compute for insensible losses= -IO mL/
kg/day (less if ventilated, more iffebrile) Sample chart order:
• Start !VF 0.45% NaCl I L x 128cc/hr
• Please do serum sodium 94-6

STEPS IN CORRECTING
WATER DEFICIT I SAMPLE CASE

Sample Case 2: A 40/F 70kg (TBW = 35) was admitted due to decreased sensorium. She
sustained skull and hip fractures three days ago. She was seen comatose hence intubated and
hooked to ventilator with BP 140/90, HR 87, afebrile, with urine output of 2 liters in the last 12
hours. Serum Na·= 168 mmol/L and K· = 4.8 mmol/L. How do you correct the hypernatremia?
1.Identify cause of hypernatremia Cause ofhypernatremia: Pure water loss
2. Determine the rate of Na· correction Desired rate of Na- correction: 6 mmol in 24h
3. Determine the type offluid infusate desired Fluid infusate desired: D5 water (D5W)

4. Estimate the effect of I liter of any infusate IL ofD5W will change serum Na by:
containing Na• and K- on serum Na-

Change in serum Na·= 0-168


(lnfusate Na-+ Infusate K-) - serum Na- Change in serum Na·=
35 + I
TBW+1 =4.7mmol/L
5. Calculate the drip rate

Amount of fluid= 6mmol


Amount of fluid = = 1.3 L
Change in serum Na· desired 4.7mmol/L
Estimated effect of1 liter infusate

1300ml
Drip rate= Drip rate= =54mL/hr
24h
Amount offluid
target number of hours

Add insensible & ongoing losses from polyuria:


Insensible losses= IO x 70 = 0.7 Lor -30 ml/h
Urine output= 167ml/hr
6. Compute for insensible losses= -IO mL/
kg/day (less if ventilated, more if febrile) Sample chart order:
• Start !VF 1LD5W at 250 cc/hour
• Please do serum sodium q6 hours
• Start desmopressin

140
POTASSIUM
I. HYPOKALEMIA
Defined as plasma potassium <3.6 mmol/L
Long-standing hypokalemia (usually from eating disorders or laxative abuse) may
predispose to acute kidney injury & lead to end-stage renal disease (it leads to
vacuolizing injury to proximal tubular cells, interstitial nephritis and renal cysts)

A. Etiopathogenesis: Causes ofHypokalemia


DECREASED
INTAKE
I CELLULAR
REDISTRIBUTION
I INCREASED LOSS
• Starvation • Metabolic alkalosis Non-renal losses:
• Clay ingestion • Insulin • Gastrointestinal loss (diarrhea)
• Beta2-adrenergic activation • lntegumentary loss (sweat)
(bronchodilators, tocolytics) Renal losses:
• Alpha-adrenergic antagonists • Increased distal delivery
• Thyrotoxic periodic paralysis (diuretics, osmotic diuresis, salt-
• Downstream stimulation wasting nephropathies)
ofNa/K ATPase pump • Increased K+secretion
(theophylline, caffeine) Mineralocorticoid excess
0

(primary hyperaldosteronism,
• Anabolic state: vitamin B12or Cushing, Bartter, Gitelman
folic acid administration, GM- syndrome)
CSF, TPN Apparent mineralocorticoid
0

• Familial hypokalemic periodic excess (Liddle syndrome,


paralysis licorice)
0 Distal deliveryofnon-reabsorbed

anions (vomiting, NGT suction,


proximal RTA, DKA)
Magnesium deficiency
0

Source:JamesonJL,et al. Harrison's


Principlesof InternalMedicine20thedition,2018

B. Manifestations
0History: medications, diet, symptoms pointing to a probable cause such as diarrhea
and periodic weakness
0Presents as cardiac (e.g., arrhythmias), skeletal (e.g., weakness, paralysis) and intestinal
(e.g., paralytic ileus) disturbances
0Predisposes to digoxin toxicity (reduced competition between K· and digoxin for
shared binding sites on cardiac Na·, K--ATPase subunits)

• Changes include: broad flat T waves, ST depression and QT


ECG prolongation (usually prominent at levels <2-7mmol/L)
• Once with ECG changes, the potassium deficit is around 400-800 mEqs
• An index reflecting the conservation ofK• in the renal collecting ducts
• Estimates the ratio of K• in the collecting duct lumen compared to that
in the peritubular capillaries, and thus useful in diagnosing causes of
hypo- or hyperkalemia
Transtubular
• TTKG >4 during hypokalemia points to renal loss
Potassium
• The transtubular potassium gradient (TTKG) can be computed using
Gradient (TTKG)
the following:
UrineK• SerumOsm
TTKG= X
SerumK• UrineOsm

• If value is <1.5mEq/mmol: hypokalemia is usually from poor dietary


Urine K· / intake, transcellular potassium shifts, gastrointestinal losses, or
Creatinine Ratio previous use of diuretics
• Higher values are indicative of ongoing renal potassium wasting
141
Hypokalemia
serum K•<3.5 mmol/l

Yes
Urgent treatment

Yes
No further workup

Yes Treat underlying


cause

No
Check urine K/crea ratio

Urine K/crea ratio Urine K/crea ratio


<15 mmol/g crea > 15 mmol/g crea

External lasses Renal loss Renal tubular acidosis


GI losses DKA
Metabolic
Poor intake Medications:
Check blood acidosis
Remote diuretics amphotericin B,
pressure/volume
acetazolamide

Magnesium deficiency
High Low or normal Variable
Non-reabsorbable ions
Mineralocarticoid excess
Renal artery stenosis Emesis
Cushing syndrome Metabolic Diureticuse
Congenital adrenal alkalosis Bartter syndrome
hyperplasia Gitelman syndrome

Source:JamesonJL,et al. Harrison's


Principles
of InternalMedicine
20thedition,2018

D. Management
1. Goals of Therapy:
• Prevent life-threatening and/or chronic consequences
• Replace the potassium deficit
• Correct the underlying cause and/or mitigate future hypokalemia

2. Replacing the Potassium Deficit


• Urgency of therapy depends on severity, associated clinical factors and rate of decline
• Urgent but cautious replacement in those with severe redistributive hypokalemia
(plasma K· <2.5 mM) and patients with serious complications
• Oral replacement is the mainstay of therapy
• Concomitant magnesium deficiency should always be corrected
Oral Replacement
I Equivalent Oral Replacement I Equivalent
1 cc 10% oral KC!solution 1.33mEqsK· KC! tablet 600 mg 8 mEqsK·
15 cc 10% oral KC!solution 20 mEqsK· KC! tablet 750 mg 10 mEqsK·
30 cc 10% oral KC!solution 40 mEqsK· 1 average-sized banana 11-12
mEqsK·

142
Correcting Hypokalemia
There is no hard & fast rule in choosing method of correction, always rely on clinical judgment!
• Careful monitoring of serum K· is needed during correction
STEPS IN CORRECTING K+ DEFICIT
I SAMPLE CASE

Sample Case I: A 24/M presented at the ER with bilateral lower extremity weakness. His
siblings share the same symptoms, which according to them occur usually after heavy
meals. Serum K· was noted to be 2.5 mmol/L.

1. Calculate for potassium deficit

K• deficit=
Desired K•-Actual K· XIOO K• deficit= 21..:::l..x 100 = 370 mEqs
0.27
0.27

• This formula is used for non-redistributive hypokalemia.


• For other cases, the potassium deficit is estimated to be a 400-800 mmol reduction for a
2.0 mmol/L fall in serum K•.
• Target K• for cardiac patients is usually 4.0; otherwise a target of 3.5 is used.

2. Oral K· correction is the preferred therapy • We can address the deficit using 10%
for hypokalemia. Since our patient can oral KC! solution (30 cc = 40 mEqs K·)
tolerate feeding, we use the oral route. • With this formulation, we can give 30 cc
of 10% oral KC! for a total of 9 doses QIO

3. The IV route is limited to patients • To speed up the correction (since patient


unable to utilize the enteral route or in the is symptomatic), we can correct using
setting of complications (e.g., paralysis, the oral and IV route simultaneously:
arrhythmia). 0 pNSS IL+ 30 mEqs KC! x 8 hrs for

• Use saline solutions rather than dextrose 3 cycles (equivalent to giving 90 mEqs/
(since dextrose-induced increase in insulin day),ANO
can acutely trigger hypokalemia) 0 Oral KC! 10% solution 30 cc TIO x
• Peripheral IV dose is usually 20-40 mmol 7 cycles(= 120 mEqs/day)
of KC! per liter; higher concentrations
can cause chemical phlebitis, irritation &
sclerosis (so use a central line)
• Ifhypokalemia is severe, IV KC! may be
given through a central vein with cardiac
monitoring at rates of 10-20 mmol/hr

Sample Case 2: A 65/M, hypertensive, came in due to lightheadedness. He is maintained on


combination antihypertensive medications with diuretics and was recently given sambong
capsules by his neighbor for kidney stones. BP was 100/50, HR 54. Serum K· was noted to be 1.5
mmol/L, Mg•· at 0.57 mmol/L. Stat 12LECG was noted to have AV block and flattened Twaves
with prolonged QTc interval.

1. Calculate for potassium deficit K• deficit= x 100 = 926 mEqs


0.27
2. Since patient has ECG changes, this would warrant use of IV K· correction for faster correction.
Oral K· can also be started as long as the patient can tolerate & is hemodynamically stable. Close
cardiac monitoring should be done during correction
• Via central IV line, start 20 mEqs KCI + 100 cc pNSS in a soluset to run for I hour x 2 cycles
• If no central line, may use peripheral IV line: IO mEqs KC!+ 100 cc pNSS for 3 hours x 4 cycles
• Recheck serum K· after fast correction and then regularly thereafter
• May continue IV KCL correction with PNSS IL+ 40 mEqs KC! x12 hours
• May also start 10% oral KC! solution 30 cc TIO

3. It is important to correct underlying hypomagnesemia since this can make hypokalemia


refractory to treatment Finally, address the causative factor for hypokalemia.
• Start MgSO4 5g in 250 cc O5W x 24hours
• Stop diuretics or shift to K•-sparing diuretics; stop herbal medications
143
II. HYPERKALEMIA
• Defined as plasma potassium <'!5.5mmol/L
• A decrease in renal potassium excretion is the most common cause
• Hyperkalemia from excessive K· intake is rare in patients with healthy kidneys due to
their adaptive capaciry to increase renal K· secretion

A. Etiopathogenesis: Causes ofHyperkalemia


INTRA-TO
"PSEUDO"
EXTRACELLULAR INADEQUATE EXCRETION
HYPERKALEMIA
I SHIFT I
• Cellular efflux • Acidosis Inhibition of the RAAS
(throm bocytosis, • Hyperosmolaliry • ACE inhibitors/ARBs/renin inhibitors
erythrocytosis, (radiocontrast, • Blockade of mineralocorticoid receptors
leukocytosis, in hypertonic dextrose, (spironolactone, eplerenone)
vitro hemolysis) mannitol) • Blockade ofENaC (amiloride, triamterene)
• Hereditary • Beta-adrenergic
Decreased distal delivery
defects in red antagonists
• CHF, volume depletion
cell membrane (noncardioselective
transport agents) Hyporeninemic hypoaldosteronism
• Digoxin and related • Tubulointerstitial disease (SLE, sickle
glycosides cell anemia, obstructive uropathy)
• Lysine, arginine, • Diabetes mellitus
aminocaproic acid • Drugs (NSA!Ds, COX-2inhibitors, beta
• Prolonged blockers, cyclosporine, tacrolimus)
immobilization • CKD, advanced age
• Rapid tumor lysis • Pseudohypoaldosteronism rype II
Renal resistance to mineralocorticoid
• Tubulointerstitial diseases
• Hereditary (defects in ENaC)
Advanced renal insufficiency

Primary adrenal insufficiency

B. Manifestations
' Clinical manifestations are predominantly cardiac in nature
0 Associated cardiac arrhythmias include sinus bradycardia, sinus arrest, slow
idioventricular rhythms, ventricular tachycardia, ventricular fibrillation and asystole

C. Diagnosis (other than measuring serum electrolytes)


DIAGNOSTIC I REMARKS
• Tall peaked T waves (at K· 5.5-6.5 mmol/L)
• Loss of P waves (at K· 6.5-7.5mmol/L)
ECG
• Widened QRS complex (at K· 7-8 mmol/L)
• Sinusoidal pattern (at K· <'!8mmol/L)

• IfTIKG >8: indicative of reduced tubular flow such as in reduced


Transtubular
ECFV or in advanced renal failure (GFR <20 mL/min)
Potassium
• IfTIKG <5: can be due to reduced distal K• secretion such as
Gradient
during tubular resistance to mineralocorticoids (drug-induced) or
(TTKG)
in hypoaldosteronism (DM, SLE, adrenal insufficiency)

144
Hyperkalemia
serum K• >5.5 mmol/L

Yes
Urgent treatment

Yes
No further workup

Yes
Treat underlying cause

No
Medications: spironolactone,
calcineurin inhibitors,trimethoprim
Pseudohypoaldosteronism
SLE
Sickle cell disease
Renal transplant
Tubulo-intersfitialdisease

Yes No

Hyperglycemia/OM
Primary adrenal insufficiency
Tubule-interstitial
disease
Medication: ACE inhibitors,ARB,,
Acute glomerulonephritis
heparin, ketoconazole
Medications:NSAIDs, beta-blockers

Source:Jameson JL, et al. Harrison'sPrinciplesof InternalMedicine20thedition,2018


D. Management
° First priority in the management is assessment of need for emergency treatment:
• ECG manifestations should be considered an emergency
• Those with plasma K· ;>:6.5,
even without ECG changes, should be managed aggressively
• Patients should be placed on continuous cardiac monitoring
1.Cardioprotection (againstthe arrhythmic effectsof hyperkalemia)
• Calcium raises the action potential threshold and reduces excitability without
changing resting membrane potential:
• Calcium gluconate IO mL of IO% IV over 2-3 mins with cardiac monitoring
• Calcium gluconate IO mL of IO% in IO0 mL D5W IV over 20-30 mins (to avoid
acute hypercalcemia which can potentiate digoxin toxicity)
• Effect starts in 1-3minutes and lasts 30-60 minutes
• Dose should be repeated if there is no change in ECG findings or if they recur
2. Potassium Excretion
THERAPY I IMPORTANT POINTS
• Sodium polystyrene sulfonate (SPS) exchanges Na· for K· in the GI
Cation
tract and increases fecal K· excretion
exchange
• Given as 15-30g premade suspension with 33% sorbitol
resins
• Full effect only afier 24 hrs & usually requires repeated dosing every 4-6 hrs
• Loop & thiazide diuretics can be utilized to reduce K• in volume-
Diuretics
replete or hypervolemic patients with sufficient renal function
• Hemodialysis is the most effective & reliable method to reduce plasma K·
Dialysis
• Peritoneal dialysis is considerably less effective
145
3. Cellular Redistribution (shifts K· inside the cells)
11-iERAPY' I DOSING I KINETICS I REMARKS
• Glucose-Insulin • Insulin causes extracellular
(GI) solution: K· to move intracellularly
50 mLof50% • Hypoglycemia is a common
• Effectin 10-20 rnins
dextrose in water side effect, therefore follow
Insulin • Peaksat 30-6o mins
(D50-50) + 10 with D10Wat 50-75 mL/h
• Lasts for 4-6 hrs
units regular • In those with glucose ~200-
insulin in 2-5 250 mg/dL, insulin should be
minutes given without glucose
• Salbutamol 10-20 • Effect in 30 mins
Beta • Use with caution in patients
mg in 4 mL pNSS • Peaks at 90 mins
agonists with cardiac disease
inhaled x 10 mins • Lasts for 2-6 hrs
• No role in routine treatment ofhyperkalemia
JV • NaHCO3 50-150
• Reserved for patients with hyperkalemia &
NaHCO3 mEqs+ 1LD5W
concomitant metabolic acidosis

CALCIUM
I. HYPOCALCEMIA
Calcium is central to normal cellular function and signaling and regulation of multiple
physiologic processes such as cardiac contractility, neuromuscular signaling, hormone
secretion and even blood coagulation
Extracellular calcium is kept within a narrow range by feedback mechanisms involving
parathyroid hormone (PTH) and active vitamin D

• Parathyroid agenesis (e.g., DiGeorge syndrome)


LowPTH • Parathyroid destruction: surgical, radiation, infiltration by
(hypoparathyroidism) metastases or systemic diseases, autoimmune
• Reduced parathyroid function (e.g., hypomagnesemia)
• Vitamin D deficiency or impaired 1,25(OH),D production/
action: nutritional, renal insufficiency
• PTH resistance syndromes
High PTH (secondary • Drugs [e.g., Ca 1 • chelators, inhibitors of bone resorption
hyperparathyroidism) (bisphosphonates, plicamycin), altered Vitamin D metabolism]
• Miscellaneous: acute pancreatitis, acute rhabdomyolysis,
hungry bone syndrome after parathyroidectomy, osteoblastic
metastases (e.g.,prostate cancer)

B. Clinical Manifestations
' Asymptomatic (if decreases in Ca'' are mild and chronic) or may present with life-
threatening complications
' Significant hypocalcemia presents with paresthesias due to neuromuscular irritability
' Chvostek's sign: twitching of circumoral muscles after gentle tapping of the facial nerve
anterior to the ear
'Trousseau's sign: carpal spasms induced by inflation of BP cuff to 20 mmHg above the
patient's SBP for 3 minutes
' Severe hypocalcemia can induce seizures, carpopedal spasm, bronchospasm,
laryngospasm and prolongation of QT interval in the ECG

146
nosis
Hypocalcemia
serum total Ca 2 • <2.1 mmol/l and/or
ionized Ca 2• < 1.16 mmol/l

Yes Renal failure


(secondary hyperparathyroidism}

No
Hypoparathyroidism
Yes Congenital/genetic
Acquired (post-surgical, post-
irradiation, hypomagnesemia}

Yes
Vitamin D deficiency

No
Yes
Vitamin0-resistant rickets

No
Pseudohypoparathyroidism
Vitamin 0-<lependent rickets

Source:SkoreckiK, et al. BrennerandRector'sTheKidney,10thEdition.Elsevier.2016


DM
MODE· 1 INDICATION I REMARKS
• Mild neuromuscular
Oral symptoms (e.g., • 1 tablet of 500 mg calcium carbonate contains
calcium paresthesia) 200 mg of elemental calcium
• Corrected Ca'· >I.9mmol/L

• IV calcium gluconate: preferred (less likely to


• Symptomatic cause tissue necrosis if extravasated)
(carpopedal spasms, • 1amp (10mL) of 10%Ca,. gluconate is equivalent to
tetany, seizures) I g Ca" gluconate & contains 93 mg elemental Ca"
• Prolonged QT interval • Given via slow infusion over 10-20 minutes,
IV • Asymptomatic + acute which may be diluted in dextrose water or saline
calcium decrease in corrected • IV solution should not contain HCO3 or phosphate
calcium to s1.9 mmol/L (this can precipitate into insoluble Ca'· salts)
• Patients who do not • Monitor response every 4-6 hours to maintain
improve despite oral serum Ca'- at 2-2.25mmol/L
supplementation • If with hypoalbuminemia and in renal failure,
monitor ionized Ca" instead

SCENARIO I CORRECTION
• Ca•- gluconate, IO mL 10% wt/vol (90 mg or 2.2 mmol) in 50 mL
Acute symptomatic 5% dextrose solution or pNSS, given IV over 5 mins
hypocalcemia
• Ca'· gluconate 10% solution IO mL 1-2amp SIVP (10-15mins)
Continuing • Constant IV infusion (IO amps calcium gluconate or 900 mg
hypocalcemia calcium in IL D5W or pNSS x 24 hours)
Hypocalcemia from • Elemental Ca'- 1,000-1,500 mg/day in divided doses
hypoparathyroidism • Vitamin 02 or 03 25,000-100,000 U daily or calcitriol [1,25(OH),D)
or vit-D deficiency 0.25-2 mcg/day (calcitriol 0.25 mcg/cap OD-BID)
Hypocalcemia • Treat concurrent hypomagnesemia first before correcting hypocalcemia
with concurrent as it is resistant to treatment if hypomagnesemia persists
hypomagnesemia • Correction of Mg" should be continued as long as it is <0.4 mmol/L
147
II. HYPERCALCEMIA

• Primary hyperparathyroidism (adenoma, hyperplasia, rarely


carcinoma)
Excessive PTH
• Tertiary hyperparathyroidism (long-term stimulation of PTH
production
secretion in renal insufficiency)
• Ectopic PTH secretion
• Overproduction of parathyroid hormone related peptide or
Hypercalcemia of
PTHrP (many solid tumors)
malignancy
• Lytic skeletal metastases (breast, myeloma)
Excessive • Granulomatous diseases (sarcoidosis, tuberculosis, silicosis)
1,25(0H),D • Lymphomas
production • Vitamin D intoxication
Primary increase • Hyperthyroidism
in bone resorption • Prolonged immobilization
Excessive calcium • Milk-alkali syndrome
intake • Total parenteral nutrition, iatrogenic, calcium supplements
• Endocrine (adrenal insufficiency, pheochromocytoma, VIPoma)
Others
• Medications (thiazides, vitamin A, antiestrogens)
Source:JamesonJL, et al. Harrison'sPrinciplesof InternalMedicine20thedition,2018
B. Dia nosis

Hypercalcemia
serum total Ca 2• >2.55 mmol/L and/or
I ionized Ca 2• > 1.32 mmol/L

Intact PTH level


I I
High1 1 Low

Urine Ca 2• excretion I PTHrP level


I

- High Primary or tertiary


hyperparathyroidism
High
- Hypercalcemia of
malignancy

- -
Lithiumtherapy
Low Low
Familial Calcitriol and
hypocalciuric calcidiol levels
hypercalcemia

I
I I I
j calcidiol, j calcitriol ! calcidiol, j calcitriol ! calcidiol, ! calcitriol
.
Vitamin D
Calcitrial overdose . Bone metastasis
Immobilization
overdose
Granulomataus
disease . Thyrotoxicosis
Milk-olkali
syndrome

Source:SkoreckiK, et al. BrennerandRector'sTheKidney,10thEdition.Elsevier.2016


148
CS
SEVERITY I VALUE* I MANIFESTATIONS**

• Usually asymptomatic
Mild • 2.75-2.88mmol/L, or • Vague neuropsychiatric symptoms (trouble
hypercalcemia • 11-11.5mg/dl concentrating, personality changes, depression),
peptic ulcer disease, nephrolithiasis or fracture risk

More severe • >3-3.25mmol/L, or • Lethargy, stupor or coma


hypercalcemia • >12-13mg/dl • GI symptoms (nausea, constipation, pancreatitis)
'Recall that the normalvalue for calciumis 2.10-2.55mmol/Lor 8.4-10.2mg/dl
"ECG changes include:bradycardia,AVblock,arrhythmiasand shortened QT-interval

Mild hypercalcemia
(Ca" <3 mmol/L) • Immediate treatment not required
• Adequate hydration (at least 6-8 glasses of water daily) to :
Asymptomatic or mildly minimize risk for nephrolithiasis ..
symptomatic
Moderate hypercalcemia
t-(_C_a_'_•
3_-_3_.5_m_m_o_l_iL_)
___ -1 • May start with saline hydration and bisphosphonates
Asymptomatic or mildly • Same precautions for mild hypercalcemia patients
symptomatic
• More aggressive therapy with volume expansion± loop
diuretics, calcitonin and concurrent bisphosphonates
• Saline hydration+ calcitonin = substantial reduction
Severe hypercalcemia
in serum Ca'· within 12-48 hrs
(Ca" >3-5mmol/L)
0 Bisphosphonate: effective by 2nd to 4th day
• Hemodialysis considered if serum Ca" 4.5-5 mmol/L and
if with neurologic symptoms with stable hemodynamics
'Avoidfactors that can worsen hypercalcemia:thiazides, lithiumcarbonate intake, volume depletion,
prolonged inactivity,and high Ca2• diet (>1000mg/day)

2. Description of Management
ASPECTS I REMARKS
• Volume expansion is the first line treatment ofhypercalcemia
Hydration
• Example: pNSS x 8 hours (may go up to 1-4liters in 24 hours)
• Use ofloop diuretics for hypercalcemia is not supported by trials &
has been criticized (however, they still remain an important tool in the
management ofhypercalcemia, especially for those with volume overload)
• Loop diuretics may be used to enhance Na• & Ca" excretion, but
Forced diuresis
should not be started until volume status restored to normal (e.g., urine
output should be >l00 cc/hr)
• Diuresis in a dehydrated patient may worsen hypercalcemia
• Example: furosemjde 20-40 mg IV QS-12hours
• Onset of action is delayed (may take up to 24-48 hours to take effect)
Bisphosphonates 0 Zoledronic acid 4 mg IV given over 15-30mins

0 Pamidronate 60-90 mg IV given over 2-4 hrs

• Used for life-threatening hypercalcemia (acts within a few hours)


Calcitonin
• Principally acts through osteoclasts, blocking bone resorption
• Steroids for malignancies (e.g., multiple myeloma, lymphoma)
• Dialysis for severe hypercalcemia complicated by renal failure
Others
• Calcimimetics for hyperparathyroidism & parathyroid malignancy
• Denosumab for refractory hypercalcemia
149
MAGNESIUM
Magnesium is the second most abundant intracellular cation
Important in different processes which include:
0 Energy transfer, storage and use
0Protein, carbohydrate and fat metabolism
0Maintenance of normal cell membrane function
0Regulation of PTH

I. HYPOMAGNESEMIA
Presents with muscular weakness, tremors, seizures, paresthesias, tetany, & nystagmus
ECG findings: prolonged QT interval, PVCs, torsades de pointes, & ventricular fibrillation
Usually coexists with hypokalemia and hypocalcemia
STEPS IN CORRECTING Mg 2 • DEFICIT I SAMPLE CASE

PR, 40/M, diagnosed case of CHF, presented with Mg'· of 0.5 mmol/L

1.Compute for the Magnesium Deficit


Mg" deficit= 1- 0.5 = 0.5 mmol/L
= Desired Mg" - Actual Mg"
,,
• Target Mg" is usually 1.0 mmol/L for those with cardiac conditions
• Otherwise, a target of o.8 mmol/L is used
• Correct deficit by starting MgSO4 drip: 5 gin
2. In correcting for the deficit, 1 g MgSO4 is
250 cc D5W x 24 hours (faster drip rates can
given per 0.1 mmol/L Mg'· deficit
be used if with no volume overload)

II. HYPERMAGNESEMIA
• Usually related to adrenal insufficiency & renal failure
• Severe cases may lead to decreased tendon reflexes and respiratory failure
• Treatment involves use of calcium gluconate, hydration and dialysis in severe cases

BICARBONATE
Usually given in acute & severe metabolic acidosis or pH <7.1(except hypercarbic acidosis), to
raise plasma HCO3· levels to -15 mEqs/L, as to improve cardiac inotropy & lactate utilization
Reacts with H· ions to form water & CO2 and acts as a buffer against acidosis by raising blood pH
Giving large amounts ofHCO3' can paradoxically depress cardiac performance, exacerbate
acidemia & may cause fluid overload, hypocalcemia, alkalosis and concomitant hypokalemia
NaHCO3 should not be given on a routine basis (correlate clinically)

CORRECTING BICARBONATE DEFICIT


I SAMPLE CASE

PC, 56/M, 75 kg, diagnosed case ofCKD Stage 5, lost to follow-up after initiation HD, presented
at the ER with dyspnea. ABG showed HCO3 of 9 and pH 6.9.
1. Compute for the estimated bicarbonate deficit.
HCO3 deficit= (12- 9) x 75 x 0.4
= (Desired HC03 -Actual HC03) x weight in kg x 04 =9omEqs

• Usual initial targec'(desired HCO3")is -I0-12 mEq/L, which should bring the blood pH to -7-20
• Subsequent goal is to increase the HCO3"to -15 mEq/L over the next 24 hours
2. There is no hard and fast rule in choosing method Sample order: Give NaHCO3 50 mEqs
of correction, always rely on clinical judgment! The IV bolus, and refer for hemodialysis
most important approach to metabolic acidosis is to (furthercorrection
may bedoneduringdialysis)
treat the underlying disorder May give half of dose Ifno risk for congestion or fluid
as IV bolus over 3-4 hours, and the remaining as drip overload & there will be a delay in
after re-assessment within 24 hours dialysis: Give NaHCO3 50 mEqs IV
bolus, then NaHCO3 50 mEqs diluted
in 250 mL D5W x 24 hours
Source:Koda-Kimble
M, et al. Handbookof AppliedTherapeutics.
LippincottWilliams& W1lk1ns,
2006
150
PERIOPERATIVE
EVALUA
APPROACH TO PERIOPERATIVE CARDIAC EVALUATION
1. Perioperative Cardiac Assessment
2. Preoperative Testing
3. Anesthesia for Patients Undergoing Surgery

QJ APPROACH TO PERIOPERATIVE PULMONARY EVALUATION


1. Perioperative Pulmonary Evaluation
2. Recommendations and Management
OVERVIEW
A focused history & physical exam is done to identify unstable or undiagnosed cardiac
conditions, estimate the risk of major adverse cardiovascular events (MACE), and
determine who may benefit from additional testing or revascularization prior to surgery
• Many patients undergoing major non-cardiac surgery are at risk for cardiovascular events
• Risk is related to patient- and surgery-specific characteristics

I. PURPOSE OF EVALUATION
The purpose of the preoperative evaluation is not to provide "clearance" but to perform
a comprehensive evaluation for perioperative and long-term benefit
Tests should be performed only if results will influence treatment

II. INITIAL APPROACH


• All patients for noncardiac surgery should be evaluated for the risk of cardiovascular
complications
Inquire for symptoms such as angina, dyspnea, syncope, palpitations
Elicit a history of ischemic/valvular heart disease, cardiomyopathy, hypertension,
diabetes, kidney disease, prior stroke, or peripheral arterial disease
A complete examination must be done, with emphasis on the cardiovascular system
• There is no "standard" set of laboratory tests for pre-op assessment (these depend on
patient characteristics)
• Performing routine screening tests in patients who are otherwise healthy is of little
value in detecting diseases and in changing the anesthetic management or outcome

III. OVERVIEW OF GUIDELINES FOR PREOPERATIVE CARDIAC RISK ASSESSMENT


CARDIAC ACC/AHA 2014 ESC/ESA 2014
EVALUATION I GUIDELINES I GUIDELINES
• Patients with known
cardiovascular disease
• All patients planning to
Who to evaluate? (CVD) or risk factors for
undergo noncardiac surgery
CVD planning to undergo
noncardiac surgery
• Combined patient-specific & , Surgical risk:
How to categorize surgical risk: 0 Low risk (<I%) ·
risk of MACE? 0 Low risk (<1%) 0 Intermediate risk (1-5%) '
0 Elevated (.er%) 0 High risk (>5%)

,RCRI
Risk calculators
,MICA ---
recommended
•ACSNSQIP
Functional • >4 METs subjectively or
• >4 METs subjectively
capacity goal objectively
ACC/AHA:AmericanCollegeof Cardiology/American HeartAssociation
ESC/ESA:European Societyof Cardiology/EuropeanSocietyof Anesthesiology

RCRI:Revised Cardiac Risk Index


MICA:MyocardialInfarctionand CardiacArrest
ACS-NSQIP:AmericanCollege of Surgeons-NationalSurgicalQualityImprovementProgram
Source:FleisherLA,et al.2014ACC/AHA
guideline
on penoperat1ve
cardiovascular
evaluation; 2014
Circulation.

153
IV. TOOLS FOR RISK STRATIFICATION
I REVISED CARDIAC
RISK INDEX (RCRI)
I GUPTA MICA I
ACS NSQIP SURGICAL
SCORE RISK CALCULATOR
SCORE (LEE INDEX)
• Procedureto be done
• Emergency case
• Hypertension,previous
cardiacevent
• Procedureto be done
• CHF,functionalstatus
(vascular,intraperitoneal,
• Diabetesmellitus
intrathoracic,or • Age
• Renalfailure,dialysis
suprainguinal surgery) • Surgerytype
• AmericanSocietyof
• Historyof ischemicheart • AmericanSocietyof
Anesthesiologists(ASA)
Components disease Anesthesiologists
PhysicalStatusClass
• Historyof CHF (ASA)class
• Woundclass
• Hisloryof strokefTIA • Functionalstatus
• Ascites
• Insulin-dependent • Creatinine>1.5mg/dl
• Systemicsepsis
diabetesmellitus
• Ventilator-dependence
• Creatinine~2 mg/dl
• Disseminated cancer
• Steroiduse
• Age& sex,BMI
• Dyspnea,smoker,COPD
Scoring • Onepointfor eachrisk • 25• percentile:0.05%
• 50• percentile:0.14%
• Webbasedcalculator
Interpretation • 0-1point:low risk(<1%) • 75• percentile:1.47% availableat www.
(and risk for • points:
elevated risk(7%) • go•
percentile:1.47%
riskcalculator.facs.org
MACEin%) • >3 points:highrisk(11%) • 95• percentile:2.60%
• 99• percentile:7.69%
• Validatedin diverse • Improvedprediction
• Internallyvalidatedin
Advantages settings for vascularsurgery&
diversepopulation
• Simpler& widelyused ambulatorysurgery
• Emergent& ambulatory
surgeriesexcluded
• Underestimates vascular
surgeryrisk • Notexternally
• Notexternallyvalidated
• Overestimates riskin validated
Disadvantage • Requireselectronicdevice
ambulatorysurgery • Requireselectronic
withinternetconnection
• Lackof universal device
serialmonitoringfor
postoperative
events
(e.g.,ECG,CK-MB)

V. SURGICAL RISK ESTIMATE ACCORDING TO TYPE OF SURGERY OR INTERVENTION


LOW RISK I INTERMEDIATE RISK I HIGH RISK
(<1%) (1-5%) (>5%)

• Superficial procedures • lntraperitoneal & intrathoracic • Aortic & major vascularsurgery


• Endoscopic procedures surgery • Adrenal resection
• Cataract surgery • Carotid endarterectomy • Pneumonectomy
• Breastsurgery (symptomatic) • Pulmonary or liver transplant
• Ambulatory surgery • Head and neck surgery • Repair of perforated bowel
• Dental surgery • Endovascular aneurysm repair • Revascularization or
• Eye surgery • Orthopedic surgery thromboembolectomy of
• Spine surgery lower limbs
• Renal transplant
• Prostatesurgery
Source:2009ACCF/AHA focusedupdate.Circulation;
2009.
154
STEPWISE APPROACH TO PERIOPERATIVE CARDIAC ASSESSMENT
Accurate assessment of perioperative risk is based on the initial determination of an
individual's risk factors combined with the risk and urgency of the surgical procedure

Patient with Risk Factors for Coronary Artery Disease Referred for Preoperative Evaluation

...
0..
w
V, Yes
Clinical risk stratification

...
0..
w
V,
_>----•I
Yes
Consider referral to Cardiology;
Evaluate and treat condition

M Proceed to

...
0..
w
V,
surgery

'<t
0..
w
li:;
.,, ~4 METS functional Capacity
0..
w Elevated risk
li:; <4 METS Functional Capacity

Refer to Cardiology

Source:FleisherLA,et al.2014ACC/AHA
guideline
on perioperative
cardiovascular
evaluation;
Circulation.
2014

STEP 1: DETERMINE THE URGENCY OF SURGERY


In patients with risk factors for or known CAD, determine the urgency of surgery
• If an emergency:
Determine the clinical risk factors that may influence perioperative management
0

Proceed to surgery with appropriate monitoring and management strategies


0

Those for emergency surgery are at increased risk of a perioperative cardiovascular event at
any level of baseline risk (risk indices derived from elective surgery cohorts are not accurate)

I
TYPE OF
DEFINITION
SURGERY

Emergency • Life or limb definitely threatened if not in the operating room (OR),
procedure with time for no or very limited clinical evaluation (typically <6 hours)
Urgent • Life or limb usually threatened if not in the operating room (OR),
procedure with time for limited clinical evaluation (typically 6-24 hours)

• One in which a delay of >I to 6 weeks to allow for an evaluation and


Time-sensitive
significant changes in management will negatively affect outcome
procedure
• Includes most oncologic procedures
Elective
• Procedure may be delayed for up to I year
procedure
Source:FleisherLA,et al.2014ACC/AHAguideline
on perioperative
cardiovascular
evaluation;
Circulation.
2014

155
STEP DETERMINE IF ACTIVE CARDIAC CONDITIONS ARE PRESENT
2:
• If surgery is urgent or elective: determine if the patient has an acute coronary syndrome (ACS)
If yes: then refer patient for cardiology evaluation & management according to guidelines
• Other conditions that may be considered active cardiac conditions:
0 Decompensated heart failure
0 High-grade arrhythmias
0 Hemodynamically significant valvular heart disease (e.g., severe aortic stenosis)
Except when emergency surgery is warranted, these conditions require adequate
evaluation and management prior to noncardiac surgery

STEP 3: ESTIMATE PERI OPERATIVE RISK OF MAJOR ADVERSE CARDIAC EVENTS (MACE)
Different guidelines have different recommendations regarding cardiac risk
stratification & testing (seepreviousdiscussion)to estimate the risk of MACE after surgery
RISK* I DEFINITION I RECOMMENDATION

• Combined surgical & patient characteristics • Can proceed to surgery


Low risk
predict a risk of MACE" <!% without additional testing

• Combined surgical & patient characteristics


Elevated predict a risk of MACE ;,1% • Should have their functional
risk • Includes both "intermediate" & "high" risk capacity in METs assessed
classifications (previous risk-stratification scheme)
• Riskcalculatorshave been developedto allowprecisecalculationof surgicalrisk(see discussionabove)
" MACEincludedeath or myocardialinfarction(Ml)

STEP 4: APPROACH TO PATIENTS WITH LOW RISK OF MACE (<1%)


• If the patient has a low risk of MACE (<I%): then no further testing is needed, and the
patient may proceed with surgery
Unnecessary evaluation for low-risk patients may yield results that could prompt
additional inappropriate invasive evaluation & intervention

STEP 5: APPROACH TO PATIENTS WITH ELEVATED RISK OF MACE


If patient has "elevated risk of MACE," then determine functional capacity
• Functional capacity is a reliable predictor ofperioperative & long-term cardiac events (e.g.,
those with reduced functional status preoperatively have increased risk of complications)
Can be estimated by:
0 History of activities of daily living
0 Objectively determined with exercise stress testing
• Often expressed in terms of metabolic equivalents (METs)
FUNCTIONAL I
CAPACITY
MET*
I ACTIVITIES

>10 METs • Strenuous sports such as swimming, singles tennis, football,


Excellent
basketball or skiing
Good 7-10METs • Climb a flight of stairs or walk up a hill
• Walk on level ground at 4 mph or running short distance
Moderate 4-6 METs • Moderate recreational activities (e.g., bowling, dancing,
doubles tennis, or throwing a baseball or football)

• Slow ballroom dancing


• Golfing with a cart
Poor .. <4 METs • Playing a musical instrument
,, • Light work around the house (e.g., dusting, washing dishes)
, • -Walking -2-3 mph
'1 MetabolicEquivalent(MET)is the restingor basal 02 consumptionof a 40-year-old,70-kgman
"Those withpoorexercisecapacity(<4 METS)are at increasedriskof perioperativecardiacevents,whereas those
withmoderate-excellent ("4 METS)functionalcapacitycan undergolowto intenmediaterisksurgerywithoutfurther
additionalcardiactestingin the absence of other cardiacindications.
Source:FleisherLA,et aL2014ACC/AHA
guidelineon penoperat1vecardiovascularevaluation;C1rculat1on.
2014
156
I. THOSE WITH GOOD FUNCTIONAL CAPACITY
Highly functional asymptomatic patients (2:4METs) may proceed with surgery without
further diagnostic testing (management will rarely change based on diagnostic results alone)

II. THOSE WITH POOR FUNCTIONAL CAPACITY


If the patient has poor (<4METs) or unknown functional capacity, the team should determine
whether further testing will impact patient decision making (e.g.,decision to perform original
surgery or willingness to undergo coronary revascularization prior to surgery)
Referral to respective specialists is often warranted

Patient with elevated risk and


poor or unknown functional
capacity

Yes No

Proceed with surgery*


.
Perform pharmacological
Consider alternative
stress testing
strategies*•

Is stress test
result normal?

No

Consider angiography ±
revascularization prior to
• Proceed with surgery surgery
Consider alternative
strategies*•

BoththeACC/AHAandESC/ESArecommend considering stresstestingfor patientswithelevated


risk (>1%)andpoorfunctionalcapacity(<4METs),if furthertestingwill impactdecisionmaking.
'For patientswithelevatedrisk& poor(orunknown)functionalcapacity& if furtherstresstestingwill
not impactdecisionmaking(e.g.,thereis an urgentneedfor surgery),proceedwithsurgeryaccord-
ing to GDMTor consideralternativestrategies.Patient& teammustbe well-informed of the clinical
assessment and risk.
"Alternativestrategiesmayincludenoninvasive
treatmentof the indicationfor surgery
Source:FleisherLA,et al. 2014ACC/AHA
guidelineon perioperative
cardiovascular
evaluation;
Circulation.
2014

157
PREOPERATIVE TESTING
DIAGNOSTIC
I COMMENTS

• Reasonable for those with known CAD, significant arrhythmias,


peripheral arterial disease, cerebrovascular disease, or other
structural heart disease
• May be considered for asymptomatic patients, except those
Resting 12-lead
undergoing low-risk surgery
ECG
• Rationale is based on having a baseline ECG should a
postoperative ECG become abnormal
• Routine preoperative resting 12-lead ECG is not useful for
asymptomatic patients undergoing low-risk surgical procedures
• Used to evaluate structural cardiac disease and left ventricular
ejection fraction (LVEF)
Resting • Reasonable for heart failure with worsening dyspnea or dyspnea
echocardiography of unknown origin
• May be considered in patients undergoing high risk surgery
,. • Routine preoperative evaluation ofLV function is not recommended
• Considered for patients with elevated risk & poor (<4 METs) or
unknown functional capacity (ifit will change management) to
assess functional capacity and detect myocardial ischemia (may be
Treadmill done with cardiac imaging)
exercise testing • Routine screening with noninvasive stress testing is not useful for
(TET) low-risk noncardiac surgery
• For patients with abnormal baseline ECGs (e.g., ST segment
abnormalities, left bundle branch block, frequent ectopy, or atrial
fibrillation),other modalities should be pursued (such as stress imaging)
··: -
• If unable to exercise, pharmacologic stress testing can be used
• Reasonable for those with elevated risk and poor functional
Pharmacological
capacity (if it will change management):
stress testing 0 Dobutarnine stress echocardiography

• Myocardial perfusion imaging


•Maybe done if indicated by an abnormal non-invasive stress test
Coronary
• Revascularization should follow existing guidelines
angiography
• Routine preoperative coronary angiography is not recommended
• Less invasive than angiography
CT angiography • Overestimates risk of vascular events in noncardiac surgery patients,
and is not recommended for perioperative risk stratification
• Preoperative NT-pro BNP for diagnosing or optimizing heart
failure may be done for patients:
0 ;,65 years old, or

BNP or NT- 0 45 to 64 years old with significant cardiovascular disease, or

proBNP • RCRI score ;,1


• Used as additional independent prognostic information for
perioperative and late cardiac events in high-risk patients
(RCRI >I for vascular surgery, RCRI >2 for other surgeries)
,_
• Used postoperatively in those with signs or symptoms of MI
• Used in high-risk patients both before &48-72hours after major surgery
• Used in patients:
0 2:65years old, or
Cardiac troponin
• 18 to 64 years old with significant cardiovascular disease or
positive BNP or NT-proBNP
0 Have qualified for BNP or NT-proBNP but were unable to have
I
the test performed
Source.FleisherLA,el al.2014ACC/AHA guidelineon penoperaUve
cardiovascular
evaluauon;
Circulation.
2014
RaslauMD,et.al.2020Preoperative
CardiacRiskAssessment,
MayoClinProc.2020
158
ANESTHESIA FOR PATIENTS UNDERGOING NON-CARDIAC SURGERY
I. GENERAL ANESTHESIA (GA)
Although laryngoscopy & intubation are considered the time of greatest stress and risk
for MI, extubation may actually pose a greater risk
Alternative methods for GA: face mask or laryngeal mask airway

II. ANESTHETIC AGENTS (no single best general anesthetic technique/or patients with CAD)
A. Inhalational Techniques
0All inhalational agents have reversible myocardial depressant effects and lead to
decreases in myocardial oxygen demand
AGENT I EFFECTS
• Negative inotropic effects and potent vascular smooth muscle
Isoflurane
relaxation and has minimal effects on baroreceptor function

Desflurane • Fastest onset of action and is commonly used in the outpatient setting
• Onset and offset of action are intermediate to those of isoflurane &
desflurane
Sevoflurane
• Major advantage is an extremely pleasant smell, which makes it the
agent of choice in children
.
B. High-Dose Narcotic Techniques
0Advantage: hemodynamic stability and lack of myocardial depression
° Frequently considered the "cardiac anesthesia" and advocated for use in all high-risk
patients, including those undergoing noncardiac surgery
0Disadvantage: requirement for postoperative ventilation
0Ramifentanil removed the need for prolonged ventilation, assists in early extubation
of patients undergoing cardiac surgery, and aids in managing short periods of intense
intraoperative stress in high-risk patients

C. Intravenous Propofol Sedation


0Alternative mode of delivering general anesthesia
° Can be used for both induction and maintenance of anesthesia
° Can cause profound hypotension because of reduced arterial tone with no change in HR
0Advantage: rapid clearance with few residual effects on awakening, but because it is
expensive, its current use tends to be limited to operations of brief duration
0Despite its hemodynamic effects, it has been used extensively to assist in early
extubation after CABG

D. Postoperative Analgesia May Reduce Perioperative Cardiac Morbidity


Effective postoperative analgesia may reduce cardiac complications because
0

postoperative tachycardia and catecholamine surges can promote myocardial ischemia


and/or plaque rupture

159
SECTION TWO
APPROACH TO PERIOPERATIVE PtJLMONARY EVALUATION

PERIOPERATIVE PULMONARY EVALUATION


Important to prevent post-operative pulmonary complications because it may prolong
the hospital stay by an average of1-2 weeks
Post-op pulmonary complications include pneumonia, respiratory failure with
prolonged mechanical ventilation, bronchospasm, atelectasis, and exacerbation of
underlying chronic lung disease

I. FACTORS TO ELICIT DURING THE PRE-OP PULMONARY EVALUATION


FACTORS I REMARKS
• Inquire about number of cigarette pack-years
• Long-term smoking damages the ciliated epithelium & the
Smoking tracheobronchial tree, resulting in bronchiolar obstruction & increased
history perioperative infection rates particularly in patients with COPD
• Three-fold decrease in the incidence of pulmonary complications after 8
weeks of smoking cessation
• Increased morbidity/mortality in patients with severe airflow obstruction
undergoing different types of surgeries
• Preoperative pulmonary preparation should be done by starting oral
COPD bronchodilator therapy,use of an expectorant,postural drainage, cough training,
nebulized bronchodilator therapy and antibiotics for purulent sputum
• A 2-week pre-op course of systemic corticosteroid is reasonable for
patients who continue to have symptoms despite bronchodilator therapy
• Increased incidence of intra-operative bronchospasm in asthma
• Aggressive management of asthma symptoms is recommended prior to
Asthma surgical procedures including treatment with corticosteroids if necessary.
• Before surgery, patients should be free of wheezing with peak flow >80%
predicted or personal best
- ,,
• Diminishes exercise capacity due to poor respiratory mechanics,
increased resistance within the respiratory system, poor respiratory
muscle function, small lung volumes, increased work and energy, cost of
Obesity breathing, control of breathing, and gas exchange.
• Despite these, obese patients do not have increased pneumonia,
bronchospasm, atelectasis, or respiratory failure compared to patients
with normal weight
• Patients with sleep apnea may have difficulty with tracheal intubation or
Obstructive airway management
sleepapnea • The use ofCPAP at home is protective for patients with sleep apnea prior
to operation

• Abdominal and thoracic surgical procedures decrease vital capacity and


functional residual capacity.
Type of
• Abdominal surgeries have a higher incidence of pulmonary
surgery
complications compared to non-abdominal surgeries (2x the risk for
upper abdominal surgery versus lower abdominal surgery)
Duration • Surgeries which last >30 minutes have a higher rate of pulmonary
of surgery complications

160
II. SUMMARY OF DIAGNOSTIC TESTS TO ORDER PRE-OPERATIVELY
TYPE OF CHEST I ARTERIAL BLOOD
SURGERY I X-RAY I SPIROMETRY
GAS*

• Smokers • Smokers and


Upper abdominal
• Patients with dyspnea, patients with
surgery
cough or exercise intolerance dyspnea
Lower abdominal, • Some with suspected
head &neck, lung disease or who need ...
•All
orthopedic surgery patients strenuous rehabilitation
Lung resection
• All patients
surgery
• Smokers
CABG
• Patients with dyspnea
'pC0 2 >45mmHg(onABG)is a strongriskfactorforcomplications
Source:American
Collegeof Phys1c1ans.
Preoperative
Pulmonary
Function
Testing.
AnnIntMed1990

RECOMMENDATIONS & MANAGEMENT


I. RISK REDUCTION STRATEGIES .
ASPECT I MANAGEMENT
• Include CPAP,incentive spirometry, deep breathing exercises & forceful
coughing
Lung • Decrease the incidence of post-operative pulmonary complications
expansion
maneuvers such as pneumonia, atelectasis, and prolonged hospital stay
• No difference in incidence of pulmonary complications among the
lung expansion maneuver strategies
• Pain aggravates pulmonary complications because it discourages
Pain control the patient from taking deep inspirations, coughing effectively and
cooperating during respiratory therapy

II. SUMMARY OF RECOMMENDATIONS


ASPECT I MANAGEMENT
• Obtain a complete history & exam focusing on pulmonary factors
• Chest x-ray in patients with active pulmonary disease
• Pulmonary function test in patients undergoing lung resection
procedures and in those who smoke and have known or suspected
Preoperative pulmonary disease
• Smoking cessation at least 8 weeks before surgery
• Aggressive management of asthma and COPD
• Treat all infections prior to operation
• Start teaching patients about lung expansion maneuvers
• Limit surgical duration to less than 3 hours
• Use spinal or epidural anesthesia
Intraoperative
• Avoid use of pancuronium
• Laparoscopic procedures are preferred
• Respiratory therapy should be initiated (deep breathing techniques
with forceful coughing, incentive spirometry)
Postoperative
• Use continuous positive airway pressure
• Adequate pain control and avoidance of oversedation

161
REFERENCES
1. Beecher H. The measured effect oflaparotomy on the respiration. J Clin Invest 1933;12:639-650.
2. Chalon J,Tayyab M, and Ramanathan S. Cytology of Respiratory Epithelium as the Predictor of Respiratory Complications after

Operation. Chest 1975;67:32-35.


3.Cohen ME, Ko CY,Bilimoria KY,Zhou L, Huffman K,Wang X et al. Optimizing ACS NSQIP modeling for evaluation of surgical
quality and risk: patient risk adjustment, procedure mix adjustment. shrinkage adjustment, and surgical focus. JAm Coll Surg.
2013Aug;217(2):336-46.e1
4.Djokovic J and Hedle-Whyte J.Prediction of Outcome of Surgery and Anesthesia in Patients over So. JAMA 1979;242:2301-23o6.
5.Eagle KA,Berger PB, Calk.insH, Chaitman BR, Ewy GA, Fleischmann KE, et al. ACC/ AHA Guideline Update for Periopermive
Cardiovascular Evaluation for Non Cardiac Surgery Executive Summary. A report or the American College or Cardiology/
American Heart Association Task Force on Practice Guidelines. Circulation. 2002 Mar,m5(10):1257-67.
6. Fleisher LA, Beckman JA, Brown KA, Calkins H, ChaikorEL, Fleischmann KE, et al. ACCF/AHA focused update on perioperative
beta blockade incorporated into the ACC/AHA 2007 guidelines on perioperative cardiovascular evaluation and care for
noncardiac surgery: A report of the American College of Cardiology Foundation/American Heart Association Task Force on
Practice Guidelines. J Am Coll Cardiol. 2014 Dec 9;64(22):e77-137.
7. Fleisher LA, Fleischmann KE, Auerbach AO, Barnason SA, Beckman JA, Bozkurt B, et al. ACC/AHA guideline on perioperative
cardiovascular evaluation and management or patients undergoing noncardiac surgery: a report or the American College or
Cardiology/American Hean Association Task Force on Practice Guidelines. Circulation. 2014;130:e278-e333
8.Ford MK. Beattie WS, and Wijeysundera ON. Systematic review: predic1ion of perioperative cardiac complications and mortality
by the revised cardiac risk index. Ann Intern Med. 2010Jan;152(1):26-35.
9.Glance LG, Lustik SJ, Hannan EL, Osler TM, Mukamel DB, Qian Fetal. The Surgical Mortality Probability Model: derivation
and valida1ion of a simple risk prediction rule for noncardiac surgery. Ann Surg. 2012Apr,255(4):696-702.
10. Gracey D, Divertie M, and Didier E. Preoperative Pulmonary Prepara1ion or Patients with Chronic Obstructive Pulmonary
Disease. Chest 1979;76: 123-129.
II. Gupta R, Parvizi J, Hanssen A, and Gay P. Postoperative complications in patients with obstructive sleep apnea syndrome
undergoing hip or knee replacemenr: A case-control study. Mayo Clin Proc. 2001;76: 897-905.
12. Howell SJ and Vohra RS. Perioperative Managemenr or Patients Undergoing Non-Cardiac Vascular Surgery. Eur J Vase
Endovasc Surg. 2007 Dec; 34(6):625-31.
13. Iskandrian AE and Garcia EV.(eds) Nuclear Cardiac Imaging. Principles and Applications. 5th Ed. UK: Oxford University Press,
2015.
14. Jameson JL, Kasper DL, Longo DL, Fauci AS, Hauser SL, Loscalzo J. Harrison's Principles of Internal Medicine. 20th Edition.
New York:McGraw Hill Education, 2018.
15. Kabalin C, Yamold P, and Grammer L Low complication rate of conicosteroid- treated asthmatics undergoing surgical
procedures. Arch Intern Med 1995;155:1379-1384.
16. Kenai MD, Boersma E, Bax JJ,Heijenbrok-Kal, MH, Hunink MGM, L'talien GJL, e1 al. A meta-analysis comparing the
prognostic accuracy of sLx diagnostic tests for predicting perioperative cardiac risk in patients undergoing major vascular
surgery. Heart 2003 Nov,89 (11):1327-1334.
17. Kristensen SD, Knuuti J, Saraste A, Anker S, Betker HE, De Hert S, et al. The Joint Task Force on non-cardiac surgery 2014ESC/
ESA Guidelines on non-cardiac surgery: cardiovascular assessment and management Eur Heart J. 2014 Sep 14;35(35):2383-431.
18. Lawrence VA, Dhanda R, Hilsenbeck SG, and Page CP. Risk of pulmonary complications after elective abdominal surgery.
Chest 1996;uo:744-50.
19. Loadsman JA and HiUman DR. Anaesthesia and sleep apnoea. Br J Anaesth 2001;86: 254-266.
20. Pad ma Sand Sundaram PS. Current Practice and Recommendation for Pre-surgical Cardiac Evaluation in Patients Undergoing
Non Cardiac Surgeries. World Journal of Nuclear Medicine. 2014Jan-Apr; 13(1):6-15.
21. Phillips E, Carroll B, Fallas M, and Pearlstein A. Comparison or laparoscopic cholecysteccomy in obese and non-obese patienrs.
The American Surgeon 1994;60:316-321.
22. Smetana GW. Pre-operative Pulmonary Evaluation. The New England Journal or Medicine. 1999Mar, 340 (12):937-944.
23. Warner M, Divenie M, and Tinker J. Preoperarive Cessation of Smoking and Pulmonary Complications in Coronary Artery
Bypass Patients. Anes1hesiology 1984;60:380-383.
24. Weinstein Hand S1eingan R. Myocardial Perfusion Imaging for Preoperative Risk Stratification. JNucl Med 2011;52=750-76o.
25. Wighrman JA A prospective survey ohhe incidence of postoperative pulmonary complications. Brit J Surg 1968;55:85-91.
26. Zipes DP, Libby P, Bonow R, Mann DL, Tomaselli GF. Braunwald's Heart Disease: A Te.xtbook of Cardiovascular Medicine. mh
Edition. Elsevier/Saunders, 2019.

162
CARDIOLO
QJ APPROACH TO DISEASES OF THE CARDIOVASCULAR SYSTEM
1. Approach to Common Cardiovascular Complaints
2. Common Diagnostic Tests in Cardiology

0 DYSLIPIDEMIA

HYPERTENSION
L:J 1. Diagnosis and Management of Hypertension
2. Hypertensive Crisis
3. Hypertension in Pregnancy

0 HEART FAILURE
1. Overview of Heart Failure
2. Management of Chronic Heart Failure

0 CHRONIC CORONARY SYNDROMES


1. Approach to Diagnosis of Chronic Coronary Syndromes
2. Management of Chronic Coronary Syndromes

0 ACUTE CORONARY SYNDROMES


1. Non-ST-Elevation Acute Coronary Syndrome
2. ST-Elevation Myocardial Infarction

RHEUMATIC FEVER AND VALVULAR HEART DISEASE

VENOUS THROMBOEMBOLISM

[!] OTHER DISORDERS OF THE CARDIOVASCULAR SYSTEM


1. Atrial Fibrillation
2. Pericarditis
3. Cardiac Tamponade
4. Peripheral Artery Disease
APPROACHTO DISEASESOF THE CARDIOVASCULARSYSTEM
APPROACH TO COMMON CARDIOVASCULAR COMPLAINTS
I. CHEST PAIN
Most common complaint of patients with cardiac conditions
• Causes of chest pain may be grouped according to the urgency of management:
DISORDER
I DURATION I QUALITY
I ASSOCIATED FEATURES

I) PainDue to an Acute or Life-ThreateningConditionthat NeedsE'!'ergentManagemen_t


• Pressure, tightness, • Symptoms even at low levels
Acute
squeezing, heaviness of exertion or at rest
coronary
• >I0-30 mins • Retrosternal with • Less relief with nitrates
syndrome
radiation to neck, jaw, • ECG & cardiac biomarkers
(ACS)•
shoulders, or arms determine approach
• Sudden • Sharp, tearing, knife- • Usually with hypertension
Aortic onset of like or ripping pain and/or underlying connective
dissection unrelenting • Radiation to the back tissue disorder (e.g., Marfan
pain between the scapulae Syndrome)

Pulmonary
embolism
• Sudden
onset
• Pleuritic for small
emboli (larger emboli
usually with dyspnea)
• Often unilateral on
• Most common symptom is
dyspnea
• Tachypnea and tachycardia
• Iflarge em bolus: hypotension
I
the side of embolism due to RV dysfunction
• Pleuritic • Dyspnea
Spontaneous • Sudden
• Unilateral on the side • Chest lag and decreased
pneumothorax onset
of pneumothorax breath sounds ipsilaterally
• Gradual • Sharp (similar to • Commonly associated with
Cardiac progression pericarditis), relieved tuberculosis or malignancy
tamponade • Sudden by sitting forward • Beck's triad: low BP, elevated
deterioration • Retrosternal )VP, muffled heart sounds
Esophageal • Sudden • Sharp, excruciating • May be preceded by straining,
rupture onset • Retrosternal retching, or vomiting
2) Pain Due to a ChronicConditionwith Potential SeriousComplications
• Pressure, tightness,
Chronic • Precipitated by exercise, cold
squeezing, heaviness
coronary weather, intense emotion or
• 2-10 mins • Retrosternal with
syndrome stress
radiation to neck, jaw,
(CCS)• • Relieved by rest or nitrates
shoulders, or arms
• Easy fatigability or syncope
Aortic
• Variable • Similar to ACS/CCS • Late-peaking systolic murmur
stenosis
radiating to carotids
• Exertional dyspnea
Pulmonary • Pressure
• Variable • Elevated )VP, parasternal lift
hypertension • Substernal
(RVH), and loud P2
*Coronary
arterydisease(CAD)is a dynamicprocess
of atherosclerotic
plaqueaccumulation.
Clinical
presentations
of CADmaybecategorized asACSor CCS.
Sources:ZipesDP,et al. Braunwald's HeartDisease,11thedition.2019
JamesonJL,et al. Harrison's
Principles
of InternalMedicine20thedition,2018
McconaghyJR. "Outpatient
evaluationof theadultwithchestpain."UpToDate, 2021
165
• Sharp pain in the retrosternal • Relieved by sitting up and leaning forward
Acute
area, which may radiate to left • Exacerbated by inspiration & lying supine
pericarditis
shoulder/trapezius ridge • Pericardia! friction rub
• Pleuritic pain, unilateral on • Dyspnea, productive cough, fever, rales,
Pneumonia
the side of involved lobe/lung occasional pleural friction rub
Herpes • Sharp or burning pain in
• Vesicular rash
zoster dermatomal distribution
4) Pain Due to Other Chronic Treatable Conditions
Esophageal • Burning pain in epigastrium • Worsened by recumbency after meals
reflux radiating to substernal area • Relieved by antacids & nitrates
Costo- • Reproduced by pinpoint pressure/palpation
chondritis • Variable, intense fleeting • No swelling of costochondral joints
Tietze pain in the sternal/chest wall • With tender swelling of costochondral
syndrome joints
Psychiatric • Variable fleeting pain • Situational factors precipitate symptoms

II. OTHER CARDIOVASCULAR COMPLAINTS


SYMPTOM I DESCRIPTION I DIFFERENTIALS
• Shortness of breath or "air hunger"
• Heart failure
• Cardiac causes of dyspnea usually
• CCS/ACS (angina! equivalent)
occur with exertion
• Pericardia! disease (e.g.,tamponade,
• Orthopnea: dyspnea when lying flat
Dyspnea constrictive pericarditis)
due to redistribution of blood from
• Pulmonary disease (e.g., asthma/
lower extremities to the lungs
COPD, tuberculosis, lLD)
• PND: dyspnea 2-4 hours after sleep,
• Deconditioning, psychological
compelling patient to sit upright
• An excess of interstitial fluid that • Heart failure
becomes clinically evident • Chronic venous insufficiency
Edema
• Imbalance between hydrostatic & • Renal insufficiency
oncotic pressures • Chronic liver disease
• Awareness of one's heartbeat, described • Atrial fibrillation
Palpitations
as pounding or skipped beats • Brady-/tachyarrhythmias
• Cardiac syncope usually occurs • Vasovagal syncope
Syncope suddenly, with a rapid restoration of • Orthostatic hypotension•
full consciousness after • Structural heart disease, arrhythmia
• Central cyanosis: due to right-to-left • Cyanotic congenital heart disease
shunting (e.g., tetralogy ofFallot)
• Peripheral cyanosis: due to reduced • Severe heart failure or shock
Cyanosis
extremity flow from vasoconstriction • Very high vasopressor doses
• Differential cyanosis: cyanosis affecting • PDA with right-to-left shunt
lower extremities, but not the upper reversal (Eisenmenger physiology)
• Pain/discomfort in affected muscle • Atherosclerotic peripheral arterial
Claudication
group with exercise;& resolveswith rest disease (PAD)
'Orthostatichypotension:
fallin SBP2:20or DBP2:10mmHgwithin3 minutesof assuming
theupright
posturefroma supineposition
166
PHYSICAL EXAMINATION OF THE CARDIOVASCULAR SYSTEM
I. TAKING THE BLOOD PRESSURE
KEY STEPS I COMMENTS
• Patient should be relaxed, sitting in a chair with arms supported (e.g.,
Prepare the resting on a desk) and both feet on the floor
patient • Abstain from coffee, exercise, & smoking for at least 30 minutes prior
• No talking during BP measurement
Use proper • Middle of the cuff is positioned on the upper arm at the level of the right
technique atrium (or midpoint of the sternum)
for BP • Cuff length & width should be 80% & 40% of arm circumference, respectively
measurements • May use either diaphragm or bell of the stethoscope for auscultation
• At first visit, measure BP in both arms (use the arm that gives the higher
reading for subsequent measurements)
• Separate repeated measurements by 1-2 minutes
• Use a palpated estimate of radial pulse obliteration pressure to estimate
Take proper systolic blood pressure (SBP); & inflate cuff 20-30 mmHg above expected SBP
measurements • Deflate the cuff pressure by 2-3 mm Hg per second
• First audible sound (Korotkoff I) is recorded as SBP; disappearance of all
sounds (KorotkoffV) is recorded as diastolic blood pressure (DBP)
• Column or dial should be read to nearest 2 mmHg
• Use an average of 22 readings obtained on 22 occasions to estimate the BP
Source: WheltonPK, et al. 2017 ACC/AHAHighBloodPressure CPG. J Am CollCardiol2018

II. INSPECTION
INSPECTION I FINDINGS/ DIFFERENTIALS
Findings on • Check for cyanosis, signs of bleeding (ecchymosis, petechiae) or tendon
the skin xanthomas (suggestive of familial hypercholesterolemia)
• LV apex may be visible in the 5th JCS left midclavicular line (visible
Findings on pulsations elsewhere are abnormal)
the chest • Precordial bulge/impulse: abnormal pulsations originating from the heart
or great vessels on chest wall (e.g., chamber enlargement, aortic aneurysm)
• Clubbing: indicates central shunting or pulmonary disease
Findings
• Endocarditis: Janeway lesions, Osier's nodes, splinter hemorrhages
on the
• Pedal/lower extremity edema: may be indicative of heart failure if
extremities
associated with an elevated )VP
• Provides an estimation of right atrial (RA) pressure (or central venous
Measuring
pressure), best examined with patient recumbent & head tilted at a 45° angle
jugular
• Measured as the vertical distance between the highest point of internal
venous
jugular venous pulsation and the sternal inflection point (angle of Louis)
pressure
• )VP distance >3.0-4.5 cmH2O is abnormal
(JVP)
• Normal venous pressure should fall (by at least 3 mmHg) with inspiration
Sources: Z1pesDP,et al. Braunwald'sHeart Disease. 11th ed1t1on.Elsevier/Saunders,2019

Relationship of JVP and Central Venous Pressure (CVP)


To compare clinical )VP & invasively measured CVP, convert the )VP (in cm H2O) to mmHg:
, Step 1) Measure )VP above the sternal angle and add 5 cm (estimated distance between
angle of Louis and mid-right atrium), which gives the CVP in cmH2O
• Step 2) Convert this value to mmHg by dividing by 1.36 (because 1.36cm H2O = 1.0 mmHg)

Example: )VP of8 cm gives a CVP ~IO mmHg (Formula: 8 + 5 = 13 cm; dividing this by 1.36
gives ~IO mmHg). The normal CVP ranges from 3-8 mmHg. An elevated CVP (>12mmHg)
may suggest fluid overload, RV/LV failure, etc. Note that using the sternal angle of Louis as
·a reference point systematically underestimates the CVP.
167
III. PALPATION
Palpation begins with patient supine at a 30° angle or in left lateral decubitus position
0 Right ventricle (RV): feel for RV with the heel of the hand at the left parasternal area
0 Left ventricle (LV):feel for the LV with the fingers across the chest (under the breast),
noting the point of maximal impulse or PMI (apex felt on the fingertips)
Palpation of arteries: should include the rhythm & fullness of the carotid artery pulse &
other peripheral pulses (radial, brachia!, femoral, popliteal, and dorsalis pedis pulses)

• Located in the 5th intercostal space, left midclavicular line


• PMI: normally over the LV apex beat & should be located in the 5th
NormalLV
JCS left midclavicular line as well
apex
• Normal apex beat: localized systolic outward thrust, <2.0-2.5cm diameter
• In LV enlargement, the PM! shifts laterally and downwards
• Upward lift best felt with the heel of the hand
Heaves • LV enlargement: sustained systolic lift in the left anterior chest wall
• RV enlargement: sustained systolic lift in the left parasternal area
• "Palpable murmur" due to turbulent & high-velocity blood flow
Thrills
• Best appreciated with the part of the palm overlying the MCP joints
PeripheralPulses
Pulsus parvus
• A weak & delayed pulse that suggests severe aortic stenosis
ettardus
• Two systolic peaks can be appreciated
Bifid pulse • Described in hypertrophic obstructive cardiomyopathy (HOCM) and
high-output states (e.g., severe aortic regurgitation, sepsis)
• Refers to a fall in SBP >IO mmHg with inspiration
Pulsus
• Present in cardiac tamponade, massive pulmonary embolism, hemorrhagic
paradoxus
shock, severe obstructive lung disease, tension pneumothorax
Pulsus • Beat-to-beat variability of pulse amplitude seen in severe systolic
alternans heart failure
Corrigan or • Abrupt upstroke with rapid fall-off; usually visible in the carotids
water-hammer • Usually also with wide pulse pressure
pulse • Seen in chronic severe aortic regurgitation (AR)
Sources:ZipesDP,et al. Braunwald's
HeartDisease.11thedition.Elsevier/Saunders,2019
JamesonJL,et al. Harrison's
Principles
of InternalMedicine20thedition,2018

IV. AUSCULTATION
Auscultate for heart sounds, murmurs and bruits in peripheral arteries (e.g., carotids,
abdominal aorta, femoral)
Use the bell of stethoscope for low-pitched sounds, & the diaphragm for high-pitched sounds

A. Common Auscultatory Areas of the Heart


AREA I LOCATION
Aortic area • md intercostal space (JCS) at the right sternal border
Pulmonic area • 2nd resat the left sternal border
Tricuspid area • Area between the 3rd to 5th JCS at the left sternal border
Mitra! area • Near the apex of the heart at the 5th resat left midclavicular line

168
B. The Heart Sounds
SOUND I DESCRIPTION I FINDINGS
• Louder in hyper kinetic states, short PR-
1st heart • Coincides with closure of the
intervals, and early rheumatic MS
sound mitral and tricuspid valves, best
• Softer with LV dysfunction, B-blocker
(S1) heard at the apex
use, long PR-intervals, & late MS
• Best heard at the base of the heart
2nd • Caused by the closure of the
• Widened A2-P2interval: RBBB,severe MR
heart aortic (A2) & pulmonic (P2)valves
• Narrow split or single S2:pulmonary HPN
sound • Splitting is normally heard upon
• Fixed splitting (constant throughout
(S2) inspiration, with P2 coming after A2
respiratory cycle): ASD
3rd heart • Coincides with early diastole or • S3 gallop in heart failure, caused by
sound phase of rapid ventricular filling flow of blood during rapid ventricular
(S3) • Heard right after S2 filling
4th • Coincides with late ventricular • Occurs when diminished ventricular
heart diastole or atrial systole (atrial compliance increases the resistance to
sound contraction/slow ventricular filling) ventricular filling
(S4) • Heard right before S1 • More common in LVH or active Ml

C. Common Murmurs
DESCRIPTION
I DIFFERENTIALS

Systolic Murmurs
Early decrescendo systolic murmur • Acute severe mitral regurgitation (MR)
Early systolic murmur that increases in
• Tricuspid regurgitation (TR)
intensity with inspiration (Carvallo sign)
Midsystolic ejection murmur • Aortic stenosis (AS)or pulmonic stenosis (PS)
Systolic click+ mid-to-late systolic murmur • Mitra! valve prolapse (MVP)
Holosystolic murmur • Ventricular septa! defect (VSD), MR or TR
Diastolic Murmurs . ...... ,,·•. .' - .. 1:1
.
Soft, early diastolic murmur • Acute severe aortic regurgitation (AR)
Decrescendo, blowing diastolic murmur • Chronic severe AR
• In left sternal border 0 AR from a primary valvular pathology

• In right sternal border 0 AR from aortic root dilatation

Mid-to-late diastolic murmur or rumble


• Mitra! stenosis (MS)
with opening snap
·Continuousmurmurs:.lieardinP:DA(classicexample),ruptureismus o.fVaJs.aiva
fistula,cezyicalvenoushwn;1l1ll1l11tl,ary
aneurysm,m:teriovenous souffleof~gnancy

• Faint; needs concentration


2 • Faint but can be heard readil b an ex erienced observer NO
3 • Moderate! loud
4 • Loud
5 Very loud; heard with stethoscope lightly pressed on the skin YES
6 • Exce tionall loud; heard with stethosco e sli htl above chest wall
169
E. Other Abnormal Heart Sounds
SOUND
I DESCRIPTION

Opening
• High-pitched and occurs after a very short interval following S2 in MS
snap (OS)

• Low-pitched mid-to-late apical diastolic murmur of chronic severe AR


Austin flint
• Due to regurgitant jet from aortic valve striking the anterior leaflet of mitral valve
murmur
• Can be confused with MS
Graham • High-pitched, decrescendo, diastolic blowing murmur along left sternal border
Steell • Heard in pulmonic regurgitation (PR) secondary to dilatation of the RV
murmur outflow tract
• Leathery or scratchy three-component sound: ventricular systole, rapid early
Pericardia!
diastolic filling, and late presystolic filling
friction rub
• -100% specific for diagnosis of acute pericarditis

Pericardia! • High-pitched diastolic sound that corresponds to the abrupt cessation of


knock ventricular expansion in constrictive pericarditis
• Lower-pitched diastolic sound in atrial myxoma, caused by prolapse of the
Tumor plop
mass through mitral orifice

MAKING A CARDIAC DIAGNOSIS


ELEMENT I COMMENTS
Etiology • Congenital, hypertensive, ischemic, or inflammatory?
• Which chambers are involved? Are they hypertrophied, dilated, or both?
Anatomic • Which valves are affected? Are they regurgitant and/or stenotic?
abnormalities • Is there pericardia! involvement?
• Has there been a myocardial infarction?
Physiologic • ls an arrhythmia present?
disturbances • Is there evidence of congestive heart failure or of myocardial ischemia?
Functional • How strenuous is the physical activity required to elicit symptoms?
disability • See table below

ASSESSMENT OF FUNCTIONAL CAPACITY (FC)


New York Heart Association Functional Classification (NYHA FC)
FC I DESCRIPTION I GENERAL GUIDE I EXAMPLES
• Can do outdoor work
• Symptoms occur • No limitation of physical
(e.g., shoveling, carrying
with greater than activity
I ordinary physical • Can climb~ 2 flights of
heavy objects), recreational
activities (e.g., basketball),
activity stairs with ease
jogat5 mph
• Symptoms occur • Slight limitation of activity • Can do gardening, walk
II with ordinary
physical activity
• Can climb 2 flights of stairs on level ground, have sex
but with difficulty without stopping
• Symptoms occur with • Marked limitation of • Can shower or dress up
III less than ordinary physical activity without stopping, do indoor
physical activity • Can climb s I flight of stairs cleaning, play bowling/golf
• Symptoms may be • Unable to carry on activity • Cannot do above activities
IV present even at rest without symptoms • Dyspnea at rest
Source:HurstJW.DisMon.1990& NewYorkHeartAssociation. CriteriaCommittee.1994
ZipesDP,et al. Braunwald's
HeartDisease.11thedition.Elsevier/Saunders,
2019
170
COMMON DIAGNOSTIC TESTS IN CARDIOLOGY
DIAGNOSTIC I DESCRIPTION
Chest radiograph • Widely used to visualize the heart and lungs

• Graphic recording of electric potentials generated by the heart


12-lead
to detect arrhythmias, conduction disturbances, and ischemia
electrocardiogram (ECG)
• See ECGchapterfor discussionon stress(ECG)testing•
24-hour ambulatory • Device that allows continuous recording of BP (usually at
blood pressure 30-minute intervals) measurements even outside of the clinic or
monitoring (ABPM) hospital setting
• Continuous ECG rhythm pattern recorded for 24 hours or more
Holter monitoring
to document paroxysmal or intermittent rhythm abnormalities
• Uses ultrasound waves to visualize heart chambers and valves
• Doppler studies can also visualize blood flow through the heart
Transthoracic
• Stress echocardiography•: hallmark of ischemia during stress
echocardiography (TTE)
echocardiography is the development of new regional wall
motion abnormalities and reduced systolic wall thickening

• Echocardiography that uses a specialized probe with an


Transesophageal
ultrasound transducer introduced into the esophagus to more
echocardiography (TEE)
accurately visualize posterior structures of the heart
Coronary computed • Offers non-invasive anatomic data of the coronary arteries to -
tomography detect stenoses
angiography (CTA) • Also useful in congenital coronary anomalies
• Simplest application of cardiac CT without the need for
administration of contrast material
Coronary CT calcium
• Quantification of coronary calcium (Agatston score):
scoring
• Minimal: 0-10 0 Moderate: 100-400

0 Mild: 10-100 • Severe: >400


• Comprehensive exam for assessment of size, morphology,
Cardiac magnetic
function, and tissue characteristics
resonance imaging
• Patterns oflate gadolinium enhancement (LGE) can aid in
(cMRI)
differentiating between ischemic & non-ischemic cardiomyopathies
• Uses radioisotopes (technetium or thallium) taken up by viable
Myocardial perfusion myocardial cells to assess perfusion, viability, and function (i.e.,
imaging (MPI) or hypoperfused or dead myocardium takes up less isotope)
radionuclide imaging • To stress heart to detect inducible ischemia, exercise (treadmill) or
pharmacologic vasodilators (dipyridamole or adenosine) are used•

• Determines patency & configuration of the coronary artery


Coronary angiography
lumen by injecting contrast material into the coronary arteries

• Uses invasive monitoring & blood sampling through a catheter


Cardiac catheterization inserted into the heart to determine function, pressures, oxygenation,
flow & volume of blood within chambers & great vessels
• Electrophysiological test of the heart involving a catheter
Electrophysiology study
with intracardiac electrodes probing the endocardium to test
(EPS)
conduction pathways and electrical activity
testing: non-invasive
*Stress tooltoevaluate to exercise
response controlledconditions(usuallyto
under
detect
CAD),usually donewithanECG,echocardiography, MPI,orcMRI
• Exercisestresstest:leastinvasive,
makesuseof.atreadmill
• Phannacologic stresstest:drugsareusedtoinducestress
-
171
OVERVIEW OF DYSLIPIDEMIA
Defined as an elevation of TC, LDL cholesterol (LDL-C), triglycerides or lipoprotein levels
It is a significant risk factor in the development of atherosclerosis, with the fatty streak
representing the initial lesion of plaque formation
Typically exhibit no symptoms, and only discovered during routine screening or when
working up for atherosclerotic cardiovascular disease (ASCVD)
Guidelines recommend use of fasting or non-fasting lipid profile (both equally estimate riskfor ASCVD)

I. THE LIPID PROFILE

TC = HDL + Non HDL • TC:totalcholesterolin mg/dl


• HDL:highdensitylipoproteinin mg/dl
= HDL + [LDL + VLDL] • LDL:lowdensitylipoproteinin mg/dl
TG • VLDL:verylow densitylipoproteinin mg/dl
=HDL+ [LDL+- 5-J • TG:triglyceridesin mg/dl

Source:Fnedewald
WT,et al. ClinicalChem,1972
II. NON-HIGH DENSITY LIPOPROTEINS (NON-HDL)
Composed ofLDL & VLDL
Lower non-HDL cholesterol is desirable and is associated with a lower risk of heart disease
(recall that low HDL values <40 mg/dL may be associated with increased risk for heart disease)
Value can be estimated from other lipid values when non-HDL level is not directly available:

Non HDL = TC - HDL


• VLDLcanbe estimatedby dividingTG levelby 5
=LDL+VLDL • Non-HDLareApoB-100containingatherogenic lipoproteins
TG • LDLcanbederivedusingthisformula:TC- HDL- (TG/5)in mg/dl
=LDL+- 5-

APPROACH & MANAGEMENT OF DYSLIPIDEMIA


For this section, we have synthesized the recommendations of recent guidelines: American College of
Cardiology (ACC 2019), European Society of Cardiology (ESC 2020), and the latest version of our local CPG.

I. RISK STRATIFICATION
Risk stratification determines the aggressiveness by which dyslipidemia should be treated,
with stricter/lower targets (particularly LDL-C) for higher risk individuals
IACC 20191 ESC 2020 I PHILIPPINE CPG (2020)

, Simplified risk factor counting


• IO-year
• SCORE • Identified risk factors include: male sex,
Risk ASCVD
risk postmenopausal women, smoker, hypertension,
stratification risk
system BM! >25 kg/m', family history of premature CAD,
calculator
microalbuminuria or proteinuria, and LVH

• Scoring systems not • Presence of;,2 risk factors (with LDL ;,130mg/dL) is
Remarks
validated for Filipinos considered elevated risk (statins are recommended)

II. LIFESTYLE MODIFICATION AND CONTROL OF COMORBIDITIES


All patients should be advised lifestyle modification
0 Smoking and vaping cessation
0 Diet low in saturated fat and high in whole grain products, fruits/vegetables, and fish
30-60 minutes of moderately vigorous exercise on most days of the week
0 Target BM! 20-25 kg/m' and waist circumference <94 cm for males or <80 cm for females
For HPN patients, target BP <130/80 mmHg (<140/90 mm Hg for very elderly "So years old)
For D M patients, HbA1c target of <t'lo
172
III. LDL-C TARGETS (PRIMARY TARGET)
0 The appropriate intensity of statin therapy should be selected in order to achieve the
desired LDL-C treatment target, which in turn is based on the patient's clinical risk profile
0 In general, individuals with higher risk demand lower LDL-C targets
0 Patients with established ASCVD automatically warrant high-intensity doses of statins

• Established clinical ASCVD


0 Acute or chronic coronary syndrome
0 Stroke and TIA <55 mg/dl
Very ° Carotid artery disease/stenosis AND
High 0 Peripheral arterial disease (PAD) ~50%from
Risk 0 History of revascularization (e.g., PC!, CABG, baseline
peripheral bypass/scenting)
• FH patients with ASCVD or other risk factor
• CKD stage IV to V (eGFR <30 mL/min)
Elevated
• TC >310mg/dL,LDL>190mg/dL,or BP;,;18o/nommHg
Risk
• Diabetics with target organ damage and/or at least <70 mg/dl
1other risk factor (RF) AND
High
• TIDM ;,,35years old; T2DM ~50 years old and/or ~50% from
Risk
DM duration ;,,10years baseline

l-
-----+-•-F_H_p_a_ti_en"'-t-s_wi_·th_o_u_tA_s_c_v_D_or_o_th_er_n_·s_k_f;_a_ct_o_rs--+--------1
• CKD stage Ill (eGFR30-59 ml/min) ..

• Diabetics without target organ damage or other RF


Moderate
• Young diabetics (T1DM <35years old; T2DM <50 <100mg/dl
Risk
years old) with OM duration <IOyears
<130mg/dl
Low Risk • None of the above
(<116mg/dlinESO)
Sources:Gonzalez-Santos
LE,et al. 2020CPGPhilippines.
J ASEAN Fed EndocrSoc.2021
MachF,et al. 2019ESC/EASGuidelinesdyslipidaemias.
EurHeartJ. 2020
GrundySM,et al. 2018AHA/ACC Cholesterol
CPG.Circulation.
2019

Four Maior Statin Benefit Grouvs (criteria for initiation of statin treatment)
• Individuals with clinical ASCVD•
• Individuals with familial hypercholesterolemia (FH)° or primary elevation ofLDL ~190mg/dL
• Individuals with diabetes mellitus
• Individuals ~45years old + LDL ~130mg/dL + ~2 risk factors
•Atheroscleroticcardiovasculardisease (ASCVD)includesacute coronarysyndrome(ACS),historyof
myocardialinfarction(Ml),stable or unstableangina,previousrevascularization,stroke/transientischemic
attack,or peripheralarterialdisease
.. FH is an autosomaldominantdisorderthat causes severe elevationsin total cholesteroland LDL-C,which
translates to prematureCADand highriskfor CVevents even in youngpatients.The DutchLipidClinic
Networkcriteriais frequentlyused to make the diagnosisof FH.

IV. OTHER LIPID TARGETS


COMPONENT I TARGET

Triglycerides • <150 mg/dl

HDL-C • >40 mg/dl in males, >50 mg/dl in females


• Target values are 30 mg/dl higher than the corresponding LDL-Ctarget
0 <85 mg/dl: for very high risk

Non-HDL-C 0 <100 mg/dl: for high risk

0 <130 mg/dl: for moderate risk

0 <160 mg/dl: for low risk

173
PHARMACOLOGIC THERAPY FOR DYSLIPIDEMIA
To summarize, the management of dyslipidemia begins with an assessment of the patient's clinical ASCVD
risk. If ASCVD risk is elevated and lipid-lowering therapy is warranted, statins are the first-line drugs. If
LDL-C treatment targets are not achieved on follow-up, ezetimibe and/or PCSK9 inhibitors may be added.

Assessment of patient's
clinical ASCVD risk

No
Lifestylemodification

No

Start moderate intensity


Start high intensity statin statin

No

Annual follow-up unless


indicated to be more Add ezetimibe
frequent

Yes

Add PCSK9 inhibitor

Source:
MachF,etal.2019ESC/EAS EurHeartJ. 2020
Guidelines.

Atorvastatin40-80 mg Atorvastatin10-20mg Simvastatin10 mg


Rosuvastatin20-40 mg Rosuvastatin5-10mg Pravastatin10-20mg
Simvastatin20-40 mg Lovastatin20 mg
Pravastatin40-80 mg
Lovastatin40-80 mg
A lipidprofileis donewithin4-12weeksafterinitiationor doseadjustmentof statins(thenevery
3-12months).For thosewith stalin-associatedmusclesymptoms(SAMS)suchas musclepain/
tenderness, checkcreatinekinase(CK),creatinine,andurinemyoglobin.Holdstatinsfor at least
2 weeksto checkfor improvement of symptoms(6 weeksif with rhabdomyolysis). If symptoms
improve,optionsincludeloweringthe statindoseor combiningwithezetimibeor PCSK9inhibitors.
Routinemonitoringof serumALTwhileon statinsis no longerrecommended.

174
, Inhibits HMG-CoA • Myositis/myopathy
reductase • 1st line of treatment • Rhabdomyolysis
• Promotes increased • LDL reduction (rare)
LDL receptor depends on dose • Reversible elevation
expression and intensity (doubling of transaminases
Statins
increased uptake of the dose leads to 6% • No association
LDL from the blood additional lowering) with dementia,
, Pleiotropic effects: • Increase HDL by 1-10% intracerebral
anti-inflammatory, • Reduce TG by I0-20% hemorrhage, or
antioxidant cancer

• Add-on to statins if
• Rarely increases
, Cholesterol LDL not on target or
transaminases
absorption inhibitor if statins not tolerated
Ezetimibe • Liver injury/failure
• Inhibits intestinal • Additional LDL
(very rare)
uptake of cholesterol reduction of 21-27%if
added to statins
• Inhibits PCSK9, • Reduces LDL by 60%
PCSK9 leading to increased • Reduces TG by -26% • Local SC injection
Inhibitors expression of LDL • Increases HDL by -9% site reactions
(Evolocumab & receptors to allow • Indicated when • Flu-like symptoms
Alirocumab) more LDL uptake statins ± ezetimibe
from the blood have been maximized
Other 1Jrr4gs
for Dyslipidemia,
• Unclear, but may be
Omega-3-Fatty related to interactions • Unpleasant fish-like
• Reduces TG by 45°/o
Acids with PPAR (decreased taste
• May raise LD L levels
(Fish Oil) VLDLsynthesis) & • GI disturbance
decreased apoB
• Agonist of PPAR-a, • Myopathy (higher risk
which leads to if with concomitant
• Primarily reduces TG
upregulation of statin use)
Fibrates (upto50%)
lipoprotein lipase • Reversible increase
(Fenofibrate, • Increases HDL by
(reduces TG) & in creatinine &
Gemfibrozil) 520%
increase of ApoA-I transaminases
• Reduce LDL by ~20%
& ApoA-II synthesis • GI disturbance
(increases HDL) • Cholelithiasis
• Increases HDL • Flushing
• Stimulates hepatic significantly • Hyperuricemia
Niacin/
ApoA-I production • Decreases TG • Impaired glucose
Nicotinic Acid
(increases HDL) • Modest decrease in tolerance
LDL • Hepatotoxicity
• GI discomfort
• Prevents intestinal
• Decreased
reabsorption of
Bile Acid • ModestLDL absorption offat-
bile acids in the
Sequestrants reduction of 18-25% soluble vitamins
terminal ileum, thus
(Cholestyramine, • Can increase • Contraindicated
forcing liver to use
Colestipol) triglycerides ifTG >200 mg/dL
cholesterol to make
(relative) or >500
more bile acids
mg/dL (absolute)
175
SECTION THREE
HYPERTENSION HPN
CLASSIFICATION OF HYPERTENSION
I. PRIMARY OR ESSENTIAL HYPERTENSION (HPN)
Type of HPN where there is no identifiable or reversible cause for the elevated BP
• Much more common (90-95% of hypertensives) than secondary hypertension

II. SECONDARY HYPERTENSION


Type of hypertension wherein a specific, remediable cause of hypertension can be identified
Clues for suspecting secondary hypertension:
Abrupt onset of hypertension or exacerbation of previously controlled hypertension
0 Age of onset <20 or >50 years old
No family history of HPN
DBP ,110-120mmHg
Sudden increase in BP in a patient with stable Stage I HPN
Poor BP control (despite good compliance to adequate drug therapy) or malignant HPN
Systemic findings (e.g., weight loss/gain, unprovoked or excessive hypokalemia)
Disproportionate target organ damage for degree of hypertension
Treatment is directed towards the underlying cause
ETIOLOGY I CLINICAL CLUES I DIAGNOSTIC TESTS
• Serum creatinine•
Renal • Risk factors for CKD
• Urinalysis"
parenchymal • Oliguria, pallor
• Renal ultrasound•
disease • Proteinuria/hematuria/casts in urine
• Kidney biopsy, if indicated
• Abrupt onset ofHPN or worsening/
Renal artery
difficult to control with risk factors • Renal arterial Doppler
stenosis
for ASCVD (atherosclerotic) ultrasound"
(atherosclerotic
• Earlyonset HPN (fibromusculardysplasia) • Abdominal CT or MRA"
or fibromuscular
• Flash pulmonary edema • Invasive renal angiography
dysplasia)
• Abdominal bruits
• HPN with spontaneous hypokalemia • Plasmaaldosterone/reninratio"
Primary
• Adrenal mass • Oral sodium loading test
hyperaldosteronism
• Muscle weakness (rare) • Adrenal CT scan
• Resistant hypertension • Berlin Questionnaire•
Obstructive • Snoring, apnea during sleeping, • Epworth Sleepiness Scale"
sleepapnea morning headache, daytime • Overnight oximetry
sleepiness, obesity • Polysornnography
Hyper- or • Signs and symptoms of thyroid
• Thyroid function tests•
Hypothyroidism hormone excess or deficiency
• Labile BP (paroxysmal HPN) • Plasma or 24-hour urinary
Pheochromocytoma • Headache, diaphoresis, palpitations, metanephrines•
and pallor • Adrenal imaging
• 1-mg dexamethasone
• History of chronic high-dose steroid use suppression test•
• Moon fades, buffalo hump, obesity • 24-hour urinary free cortisol•
Cushing's syndrome
• Violaceous striae, easy bruisability • Midnight salivary cortisol
• Proximal muscle weakness • Plasma ACTH
• Cranial & adrenal imaging
• Systolic BP difference ;t20 mmHg • 2D echocardiogram•
Coarctation
between upper & lower extremities • CXR: rib notching
of the aorta
!e and/or between right and left arms • CT or MR aortography
'Usuallyconsideredas screeningtests
176
DIAGNOSIS AND WORK-UP OF HYPERTENSION
HPN is defined as the level of BP at which benefits of treatment outweigh the risks
• Based on evolving trial evidence, recent guidelines from various societies have arbitrarily set
different cut-off values and classifications for hypertension

I. CRITERIA FOR HYPERTENSION BASED ON CLINIC/OFFICE BP MEASUREMENTS

PHILIPPINE I ESC I ESH ACC /AHA•


CPG (2020) (2018)

Optimal
I (2017)

Normal
<120 <80 Normal
120-129 80-84 Normal ElevatedBP (120-129/<80)
Borderline
130-139 85-89 HighNormal Stage 1 HPN(130-139/80-89)
140-159.. 90-99 .. Grade 1 HPN
Hypertension
160-179 100-109 Grade2 HPN Stage 2 HPN
(HPN)
2:180 2:110 Grade3 HPN
'Comparedto the otherguidelines,the 2017ACC/AHA
Guidelinesset a strictercut-offvalueof 2:130/80
"Isolatedsystolichypertension:SBP2:140mmHgANDDBP<90mmHg

II. CRITERIA FOR HPN BASED ON OUT-OF-OFFICE BP MEASUREMENTS


Home BbPmhonihtorindg(HBPM)and afimbuhlatory BP monitoring (ABPM)offer alternative
means y w ic to iagnose or con rm ypertension
HBPMand ABPMare most useful for the diagnoses of white coat HPN & masked HPN
Compared to HBPM,ABPMcan measure nocturnal BPreadings & dynamic BP measurements
throughout the patient's day, but is more costly and is not readily available in some areas

Home BP Monitoring (HBPM) 2: 135/85 2:130/80


Ambulatory BP Monitoring (ABPM)
Mean Daytime (Awake) ABPM 2: 135/85 2: 130/80
Mean Nighttime (Asleep) ABPM 2: 120/70 2: 110/65
Mean 24-hour ABPM 2: 130/80 2: 125/75
'The 2018 ESC/ESHcut-offvaluesare the ones followedby mostcentersinthe Philippines

III. OTHER BP PATTERNS BASED ON OFFICE AND OUT-OF-OFFICE MEASUREMENTS


PATTERN I OFFICE OR CLINIC BP AMBULATORY OR HOME BP

Normotensive Normal Normal


White coat hypertension• High Normal
Masked hypertension .. Normal High
Sustained hypertension High High

WhitecoatHPN:daytimeBP <135/85mmHg& no evidenceof targetorgan damagedespiteconventional


officeBP consistently2:140/90mmHg(associatedwithless HMODand riskof CVevents)
MaskedHPN:daytimeBP 2:135/85or nighttimeBP 2:120/70despite conventionalofficeBP consistently
<140/90mmHg(commonin youngermen,smokers,and those withhigherlevelsof stress/alcohol)
Resistant HPN:high BP uncontrolledwiththree OR controlledwithat least fouranlihypertensivedrugs
(one of whichis a diuretic)at maximallytolerateddoses
177
IV. DIAGNOSTIC WORK-UP
The work-up of the hypertensive patient is directed towards screening for co-morbidities &
evidence of hypertension-mediated organ damage (HMOD)

CBC• • May reveal anemia ofCKD


• Check eGFR and screen for renal HMOD
Serum creatinine,
• Check for hyperkalemia of CKD, unprovoked hypokalemia of
sodium, potassium•
primary aldosteronism
Serum AST, ALT' • Baseline liver enzymes
• Fasting blood sugar (FBS) & HbArc to screen for OM'
Sreen for co-
• Fasting lipid profile to screen for dyslipidemia'
morbidities
• Serum uric acid (SUA) to screen for hyperuricemia'

ScreeningforHy:eeratension·Mediated
Organ Damage (HMOD)
12-lead ECG' • Screen for LVH, ischemia, arrhythmias (e.g., atrial fibrillation)
Urine albumin to • Screen for albumin excretion indicative of early hypertensive
creatinine ratio• nephropathy
Fundoscopy• • Screen for hypertensive retinopathy

• Evaluate cardiac structure and function if with ECG abnormalities or


Echocardiography
signs and symptoms of heart failure/L V dysfunction

Abdominal • Evaluate renal size and structure, exclude urinary tract obstruction
ultrasound • Evaluate abdominal aorta for AAA, especially for heavy smokers

• Screen for carotid plaque or stenosis, especially if with history of


Carotid ultrasound
stroke/TIA or evidence of ASCVD elsewhere
Cranial imaging • Screen for ischemic or hemorrhagic brain injury, esp. if with history
(CT of MRI) of stroke/TIA or with signs of cognitive decline
Ankle brachia! index • Screen for lower extremity peripheral arterial disease (PAD)
• Minimum
testingrequired

MANAGEMENT OF HYPERTENSION
I. NON-PHARMACOLOGIC MANAGEMENT (LIFESTYLE MODIFICATIONS)
MANAGEMENT I REMARKS
Dietary Approaches • Rich in fruits, vegetables, whole grains, nuts, low-fat dairy products
to Stop Hypertension • Prefer lean meat, poultry and fish over red meat and pork
(DASH)diet • Less sweets and sugar-sweetened beverages

Dietary salt • Optimal: <1500mg/day, but at least a 1000-mg/day reduction


restriction • 1 teaspoon of salt= 5000 mg

Weight loss • Optimal target: ideal body weight (aim for at least 1-kg reduction)
;1
Physical activity • 90-150 minutes of moderate-intensity aerobic exercise per week

Moderate alcohol • 2 drinks daily for men; and 5 I drink daily for women
intake • I drink= 12 oz beer= 5 oz wine= 1.5oz distilled spirits

Smoking cessation ,1 • bupoprion,


Quit rates improved by pharmacological measures (varenicline,
nicotine replacement) and behavioral support
Sources:2018ESC/ESH EurHeartJ 2018& 2017ACC/AHA
Guidelines. CPG.J AmCollCardiol2018
178
II. TREATMENT THRESHOLDS AND TARGETS
The aggressiveness of the treatment strategy for hypertensive patients is influenced by
the patient's age, comorbidities, risk profile, and evidence ofHMOD
All should be advised regarding lifestyle modification/non-pharmacologic aspects of BP control

Th h Id
ADULTS <80 VERY ELDERLY
I YEARS OLD
?140/90mmHg*
I (?80YEARSOLD)
I REMARKS

Treatment thresholds 2:150/90mmHg When to start therapy?

Treatment goals <130/80mmHg <140190mmHg What is the target BP?

Thresholds for 2-drug Core regimen:


combination as >150/100mmHg >160/100mmHg RAAS blocker+
initiation therapy .. CCB or diuretic
*ESC/ESH2018:may already initiatetreatment if BP .!:130/85
mmHg(high normal)in very high risk patients
'*At treatment-na'iveBP levels above these thresholds, monotherapywilllikelybe insufficientto achieve
the BP target. Also, low-dosecombinationtherapy has been shown to be more effectivethan maximaldose
monotherapy. Monotherapyshould be reserved for low risk patients with Grade 1 HPN(especiallyif SBP
<150mmHg),or frailolder patients.

High Risk Features


The presence of these high-risk features should prompt physician to initiate drug/pharmacologic ..
therapy earlier (or upon diagnosis if already with HMOD or concomitant CV disease)
CARDIOVASCULAR RISK I HMOD I CARDIOVASCULAR
FACTORS DISEASE
• Age >65years • LVH on ECG or 2D echo • Coronary heart disease/
• Male sex • Moderate to severe CKD myocardial infarction
• BM! ~25kg/m' (eGFR <60 ml/min/1.73m') • Heart failure
• Diabetes • Microalbuminuria • Stroke or TIA
• LDL,130mg/dLorTG ,150 mg/dL (30-300 mg/24 hours) • Peripheral vascular disease
• Family history ofCVD or HPN • Retinopathy • Atrial fibrillation
• Early-onset menopause
• Smoking

III. PHARMACOLOGIC MANAGEMENT


First-line agents for HPN are renin-angiotensin-aldosterone inhibitors (RAAS-1),
calcium channel blockers (CCB), and thiazide/thiazide-like diuretics
Beta blockers (BB) are no longer considered first-line therapy, but are indicated for
patients with CAD, post-Ml, HFrEF, AF, high sympathetic drive, and pregnancy
Single pill combinations of~2 anti-hypertensive agents are encouraged for most patients

Compelling Indications of Anti-Hypertensive Classes for Specific Co-Morbidities


CO-MORBIDITY ANTI-HYPERTENSIVE CLASS

Diabetes • RAAS-1, CCB, diuretic

Heart failure • RAAS-I, BB, MRA, diuretic

CAD/Post-MI/Angina • ACE-I, CCB, BB, diuretic

Post-stroke • ACE-I+ diuretic, CCB

Chronic kidney disease • ACE-I, BB, diuretic

Atrial fibrillation • BB, non-DHP CCB

Aortic aneurysm • BB

Pregnancy , Methyldopa, BB (labetalol), CCB (nifedipine)

179
Summary of Pharmacologic Therapy for Hypertension
(table below summarizes anti-h ertensives, iven in TOTAL DOSE PER DAY & dail

• Captopril 12.5-150
mg/d (BID-TIO) • Inhibits ACE, • Cough (due
ACE- • Enalapril 5-40 mg/d (OD-BID) therefore, to more
Inhibitors • Lisinopril 10-40 mg/d (OD) angiotensin I is bradykinin)
(ACE-I)• • Perindopril 2.5-10mg/d (OD) not converted to • Angioedema
• Ramipril 2.5-20 mg/d (OD) angiotensin II • Hyperkalemia

• Candesartan 8-32 mg/d (OD)


• Competitive
Angiotensin • Irbesartan 150-300 mg/d (OD) • Less cough and
antagonism with
Receptor • Losartan 50-100 mg/d (OD-BID) angioedema
angiotensin-11 on
Blockers • Olmesartan 20-40 mg/d (OD) than ACE-I
type 1 angiotensin
(ARBs)• • Telmisartan 20-80 mg/d (OD) • Hyperkalemia
receptors
• Valsartan 80-320 mg/d (OD)

Dihydro- • Amlodipine 2.5-10mg/d (OD) • Blocks L-type • Ankle edema


calcium channels
pyridines , Felodipine 2.5-10mg/d (OD) • Headaches
• Vascular effect >
(DHP)' • Nifedipine LA 30-90 mg/d (OD) AV node effect • Flushing

Non- • DiltiazemSA120-36o
mg/d(B!D-QID) • Blocks L-type • Bradycardia
Dihydro- , Diltiazem ER 120-360mg/d (OD) calcium channels (avoiduse with
pyridines , Verapamil IR 120-360mg/d (f!D) • AV node effect > BBs)
(Non-DHP) , VerapamilSR120-36omg/d(OD-BID) vascular effect • Worsening HF

• Hydrochlorothiazide (HCTZ) • Hyponatremia


• Inhibit Na/Cl
Thiazide/ 25-50 mg/d (OD) • Hypokalemia
symporter in distal
Thiazide- • Chlorthalidone 12.5-25mg/d • Hyperglycemia
convoluted tubules,
like (OD) • Elevated TG
leading to enhanced
Diuretics• • Metolazone 2.5-5mg/d (OD) • Hyperuricemia
NaCl excretion
• Indapamide 1.25-2.5mg/d (OD) • Erectiledysfunction

• Acts on Na-K-2Cl • Hypokalemia


• Hypocalcemia
Loop • Bumetanide 0.5-2 mg/d (BID) symporter in thick
• Hypomagnesemia
Diuretics • Furosemide 20-80 mg/d (BID) ascending limb of
• Metabolic alkalosis
the loop of Henle • Ototoxicity
Mineralo- • Inhibits aldosterone,
corticoid • Eplerenone 50-100mg/d (OD-BID) • Hyperkalemia
leading to decreased
Receptor • Spironolactone 25-100mg/d • Gynecomastia
Antagonists expression ofENaC
(OD) • Erectiledysfunction
(MRA) in collecting tubules

BetaBk
• Atenolol 25-100 mg/d (BID)
• Bisoprolol 2.5-10mg/d (OD) • Selectively inhibits • Bronchospasm
Cardio- • Metoprolol tartrate ~-1 receptors (less • Bradycardia, AV
selective BB 100-200 mg/d (BID) pulmonary side
• Metoprolol succinate blocks
effects) • Easy fatigability
50-200 m Id (OD)
• Weight gain
Non- • Propranolol 80-160 mg/d (BID) • Inhibits both ~-1 & • Glucoseintolerance
selective BB • Nadolol 40-320 mg/d (OD) ~-2 receptors
• Sleep disturbance
• Carvedilol 12.5-50mg/d (BID) • Combined a-1 • Depression
Vasodilating
• Carvedilol CR 20-80 mg/d (OD) & ~-adrenergic • Erectiledysfunction
BB
• Nebivolol 5-40 m /d (OD) rece tor blockade
180
CLASS I REPRESENTATIVE DRUGS** I MECHANISM I SIDE EFFECTS
Other Anti-Hypertensives
• Block post- • Orthostatic
• Doxazosin 1-16mg/d (OD)
Alpha synaptic a-1 hypotension
• Prazosin 2-20 mg/d (BID-TID)
Blockers receptors in • Reflex
• Terazosin 1-20mg/d (OD-BID)
vessels tachycardia
• Activates a-2
receptors in • CNS side effects
• Clonidine 150-450mcg/d
CNS, resulting • Xerostomia
(BID-TID)
Central in reduced • Male sexual
• Methyldopa 250-rooo mg/d
Sympatholytics sympathetic dysfunction
(BID)
outflow& • Rebound HPN
peripheral vascular on withdrawal
resistance

• Reflex
Ii tachycardia
• Hydralazine roo-200 mg/d • Fluid retention
(BID-TID) • Open vascular • Headache
Direct
• Minoxidil 5-roo mg/d ATP-sensitive K+ • Lupus-like
Vasodilators
(OD-TID) channels syndrome (for

I
hydralazine)
• Hypertrichosis
(for minoxidil)
ACE:angiotensin-converting
enzyme CHF:congestiveheart failure
'First-lineagents, preferablyas single-pillcombinationinstead of monotherapy
"Drugs givenin total dose per day (and dailyfrequency)

, , ,

DRUG
I DOSE

• 5-15mg/hr as continuous infusion


I ADVERSEEFFECTSAND
CONTRAINDICATIONS

• Liver failure
• Starting dose 5 mg/hr, increase q15- • Headache
Nicardipine
30 mins by 2.5 mg/hr until goal BP • Reflex tachycardia
achieved; thereafter decrease to 3 mg/hr • Contraindicated in severe AS
• 5-200 mcg/min as continuous infusion
Nitroglycerin • Headache
• Starting dose 5 mcg/min, increase q5
(NTG) • Reflex tachycardia
mins by 5 mcg/min
• 0.25-0.5mg/kg IV bolus, then 2-4 mg/
Labetalol min infusion until goal BP achieved; • Contraindicated in advanced
thereafter 5-20 mg/hr AV blocks, bradycardia, systolic
• 0.5-1.0mg/kg IV bolus, then 50-300 heart failure, and asthma/COPD
Esmolol
mcg/kg/min infusion
• Initial 10 mg via slow IV infusion • Unpredictable response &
Hydralazine (maximum initial dose 20 mg) prolonged duration of action
• Repeat every 4-6 hours as needed (not a desirable first-line agent)
Source:Vanden BornBJ,et al. Dutchguidelineforthe managementof hypertensivecrisis.NethJ Med.2011
ZipesDP,et al. Braunwald's HeartDisease.11thedition.Elsevier/Saunders,
2019
Williams B,et al. 2018ESC/ESHGuidelines.EurHeartJ 2018
WheltonPK,et al. 2017ACC/AHA HighBloodPressureCPG.J AmCollCardiol2018
J ClinHypertens(Greenwich)
Ona DID,el al. 2020CPGin the Philippines. 2021

181
HYPERTENSIVE CRISIS
I. TYPES OF UNCONTROLLED HYPERTENSION
HYPERTENSIVE HYPERTENSIVE
I URGENCY I EMERGENCY
Blood Pressure • SBP >180&/or DBP >120mm Hg
AcuteHMOD• • None • Present
• Reinstitute or intensify oral • Admit to ICU & manage based
Management drug therapy & arrange close on the presence of compelling
outpatient follow-up conditions•
*AcuteHMODmay includeacute heart failureor pulmonaryedema, aorticdissection,renalfailure,
preeclampsia/eclampsia/HELLP,
stroke,encephalopathy,or subarachnoidhemorrhage
GeneralGuidein theManagement:
• For patientswitha compellingcondition(e.g., aorticdissection,ACS,pulmonaryedema), SBP
shouldbe reducedto <140 riimHgduringthe 1st hourand to <120 mmHgin aorticdissection
• For patientswithouta compellingcondition,SBP shouldbe reduced by no morethan 25%withinthe
1st hour;then ifstab[e, to 160/100mmHgwithinthe next 2-6 hours;and then cautiouslyto normal
duringthe following24-48hours

II. TREATMENT OF HPN EMERGENCIES BASED ON END-ORGAN IVOLVEMENT


EMERGENCY
I TIMELINE & TARGET BP I PREFERRED DRUGS
HPN • Immediate
encephalopathy • Target MAP: decrease by 20-25%
• If candidate for thrombolysis:
target BP <185/nobefore & during
thrombolysis
Acute ischemic
• If not a candidate for thrombolysis
stroke • Nicardipine
and BP >220/120:reduce MAP by 15%
• Labetalol
during the first 24 hours, target MAP
no-130 mmHg for neuroprotection
Cerebral
• Several hours (as long as SBP not >220)
hemorrhage
• Target BP: 140-160mmHg (avoid
(with SBP>180or
reductions 2!60mmHg/hr)
MAP >130mmHg)
• Esmolol (~-blockade
should precede vasodilator
Acute aortic • Immediate (within 20 mins)
to preventreflextachycardia)
dissection • Target SBP <120mmHg & HR <60 bpm
• Nitroglycerin
, Nicardipine
Acute • Nitroglycerin (with loop
cardiogenic diuretic)
pulmonary • Immediate • ~-blockers
edema • Target SBP <140mmHg contraindicated
Acute coronary • Nitroglycerin
syndrome • Labetalol
Severe • Labetalol
• Immediate
preeclampsia/ • Nicardipine
• Target BP: <160/105mmHg
eclampsia • Magnesium sulfate
182
HYPERTENSION IN PREGNANCY
I. HYPERTENSIVE DISORDERS OF PREGNANCY
TERM I DEFINITION
• New-onset BP elevations after 20 weeks age of gestation (AOG) but in the
Gestational
absence of proteinuria
hypertension
• Failure of BP to normalize postpartum changes diagnosis to chronic HPN
ChronicHPN • High BP known to predate conception or detected before 20 weeks AOG
• Pregnancy-specific hypertensive disease with multisystem involvement
• Defined by new-onset hypertension+ new-onset proteinuria
• SBP ~140 or DBP ~90 mm Hg on 2 occasions at least 4 hours apart after
20 weeks AOG in a woman with previously normal BP, AND
Preeclampsia 0 Proteinuria of~300 mg per 24 hour urine collection, OR protein/
creatinine (mg/dL) ratio ~0.3, OR dipstick reading 1+
• In the absence of proteinuria, it may also be defined by new-onset HPN
with thrombocytopenia, renal insufficiency, impaired liver function,
pulmonary edema or cerebral/visual symptoms
• Presence of new-onset grand ma) seizures in a woman with preeclampsia
Eclampsia
• Can occur before, during, or after labor
• Diagnosis more likely in women with HPN only in early gestation who
develop proteinuria after 20 weeks AOG, or women with proteinuria
before 20 weeks AOG who develop:
Chronic
• Sudden exacerbation of HPN or need to escalate therapy
HPNwith
0 New-onset signs/symptoms (increase in liver enzymes, RUQ pain, headache)
superimposed
0 Platelet decrease to <100,000/uL
preeclampsia 0 Pulmonary edema
• Renal insufficiency
0 Sudden and sustained increase in protein excretion

II. MANAGEMENT
Threshold for drug treatment initiation:
0 ,140/90 if not known to have chronic HPN or if already with HMOD from chronic HPN
• ~150/95in all pregnant women
Target BP once treatment started: <140/90 mmHg
DRUG* I DOSE I REMARKS
Oral Antihypertensive Agents in Pregnancy
Methyldopa • 0.5-3g/day in 2-3 divided doses • May not be as effective for severe HPN
Nifedipine • 30-120 mg/day • Do not use sublingual, short-acting form
Labetalol • 200-2400 mg/day in 2-3 doses • Well-tolerated
Agents for Urgent Blood Pressure Control in Pregnancy
• 10-20 mg IV, then 20-80 mg q20- • First-line agent
Labetalol 30 mins (max of300 mg), OR • Do not exceed 800 mg/24 hr to prevent
• IV infusion 1-2mg/min fetal bradycardia
• 5 mg IV, then 5-10 mg IV • Higher dosage associated with maternal
Hydralazine q20-40 mins, OR hypotension, headaches, fetal distress
• IV infusion 0.5-10 mg/hr (hence no longer the drug of choice)
• to-20 mg PO, repeat in 30 mins if • May cause reflex tachycardia and
Nifedipine
needed, then I0-20 mg q2-6 hrs headaches
*Women withHPNwhobecomepregnant (orareplanning
to becomepregnant)
shouldbetransitioned
to methyldopa,
nifedipine,
and/orlabetalol
duringpregnancy(theyshouldNOTbetreatedwithACEiorARBs)
Sources:RobertsJM,et al. Hypertension
in pregnancy.
ObstetGynecol.2013
2017ACC/AHACPGJ AmCollCardiol2018& 2018ESC/ESH EurHeartJ 2018
Guidelines.
183
OVERVIEW OF HEART FAILURE (HF)
I. ETIOPATHOGENESIS
Syndrome characterized by symptoms (dyspnea & fatigue) & signs (edema, rales, elevated
)VP) that lead to frequent hospitalizations, poor quality oflife, and shortened life expectancy
Due to a structural and/or functional abnormality that results in elevated intracardiac
pressures and/or inadequate cardiac output at rest and/or during exercise
Coronary artery disease (CAD): most common cause of HF in industrialized countries (60-75%)

A. Classification Based on LV Function/Ejection Fraction (LVEF)


TYPE*
I LVEF I DESCRIPTION I COMMON EXAMPLES

• Systolic dysfunction
HF with • Progressive disorder initiated by • CAD (i.e. MI)
Reduced EF :540% an index event (e.g. MI, volume • Dilated cardiomyopathy
(HFrEF)" overload) that leads to a decline in
the pumping capacity of the heart

HF with
• Have primarily mild systolic dysfunction with features of diastolic
Mildly
41-49% dysfunction
ReducedEF
• May benefit from similar therapies for HFrEF, but trials still lacking
(HFmrEF)

• Due to diastolic dysfunction and • Pathologic hypertrophy


HF with
increased vascular stiffness (e.g., HOCM, HPN)
Preserved ,::50%
• Data for therapy still evolving with • Aging, fibrosis
EF(HFpEF)
new trials • Restrictivecardiomyopathy
'High-output states include thyrotoxicosis, nutritionaldisorders (beriberi),excessive blood flow
requirements, chronic anemia
"Patients with overtly reduced EF 910% that later on demonstrate EF ~50% are termed as 'HF with
improved EF' or HFimpEF(considered as recovered HFrEF rather than HFpEF)

B. American College of Cardiology/American Heart Association Stages of Heart Failure


STPG: I DESCRIPTION I SAMPLE PATIENTS I NYHAFC
• At high risk for HF but • Patients with HPN, CAD, OM,
A without structural heart cardiotoxin exposure, or family history None
disease or HF symptoms of cardiomyopathy
• Structural heart disease • Patients with previous Ml, systolic
B but without signs or dysfunction, or asymptomatic valvular
I
symptoms of HF disease

• Patients with known structural heart


• Structural heart disease
disease (as above) but with shortness II
C with previous or current
of breath, fatigue, reduced exercise
symptoms of HF
tolerance III
,i
• Refractory HF • Patients with recurrent hospitalizations
D requiring specialized or cannot be safely discharged without IV
interventions specialized interventions
Sources:McDonaghTA,et al. 2021ESCGuidelines.EurHeartJ. 2021
and dassificationof heartfailure.EurJ HeartFail.2021
BozkurtB,et al. Universaldefinition
ZipesDP,et al. Braunwald'sHeartDisease.11thedition.Elsevier/Saunders, 2019
YancyCW,et al. 2013ACCF/AHA HeartFailureGuideline.Circulation. 2013
184
Sample Case: 58-year-old male with marked limitation of physical act1v11y.Less than
ordinary activity causes dyspnea and fatigue. He is comfortable at rest. Echocardiography
revealed depressed LV ejection fraction of 35%. Stress testing and coronary angiography
were compatible with coronary artery (ischemic heart) disease.
Diagnosis: HFrEF from CAD, ACC/AHA Stage C, NYHA Functional Class III

II. CLINICAL MANIFESTATIONS

I REMARKS

Symptoms , .. ,~ - ..
Fatigue& • Cardinal symptoms of heart failure
dyspnea • Due to pulmonary congestion (juxtacapillary )-receptors are activated)
Orthopnea/ • Dyspnea in the recumbent position
nocturnal • Redistribution of fluid from splanchnic circulation and lower extremities
cough into the central circulation on recumbency
• Dyspnea in the lateral decubitus position
Trepopnea
• Associated with predominantly right-sided pleural effusion
Paroxysmal
• Severe dyspnea that awakens patient from sleep after a few hours
nocturnal
• Increased pressure in the bronchial arteries
dyspnea
Cheyne-
• Periodic or cyclic respiration: series of apnea hyperventilation hypocapnia
Stokes
• Caused by an increased sensitivity of the respiratory center to arterial PC02
respiration
• GI: anorexia, nausea, early satiety, and abdominal fullness may be due to
congested liver and/or bowels
Others
• CNS: confusion, disorientation, sleep and mood disturbance may be due
to reduced cerebral perfusion
:
Signs ,,;,.

General • Uncomfortable when lying flat, labored breathing
appearance • Normal or low BP
& vital signs • Cardiac cachexia
• Elevated jugular venous pressure ()VP)
• Sinus tachycardia due to increased adrenergic activity
• Point of maximal impulse displaced due to cardiomegaly
Cardio-
• Regurgitant murmurs: MR and TR
vascular
• S3 (protodiastolic gallop) at apex: usually in volume overloaded patients
• S4: usually in diastolic dysfunction from LV hypertrophy
• Pulsus altemans &/or narrow pulse pressure/thready pulse in severe disease
• Crackles/rales: transudation of fluid from intravascular space to alveoli
• Expiratory wheezes: cardiac wheezing from peribronchial cuffing from
Pulmonary
congestion
• Pleural effusions often bilateral; if unilateral, more often on the right
• Hepatomegaly with pulsation (if with significant TR)
• Ascites: increased pressure in the hepatic veins
• Jaundice: impairment of hepatic function due to congestion
Abdomen
• Abdominojugular reflex (pressure over the RUQ for at least IO seconds
causing a sustained rise >3 cm in )VP for ~15seconds after release of the
hand) is considered a predictor of heart failure
• Peripheral edema: ankles and pre-tibial region, usually pitting
• Indurated and pigmented skin: longstanding edema
Extremities
• Peripheral vasoconstriction: cool extremities in late stages
• Chronic venous stasis changes (e.g., hyperpigmentation, venous ulcers)
185
III. DIAGNOSIS OF HEART FAILURE
A. A roach to the Dia nosis of Heart Failure

Consider Heort Failure (HF) ii with


Risk factors or setting
Signs and symptoms
Abnormal ECG

HFrEFS40%
Determine
HF diagnosis confirmed;
HFmrEF41- 49% etiology and
classify according to LVEF TREAT
HFpEF2'50%

Basedon patient'srisk factors,signsand symptoms,and ECGfindings,one mayhavea high indexof


suspicionfor HF.Usenatriureticpeptidesandechocardiography to confirmthediagnosis,andthenclassify
andtreataccordingly.
Theunderlying etiologyfor HFshouldbe identifiedandaddressedas well.

B. Diagnostic Tests for Heart Failure


DIAGNOSTIC I COMMENTS/EXPECTED FINDINGS
• B-type natriuretic peptide (BNP) & N-terminal pro-BNP (NT-proBNP)
• Cutoff in chronic HF (different values in acute decompensated HF):
• BNP <20-40 pg/mL
Natriuretic • NT-proBNP <125pg/mL (in <75yrs) or <450 pg/mL in ?.75yrs)
peptides (BNP • Relatively sensitive markers for the presence of heart failure
and NT-proBNP) • Higher in patients with other cardiac conditions (e.g., !HD, valvular or
congenital heart disease, AF), pulmonary embolism, stroke, or sepsis
• Increase with age & renal impairment
• May be falsely low in obese patients
2D • Most useful test for evaluation of ejection fraction (EF) or LV function
echocardiography • Semi-quantitative assessment ofLV size, function, wall motion
with Doppler abnormalities, valvular lesions
• Assesses cardiac rhythm, LV hypertrophy, prior MI
12-leadECG
• A normal ECG virtually excludes LV systolic dysfunction
• Assess the cardiac size and shape and state of pulmonary vasculature
• Kerley B-lines: thin, horizontal linear opacities extending to the
Chest
pleural surface due to fluid in the interstitial space
radiography
• Others: peribronchial cuffing,prominent upper vasculature (cephalization)
• May also reveal other causes of symptoms (e.g., pneumonia)
• Gold standard for measurements of volumes, mass, & EF ofLV/RV
Cardiac MRI
• Characterize myocardial structure/tissue edema & myocardial viability
.
CBC • Check for anemia, signs of infection, and bleeding (may worsen HF)
Electrolytes, • Assess for electrolyte disturbances or beginning cardiorenal
BUN, creatinine, syndrome, ischemic hepatitis or chronic passive congestion of the liver
AST,ALT • Provides prognostic information and guides potential therapy
Sources.McDonagh EurHeartJ. 2021
TA,etal.2021ESCGuidelines.
186
MANAGEMENT OF CHRONIC HEART FAILURE WITH REDUCED EF
I. ALGORITHMIC APPROACH FOR HFrEF
Goal-directed medical therapy (GDMT) is the cornerstone of treatment for HFrEF
Proven to reduce mortality & HF hospitalization, the pillars of pharmacotherapy include:
0 Angiotensin-receptor neprilysin inhibitor (ARNI) or ACE-inhibitor (ACE-I)
0 Beta-blockers (BB)
0 Mineralocorticoid receptor antagonists (MRA)
0 SGLT-2 inhibitors (e.g., dapagliflozin, empagliflozin)

Patientswith diagnosed
HFrEF
(LVEF:S40%)

ACE-I/ARNI
BB GOAL-DIRECTEDMEDICALTHERAPY
MRA Reducemortality
SGLT2-I
Relievesymptoms
Diureticsfor fluid overload Preventhospitalization
Revascularization if indicated

Titrate meds;
Recheck LVEFafter 3-6 months

I
titrating medications I I I
IContinue
to maximal doses Screen for device therapy

ICD
1-----i LVEF:S35%
ECG QRS < 130 ms

CRT-D/-P
LVEF:S35%
ECG QRS ~130 ms
(especially if with LBBB)
Sinusrhvthm

No device
LVEF>35%
ECG QRS <130 ms

ICD:implantablecardioverter-defibrillator [VTor VF]for


(deviceto treattachyarrhythmias
prophylaxis
againstsuddencardiacdeath)
CRT:cardiacresynchronizationtherapy(deviceusedto restoresynchronized contractionof LV&
RVin patientswithHFrEFin sinusrhythm& wideQRScomplex[i.e.,CLBBB])
• CRT-D:cardiacresynchronizationtherapy-defibrillator
CRT-P:cardiacresynchronizationtherapy-pacemaker

II. NON-PHARMACOLOGICMANAGEMENT
• Na· restriction: limit to 2-3 g/day in all patients with HF (<2 g/day in moderate to severe HF)
Fluid restriction: limit total fluid intake to <2 L/day if with hyponatremia (<130meq/L) or
refractory volume overload despite high-dose diuretics and sodium restriction
• Monitor weight & adjust diuretics if with unexpected weight gain of 3-4 lbs over a 3-day period
Source:
McDonagh EurHeartJ. 2021
TA,etal.2021ESCGuidelines.
187
III. PHARMACOLOGIC MANAGEMENT OF HFrEF
DRUG STARTING DOSE
CLASS I (TARGET DOSE)' I DESCRIPTION/MECHANISM

p Illarsoif HFrEFPh_9r-macQt
herapy
• Captopril 6.25mg TIO (50 mg TIO) • Inhibits the conversion of angiotensin I to
ACE-
• Enalapril 2.5mg BID (10-20mg BID) angiotensin II
Inhibitors
• Lisinopril25-5mg OD (25-35mg OD) • Also inhibits kininase which may lead to
(ACE-I)
• Ramipril 2.5 mg BID (5 mg BID) increase in bradykinin & ACE-I induced cough
• Sacubitril inhibits neprilysin, which prevents
breakdown of natriuretic peptides & angiotensin
II (hence the need for ARB co-drug valsartan)
Angiotensin
• Recommended to replace ACE-ls & ARBs
Receptor-
• Sacubitril/valsartan 24'26 or 50 in ambulatory HFrEF patients who remain
Neprilysin
mg BID (97/103or 200 mg BID) symptomatic despite optimal therapy
Inhibitor
• Allow at least 36 hours washout period for
(ARNI)
ACE-I before starting ARNI to minimize
potential for angioedema
• May be considered in ACE-I/ARB-naive patients
• Bisoprolol 1.25mg OD (10mg OD) • Interferes with sustained activation of the
Beta • Carvedilol 3.125mg BID (25mg BID) adrenergic nervous system, particularly the
Blockers • Metoprolol succinate 12.5-25
mg OD deleterious effects of beta-I activation
(BB) (2oomgOD) • In acute HF, initiate with caution once patient
• Nebivolol 1.25mg OD (m mg OD) is euvolemic & hemodynamically stable

Mineralo-
• Inhibits aldosterone on the collecting duct
corticoid • Eplerenone 25mg OD (50 mg OD)
• May also be used for fluid retention (diuretic)
Receptor • Spironolactone25mgOD(5omgOD)
• Caution in renal failure & hyperkalemia
Antagonist
• Inhibits SGLT-2 receptor in proximal tubule,
reducing sodium & glucose reabsorption
Na"-Glucose
• Cardiac/renal protective effects independent of
Linked • Dapagliflozin 10 mg OD
glucose lowering (beneficial even in non-diabetics)
Transporter • Empagliflozin 10 mg OD
• Putative mechanisms for HF: diuresis/
2 (SGLT2)
natriuresis, BP reduction, improved cardiac
Inhibitors
energy metabolism, inflammation reduction, &
prevention of cardiac remodeling

Other Agents ii
Angiotensin
• Candesartan 4 mg OD (32mg OD) • Competitive antagonist of angiotensin II at
Receptor
• Losarran 50 mg OD (150mg OD) its receptor
Blockers
• Valsartan 40 mg BID (160mg BID) • Used if ACE-I intolerant (i.e., cough, angioedema)
(ARB)C

• Reduces heart rate by inhibition of the "funny


I{Channel • lvabradine 5 mg BID channel" (I) in the SA node
Inhibitor (7.5 mg BID) • May be used on top ofBB for HFrEF patients
with sinus rhythm and heart rate '270 bpm

• Inhibits Na-K-ATPase pump, increasing


Na-K-
intracellular Ca'", which leads to increased
ATPase • Digoxin 0.0625 mg OD
cardiac contractility
Pump (0.25 mg OD)
• For symptomatic LV dysfunction with
Inhibitor
concomitant atrial fibrillation

• Tomaximize clinicalbenefit,slowlytitrateto maximally


tolerated
dosesforeachdrug
b Pillars
of therapyshownto reducemortality in HFrEF(donotgiveACE-I,ARB,&/orARNIsimultaneously)
0 ARBis recommended forpatientsunableto tolerateanACE-IorARNI
188
JV. PHARMACOLOGIC MANAGEMENT OF FLUID RETENTION/VOLUME OVERLOAD
CLASS I DAILY DOSE I DESCRIPTION/MECHANISM
• Act on the loop of Henle by reversibly inhibiting
the reabsorption of Na·, K·, Cl in the thick
Loop • Furosemide 20-240 mg
ascending limb
Diuretics • Bumetanide 0.5-5 mg
• Also act as venodilators and reduce RA pressure
andPCWP
• Reduce the reabsorption of Na• and Cl in the first
Thiazide/ • HCTZ 12.5-100mg
half of the distal convoluted tubule
Thiazide-Like • Indapamide 2.5-5mg
• Tend to lose their efficiency with moderate to
Diuretics • Metolazone 2.5-10mg
severe renal insufficiency (Crea >2,5mg/dt)

• Competitive antagonism with vasopressin at V2


Arginine
receptors of the renal collecting ducts
Vasopressin
• Tolvaptan 15-60mg • Primarily used for treatment ofhyponatremia
(AVP)
by stimulating free-water excretion & improving
Antagonists
serum Na.. concentration

MANAGEMENT OF CHRONIC HEART FAILURE WITH PRESERVED EF


Previously termed as heart failure with normal EF, diastolic heart failure, or heart
failure with diastolic dysfunction
Diastolic dysfunction refers to abnormalities in LV performance, and abnormalities in
diastolic function can occur in both the presence or absence of the clinical syndrome of HF ;

I. DIAGNOSIS OF HFpEF
A simplified diagnostic approach to HFpEF still includes the signs and symptoms of
the clinical syndrome of HF, supported by an LVEF ;,:50% & other objective measures
involving natriuretic peptides & echocardiography

Signs and symptoms • Dyspnea, fatigue, orthopnea, PND


ofHF • Elevated )VP, rales, pedal edema

• Most commonly measured through echocardiography


LVEF~50%
• Other options: radionuclide imaging or cardiac MRI
Objective evidence • NT-proBNP ~125pg/mL (sinus) or ~365pg/mL (AF)
of structural &/ • BNP ~35pg/mL (sinus) or ~!05 pg/mL (AF)
or functional • Echocardiographic findings:
abnormalities 0 LV geometry consistent with concentric remodeling or
3 consistent with LV hypertrophy (normal geometry does not exclude HFpEF)
diastolic dysfunction 0 Dilated/enlarged left atrium (in the absence of valvular disease)

or raised LV filling 0 Pulmonary artery (PA) systolic pressure >35 mm Hg (TR jet
pressures velocity >2.8 mis)
TA,et al. 2021ESCGuidelines.EurHeartJ. 2021
Source:McDonagh

II. THERAPY FOR HFpEF


Evidence is lacking for the management ofHFpEF, despite numerous trials involving
drugs that have proven long-term morbidity and mortality benefits for HFrEF
Only SGL T2 inhibition (specifically empagliflozin 10 mg OD) has been demonstrated so
far to have positive results in HFpEF/HFmrEF (LVEF >40%)
A practical clinical approach includes the following:
0 Screen and manage individual risk factors (HPN, DM, CAD, etc.)
0 Reduce symptoms of congestion (loop diuretics may be used to relieve volume
overload)
0 Prevent/minimize acute decompensation and hospitalization
0 Improve exercise tolerance
189
ACUTE HEART FAILURE (AHF)
Refers to the new onset or recurrence of signs/symptoms of HF, requiring urgent
medical attention and resulting in an unscheduled hospital admission or ER visit
Possible etiologies for AHF:
° C: acute Coronary syndrome
0 H: Hypertensive emergency
0 A: Arrhythmia
0 M: acute Mechanical cause
0 P: Pulmonary embolism
0 I: Infection
0 T: cardiac Tamponade
I. CLINICAL PRESENTATIONS OF ACUTE HEART FAILURE
MECHANISMS &
TRIGGERS I PROFILES I CLINICAL
ASSESSMENT I TREATMENT

.ai1ure (ADHF)- Most Common


Acute y Decompensated Heart F.i
• Already with previous • Warm& • SBP: normal to low • Diuretics
LVsystolic &for diastolic wet,OR • CXR: often clear • I notropes/
dysfunction • Cold and wet lungs despite vasopressors
• Triggered by non- elevated filling (if with
compliance to medications/ pressures hypo perfusion)
salt restriction, ischemia, or • Echo: preserved or • Short-term MCS or
infections reduced EF RRT if needed
• Gradual progression (days)
with high rehospitalization

Acute PulmonaryEd}ma
.. '•

• Fluid accumulation in lungs • Warm&wet • SBP: >140 in most • Diuretics


leads to respiratory failure • CXR: pulmonary • Vasodilators
• Triggered by hypertensive edema
emergency,ACS,arrhythmias, • Echo: preserved EF
acuteMR&AR • Hypoxemia
• Sudden onset (hours) & common
very ill at presentation, but
responds well to therapy
..,
Isolated RV Failure It " !I! ·T
·~

• RV dysfunction • Cold&wet • SBP: low to normal • Diuretics for


• Pre-capillary pulmonary • CXR: may have peripheral
hypertension (cor pulmonary disease, congestion
pulmonale or primary prominent MPA • May consider
pulmonary arterial HPN) • Echo: dilated/ careful fluid
• Triggered by submassive dysfunctional RV, administration
to massive PE or RV moderate/severe • lnotropes/
infarction TR, LVEF preserved vasopressors
• Can be acute or chronic • Systemic congestion: • Short-term MCS or
onset ascites, pedal edema RRT ifneeded
.;,
-·~ :

-
<\ ,
I;,-. ~;l:i, . ~. "
Ctmliogpric Sliock; .
• Severe pump failure
'
• Cold&wet
""' -
• SBP: low
·-- • Inotropes/
• Progression of advanced • Echo: severely vasopressors
HF or development of a depressed EF • Revascularization
major myocardial insult • End-organ failure if needed
(large AMI, myocarditis) (e.g., renal, hepatic) • MCS and/or RRT
MCS:mechanical.circulatory
support
RRT:renalreplacement
therapy
Sources:McDonaghTA, et al. 2021 ESC Guidelines.Eur HeartJ. 2021
ZipesDP,et al. Braunwald'sHeartDisease.11thedition.Elsevier/Saunders,
2019
190
II. HEMODYNAMIC PROFILES AND CORRESPONDING MANAGEMENT
Assessing the AHF patient for systemic congestion and/or signs ofhypoperfusion can
guide the treatment approach & provide prognostic information

PROFILE I DESCRIPTION I MANAGEMENT


Warm& • Good perfusion • Adjust oral therapy (since patient is adequately
Dry and no congestion perfused & compensated)
• If HPN predominates (i.e., HPN emergency):
• Good perfusion
Warm& vasodilators (first-line) & diuretics
but with
Wet • If congestion predominates: diuretics,
congestion
vasodilators, ultrafiltration if diuretic-resistant

Cold& • With hypoperfusion • Consider fluid challenge first


Dry but no congestion • lnotropic support if still hypoperfused

• If SBP ~90: vasodilators, diuretics, consider


inotropic agents ifrefractory
•With
Cold& • If SBP <90: inotropic agent, consider vasopressors
hypoperfusion and
Wet if refractory, diuretics when perfusion is corrected,
congestion
consider mechanical circulatory support if no
response to drugs
• Signsof hypoperfusion("cold"):
coldextremities,oliguria,mentalconfusion,dizziness,narrowpulse pressure
• Signs of congestion("wet"):pulmonarycongestion/rales,orthopnea/PND,jugularvenous distention,
ascites, edema

lll. ADDITIONAL DIAGNOSTIC TESTS FOR ACUTE HEART FAILURE


Aside from the tests that are usually ordered for chronic heart failure, other tests that
have more clinical utility in the acute heart failure setting include the following:

DIAGNOSTIC I COMMENTS/EXPECTED FINDINGS


• Generally higher cut-off values than that of chronic HF
• Cut-off values to exclude acute heart failure:
Natriuretic 0 BNP <30-50 pg/mL
peptides (BNP 0 NT-proBNP <300 pg/mL
& NT-proBNP) • Cut-off values to identify acute heart failure:
0 BNP ;,100 pg/mL
0 NT-proBNP ;,900 pg/mL

Troponin • To rule out acute myocardial injury/infarction


D-dimer • To rule out pulmonary embolism
Procalcitonin • If considering sepsis as a possible trigger for the acute heart failure
Lactate • To assess perfusion status (lactic acidosis is a marker for hypoperfusion)
Pulse oximetry
• To assess oxygenation status for imminent respiratory failure
&ABG
• To visualize comet tails (the ultrasound equivalent of Kerley B-lines)
Lung
in pulmonary congestion
ultrasound
• Can also characterize/quantify amount & loculation of pleural effusion
Source:McDonagh
TA,et al. 2021ESCGuidelines. EurHeartJ. 2021
Ponikowski
P,et al. 2016ESCGuidelines. EurHeartJ. 2016
NohriaA,et al. J AmCollCardiel2003

191
IV. THERAPY FOR ACUTE HEART FAILURE
Initial management of AHF patients involves stabilization of the hemodynamic and
respiratory/ventilation status, followed by prompt work-up for the etiology of the AHF
and initiation of the corresponding treatment
CLASS I SAMPLE DRUGS I REMARKS
• Furosemide (20-160 mg/day or • Cornerstone of AHF treatment
5-40 mg/hr infusion) • Starting IV dose should be 1-2.5
• Bumetanide (0.5-4 mg/day or times the pre-admission oral daily
Diuretics
0.5-2 mg/hr infusion) dose of the patient
• Metolazone 2.5-10 mg OD (more • Satisfactory diuretic response: UO
potent if eGFR <30 mL/min) >I00-150 mL/hr in the first 6 hours

• Nitroglycerin (40-400 mcg/min) • Initial therapy for hypertensive


• Isosorbide dinitrate (2-10 mg/h) AHF (i.e., hypertensive emergency)
Vasodilators
• Others: nitroprusside, nesiritide, • Used for patients with pulmonary
serelaxin congestion for rapid relief of dyspnea
• Dobutamine (2-20 mcg/kg/min)
• Used for patients with hypotension,
• Dopamine (3-5 mcg/kg/min)
end-organ hypoperfusion, and/
Inotropes • Milrinone (optional 25-75 mcg/kg
or shock secondary to myocardial
bolus over 10-20 mins, then
pump failure
0.375-0.75 mcg/kg/min)

• Norepinephrine (0.2-1.0 mcg/kg/min) • Considered for refractory


Vasopressors • High-dosedopamine (>5mcg/kg/min) cardiogenic shock despite inotropic
• Epinephrine (0.05-0.5 mcg/kg/min) support
Source:McDonagh TA,et al. 2021ESCGuidelines.EurHeartJ. 2021
ZipesDP,et al. Braunwald's
HeartDisease.11thedition.Elsevier/Saunders,
2019

COR PULMONALE
I. ETIOPATHOGENESIS
Often referred to as "pulmonary heart disease"
Defined as altered RV structure and/or function in the context of chronic lung disease
and is triggered by the onset of pulmonary hypertension

II. MANIFESTATIONS
Dyspnea is the most common symptom and occurs due to increased work of breathing
Effort-related syncope due to inability of RV to deliver adequate blood volume to the LV
Abdominal pain and ascites: occurs due to backflow from right-sided heart failure
Orthopnea and PND: uncommon and occurs only with concurrent LV failure
RV heave: points to RV volume and pressure overload

III. DIAGNOSIS
DIAGNOSTIC I COMMENTS/EXPECTED FINDINGS

• P-pulmonale (P waves >2.5 m Vin leads II and/or V1)


12-lead ECG
• Right axis deviation and RV hypertrophy

• Enlargement of the main pulmonary artery, hilar vessels and


Chest radiographyj
descending right pulmonary artery
2D • Right sided chamber enlargement with associated dysfunction
echocardiography • Elevated pulmonary artery pressure

IV. MANAGEMENT
Target the underlying pulmonary disease: to decrease the underlying pulmonary
vascular resistance and lessen RV afterload
Pulmonary vasodilators: modest reduction of pulmonary pressure and RV afterload
192
CHRONIC CORONARY SYNDROMES (CCS)
OVERVIEW OF ISCHEMIC HEART DISEASE (IHD)
Patients with ischemic heart disease (!HD) fall into two large groups:
• Chronic coronary syndromes (CCS) which commonly present with chronic stable
angina pectoris (CSAP)
• Acute coronary syndromes (ACS), which are composed of:
0Non-ST-segment elevation acute coronary syndrome (NSTE-ACS), which includes:
• Unstable angina (UA)
• Non-ST-segment elevation myocardial infarction (NSTEMI)
0ST-segment elevation myocardial infarction (STEM!)

I. ETIOPATHOGENESIS
Inadequate supply of blood flow & 0, to a portion of the myocardium causing inadequate
perfusion of myocardium (ischemia) supplied by an involved coronary artery
!HD is most commonly caused by obstructive atherosclerotic plaque accumulation in
epicardial coronary arteries (coronary artery disease or CAD)
Guidelines have abandoned the term 'stable !HD' or 'stable CAD' in favor of'chronic
coronary syndrome,' acknowledging that CAD itself is a dynamic pathological process
Risk factors: hypertension, dyslipidemia, diabetes/insulin resistance, obesity, smoking

II. CLINICAL MANIFESTATIONS


A. Chronic Stable Angina Pectoris (CSAP)
0 Typical history involves a man >50 or woman >60 years old with chest discomfort:
• Described as heaviness, pressure, squeezing, smothering, or choking
• Crescendo-decrescendo in nature
• Usually lasts 2-5 minutes
• Associated with physical exertion or stress
• Radiation to either or both shoulders/arms, but does not radiate to trapezius muscles
• Relieved within 5 minutes by rest and/or sublingual nitrates
0 Levine's sign: hand placed over sternum with a clenched fist to indicate a squeezing,
central, substernal discomfort

I.Canadian Cardiovascular Society (CCS) Classification of Angina


ccs 1 DESCRIPTION I SAMPLE ACTIVITIES
• Angina occurs with greater than • Can perform to completion
ordinary physical activity (e.g., activities requiring >7 METs (e.g.,
I prolonged/rapid/strenuous exertion) basketball, jogging, climbing
• Ordinary activity does not cause angina stairs)
• Angina and slight limitation with • Can perform to completion activities
II ordinary physical activity (e.g., requiring >5 METs (e.g.,sex without
walking >2 blocks on the level) stopping, dancing, gardening)
• Can perform to completion
• Angina and marked limitation with
activities requiring >2 METs (e.g.,
III less than ordinary physical activity
showering, dressing up, playing
(e.g., walking 1-2 blocks on the level)
golf, bowling)
• Angina may be present even at rest
• Cannot perform activities
IV • Inability to carry out any physical
requiring ?:2 METs
activity without discomfort
Source:CampeauL. Gradingof anginapectoris(letter).Circulation.
1976
193
• Substemal chest discomfort of characteristic • Typical angina: meets all of the
#I
quality & duration (seedescriptionabove) 3 manifestations listed
• Atypical angina: meets only 2
#2 • Provoked by exertion or emotional stress of the manifestations listed
• Non-angina! pain: meets only 1
#3 • Relieved by rest &/or nitrates within mins or none of the manifestations
Source:KnuutiJ, et al. 2019ESCGuidelinesforCCS. EurHeartJ. 2020

B. Physical Examination
Assess for anemia, hypertension, valvular heart disease, or arrhythmias
0

Obtain blood pressure, heart rate, rhythm, and body mass index (BM!)
0

Search for other vascular diseases by palpating peripheral pulses or auscultating for bruits
0

Perform full cardiac examination to check for murmurs, S3, S4, chamber enlargements
0

APPROACH TO THE DIAGNOSIS OF CCS


DIAGNOSTIC I COMMENTS/EXPECTED FINDINGS
• CBC (hemoglobin levels): anemia may trigger ischemia
Basic
• Fasting lipid profile and fasting blood sugar/HbA1c
laboratory
• Baseline liver (AST, ALT) & kidney function (creatinine, BUN)
work-up
• Others: consider thyroid function tests, BNP/NT-proBNP
• May be normal at rest - but it does not exclude diagnosis of
ischemia (stress testing may be needed)
12-lead ECG
• May find ST-segment and T-wave changes, LV hypertrophy,
intraventricular conduction disturbances, arrhythmias
Chest radiograph • May be useful to rule out other non-cardiac causes of chest pain
• Used to assess left ventricular function and patients with a history of
2D
a prior MI, pathologic Q waves, or clinical evidence of heart failure
echocardiography
• Assess for wall motion abnormalities, LVEF, etc.
Dfa9,n1mfcs
.
Spl!cifi§JorCCS
..11' .. ,.
,.
Iii'"'"". - :;~
• To diagnose obstructive CAD by detecting ischemia provoked by
exercise or pharmacologic stressors (e.g., dobutamine stress)
Functional 0 ECG exercise testing: most widely used for both diagnosis of!HD
non-invasive and estimating prognosis
tests• 0 Stress imaging (stress echocardiography, radionuclide perfusion
or myocardial perfusion imaging, stress cardiac MRI): preferred
when the resting/baseline ECG is already abnormal
• Imaging technique for anatomical non-invasive evaluation of
Coronary CT
obstructive coronary lesions using IV contrast agents
angiography
• Used to rule out CAD, since it is a high sensitivity test
(CCTA)"
• Calcium score: calcified lesions are quantified using Agatston score
• Indicated if assessment suggests high risk for cardiovascular events
Invasive
• For planning of options for revascularization (PC! vs. CABG)
coronary
• To diagnose CAD if with high clinical likelihood & severe symptoms
angiography
refractory to medical therapy, or typical angina at a low level of exercise
"Non-invasivefunctionalimaging(forischemia)and/or coronaryCTA(foranatomy)are recommendedas
the initialtests for diagnosingCADin symptomaticpatients in whomobstructiveCADcannot be excluded
by clinicalassessment alone
Source:KnuutiJ, et al. 2019ESC GuidelinesforCCS.EurHeartJ. 2020
194
MANAGEMENT OF CCS
I. CONTROL OF RISK FACTORS AND LIFESTYLE MODIFICATION
ASPECT I RECOMMENDATIONS
• Diet low in saturated fats & salt; high in fruits, vegetables, whole grain, fish
Lifestyle • Limit consumption of alcohol (1-2drinks/day) & sugar-sweetened beverages
modifcation • 30-60 minutes of moderately vigorous exercise on most days of the week
• Smoking cessation (e.g. nicotine replacement therapy, bupropion)
• Summary of targets:
0 BM! <25 kg/m'; waist circumference <94 cm for males, <80 cm for females
0 LDL cholesterol <55mg/dL AND ~50% reduction from baseline
Control of
0 BP <130/80 mmHg (< 140/90 mmHg if~8o years old)
risk factors
•HbA1c<7.0%
• See guidelines for dyslipidemia, hypertension, & diabetes for more details
(discussedin other sections/chapters)
forCCS.EurHeartJ. 2020
Source:KnuutiJ, et al. 2019ESCGuidelines

II. PHARMACOLOGIC TREATMENT FOR PREVENTION OF MORBIDITY & MORTALITY


Three drug classes focus primarily on reducing acute thrombotic events (i.e., plaque
stabilization) & LV dysfunction in patients with CCS (shown to reduce mortality in CCS):

I
0 Aspirin
0 ACE-inhibitors (angiotensin receptor blockers as alternative)
0 Stalins
B-blockers &ACE-I: shown to reduce mortality in CCS with LV dysfunction following MI
CLASS I EXAMPLES I MECHANISM & REMARKS
• Irreversible inhibitor of platelet
cyclooxygenase activity, interfering with
• Aspirin 75-162mg OD platelet activation
• Chronic administration has been shown to
reduce coronary events
Antiplatelets • Blocks ADP receptor-mediated platelet
aggregation
• May be substituted for aspirin in those with
• Clopidogrel 75 mg OD
aspirin intolerance
• Preferable to aspirin if with history of PAD or
ischemic stroke/TIA

• RAAS inhibition
• For CAD: reduce LV & vascular hypertrophy,
ACE • Enalapril 2.5-20 mg BID
progression of atherosclerosis, plaque
inhibitors • Ramipril 2.5-5mg BID
rupture, & thrombosis
(ACE-I)& • Candesartan 4-32 mg OD
• ACE-I preferred over ARBs if tolerated
ARBs • Valsartan 40-160 mg BID
• Indicated for CCS patients with HPN, DM, LV
dysfunction/HFrEF, history of ACS, or CKD
• Inhibit HMG-CoA reductase
• Exhibit pleiotropic effects: plaque
• Rosuvastatin 20-40mg OD stabilization & anti-inflammatory effects
Statins
• Atorvastatin 40-80 mg OD • High-intensity therapy recommended in
established CAD to target LDL <55mg/dL &
~50% reduction from baseline
for CCS.EurHeartJ. 2020
Sources:KnuutiJ, et al. 2019ESCGuidelines
ZipesDP,et al. Braunwald's
HeartDisease.11thedition.Elsevier/Saunders,
2019

19S
III. PHARMACOLOGIC TREATMENT FOR RELIEF OF ANGINA (ANTI-ISCHEMIC DRUGS)
Beta-blockers and/or CCBs are recommended as the first choice for relief of angina
• Second line drugs include nitrates, ivabradine, trimetazidine, and ranolazine
CLASS I EXAMPLES I MECHANISM I REMARKS
• Reduce
• Cornerstone therapy for
• Atenolol 50-100 mg OD myocardial
angina
• Bisoprolol 5-20 mg OD 02 demand
• Target HR: 55-60 bpm
Beta- • Carvedilol 3.125-25mg BID by inhibiting
• Shown to improve life
Blockers • Metoprolol succinate increases in HR,
expectancy after recent MI
(BB) 50-200 mg OD arterial pressure
• Avoid in those with
• Metoprolol tartrate & contractility
asthma or COPD with
50-100 mg BID-QID from adrenergic
wheezing
activation
Non-Dihydropyridines • Indicated in patients with:
• Diltiazem 30-90mgTID-QID 0 Inadequate response
• Diltiazem ER 120-480mg OD to BB (DHP-CCB can
• Coronary
• VerapamilSo-120mgTID-QID be added)
vasodilators
• Verapamil SR 180-480mg OD 0 Adverse reactions to
that produce
Calcium ~-blockers
variable&
Channel 0 Angina & history of
dose-dependent
Blockers asthma or COPD
Dihydropyridines reductions in
(CCB) 0 Prinzmetal's angina

• Amlodipine 2.5-10mg OD O2demand,


• Side effect: ankle edema,
• Felodipine 2.5-10mg OD contractility, and
headache
• Nifedipine SR 90 mg OD arterial pressure
• Avoid combining BBs&
non-DHP CCBs (risk of
heart block, HF)

Short-acting for Acute Relief • Systemic


• SL nitroglycerin: most
• Isosorbide dinitrate (ISDN) venodilation
effective for acute relief
5 mg sublingual (SL) up to with reduction
of angina (SL ISDN has
3 doses, 5 mins apart in LVend-
slightly slower onset)
• Nitroglycerin 0.3-0.6 mg diastolic volume
•Maybe tapered off with
SL, up to 3 doses, 5 mins & pressure,
resolution of chest pain
apart thereby reducing
• Side effect: headache
myocardial wall
Nitrates • At least IO· to 14-hour
Long-acting for Prophylaxis tension &02
nitrate-free interval to
• lsosorbide mononitrate requirements
avoid nitrate tolerance
(ISMN) 30-240 mg OD • Dilation of
• Contraindicated in
• Nitroglycerin transdermal epicardial
HOCM, severe aortic
patch 0.2-0.8 mg/hr coronary vessels
stenosis, concomitant
(remove at bedtime for • Increased blood
use of phosphodiesterase
12-14hours) flow in collateral
inhibitors (e.g., sildenafil)
vessels

196
OTHER
REMARKS
DRUGS I MECHANISM
I
• Only works in patients who are in
• Inhibitor of the I, ion channel
sinus rhythm (cannot use in AF)
(principal determinant of the
• Approved for patients with CSAP
lvabradine SA node)
who are intolerant to BB or who are
2.5-7.5mg BID • Slows the heart rate through a
on optimal BBdose but still above
mechanism that is not associated
target HR
with negative inotropic effects
• Slightly increased risk of AF

• Dilates peripheral & coronary


• Has anti-angina! efficacy similar to
resistance vessels via ATP-
Nicorandil BBs, nitrates, and CCBs
sensitive K• channels
10-4omgBID • Side effects: nausea, vomiting, GI
• Has a nitrate moiety that promotes
tract ulcerations
venous & coronary dilation

• Inhibits fatty acid metabolism


• Reduces frequency of angina
Trimetazidine by blocking 3-ketoacyl-CoA
without hemodynamic effects
20 mgTID,or thiolase, which enhances glucose
• Contraindicated in Parkinson's
35 mg BID oxidation and produces more
disease and other motion disorders
ATP for less 0, consumption

Ranolazine
375-1000 mg BID
• Inhibits late inward sodium
current (IN,),leading to a shift
in myocardial substrate uptake
from fatty acid to glucose
• Favorable metabolic effects, including
modest reduction in HbAic
• Contraindicated with hepatic
impairment or use with drugs that
I
prolong the QT interval
• Anti-ischemic effects without
• Side effects:nausea, weakness,
hemodynamic consequences
constipation
Sources:KnuutiJ, et al. 2019ESCGuidelines for CCS.EurHeartJ. 2020.
ZipesDP,et al. Braunwald'sHeartDisease.11thedition.Elsevier/Saunders,
2019

IV. CORONARY REVASCULARIZATION


METHOD I REMARKS
• Balloon dilatation (angioplasty) usually followed by scenting of
coronary stenosis
Percutaneous • Most common indication is persistent or symptom-limiting angina
coronary pectoris, despite medical therapy, accompanied by evidence of
intervention ischemia during a stress test
(PCI) • Patients who have received a drug eluting stent (DES) will need:
0 Dual antiplatelets for I year (aspirin+ ADP-P2Y,,-inhibitor), then
0 Single antiplatelet indefinitely (usually aspirin)

• Achieves complete revascularization by grafting all coronary arteries


of sufficient caliber that have significant proximal stenoses
Coronary • Preferred approach for:
artery bypass 0 Left main CAD
grafting 0 3-vessel CAD, especially if with LV dysfunction
(CABG) 0 2-vessel CAD involving proximal left anterior descending artery,
with LV dysfunction or high risk findings on non-invasive tests
0 Patients with multi-vessel CAD & diabetes

197
SECTION SIX

UNIVERSAL DEFINITIONS OF MYOCARDIAL INJURY & INFARCTION


I. CLINICAL CLASSIFICATIONS OF MYOCARDIAL INFARCTION (Ml)
TYPE OF Ml
I CRITERIA
• Atherosclerotic plaque rupture, ulceration, fissure, or erosion with
resulting intraluminal thrombus in the coronaries
Spontaneous
I • Rise and/or fall of cTn with at least one value >99th percentile URL
MI
• At least one clinical criteria for ischemia/infarction' or identification of
a coronary thrombus by angiography or autopsy
• Imbalance between oxygen supply & demand unrelated to coronary
MI thrombosis (e.g., endothelial dysfunction, coronary spasm, coronary
secondary embolism, tachy- or bradyarrhythmias, anemia, respiratory failure,
2
to ischemic hyper- or hypotension)
imbalance • Rise and/or fall of cTn with at least one value >99th percentile URL
• At least one clinical criteria for ischemia/infarction•
MI resulting • Suffered cardiac death + symptoms suggestive of ischemia + presumed
in death new ischemic ECG changes or ventricular fibrillation, and:
when 0 Died before blood samples for biomarkers could be obtained, or
3
biomarkers 0 Before increases in biomarkers could be identified, or
unavailable 0 MI was detected by autopsy

• Elevation of post-procedure cTn values:


0 >5X the 99th percentile URL in patients with normal baseline values, or

PCI-related 0 Elevation >20% in patients with elevated pre-procedure cTn in


MI,;48 whom cTn levels are stable (:520%variation) or falling (absolute post-
43. hours procedural value must still be >5Xelevated)
after index • At least one clinical criteria for ischemia/infarction', or angiographic
procedure findings with flow-limiting complication (e.g., coronary dissection,
occlusion of epicardial artery or a side branch occlusion, disruption of
collateral flow, or distal embolization)
• Stent thrombosis detected by coronary angiography or autopsy
• Rise and/or fall in cTn & criteria for ischemia/infarction' as in type I MI
Stent
• Temporal categories (time after stent implantation):
thrombosis
41>associated 0 Acute: 0-24 hr

0 Subacute: >24 hr to 30 days


with PCI
0 Late: >30 days to I year

0 Very Late: >I year

Restenosis • Focal or diffuse restenosis, or a complex lesion on angiography


4c withPCI • Rise and/or fall in cTn & criteria for ischemia/infarction• as in type I MI
• Elevation of post-procedure cTn values:
0 >10 times the 99th percentile URL if with normal baseline cTn values
CABG-
0 Elevation >20% in patients with elevated pre-procedure cTn in
related MI
whom cTn levels are stable (,;20% variation) or falling (absolute post-
5 ,;48 hours
procedural value must still be >IOtimes elevated)
after index
• At least one clinical criteria for ischemia/infarction', or
procedure
angiographically documented new graft occlusion or new native
coronary artery occlusion
*Clinicalcriteriafor ischemia/infarction:
• Symptoms of acuteischemia
• NewischemicECGchanges
• Development of newpathologicalQ waves
• Imagingevidenceof newlossof viablemyocardium
or newregionalwallmotionabnormality
198
II. CRITERIA FOR MYOCARDIAL INJURY AND INFARCTION
. TERM I CRITERIA

• Detection of an elevated cardiac troponin (cTn) value >99th percentile URL


• Injury is considered acute if there is a rise and/or fall of cTn values
• In patients with increased cTn values, clinicians must distinguish
Myocardial
whether patients have suffered a non-ischemic myocardial injury or one
injury
of the MI subtypes (seeprevious table)
• If there is no evidence to support the presence of myocardial ischemia, a
diagnosis of myocardial injury should be made

• The clinical definition of MI denotes the presence of acute myocardial


Myocardial
injury detected by abnormal cardiac biomarkers in the setting of
infarction (Ml)
evidence for acute myocardial ischemia
Source:Thygesen definitionof myocardial
K,et al. Fourthuniversal 2018
J AmCollCard1ol.
infarction.

III. MYOCARDIAL INFARCTION WITH NON-OBSTRUCTIVE CORONARYARTERIES (MINOCA)


AMI occurring in the absence of obstructive (>50%) CAD
Alternative diagnoses must be ruled out, including sepsis, pulmonary embolism, and myocarditis
Cardiac MRI is helpful in tissue characterization: wall motion, edema, and scar/fibrosis
Etiologies can be grouped into:
• Coronary artery disorders with non-obstructive plaque or no CAD (Ml type 1)
Imbalance between 02 supply & demand (e.g. coronary spasm or embolism) (Ml type 2)
Coronary endothelial dysfunction (Ml type 2)
Myocardial disorders without involvement of coronaries (e.g.myocarditis,Takotsubo syndrome)
I
;_

IV. PRINZMETAL VARIANT ANGINA


Syndrome of severe ischemic pain that usually occurs at rest and associated with transient
ST-segment elevation
Caused by focal spasm of an epicardial coronary artery (most commonly the right coronary artery)
Diagnostic hallmark: coronary angiography demonstrates transient coronary artery spasm
Treatment: nitrates and calcium channel blockers (avoid aspirin and beta blockers - may
worsen spasm)

OVERVIEW OF ACUTE CORONARY SYNDROME (ACS)


Operational term that refers to a spectrum of conditions compatible with acute
myocardial ischemia &/or infarction due to an abrupt reduction in coronary blood flow
Hallmark of ACS: sudden imbalance betvveen myocardial oxygen consumption (MVO2)
and demand usually due to coronary obstruction

Non-ST Segment Elevation ACS


____
L-----...:.(_NS,--TE_-A_C_S...:) --t ST-Segment Elevation Ml
FEATURES Non-ST-Segment
Unstable (STEM!)
Elevation Ml
Angina (UA)*
(NSTEMI)
• Persistent or transient ST-segment • Persistent (>20 mins) ST-
ECG findings depression, T-wave inversion, flat T-waves segment elevation (see ECG
• Transient ST-segment elevation chapter for specific criteria)

Myocardial
• No • Yes • Yes
necrosis
'UA is definedas myocardialischemiaat resVminimal exertionin theabsenceof necrosis.Theintroduction
of highsensitivitycTnmeasurements resulledin an increasein the delectionof Ml anda decreasein the
diagnosisof UA. ElevaledbiomarkerlevelsdislinguishMl fromunstableangina.
Source:Collet,et al. 2020ESCGuidelines EurHeartJ. 2021
for NSTE-ACS.
199
I. MANIFESTATIONS OF ACUTE CORONARY SYNDROME (ACS)
• Leading symptom of suspected ACS is acute chest discomfort described as pain,
pressure, tightness, or burning
Angina or equivalent ischemic discomfort with at least one of the following:
0 Occurs at rest (or with minimal exertion), usually lasting >IO minutes
0 Severe and of new onset (i.e., within the prior 4-6 weeks) of at least CCS Ill severity
0 Occurs with a crescendo pattern (i.e., distinctly more severe, prolonged, or frequent
than previous episodes)
A K"ll" Cl s "fi t" f. P . ( (

CLASS I DESCRIPTION I RISK OF


MORTALITY*

• No rales or signs of pulmonary or venous congestion


I • Normal BP
6%

• Moderate heart failure


• Rales
II •

S3 gallop
Elevated JVP, hepatomegaly, and/or ankle edema
17%
• Normal BP
• Severe heart failure
III • Frank pulmonary edema
• Normal BP
38%

• Cardiogenic shock with SBP <90 mmHg & evidence of


peripheral vasoconstriction
IV • Peripheral cyanosis
81%
• Mental confusion, oliguria, diaphoresis
• Mortalitydata based on original1967paper of Killipand Kimball;due to advances in pharmacotherapy,
PCI techniques & equipment, and overallmanagement, figures may now be lower.

• Typical pain with radiation to shoulders • Inframammary location of pain


• Typical pain associated with exertion, • Pain reproducible with palpation
diaphoresis, nausea, or vomiting • Pain described as sharp, positional, or
• Pale cool skin, tachycardia, S3 or S4, pleuritic
basilar rales, hypotension•
• Initialexaminationof patients may be normal

II. DIAGNOSIS OF ACUTE CORONARY SYNDROME


A. 12-Lead Electrocardiogram (ECG)
0 Should be performed and interpreted within IO minutes of patient's arrival (or
preferably in the ambulance en route to the hospital)
0 Differentiates between STEMI & NSTE-ACS, which determines urgency of
revascularization (STEMI patients require emergent PC! or fibrinolysis, discussedlater)
0 May be initially normal, hence serial ECGs should be done to detect dynamic ischemic
changes if patient remains symptomatic with a high index of suspicion for ACS

B. Conventional Cardiac Troponin (Traditional Assays)


0 Serial cardiac troponin I or T levels should be obtained to identify a rising and/or
falling pattern of values (two negative cTn assays at least 6 hours apart are needed to
exclude MI with these less-sensitive assays):
• First level taken at presentation
• Second level taken at 3-6 hours after symptom onset
• Another sample taken beyond 6 hours in those with normal levels on serial
examination, when ECG &/or presentation confer an intermediate to high index of
suspicion for ACS
° Cut-off troponin value for necrosis: 99th percentile of the upper reference level
° Can detect recent MI up to 2 weeks after onset (remain elevated 7-IO days after Ml)
200
C. High-Sensitivity Cardiac Troponin (hs-cTn)
0 Due to its higher sensitivity & accuracy, the time interval to 2nd hs-cTn determination
can be shortened to I hour, resulting in faster disposition & discharge times at the ER
0 hs-cTN is a quantitative marker of myocyte damage (i.e., higher levels suggest a greater
likelihood of MI, with elevations >5Xhaving a 90% positive predictive value for Ml)
0 Other conditions may result in hs-cTn elevations: tachyarrhythmias, heart failure,
hypertensive emergencies, critical illness, myocarditis, valvular heart disease,
pulmonary embolism, renal failure, and stroke/subarachnoid hemorrhage
DI : :£ ACS
DIAGNOSTIC I REMARKS
• To identify other differentials of chest pain
Chest radiograph
• Widened mediastinum: may be seen in aortic dissection
Transthoracic • Abnormalities of wall motion are almost universally present in ACS
echocardiography • Identifies pericardia! effusion, concomitant valve abnormalities, etc.
• Recommended as an alternative to invasive angiography to exclude
Coronary CT
ACS when there is a low-to-intermediate likelihood of CAD &
angiography
when troponin and/or ECG are normal or inconclusive
(CCTA)
• Can rule out an ACS if with normal findings
• Myocardial perfusion imaging for prognostication and to check for
Others myocardial viability (e.g., MP! with thallium or sestarnibi)

I
• Cardiac MRI can assess perfusion and wall motion abnormalities

III. OVERVIEW OF MANAGEMENT OF ACS


A. Anti platelet Therapy for ACS
1.Aspirin
• Established first-line therapy for ACS
• Platelet cyclooxygenase inhibitor which blocks synthesis & release of thromboxane-A2
(platelet activator), causing decreased platelet aggregation & thrombus formation
• Dose: 162-325mg LD chewed (non-enteric-coated), then 81 mg OD indefinitely
• Contraindications: active bleeding or aspirin intolerance
2. P2Y,,-Inhibitors
• In the absence of a high risk of bleeding, patients with ACS should also receive a P2Y,,
inhibitor for up to 12months
• Thrombotic/ischemic risk must be balanced with bleeding risk in determining the
duration of dual anti platelet therapy (DAPT), particularly for those who undergo PC!

I CLOPIDOGREL
I TICAGRELOR*
I PRASUGREL*

Class • Thienopyridine • Non-thienopyridine • Thienopyridine


Inhibition • Irreversible • Reversible • Irreversible
Onset of effect • 2-6 hours • 0.5-4 hours • 0.5-2 hours

• 3oomgLD • 60 mg LD, then


(conservative) or 600 • 180 mg LD, then 0 10 mgODx1 year
Dose
mg LD (invasive),then • 90mg BID x I year (5 mg OD if age 275
• 75 mg OD x1 year or weight <60 kg)

Withdrawal
• 5 days • 5 days • 7 days
before surgery

• For those managed conservatively or • Only for those


Use in ACS
invasively (PC!) managed with PCI°
LD:loadingdose PCI:percutaneous
coronaryintervention
'Ticagrelorandprasugrel
arepreferred overclopidogrel (forACS)
"Prasugrelis contraindicated
in patientswitha priorstrokeor transientischemic
attack(TIA)
201
• Sublingual ISDN (5 mg) or • Decreases pre load (venodilation) & afterload
NTG (0.3-0.6 mg) every 5 (arterial dilation)
minutes (max of3 doses • Benefits may be offset by reflex increase in HR &
Nitrates • IV NTG in the first 48 hours contractility (unless a BB is given concurrently)
(5-10mcg/min) for persistent • Avoid for hypotensive patients, suspected RV
ischemia, heart failure or infarction, and on sildenafil (or other PDE-5
hypertension inhibitors) for the past 24 hours
• Decreases HR, contractility & BP, resulting in
• Bisoprolol 5-20 mg PO OD
decreased MVO2
• Carvedilol 3.125-25mg PO BID
Beta- • Initiated in the first 24 hours, except if with
• Metoprolol succinate
blockers acute heart failure, low output states, PR
50-2oomgOD
(BB) interval ?:0.24seconds, 2nd or 3rd degree AVB
• Metoprolol tartrate
without a pacemaker, or active asthma
25-50 mg PO q6-12
• Target HR 50-60 bpm is recommended
• Amlodipine 2.5-10mg OD • Induce coronary vasodilation
Calcium
• Felodipine 2.5-10mg OD • Recommended for recurrent ischemia after
channel
• Diltiazem 30-90 mg TID-QID appropriate use of BB and nitrates
blockers
• Verapamil 80-120mgTID-QID • For those with contraindications to B-blockers
(CCB)
• Do not use Nifedipine • Preferred over BB for vasospasm (Prinzmetal)

• Venodilator effects reduce preload


• Increase in vagal tone also reduces HR & BP
• Morphine 1-5mg IV, repeated
Opioids • Given for persistent pain despite nitrates & BB
q5-30 mins until pain relieved
• Adverse events: hypotension, respiratory
depression
• Not routinely given
• Nasal cannula or face mask at
• Administer supplemental oxygen only if with
Oxygen 2-4 L/min for the first 6-12hrs
hypoxemia (02 sat <90% or PaO2 <60 mmHg),
support • Higher support may be given
cyanosis, or respiratory distress
if with respiratory failure
• Target 02 sat >95%
Sources:ZipesDP,et al. Braunwald's HeartDisease.11thedition.Elsevier/Saunders,
2019
Amsterdam EA,et al. 2014AHNACCNSTE-ACS Guideline. J AmCollCardiel.2014
Collet,et al. 2020ESCGuidelines for NSTE-ACS. EurHeartJ. 2021

C. Standard Medications for ACS


CLASS I EXAMPLES I REMARKS
• Captopril 6.25-50mg TIO • Reduces mortality in those with recent MI
ACE-I • Enalapril 2.5-5mg BID • Should be initiated in the first 24 hours
• Ramipril 2.5-5mg BID • Indications:anterior STEM!, LVEF s,io%, HPN, OM

• Candesartan 4-32mg OD
• Second-line drug if intolerant to ACE-
ARB • Losartan 50 mg BID
inhibitors
• Valsartan 40-160mg BID

• Recommended in post-Ml patients without


• Eplerenone 25-50 mg OD significant renal dysfunction or hyperkalemia,
MRA
• Spironolactone 12.5-50mg OD on top of ACE-I and BB
• Indications: HF with LVEF s,io%, OM

• High intensity statin therapy should be initiated


• Atorvastatin 40-8o mg OD HS
Statins or continued (early administration reduces
• Rosuvastatin 20-40 mg OD HS
adverse outcomes)
forSTEMI.EurHeartJ. 2018
Sources:IbanezB,et al. 2017ESCGuidelines
O'GaraPT,et al. 2013ACCF/AHA STEMIGuideline.
Circulation.
2013
202
• Mainstay of therapy
• 60 JU/kg IV bolus (max of • Prevents coagulation by blocking
4000 IU), then 12IU/kg/hr thrombin (factor Ila) and factor Xa
Unfractionated
infusion (max of1000 JU/ • Monitoring of aP1T is needed to
heparin (UFH)
hr) for 24-48 hours or until target 50-70 seconds or 1.5-2.5xcontrol
PC! is performed • Risk for heparin-induced
thrombocytopenia (HIT)
• Superior to UFH in reducing
• 30 mg IV LD, then I mg/
recurrent cardiac events, especially in
kg SC q12 for the duration
patients managed conservatively
of hospitalization or until
Low molecular • Greater anti-factor Xa activity
PC! is performed
weight heparin (relative to factor Ila) inhibits
• For '?.75years old: no IV
(enoxaparin)• thrombin generation more effectively
bolus needed
• No aPTI monitoring needed (unlike
• Adjustment: 1 mg/kg SC
UFH)
q24 for CrCl <30 mUmin
• Less risk for HIT
• Indirect factor Xa inhibitor
• Equivalent in efficacy to enoxaparin,
• 2.5 mg SC OD for the
but with lower risk of major bleeding
Fondaparinux• duration of hospitalization
• Should not be used as sole
or until PC! is performed
anticoagulant to support PC!, due to
increased risk of catheter thrombosis
'Durationis upto 8 days,untilhospitaldischarge,
or untilPCIis performed,
whichever
comesfirst.Consider
discontinuation aftersuccessfulPCI (unlessprocedurehadthromboticcomplications or thereare other
clinicalindicationsforanticoagulation;
e.g.,intracardiac
thrombus, AF)
Source:
Amsterdam
EA,et al.2014AHA/ACC
NSTE-ACS J AmCollCardiol.2014.
Guideline.

E. Supportive Care & Long-Term Management


ASPECT I DESCRIPTION

• First 12hours: bed rest with continuous ECG monitoring


• Next 12hours: dangling of feet at bedside and sitting in a chair
Activity • 2nd and 3rd day: ambulation in the room with increasing duration
and frequency to a goal of 185m (600 ft) at least 3x a day
• 2 weeks: resumption of work and sexual activity
• Nothing or only clear liquids (due to risk of emesis and aspiration) for
Diet
the first 4-12 hours
Bowels • Use of stool softener to avoid straining during bowel movement
• Many patients require sedation during hospitalization to withstand
Sedation
periods of enforced inactivity
Secondary • Management for secondary prevention & risk reduction: same as in
prevention CCS (discussedin previoussection)

203
NON-ST-ELEVATION ACUTE CORONARY SYNDROME (NSTE-ACS)
Most commonly caused by plaque rupture or erosion with superimposed non-occlusive
thrombus, leading to an imbalance between oxygen supply and demand

I. RISK STRATIFICATION SCORES FOR NSTE-ACS


Clinical variables and findings from the 12-lead ECG & cardiac biomarkers can be
consolidated in order to better prognosticate patients who are admitted for NSTE-ACS
Helps in identifying high-risk patients who would benefit from an early invasive strategy
Two examples of commonly used risk scoring systems are:
0Thrombolysis in Myocardial Infarction (TIMI) Risk Score
0Global Registry of Acute Coronary Events (GRACE) Risk Score

A. TIMI Risk Score for UA/NSTEMI


COMPONENT I POINTS
Age 2:65years
2:3CAD risk factors (family historyof CAD,HPN,dyslipidemia,DM, currentsmoker)
Known CAD (2:50%stenosis)
Aspirin use in the past 7 days
Severe angina in last 24 hours
Elevated cardiac markers
ST deviation 2:0.5mm
• Risk Total (0-7points)
• High risk: TIMI~3 points (13%risk of death, recurrent Ml,or urgent revascularization in 14days)
Source:AntmanEM,et al. The TIMInsk score for UA/NSTEMI. JAMA.2000.

B. GRACE Risk Score for ACS (available online or on smartphone apps)


0There are 8 components in the GRACE score: age, SBP, HR, Killip, creatinine, cardiac
arrest on admission, ST-segment deviation on ECG, and elevated cardiac enzymes
0These components are all scored upon patient admission
0High risk: GRACE score >140 (2.9% in-hospital mortality)

II. INVASIVE APPROACH TO NSTE-ACS (ROUTINE VS. SELECTIVE INVASIVEAPPROACH)


• Routine versus selective invasive strategy:
0Routine invasive strategy: all NSTE-ACS patients to undergo invasive angiography
0Selective (ischemia-guided) strategy: invasive angiography done if with recurrent
symptoms, inducible ischemia on non-invasive testing, or obstructive CAD on CCTA
Timing of angiography is dependent on the criteria below:
RISK I CLINICAL CHARACTERISTICS I STRATEGY
• Hemodynamic instability or cardiogenic shock
• Recurrent/refractory chest pain despite medical therapy
Very • Life-threatening arrhythmias/electrical instability • Immediate
High • Mechanical complications of MI Invasive
Risk • Acute heart failure clearly related to NSTE-ACS (<2 hours)
• ST segment depression >I mm in 6 leads and ST segment
elevation in aVR and/or V1 (likely left main disease)
• Established NSTEMI diagnosis (elevated cTn)
, Early
High • Dynamic new or presumably new contiguous ST-T wave changes
Risk Invasive
• Resuscitated cardiac event without ST elevation or cardiogenic shock
(<24hours)
• GRACE risk score >I.40
Low • None of the above clinical criteria • Selective
Risk • No recurrence of symptoms Invasive
*Ifa highor very highrisk patientis initiallybroughtto a non-PCl-capablecenter, transferto the nearest PCI-
capable center so that time targets for invasiveangiography& urgent PCI are met
Source:Collet,et al. 2020ESCGuidelinesforNSTE-ACS. EurHeartJ. 2021
FoxKA,et al. BMJ.2006
204
ST-ELEVATION MYOCARDIAL INFARCTION (STEMI)
Acute plaque rupture is central to the pathogenesis of STEM!
STEM! occurs when coronary blood flow decreases abruptly after a thrombotic (usually
total) occlusion of a coronary artery previously affected by atherosclerosis

I. RISK STRATIFICATION SCORE FOR STEM!


TIMI risk score for STEM! predicts 30-day mortality
Designed for risk assessment early after patient presentation, hence does not
incorporate noninvasive and invasive data
COMPONENT I POINTS COMPONENT I POINTS
History & Presentation Physical Exam _,
• Age 65-74 2 • Systolic BP <JOOmmHg 3
• Age 2 75 3 • Heart rate> IOO bpm 2
• DM, HPN, angina 1
• Anterior ST Elevation or LBBB(ECG) 1 • Killipll-IV 2
• Time to Treatment >4 hours 1 • Weight <67kg 1

• Risk Total (0-14points)


• High risk: TIMI score 25 points (12% mortality at 30 days)
Source:MorrowDA,et al.TIMIRiskScoreforSTEMI.Circulation.
2000

II. REPERFUSION THERAPY FOR STEM!: PRIMARY GOAL OF MANAGEMENT


Initial goal in the management of STEM! would be to identify candidates for urgent
reperfusion (either by primary PC! or fibrinolysis), ideally within 12hrs of symptom onset .:_.
• Reperfusion is reasonable if symptom onset is between 12-24 hours + evidence of
ongoing ischemia (primary PC! is preferred in these patients)

Time to ECG • FMC to ECG: s10 mins • FMC to ECG: ,;w mins
• Time from STEM! diagnosis to start of
Thrombolysis • Door-to-needle time: s30 mins
fibrinolysis: s10 mins
• Time from STEM! diagnosis to wire
PC! • Door-to-balloon time: s90 mins
crossing: ,;60 mins

Decision between Primary PCI vs. Fibrinolysis

• Time-to-treatment delays are short: symptoms <12hours & PCI can be


performed ,;120mins from STEM! diagnosis
Primary PC! • Contraindications to fibrinolysis (high bleeding risk), irrespective of time
preferred in delay from FMC
the following • High risk from STEM! (cardiogenic shock, Killip III-IV)
• Late presentation of patient (symptom onset> 3 hours)
• Diagnosis of STEM! is in doubt
• Symptoms <12hours and primary PC! cannot be performed within 120mins
Fibrinolysis from STEM! diagnosis
preferred in • Prolonged transport to PCI-capable center with FMC-to-device time
the following >120mins (if initially received in non-PCI-capable center)
• PC! is not available
Definition
of terms:
• FMCor FirstMedicalContact:timepointwhenpatientis initiallyassessedby medicalpersonnel
• STEMIdiagnosis: timeat whichECGof patientwithischemicsymptoms is interpreted
as "ST-segment
elevation
or equivalent"
• Door-to-needle:
timefromarrivalin theemergency department to thetimefibrinolysisstarted
• Door-to-balloon:
timefromarrivalin theemergencydepartment to thetimea coronaryguidewire crosses
theculpritlesionduringPCI
205
• Emergent PC! with balloon, scent, or other device, performed on the
infarct related artery without previous fibrinolytic treatment
• For symptoms s12 hours: PC! is recommended (over fibrinolysis) when
Primary PC! it can be performed in a timely fashion (see previous table)
• For symptoms >12 hours: PC! is recommended in the presence of ongoing
ischemia, hemodynamic instability, or life-threatening arrhythmias
• In asymptomatics, PC! >48 hrs after onset of STEM! is not indicated
Pharmacoinvasive • Fibrinolysis combined with rescue PC! (in case of failed fibrinolysis) or
strategy routine early PC! strategy (in case of successful fibrinolysis)
• Emergent PC! performed immediately for failed fibrinolysis (persistent
Rescue PC!
chest pain & <50% ST-segment resolution at 60-90 ruins after)

Routine early
• Angiography with PC! of infarct related artery, performed between
PCI strategy
2-24 hours after successful fibrinolysis
after fibrinolysis

B. Fibrinolysis/Thrombolysis for STEM!


0 Largest benefit when offered <2 hours after symptom onset
0 It is the re perfusion strategy when PC! cannot be done in a timely manner
0 Mechanism: promotes conversion of plasminogen to plasmin, which then lyses fibrin thrombi
0 With the pharmacoinvasive approach, routine angiography & early PC! is still recommended
even after successful fibrinolysis (transfer to PCI-capable center within 24 hours if necessary)

1. Fibrinolytic Agents•
• Alteplase (tPA) 15mg IV bolus, then 0.75 mg/kg IV x 30 min, then 0.5 mg/kg IV x 60 min
• Reteplase (rPA)
• Tenecteplase (TNK-tPA)
• Streptokinase 1.5million units IV x 30-60 mins
*Fibrin-specific
agentsarerecommended
(tenecteplase,
alteplase,
reteplase)

• Any prior intracranial hemorrhage (!CH) • History of chronic, severe, poorly


• Structural cerebral vascular lesion (e.g., controlled hypertension
arteriovenous malformation) • Significant HPN at initial evaluation
• Malignant intracranial neoplasm (SBP >180 or DBP >IIO mmHg)
• Ischemic stroke within 3 months (except • History of previous ischemic stroke
acute ischemic stroke within 4.5 hours) >3 months
• Suspected aortic dissection • Dementia
• Active bleeding or bleeding diathesis • Intracranial pathology not covered in
(excluding menses) Absolute Contraindications
• Closed-head or facial trauma within • Traumatic or prolonged (>IO minutes)
3 months cardiopulmonary resuscitation
• lntracranial or intraspinal surgery within • Major surgery (<3 weeks)
2 months • Recent (within 2-4 weeks) internal bleed
• Severe uncontrolled hypertension • Noncompressible vascular punctures
(unresponsive to emergency therapy) • Pregnancy
• For streptokinase, previous treatment • Active peptic ulcer
within the last 6 months • On oral anticoagulant therapy
Source:O'GaraPT,et al.2013ACCF/AHA STEMIGuideline.
Circulation.
2013
forSTEMI.EurHeartJ. 2018
IbanezB, et al. 2017ESCGuidelines
206
SECTION SEVEN
RHEUMATIC FEVER AND VALVULAR HEART DISEASE
RHEUMATIC FEVER (RF)
I. ETIOPATHOGENESIS
Autoimmune reaction to infection with Group A Beta-Hemolytic Streptococcus
• Antibodies against M-proteins of certain strains of Streptococcuscross-react with tissue
glycoproteins in the heart, joints, and other tissues (molecular mimicry)

II. CLINICAL MANIFESTATIONS


Latent period of3 weeks (ranges from I to 5 weeks) between the precipitating infection
and the appearance of the clinical features of acute RF
MANIFESTATION I REMARKS

• Pancarditis involving the pericardium, myocardium, and endocardium


Carditis • Hallmark is valvular damage (only valvulitis leads to permanent damage)
(50-60%) • Characteristic manifestation is mitral regurgitation (sometimes with AR)
• Single most important prognostic factor in RF
• Most frequent major manifestation of RF (responsive to salicylates & NSA!Ds)
Migratory • Typically migratory over a period of hours
polyarthritis • Most often asymmetric & affecting large joints (ankles, knees, hips, elbows)
(60-75%) • Monoarthritis & polyarthralgia now considered as major manifestation
in high-risk populations
Sydenham's • Involuntary jerking movements mostly affecting the head & upper limbs
chorea • Usually resolves completely within 6 weeks
Erythema • Evanescent pink macular eruption with round borders & central clearing
marginatum • Concentrated on trunk, sometimes on the limbs, but almost never on the face
Subcutaneous • Round, firm, freely movable lesions varying in size from 0.5-2.0 cm
nodules • Painless small lumps found over extensor surfaces of joints

III. DIAGNOSIS (REVISED JONES CRITERIA FOR THE DIAGNOSIS OF RF)


Initial ARF: 2 major, or I major+ 2 minor AND evidence of preceding GAS infection
• Recurrent ARF: 2 major, or I major+ 2 minor, or 3 minor AND preceding GAS infection
LOW-RISK POPULATIONS* I MODERATE & HIGH-RISK POPULATIONS

'MajorCriteria ,, ·•,

• Carditis (clinical or subclinical) • Carditis (clinical or subclinical)


• Arthritis (polyarthritis only) • Arthritis (polyarthritis,monoarthritis, or polyarthralgia.. )
• Chorea • Chorea
• Erythema marginatum • Erythema marginatum
• Subcutaneous nodules • Subcutaneous nodules
MinorCriteria
-
- i,' ...

• Polyarthralgia • Monoarthralgia
• Fever 2: 38.5 °C • Fever 2: 38 °C
• Elevated ESR 2: 60 mm in the first • Elevated ESR 2: 30 mm/hr &/or CRP 2: 3.0 mg/dL
hour &/or CRP 2: 3.0 mg/dL • Prolonged PR interval on ECG
• Prolonged PR interval on ECG

Evidenceof Pre~edingGroupA Strep,tpeaccal(9AS) infection ~J


...
• A positive throat swab culture, or
• Elevated or rising anti-streptolysin-O or anti-DNase B titers, or
• Rapid antigen test for group-A streptococcus
'Low-riskpopulations:
rheumatic
heartdisease
prevalence
of S1per1000populationperyear
shouldonlybe considered
"Polyarthralgia asa majormanifestation
in moderate-
to high-risk
populations

207
IV. MANAGEMENT

For treatment
• Also known as primary prophylaxis or primary prevention of RF
of streptococcal
• See table below
tonsillopharyngitis
• First-line: aspirin IOO mg/kg/din 4-5 divided doses up to 2 weeks
For arthritis/mild
• Dose can be decreased to 60-70 mg/kg/day for an additional 3-6 weeks
carditis
• For aspirin allergy: naproxen I0-20 mg/kg/d (BID dosing)
• May add prednisone 1-2 mg/kg/day (max 80 mg/day) up to
For moderate-
maximum of3 weeks (then dose decreased and tapered by 20-25%
severe carditis
each week)
For severe chorea • Carbamazepine or valproic acid preferred over haloperidol
Sources:
ZipesDP,etal.Braunwald's HeartDisease.
11thedition.
2019
JamesonJL,etal.Harrison's
Principles
of Internal
Medicine
20thedition,
2018
WHO.Rheumatic
feverandrheumaticheartdisease:
reportofa WHOexpertpanel.2004
Gewitz
MH,etal.RevisedJonesCriteria
forAcuteRheumaticFever.
Circulation.
2015

B. Prophylaxis for Rheumatic Fever (RF)


PROPHYLAXIS I GOAL I DRUGS

• Penicillin:
0 Penicillin VK PO 250-500 mg TID x
IO days, OR
0 Amoxicillin I g PO OD x IO days, OR
• Treat group-A 0 Benzathine penicillin G 1.2 million
streptococcal URTI
Primary units IM injection as single dose
& eradicate the
prophylaxis • If with penicillin allergy:
organism to prevent
forRF ° Clindamycin PO 300-600 mg PO TID x
an initial attack of
IO days, OR
acute RF
0 Azithromycin PO 500 mg PO OD x
5 days, OR
° Clarithromycin 250 mg PO BID x
IO days

• Prevent colonization
&/or infection in • Benzathine penicillin G 1.2 million units
Secondary patients who have IM injection every 3-4 weeks, OR
prophylaxis had a previous • Penicillin VK PO 250 mg PO BID, OR
forRP attack of RF (to • Erythromycin 250 mg PO BID (if with
prevent recurrence penicillin allergy)
ofRF)

*Duration
of Secondary RFProphylaxis:
• RFwithoutcarditis:5 yearsafterlastattackor until21 yearsold (whicheveris longer)
• RFwithcarditis,but no residualvalvulardisease:10yearsafterlastattackor until21 yearsold
(whicheveris longer)
• RFwithcarditisand persistentresidualvalvulardisease:10yearsafterlastattackor until40
yearsold (whicheveris longer,sometimeslifetimeprophylaxis)
• Lifetimeprophylaxisshouldbe continuedevenaftervalvereplacement
Sources:
WHO.Rheumatic feverandrheumatic
heartdisease:
reportofa WHOexpertpanel.2004
Gerber
MA,etal.AHAScientific
Statement
onRheumatic
FeverandStreptococcal
Pharyngitis.
Circulation.
2009

208
VALVULAR HEART DISEASE (VHD)
Refers to cardiovascular diseases that involve one or more of the four valves of the heart
(i.e., aortic & mitral valves on the left side; pulmonic and tricuspid valves on the right side)
Degenerative disease is the most common cause in developed countries
Rheumatic heart disease (RHD) is most common in developing nations

• Dyspnea
, Dyspnea • Acute: pulmonary edema & acute HF
• Fatigue
Presentation • Angina • Chronic: fatigue, palpitations, dyspnea,
• Exercise
• Syncope heart failure
intolerance
• Irregular pulse • Displaced apex
, Weak & delayed • Widened pulse
(if AF) beat with brisk
pulse (pulsus pressure
Physical • Elevated )VP systolic impulse
parvus et tardus) • Inferolaterally
exam • Malar flush; • Soft or absent S1
• Sustained LV displaced LV
pinched, blue • S3 in severe
apical impulse apical impulse
facies acute MR
• Midsystolic • High-pitched,
ejection at 2nd decrescendo,
• Apical diastolic
JCS RPSB (± blowing diastolic
rumble • Apical
Murmur thrill) at 3rd !CS LPSB
preceded by holosystolic
• Gallavardin effect: • Austin Flint: low-
opening snap
transmitted to apex pitched rumbling
(resembles MR) diastolic at apex
• Acute MR: • Acute AR: IE,
• Degenerative papillary muscle aortic dissection
• RHD(most
calcification rupture (from • Chronic AR: RHD,
Common common)
(most common) ACS), IE degenerative,
etiologies • Congenital
• RHD • Chronic MR: aortic dilatation
• Calcific
• Congenital BAV RHD,MVP, (e.g., Marfan's
cardiomyopathy syndrome)
Diagnosis
• LAE • Rounding of apex • LV apex displaced
• LAE,LVH
Chest X-ray • RAE&RVH • Dilated proximal inferolaterally in
• Sometimes RAE
eventually ascending aorta chronic AR
• lAE±RAE&RVH • LVH with strain • LAE,LVH • LVH with strain
ECG
• MayhaveAF pattern • May have AF pattern
Management•
• Rate control: • Caution with
BBs, non-DHP nitrates/ vasodilators
• Vasodilators decrease afterload & may
Drugs CCBs, digox.in (can precipitate
reduce MRI AR severity
• Warfarin if hypotension)
in AF • Avoid BBs
• Transcatheter
Structural • PTMC/PMBV , TAVI/TAVR
MY repair • A YR surgery
intervention • MVR surgery • AYR surgery
• MVR surgery
'Diuretics are given across all types of VHD,as needed for fluidoverload. RF prophylaxisgiven ifwith RHO.
ACS: acute coronarysyndrome PTMC:percutaneous transseptal mitralcommissurotomy
IE: infectiveendocarditis PMBV:percutaneous mitralballoonvalvuloplasty
RHO:rheumaticheart disease MVR:mitralvalve replacement (surgical}
BAV:bicuspidaortic valve TAVI/TAVR: transcatheter aortic valve implantation/replacement
MVP:mitralvalve prolapse AVR:aortic valve replacement (surgical)

209
SECTION EIGHT
VENOUS THROMBOEMBOLISM
VENOUS THROMBOEMBOLISM (VTE)
Simply means "a blood clot in the veins"
Encompasses deep venous thrombosis (DVT) and pulmonary embolism (PE)
Virchow's triad: venous stasis+ hypercoagulability + endothelial injury

I. DEEP VENOUS THROMBOSIS (DVT)


• Presence of thrombus in one of the deep venous conduits that return blood to the heart
Most commonly involves the deep veins of the leg or arm, often resulting in potentially
life-threatening emboli to the lungs (e.g., pulmonary embolism) or debilitating valvular
dysfunction and chronic leg swelling

A. Clinical Manifestations
° Cramp in the lower calf (most frequent symptom)
0Leg swelling and pain
0Presence of risk factors (e.g., older age, prolonged immobilization, paralysis,
hyperviscosity syndromes)

B. Diagnostics for DVT


DIAGNOSTIC I REMARKS
Venous duplex • Best non-invasive diagnostic method which detects vein non-
scan compressibility (most definite sign of thrombosis)
• High negative predictive value (rule out test)
D-dimer
• Used for a suspected DVT in a low-probability situation
Computed
• Simple way to diagnose DVT
tomographic
• Can be combined with CT pulmonary angiography to diagnose PE
(CT) venography
.. for acutePE.EurHeartJ. 2020
SV,et al. 2019ESCGuidelines
Source:Konstantm1des

II. PULMONARY EMBOLISM (PE)


Blockage of the main artery of the lung (or its branches) by a substance from elsewhere
in the body (embolism)
Increase in pulmonary vascular resistance from PE causes increase in RV wall tension,
leading to progressive right heart failure (the usual cause of death from PE)
Increase in anatomic dead space: a completely obstructed pulmonary artery (PA) would
increase the proportion of the tracheobronchial tree incapable of effective gas exchange
Increase in physiologic dead space: air ventilation to gas exchange units exceeds venous
blood flow through pulmonary capillaries (increased ventilation to perfusion ratio> 1.0)

A. Sources ofEmbolization
0Pelvic vein thrombosis or proximal leg DVT
0Isolated calf thrombi (most common source of paradoxical embolism - e.g.,through an ASD)
0Upper extremity venous thrombosis
0Others: air em bolus, fat em bolus, amniotic fluid

B. Clinical Manifestations
0Most frequent history is unexplained breathlessness
0Dyspnea is the most frequent symptom, and tachypnea is the most frequent sign of PE
° Clinical presentation depends on the size of pulmonary embolism

210
SMALL TO
MODERATE MASSIVE PE
(5-10%)
I PE (70%)
• Pleuritic
Usual • Dyspnea,
pain, cough, • Variable
symptoms syncope
hemoptysis
• Unstable (see
definitionbelow)
Hemodynamics • Stable
• Extensive
thrombosisb
• Either elevated
Biomarkers' • Elevation ofbiomarkers AND
• Both are biomarkers or
and RV size/ moderate/severe RV dysfunction/
normal RV dysfunction
function enlargement
(but not both)
• Options for • Anticoagulation,
• Anticoagulation
anticoagulation: followed by
and consider • Anticoagulation
0 Heparin+ "watch & wait"
Management advanced plus advanced
warfarin • Advanced
therapy therapy'
° Fondaparinux therapy if with
(controversial)
0 DOAC deterioration
• Biomarkersincludecardiac troponinor NT-proBNP2:600ng/L
b Extensivethrombosiswouldincludea saddle embolus or a rightor left main pulmonaryartery thrombus

'Advanced therapy: systemic thrombolysis,catheter-directedtherapy, surgicalembolectomy,IVCfilter

,
CRITERIA I REMARKS
Cardiac arrest • Need for cardiopulmonary resuscitation

• SBP <90 mmHg or vasopressors required to achieve SBP 2:90mmHg, AND


Obstructive
• End-organ hypoperfusion (e.g., altered mentation, cold clammy skin,
shock
oliguria/anuria, increased serum lactate)
Persistent • SBP <90 mmHg or SBP drop ~40 mmHg, lasting >15mins and not
hypotension caused by arrhythmias, hypovolemia, or sepsis
Sources: KonstantinidesSV,et al. 2019 ESC Guidelinesfor acute PE. Eur HeartJ. 2020
Zipes DP,et al. Braunwald'sHeart Disease. 11thedition.Elsevier/Saunders,2019

Simplified Pulmonary Embolism Severity Index (sPESI)


• Prognostic model that can facilitate decision to treat patients as an outpatient and
identify those that require vigilant monitoring as inpatient
sPESI may have a prognostic accuracy similar to the PESI (more cumbersome to use)
• Assessment of the RV by imaging or laboratory biomarkers should be considered, even
in the presence of a low PESI or a sPESI of zero

Components of sPESI (one point each) Interpretation of sPESI:


• Age >BOyears old • Low risk: score of zero (may consider
• History of cancer outpatient anticoagulation)
• Chronic cardiopulmonary disease • High risk: score oh1
• Heart rate ~110bpm
• Systolic BP <IOO
• Arterial oxygen saturation <90%
Source:Jimenez D, et al. ArchInternMed.2010
211
C. Diagnostics for Pulmonary Embolism (PE)
DIAGNOSTIC I COMMENTS/EXPECTED FINDINGS
• High sensitivity for PE
• Useful rule-out test: patients with a normal D-dimer do not have
Plasma
pulmonary embolism
D-dimer ELISA
• Not specific (levels can increase with ACS, pneumonia, sepsis,
cancer, post-operative state, 2nd & 3rd trimesters of pregnancy)
• Most common gas exchange abnormalities:
Arterial blood 0 Hypoxemia (decreased arterial P02)

gas(ABG) 0 Increased alveolar-arterial 02 tension (A-a) gradient (represents


inefficiency of 02 transfer across the lungs)
• Most common abnormality in PE is T-wave inversion in leads V1 to
V4 (due to RV strain)
• Most frequently cited abnormality in PE (in addition to sinus
ECG tachycardia) is the McGinn-White sign or the S1Q3T3pattern:
0 S wave in Lead I
0 Q wave in Lead III
0 Inverted T wave in Lead III
Cardiac
• May be increased in RV microinfarction
biomarkers
• Westermark's sign: focal oligemia
• Hampton's hump: peripheral wedged-shaped density above the
Chest X-ray
diaphragm
• Palla's sign: enlarged right descending pulmonary artery
• Method of choice for imaging the pulmonary vasculature in
Computed
suspected PE
tomographic
• Can image small peripheral emboli
pulmonary
.• A normal CTP A excludes PE in patients with a Well' s score of "PE-
angiography
unlikely" or score <5
(CTPA)
• CTP A showing segmental or more proximal thrombus confirms PE
Ventilation-
• Second-line diagnostic test for PE
Perfusion
• High probability scan for PE: two or more segmental perfusion
scintigraphy
defects in the presence of normal ventilation
(V/Qscan)
• Acute PE may lead to RV pressure overload and dysfunction
• McConnell's sign: hypokinesis of the RV free wall with normal
motion of the RV apex (best known indirect sign of PE on
2DEcho
transthoracic echo)
• Other signs: dilated RV, flattening of the IV septum, elevated PA
pressure, visualized RA and/or RV thrombi
• Definitive diagnosis of PE depends upon visualization of an
Invasive
intraluminal filling defect in more than one projection
pulmonary
• Rarely performed (because CTPA is less invasive & has similar
angiography
accuracy)
Sources:Konstantinides
SV,et al. 2019ESCGuidelines for acutePE.EurHeartJ. 2020
JamesonJL,et al. Harrison's
Principles
of InternalMedicine20thedition,2018

212
III. DIAGNOSIS OF VENOUS THROMBOEMBOLISM (VTE)
A. Determine the Likelihood of Venous Thromboembolism (Wells Scoring)
DEEP VENOUS THROMBOSIS (DVT) PULMONARY EMBOLISM (PE)

Active cancer 1 Signs and symptoms ofDVT 3


Paralvsis, oaresis, or recent cast 1 Alternative diagnosis less likely than PE 3
Bedridden >3 davs; surgery <12 weeks 1
Heart rate >IOO beats/min 1.5
Tenderness in distribution of deep veins 1
Entire leg swelling 1 Immobilization >3 days; surgery
1.5
within 4 weeks
Unilateral calfswelline: >~ cm 1
Pitting edema 1 Prior PE or DVT 1.5
Collateralsuperficialnon-varicoseveins 1 Hemoptysis 1
Alternative diagnosis at least as likely
asDVT
-2 Cancer 1
Lowclinicallikelihoodof DVTifscore: s Opis PE is likelyifscore is ;::5points
Moderateclinicallikelihoodof DVTifscore: 1-2pts PE is unlikelyifscore is <5
Highclinicallikelihood
of DVTifscore:;::3pts
Sources:WellsPS, et al. AnnInternMed.2001& WellsPS, et al. N EnglJ Med.2003
B. Approach to Diagnosis once VTE is Suspected

VTEsuspected

Assess clinical likelihood

PE
likely

D-dimer

Venous CT
ultrasound pulmonary
angiography

No VTE

Requestfora D-dimer(ruleout test) if: Requestforan imagingtest if:


• Likelihood
forDVT is low • Likelihood
for DVT is not low(venousultrasound)
• Likelihood
forpulmonaryembolismis not high • Likelihood
forPE is high(CTPA)

Plasma D-dimer Measurement


0 Recommended in those with low or intermediate clinical probability, or those that are
PE-unlikely (to reduce the need for unnecessary imaging/irradiation)
0 A negative D-dimer test using an age-adjusted cut-off (seeformula below)should be
considered for excluding PE in those with low/intermediate probability or PE-unlikely
• Improves performance of D-dimer in the elderly
D-dimer cut-off= Age x 10 mcg/L
• Used in patients >50 years old
Sources:Konstantinides
SV,et al. 2019ESC Guidelinesforacute PE. EurHeartJ. 2020
213
IV. MANAGEMENT OF VENOUS THROMBOEMBOLISM (DVT AND PE)
A. Anticoagulation
° Foundation for successful treatment ofDVT and PE
0 Parenteral agents are started over the first 5-10 days (acute phase treatment) and
continued as a transition or bridge to stable, long-term oral anticoagulation
DRUG I MECHANISM I DOSE
arenteralA nt1coa9uat1on
• Binds & accelerates activity
• Initial IV bolus of8o units/kg, then
of antithrombin, thus
Unfractionated 18 units/kg/hour
preventing additional
Heparin • Maintain PTT ratio 1.5-2.5xnormal
thrombus formation
(UFH) (monitor every 4-6 hours & adjust
• Advantage: has a short
UFH dose as needed)
half-life

Low molecular • No monitoring or dose


• Enoxaparin I mg/kg SC q12
weight heparin adjustment needed, unless
• eGFR <30 mL/min: 1 mg/kg SC q24
(LMWH) obese or has CKD

• Anti-Xa pentasaccharide • 5 mg SC OD if weight <50 kg


Fondaparinux that specifically inhibits • 7.5 mg SC OD if weight 50-100 kg
factor X • IO mg SC OD if weight >JOO kg
>

Or,a.lll.nticoagulA~ts•
• Vitamin K antagonist • 2.5-5 mg OD to be overlapped with
(prevents activation of parenteral anticoagulation (e.g.,
factors II, VII, IX, X) UFH, LMWH) to be started on day
• Full effect requires 5 days 1-2ofLMWH or UFH
Warfarin • Overlapping with UFH, • Maintain overlap with parenteral
LMWH, or fondaparinux anticoagulation for 5 days until INR
counteracts the early is 2-3 for at least 24 hours
procoagulant effect of • Monitor prothrombin time (PT) with
unopposed warfarin a target INR range of 2.0-3.0

• 150mg BID after initial treatment with


• Direct thrombin parenteral anticoagulant (start the day
Dabigatran
(factor Ila) inhibitor after last parenteral dose is given)
• Age ~80 years: uo mg BID

• 15mg BID x 3 weeks, then 20 mg OD


Rivaroxaban • eGFR 15-50 mL/min: 15mg OD
(instead of 20 mg OD)
• 10 mg BID x 7 days, then 5 mg BID
Apixaban • eGFR 15-50 mL/min: 2.5 mg BID
• Factor Xa inhibitors
(instead of 5 mg BID)

• 60 mg OD after initial treatment with


parenteral anticoagulant
Edoxaban
• Weight <60 kg or eGFR 15-50mL/min:
3omgOD
*Directoral anticoagulants(DOAC)are the first-choiceanticoagulantif eligible.Warfarinis an alternative.
Sources: KonstantinidesSV,et al. 2019 ESC Guidelinesfor acute PE. Eur Heart J. 2020
Zipes DP,et al. Braunwald'sHeart Disease. 11thedition.Elsevier/Saunders,2019

214
Duration of Anticoaaulation
, Therapeutic anticoagulation for at least 3 months is recommended for all with PE/VTE
, For first PE/VTE due to a major transient/reversible risk factor': discontinue therapeutic
anticoagulation after 3 months
• For recurrent VTE not related to a major transient/reversible risk factor or in patients
with antiphospholipid antibody syndrome (APAS):indefinite duration of anticoagulation
*Temporaryor reversibleriskfactorsincludetrauma,immobilization,surgery,pregnancy,
oral
contraceptive
use(withinthe last6-12weekspriorto diagnosis)
Sources:Konstantinides foracutePE.EurHeartJ. 2020
SV,et al. 2019ESCGuidelines
KearonC,et al. CHESTGuideline.
2016

Contraindications to Anticoagulation
• Severe uncontrolled hypertension • Major bleeding diathesis
• Acute bacterial endocarditis • Allergy to anticoagulants
• Hemorrhagic stroke • Severe liver and renal failure
• Active ulcerative condition • Impaired bleeding parameters
• Uncontrolled active bleeding 0 Platelet count <50 x 10•/L
• Cerebral lesions at high risk for bleeding 0 INR >3.0 or PTT >2X normal

B. Fibrinolysis/Thrombolysis
0 The only FDA-approved indication for systemic fibrinolysis is massive PE
0 Rapidly reverses right heart failure & may result in a lower rate of death/recurrent PE
0 Preferred: Recombinant Tissue Plasminogen Activator (tPA)

C. Inferior Vena Cava (IVC) Filters


0 Indications for IVC filters include (can be retrievable for some models):
• Contraindications to anticoagulation in DVT patients
• Failure of anticoagulation (i.e.,patient still had PE despite adequate anticoagulation)
• Prophylaxis prior to abdominopelvic or lower extremity surgeries in acute DVT

D. Other Treatment Modalities


0 Percutaneous catheter-directed fibrinolysis for patients with PE and high bleeding risk
0 Pulmonary embolectomy (surgical)
0 Thromboendarterectomy for chronic thromboembolic pulmonary hypertension (CTEPH)
0 Maintenance of adequate hydration (for patients with massive PE and hypotension)
0 Graduated compression stockings (30-40 mmHg) are not routinely recommended in
DVT for prevention of post-thrombotic syndrome, but a trial of compression stockings
may be justified for symptomatic relief (if without significant PAD)

V. PREVENTION OF VENOUS THROMBOEMBOLISM


A. Padua Prediction Score for Identification of Hospitalized Patients at Risk for VTE
0 Most widely used risk assessment tool to decide whether to administer prophylaxis to
hospitalized medical patients
0 Meant to risk strati atients who have a

• Cancer 3 • Heart/respiratory failure


• Previous VTE 3 • Acute MI or stroke
• Immobility 3 • Infection/rheumatologic disorder
• Thrombophilia 3 • Obesity
• Trauma/surgery 2 • Hormonal treatment
• Age "?:70years 1
Paduascore.::4points:highriskofVTE(thromboprophylaxis
is recommended
if nocontraindications)
Score.J ThrombHaemost.
Source:BarbarS,et al.ThePaduaPrediction 2010
215
• Unfractionated heparin 5000 units SC BID or TIO, OR
• Enoxaparin40 mg SC OD, OR
Hospitalization • Dalteparin 2500-5000 units SC OD, OR
with medical • Fondaparinux 2.5 mg SC OD with normal renal function (in
illness heparin allergy such as HIT), OR
• Compression stockings/intermittent pneumatic compression (if
with contraindications to anticoagulation)
• Unfractionated heparin 5000 units SC B1D or TIO, OR
General surgery • Enoxaparin 40 mg SC OD, OR
• Dalteparin 2500-5000 units SC OD
• Warfarin (target INR 2-3), OR
• Enoxaparin 30 mg SC BID, OR
• Enoxaparin 40 mg SC OD, OR
• Dalteparin 2500-5000 units SC OD, OR
• Fondaparinux 2.5 mg SC OD, OR
Major orthopedic
• Rivaroxaban IO mg OD, OR
surgery
• Aspirin 81 mg OD, OR
• Dabigatran 220 mg OD, OR
• Apixaban 2.5 mg BID, OR
• Intermittent pneumatic compression (with or without
pharmacologic prophylaxis)
Source:ZipesDP,et al. Braunwald's
HeartDisease.11thedition.2019

ANTICOAGULATION WITH UNFRACTIONATED HEPARIN DRIP


Heparin is a sulfated polysaccharide usually isolated from mammalian tissues rich in mast cells
• Acts as an anticoagulant by activating antithrombin (AT III) and accelerating the rate at
which antithrombin inhibits clotting enzymes, particularly thrombin and factor Xa

• 10,000 units ofUFH in enough pNSS to make 100 cc in a


Sample preparation
soluset (concentration ofJO,ooo units/JOO cc or JOOunits/cc)

Dose for myocardial • 60 units/kg IV push as loading dose (LO), then


infarction: "60-12" • Start IV drip at 12 units/kg/hr
Dose for DVT or PE: • 80 units/kg IV push as LD, then
"80-18" • Start IV drip at 18units/kg/hr

Heparin Drip Adjustment


PTT is ideally monitored every 6 hours (after a dose change) and IV drip adjusted
accordingly to reach target PTT of 1.5-2.3times the control (or 46-70 sec)

Heparin Adjustment using Raschke's Protocol

<35 , <1.2xcontrol • 80 u/kg IV bolus then add 4 u/kg/hr to infusion rate


35-45 1.2to 1.5xcontrol • 40 u/kg IV bolus then add 2 u/kg/hr to infusion rate
46-70 1.5to 2.3xcontrol • No change (OPTIMAL TARGET)
71-90 2.3 to 3.0xcontrol • Decrease infusion rate by 2 u/kg/hr
>90 >3.0xcontrol • Discontinue for I hour, then decrease rate by 3 u/kg/hr
Source:RaschkeRA,et al.Ann InternMed.1993
216
Sample Case
An So-year-old male (60 kg) with pulmonary embolism comes in for dyspnea. How
will you give heparin drip?
• Dose of heparin for pulmonary embolism is "80-18" (So units/kg as loading dose, then
18 units/kg/hr as drip)
• Loading dose is given at 80 units/kg x 60 kg= 4800 units ofUFH as IV bolus
• Then, start the drip by following the steps
Step 1: Determine • If the drip formulation is 10,000 units ofUFH + 100 mL of!VF,
drug concentration the drug concentration is 10,000 units/roo mL or 100 units/mL
in the drip
Step 2: Convert units • No conversion is necessary because the desired dose (18 units)
as needed has the same unit as the computed concentration (100 units)
• The formula for drip rate (in mL/hr or cc/hr) can be simplified
as follows:

= Desired Dose (units/kg/hr)x Body weight (kg)


Step 3: Compute for (Drug Concentration (units/ml) )
drip rate (mL/hr)
= 18 (units/kg/hr)x 60 (kg)
100 (units/ml)

= 10.8 or ~II mL/hr (or cc/hr)

• Give LD of 4,800 units UFH (So units/kg) as IV bolus


• Start heparin drip: 10,000 units UFH + enough pNSS to make
Order in the Chart 100 cc in a soluset to run at II mL/hr (18 units/kg/hr) delivered
via an infusion pump
• Monitor PTT & adjust using Raschke's protocol

Continuation of the case: After 6 hours, the PTT control value is 37.1 sec and the
patient's value is 33.3 sec. How will you adjust the drip using the same formulation
(100 units/ml)?

• Since 33.3 / 37.1 = o.9x the control value, using the Raschke I

protocol:
0 Give 80 u/kg bolus, then

0 Increase the dose of heparin being given by 4 u/kg/hr

Adjusting the drip • Computation: 4 u/kg/hr x 60 kg= 240 u/hr should be added to
the current infusion rate
• Adjust the heparin drip by adding 2 mL/hr (or 2 ugtts/min) to
the baseline drip rate (note that in I ml of the drip, there are
100 units ofUFH)

217
SECTION NINE
OTHER DIS©RDERS OF THI;: CARDIOVASCUl:AR SYSTEM

ATRIAL FIBRILLATION (AF)


I. ETIOPATHOGENESIS
• Supraventricular tachyarrhythmia with uncoordinated atrial activation & ineffective
atrial contraction, characterized by irregularly irregular RR intervals, absence of
distinct P waves, and irregular atrial activation
Most common sustained arrhythmia in adults
Precipitating factors: hyperthyroidism, alcohol intoxication, post-operative state, acute
illness (e.g., ACS, pulmonary embolism)
Risk factors: advanced age, hypertension, diabetes, sleep apnea, structural heart disease
(e.g., valvular heart disease)

II. MANIFESTATIONS AND DIAGNOSIS


Clinical AF: symptomatic or asymptomatic AF that is documented by surface ECG
Subclinical AF: asymptomatic AF with no surface ECG documentation of AF (usually
detected during pacemaker/I CD interrogation instead)
Minimum duration of ECG documentation: at least 30 seconds of a single-lead strip, or
the standard 12-lead ECG
• Patients may be asymptomatic (silent), or have palpitations, dyspnea, fatigue, chest
pain, or may have hemodynamic instability

Clinical Patterns of AF
TYPES OF AF* I DESCRIPTION

First- • AF not diagnosed before, irrespective of its duration or the presence/


diagnosed AF severity of AF-related symptoms
Paroxysmal AF • AF.that terminates spontaneously or with intervention within 7 days of onset

• AF that is continuously sustained beyond 7 days, including episodes


Persistent AF
terminated by cardioversion (drugs or electrical cardioversion) after '2:7days
Long-standing • Continuous AF of >12 months' duration when decided to adopt a
Persistent AF rhythm control strategy
• AF that is accepted by the patient and physician, and no further
Permanent AF
attempts to restore/maintain sinus rhythm will be undertaken
*Terminologies
thatshouldbe abandonedincludeloneAF,valvular/non-valvular
AF,andchronicAF.
Source:Hindricks forAF.EurHeartJ. 2021
G,et al.2020ESCGuidelines

III. MANAGEMENT OF ATRIAL FIBRILLATION


A. Acute Management of AF
0 If patient is unstable, immediate transthoracic electrical cardioversion may be appropriate
0 If patient is stable, decision to restore sinus rhythm by cardioversion is based on
symptoms, previous AF episodes, age, LA size, & current antiarrhythmic drug therapy
MODE I MANAGEMENT
Pharmacologic • Options: flecainide, dofetilide, propafenone, ibutilide, amiodarone
cardioversion • Digoxin & sotalol may be harmful when used for cardioversion
Transthoracic
electrical
• Appropriate first-shock strength: biphasic waveform 150-200 J
• Higher output shocks if needed
cardioversion
Source:
ZipesDP,et al. Braunwald's
HeartDisease.
11thedition.Elsevier/Saunders,
2019
218
• IV: 300 mg+ 250 mL D5W over • JV:5-7 mg/kg over 1-2hours, followed by
30-60 mins, followed by 900-1200 mg+ 50 mg/hour infusion (maximum of1.2 g
500-1000 mL D5W over 24 hours for 24 hours)
• PO: 200 mg TID over 4 weeks, then
2oomgOD
*Usein patientswiththyroiddiseaseonlyif nootheroptions.Amiodarone is contraindicated
in
patientswithliverfailure/cirrhosis.
Source:HindricksG, et al. 2020ESCGuidelinesforAF.Eur HeartJ. 2021

• Pharmacologic therapy to lower the HR to improve AF-related symptoms


• Criteria used for strict rate control:
Rate 0 <80 bpm at rest
control 0 <ll0 bpm during moderate exercise
• Options: beta blockers (first-line), calcium channel blockers (verapamil,
diltiazem), digoxin, amiodarone
• Attempts to restore & maintain sinus rhythm (i.e., convert AF back to sinus)
Rhythm • Includes cardioversion, antiarrhythmic drugs, and catheter ablation
control • If with structural heart disease: amiodarone
• If without structural heart disease: flecainide, propafenone, ibutilide
Sources:
Hindricks
G,et al.2020ESCGuidelinesforAF.EurHeartJ. 2021
ZipesDP,et al. Braunwald's
HeartDisease.
11thedition,2019

IV. ANTICOAGULATION FOR STROKE PREVENTION


AF increases the risk of stroke depending on the presence of specific risk factors
(summarized by the CHA2DS2-VASc)
Usually use two scoring systems in decision-making:
° CHA2DS2-VASc score: to determine risk of having a stroke in the presence of AF
HAS-BLED score: to determine the risk of bleeding (since patients with AF should
0

be anticoagulated)
A. CHA2DS2-VASc Score
0 Estimates the risk of ischemic (embolic) stroke in patients with AF (higher score
indicates higher stroke risk)
0 Those likely to benefit from anticoagulation:
• Men: score ~I
• Women: score <!2
VARIABLE I SCORE
C • Congestive heart failure or left ventricular dysfunction 1

H • Hypertension (<!140/90mmHg) ~- 1
A2 • Age <!75years 2

D • Diabetes mellitus 1

S2 • Prior stroke/ TIA/ thromboembolism 2

V • Vascular disease (CAD, prior MI, PAD, aortic plaque) 1

A • Age 65-74 years 1


... ..
Sc • Sex cat~gory (i.e., female sex) 1
Source:Lip GY,et al. Chest.2010
219
B. HAS-BLED Score
0 Bleeding risk score to aid in decision-making for thromboprophylaxis {to balance the
risk of stroke versus risk of major bleeding)
0 High risk for bleeding: HAS-BLED score 2:3(would opt for regular monitoring and

correction of potentially reversible risk factors for bleeding)


VARIABLE I SCORE
H • Hypertension (uncontrolled: SBP >160mmHg) 1

• Abnormal renal/liver function


0 Renal: dialysis or renal transplantation or creatinine 2:200umol/L 1
A 0 Liver: chronic hepatic disease (cirrhosis), or bilirubin >2x upper 1
limit with AST/ALT/alkaline phosphatase >3Xupper limit
s • Stroke (previous ischemic/hemorrhagic) 1

B • Bleeding history or predisposition (e.g., bleeding diathesis, anemia) 1

L • Labile INR (unstable/high INR: <60% of the time in therapeutic range) 1

E • Elderly (age >65years) 1

• Use of drugs predisposing to bleeding (e.g., anti platelets, NSA!Ds) 1


D
• Alcohol use (>8 drinks per week) 1

C. Al orithmic A roach to Anticoa ulation in AF


Patient with AF

Compute for CHA20S2-VASc


Worforin (VKA)

Target INR depending on


valve lesion or type of prosthesis
Male2:2 Mole 1 Male 0
Female 2:3 Female 2 Female 1

Oral anti- Consider No anti-


coogulation0 oral anti- coagulation
coagulation0
{Class I) {Class Ila) {Low risk)

Compute HAS-BLE0b and


address modifiable risk factors

'DOACsasfirst-lineoralanticoagulant (VKAif DOACscontraindicated or notavailable).


Antiplatelet
therapyaloneis notrecommended for strokeprevention
in AF.
b If HAS-BLED ~3, flagpatientfor regularreviewandfollow-up.HighHAS-BLED scoresshouldnot
beanabsolutecontraindication to OAC.
Source:HindricksG,et al.2020ESCGuidelines forAF.EurHeartJ. 2021
220
D. Oral Anticoagulant Drugs for AF
1. Warfarin
• Mechanism: vitamin K antagonist (VKA)
• Usual lNR target: 2.0-3.0 (but depends on valvular lesion)
• DOA Cs are preferred over warfarin, except those with mechanical heart valves or
moderate-to-severe mitral stenosis

2. Direct Oral Anticoagulants (DOACs) or Non-VKA Oral Anticoagulants (NOAC)


• Preferable to warfarin in the majority of patients with non-valvular AF (non-inferior
to warfarin, but with better safety profile)
• Assessment of renal function is mandatory for all DOA Cs, especially for dabigatran
• Do not require dose adjustment on the basis of a specific coagulation test (in contrast
to INR monitoring in warfarin)
• Patients with ESRD were excluded from DOAC trials, but adjusted regimens of
rivaroxaban, apixaban, & edoxaban are options for CKD (CrCI 15-30 mL/min)
DRUG I MECHANISM I DOSE ADJUSTMENT
• Direct thrombin inhibitor • no mg BID for:
Dabigatran • 150 mg BID: superior to warfarin in stroke 0 Age 80 years
15oornomg prevention, with similar risk for bleeding 0 HAS-BLED~ 3
BID • no mg BID: non-inferior to warfarin in ° Concomitant interacting
stroke prevention, with fewer major bleeds drugs

I
• Direct factor Xa inhibitor • 15 mg OD for:
Rivaroxaban
• Non-inferior to warfarin in stroke 0 HAS-BLED~ 3, OR
2omgOD
prevention, with similar risk for bleeding ° CrCl 15-50 mL/min
• 2.5 mg BID if with at least
• Direct factor Xa inhibitor two of the following:
Apixaban
• Superior to warfarin in stroke prevention, 0 Age 80 years
5mgBID 0 Weight,;; 60 kg
with lower rates of major bleeding
° Creatinine 1.5 mg/dL
• Direct factor Xa inhibitor • 30 mg OD for:
Edoxaban 0 Weight,;; 60 kg, OR
• Superior to warfarin in stroke prevention,
6omgOD
with lower rates of major bleeding ° CrCI 15-50 mL/min
Antidotes/reversal agents in case of major bleeding (idarucizumab for dabigatran, andexanet alfa for
rivaroxaban& apixaban) are not yet widelyavailable,so management is usuallysupportive(e.g., transfusion)
Source: HindncksG, et al. 2020 ESC Guidelinesfor AF.Eur Heart J. 2021

PERICARDITIS
• Most common pathology affecting the pericardium ("inflammation")
• Clinically, it may be acute (<6 weeks), subacute (6 weeks to 6 months), or chronic (>6 months)

I. ETIOPATHOGENESIS
ETIOLOGY I EXAMPLES
• Viral: coxsackievirus, echovirus, herpesvirus, adenovirus, hepatitis, HIV
Infectious • Bacterial: pneumococcus, Streptococcus,Staphylococcus,Neisseria
• Others: tuberculous, fungal, syphilis, protozoa!, parasitic
• Acute idiopathic, renal failure, trauma, aortic dissection, acute MI,
Non- post-irradiation, metabolic
infectious • Neoplastic: mesothelioma, metastasis to pericardium (e.g., lungLbreast
CA, lymphoma, leukemia)
• Rheumatic fever, SLE, rheumatoid arthritis, scleroderma
• Drugs: procainamide, hydralazine, phenytoin, isoniazid, minoxidil,
Hypersensitivity
anticoagulants
or Autoimmune
• Postcardiac injury: post-pericardiotomy, post-traumatic, Dressler's
syndrome (post-MI pericarditis)
221
II. CLINICAL MANIFESTATIONS
ACUTE PERICARDITIS CHRONIC CONSTRICTIVE
I (<6 Weeks)
I PERICARDITIS (>6 Months)
• Healing of an acute fibrinous
I
or serofibrinuous pericarditis
• Inflammation of the
or the resorption of a chronic
pericardium
Description pericardia! effusion is followed
• May be infectious or non-
by obliteration of the pericardia!
infectious
cavity with formation of
granulation tissue
• Chest pain is severe, pleuritic,
may be retrostemal or left • Weakness, weight gain, fatigue
precordial, & may be referred to • Increased abdominal girth/ascites,
Symptoms neck, arms or left shoulder and edema
• Pericardia! pain may be relieved • Common in the Philippines: TB,
by sitting up & leaning forward; malignancy, radiation-induced
worsened by lying supine
• Pericardia! friction rub: high- • Kussmaul's sign: rise in the JVP
pitched, rasping, scratchy or (normally should decrease) with
Signs grating & heard most frequently inspiration
at end-expiration with patient • Pericardia! knock: early diastolic
upright & leaning forward sound in the left sternal border

1l
~.
Diagnostics " ,,
"'
.,. ,c.

Biomarkers • Modest increase • Normal to minimally increased


• Low voltage QRS complexes
ECG • Subepicardial inflammation • Flattened or inverted T waves
• Atrial fibrillation in 1/3 of patients
• Pericardia! thickening
• Pericardia! fluid or thickening
• Dilation of the IVC and hepatic
2D Echo • Differentiate pericarditis from
veins
Ml: assessment of wall motion
• Flattening of the LV posterior wall
• Pericardia! thickening
Cardiac CT/ • Pericardia! fluid collection • CT/MRI more accurate for
MRI • Pericardia! thickening imaging pericardia! thickening
than echo
Sources:JamesonJL, et al. Harnson'sPrinciplesof InternalMedicine20thed1t1on,
"
2018

III. DIAGNOSIS BY ELECTROCARDIOGRAM (ECG)


A. Stages of Acute Pericarditis
STAGE I ECG CHANGES
I TIMELINE
• Widespread ST elevation (with upward concavity)
Stage 1 • First 2 weeks
and PR depression with reciprocal changes in aVR
• Normalization of ST changes
Stage2 •Ito 3 weeks
• Generalized T wave flattening
Stage 3 • Flattened T waves become inverted • 3 to several weeks
• Several weeks
Stage4 • ECG returns to normal
onward

222
· · · · · · .. · • Diffuse ST • ST elevations
· elevations convex upward,
usually in leads
ST elevation which are representing
concave ischemic LV
upward .. _ . . . . . territory
• Usually not inverted until after • May begin to invert within hours
Twaves
ST se ment becomes isoelectric before ST se ment becomes isoelectric
waves • Absent • U suall resent
Residual ST-
• Unusual • Common
• Usual! resent • Absent

IV. MANAGEMENT OF PERICARDITIS


ACUTE PER!CARDITIS I CHRONIC (CONSTRICTIVE) PERICARDITIS
• Pericardiocentesis if with tamponade • Pericardia! resection/pericardiectomy
• Bed rest & anti-inflammatory drugs: • Sodium restriction & diuretics
Aspirin 750-1000 mg TIO
0 • Anti-Koch's for TB pericarditis
Ibuprofen 600-800 mg TIO
0 • Steroids (uncertain benefit)

° -In_d_o_m_e_th_a_c_i_n_2_5;;..-5_o_m_g_T_l_D
Colchicine 0.5 mg BID (OD___ ;;.._..__________________ _.
0

L.
--
if <70 kg)

CARDIAC TAMPONADE
I. ETIOPATHOGENESIS
Accumulation of fluid in the pericardia! space causes increased intracardiac pressures,
which limit ventricular filling and decreased cardiac output
• Three most common causes: neoplastic disease, idiopathic pericarditis, & renal failure

II. CLINICAL MANIFESTATIONS


Dyspnea, orthopnea and fatigue
Physical examination:
Beck's triad (hypotension, neck vein engorgement and muffled/distant heart sounds)
0

Tachycardia, tachypnea & pulsus paradoxus (>IO mmHg decrease in SBP with inspiration)
0

Ill. DIAGNOSTICS
DIAGNOSTIC I COMMENTS/EXPECTED FINDINGS
• Low voltage QRS complexes
• Electrical alternans: beat to beat variation in the QRS amplitude due to
12-LeadECG heart"

• Multi-chambered cardiomegaly with a "water-bottle" sign


Chest X-ray
• No pulmonary venous congestion
• Large pericardia! effusion
2D Echo
• Right atrial and right ventricular diastolic collapse

IV. MANAGEMENT
• Emergency pericardiocentesis
• Tube pericardiostomy with creation of pericardia! window (for recurrent, infectious,
malignant and other chronic causes)
223
PERIPHERAL ARTERY DISEASE (PAD)
I. ETIOPATHOGENESIS
Disorder characterized by stenosis or occlusion in the aorta or arteries of the limbs
• Atherosclerosis is the leading cause of PAD in patients >40 years old

II. CLINICAL MANIFESTATIONS


A. History and Symptoms
0 Most are asymptomatic, though some may present with slow gait
0 Most common symptom: intermittent claudication (pain, ache, cramp, numbness, or
sense of fatigue in the muscles which occurs during exercise and is relieved by rest)
0 Others: rest pain or feeling of coldness/numbness in feet/toes (critical limb ischemia)

B. Physical Examination
0Decreased or absent pulses distal to obstruction
0Bruits over narrowed artery
0Muscle atrophy, hair loss, thickened nails, smooth and shiny skin
0Reduced skin temperature, pallor, cyanosis, ulcers or gangrene

C. Classification of PAD
FONTAINE CLASSIFICATION • I • I

c-• • I

I Asymptomatic 1 • Mild claudication
II Intermittent claudication I 2 • Moderate claudication
3 • Severe claudication
Ila • Claudication walking >200 m
llb • Claudication walking <200 m 4 • lschemic rest pain
II
Ill Rest and nocturnal pain
5 • Minor tissue loss
IV Necrosis, gangrene Ill 6 • Majortissuelossabovetransmetatarsals
Source:FontaineR, et al. HelvCh1rActa. 1954
RutherfordRB,et al. J VaseSurg. 1997
III. DIAGNOSIS
DIAGNOSTIC I COMMENTS/EXPECTED FINDINGS
• AB!: ratio of ankle to brachia] artery pressure
0>I.4: noncompressible lower extremity arteries due to calcification
Ankle-brachia! 01.0-1.4:normal individuals
index (AB!) by
00.91-0.99: borderline
Doppler
0so.9: indicative of PAD
0so.4: signifies severe ischemia
..
• 1JBI:ratio of toe to brachia! artery pressure
Toe brachia! 0 :?.0.7:normal
index (TB!) by 0 <0.7: abnormal
Doppler
• Useful for patients whose AB! >1.4(usually DM or. CKD patients)
Segmental • Presence of pressure gradients between sequential cuffs denote
pressures & hemodynamically significant stenosis
pulse volumes • Amplitude of pulse volume contour becomes blunted in significant PAD
• Images and detects stenosis and/or thrombi
Arterial duplex
• Changes in peak systolic velocity & spectral waveforms can estimate
ultrasonography
the degree of stenosis <

• CT/MR angiography or invasive (conventional) angiography


Other imaging • Useful to diagnose anatomic location and severity of stenosis for
patients undergoing revascularization
Sources:AboyansV,et al. 2017 ESC Guidelineson PeripheralArtenalDiseases. Eur HeartJ. 2018
Gerhard-HermanMD,et al. 2016AHA/ACC LowerExtremityPADGuideline.J AmCollCardiel.2017
224
IV. MANAGEMENT
A. Non-Pharmacologic Management
Goals are to reduce CV events, improve limb symptoms, and preserve limb viability
0

Risk factor modificaton: smoking cessation and management of comorbidities


0

Daily foot care with regular exercise (walk until nearly maximum claudication
0

discomfort is experienced, and then rest until symptoms resolve)


0 Elastic compression stockings should be avoided (may worsen perfusion)
B. Pharmacologic Management
Drugs for improving CV and mortality risk in PAD are similar to that of CAD (e.g.,
0

antiplatelets, ACE-1/ARBs, and high-intensity statins)


Some drugs for improving symptoms are discussed below:
0

DRUG I REMARKS

Cilostazol • Increases claudication distance and improves quality of life


(50-rno mg BID) • Contraindicated in patients with heart failure
• Not recommended (Class III) in 2016 ACC/AHA guidelines due to
Pentoxyfylline
unclear/marginal benefit in improving claudication/walking distance

C. Indications for Revascularization (Percutaneous or Surgical)


0 Disabling, progressive or severe symptoms despite medical and exercise therapy
° Critical limb ischemia with pain even at rest
0 Non-healing wounds/ulcers

Sources:AboyansV, et al. 2017ESCGuidelineson Peripheral ArterialDiseases.EurHeartJ. 2018


Gerhard-HermanMD,et al. 2016AHA/ACCLowerExtremityPADGuideline.J Am CollCardiel.2017 ·.
CoccheriS, et al. EurHeartJ. 2002& GaddiA, et al. J Int MedRes.1996
REFERENCES
1. Aboyans V, Ricco JB, Banelink MEL, Bj0rck M, Brodmann M, Cohnert T, et al. 2017 ESC Guidelines on the Diagnosis and Treatment of
Peripheral Arterial Diseases, in collaboration wi1h the European Society for Vascular Surgery (ESVS):Document covering athcrosdero1ic
disease of exn-acranial caro1idand vertebral, mesenteric, renal, upper and lower extremity arteries. Eur Heart J,2018Mar 1;39(9):763-816.
2. Amsterdam EA, Wenger NK, Brindis RG, Casey DE Jr, Ganiats TG, Holmes DR Jr, ct al. 2014 AHNACC Guideline for the Management of
Patients with Non-ST-Elevation Acute Coronary Syndromes: a report or the American College of Cardiology/American Heart Association
Task Force on Practice Guidelines. I Am Coll Cardiol. 2014 Dec 23;64(24):e139-c228.
). Anker SD, Butler J,Filippatos G, ct al. Empagliflozin in Heart Failure with a Preserved Ejection Fraction. N Engl J Med. 2021Aug 27.
4-Antman EM, Cohen M, Bern ink PJ, McCabe CH, Horacek T, Papuchis G, et al. The TIMI risk score for unstable angina I non-ST elevation
Ml: a method for prognos1ication and therapeutic decision making. JAMA. 2000; 284(7):835-842.
5. Amen DK, Blumenthal RS, Alben MA, el al. 2019 ACC/AHA Guideline on the Primary Prevention of Cardiovascular Disease: Executive
Summary: A Report or the American College of Cardiology/American Heart Association Task Force on Clinical Practice Guidelines.
Circula1ion. 2019 Sep 10;14o(11):e563-e595.
6. Barbar S, Noventa F, Rosseuo V, et al. A risk assessment model for 1he identification of hospi1alizcd medical patients at risk for venous
thromboembolism: 1he Padua Prediction Score. J Thromb HaemosL 2010 Nov,8(11):2450-7.
7. Bozkurt B, Coats AJS, Tsutsui H, el al. Universal definition and classification ofhean failure: a report of the Heart Failure Society of America,
Hean Failure Association of1he European Society of Cardiology, Japanese Heart Failure Society and Wri1ing Commiuee or the Universal
Definition or Heart Failure: Endorsed by the Canadian Hean Failure Society, Hean Failure Association of India, Cardiac Society or
Australia and New Zealand, and Chinese Hean Failure Association. Eur J Hean Fail. 2021 Mar,23(.)):352-380.
8. Brockington GM, Zebede J, Pandian NG. Constrictive pericarditis. Cardiol Clin. 1990 Nov;8(4):645-61.
9. Campeau L Grading or angina pectoris (leuer). Circulation 1976;52:522-523.
10. Civeira F, lntema1ional Panel on Management or Familial Hypercholes1erolemia. Guidelines for the diagnosis and management or
heterozygous familial hypcrcholesterolemia. A1herosclerosis. 2004 Mar.173(1):55-68.
11. Coccheri S, Scondouo G, Agnelli G, Palazzini E, Zamboni V.et al. Sulode.xide in the trea1ment or intermittent claudication. Results or a
randomized, double-blind, multicentre, placebo-controlled s1udy. Eur Hean J. 2002 Jul;23(13):1057-65.
12. Collet JP, Thiele H, Barbato E, et al. 2020 ESC Guidelines for the management or acute coronary syndromes in pa1ien1Spresenting without
persistenl ST-segment elevation. Eur Heart J.2021Apr 7;42(14):1289-1367.
13. Dolgin M, Association NYH, Fox AC, Corl in R.and Levin RI, New York Hean Associalion. Criteria Commiuee. Nomenclature &criteria for
diagnosis or diseases of the heart and great vessels. 9th Edi1ion. Boston, MA: Lippincott Williams & Wilkins; 1994.
14. Fihn SD, Blankenship JC, Alexander KP,Bittl JA, Byrne JG, Fle1cherBJ.el al. 2014ACC/AHNAATS/PCNNSCAI/STS
focused update
of the guideline for the diagnosis and management of patients with stable ischemic hcan disease: a repon of the American College of
Cardiology/American Heart Association Task Force on Practice Guidelines, and the American Association for Thoracic Surgery, Prevemive
Cardiovascular Nurses Association, Society for Cardiovascular Angiography and Interventions, and Society of Thoracic Surgeons. J Am
Coll Cardiol. 2014 Nov 4;64(18):1929-49.
15. Fontaine R.Kim M, Kieny R. Die chirurgische Behandlung der peripheren Durchblutungsst0rungen [Surgical treatment of peripheral
circulation disorders]. Heh· Chir Acta. 1954Dec.21(5-6):499-533.German.
16. Fox KA, Dabbous OH, Goldberg RJ, Pieper KS, Eagle KA. Van de Werf F, et al. Prediction of risk or death and myocardial infarction in
the six momhs after presentalion with acute coronary syndrome: prospec1ive multinational observa1ional study (GRACE). BMJ. 2006
J7. Friedewald wr, LevyRI and Fredrickson DS. Estimation or the concentration orlow-density-lipoprotein cholesterol in plasma, wi1hout
use ofthe preparative ultracentrifuge. Clin Chem. 1972;18(6):499-502.
18. Gaddi A, Galetti C, Illumina1i B, Nascetti S. Meta-analysis or some resullS of clinical trials on sulodexide therapy in peripheral occlusive
anerial disease. J Int Med Res. 1996Sep-Oct,24(5):389-406.
19. Gerber MA, Baltimore RS, Eaton CB, Gewitz M, Rowley AH, Shulman ST, Tauben KA. Prevention or rheumatic fever and diagnosis and
trcatmen1 of acute Strep1ococcal ph:nyngitis: a scientific statement from the American Heart Association Rheumatic Fever, Endocardi1is,
and Kawasaki Disease Committee oflhe Council on Cardiovascular Disease in the Young, the Imcrdisciplinary Council on Functional
Genomics and Translational Biology, and the Imerdisciplinary Council on Quality of Care and Outcomes Research: endorsed by the
American Academy or Pediatrics. Circulation. 2009 Mar 24;119(11):1541-51.

225
20. Gerhard-HenTian MD, Gomik HL, Barrett C, Barshes NR, Corriere MA, Drachman DE, et al. 2016 AHNACC Guideline on the
Managemem of Patients V\'ith Lower Extremity Peripheral Anery Disease: A Repon of the American College of Cardiology/American
Hean Association Task Force on ClinicaJ Practice Guidelines. J Am Coll Cardiol. 2017Mar 21;69(11):e71-e126.
21. Gewitz MH, Baltimore RS, Tani LY.Sable CA, Shulman ST, Carapetis J, et al. Revision of the Jones Criteria for the diagnosis of acuie
rheumatic fever in 1he era of Doppler echocardiography: a scientific statement from the American Hean Association. Circulation. 2015
May 19;131(20):1806-18.
22. Gonzalez-Santos LE, Oliva R, Jimeno C, et al. Execuiive Summary of the 2020 Clinical Practice Guidelines for the Management of
Dyslipidemia in the Philippines. J ASEAN Fed Endocr Soc. 2021;36(1):5-11.
23. Grundy SM, Stone NJ, Bailey AL. et aJ. 2018AHAIACC/AACVPRJAAPNABC/ACPM/ADNAGS/APhA/ASPC/NLA/PCNA Guideline on the
Management of Blood Cholesterol: Executive Summary: A Repon of the American College of Cardiology/ American Hean Association
Task Force on Clinical Practice Guidelines. Circulation. 2019Jun 18;139(25):e1046-e1081.
24. Hindricks G, Potpara T, Dagres N, Arbela E, Ba~ JJ, Blomstri>m-Lundqvist C, et al. 2020 ESC Guidelines fort he diagnosis and management
of atrial fibrillation developed in collaboration with the European Association for Cardio-Thoracic Surgery (EACTS): The Task Force for
the diagnosis and management of atrial fibrillation of the European Society of Cardiology {ESC) Developed with the special contribution
of the European Hean Rhythm Association (EHRA) of the ESC. Eur Hean J. 2021Feb 1=42(5):373-498.
25. Hurst JvV. The examination of the hean: the imponanceofinitial screening. Dis Mon. 1990 May,36(5):245-313.
26. Ibanez B, James S, Agewall S, et al. 2017ESC Guidelines for the management of acute myocardial infarction in patients presenting with ST-
segment elevation: The Task Force for the management of acute myocardial infarction in patients presenting with ST-segment elevation of
the European Sociery of Cardiology (ESC). Eur Hean J.2018Jan 7;39(2):119-177-
27. Jameson JL. Kasper DL. Longo DL. Fauci AS, Hauser SL, Loscalzo J. Harrison's Principles of Internal Medicine. 20th Edition. New York:
McGraw Hill Education, 2018.
28. Jimenez D, Aujesky D, Moores L, et al. Simplification of the pulmonary embolism se\'erity inde.x for prognostication in patients with acute
symptomatic pulmonary embolism. Arch Intern Med. 2010 Aug 9;170(15):1383-9.
29. Kearon C, Akl EA, Ornelas J, et al. Antithrombodc therapy for vrE disease: CHEST guideline and expen panel report ChesL 2016;149:315
30. Killip T, Kimball JT.Treatment of myocardial infarction in a coronary care uniL A two year experience with 250 patients. Am J Cardiol.
1967;20(4):457-64.
31. Knumi J,Wijns W, Sarasre A, et al. 2019 ESC Guidelines for the diagnosis and management of chronic coronary syndromes. Eur Hean J.
2020 Jan 14=41(3)=407-4n.
32. Konstantinides SV, Me)'er G, Becattini C, et al. 2019 ESC Guidelines for the diagnosis and management of acUle pulmonary embolism
developed in collaboration with 1he European Respiratory Society (ERS). European Hean JoumaJ; 2020:41:543-603.
33. Lip GY. Implications of the CHA(2)DS(2)-VASc and HAS-BLED Scores for thromboprophylaxis in a1riaJ fibrillation. Am J Med. 2011
Feb;124(2):111-4.
34. Lip GY, Nieuwlaat R, Pisters R, Lane DA, Crijns HJ. Refining clinical risk stratification for predicling stroke and thromboembolism in atrial
fibrillation using a novel risk fac1or-based approach: the euro hean survey on atrial fibrillation. ChesL 2010 Feb;137(2):263-72.
35. Lopaschuk GD, Verma S. Mechanisms of Cardiovascular Benefils of Sodium Glucose Co-Transporter 2 (SGLT2) Inhibitors: A Sta1e-of-the-
An Review. JACC Basic Transl Sci. 2020 Jun 22;5(6):632-644.
36. Mach F, Baigent C, Catapano AL. et al. 2019 ESC/EAS Guidelines for the management of dyslipidaemias: lipid modification to reduce
cardiovascular risk. Eur Hean J.2020 Jan 1:41(1):111-188.
37. Mann SJ, James GD, Wang RS, Pickering TG. Elevation of ambulatory systolic blood pressure in hypertensive smokers. A case-control
study. JAMA. 1991May 1;265(17):2226-8.
38. McConaghy JR. "Outpatient evaluation of the adult with chest pain." UpToDate, 26 Feb 2021.
39. McDonagh TA, Metra M, Adamo M, et al. 2021 ESC Guidelines for the diagnosis and u-eatmem of acute and chronic heart failure. Eur
Hean J. 2021Sep 21;42(36):3599-3726.
40. Morrow DA, Antman EM, Charleswonh A, Cairns R, Murphy SA, de Lemos JA, et al. TIMI risk score for ST-elevation myocardial
infarction: A convenient, bedside, clinical score for risk assessment al presentation: An intravenous nPA for treatment of infarcting
myocardium early II triaJ substudy. Circulation. 2000 Oct 24;102(17):2031-7.
41. Nohria A, Tsang SW, Fang JC, Lewis EF, Jarcho JA, Mudge GH, Stevenson LW. Clinical assessment identifies hemodynamic profiles that
predict outcomes in patients admitted with hean failure. J Am Coll Cardiol 2003w1:1797-18o4.
42. O'Gara PT, Kushner FG, Ascheim DD, Casey DE Jr, Chung MK, de Lemos JA, ct al. 2013ACCF/AHA guideline for the management ofST-
elevation myocardial infarction: a report of the American College of Cardiology Foundation/American Heart Association Task Force on
Practice Guidelines. Circulation. 2013Jan 29;127(4):e362-425.
43. Ona DID, Jimeno CA, Jasul GV Jr, et al. Executive summary of 1he 2020 clinical practice guidelines for the management of hypertension in
the Philippines. J Clin Hypenens (Greenwich). 2021Aug>-
44. Otto CM, Nishimura RA, Bonow RO, Carabello BA,Erwin JP Ill, Gentile F, et al. 2020 ACC/AHA Guideline for the Management of Patients
With Valvular Hean Disease: A Repon of the American College of Cardiology/American Hean Association Joint Committee on Clinical
Practice Guidelines. Circulation. 2021Feb 2;143(5):e72-e227.
45. Pisters R, Lane DA. Nieuwlaat R, de Vos CB, Crijns HJ, Lip GY. A novel user-friendly score (HAS-BLED) 10 assess 1-year risk of major
bleeding in patients with atrial fibrillation: the Euro Hean Survey. ChesL 2010 Nov;138(5):1093-100.
46. Ponikowski P, Voors AA, Anker SD, et al. 2016 ESC Guidelines for the diagnosis and treatment of acute and chronic hean failure: The Task
Force for the diagnosis and treatment of acute and chronic heart failure of 1he European Society of Cardiology (ESC). Developed with the
special contribution of the Heart Failure Association (HFA) of the ESC. Eur HeanJ. 2016Jul 14;37(27):2129-2200.
47. Raschke RA, Reilly BM, Guidry JR, Fontana JR, Srinivas S. The weight-based heparin dosing nomogram compared with a "standard care"
nomogram. A randomized conrrolled trial. Ann Intern Med. 1993Nov 1;119(9):874-81.
48. Remenyi B, Wilson N, S1eer A, et al. World Hean Federation criteria for echocardiographic diagnosis of rheumatic hean disease: an
evidence-based guideline. Nat Rev Cardiol 2012;9:297-309.
49. Robens JM, August PA, Bakris G, Banon JR, Bernstein IM, Druzin M, et aJ. Hypertension in pregnancy. report of the American College of
Obstetricians and Gynecologists' Task Force on Hypenension in Pregnancy. Obstet Gynecol. 2013Nov;122(5):1122-1131.
50. Robens WC. The rule of 5 and the rule of 7 in lipid-lowering by statin drugs. Am J Cardiol. 1997Jul 1;80(1):106-7.
51. Rutherford RB, Baker JD, Ernst C, Johnston KW, Poner JM, Ahn S, Jones ON. Recommended standards for reports dealing with lower
e.,nremity ischemfa: revised version. J Vase Surg. 1997 Sep;26(3):517-38.
52. Thygesen K, Alpen JS, Jaffe AS, et aJ. Founh universal definition of myocardial infarction (2018).J Am Coll Cardiol. 2018 Oct 30;72(18):2231
53- Whelton PK, Carey RM, Aronow WS, et al. 2017ACC /AHA/ AAPA /ABC/ ACPM / AGS I APhA /ASH/ ASPC / NMA I PCNA Guideline
for 1he Pre\'ention, Detection, Evaluation, and Management of High Blood Pressure in Adults: A Repon of the American College of
Cardiology/American Hean Association Task Force on Clinical Practice Guidelines. J Am Coll Cardiol. 2018 May 15i71(19):e127-e248.
54. Williams B, Mancia G, Spiering W, et al. 2018 ESC/ESH Guidelines for 1he management of anerial hypertension: The Task Force for 1he
management of anerial hypenension of the European Society of Cardiology (ESC) and the European Society of Hypenension (ESH). Eur
Heart J 2018Sep 1;39(33):3021-3104.
55. World HeaJth Organization. Rheumatic fever and rheumatic hean disease: repon of a WHO expen panel. WHO Technical Report Series
No. 923. Geneva: WHO; 2004.
56. Vahanian A, Beyersdorf F, Praz F, Milojevic M, Baldus S, Bauersachs J, et al. 2021 ESC/EACTS Guidelines for the management of valvular
hean disease. Eur Hean J. 2021Aug 28:ehab395.
57. Van den Born BJ, Beutler JJ,Gaillard CA, de Gooijer A, van den Meiracker AH, Kroon AA Dutch guideline for the management of
hypenensive crisis - 2010 revision. Neth JMed. 2011;69(5):248-55.
58. Wells PS, Anderson DR. Rodger M, et al. Evaluation of D-dimer in the diagnosis of suspected deep-vein thrombosis. N Engl J Med. 2003
Sep 25;349(13):1227-35.
59. Wells PS, Anderson DR, Rodger M, et al. Excluding pulmonary embolism at the bedside withom diagnostic imaging: management of
patients with suspected pulmonary embolism presenting 10 the emergency depanment by using a simple clinical model and d-dimer.
Ann lmem Med. 2001 Jul 1;.135(2):98-107.
6o. Yancy CW, Jessup M, Bozkun B, Butler J, Casey DE Jr., Drazner MH, et al. 2013ACCF/AHA guideline for the management ofhean failure:
executive summary: a repon of the American College of Cardiology Foundation/American Hean Association Task Force on Practice
Guidelines. Circulation. 2013;128(16):1810-1852.
61. Zipes DP, Libby P, Bonow R, Mann DL, Tomaselli GF. Braunwald's Hean Disease: A Textbook of Cardiovascular Medicine. 11th Edition.
Elsevier/Saunders, 2019.
226
PULMONOL
APPROACH TO DISEASES OF THE PULMONARY SYSTEM
1. Approach to Common Pulmonary Complaints
2. Common Diagnostics in Pulmonology

0 BRONCHIAL ASTHMA
1. Approach to Bronchial Asthma
2. Management of Bronchial Asthma
3. Exacerbations in Asthma
4. Asthma-COPD Overlap (ACO)

0 CHRONIC OBSTRUCTIVE PULMONARY DISEASE


1. Approach to COPD
2. Management of Stable COPD
3. Exacerbations in COPD

0 PNEUMONIA
1. Community-Acquired Pneumonia
2. Hospital-Acquired & Ventilator-Associated Pneumonia

0 TUBERCULOSIS
1. Screening & Diagnosis of Tuberculosis
2. Management of Tuberculosis

0 RESPIRATORY FAILURE AND ARDS

OTHER DISORDERS IN PULMONOLOGY


1. Disorders of the Pleura
2. Pulmonary Nodules
SECTION ONE
TO DISEASESOF THE PULMONARYSYSTEM·
AP,PROACH
APPROACH TO COMMON PULMONARY COMPLAINTS
I.COUGH
Act of forcefully expelling air from the lungs, usually in response to throat or airway irritation
An effective cough reflex serves a protective function against aspiration and can help
clear airway secretions
Often a clue to underlying respiratory disease
AD t" fC :h
DURATION I COMMON CAUSES

Acute • Respiratory tract infection or aspiration event


<3weeks
cough • Inhalation of noxious chemicals or smoke
Subacute 3-Bweeks • Residuum from tracheobronchitis (e.g., in pertussis or
cough "post-viral tussive syndrome")
Chronic >8weeks • Inflammatory • Neoplastic
cough • Infectious • Cardiovascular etiology
of cough.Chest2006
and management
Source:IrwinRS,et al.ACCPd1agnos1s
B. Differential Diagnoses for Cough with Normal Chest PE and Radiography
° Cough-variant asthma

I
0 Gastroesophageal reflux
0 Nasopharyngeal drainage
0 Medications (ACE inhibitors)

• Carbocisteine 500 mg TIO


• Reduces the viscosity of
• Ambroxol 75 mg OD
Mucolytics sputum to expel sputum
• Acetylcysteine 200 mg sachet
more easily
with water BID-TID
• Guaifenesin 600-1200 mg q12h • Increases the volume &
Expectorants
up to a max of 2.4 g/day reduces viscosity of secretions
Local • Benzonatate 100-200 mg PO q8h • Localanesthetic effecton stretch
anesthetics (not to exceed 600 mg/day) receptors in respiratory passages
• 60 mg TIO (not longer than • Inhibitory action at the level
Levodropropizine
7 days) of the airway sensory nerves
• Decreases viscosityof mucus
Lagundi • 500-600 mg TIO
for easier expectoration
• Bronchodilators
Others • Antihistamines • See discussion below
• Montelukast
Centrall -Actin Dru s
Opioids/ • Codeine 7.5-20 mg PO q4-6h PRN • Thought to act in the "cough
narcotics (not to exceed 120 mg/day) center" in the brainstem
• Dextromethorphan 10-20 mg PO
• Decreases sensitivity of
Non-narcotics q4h or 30 mg PO q6-8h
cough receptors
• Butamirate citrate 50 mg BID-TIO
229
II. HEMOPTYSIS
Hemoptysis is the expectoration of blood from the respiratory tract
• The origin of blood can be identified by observing its color:
Bright-red, foamy blood: usually from the respiratory tract
0

Dark-red, coffee-colored blood: usually from the gastrointestinal


0 tract
A. Causes ofHemoptysis
070-90% due to bronchitis, bronchiectasis, necrotizing pneumonia, tuberculosis (owing
to high prevalence and its predilection to cavity formation)

• B: Bronchitis, Bronchiectasis • C: Coagulopathy


• A: Aspergilloma • A: Autoimmune, AV malformation, Alveolar
• T:Tumor hemorrhage
• T: Tuberculosis • M: Mitra! stenosis
• L: Lung abscess • P: Pneumonia
• E: pulmonary Embolus

B. Principles of Management
0 Maintain airway patency and oxygenation (intubate as necessary; use double lumen
endotracheal tube if available)
0 Localize the source of bleeding (radiologic imaging, bronchoscopy)
0 For hemorrhage in intubated patients, give racemic epinephrine ET flushing: 1 ampule of
epinephrine (I:IOOO solution) in 9 mL normal saline solution, as 2 mL flushing q6 hours

III. DYSPNEA
Dyspnea (shortness of breath) is a common symptom of patients with pulmonary disease
• May also be the primary manifestation of cardiac disease, anemia, neuromuscular
disorders, or obesity/deconditioning
A. Pathogenesis ofDyspnea
DESCRIPTION I PATHOPHYSIOLOGY
Chest tightness or • Bronchoconstriction
constriction • Interstitial edema (e.g., asthma, myocardial ischemia)
Increased work or effort of • Airway obstructio!} (e.g., COPD, uncontrolled asthma)
breathing • Neuromuscular disease (e.g., myopathy, kyphoscoliosis)
Air hunger, need to • Increased drive to breathe (e.g., heart failure, pulmonary
breathe, urge to breathe embolism, moderate-severe airflow obstruction)
Cannot get a deep breath, • Hyperinflation (e.g., asthma, COPD)
unsatisfying breath • Restricted tidal volume (e.g., pulmonary fibrosis)
Heavy breathing, rapid
• Deconditioning
breathing, breathing more

BY fD .
SYMPTOM I COMMENTS

• Common indicator of heart failure, mechanical


Orthopnea impairment of the diaphragm associated with obesity, or
asthma, triggered by esophageal reflux
Paroxysmalnocturnal dyspnea • Highly suggestive of heart failure
Acute, intermittent episodes • More likely to reflect episodes of myocardial ischemia,
ofdyspnea bronchospasm, or pulmonary embolism
• Typical of COPD, interstitial lung disease, and chronic
Chronic persistent dyspnea
thromboembolic disease
Platypnea • Left atrial myxoma or hepatopulmonary syndrome
230
Respiratory • Stigmata of infection (e.g., • Infiltrates present in
• Variable
tract infection fever, cough, colds) pneumonia
• Onset often in childhood
• Symptoms vary from day
• Variable to day and worse at night &
Asthma •May • Spirometry
early morning
present • Associated with allergy,
with rhinitis, eczema
exacer-
bations • Onset usually in mid-life
• Symptoms are slowly • Smoking history
COPD
progressive, but may have • Spirometry
exacerbations
• Sudden onset of severe
dyspnea • D-dimer
Pulmonary
• With predisposition to • CT pulmonary
Embolism
VTE (e.g., immobilization, angiogram
• Usually malignancy)
acute
• Radiography shows
• Sudden onset of severe
no lung markings
Pneumothorax dyspnea
peripheral to visceral
• Symptoms depend on size
pleural line
• Imaging shows
• Commonly associated with
bronchial dilation,
Bronchiectasis bacterial (or tuberculous)
bronchial wall
infection
thickening

• Chronic • Living in an area with


high TB prevalence
• Associated with cough &
Tuberculosis • Apical infiltrates or
sputum production, weight
(TB) cavitation on CXR
loss, fever
• Sputum examination
for confirmation

Cardiac Causes
Congestive
• Associated with • History and PE
Heart Failure
manifestations of cardiac • Imaging shows
Ischemia • Variable disease (e.g., chest pain, cardiomegaly or
or ACS palpitations, orthopnea, other structural heart
edema) disease
Arrhythmias
Others
Panic disorders • Associated with social or emotional stressor
• Family history/pedigree
• Variable • Associated abnormalities
Neuromuscular • Creatine kinase
depend on the specific
causes ,EMG-NCV
disease
,MRI
Source:JamesonJL,et al. Harrison'sPrinciplesof InternalMedicine20thedition,2018
231
PHYSICAL EXAMINATION
I. COMPONENTS OF PHYSICAL EXAMINATION
• Note for rate of breathing, abnormal retractions in intercostal spaces
• Describe any defects in chest wall (e.g., pectus excavatum/carinatum)
• Note for any abnormal bulging/masses
Inspection
• Note if there is local lagging or impairment of respiratory movement
• Check for other associated extrapulmonary findings (e.g., clubbing,
facial puffiness, prominent veins in anterior chest wall)
• Check for cervical lymph node enlargement or tenderness
• Respiratory excursion: put thumbs parallel to roth rib posteriorly
& grasp the lateral rib cage; while patient breathes deeply, look at
Palpation excursion of fingers
• Assess tactile fremitus: using ulnar surface of hand, compare tactile fremitus
on both sides from upper to lower lung zones (instruct patient to say "tres•tres")
• Note for tracheal deviations, crepitations, and areas of tenderness
, Hyperextend pleximeter finger on surface to be percussed
• Position other hand close to pleximeter with finger cocked upwards
• Strike pleximeter finger with a quick and relaxed wrist motion
Percussion
• Withdraw flexor briefly to avoid damping
• Percuss chest in proper sequence
• Produce appreciable percussion note

• Use of stethoscope:
0 Bell for low pitched sounds
Auscultation
0 Diaphragm for high pitched sounds
• Start at the apices from left to right; back and front of chest

II. SUMMARY OF CHEST EXAMINATION FINDINGS

CONDITION
BREATH I
I PERCUSSION I FREMITUS I SOUNDS VOICE I ADVENTITIOUS
SOUNDS
TRANSMISSION

Normal Resonant Normal Vesicular Normal Absent

Consolidation
or atelectasis Bronchophony,
Dull Increased Bronchial Crackles
(with patent Egophony
airway)
Consolidation
or atelectasis
Dull Decreased Decreased Decreased Absent
(with blocked
airway)

Asthma Resonant Normal Vesicular Normal Wheezing


COPD/ Absent or
Hyperresonant Decreased Decreased Decreased
emphysema wheezing
Pneumothorax Hyperresonant Decreased Decreased Decreased Absent
Absent (with
Pleural
Dull Decreased Decreased Decreased pleural friction
effusion
rub in some)
Dull
Pulmonary Decreased
(over the Normal Normal Normal
mass (over the mass)
mass)

232
COMMON DIAGNOSTICS IN PULMONOLOGY
DIAGNOSTIC I DESCRIPTION
, Routine chest radiography (posteroanterior and lateral views taken
in the upright position)
, Integral part of the diagnostic evaluation involving the parenchyma,
Radiography pleura, airways, and mediasrinum
, Lateral decubitus: to determine whether pleural abnormalities
represent freely-flowing fluid
• Apicolordotic views: visualize disease at the lung apices
• Most common pulmonary function test (others include gas diffusion
rests, body plethysmography, inhalation/bronchodilation challenge test)
Spirometry
• Measures how much/quickly an individual can move air out oflungs
, Individual breathes into a mouthpiece attached to a spirometer
• Produces images using echoes or reflection of the UTZ beam
Ultrasound • Can detect & localizepleural abnormalities & peripheral lung parenchyma
(UTZ) • Quick and effective way of guiding percutaneous needle biopsy of
peripheral lung, pleural, or chest wall lesions
• Allows distinction between densities that would be superimposed on
Computed
plain radiographs
tomography
(CT) • Better in characterizing tissue density and providing accurate size
assessment and location oflesions
High-resolution • Higher resolution than a conventional CT scan because of thinner
CT(HRCT) collimation slices, but at the cost of higher radiation dose/exposure
• Used for evaluation of chronic thromboembolic pulmonary
Ventilation- hypertension (CTEPH) and pulmonary embolism (PE)
perfusion (VQ) , Abnormal finding is VQ mismatch (e.g., regions in which there is a
lung scanning defect in perfusion that follows the distribution of a vessel and that is
not accompanied by a corresponding defect in ventilation)
• Allows simultaneous detection of parenchymal abnormalities and
CT pulmonary
pulmonary vasculature
angiography
• Test of choice in evaluation of pulmonary embolism

Pulmonary • Radiopaque contrast medium is injected through a catheter placed


angiography in the pulmonary artery to visualize the pulmonary vessels
• Allows direct visualization of the tracheobronchial tree and
Bronchoscopy evaluation of the bronchopulmonary airway segments
• Performed usually with flexible fiberoptic instruments

Video-assisted • Minimally invasive technique for diagnosis and management of


thoracoscopic pleural and some parenchymal lung diseases
surgery (VATS) • Thoracoscope is inserted through a small incision between the ribs
Source:JamesonJL, et al. Harrison'sPrinciplesof InternalMedicine20thedition,2018

233
SECTION TWO
BRONCHIAL ASTHMA
OVERVIEW OF BRONCHIAL ASTHMA
I. ETIOPATHOGENESIS
Heterogenous disease characterized by airway hyperresponsiveness, chronic airway
inflammation, and expiratory airflow limitation that varies markedly over time and in
intensity, both spontaneously and with treatment
Airflow limitation may later become persistent
Mast cells, eosinophils, T-lymphocytes, and neutrophils all play a role in the pathogenesis

II. DEFINITION OF SPIROMETRY TERMS


Forced Expiratory
• Measures how much air an individual can exhale during the first
Volume in I
Second (FEV1) second of a forced expiration

Forced Vital
• Total amount of air exhaled during the FEV test
Capacity (FVC)

• Normally should be 75-80%


FEV1/FCV Ratio
• Reduced in obstructive disease (e.g., asthma, COPD)
(FEV1%)
• May be normal or increased in restrictive diseases (e.g., fibrosis)

Peak Expiratory • Individual's maximum speed of expiration, measured by a peak


Flow(PEF) flow meter (handheld device to monitor ability to breathe out air)

III. MANIFESTATIONS
Typical symptoms: any combination of dyspnea, wheeze, cough, or chest tightness,
worse at night or early morning and with variable time course and intensity
Symptoms usually demonstrate reversibility and variability:
0 Reversibility: applies to rapid improvements in FEV1 (or PEF) measured within
minutes after inhalation of a rapid-acting bronchodilator, or more sustained
improvement over days or weeks after controller treatment
0 Variability: refers to improvement or deterioration in symptoms and lung function
occurring over time
Physical examination is often normal, but the most frequent abnormality is expiratory
wheezing or rhonchi on auscultation
PHENOTYPE
I DESCRIPTION

• Most easily recognizable phenotype, which often starts in childhood


• Associated with a personal or family history of atopy (eczema,
Allergic asthma allergic rhinitis, food or drug allergy)
• Sputum examination: eosinophilic airway inflammation
• Responds well to inhaled corticosteroids (JCS)
Non-allergic • Sputum exam: neutrophilic or paucigranulocytic airway inflammation
asthma • Less responsive to ICS
Adult-onset/ • Common in women, presenting with symptoms usually in adulthood
late-onset asthma • Requires higher doses of!CS or are relatively refractory to !CS
Asthma with
• Seen in patients with long-standing asthma who develop fixed
persistent airflow
airflow limitation due to airway wall remodeling
limitation

Asthma with • Seen in obese patients with asthma who present with little
obesity eosinophilic inflammation but with prominent respiratory symptoms
for Asthma(GINA),2021& MooreWC,et al.Am J Resp1r
Sources:Globallrnt1at1ve GritCareMed2010
234
IV. DIAGNOSIS
A. Dia nostic Criteria (Definition) for Asthma

History of variable respiratory symptoms, AND


confirmed variable expiratory airflow limitation

Features that Support the Diagnosis


• Cough
• More than one (combination) respiratory symptom
• Dyspnea
• Occurs variably over time & in intensity
• Wheezing
• Worse at night or early morning
• Chest tightness
• Triggered by exercise, allergens, infections, cold air, laughter

2. ConfirmedVariableExpiratoryAirflow Limitation
• Documented excessive
variability in lung • The greater the variation, the more likely the diagnosis
function (oneor moreof of asthma
the tests below), AND

• Documented expiratory • When FEV1 is reduced, confirm that FEV1/FVC is also


airflow limitation low (normal for adults is >0.75-0.80)
Source: GlobalInitiativeforAsthma (GINA),2021

B. Tests to Document Variable Expiratory Airflow Limitation (to meet above criteria)
CRITERIA FOR EXCESSIVE VARIABILITY
DIAGNOSTIC
I IN LUNG FUNCTION
• Increase in FEV1 ,12% and >200 mL from baseline, measured
Positive
I0-15 minutes after salbutamol 200-400 mcg or equivalent
bronchodilator
• More likely to be positive ifbronchodilator medication is
reversibility test
withheld before test (2:4hours for SABA,2:15hours for LABA)

• Average daily diurnal PEF variability >IO%


• PEF variability is calculated from twice daily readings
(best of 3 each time), averaged over 2 weeks
Excessive variability in
peak expiratory flow PEF Variability:
(PEF)
= (Da;i::'shi 2hest PEF - Dats lowest PEF)
(Mean of the day's highest and lowest PEF)

Increase in lung • Increase in FEV1 by >12% and >200 mL (or PEF by >20%)
function after anti-
inflammatory treatment from baseline after 4 weeks of treatment

• Positive exercise challenge test (fall in FEV1 >IO%& >200 mL


from baseline)
• Positive bronchial challenge test (fall in FEV1 2:20% with
Other tests
methacholine or histamine or 2:15%with hyperventilation,
saline, mannitol)
• Excessivevariation in lung function between visits (less reliable)
FEV1:forced expiratory volume in 1 second
PEF:peak expiratory flow (besVhighest of three readings)
SABA:short-acting beta-2 agonist
LABA:long-acting beta-2 agonist
..
Source: GlobalInitiativeforAsthma (GINA),2021
235
V. ASSESSMENT OF ASTHMA CONTROL
• Level of asthma control is the extent to which the manifestations can be observed, or
have been reduced or removed by management
Control has 2 domains to be assessed: symptom control & risk of future adverse outcomes

• Daytime asthma symptoms >2x/week?


Well-controlled: none present
• Any night waking due to asthma?
Partly controlled: 1-2present
• Reliever needed for symptoms >2x/week?
Uncontrolled: 3-4 present
• Any activity limitation due to asthma?
B. Risk Factors for Poor Asthma Outcomes
0 Assess risk factors at diagnosis and periodically
0 Measure FEV1at start of treatment, after 3-6 months of controller treatment, then periodically
The presence of any of the following increases the risk of exacerbations:
Potentially modifiable independent risk factors for exacerbations:
• Uncontrolled asthma symptoms
• High SABA use (,! one 200-dose canister per month)
• Inadequate !CS, or not prescribed !CS
• Low FEV1 (especially if <60% predicted), high bronchodilator reversibility
• Poor adherence or incorrect inhaler technique
• Major psychological or socioeconomic problems
• Exposure: smoking, allergen exposure, air pollution
• Co-morbidities: obesity, rhinosinusitis, GERO, food allergy
• Pregnancy
Other major independent risk factors for exacerbations:
• History of intubation or ICU admission for asthma
• ,!I severe exacerbation in the past year

VI. ASSESSMENT OF ASTHMA SEVERITY


Severity is assessed from the level of treatment required to control symptom exacerbations
Can be assessed once patient has been on controller treatment for several months
Treatment step down can be attempted to settle at a minimum effective level of treatment
Asthma severity is therefore dynamic and may change over months or years
SEVERITY I DESCRIPTION
• Asthma well-controlled with step I or step 2 treatment (e.g., as-needed
Mild
ICS-formoterol alone, or with low-intensity maintenance controller
asthma
treatment such as low-dose !CS, LTRA or cromones)
Moderate • Asthma well-controlled with step 3 or step 4 treatment (e.g., low or
asthma medium dose !CS-LABA)
• Asthma that remains uncontrolled (see definition below) despite adherence
Severe
to maximal treatment (e.g., high-dose !CS-LABA) & management of
asthma
contributory factors; or that worsens when high dose treatment is decreased
• Includes either of the following:
0 Poor symptom control (frequent symptoms or reliever use, activity
Uncontrolled
limited, night waking due to asthma)
asthma
° Frequent exacerbations(<! 2/year) requiring OCS, or serious
exacerbations (<!1/year) requiring hospitalization
Difficult- • Uncontrolled despite medium or high dose !CS with a 2nd controller
to-treat (usually LABA); or with maintenance OCS; or that requires high dose
asthma treatment to maintain good symptom control & reduce risk of exacerbations
ICS:inhaledcorticosteroids OCS:oralcorticosteroids
LABA: long-acting
beta-2agonist LTRA:leukotriene
receptorantagonist
forAsthma(GINA),2021
Source:Globallnit1a!ive
236
MANAGEMENT OF BRONCHIAL ASTHMA
I. ASTHMA TREATMENT TRACKS FOR ADULTS/ADOLESCENTS
• There are two treatment tracks for the treatment of asthma
Key difference is the medication used for symptom relief:
0 Track 1: as-needed low dose ICS-formoterol
0 Track 2: as-needed SABA

A. Recommended Options for Initial Asthma Treatment


° For safety, "SABA alone treatment" is no longer recommended
0 Patients should receive JCS-containing controller to reduce risk of serious
exacerbations and to control symptoms
0 JCS-containing controller can be delivered with regular daily treatment, or with as-
needed ICS-formoterol combination for symptomatic relief in mild asthma
0 Recommendedoptionsfor initial controllerare listed below:

I
I
PRESENTING PREFERRED INITIAL ALTERNATIVE INITIAL
SYMPTOMS TREATMENT (Track 1) TREATMENT (Track 2)
• Infrequent symptoms • Low dose JCS taken
(<2x/month), AND whenever SABA is taken
• No risk factors for • As needed low dose JCS- (in combination or
exacerbations formoterol separate inhalers)
• Symptoms or need for • Low dose ICS with as-
reliever ~2x/month needed SABA
• Troublesome
symptoms most days;
OR • Low dose JCS-LABA with
• Low dose JCS-
• Waking due to asthma as-needed SABA; or
formoterol maintenance
~Ix/week • Medium dose !CS with
and reliever therapy
• Especially if with as-needed SABA
risk factors for
exacerbations
• Medium dose JCS- • High dose JCS or medium
• Severely uncontrolled
formoterol maintenance dose JCS-LABAwith as-
asthma;OR
& reliever therapy needed SABA
• With acute
• Short course OCS may • Short course OCS may be
exacerbation
be needed needed
*Checkadherenceandinhalertechnique

SABA:short-actingbeta-2agonist
LABA:long-actingbeta-2agonist
ICS:inhaledcorticosteroids
OCS:oralcorticosteroids
Source: Global Initiativefor Asthma (GINA), 2021

237
B. Stepwise Approach to Control Symptoms and Minimize Future Risk
0 Asthma treatment is a continuous cycle: assess, adjust treatment, & review response
° For the best outcomes, regular controller treatment should be initiated as soon as
possible after the diagnosis of asthma is made
0 After starting initial controller treatment (as discussed above), review response after 2-3
months, or according to urgency
• Consider stepping up if: uncontrolled symptoms, exacerbations or risk factors
present (but also assess adherence to medications, correct inhaler technique, and
persistent exposure to triggers)
• Consider stepping down if: symptoms controlled for 3 months and with low risk
for exacerbations
° Find the patient's lowest treatment step/dose that controls both symptoms & exacerbations

STEP I TRACK 1 *
I TRACK 2 **

I • Take JCS whenever SABA taken


,___ • As-needed low dose ICS-formoterol
2 • Low dose maintenance JCS

• Low dose maintenance JCS-


3 • Low dose maintenance !CS-LABA
formoterol
• Medium dose maintenance • Medium/high dose maintenance
4 ICS-formoterol !CS-LABA
• Add-on LAMA • Add-on LAMA
• Refer for phenotypic assessment± • Refer for phenotypic assessment±
5 anti-IgE, anti-lL5/5R, anti-lL!R anti-IgE, anti-IL5/5R, anti-lL!R
• Consider high dose ICS-formoterol • Consider high dose !CS-LABA
•Track1 relieveris as-needed low-doseICS-formoterol(preferredrelieverto reduce exacerbations)
••Track2 relieveris as-needed SABA(alternativereliever) '
SABA:short-actingbeta-2 agonist
LABA:long-actingbeta-2 agonist
LAMA:long-actingmuscarinicantagonist
ICS: inhaledcorticosteroids
IL:interleukin
Source:GlobalInitiativeforAsthma(GINA),
2021

II. PHARMACOLOGIC THERAPY OF ASTHMA


A. General Categories of Asthma Medications
CATEGORY I DESCRIPTION
• Provided for as-needed relief of breakthrough symptoms &
prevention of exercise-induced asthma
Reliever
• Include as-needed low dose ICS-formoterol (preferred reliever,
medications
but not if maintenance controller contains a different !CS-LABA)
or as-needed SABA

• Used to reduce airway inflammation, control symptoms, and


Controller reduce future risks (e.g., exacerbations, decline in function)
medications • In mild asthma: may be delivered through as needed low dose
ICS-formoterol (taken when symptoms occur & before exercise)
Add-on • Considered for persistent symptoms and/or exacerbations
therapies for despite optimized treatment with high-dose controller
severe asthma medication

238
BO .0
COMMENTS &
CLASS
I EXAMPLES
I MECHANISM
I ADVERSE EFFECTS

BronchodilatorTherapies -
• Stimulates adenylyl • No effect on chronic
cyclase thus increasing inflammation
Short-acting • Salbutamol/ cAMP levels and • May cause tremors and
beta-2 albuterol causing bronchodilation palpitations (usually in
agonists • Procaterol • Rapid onset of the elderly) and a small
(SABA) • Terbutaline bronchodilation & best fall in potassium
used for symptom relief • Formoterol has a rapid-
• Duration: 3-6 hours onset and is as effective
as SABA as reliever
• Same as SABA medication (but use of
• Formoterol • Improve asthma regular LABA without
Long-acting • Salmeterol control and reduce JCS is discouraged)
beta-2 • Bambuterol inflammation when • LABA replaced regular
agonists • Vilanterol added to JCS, thereby use of SABA
(LABA) • Indacaterol allowing lower doses of • LABA improves asthma
• Olodaterol JCS to be given control & reduces
• Duration: >12hours exacerbations when
added to !CS
• M uscarinic receptor • Provide additional
antagonists benefit when used in
Short-acting
• Inhibit only the cholinergic combination with SABA
muscarinic
• Ipratropium reflex component of in those with more
antagonists
bronchoconstriction, so severe symptoms
(SAMA)
less effective than B2- • Adverse effects: dry
agonists mouth (most common),
urinary retention,
glaucoma
• Same as SAMA • Tiotropium as mist
Long-acting • Additional inhaler is an add-on
muscarinic bronchodilator in treatment for patients
• Tiotropium
antagonists those not controlled by with history of
(LAMA) maximal doses ofICS- exacerbations (Step 4)
LABA combinations • Not for children <12
years old
• Nausea, vomiting &
headache (most common)
• Arrhythmias, seizures
and death at high
• Inhibit
concentrations
• Theophylline phosphodiesterase
Methyl- • IV aminophylline &
• Aminophylline activity causing increase
xanthines theophylline should
• Doxofylline in cAMP levels and
not be used for asthma
bronchodilation
exacerbations due to
their poor efficacy &
safety profile (compared
to SABA)
'

239
I
CLASS

Contri>fler, Md
I EXAMPLES

e ,cat 1,!)ns
I MECHANISM COMMENTS &
ADVERSE EFFECTS
I

• Hoarseness/
• Most effective anti- dysphonia and
inflammatory agents oral candidiasis,
for asthma control pneumonia
Inhaled • Beclomethasone • Best route for • Given as first-line
corticosteroids • Budesonide steroids as it therapy for persistent
(ICS) • Fluticasone provides targeted asthma; but LABA
drug delivery, acts may be added if
faster, and requires they do not control
smaller doses symptoms at low
doses
• Truncal obesity, easy
• Prednisone
Systemic bruisability,osteoporosis,
• Methyl- • Useful for
corticosteroids DM, hypertension,
prednisolone treatment of acute
(oral or gastric ulceration,
• Hydrocortisone exacerbations
intravenous) proximal myopathy,
depression, cataracts

• Less effective than JCS


in controlling asthma
and have less effect on
• Block leukotriene
airway inflammation
receptors
Leukotriene- • Montelukast • Useful as an add-on
(montelukast,
modifying drugs • Zafirlukast therapy in those not
zafirlukast) or
(antileukotrienes) • Zileuton controlled with low
inhibit lipoxygenase
doses oflCS
(zileuton)
• Indicated for aspirin-
or exercise-induced
l asthma

• Cromolyn • Very short duration


• Inhibit mast cell
sodium of action needing
Cromones and sensory nerve
• Nedocromil frequent dosing
activation
sodium • Favorable safety profile

I
.•.
Add-on'Therapies _.• {<

• Very expensive
Anti-IgE • Inhibit lgE-mediated
• Omalizumab • Limited to patients with
antibodies reactions
elevated serum IgE
• Reduce exacerbations
• Block IL-5or its
• Mepolizumab in those with
Anti-IL5 receptor
• Reslizumab persistently increased
antibodies • Reduce blood & tissue
• Benralizumab sputum eosinophils
eosinophils
despite maximal JCS
• Builds tolerance to • Potential for
• Injected extracts
triggers through anaphylaxis
Immunotherapy of pollen or
gradual introduction • Sublingual dosing may
house dust mites
of allergens be safer
Sources:JamesonJL,et al. Harrison'sPrinciples
of InternalMedicine
20thedition,2018
ExpertPanelReport3: Guidelines
for theDiagnosisandManagement ofAsthma,NHLBI2007
240
III. DAILY DOSES OF INHALED CORTICOSTEROIDS (JCS) FOR ADULTS

DRUG

Beclometasone
dipropionate (pMDI, 200-500 >500-1000 >1000
standard particle, HFA)
Beclometasone
dipropionate (DP! or pMDI, 100-200 >200-400 >400
extra fine particle, HFA)
Budesonide (DP! or pMDI, 200-400 >400-800 >800
standard particle HFA)
Fluticasone furoate (DP!) 100 100 200

Fluticasone propionate 100-250 >250-500 >500


(DP!)

Fluticasone propionate
(pMDI, standard particle, 100-250 >250-500 >500
HFA)

Mometasone furoate (pMDI, 200-400 200-400 >400


standard particle, HFA)
• Thistable does NOTimplypotencyequivalence.
pMDI:pressurizedmetereddose inhaler
HFA:hydrofiuoroalkane
DPI:drypowderinhaler
Source:GlobalInitiative
forAsthma(GINA),2021

IV. TYPES OF INHALERS


TYPE I HOW TO USE*
Pressurized Metered Dose • Remove cap, hold inhaler upright, and shake
Inhaler (pMDI) • Breathe out normally, then seal lips around
mouthpiece
• Start breathing slowly and at the start of inspiration,
which should be slow and deep, press down on the
canister to release the dose while continuing to inhale
deeply through the mouth
• Hold your breath for IO seconds, or as long as possible
• Breathe out gently
• Repeat above steps as necessary
Metered Dose Inhaler with • Remove cap, hold inhaler upright, and shake
Spacer • Insert inhaler mouthpiece into the receiving end of the
spacer
• Breathe out normally, then seal lips around the
mouthpiece of the spacer while keeping it horizontal
• Press down on the canister to release the dose
• Continue to inhale slowly through the mouth, and
hold breath for around IO seconds
• While holding breath, remove spacer from mouth,
then breathe out gently
• Repeat above steps as necessary

241
TYPE I HOW TO USE*

Dry Powder Inhaler (DP!) - • Unscrew and remove cover


Turbohaler • Keep inhaler upright while twisting grip at the base:
twist around and then back until a click is heard
• Breathe out gently (away from inhaler)
• Place mouthpiece between teeth (without biting) and
close lips to form a good seal (do not cover the air vents)
• Breathe in steadily and deeply
• Remove inhaler from mouth and hold your breath for IO
seconds (or longer, if possible)
• Breathe out gently (away from inhaler)
• Repeat above steps as necessary
Dry Powder Inhaler (DP!) - • Open cover by using the thumb and pushing until a click
Diskus is heard, revealing the mouthpiece
• Load dose: keep device horizontal while sliding lever
until another click is heard (do not shake)
• Breathe out gently (away from inhaler)
• Place mouthpiece in mouth (without biting) and close
lips to form a good seal (keep inhaler horizontal)
• Breathe in steadily and deeply
• Hold breath for about IO seconds (or longer, if possible)
• While holding breath, remove inhaler from mouth, then
breathe out gently (away from inhaler)
• Repeat above steps as necessary
Dry Powder Inhaler (DP!) - • Open cap of the inhaler device, then open cap of the
Handihaler mouthpiece inside the device
• Remove the capsule from the blister pack (do not swallow
capsule), and place the capsule in the capsule reservoir
• Close the mouthpiece cover
• Push the button on the side of the inhaler to puncture the
capsule and allow the medication to be released and inhaled
• Put mouth on mouthpiece & tightly seal the lips around it
• Breathe in steadily and deeply
• Remove inhaler from mouth
• Hold your breath for IO seconds, or as long as possible
• Breathe out gently
• Open mouthpiece cover and discard used capsule
• Close cover
Dry Powder Inhaler (DPI) - • Hold rotahaler vertically and put capsule (clear end first)
Rotahaler (S) into the hole
• Hold rotahaler horizontally & twist barrel to open capsule
• Breathe out gently (away from inhaler)
• Keep rotahaler horizontal & place mouthpiece between lips
• Breath in the powder quickly and deeply
• Hold breath for IO seconds (or longer, if possible), then
breathe out gently
• If any powder is left, repeat above steps as necessary
• Open rotahaler and discard empty capsule
'If inhalercontainsan ICS,rinsemouthwithwaterafteruseto avoidoralthrushfromsteroidresidue.
Someexamples of commoninhalersavailable:
+ Fluticasone
• Salmeterol 25/250mcgMDIor 25/125mcgMDI
• Salmeterol
50mcg+ Flulicasone250mcgDiskus250or Diskus500(S 50 mcg+ F 500mcg)
+ Formoterol
• Budesonide 160/4.5mcgDPIor 320/9mcg
242
V. NON-PHARMACOLOGIC THERAPY OF ASTHMA
Smoking cessation
Regular physical activity
Avoidance of occupational exposures
Avoidance of medications that may worsen asthma (e.g., aspirin, NSAIDs)
Healthy diet
Weight reduction for obese patients
Breathing techniques
• Immunizations (influenza and pneumococcal vaccines)

VI. LONG-TERM GOALS OF ASTHMA THERAPY


To achieve good control of symptoms
To maintain normal activity levels (i.e., no limitation in activities), including exercise
To minimize risk of asthma-related death
Minimal (ideally no) chronic symptoms, including nocturnal symptoms
Minimal (infrequent) exacerbations
• Minimal (or no) adverse effects from medications

EXACERBATIONS IN ASTHMA

I. ETIOPATHOGENESIS & MANIFESTATIONS


Exacerbations represent an acute/subacute worsening of symptoms & lung function
from the usual status that may require a step-up in treatment
May be the initial presentation of asthma
Occur in response to an external agent (e.g., viral upper respiratory tract infection, :
pollution, etc.) and/or poor adherence to medications
Manifests as episodes of progressive increase in symptoms of dyspnea, cough, wheezing ·
or chest tightness and progressive decrease in lung function

II. DIAGNOSIS
DIAGNOSTIC I REMARKS

• Can quantify the decrease in expiratory airflow during exacerbations


PEForFEV1
(more reliable than symptoms)
• Not routinely required
Arterial
• Considered if PEF or FEV1 <50% predicted or those who do not
blood gas
respond to initial treatment or are deteriorating
(ABG)
• Respiratory failure: suggested by PaOz <60 rnmHg&/or PCO2 >45 rnmHg
• Not routinely recommended
Chest X-ray • Considered if a complicating alternative diagnosis is considered (e.g.,
pneumothorax) or for those who are not responding to treatment

Oxygen
• Monitored closely, preferably by pulse oximetry
saturation

PEF: Peak expiratoryflow


FEV1:Forced expiratoryvolume in 1 second
Source:GlobalInitiativeforAsthma(GINA),2021

243
III. MANAGEMENT OF EXACERBATIONS
• Mainstay of treatment: high doses of SABA (either by nebulizer or via MDI with a spacer)
Nebulized anticholinergic (ipratropium bromide) may be added if needed
Prophylactic intubation for impending respiratory failure (i.e., when PC02 is normal or rises)

I MILD OR MODERATE
I SEVERE ILIFE-THREATENING
Assessment oif p,at1e!!t'fior sev~tyo ifExacerbanon -. .. D. ,,.
• Talks in phrases • Talks in words
Clinical • Prefers sitting to lying • Sits hunched forward
• Not agitated • Agitated

Respiratory • Increased • >30/min


• Drowsy, confused
rate • No accessory muscle use • Use of accessory muscles
• Silent chest
Pulse rate , wo-120 bpm • >120 bpm

02 saturation • 90-95% • <90%

PEF • >50% predicted or best • ~50% predicted or best


r,, l! ,,,. ···--·
Management Acco11~ingto Setting -~ --;
=
• SABA (4-IO puffs by
pMDI + spacer • Transfer to acute care facility
Management 920 mins for I hour)' • Give inhaled SABA and ipratropium bromide,
in primary • Prednisolone 40-50 mg 02 support, and systemic corticosteroids (while
care units • Oxygen (target 02 93-95%) awaiting transfer)
• If worsens at I hr, transfer • Consider referral to specialists
to acute care facility

• SABA • ICU admission


• Ipratropium bromideb • SABA & 02 support
,SABA
• Oxygen (target 02 93-95%) • Hydrocortisone200mg/
• Consider ipratropium
Management • Prednisolone (50 mg OD day IVin divideddoses
bromide
in an acute PO) or hydrocortisone • Intubation
• 02 (target 02 93-95%)
care facility (200 mg/day IV in divided • Evidence for non-
• Prednisolone 50 mg OD
doses) for 5-7 days invasive ventilation
PO (or equivalent)
• Consider IV magnesium (NIV) is weak
• Consider high dose !CS • Refer to specialists

• If continuing deterioration, treat as severe and admit to ICU


Monitoring
• Consider referral to specialists

Disposition' II
Admit to • Pre-treatment FEV1 or PEF is <25% of predicted or personal best
ward or ICU • Post-treatment FEV1 or PEF is <40% of predicted or personal best
Treat&
• Post-treatment FEV1 or PEF is 40-60% of predicted or personal best
reassess

• Post-treatment FEV1 or PEF is >60% of predicted or personal best


Consider
• Symptoms improved, not needing SABA
discharge
• 02 saturation >94%
from ER•
• Resources at home are adequate
• Afterfirst hour of inhaled'SABA,dose varies from4-10puffsq3-4h·oursup to 6-10puffsq1-2hours
'Treatment with both SABAand ipratropiumfor moderate-severe exacerbations is better than SABAalone
' Clinical status should be reassessed frequently. Measure lung function in all patients 1 hour after initial
treatment Consider ris~factors & availabilityof follow-upcare whenever discharging patients from the ER
'On discharge, prescribe controller (e.g., prednisolone 1 mg/kg/day or equivalent 40-50mg/day x 5-7days
& initiate regular !CS). Ifalready on regular !CS prior to exacerbation, increase dose. Follow-upin 2-7days
.. for Asthma (GINA),2021& Grundeld A, et al. Can Resp1rJ. 1996
Source: Global lrnt1at1ve
244
ASTHMA-COPD OVERLAP (ACO)
Describes patients who have features of both asthma and COPD, which are conditions
characterized by airway obstruction
Reported prevalence of 9% to 55% with variation by gender and age
• Referral to specialists for confirmation of diagnosis and management is recommended
• Outcomes for ACO are often worse than for asthma or COPD alone due to greater
burden of symptoms, more frequent exacerbations, poorer quality of life, more rapid
decline in lung function, and higher mortality

Approach to Initial Management in Patients with Asthma and/or COPD

I
HIGHLY LIKELY TO BE
ASTHMA
I
FEATURES OF BOTH
ASTHMA & COPD
I LIKELY TO BE COPD

History & Lung Function '!',


-
• Vary over time and in • Intermittent or
Symptoms • Dyspnea persistent
intensity episodic
• Before or after 40
Onset • <40 years old • >40 years old
years old

• Laughter, exercise, • History of smoking, toxic exposures


Triggers
allergens, seasonal • History oflow birth weight or tuberculosis
Resolution
• Symptoms improve spontaneously or with • Limited relief with
of
bronchodilators or !CS bronchodilators
symptoms
• Variable expiratory
airflow limitation
Lung • Persistent expiratory airflow limitation
• Persistent airflow
function • With or without bronchodilator reversibility
limitation may be
present
ii
Management -
Overview • Treat as asthma • Treat as asthma • Treat as COPD
• LAMA &/or LABA
• Add !CS as per
• !CS-containing • !CS-containing GOLD guidelines
treatment is essential treatment is for patients with
• As needed low dose essential hospitalizations or
Drug ICS-formoterol may • Add-on LABA and/ ~2 exacerbations/
therapy be used as reliever or LAMA as needed year requiring OCS
• Do not give LABA • Do not give LABA • Avoid high dose !CS
and/or LAMA without and/or LAMA (risk of pneumonia)
an!CS without!CS • Reliever containing
!CS is not
recommended
Sources:GlobalInitiativefor Asthma(GINA),2021

245
SECTION THREE
CHRONIC OBS1'RUCTIVE PULMONARY DISEASE

CHRONIC OBSTRUCTIVE PULMONARY DISEASE (COPD)


I. ETIOPATHOGENESIS
Characterized by persistent respiratory symptoms and airflow limitation due to airway
and/or alveolar abnormalities caused by a significant exposure to noxious particles/gases
• Unusual in the absence of current or prior smoking history, except for patients with
alpha-I antitrypsin deficiency
COPD encompasses the following conditions:
0 Emphysema: anatomically defined condition characterized by enlargement and
destruction of alveoli
° Chronic bronchitis: condition characterized by chronic cough & sputum production
0 Small airways disease: condition where the bronchioles & smaller airways are narrowed
II. MANIFESTATIONS
• Most common symptoms: cough, sputum production, exertional dyspnea
Symptoms
• COPD may be punctuated by exacerbations (acute worsening of symptoms)
• May be normal in early stages
• Pink puffers (predominantly emphysema): thin, non-cyanotic, prominent use of
accessory muscles
• Blue bloaters (predominantly chronic bronchitis): heavy and cyanotic
Signs • Tripod position: to facilitate use of accessory muscles
• Signs of hyperinflation: barrel chest, hyperresonance on percussion
• Pursed-lip breathing, expiratory wheezing, systemic wasting, weight loss
• Signs of cor pulmonale (bipedal edema, ascites) in severe cases
• Clubbing is not a sign of COPD (search for other causes of clubbing)

III. DIAGNOSIS
A clinical diagnosis ofCOPD should be considered in any patient who has dyspnea,
chronic cough or sputum production, a history of recurrent lower respiratory tract
infections, and a history of exposure to risk factors for the disease
Risk factors include tobacco smoke (main risk exposure), smoke from home cooking
and biomass fuels, occupational dusts and chemicals
DIAGNOSTIC I COMMENTS/EXPECTED FINDINGS
• Required to make the diagnosis ofCOPD
• Post-bronchodilator FEV1/FVC <0.70 confirms the presence of
Spirometry
persistent airflow limitation
• FEV1, FEV1/FVC & all other measures of expiratory airflow are reduced
• Useful for excluding other differential diagnosis (not useful to establish
diagnosis ofCOPD)
Chest • Low flattened diaphragms, increase in the volume ofretrosternal
radiograph airspace (hyperinflation)
• Hyperlucent lung zones with possible bullae formation and diminished
vascular markings
• Noc routinely requested
CT scan • Helpful when the diagnosis is in doubt to rule out concomitant diseases
• Useful if surgical procedures such as lung volume reduction is contemplated

• To evaluate 02 saturation & need for supplemental oxygen therapy


Pulse • Should be used to assess all stable patients with FEV1 <35%predicted or
oximetry with clinical signs suggestive of respiratory failure or right heart failure
• If peripheral saturation is <92%, arterial blood gas should be determined
246
DIAGNOSTIC I COMMENTS/EXPECTED FINDINGS

• Resting or exertional hypoxemia


Arterial
• Increased alveolar-arterial oxygen tension gradient (A-a gradient)
blood gases
• In long-standing disease, may have chronically increased arterial Pa CO2 but
(ABG)
metabolic compensation (increased HCO3) maintains pH at near normal levels
• Documented by body plethysmography or helium dilution lung volume
measurement (less accurate)
Lung
• COPD patients exhibit gas trapping (i.e., rise in residual volume or RV),
volumes&
and as airflow limitation worsens, static hyperinflation (i.e., an increase
diffusing
in total lung capacity or TLC) occurs
capacity
• DLCO (diffusing capacity of the lung for carbon monoxide) may be
reduced as a functional impact of emphysema
Sources:JamesonJL, et al. Harrison'sPrinciplesof InternalMedicine20thed1t1on,
2018
GlobalInitiativefor ChronicObstructive LungDisease(GOLD),2021

APPROACH TO CLASSIFICATION OF COPD


New "ABCD"assessment tool incorporates patient-reported outcomes & highlights importance
of exacerbation prevention (the old GOLDclassificationis only a "spirometric
gradingsystem')

I. STEP 1: CONFIRM THE DIAGNOSIS OF COPD


COPD is considered in patients with dyspnea, chronic cough, and sputum production
with a history of exposure to risk factors (e.g., smoking)
Spirometry is required to confirm diagnosis: post-bronchodilator FEV1/FVC <0.70
° FEV1/FVC ratio: reflects the proportion of a person's forced expiratory capacity that
they are able to expire in the first second of forced exhalation to the full vital capacity :
0 Interpretation: value <70% means the patient can only expire <70%of the air inside the
lungs in I second (the lower the value, the more severe the obstructive airway disease) ·

II. STEP 2: ASSESS AIRFLOW LIMITATION BY SPIROMETRY


Classificationis based on severityof airflow limitation on spirometry (post-bronchodilator FEV1)
Spirometry should be performed after the administration of an adequate dose of a
short-acting inhaled bronchodilator (to minimize variability)

GOLD
I CLINICAL FINDINGS

• Chronic cough & sputum production


GOLDI :?:80%
• Patient unaware that lung function is
Mild predicted
abnormal
• Chronic cough & sputum production
GOLD2 • Shortness of breath on exertion 50 to <80%
Moderate • Stage wherein patients typically seek predicted
medical attention Post-
Bronchodilator
• Greater shortness of breath FEV1/ FVC<0.70
GOLD3 • Reduced exercise capacity 30 to <50%
Severe • Fatigue predicted
• Repeated exacerbations

GOLD4 • Respiratory failure (PaO2 <60 mmHg


<30%
Very +/- PaCO2 >50 mmHg) predicted
Severe • Cor pulmonale
FEV1:forced expiratoryvolume in 1 second
FVC:forced vitalcapacity
Source:GlobalInitiativefor ChronicObstructiveLungDisease (GOLD),2021
247
III. STEP 3:ASSESS FOR SYMPTOMSAND RISKOF EXACERBATIONS
In the refined scheme, the spirometric (GOLD) grades are separated from the "ABCDgroup"
ABCD groups are derived exclusively from the patient's history of exacerbation and
symptom assessment (e.g., mMRC, CAT)

Low symptom severity S1 (not leading to


A Low exacerbation risk hospital admission)
mMRC0-1 CAT<10

High symptom severity s1 (not leading to


B mMRC,!:2 CAT,!:10
Low exacerbation risk hospital admission)
Low symptom severity ,!:2or ,!:1leading to
C High exacerbation risk hospital admission
mMRC0-1 CAT<10

High symptom severity ,!:2or ,!:1leading to


D mMRC,!:2 CAT,!:10
High exacerbation risk hospital admission
'SymptomAssessment formalsymptomaticassessment is required,since there is a weakcorrelation
betweenFEV1(spirometry),
symptoms,and impairmentof a patient'shealthstatus
mMRC:modifiedMedicalResearchCouncil;CAT:COPDAssessmentTest
Example1) PatientwithFEV125%of predictedand CATscore of 16. He had no exacerbationsin the past
year.Diagnosis:GOLDGrade4; GroupB
Example2) PatientwithFEV125%ofpredictedandCATscoreof 16. Hehadthreeexacerbations
inthe pastyear.
Diagnosis:GOLDGrade4; GroupD
Source:GlobalInitiativeforChronicObstructiveLungDisease(GOLD),2021

A. Classification Based on Exacerbations


0Exacerbation ofCOPD: an acute event characterized by worsening of the patient's
symptoms that is beyond normal day-to-day variations & leads to a change in medication
0The best predictor of having frequent exacerbations (~2 per year) is a history of a
previous exacerbation

R 1 11 h fh 1 h & d' f .
B. Modified Medical Research Council (mMRC) Questionnaire (Severity of Breathlessness)
I' .k
mMRC
Grade
I DESCRIPTION

0 • I only get breathless with strenuous exercise

1 • I get short of breath when hurrying on the level or walking up a slight hill
• I wa]k slower than people of the same age on the level because of breathlessness,
2 or I stop for breath when walking on my own pace on the level
3 • I stop for breath after walking !00 meters or after a few minutes on the level
4 • I am too breathless to leave the house or I am breathless when dressing or undressing

C. COPD Assessment Test (CAT)


° Comprehensive assessment of symptoms
° Consists of8 items which pertain to symptoms of CO PD- patient will give a score (0-5
point rating scale) and the points will be added
0 Questionnaire is available online, but still to be validated in the Philippine setting

Sources:FletcherCM.MRCbreathlessnessscore. BMJ1960
GlobalInitiativeforChronicObstructiveLungDisease(GOLD),2021
Jones et al., ERJ 2009
248
MANAGEMENT OF STABLE COPD
Based on individualized symptom assessment & future risk of exacerbations
• Main goals of management:
0 Reduction of symptoms (relief of symptoms, improve exercise tolerance & health status)
0 Reduction of future risk (prevent disease progression, exacerbations, and mortality)

I. ONLYTHREE INTERVENTIONS DEMONSTRATED TO IMPROVE SURVIVALIN COPD


THERAPY I REMARKS
• Biggest impact in the natural history ofCOPD
• Nicotine replacement therapy (gum, inhaler, nasal spray, transdermal
Smoking patch) reliably increases long-term smoking abstinence rates
cessation • Varenicline, bupropion, and nortriptyline increase long-term quit
rates, but should be used as part of an interventional program
• E-cigarettes as a smoking cessation aid: more harm than benefit

• The only pharmacologic therapy demonstrated to unequivocally


decrease mortality rates in COPD
Oxygen • Prescribe oxygen and titrate to keep SaO2 2e90%in patients with
therapy arterial hypoxemia, defined as:
0 PaO2 <55 mmHg or SaO2 <88%, or
0 PaO2 55-59 mmHg with right heart failure or erythrocytosis
Lung reduction • Segmentectomy or lobectomy of focal emphysematous areas of the

1
surgery lung
Sources:JamesonJL,et al. Harrison'sPrinciples
of InternalMedicine20thedition,2018
GlobalInitiativefor ChronicObstructiveLungDisease(GOLD),2021 .

II. PHARMACOLOGIC THERAPY FOR STABLE COPD


In general, inhaled bronchodilators are the primary treatment for almost all patients ·
with COPD (e.g., B2-agonists, anticholinergics and methylxanthines)
None of the existing medications have been shown to modify the long-term decline in
lung function
GROUP I EXAMPLES I COMMENTS I SIDE EFFECTS
Short-acting • Alters airway smooth muscle
(SABA): tone, improving emptying of
• Salbutamol the lungs
• Terbutaline • Effects usually last 4-6 hours
• Tachycardia
(short-acting) or >12hours
Long-acting • Arrhythmia
Beta (long-acting)
(LABA): • Tremors
2-agonists • Regular treatment with
• Formoterol • Hypokalemia
LABA is more effective &
• Salmeterol convenient than SABA
• Vilanterol • Givessubjective benefit in acute
• Olodaterol episodes, but not necessarily
• Indacaterol helpful in stable disease
Short-acting • Blocks acetylcholine's effect
(SAMA): on muscarinic receptors
• Ipratropium • Bronchodilating effect
bromide of short-acting inhaled • Dry mouth
Anti- • Oxitropium anticholinergics lasts longer • Bitter metallic
cholinergics bromide than that of short-acting taste
(Muscarinic
Long-acting beta2-agonists • Arrhythmias
antagonist)
(LAMA): • Improves symptoms, health
• Tiotropium status, and effectiveness of
• Umeclidinium pulmonary rehabilitation;
• Glycopyrronium reduces exacerbations

249
GROUP I EXAMPLES I COMMENTS I SIDE EFFECTS
• Nonselective • Tachycardia
• Theophylline
Methyl- phosphodiesterase inhibitor • Arrhythmias
• Aminophylline
xanthines • Improves FEV1 & dyspnea • Headache
• Doxofylline
when added to salmeterol • Insomnia

, Adding !CS to
bronchodilators is
appropriate for:
• Beclomethasone 0 Symptomatic patients
• Hoarseness
Inhaled • Budesonide with FEV1 <50%
• Prone to
corticosteroids • Mometasone predicted (stage III & IV)
candidiasis
(JCS) • Fluticasone 0 Repeated exacerbations
• JCS combined with LABA
in moderate to very severe
COPD is more effective than
either component alone

• Improves lung function


Phospho- • Reduces moderate & severe
• Weight loss
diesterase-4 exacerbations in chronic
• Roflumilast • Diarrhea
(PDE-4) bronchitis or those who are
• Headache
inhibitors on fixed-dose LABA-ICS
combinations
• Azithromycin
250 mg/day, OR
• Azithromycin • Long-term macrolide use reduces exacerbations
Antibiotics
500 mg 3x/week,OR if taken over I year
• Erythromycin
500 mg 2x/day

• N-acetylcysteine
Mucolytics/ • Antioxidants reduce the risk of exacerbations in
• Erdosteine
antioxidants selected populations
• Carbocysteine

• Influenza vaccine: decreases incidence of lower respiratory tract infections


• PCV13& PPSV23 are recommended for all ~65 years of age:
0 23-valent pneumococcal polysaccharide vaccine (PPSV23):reduces CAP in

COPD patients <65years & FEV1<40%predicted & in those with comorbids


Vaccinations 0 13-valent conjugated pneumococcal vaccine (PCV13):reduces bacteremia

& invasive pneumococcal disease in the general population ~65 years


• CDC recommends Tdap (dTaP/dTPa) vaccination for adults with COPD
who were not vaccinated in adolescence to protect against pertussis
Sources:JamesonJL,et al. Harrison'sPrinciples
of InternalMedicine20thedition,2018
GlobalInitiativefor ChronicObstructiveLungDisease(GOLD),2021

III. NON-PHARMACOLOGIC MANAGEMENT OF COPD


DEPENDING ON
I
PATIENT
ESSENTIAL RECOMMENDED
GROUP LOCAL GUIDELINES

A • Smoking cessation • Flu vaccine


• Smoking cessation • Pneumococcal
• Physical activity
vaccine
B-D • Pulmonary
rehabilitation • Pertussis vaccine

Source:GlobalInitiativefor ChronicObstructive
LungDisease(GOLD),2021
250
IV. TREATMENT FOR STABLE COPD BASED ON GOLD ABCD CLASSIFICATION
Most treatments are inhaled (proper inhaler technique is important)
LABAs & LAMAs: preferred over short-acting agents, except if with only occasional dyspnea
May start with a single long-acting bronchodilator or dual long-acting bronchodilator
• Long-term monotherapy with JCS is not recommended

GROUP I PREFERRED
TREATMENT I
NEXT STEP IF NO
IMPROVEMENT* I OTHER OPTIONS

• Any bronchodilator • Continue, stop or try


A (short- or long-acting) alternative bronchodilator
• Antioxidant mucolytics

B • LAMA or LABA • LAMA+ LABA • Add SAMA, SABA

•LAMA+ LABA if with


C • LAMA further exacerbations • Add SAMA, SABA
• Alternative: LABA+ !CS
• Add SAMA, SABA
• LAMA+ LABA+ JCS
• Consider PDE-4 inhibitor if
• LAMA,or (triple therapy)
FEV1 <50% predicted & has
D •LAMA+ LABA', or • Alternative: LABA+ JCS
chronic bronchitis
•JCS+ LABAh before going to triple
• Consider macrolide (in
therapy
former smokers)
• Consider to initiatewith LAMA+ LABAif with severe symptoms (CAT>20)
'Consider to initiatewith ICS + LABAif with:
• Historyof hospitalizationfor exacerbations
• <!:2moderate exacerbations/year

I
• Bloodeosinophils >300 cells/ul
• Historyof/concomitantasthma
• Defer ICS if with repeated pneumonia, blood eosinophils <100 cells/ul, history of mycobacterialinfection

EXACERBATIONS IN COPD
I. ETIOPATHOGENESIS
Increased airway inflammation, increased mucus production, and marked gas trapping
• Mainly triggered by respiratory viral infections (others: bacterial infections, environment)
Strongest predictor of COPD exacerbation is a history of previous exacerbation

II. MANIFESTATIONS
Acute change in patient's baseline dyspnea (key symptom during exacerbations), cough
&/or sputum beyond normal day-to-day variations & warranting additional therapy
Change in mental status is the most important sign of a severe exacerbation
Symptoms usually last between 7-10 days, but 20% of patients do not fully recover

PARAMETER I NO RESPIRATORY
FAILURE

• 20-30 breaths/min
Respiratory • >30 breaths/min
• No use of accessory
rate • With use of accessory muscles
muscles
Changein
• None • Yes (acute changes)
mental status
• Improved with 02 • Not improved with 02
• Improved with 02
Hypoxemia support at 28-35% via Venturi mask, or
support at 35-40% FiO2
FiO2 • Requiring FiO2 >40%
• Hypercarbia (increased • Hypercarbia (increased
from baseline or from baseline or
PaC02 • Not increased
elevated at 50-60 elevated >60 mmHg or
mmHg) with acidosis (pH 7.25)
Sources: Global Initiativefor Chrome Obstructive Lung Disease (GOLD),2021
251
III. MANAGEMENT OF ACUTE COPD EXACERBATIONS
Goal is to minimize the negative impact of current exacerbation & prevent subsequent events
Assess severity of symptoms, blood gases and chest radiograph
• Administer controlled oxygen therapy and repeat ABG after 30-60 minutes
Consider non-invasive ventilation (NIV) or intubation if indicated
Most commonly used drugs for exacerbations: inhaled bronchodilators (SABA+/-
SAMA), systemic corticosteroids, antibiotics

Mild • Short-acting bronchodilators


Moderate • Short-acting bronchodilators +antibiotics+/- oral corticosteroids
• Requires hospitalization or ER visits
Severe
• Some managed as acute respiratory failure (e.g., ventilator)

• Initial bronchodilators: inhaled short-acting B-agonists (SABA)


+/- short-acting anticholinergics
Bronchodilators • Frequency depends on severity of exacerbation
• Maintenance long-acting bronchodilators should be initiated
before hospital discharge
• Evidence supports use of antibiotics in patients with
exacerbations and increased sputum purulence
• Bacteria frequently implicated in exacerbations: Streptococcus
Antibiotics pneumoniae, Haemophilus influenzae,Moraxella catarrhalis
• Duration: 5-7 days
• Procalcitonin-guided antibiotic treatment may reduce
antibiotic exposure and side effects with same clinical efficacy
• Reduces hospital stay, hastens recovery, and reduces chances
of subsequent exacerbations/relapses
• Prednisone 40 mg/day (or equivalent) can improve lung
Systemic
function (FEV1)and oxygenation
corticosteroids
• Duration: not more than 5-7 days (avoid prolonged/chronic use
of systemic steroids)
• Most common acute complication of steroids: hyperglycemia
Sources:
Jameson
JL,et al. Harrison's
Principles
of InternalMedicine
20thedition,2018
Global Initiativefor Chronic ObstructiveLung Disease (GOLD), 2021

IV. RESPIRATORY SUPPORT


A. Oxygen Therapy
0 Should be titrated to improve hypoxemia with a target saturation of88-92%
0 Administration of oxygen does not reduce minute ventilation

B. High-Flow Nasal Therapy (HFNT)


0 Delivers heated & humidified air-oxygen blends via special devices at rates up to 60 Umin
0 Associated with:
• Decreased respiratory rate & effort
• Decreased work of breathing
• Improved gas exchange
• Improved lung volume & dynamic compliance
• Increased transpulmonary pressures
2S2
C. Ventilatory Support
0 Non-invasive ventilation should be the first mode of ventilation in COPD patients
with acute respiratory failure who have no absolute contraindication

NON-INVASIVE VENTILATION I INVASIVE MECHANICAL


VENTILATION
• Selection Criteria (at least one of the • Indications:
following): 0 Unable to tolerate NIV or NIV failure
Severe dyspnea with use of accessory
0 0 Respiratory or cardiac arrest
muscles & paradoxical abdominal 0 Somnolence, impaired mental status
motion 0 Massive aspiration or persistent
Respiratory acidosis (pH s7.35)and/or
0 vomiting
hypercapnia (PaC02 >45 mmHg) 0 Persistent inability to clear respiratory
Persistent hypoxemia despite
0 secretions
supplemental 02 therapy 0 Severe hemodynamic instability
without response to fluids &
• Exclusion Criteria (presence of any of
vasoactive drugs
the following): 0 Severe ventricular or supraventricular
0Respiratory arrest
arrhythmias
° Cardiovascular instability
0 Life-threatening hypoxemia in those
° Change in mental status or
unable to tolerate NIV
uncooperative patient
0High aspiration risk
0Viscous or copious secretions
0Recent facial/gastroesophageal
surgery
° Craniofacial trauma
° Fixed nasopharyngeal abnormalities
0 Burns

0Extreme obesity
Source:GlobalInitiativefor ChronicObstructive
LungDisease(GOLD),2021

V. HOSPITAL DISCHARGE CRITERIA


Review clinical and laboratory data
• Check maintenance therapy and understanding
Reassess inhaler technique
• Ensure understanding of withdrawal of acute medications (steroids and/or antibiotics)
Assess need for continuing oxygen therapy
• Provide management plan for comorbidities and follow-up
• Early follow up at <4 weeks, or late follow-up <12weeks as indicated

253
SECTION FOUR
PNEUMONIA
COMMUNITY-ACQUIRED PNEUMONIA (CAP)
I. ETIOPATHOGENESIS
Lower respiratory tract infection acquired in the community within 24 hours to <2 weeks
Results from the proliferation of microbial pathogens at the alveolar level and the host's
response to those pathogens
Most common access of microorganisms to the lower respiratory tract is through
aspiration from the oropharynx
Classic pneumonia (lobar pneumococcal pneumonia) evolves through a series of changes:

PHASE I DESCRIPTION
• Initial phase with the presence of a proteinaceous exudate, and often with bacteria
Edema
in the alveoli
• Presence of erythrocytes in the cellular intraalveolar exudate
Red
• Neutrophil influx is more important from the standpoint of host defense
hepatization
• Bacteria are occasionally seen in pathologic specimens
• No new erythrocytes are extravasating; those already present have been degraded
Gray • The neutrophil is the predominant cell, fibrin deposition is abundant, and bacteria
hepatization have disappeared
• Corresponds with successfulcontainment of infection& improvement in gas exchange
Resolution • Macrophage reappears as the dominant cell type in the alveolar space
(final phase) • Inflammatory response and cellular debris have been cleared

II. MANIFESTATIONS
Commonly presents with acute cough, tachypnea, tachycardia, and fever with at least
one abnormal chest finding of diminished breath sounds, rhonchi, crackles or wheezing
Defined as "new lung infiltrate+ evidence that infiltrate is of an infectious origin, which
includes new onset offever, purulent sputum, leukocytosis, & decline in oxygenation"

III. DIAGNOSIS
A. Classification and Disposition

I LOW-RISK CAP I MODERATE-RISK CAP I HIGH-RISK CAP

Stable Unstable
• RR <30/min • RR2c30/min
Vital signs • PR <125bpm • PR2c125bpm
• Temp of 36-40 °C • Temp 2c40°C or s;36°C Any of the criteria
• BP ,90/60 mmHg • BP s;90/60 mmHg under Moderate-Risk
CAP, PLUS any of the
• No altered mental • Altered mental state following:
state of acute onset of acute onset • Severe sepsis and
Features • No suspected • Suspected aspiration septic shock, OR
aspiration • Decompensated • Need for mechanical
• No or stable comorbids comorbids* ventilation

• Localized infiltrates • Multilobar infiltrates


Chest
• No pleural effusion • Pleural effusion
X-ray
• No abscess • Abscess
Disposition • Outpatient • Ward admission • ICU admission
*Unstable
or decompensated comorbidconditions
include:uncontrolled
DM,activemalignancies,
neurologic
diseaseinevolution,
CHFFCII-IV,unstable
CAD,renalfailureondialysis,
uncompensated
COPD,decompensated liverdisease
..
Source:Ph1hpp1ne
CPGon Commurnty-Acqu1red Pneumonia(CAP),2016
254
B. Diagnostics
DIAGNOSTIC I DESCRIPTION
• Essential in the diagnosis of CAP, assessing severity, differentiating
pneumonia from other conditions, and for prognostication
Chest
• Best radiologic evaluation consists of standing posteroanterior and
radiography
lateral views of the chest
• Does not predict the likely etiologic agent
• Strongly influenced by the quality of collection, transport, & processing
Sputum • Main purpose of gram stain is to ensure that a sample is suitable for
gram stain& culture - an adequate sputum sample must have:
culture 0 >25neutrophils/LPF

0 <10squamous epithelial cells/LPF

• Yield is relatively low, therefore it is optional for hospitalized patients


Blood • Most common isolate: S. pneumoniae
cultures • Strongest indication for blood cultures: severe CAP (more likely to be
infected with S. aureus, P. aeruginosa, other gram-negative bacilli)
• Includes transtracheal, transthoracic biopsy, bronchoalveolar lavage,
and protected brush specimen
• Options for non-resolving pneumonia, immunocompromised
Invasive
patients, and in patients in whom no adequate respiratory specimens
procedures
can be sent despite sputum induction and routine diagnostic testing
• Most clear-cut indication for extensive diagnostic testing is in the
critically-ill CAP patient
Sources:Jameson
JL,et al. Harrison's
Principles
of InternalMedicine
20thedition,2018
MetlayJP,et al.ATS/IDSACPGonCommunity-Acquired Pneumonia. AmJ RespirGritCareMed.2019

C. Pneumonia Risk Score (CURB-65):predicts mortality in CAP


• Confusion of new onset

=1-•-U_r_e_a_(_B_U_N_)_2:_7_m_m_o_l_i_L_(_19_m_;;g_ld_L_)
___ Jnterpretation:
• 0-1 point: treat as outpatient
• Respiratory rate 2:30breaths per minute
• 2 points: admit patient

• Blood pressure <90/60 mmHg • 2:3points: consider ICU admission

• Age 2:65years old


Source:LimWS,et.al.Thorax.2003

IV. MANAGEMENT
For those requiring hospitalization, empiric therapy should be initiated as soon possible
Most bacterial pneumonias can be treated for 5-7days
Response to therapy is expected within 24-72 hours of initiating treatment (e.g., fever
decreases within 72 hours, temperature normalizes within 5 days, and respiratory signs,
particularly tachypnea, return to normal)

General Guide for the Duration of Therapy:


• 5-7 days:for low-risk(uncomplicated)
CAP
• 7-10days:for moderate-risk CAP
• Prolongedto 28 days:for moderate-& high-riskCAPor thosewithsuspected/confirmed
gram-
negative,S. aureus,or P. aeruginosa
pneumonia
Sources:
Philippine
CPGonCommunity-Acquired
Pneumonia (CAP),2016
MetlayJP,et al.ATS/IDSA
CPGonCommunity-Acquired AmJ RespirGritCareMed.2019
Pneumonia.
DOHNational
AntibioticGuidelines
2017
255
Without co-morbids: Amoxicillin, OR
• Streptococcuspneumoniae • Amoxicillin I gm TJD, or Azithromycin or
• Haemophilus influenzae • Extended macrolides' Clarithromycin
Low-risk • Chlamydophilapneumoniae With stable co-morbids:
CAP • Mycoplasma pneumoniae (Co-amoxiclav or
• ~-lactam/~-lactamase
Cefuroxime)
• Moraxella catarrhalis inhibitor combination
(BLIC),or 2nd-gen oral +/-
• Enteric Gram(-) bacilli' (Azithromycin or
cephalosporins, +/-
Clarithromycin)
• Extended macrolides'
(Ampicillin-
Sulbactam JV or
• JV non-antipseudomonal
• Streptococcuspneumoniae Cefuroxime IV or
~-lactam (BLIC,
Ceftriaxone JV)
• Haemophilus injluenzae cephalosporin), PLUS
• Extended macrolides' +
• Chlamydophilapneumoniae (Azithromycin or
or respiratory
Clarithromycin or
Moderate- • Mycoplasma pneumoniae fluoro9uinolones PO Levofloxacin or
risk CAP • Moraxella catarrhalis Moxifloxacin)
• Enteric Gram (-) bacilli • If with suspected aspiration & a regimen
with ampicillin-sulbactam (3 g JV 96) and/or
• Legionellapneumophila
Moxifloxacin (400 mg OD) is used, there is no need
• Anaerobes' to add additional anaerobic coverage. If another
combination is used, add Clindamycin (600 mg IV
98) to cover for microaerophilic streptococci.
No risk for P. aeruginosa: (Ceftriaxone IV or
• JV non-antipseudomonal Ertapenem IV)
~-lactam, PLUS +
• JV extended macrolide (Azithromycin JV or
or JV respiratory Levofloxacin IV or
fluoro9uinolone Moxifloxacin IV)
• Streptococcuspneumoniae With risk for (Piperacillin-Tazobactam
P. aeruginosa (Option 1): IV or Cefepime IV
• Haemophilus influenzae
• IV anti-pneumococal or Meropenem IV)
• Chlamydophilapneumoniae anti-pseudomonal ~-lactam +
(BLIC, cephalosporin, Azithromycin IV
• Mycoplasma pneumoniae
carbapenem), PLUS +
High- • Moraxella catarrha/is • IV extended macrolide, PLUS (Gemamicin IV or
riskCAP • Aminoglycoside Amikacin IV)
• Enteric Gram (-)bacilli
With risk for
• Legionel/apneumophi/a
P. aeruginosa (Option 2):
(Piperacillin-Tazobactam
• Anaerobesc • IV anti-pneumococcal
IV or Cefepime IV
• Staphylococcusaureus anti-pseudomonal ~-Iactam
or Meropenem IV)
(BLIC, cephalosporin,
• Pseudomonasaeruginosa carbapenem), PLUS +
(Ciprofloxacin IV or
• JV Ciprofloxacin (400 mg
Levofloxacin IV)
98-12)or high-dose
Levofloxacin (750 mg OD)
If MRSA suspected, add:
• Vancomycin 15mg/kg JV 98-12,or
• Linezolid 600 mg IV 912
• Useextended
macrolides if witha historyof allergyto ~-lactams
b Amongthosewithco-morbid illness
c Amongthosewithriskof aspiration
2S6
B. Common Antibiotics Used in CAP
DRUG CLASS I COMMON EXAMPLES
• Azithromycin 500 mg OD PO or IV
Extended macrolides
, Clarithromycin 500 mg BID PO or IV

Oral ~-lactam/~-lactamase inhibitor


• Amoxicillin-clavulanic acid I g BID PO
combination (BLIC)

Oral second-generation cephalosporin • Cefuroxime axetil 500 mg BID PO

• Ampicillin-sulbactam 1.5g q6 IV
IV non-antipseudomonal ~-lactam (BLIC, • Cefuroxime 1.5g q8 IV
cephalosporin, carbapenem•) • Ceftriaxone 2 g OD IV
• Ertapenem I g OD IV
• Levofloxacin 500-750 mg OD PO or IV''
Respiratory fluoroquinolones
• Moxifloxacin 400 mg OD PO or IV

IV anti-pneumococcal, anti-pseudomonal • Piperacillin-tazobactam 4.5 g q6 IV


~-lactam (BLIC,cephalosporin or • Cefepime 2 g q8-12 IV
carbapenem) • Meropenem I g q8 IV

• Gentamicin 3 mg/kg OD IV
Aminoglycosides
• Amikacin 15mg/kg OD IV
•carbapenems used for high-riskCAP.Reserve its use for risk of potentiallyresistant strains.
**High-doselevofloxacin(750mg) is used for high-riskCAP with risk for Pseudomonas
Sources:PhilippineCPGon Community-Acquired
Pneumonia(CAP),2016
DOHNationalAntibioticGuidelines2017 .

C. Duration of Treatment based on Etiology


ETIOLOGIC ORGANISMS I DURATION OF TREATMENT

• 5-7 days
Most bacterial pneumonias•
• 3-5 days (azalides) for S. pneumoniae

Enteric Gram-negatives, S. aureus,P.aeruginosa


•MSSA-CAP • 7-14 days (up to 21days ifbacteremic)
• MRSA-CAP • 7-21days (up to 28 days ifbacteremic)
• P. aeruginosa • 14-21days (up to 28 days ifbacteremic)
Mycoplasma and Chlamydophila • 10-14days
Legione/la • 14-21days (10 days for azalides)
MSSA:methicillin-sensitiveS. aureus
MRSA:methicillin-resistantS. aureus

*Exceptenteric gram-negative pathogens, S.aureus,and P.aeruginosa


Source:PhilippineCPGon Community-Acquired
Pneumonia(CAP),2016

D. Discharge Criteria: during the 24 hours before discharge, the patient should have:
0 Temperature of 36-37.5•c
0 Pulse rate <IOO bpm
0 RR between 16-24/min

0 SBP >90 mmHg


0 Blood 02 saturation >90%
° Functioning GI tract (allowing use of oral antibiotics)

257
HOSPITAL-ACQUIRED & VENTILATOR-ASSOCIATED PNEUMONIA
I. ETIOPATHOGENESIS (conceptof healthcare-associatedpneumonia (HCAP) has been discarded)
A. Hospital-Acquired Pneumonia (HAP)
0 Episodes of pneumonia that are not associated with mechanical ventilation
0 HAP in non-intubated patients, both inside & outside ICU, is similar to VAP (save for the
higher frequency of non-MOR pathogens & better host immunity in non-intubated patients)
0 Lower frequency of multi-drug resistant (MOR) pathogens allows for monotherapy in HAP
0 The only pathogens that may be more common in the non-V AP population are anaerobes
(higher risk of macroaspiration)
0 More difficult to obtain lower respiratory samples for culture in non-intubated patients

B. Ventilator-Associated Pneumonia (VAP)


0 The greatest difference between V AP and HAP is the return to dependence on expectorated
sputum for a microbiologic diagnosis ofVAP, which is further complicated by frequent
colonization by pathogens in patients with HAP
0 Mechanisms: oropharyngeal colonization, cross-infection from other patients, large-volume
aspiration, microaspiration around endotracheal tube, and altered host defenses
NON-MDRPATHOGENS I MDR PATHOGENS
• Streptococcuspneumoniae • Pseudomonasaeruginosa
• Other Streptococcusspp. • Methicillin-resistant S. aureus
• Haemophilus influenza • Acinetobacter spp.
• Methicillin-sensitive S. aureus • Antibiotic-resistant Enterobacteriaceae
(e.g., ESBL-positive strains, carbapenem-
• Antibiotic-sensitive Enterobacteriaceae
resistant strains)
(e.g., E. coli, K. pneumoniae, Proteus spp., • Legionel/apneumophila
Enterobacterspp., Serratia marcescens) • Burkholderiacepacia
• Aspergillus spp.

II. MANAGEMENT
Once an etiologic diagnosis is made, broad-spectrum empirical therapy can be modified to
specifically address the identified pathogen (consider referral to a specialist)
RISK FOR MDR
PATHOGENS?
I EMPIRIC ANTIBIOTIC THERAPY

Majority can be treated with a single agent


• Piperacillin-tazobactam 4.5 g IV q6
Without risk
• Cefepime z g IV q8
factors
• Levofloxacin 750 mg IV q24
• Meropenem I g IV q8
Treatment with 3 antibiotics (two directed at P. aeruginosa and one at MRSA)
• First agent is a ~-lactam:
° Ceftazidime 2 g IV q8 or Cefepime 2 g IV q8
·1,
0 Piperacillin/tazobactam 4.5 g N q6
0 Meropenem I g N q8
0 Aztreonam 2 g IV q8
With risk • Second agent active against gram-negative pathogens:
factors 0 Gentamicin or tobramycin 7 mg/kg IV q24 or amikacin 20 mg/kg IV q24
° Ciprofloxacin 400 mg IV q8 or Levofloxacin 750 mg IV q24
° Colistin 5 mg/kg IV loading dose, then 2.5 mg x (1.5x CrCI + 30) IV q12
0 Polymyxin B 2.5-3 mg/kg/day IV in 2 divided doses
• Third agent active against MRSA:
0 Linezolid 600 mg IV q12
0 Vancomycin 15 mg/kg (up to I g IV) q12
RiskFactorsfor Multidrug-Resistant (MDR)Pathogens:
• Riskfor MORVAP:priorIVantibioticusewithin90days,septicshockat timeof VAP,ARDSpreceding
VAP,~5 daysof hospitalization
priorto occurrence
of VAP,dialysispriorto VAPonset
• RiskforMORHAP,MRSAVAP/HAP, MORPseudomonas VAP/HAP: priorIVantibioticusewithin90 days
Sources:KalilAC,et al. 2016ATS/IDSA CPG.ClinInfectDis.2016
258
SECTION FIVE
TUBEReULOSIS

ETIOPATHOGENESIS OF TUBERCULOSIS (TB)


Infectious disease caused by Mycobacteriumtuberculosis,
most commonly affecting the lungs
Usually transmitted from person with infectious pulmonary tuberculosis (PTB) to
others by droplet nuclei, which are aerosolized by coughing, sneezing or speaking
The most infectious patients are those with cavitary pulmonary disease & laryngeal TB
The typical TB lesion is an epithelioid granuloma with central caseation necrosis

SCREENING FOR TUBERCULOSIS


Case finding pertains to the identification of presumptive TB, either by clinical manifestations or chest
X-ray, followed by the diagnosis of active TB disease through bacteriological testing or clinical diagnosis.

I. DEFINITION OF TERMS
TERM I DEFINITION
Systematic • Systematic identification of presumptive TB in a predetermined target group
screening • Active case finding: systematic screening implemented outside healch
for active TB facilities (e.g., high risk populations) by bringing screening tests to community
• Screening using any of the four cardinal TB symptoms/signs
(at least 2 weeks duration):
Symptom-
°Cough
based
0 Unexplained fever
screening
0 Unexplained weight loss
0 Night sweats
Screening by • Using chest X-ray to identify presumptive pulmonary tuberculosis (PTB)
chest X-ray • Recommended annually among all health facility consults
6thEdition,2020
ControlProgramManualof Procedures
Source:DOHNationalTuberculosis

II. PRESUMPTIVE TUBERCULOSIS


PRESUMPTIVE I DEFINITION
TB
• Refers to person with either:
Presumptive
0 2:2 weeks of cough, unexplained fever & weight loss, or night
Pulmonary TB
sweats; OR
(PTB)
° Chest X-ray finding suggestive of TB
• Refers to person with signs/symptoms specific to the suspected
extrapulmonary site with or without general constitutional signs/
symptoms (e.g., unexplained fever/weight loss, fatigue, anorexia)
• The following may identify presumptive EPTB:
Presumptive 0 Gib bus deformity (vertebral TB)
Extrapulmonary 0 Non-painful cervical lymphadenopathy (TB lymphadenitis)
TB(EPTB) 0 Neck stiffness and/or drowsiness (TB meningitis)
0 Pleural or pericardia! effusion
0 Distended abdomen with ascites (TB peritonitis)
0 Non-painful enlarged joint (TB arthritis)
0 Signs of tuberculin hypersensitivity (i.e., PPD-positive)
6thEdition,2020
ControlProgramManualof Procedures
Source:DOHNationalTuberculosis

259
Ill. SYSTEMATIC SCREENING FOR PULMONARY TUBERCULOSIS (PTB)

Yes No

Cough
Unexplained fever
Unexplained weight loss
Night sweats

PRESUMPTIVETB
RequestXpert MTB RifTest
(SM/TB LAMP if
Xpert not available)

Yes

Cansultationfor determining
other etiology of disease

• Systematicscreeningin facilitiesshouldbe donefor all clientsvisitingthe facility,regardlessof the


reasonfor consult(thisalgorithmis recommended to screenfor PTBamongadults;?15yearsold)
• Forpeoplelivingwith HIV(PLHIV),screeningby bothsymptomsandchestX-rayshouldbe doneat
the timeof diagnosisof HIV/AIDSandannuallythereafter
• CXRshouldnot be usedas sole reasonfor initiatinganti-TBtreatment
Source:DOHNational Tuberculosis
ControlProgram Manualof Procedures 6thEdition,2020

DIAGNOSIS OF TUBERCULOSIS
I. DEFINITION OF TERMS
TERM I DEFINITION

• A presumptive TB case that is either bacteriologically-confirmed


Active TB Disease
or clinically diagnosed
• Patient from whom a biological specimen (either sputum or non-
Bacteriologically
sputum sample) is positive for TB by smear microscopy (SM),
Confirmed TB
culture, or rapid diagnostic tests (e.g., Xpert MTB/RIF, line probe
(BCTB)
assay for TB, or TB LAMP)

• Patient for which the criterion for bacteriological confirmation


Clinically
is not fulfilled, but diagnosis is made on the basis of clinical
Diagnosed TB
findings, X-ray abnormalities, suggestive histology, and/or other
(CDTB)
biochemistry or imaging tests
Source:DOHNational
Tuberculosis
ControlProgram
Manualof Procedures
6thEdition,2020

260
11.CLASSIFICATION OF TUBERCULOSIS
A Based on Site oflnvolvement
• TB involving the lung parenchyma
Pulmonary
• Patient with both pulmonary and extrapulmonary TB should
Tuberculosis (PTB)
be classified as a case of PTB
• TB involving organs other than lungs (e.g., larynx, pleura,
Extrapulmonary
lymph nodes, abdomen, genitourinary tract, skin, joints,
Tuberculosis (EPTB)
bones, meninges)

B. Base d on Drug- suscepti b i Iity Testing


Drug-Susceptible • For patients with Xpert result: MTB without rifampicin
TB(DS-TB) resistance
• For MOR-TB risk groups (retreatment, contact of DR-TB,
Drug-Resistant TB non-converter of DS-TB regimen) with Xpert result: MTB
(DR-TB) detected with rifampicin resistance
• See subclassificationsin sectionon Managementof DR-TB

• Never had treatment for TB or who has taken anti-TB


New Case medications for <I month (prophylaxis & treatment for LTB!
is not counted as "treatment")

I
Retreatment Case (previouslytreatedwith anti-TBdrugsfor at leastI monthin thepast)
• Previously treated for TB, was declared cured or treatment
Relapse
completed, and is now diagnosed with active TB disease
• Previously treated for TB but failed most recent course
Treatment after based on a positive SM follow-up at five months or later, or
failure a clinically diagnosed TB patient who does not show clinical
improvement anytime during treatment
Treatment after lost • Previously treated for TB but did not complete treatment &
to follow-up (TALF) lost to follow-up for at least 2 months in the most recent course
Previous treatment
• Previously treated for TB but whose outcome in the most
outcome unknown
recent course is unknown
(PTOU)
Patients with • Do not fit any of the categories listed above or previous
unknown previous treatment history is unknown (thisgroupwill beconsidered as
TB treatment history previouslytreatedalso)
Source:DOHNationalTuberculosis
ControlProgramManualof Procedures
6thEdition,2020

III. INITIAL DIAGNOSTICS IN MANAGING TUBERCULOSIS


DIAGNOSTIC REMARKS
• Patient with CXR findings suggestive of PTB is considered "Presumptive
PTB",& should be followed through with bacteriological confirmation
Chest or clinical evaluation
radio graph • Typical findings:
or X-ray 0Apical infiltrates/blebs, cavitations
(CXR) 0Miliary disease pattern (diffuse nodules likened to millet seeds)
0Bronchiectatic changes
0 Pleural effusion

• Baseline serum ALT & creatinine before starting treatment for TB


Chemistry
• Blood uric acid and FBS 9r 75g OGTT (for screening for diabetes)
261
IV. DETECTION OF TUBERCULOSIS
DIAGNOSTIC REMARKS
• Nucleic acid amplification testing for M. tuberculosis & rifampicin
(RIF) resistance
• Now considered the primary diagnostic test for PTB & EPTB
XpertMTB/ • Sputum sample should be at least I mL
RIF Assay • Should be used for TB diagnosis among:
0 Smear-negative adults with CXR findings suggestive of TB
(rapid molecular 0 Presumptive drug-resistant TB
diagnostic tests) 0 Individuals with HIV and manifestations of TB
• EPTB can be confirmed by Xpert MTB/RIF using fluid/tissues (e.g.,
CSF, gastric aspirate, biopsy specimens, lymph node aspirate)
• Pleural fluid should not be used for Xpert MTB/RIF
• Directly visualizing the TB bacilli using either brightfield microscopy
(Ziehl-Neelsen technique) or fluorescence microscopy (FM)
• Smear microscopy or loop mediated isothermal amplification (TB
LAMP, see below) shall be the alternative test ifXpert is not accessible
Smear
• Used to monitor progress of BCTB patients during treatment &
Microscopy
confirm cure upon course completion
(SM) or Direct
• Sample should be atleast 3-5mL (check quality:just saliva is not acceptable)
Sputum Smear
• Two specimens are sent (to be collected within 3 days, at the most):
Microscopy
0 1st sample at time of consultation (spot sample)
(DSSM)
0 2nd sample after at least I hour or early morning specimen the

following day
• At least I s_putum smear-positive is considered bacteriologically-
confirmed TB
Loop Mediated
Isothermal • A lower cost DNA amplification technique
Amplification • Sputum sample should be at least I mL
(TB LAMP)
• Primarily recommended for patients at risk for drug resistance
• Baseline TBC & DST should be done for:
TB Culture 0 All cases of retreatment

and Drug 0 All cases of treatment failure (non-converter after treatment completion)

Susceptibility 0 Household contacts of DR-TB index case

Testing (TBC 0 In patients with HIV

&DST) 0 Xpert result: MTB detected with rifampicin resistance (RR)

• Follow-up TBC is also used to monitor treatment response (culture


conversion) in DR-TB patients
• Tuberculin (purified protein antigen from MTB) is injected
inrradermally to elicit a skin reaction
Tuberculin
• Screening tool for TB infection in children
Skin Test
(TST) • Positive TST: induration of skin with diameter ;,w mm after 48-72hours
• Used to screen for patient's eligibility for receiving treatment for
latent TB infection (TB preventive treatment)
• Blood assay measures interferon gamma levels, which is released by
Interferon- CD4 & CDS T cells in response to MTB antigens that are mixed with
Gamma the patient's blood sample
Release Assays • Unlike TST, does not require a repeat visit from the patient for
(IGRA, e.g., interpretation of the test
Quantiferon) • Used to screen for patient's eligibility for receiving treatment for
latent TB infection (TB preventive treatment)
Sources:DOHNationalTuberculosis
ControlProgramManualof Procedures
6th Edition,2020
WHO,Definitions
andReportingFramework forTuberculosis,
2013
262
V. APPROACH TO DIAGNOSIS OF TB
Once presumptive TB case is identified, bacteriologic confirmation must be conducted by:
• Collecting necessary specimens for testing
• Performing test (Xpert, smear microscopy, or TB LAMP)
• Making a diagnosis based on results
The only contraindication to collecting sputum for diagnosis is massive hemoptysis
Blood-streaked sputum is still a viable specimen for analysis

• T: MTB detected, rifampicin resistance not detected


• RR: MTB detected, rifampicin resistance detected
Xpert MTB/
• Tl: MTB detected, rifampicin resistance indeterminate
RIF
• N: MTB not detected
• I: invalid/no result/error
Smear • Positive: at least one sputum smear is positive for AFB (+n, I+, 2+, 3+)
microscopy (SM) • Negative: both sputum smears are negative for AFB

B. Dia nosis and Clinical Decision


PRESUMPTIVE
TB
Spot sputum specimen (or other body fluid/tissue) for Xpert MTB/Rif Test
DSSM/TB LAMP if Xpert unavailable

No

On clinical
follow-up,
is TB still
Treat as suspected?
OS.TB
Yes No

Yes No
If still symptomatic,
may investigate for
High riskfor MORTB Lowriskfor MORTB Consider work-up for
other medical
conditions
EPTBor referral to
specialist (possibly to
May initiate LTBJ
Recollect/ start anti-TB
See treatment for treatment for
repeat Xpert MTB/Rif treatment)
DR-TB; consider referral contacts of
test and follow the
to specialist bocteriologicolly
second test result.
confirmed TB cases

RR:rifampicinresistance DS-TB:drug-susceptible
tuberculosis
LTBI:laten!TBinfection DR-TB:drug-resistant
tuberculosis
Approachto Patients:
ForMTSwithoutRR,classifyas DS-TB
ForMTSwithRR:
° ForMOR-TBriskgroups(retreatment, contactof DR-TB,non-converter
of DS-TB
regimen):classifyas DR-TB
° Forlowriskfor MOR-TB(i.e.newTB caseswhoare notDR-TBcontacts), recollecta
sputumsample& followthesecondresultfor thetreatmentdecision
• Forindeterminate,
invalid
orerrorresults,
recollecl
a freshsputum
sample,
repeattheXpertMTB/RIF test
Source:DOHNational Tuberculosis
ControlProgramManualof Procedures6thEdition,2020
263
MANAGEMENT OF DRUG-SUSCEPTIBLE TB (OS-TB)
I. TREATMENT REGIMENS FOR DRUG-SUSCEPTIBLE TB (DS-TB)
Standard treatment for DS-TB is given based on results ofXpert MTB/RIF (or if not available,
on history of treatment)
TB regimen is comprised of at least 4 first-line drugs (fixed dose combination should be used)
A patient with negative Xpert result (MTB not detected) but positive SM/TB LAMP is still
considered BCTB and should be treated as DS-TB
EPTB of the CNS, bones & joints extends the standard 6-month DS-TB regimen to 12 months

ELIGIBLE PATIENTS

• PTB or EPTB (except CNS, bones, joints) whether new


or retreatment, with final Xpert result:
0MTB, RIF sensitive
1 2-HRZE 4-HR
0MTB, RIF indeterminate
• New PTB or new EPTB (except CNS, bones, joints), with
positive SM/TB LAMP or clinically diagnosed, and:
0Xpert not done••
0Xpert result is MTB not detected
• EPTB of CNS, bones, joints whether new or
retreatment, with final Xpert result:
0 MTB, RIF sensitive
2 2-HRZE 10-HR
0 MTB, RIF indeterminate
• New EPTB of CNS, bones, joints, with positive SM/TB
LAMP or clinically diagnosed, and:
0 Xpert not done ••

0 Xpert result is MTB not detected

'Intensivephaseis theinitialpartof standardizedregimens, whichusuallyconsistsof ~4 anti-TBdrugs.


Oncesomedrugsarediscontinued, thisis nowreferredto as thecontinuation
phase.
"All effortsshallbeexertedto ensurethatall retreatment casesaretestedwithXpertMTB/RIF.

II. PHARMACOLOGY OF FIRST-LINE TB DRUGS

• Inhibits fatty acid synthase and mycolic acid synthesis


Isoniazid 5 mg/kg • Excellent bactericidal activity against both
(HorlNH) (Max400 mg) intracellular and extracellular actively-dividing MTB
• Bacteriostatic against slowly-dividing organisms

• Binds to and inhibits mycobacterial DNA-dependent


Rifanipicin 10 mg/kg RNA polymerase thereby blocking RNA synthesis
(RorRIF) (Max600 mg) • Has both intracellular and extracellular bactericidal
activity, both in dividing and non-dividing MTB
• Most active anti-mycobacterial agent available
• Exact mechanism is unclear (fatty acid synthetase-1
may be the primary target)
Pyrazinamide 25 mg/kg
(Z) (Max2 g)' • More active against slowly replicating organisms
• Active only in acidic environment (pH<6.o) as are
found within phagocytes or granulomas

15 mg/kg, • Inhibits arabinosyltransferases involvedin cell wall synthesis


Ethambutol
• Bacteriostatic anti-mycobacterial agent
(E) (Max 1.2g)*
• Least potent against MTB
*Frequencyof Zand E mustbe adjustedif individualhas creatinineclearance<30 mUminor is
undergoinghemodialysis(i.e.,giventhreetimesperweek)
264
B St d dR f, Ad It 'th DS TB

BODY
WEIGHT

25-37 kg 2 2
38- 54 kg 3 3
55-70 kg 4 4
>70kg 5 5

C. Managing Drug Side-Effects


SIDE EFFECTS I CULPRIT DRUG I MANAGEMENT

Minor Side Effects (may continueanti-TB drugs & checkdoses) ,-


Anorexia, nausea, • Pyrazinamide, • Give drug with small meals or at
abdominal pain Rifampicin, lsoniazid bedtime
Joint pains (hyperuricemia) • Pyrazinamide • Give aspirin or NSAID
Peripheral neuropathy • Isoniazid • VitaminB61omgODforprevention
Orange/red-colored urine • Rifampicin • Reassure the patient
Drowsiness • Jsoniazid • Reassurance & give drug at bedtime
Flu-like symptoms • Rifampicin • Give anti-pyretics
Major Side Effects .. ,.
Severehypersensitivityrash • Any drug

Jaundice due to hepatitis • Any drufl


(especia y H-R-Z)
Optic neuritis (visual) • Ethambutol • Discontinue taking medications
Decreased urine output • Rifampicin & refer to clinician urgently

Psychosis & convulsion • Isoniazid


Thrombocytopenia, • Rifampicin
anemia, shock:

III. SCHEDULE OF SPUTUM FOLLOW-UP EXAMINATIONSFOR PTB ON DS-TB REGIMEN


Response is monitored through follow-up smear microscopy (SM) & clinical assessment
Xpert MTB/RIF test is not used for follow-up examination to monitor treatment
because current generation PCR-based tests are unable to determine MTB viability &
may test positive even with nonviable or dead bacilli
TYPE I FOLLOW-UP 1* I FOLLOW-UP 2 ** I FOLLOW-UP 3 **
• Only if positiveat end • Only if positive at end of
• End of intensive
New,CD-TB of intensive phase intensive phase
phase (2 months)
• End of 5th month • End of treatment (6th month)
New,BC-TB • End of intensive
• End of 5th month • End of treatment (6th month)
Retreatment phase (2 months)
'If SMis positiveat end of intensivephase (2ndmonth),requestforXpertMTB/RIFand proceedwith
continuationphase whilewaitingforresults.IfRIFresistant,shiftto DR-TBregimen.
"If positiveSMafter5thor 6th month,stoptreatmentand declareas treatmentfailure(referto
Programmatic Managementof DrugResistantTBor PMDTTreatmentCenter)
265
IV. TREATMENT OUTCOMES FORDS-TB
OUTCOME I DEFINITION

• BCTB when treatment started, then became smear or culture (-) on last month
Cured
of treatment AND on at least I previous occasion in continuation phase

• Completes treatment without evidence of failure but no sputum smear(·)


Treatment
results (tests not done or results are unavailable)
completed
• Includes CD-TB patients who completed treatment

• Sputum smear or culture(+) at ~5 months during treatment


• Treatment terminated because of evidence of additional acquired
Treatment resistance (e.g., RIF resistance on Xpert at 2nd month)
failed • EPTB patients (unable to do sputum exam) who do not show clinical
improvement anytime during treatment
• Severe uncontrolled adverse drug reaction

Died • Dies for any reason during the course of treatment

Lost to • Treatment was interrupted for at least two consecutive months


follow-up • Diagnosed with active TB but not started on treatment (i.e., initial LTFU)

• No treatment outcome is assigned


Not
• Includes those transferred to another facility for continuation of
evaluated
treatment but the final outcome was not determined
Source: DOHNationalTuberculosisControlProgramManualof Procedures 6th Edition,2020

LATENT TUBERCULOSIS INFECTION (LTBI)


State of persistent immune response to stimulation by MTB antigens with no evidence
of clinical manifestations of active TB disease
Majority of infected have no signs or symptoms of TB but are at risk for active TB disease
TB Preventive Treatment (TPT, i.e., LTBI treatment or preventive therapy) is offered to
individuals who are at high risk of developing active TB disease in order to reduce that risk

I. UTILITY OF TST OR IGRA PRIOR TO TB PREVENTIVE TREATMENT (TPT)


No "goldstandard"test for diagnosing LTBI, but tuberculin skin test (TST) or interferon-gamma
release assays (IGRA) may be used to determine an individual's eligibility for receiving TPT
Active TB disease should be excluded prior to initiation ofTPT by screening for
manifestations and ordering for a chest X-ray
TST/IGRA
not required • Household contact' of a BCTB index case, AND with TB risk factor/s 6
(automatically • PLHIV (regardless of history of contact)
eligible for TPT)
• Household contact• of a BCTB index case, with no TB risk factor/s'
TST/IGRA • Close contactc::of a BCTB index case
required (eligible • Patients receiving dialysis
for TPT only if • Patients preparing for organ or hematological transplantation
positive) • Patients initiating anti-TNF treatment
• Patients with silicosis
Not eligible for • Contacts of CDTB index case (except household contacts <5 years old)
TPT • Contacts ofMDR-TB or RR-TB index case
PLHIV:people livingwith HIV BCTB:bacteriologically-confirmed
tuberculosis
CDTB:clinicallydiagnosed tuberculosis MOR-TB:multidrug-resistanttuberculosis
RR-TB:rifampicin-resistant
tuberculosis
• Householdcontact:shared the same enclosed livingspace as the index person withTB
6 TB riskfactors: OM,smoking,iinmunosuppression,malnourished,multiplepeople withTB in household

'Close contact:shared an enclosed space (socialgathering,workplaceor facility)for extended periods


duringthe day withthe indexperson withTB during3 months before commencementof currenttreatment
266
II. TREATMENT REGIMENS FOR LTBI
PREVENTIVE
REGIMEN I DOSE
I REMARKS

• Regimen of choice
3HP (isoniazid + • H: 15mg/kg
• Can be initiated even without
rifapentine once • P: 900mg (750mg if $50 kg
baseline LFTs
weekly x 3 months) body weight)
• Contraindicated in pregnant women
4R (rifampicin daily • 10mg/kg (range: 8-12mg/kg) • Preferred if 3HP regimen not
x4months)• • Not to exceed 600mg daily available or contraindicated
6H (isoniazid daily x • 5 mg/kg (range 4-6mg/kg) • Currently available under the
6months)" • Not to exceed 300mg daily program
'For those withriskfactorsfor hepatoxicity,do not give LTBItreatmentifliverfunctiontests cannot be done
at baselineand monthlyduringtreatment(exceptionis PLHIVwithoutother riskfactorsfor hepatotoxicity)

III. MONITORING
Follow up 2 weeks after initiation ofTPT and then at least monthly thereafter
Check for signs or symptoms of TB and adverse reactions to drugs
If diagnosed with active TB disease after appropriate evaluation, stop TB preventive
treatment and shift to treatment for active TB disease
OUTCOME I DEFINITION
Completed • Finished prescribed duration of treatment & remains asymptomatic

Failed • Developed active TB disease anytime while on TPT

Died • Dies for any reason during the course ofTPT

Lost to Follow-up • Interrupted TPT for two consecutive months or more

• Transferred to another health facility with proper referral slip for


continuation ofTPT and whose treatment outcome is not known
Not Evaluated
• Included here are patients whose treatment was discontinued
because they could not tolerate adverse effects ofTPT

MANAGEMENT OF DRUG-RESISTANT TUBERCULOSIS (DR-TB)


MDR-TB and RR-TB should be treated within 7 days of diagnosis
DR-TB patients should be referred to a Programmatic Management of Drug-Resistant
TB (PMDT) clinic/center

Classification Based on Drug-Susceptibility Testing


CLASSIFICATION I DEFINITION
• Positive for MTB with resistance to RIF
BC Rifampicin-Resistant •Maybe resistant to isoniazid (i.e., MDR-TB), resistant to
TB (BC RR-TB) other first-line TB medicines (poly-resistant), or resistant to
second-line TB medicines (e.g., XDR-TB)
BC Multidrug-Resistant
• Positive for MTB with resistance to at least both !NH & RIF
TB (BC MOR-TB)

BC Extensively Drug- • Positive for MTB with resistance to any fluoroquinolone


Resistant TB (BC and to at least one second-line injectable drug (e.g.,
XDR-TB) amikacin, streptomycin) in addition to multidrug resistance
..
Source: DOHNationalTuberculosisControlProgramManualof Procedures6th Ed1t1on,
2020

267
RESPIRATORY FAILURE
Syndrome in which the respiratory system fails in one or both of its gas exchange
functions: oxygenation (hypoxemic) & carbon dioxide (CO,) elimination (hypercapnic)

I. TYPE I (HYPOXEMIC) RESPIRATORY FAILURE


• Most common form which can be associated with most acute diseases of the lung
Characterized by an arterial 02 tension PaO2 <60 mmHg with a low or normal PaCO2
• Usually from alveolar flooding with intrapulmonary shunting
Alveolar flooding may be a consequence of:
0 Pulmonary edema (cardiogenic or non-cardiogenic)
0 Pneumonia
0 Pulmonary (alveolar) hemorrhage
0 ARDS

II. TYPE II (HYPERCAPNIC/HYPERCARBIC) RESPIRATORY FAILURE


Type which generally has a PaCO2 >45-50mmHg (pH depends on the level of
bicarbonate, which in turn depends on the duration of the hypercapnia)
0 Acute hypercapnic failure: develops over minutes/hours with a pH <7-3
° Chronic hypercapnic failure: develops over days or more with a normal/only
slightly decreased pH because of renal compensation (i.e., increase in HCO3 levels)
• Result of alveolar hypoventilation and leads to the inability to eliminate CO, effectively

• Drug overdose
Diminished
• Brainstem injury
breathing
• Sleep-disordered breathing
effort of CNS
• Hypothyroidism
• Myasthenia gravis
, Impaired
• Guillain-Barre syndrome
Reduced neuromuscular
• Amyotrophic lateral sclerosis
strength of transmission
• Phrenic nerve injury
neuromuscular
function , Myopathy
• Respiratory muscle
, Electrolyte derangements
weakness
• Fatigue
• Increased resistive • Bronchospasm
loads • COPD, asthma, suffocation
• Alveolar edema
• Loads due to reduced • Atelectasis
Increased lung compliance • Intrinsic positive end-expiratory pressure
overall (auto-PEEP)
load on the
respiratory • Pneumothorax
• Loads due to reduced
system • Pleural effusion
chest wall compliance
• Abdominal distention
• Loads due to increased • Sepsis
minute ventilation • Pulmonary embolism with increased dead
requirements space
Source:JamesonJL,et al. Harrison's
Principles
of InternalMedicine20thedition,2018
268
III. TYPE III RESPIRATORY FAILURE
This form of respiratory failure occurs as a result of lung atelectasis
• Also called perioperative respiratory failure as this occurs commonly during the
perioperative period (usually from pain)

IV. TYPE IV RESPIRATORY FAILURE


Results from hypoperfusion of respiratory muscles of patients in shock (respiratory
muscles normally consume <5%of the total cardiac output and 02 delivery)
Patients in shock often experience respiratory distress due to pulmonary edema (e.g.,
patients in cardiogenic shock), lactic acidosis, and anemia
Up to 40% of the cardiac output may be distributed to the respiratory muscles
Intubation and mechanical ventilation can allow redistribution of the cardiac output
away from the respiratory muscles and back to vital organs while the shock is treated

ACUTE RESPIRATORY DISTRESS SYNDROME (ARDS)


I. ETIOPATHOGENESIS
ARDS is caused by diffuse lung injury from many underlying medical & surgical disorders
• Most cases are caused by pneumonia and sepsis

• Pneumonia • Sepsis
• Aspiration of gastric contents • Severe trauma (frfactures,flailchest, head trauma, bums)
1 1
• Pulmonary contusion , Mu tip e trans usions
• Near-drowning • Drug overdose
• Toxic inhalation injury • Pancreatitis
• Post-cardiopulmonary bypass
Source:JamesonJL,et al. Harrison's
Principles
of InternalMedicine20thedition,2018

B. Three Phases in the Natural History of ARDS


PHASE I REMARKS
• Alveolar capillary endothelial cells and type-I pneumocytes (alveolar
epithelial cells) are injured, leading to the loss of the normally tight
Exudative alveolar barrier to fluid and macromolecules
phase • Edema fluid rich in protein accumulates in the interstitial & alveolar
(day 1-7) spaces
• X-ray reveals alveolar and interstitial opacities involving at least three-
quarters of the lung fields
• Most recover rapidly & liberated from mechanical ventilation during
Proliferative this phase ·
phase • Histologically, the first signs of resolution are often evident in this phase
(day7-21) • Lymphocyte-predominant pulmonary infiltrate
• ~3 weeks after the initial pulmonary injury, most patients recover
• While many patients with ARDS recover lung function 3-4 weeks after
Fibrotic the initial pulmonary injury, some will enter a fibrotic phase
phase • Biopsy evidence for pulmonary fibrosis in any phase of ARDS is
associated with increased mortality
Source:JamesonJL,et al. Harrison'sPrinciples
of InternalMedicine20thedition,2018

269
II. CLINICALMANIFESTATIONS
ARDS is a clinical syndrome of:
Severe dyspnea of rapid onset
0

Hypoxemia
0

Diffuse pulmonary infiltrates


0

III. DIAGNOSIS OF ARDS (Berlin Definition)


OXYGENATION IN ARDS
(Pao,/ FiO,) I OTHER CRITERIA FOR DIAGNOSIS

• Onset is acute, within I week of known clinical


Mild >200 to 300 mmHg•
insult or new or worsening symptoms
• Chest imaging (CXR or CT): bilateral opacities not
Moderate >IOO to 200 mmHg•
explained by effusions, lung collapse, or nodules
• Respiratory failure not fully explained by CHF
Severe SJ00mmHg• or fluid overload (i.e., absence of left atrial
hypertension) ..
*WithPEEP ~5 cm H,O
**Needsobjective assessment to exclude hydrostaticedema if no risk factor present (e.g., 2D echo,
PCWP S18 mmHgor no clinicalevidence of increased left atrial pressure)
Source:ARDSDefinition
Task Force. BerlinDefinition.JAMA.2012.

IV. MANAGEMENT
Aim is to protect the lung
Patients with ARDS frequently become fatigued from increased work of breathing and
progressive hypoxemia, requiring mechanical ventilation for support
Done by decreasing tidal volume & giving adequate positive end-expiratory pressure

A. Mechanical Ventilation
° Frequently needed due to fatigue from increased work of breathing and progressive
hypoxemia
0However, mechanical ventilation can aggravate lung injury (barotrauma/volutrauma)
0Lower tidal volume (VT) would protect against ventilator-induced lung injury and
improve clinical outcomes
• Low VT: 6 mL/kg predicted body weight (preferred in ARDS)
• Conventional VT: 12 mL/kg (8-10 mL/kg in Asians)
0This improvement in survival represents the most substantial benefit in ARDS
mortality demonstrated for any therapeutic intervention in ARDS to date

B. Summary of Treatment Strategies for ARDS


0 Low tidal volume (best level of evidence)
0 Minimize left atrial filling pressures
0 High PEEP
0 Prone position
0 Recruitment maneuvers that transiently increase PEEP to high levels
0 Extracorporeal membrane oxygenation (ECMO)
0 Neuromuscular blockade

Source:Jameson JL, et al. Harrison'sPrinciplesof InternalMedicine20th edition,2018

270
SECTION SEVEN
OTHER DISORDERS IN PULMONOLOGY
DISORDERS INVOLVING THE PLEURA
I. PLEURAL EFFUSION
The pleural cavity is a potential space between the lungs & chest wall and normally
contains only a very thin layer of fluid that serves as a lubricant between the membranes
Pleural effusion is defined as an excess quantity of fluid in the pleural cavity/space, &
classified as transudative or exudative based on Light's criteria (seeChapter 2)
TYPE I ETlOPATHOGENESlS I EXAMPLES
• Occurs when systemic factors that • Heart failure
influence formation & absorption • Cirrhosis (hepatic hydrothorax)
Transudative
of pleural fluid are altered • Nephrotic syndrome
pleural
• 25% of transudative effusions may • SVC obstruction
effusion
be misclassified as exudative by • Myxedema
Light's criteria • Urinothorax
• Parapneumonic effusion {bacterial
• Occurs when local factors that
pneumonia, lung abscess,
influence formation & absorption
Exudative bronchiectasis, empyema)
of pleural fluid are altered
pleural • Malignancy
• Additional work-up is needed to
effusion • Pulmonary embolism
determine the cause of the local
• Tuberculous effusion
disease process
• Hemothorax

II. PNEUMOTHORAX (PTX)


Presence of gas in the pleural space
Usually presents as a sudden onset of dyspnea and pleuritic pain, depending on etiology
Management depends on symptoms, size, and etiology of PTX
TYPE I ETlOPATHOGENESlS I MANAGEMENT
• Occurs in the absence of underlying • Initial treatment: simple
Primary lung disease aspiration
Spontaneous • Usually due to rupture of apical • !flung does not expand or PTX
PTX pleural blebs recurs, thoracoscopic stapling of
• Occurs almost exclusively in smokers blebs and pleural abrasion
• Tube thoracostomy or
• Occurs in those with underlying lung
thoracoscopy or thoracotomy with
Secondary disease
bleb stapling and pleural abrasion
PTX • Most due to COPD (but have been
• If patient refuses surgery,
reported in all lung diseases)
pleurodesis is an option
• Tube thoracostomy unless very
• Penetrating or non-penetrating chest
small & can be managed with
injuries
Traumatic supplemental oxygen or aspiration
• Iatrogenic PTX becoming increasingly
PTX • For hemopneumothorax: one
more common (needle biopsies,
chest tube to evacuate air,
thoracentesis, central line insertion)
another tube to drain blood
• Usually occurs during mechanical • Medical emergency
ventilation or resuscitative efforts • Large-bore needle should be
• Pressure in the pleural space is inserted into the pleural space
TensionPTX
positive throughout the respiratory through the 2nd anterior JCS
cycle decreased venous return to & should be left in place until a
the reduced cardiac output thoracostomy tube can be inserted
..
Sources forthe tables: Jameson JL,et al. Harrison'sPnnc1plesof InternalMed1c1ne
20th ed1t1on,
2018
271
PULMONARY NODULES
I. ETIOPATHOGENESIS
• Asymptomatic pulmonary nodules that are incidentally discovered on chest CT
imaging is a common clinical dilemma
• A lung nodule is a well-defined rounded opacity measuring ,,;3cm in diameter (lesions
>3 cm are termed as pulmonary/lung masses)
Nodules may be solitary or multiple

AT ypeso fP u 1monary Ndo u 1es


• Most common type
Solid nodules
• Characterized by homogenous soft tissue attenuation
• Nonuniform in appearance with hazy inGrease in local
Ground-glass
attenuation oflung parenchyma
nodules
• Type that is least suspicious for malignancy
Part-solid nodules • Comprising both solid and ground-glass attenuation properties

B. Differential Diagnosis of Pulmonary Nodules


NEOPLASMS

• Hamartoma
• Bronchogenic lung cancer
• Chondroma
• Lymphoma
• Lipoma
• Carcinoid
• Respiratory papillomatosis
• Sarcoma
• Pulmonary benign metastasizing
• Lung metastases
leiomyoma

INFECTIONS I IMMUNE-MEDIATED I CONGENITAL


• Mycobacteria • Rheumatoid arthritis
• Arteriovenous
• Fungi • Granulomatosis with
malformation
• Round pneumonia polyangiitis
• Bronchogenic cyst
• Lung abscess • Nodular sarcoidosis
• Pulmonary sequestration
• Septic emboli • Organizing pneumonia
• Pulmonary venous varix
• Nocardia spp. • Lymphoid granulomatosis
• Bronchial atresia with
• Hydatid cyst • Necrotizing sarcoid
bronchocele
• Q fever granulomatosis
Source:LoverdosK, et al.AnnThoracMed.2019

11.INITIAL ASSESSMENT OF PULMONARYNODULES


• Current or past history of tobacco smoking: major risk factor for lung cancer
Assess the • Age: older age correlates with an increased probability of malignancy
pretest
• Occupational exposure to carcinogenic agents (e.g., asbestos, silica, soot,
probability
of beryllium, chromium, arsenic, nickel, cadmium, radon, diesel fumes)
malignancy • History of previous lung cancer, family history of lung cancer
• Co-morbid chronic lung disease (e.g.,COPD, idiopathic pulmonary fibrosis)
• Nodule size: lesions >3 cm should be considered indicative of
malignancy until histologically proven otherwise
Assess
• Nodule growth rate: probability of malignancy significantlyincreases in nodules
imaging
with lower/shorter volume doubling time or VDT (i.e., faster growth rate)
features of
the nodule • Presence of spiculations: increased risk for lung cancer
• Location: majority oflung cancers occur in the upper lobes
• Predictors ofbenign etiology:perifissural nodules, calcification,fat attenuation
272
111.MANAGEMENT OF PULMONARY NODULES
Options for the management of pulmonary nodules include:
0 No further action
° CT surveillance at intervals dictated by nodule size and patient's clinical risk
0 PET/CT
° CT-guided needle biopsy (most appropriate for peripheral nodules)
0 Surgical excision (]obectomy or rarely sublobar excision)
0 Stereotactic body radiation therapy or radiation therapy ablation if with high surgical risk
° Combinations of the above
Decision is best arrived at with the help of a multidisciplinary team, considering nodule size,
clinical risk of malignancy, patient preferences, and overall health status & co-morbidities

Fleischner Society Guidelines for Approach to Pulmonary Nodules

TYPE

• Solid nodule: no
surveillance or optional • Subsolid nodules:
CT at 12months depending CTat3-6
Small <6 <100 on risk' months and then
• Subsolid nodule: more optionally at 2
extensive follow-up at 2 years and 4 years
years and 4 years
Intermediate • CT at 6-12months and
6-8 100-250 • CT at 3-6 months
Solid then 18-24months
and then at 18-24
• CT at 3 months or PET/ months (optional
Larger Solid >8 >250 CT scan, needle biopsy, or for low risk)
surgical excision
• Pure ground-glass nodule:
CT at 6-12months, then
every 2 years for 5 years
• CT at 3-6 months
• Part-solid nodule, solid
• Further
component stable & <6
management
Larger mm: CT at 3-6 months,
>6 >100 based on the
Subsolid then annually for 5 years
most suspicious
• Part-solid nodule, solid
nodule/s
component ~6 mm or
growing: proceed to PET/
CT scan, needle biopsy, or
surgical excision
•Largernodulesizes/volumesrequiremorefrequentCTmonitoring or moreaggressivework-up.Nodules
withconstantsize overthe span of 2 years are generallybenign,permittingdiscontinuation
of CT
surveillance& dischargeof the patientfromthe clinic.Forgrowingnodules,proceedwithfurtherwork-up
(PET-CTscan or histologicdiagnosis).
binpatientswithmultiplenodules,the largestnoduleis not necessarilythe malignantone. Work-upshould
be dictatedby the mostsuspiciousnodulebased on size, appearance,and tissueattenuation
clow riskformalignancy(<5%):youngage, nonsmoker,small,smoothnodule& non-upperlobelocation
Highrisk(> 5%):any of the oppositefeaturesare present
Sources:MacMahonH,et al. Radiology.2017,LoverdosK,et al. AnnThoracMed.2019.

273
REFERENCES (for Chapters 11 & 12)
'· trt1'~~£;;7~ 2~~•~~ 5;,7 5
3 2~~ ~twell E.Fan E.Camporota SlutskyAS.
2. Ambrosino N, VaghegginiG. Noninvasive positive pressure ventiJation in the acute care setting: where are we? Eur Respir 12oo8;31:874
3. Busse V\'W, Cloutier M, DombrO\vskiM, Nelson HS, Reed M, Schatz M, etal. National Astfima Education and Prevention Program.
Working Group Repon on Managing Asthma during Pregnancy:Recommendations for PharmacologicTream1ent 2004-Available
at: hnps://W\\'\v.nhmi.nih.gov/filesfdocs/resources/lunglastpreg_full.pdf
4- Bojar RM.Manual ofPerioperative Care in Adult Cardiac Surgery.6tri Edition. Wiley-Blackwell,2021.
5. Broaddus VC,Ernst JD, KingTE Jr,et a1.(editors).Murray & Nadel'sTextbook of Respiratory Medicine.7thEdition. Elsevier,2021.
6. Buensa.lidoJAL Malundo AFG, Aherrera JAM, et al Clinical Practice Guidelines for Sepsis and Septic Shock in Adults in the
Philippines 2020.Availablefrom:w\.vw:psmid.org/,V}>'-COntent/uploads/2020/031'2020-CPG-for-Sepsis-in-Adults-Full-Manuscriptpd(
7. Busse L, Davison DL, Junker C, Chawla LS. Hemcxlynamicmonitoring in the criticalcare environment Adv Chronic Kidney Dis. 20!3
8. Chetana Shanmukhappa S, Lokesh\...-aran S. Venous Q:,.,)'genSaturation. fUpdated 2020 Nov 1].In: StatPearls (Internet].Treasure
Island (FL):StatPearls Publishing;2021Jan. Availablefrom:ww\.v.ncbi.nlm.nih.gov/books/NBK56439;/
9. Depanment of Health. National Antibiotic Guidelines 2017.Philippines:Department of Health, 2017.
to. Depanment of Health. National Tuberculosis Control Progta!O Manual of Procedures.6th Edition. Philippines:DOH, 2020.
n. DeMers D, Wachs D. Ph)>iology,Mean Anerial Pressure. (Updated 2021 Apr 21).In: StatPearls (Internet). Treasure Island (FL),
12. Zt\i:iw~
l ~&~~~itG~ric?.
COVJD-19.Am Respir Crit Care Med. 2020Od 1;202(7):1039-1042-
1
~~,~-~~~~F{:,";.:B~ :1~
2 in CriticallynJ P-atientswith Severe

f~~e~~es-~ 0~~~~~agi=~t~is~;~1n:~;idel~nc:nm1:!1n~;~~~t 2 1~a~;~~


it~;~
shock 2021.IntensiveCare Med. 2021Nov-,47(11):1181-1247.
15• ~:f~:~o/s~~
Ji;;!;!~~~~ ~~rx::1rs~~:i~~ 2 Ratio oflli1,gen Saturation Index to Guide
16. Fletcher CM. Standardised questionnaire on respiratory symptoms: a statement prepared and approved bythe MRC Committee on
the Aetiolow of Chronic Bronchitis(MRCbreathlessness score).BMJ.1C)6o;2:1662
:~.g:~~
l~~~::: ~~ic(g=~ebfu~ro1~= ~OTo).~~b!tS~~:~
~th!Di~~~~~:~:,:~;;,~~t~~~~
Chronic Obstructive Pulmon~ Disease, 2021.Available&om:\.'/\.V\.\'.poldcopd.org.
2

19. Grunfeld A, FitzgeraldJ. Discharge considerations for adult aslhmanc patients treated in ED.Can Respir J.1996;3:322
20. Irwin RS, Baumann MH, Bolser DC, et al. Diagnosis and management of cough e.xecutivesummary: ACCP evidence-based clinical
r.,ractice&J:idelines.Chest 2006;129.
21. ¼rkMc61;;~Jiu~~1t 0n~~~; Fauci AS, Hauser SL. LoscalzoJ. Harrison's Principles of Internal Medicine. 20th Edition. New
22. Jentzer JC, Coons JC, Link CB,Schmidhofer M. Pharmacotherapy update on the use of vasopressorsand inotropes in the intensive
care unit JCardiovasc Pharmacol Tuer. 20J5May;20(3):249-6o.
23- Jon~ PW',H~i_ng G, Berry P,Wiklund I, Chen W-H, Leidy K Development and first validation ofthe COPD assessment tesLEur
Resp1rJ.2009,34.648-654-
24- KalilAC,Meterskv ML, Klompas M, et al Management of adults with hosp11al-acqmredand ventilator-associatedpneumoma: 2016
Clinical Pracace Cu1delmesby the Infect1ousDLSeasesSOC1ety of Amenca II Amencan Thoraac Society.Chn InfectDis. 2016
2; ~~y.al'f~~~1~3c,;~;~r;;1~acqu1red pneumorua se\'enty on presemanon to hospital· an
26 Loverdos K, Foaad1sA, Kontogianru C, Ihopoulou M, Gaga M. Lung nodules: A comprehensrve review on current approach and
man~emenL Ann Thorac Med. 2019Oct-Dec;14(4):226-238.
r,. LlJ, Fmk JB,Ehrmann S. High-flownasal cannula forCOVJD-19patients: low risk ofbio-aero.soldispersion. Eur Respir J.2020May
28. LieschingT, KwokH, Hill N. Acute lplications of NoninvasivePositivePressureVentilation.Chest. 2003~699-713-
29. MacIntyre NR, Cook DJ, Ely EW r, et al. Evidence-Based Guidelines for Weaning and Discontinuing Ventilatory Support a
CollecaveTask Force faciUtatedbvthe American College of Ches! Physicians;the American Association for Respiratory Care; and
the American College of CriticalCareMedicine.Chest 2001;120:375S-396S.
30. Malbrain MLNG,Van Regenrnortel N, Saugel B, et al. Principlesofl1uid management and stewardship in septic shock: it is rime to
consider the four D's and tfle four phases offluid therapy.Ann lmensive Care. 2018May 22;8(1):66.
31. Mellema MS. Ventilatorwaveforms.Top Companion Anim Med. 2013Aug,28(J):112-23-
32. McDonagh TA. Metta M, Adamo M, et al. 2021ESC Guidelines for the diagnosis and treatment of acute and chronic hean failure..
Eur Hean J.2021Sep 21:42(36),3599-3726.
33- MacMahon H, Naidich DP,Goo JM, Lee KS,Leung AN,MayoJR,et al. Guidelines for management of incidental pulmonary nodules
detected on CT images:from the Fleischner Society2017.Radiology201-,;z84;228-43-
34 Metlay JP, Waterer GW, Long AC, et al. Diagnosis and Treannent of Adults with Community-acquired Pneumonia An Official
Clinical PracticeGuideline of American Thoracic Society& Infectious DiseasesSocietyof Amenca. Am J Respir Crit Care Med. 2m9
35. Moore WC, Meyer.;DA,Wenzel SE, et al. Identificationof asthma phenotypes using duster analysis in the Se\'ere Asthma Research
Pro~. Am J RespirCrit Care Med. 2010;181:315-23-
36. National Asthma Education and Prevention Program, Third Expen Panel on the Diagnosis and Management of Asthma. Ex_pen
Panel Report 3:Guidelines for the Diagnosis and Man~cment or Asthma Bethesda (MD):National Heart, Lung,and Blood Institute
(US);2007Aug.Availablefrom, hnps1/ww,v.ncbi.nlm.mh.gov/books/NBK7232i
r,. Neligan PJ.Pos1o~rative noninvasrveventilation.Anesthesiol Clin. 2012Sep;3CX3)=495-511.
38. Nisl-i1muraM. High-flownasal cannula oxygen therapy in adults. J IntensiveCare. 2015Mar 31;3(1):15-
39. O.vens W. The VentilatorBook.2nd Edition. Fim Draught Press, 2018.
40. PanchalAR, Bartos JA,Cabanas JG,Donnino MW,Drennan JR, Hirsch KG,et al. Part3' Adult Basicand Advanced LifeSupport: 2020
American Hean Association Guidelines for Cardiopulmonary Resuscitation and Emergency CardiovascularCare. Circulation.2020
41. Pham T, Brochard Lj, Slutsky AS.Mechanical Ventilation,State of the Art. MayoClin Proc.2017Sep;92(9),1382-t.100.
42. Pinsky MR.Teboul J,Vincent J.Hemodynamic Monitoring. Springer, 2019.
43• :i~~: 01~;n for the rapid and reproducible annotation of ventilator \.vaveformdata
1,
44 Roca 0, Carah Messika J,Sampe:r M, Sztrymf B, Hem.indez G, et aL An index:combining respiratory rate and o>..-ygenation to
predid outcome of nasal high-flowcherapy.Am J RespirCrit Care Med. 2019Jun 1;1~11):1368-tr,6.
45. RochwergB, Einav S, Chaudhuri D, Mancebo J,Mauri T.HelvizY,et al. The role for · h flow nasal cannula as a respiratory support
strategy in adults: a clinical practiceguideline. IntensiveCare Med. 2020Dec-A6(12):222 22-,,.
46. Singer M, Deut.schrnan CS, Seymour CW, et al. The Third International Consensus Definitions for Sepsis and Septic Shock
(Sepsis-3).JAJ'v!A. 2016Feb 23;315(8),8o1-10.
47. Tansey EA.Montgomery LEA, Quinn JG, Roe SM, Johnson CD. Understanding basic vein physiologyand venous blood pressure
through simple pliysicalassessments. Adv PhysiolEduc. 2019Sep 1:43(3)'423-429.
48. Theodore AC."Measures of oxygenationand mechanisms ofhypoxemia" UpToDate,4 Mar 2020.
49. Tobin, MJ. Principlesand practJceof mechanical ventilation.3rd Edition. McGraw Hill,2013-
50. The Task Force on Community ~Ulred Pneumonia The Philippine Clirucal Pracace Gmdelines on the D1agnos15,Empmc
Management, and Prevent.Lonof Commumty-a~u1red Pneumoma (CAP) m lmmunocompetent Adults: 2m6 Update Jome
Statement of the Phihppme Society for M1crob1ologyand lnfecnous Diseases, Phihppme College of Chest Physicians,Philippine
Academy of Family Physicians,Philippine Collegeof Radiology,2016.
51. Van D1epen S, Katz JN, Albert NM, Henry TD, Jacobs AK, Kapur NK, et al Contemporary Management ofCan:!iogenic Shock, A
ScientificStatement From the Amencan Hean Associanon.Cuculanon. 2017Oct 1;,136(16).e232-e268.
52. Vincent JI., Moreno R, Takala J, et al The SOFA (Sepsis-relatedO<ga!1Failure Assessment) score to describe 0'll"" dysfunction/
failure. On behalf of the Working Group on Sepsis-RelatedProblems of the European Societyof IntensiveCare Medicine. Intensive
Care Med. 1996Jul;22(7l:707-10.

~~~i!!o/r,s~~d~;;,
~
53- World Health Of.Eization. Treaonent ofTuberculosis Guidelines,4th Edition. Geneva, S,vitzerland:WHO, 2010.
54 1~ann DI., TomaselLiGF. Braunwald's Hean Disease, A Textbook of Can:!iovascularMedicine. ttth

274
CRITICAL
CARE
MEDIC
QJ SHOCK
1. Hemodynamics and Shock
2. Sepsis & Septic Shock
3. Cardiogenic Shock
4. Selecting Vasopressors & lnotropes

0 RESPIRATORY SUPPORT & MECHANICAL VENTILATION


1. Oxygen Delivery
2. Principles of Ventilation
3. High-Flow Nasal Cannula (HFNC)
4. Non-invasive Positive Pressure Ventilation
5. Invasive Mechanical Ventilation
6. Basic Monitoring and Troubleshooting
7. Spontaneous Breathing Trial & Weaning

0 ADVANCED CARDIAC LIFE SUPPORT


1. Adult Cardiac Arrest Algorithm
2. Adult Bradycardia Algorithm
3. Adult Tachycardia with a Pulse Algorithm
4. Advanced Therapeutic Modalities
SHOCK
Refers to an imbalance between cellular 0, supply & demand, resulting in organ dysfunction
Clinical indicators include reduced mean arterial pressure (MAP), tachycardia,
tachypnea, cool skin and extremities, acute altered mental status, and oliguria
Most common presentation is hypotension (but not always present), generally defined as:
0 Systolic blood pressure (SBP) <90 mmHg, AND
0 Mean arterial pressure (MAP) <65 mmHg
TYPE OF
SHOCK I MECHANISM*

! CO (& oxygen
I PREDISPOSING
CONDITIONS
• Hemorrhage
I PRINCIPLES OF
TREATMENT

• Fluid/volume resuscitation
Hypovolemic delivery) via a • GI loss (e.g.,diarrhea)
• Blood transfusion if needed
reduction in preload • Burns
• Myocardial infarction
• Reperfusion/revascularization
(most common)
of infarct-related artery for Ml
! CO from intrinsic • Cardiomyopathy
• Vasopressors and inotropes
Cardiogenic cardiac pathology (e.g., • Myocarditis
• Mechanical circulatory support
left ventricular failure) • Critical valvular heart
• Valve replacement
disease
• ACLSalgorithmsfor arrhythmias
• Arrhythmias
! CO from extracardiac • Tension pneumothorax • Needling of pneumothorax
processes that impair • Pulmonaryembolism • Fibrinolysis or embolectomy
Obstructive
blood flow/venous • Cardiac tamponade • Pericardiocentesis
return • Aortic dissection • Surgical repair of dissection
Distribu!ive Shock ..
• Chronic diseases • Fluid Resuscitation,
! SVRfrom
• Immunosuppression Optimization, Stabilization and
dysregulated host
Septic • Advanced age Evacuation (ROSE)
response from
• Recent ICU admission • Vasopressors
infection
or hospitalization • Respiratory support
• Epinephrine
• Allergens (e.g.,food,
! SVR from IgE and • Fluids & vasopressors
medications, insect
Anaphylactic histamine-mediated • Steroids & antihistamines
bites, or contrast
allergic reaction • Beta-agonists & 02 support for
media)
respiratory symptoms
! SVRfrom disruption • Brain injury/surgery
• Fluid resuscitation &
Neurogenic of autonomic pathways • Spinal cord injury/
vasopressors
regulating vascular tone surgery
• Chronic steroid use
! SVR from cortisol
Hypoadrenal • Adrenal hemorrhage, • Fluid resuscitation &
deficiency and
(adrenal metaStasis, or infection vasopressors
hypovolemia from
insufficiency) • Autoimmune • Steroid replacement
aldosterone deficiency
adrenalitis
CO: cardiac output
SVR:systemic vascular resistance
'Clinicalevidence of diminishedCO includenarrowpulse pressure and cool extremitieswithdelayed capillary
refill.Signs of increased CO includewidened pulse pressure, warm extremitieswithboundingpulses, and
rapid capillaryrefill.Ifa hypotensivepatient has signs of increased CO, it can be inferredthat the reduced
bloodpressure is fromdecreased SVR.
"L SVR is the commonprimarydisturbanceof distributiveshock, witha compensatoryincrease in cardiac
output (other examples includepancreatitisand severe burns)
Sources:MalbrarnMLNG,et al.Annalsof IntensiveCare.2018
JamesonJL, et al. Harrison'sPrinciplesof InternalMedicine20thedition,2018
277
HEMODYNAMICS AND SHOCK
I. IMPORTANT CONCEPTS IN HEMODYNAMICS
Determinants of Oxygen Delivery (DO2)
Oxygen delivery is the primary basis for use ofhemodynamic suppon
Its determinants are:
° Cardiac output (CO)
0 Arterial oxygen content (CaO2)
To maintain hemodynamic stability, there should be adequate CO and CaO2

Arterial Oxygen Content (CaO2)


Composed of oxygen carried by hemoglobin and oxygen dissolved in blood
Hypoxemia and anemia must be corrected to ensure adequate oxygen delivery and
maintain hemodynamic stability
DO2 =CO x CaO2 CaO2 =(Hgb x 1.34 x SaO2) + (PaO2 x 0.03)
• DO2: oxygendelivery • Hgb:hemoglobin level(g/l)
• CO:cardiacoutput • PaO,:partialpressureof oxygen(N:~80 mmHg)
• CaO1: arterialoxygencontent • Sao,:arterial0 1 saturationexpressed
asdecimal(N:>0.95or95%)

Stroke Volume (SV)


Volume of blood pumped with each heartbeat
If the problem in CO is the SV,we need to determine which of its determinants are impaired
Determinants of stroke volume:
0 Preload (volume): myocardial fiber length before contraction (EDV)
° Contractility (ventricle):ability of ventricle to contract (independent of preload & afterload)
Afterload (resistance to outflow): force against which the ventricle must contract
(i.e., systemic vascular resistance)

Cardiac Output (CO) & Cardiac Index (CI)


CO is the volume of blood being pumped by the hean per minute
CI is a marker of cardiac function in relation to BSA, thus relating hean performance to
the size of the individual

SV=EDV-ESV • SV:strokevolume(N: 55-100ml; -70 ml for a 70-kgmale)


• EDV:end-diastolicvolume(N: 65-240ml)
• ESV:end-systolicvolume(N: 16-143ml)
CO=HRxSV
• CO:cardiacoutput(N: 4-8 Umin)
co • Cl: cardiacindex(N: 2.6-4.2Umin/m1)
CI= ---
BSA


HR:heartrate (beatsper minute)
BSA:bodysurfacearea(m')

Blood Pressure (BP)


• Simply defined as the force of circulating blood on the walls of the systemic anerial system
In a patient with hemodynamic instability, we must identify which determinant (CO or
SVR) is the primary problem
• BP:bloodpressure(mmHg)
BP=COxSVR • CO:cardiacoutput(Umin)
• SVR:systemicvascularresistance(dynes•s/cm·
5)

Systemic Vascular Resistance (SVR)


Resistance to blood flow through peripheral vascular system
Computed as pressure drop across the systemic vascular bed (MAP - CVP) divided by
the cardiac output (CO)
• SVR:systemicvascularresistance(N:700-1600dynes•s/cm·
5)
MAP-CVP
SVR= • MAP:meanarterialpressure(mmHg)
co • CVP:centralvenouspressure(mmHg)
278
Mean Arterial Pressure (MAP)
Defined as the average arterial pressure during a single cardiac cycle
The target MAP is computed to maintain organ perfusion
MAP targets depend on certain conditions (e.g., higher target in acute ischemic stroke)
• MAP:mean arterialpressure (mmHg)
• SBP:systolicbloodpressure (mmHg)
MAP= 2 (DBP) + SBP
• DBP:diastolicbloodpressure (mmHg)
3 • NormalMAP=70-110mmHg(>60mmHgto sustain perfusion)

Pulse Pressure (PP)


Represents the force that the heart generates each time it contracts
Gives an indirect clue of the hemodvnamic status of a patient
• PP: pulse pressure (N:30-40 mmHg)
• NarrowPP: if PP <25%of the systolicvalue (e.g., lowstroke
volume,blood loss, aortic stenosis, tamponade)
PP=SBP-DBP • Wide PP: up to 100 mmHg(e.g., exercise, atherosclerosis, aortic
regurgitation,AVmalformation,hyperthyroidism,aortic dilatation/
aneurysm, fever, anemia, pregnancy, anxiety,beri-beri)

Pulse Pressure Variation (PPV)


A dynamic measure of volume status and fluid responsiveness
• Most reliable method to know if preload (a determinant of stroke volume and cardiac
output) is optimized, provided parameters are met
PPm,.:maximumpulse pressure (duringinspirationin a positive
pressure-ventilatedpatient)
(PPmax- pp min) PPm;,:minimumpulse pressure (duringexpirationin a positive
PPV=------ pressure-ventilatedpatient)
pp
mean PP ": mean pulse pressure (average of PPm & PPm;,)
PPv' 2:12%predicts a fluid-responsivestate

Mixed Venous 02 Saturation (Sv02)


Measure of02 extraction in peripheral tissues & used to assess D02 & adequacy of CO
If Sv02 is decreased, CO is not meeting tissue oxygenation needs, and the body has
several methods for compensating:
° Cardiac output is increased, usually by increasing the heart rate (increases the D02)
0Tissues will attempt to extract a higher percentage of 02 as RBCs pass through the
capillary system (increases the oxygen consumption or V02)
0Anaerobic metabolism (also increases D02), which leads to accumulation of lactic
acid (manifesting as elevated serum lactate levels and metabolic acidosis)
Normal Sv02: 65-70%
0Decreased in low-CO shock
0Normal or elevated in high-CO shock

• High CO states • Sedation, analgesia, • Low CO states • Pain


0Early sepsis anesthesia •ACS/Ml • Stress/agitation/
0Hyperthyroidism • Ventilation or 0 Heart failure anxiety
0Liver cirrhosis respiratory support • Decrease in arterial • Shivering
0AV shunts • Hypothermia 02 content
• Fever/
• Resuscitation • Inefficient 02 0 Hypoxia
0IV fluids extraction 0 Anemia hyperthermia
0lnotropes 0 Late sepsis 0Hemorrhage • Infection
0Blood transfusion ° Cyanidepoisoning • Hypovolemia • Seizures
Sv02: mixedvenousoxygensaturation
V02 : oxygenconsumption
D02: oxygendelivery
279
II. HEMODYNAMIC PROFILES OF THE MAJOR TYPES OF SHOCK
In circulatory failure, three windows of perfusion that immediately give a clue of
hemodynamic instability are the following:
0 Mental state (restlessness, delirium, changing attention span, comatose state)
0 Skin perfusion (cold, clammy, mottled, or pale white; prolonged capillary refill time)
0 Oliguria
In those with depressed CO from cardiogenic shock, patients have cold & clammy skin
due to an increase in SVR as compensatory mechanism to maintain BP
In patients with micro-perfusion abnormalities and oxygen extraction issues, mottling
is frequently observed and is treated as an emergency regardless of blood pressure

Low-Cardiac Output vs. High-Cardiac Output Shock


Shock can be classified easily based on patient's cardiac output (because CO represents
oxygen delivery with the assumption of stable hemoglobin and oxygenation)
Sv02 is helpful in differentiating between the two:
• Shock with a decreased CO (e.g., cardiogenic, obstructive, hypovolemic)
Low-CO
• Higher oxygen extraction of tissue (V02) as a result of decreased oxygen
shock
delivery (D02), therefore Sv02 is typically decreased
• Shock with a normal or increased CO (e.g., distributive shock)
• P.roblem·is the oxygen extraction (V02) in the peripheral tissue, resulting
High-CO in high SV02 (D02 is generally preserved)
shock • Most common type of shock in the ICU (majority from septic shock)
• Systemic vasodilation occurs from inflammatory mediators (e.g., sepsis,
, anaphylaxis) or decrease in sympathetic tone (e.g., neurogenic)

• Small chambers with • Skin: cold & pale


Hypovolemic
preserved contractility • Jugular veins: flat

• Dilated chamber/s • Skin: cold, edema


Cardiogenic
i i with impaired
contractility
• Jugular veins:
distended

l i l • Tamponade: effusion;
RA/RV collapse
• Skin: cold

i
• Tension pneun10thorax:
Obstructive • Jugular veins:
small cardiac chambers
distended
• Pulmonary embolism:
small LV,dilated RV

High-Cardiac; Outp!ft ·shock

• Normal chambers with • Skin: mottling


normal contractility • Tachycardia
Distributive
l l i l i (unless with
cardiomyopathy)
• Temp: increased
or decreased

CVP:centralvenouspressure(N:0-8 cmHp or 0-6 mmHg)


PCWP:pulmonary capillarywedgepressure(N:4-12mmHg)
CO:cardiacoulput(N:4-8 Umin)
SVR:systemicvascularresistance
(N:700-1600dynes•s/cm·
5)

SvO2:mixedvenousoxygensaturation(N:65-70%)·
Sources:TanseyEA,et al.AdvPhysiolEduc,2019& PinskyMR,et al. Hemodynamic Monitoring,2019
JamesonJL,et al. Harrison's
Principles
of InternalMedicine20thedition,2018
DeMersD,WachsD. "Physiology, MeanArterialPressure." StatPearls,
2021
ChetanaShanmukhappa S, et al."VenousOxygenSaturation." StatPearls,
2021
TheodoreAC."Measures of oxygenation andmechanisms of hypoxemia."
UpToDate, 2020
280
III. MEASUREMENTS OF VOLUME STATUS & FLUID RESPONSIVENESS
The goal ofhemodynamic monitoring & assessment of fluid responsiveness is to ensure
adequate DO2
Volume expansion is most often first-line therapy during shock, but with a caveat
In fluid responders, it may increase CO and improve oxygen delivery
0 In non-fluid responders, it may aggravate tissue edema, lung edema, hemodilution,
and organ dysfunction
Frank-Starling Law: an increase in preload (usually by 300-500 cc of fluid) results in an
increase in SV and/or CO by 10-15%(degree of improvement is also variable)
Hemodynamic monitoring & fluid responsiveness tests will tailor management approach:
0 Volume expansion ° lnotropic support
0 Vasopressor use O Diuresis

A. Static Measurements
0Poor surrogates for volume status & do not accurately predict fluid responsiveness
(whether a fluid challenge will lead to an improvement in cardiac performance)
MEASUREMENT NORMAL I I REMARKS
0-8 cmH,O • Measured using a central line or catheter inserted via
Central venous
or internal jugular (IJ) or subclavian vein
pressure (CVP)
0-6 mmHg • Equilibrium between venous return function & cardiac function
• Measured using a pulmonary artery (Swan-Ganz) balloon
Pulmonary
flotation catheter inserted via lj, subclavian, or femoral vein
capillary wedge 4-12 mmHg
• Allows measurement of pressure in the pulmonary
pressure (PCWP)
capillary bed and left atrium

B. Dynamic Measurements
0 Based on the principle of pulsus paradoxus (variation of SV & BP with respiration),
with better correlation with a patient's fluid responsiveness
METHOD ITHRESHOLD* I LIMITATIONS

Pulse Pressure Variation (PPV) ii,

• Uses an arterial line (A-line) to demonstrate • Not used in patients


arterial pressure wave forms & determine with spontaneous
PPV>12%
maximum and minimum pulse pressures breathing, arrhythmia,
or on low tidal volume
,~
Passjve Leg Raising ,,
-' -·-
• Change in position leads to transfer of venous • Requires direct
blood from splanchnic & lower limbs to the measurement of
cardiac cavities, increasing mean systemic cardiac output
pressure and cardiac preload • Intra-abdominal
• Measure CO in real-time (using monitors or 2:10%increase hypertension causes a
echocardiography) at a 45°semi-recumbent in CO false negative test
position, then shift patient to a Trendelenburg
position (legs raised at a 45°angle)
• This is equivalent to a 300-cc standard fluid
challenge test
-w
Fluid Cfial/en,ge ' ·'
• Most direct way to assess responsiveness is to Mini:2:10% • Requires direct
administer fluid increasein CO measurement of CO
• Both diagnostic and therapeutic • Risk offluid overload
• Mini-fluid challenge: 100 mL Standard:2:15%
• Standard fluid challenge: 300-500 mL increasein CO
thatpatientis a "fluid-responder"
valuethatwouldindicate
'Threshold:
TanseyEA,et al.AdvPhys1ol
Sources: Educ,2019& BusseL, et al.AdvChronicKidneyDis:2013
2019
Monitoring,
PinskyMR,et al. Hemodynamic
281
SEPSIS AND SEPTIC SHOCK
I. ETIOPATHOGENESIS
Sepsis is the primary cause of death from infection, especially if not managed promptly
TNF-alpha is a central mediator
Intra vascular thrombosis is the hallmark of the local inflammatory response
• Endothelial injury is the major mechanism of multi-organ dysfunction
• Bacteremia vs. septicemia
0 Bacteremia: presence of bacteria in blood, as evidenced by positive blood cultures
0 Septicemia: presence of microbes or their toxins in blood

II. UPDATED DEFINITIONS BASED ON SEPSIS-3 (3rd International Consensus Definitions)


The 2016 Sepsis-3 Consensus revised the definition of sepsis, making it equivalent to the
severe sepsis of old
The new definition makes the condition more specific, as it removes those infections
that are not life-threatening and present with at least two SIRS criteria (which could
actually be just a normal host response to infection)
TERM I DEFINITION
• Life-threatening organ dysfunction (see definition below) due to a
Sepsis dysregulated host response to infection
• Arises when the body's response to an infection injures its own tissues & organs

Organ • Identified as an acute change in total Sequential (Sepsis-Related) Organ


dysfunction Failure Assessment (SOFA) score of;>:2points (consequent to the infection)
• Subset of sepsis where underlying circulatory and cellular/metabolic
abnormalities are profound enough to substantially increase mortality
Septic • Clinical criteria identifying this condition include:
shock 0 Need for vasopressors to obtain a mean arterial pressure (MAP) ;,65 mmHg

despite adequate fluid resuscitation


0 Increase in serum lactate >2 mmol/L (18 mg/dL) in the absence ofhypovolemia

Source:Singer,et al.The3rd International


Consensus
Definitions
for SepsisandSepticShock.JAMA,2016

III. DIAGNOSIS
AD'
DIAGNOSTIC I REMARKS

• CBC to check for leukocytosis/leukopenia, anemia, thrombocytopenia


• Blood chemistry to check for organ dysfunction (e.g., kidney, liver)
Basic work-
• Imaging as needed (e.g., chest, abdomen, pelvis, brain)
up
• Cardiac studies (e.g., ECG, cardiac markers)
• Gram stain & cultures as needed (e.g., sputum, urine, secretions/discharge)

• Obtained before administering antibiotics to patients suspected of sepsis or


Blood septic shock, if it will not cause a substantial delay in initiation of antibiotics
cu1tures • Complemented by appropriate cultures taken from suspected focus of infection
• Appropriate cultures include at least 2 sets of blood cultures (aerobic & anaerobic)

• Acute-phase reactant that is elevated in severe bacterial infections (high


PCT has been shown to predict bacteremia)
• Surviving Sepsis Campaign 2021 Guidelines now suggest against using PCT
Procalcitonin as an adjunct to clinical evaluation when deciding to initiate antibiotic
(PCT) therapy (clinical evaluation alone should suffice)
• May be used to guide antibiotic discontinuation if optimal duration of
therapy is unclear despite adequate source control (a decreasing trend or
low values of PCT may support discontinuation of antimicrobials)

• Level >2 mmol/L (18 mg/dL) after adequate fluid resuscitation can identify
Serum
patients with septic shock
lactate
• Guides hemodynamic resuscitation, with the goal of normalizing lactate levels

282
B. Approach to Identifying Patients with Sepsis and Septic Shock
Potient with suspected infection

Ye,

Ye,

No Monitor clinical condition;


reevaluate for possible sepsis if
clinically indicated

Sepsis

No

1. asopressors required to main 01n Ye,


M 65 mmHg AND Septic shock

ThebaselineSOFAscoreisassumedto be zerounlesspatientis knownto havepreexisting (acuteorchronic)


organ
dysfunction
beforetheonsetofinfection.
Neither
SOFAnorqSOFA is intendedtobe a stand-alone
definition
ofsepsis.
Source:Singer,et al. The 3rd International
ConsensusDefinitions
forSepsisand SepticShock.JAMA,2016
C. Quick SOFA (qSOFA)
° Facilitates prompt identification of an infection that poses a greater threat to life
0 Used to quickly assess patients at bedside without the use of laboratory parameters
• Respiratory rate <!:22breaths/min
• Altered mental status (GCS <15)
• Systolic blood pressure s10OmmHg
0 Score <!:2 warrants further investigation for organ dysfunction &/or escalation of therapy
0 The Surviving Sepsis Campaign 2021Guidelines recommend against using qSOFA as a
single-screening tool for sepsis/septic shock due to its poor sensitivity; its utility lies in
its ability to predict poorer outcomes when a patient is qSOFA positive (score <!:2)

D. Sequential (or Sepsis-Related) Organ Failure Assessment Score (SOFA)


° Composed of scores from 6 organ systems,graded from o to 4 according to degree of dysfunction
° Calculated on admission and every 24 hours until discharge, using the worst parameters
measured during the prior 24 hours

SYSTEM

Respiration(PaO/FiO2 in mmHg) 2:400 <400 <300 <200" <100*


Coagulation(platelet
countin103/ul) 2:150 <150 <100 <50 <20
Liver(bilirubinin mg/dl) <1.2 1.2-1.9 2.0-5.9 6.0-11.9 >12.0
MAP MAP Dopa5.1-15,or
Dopa>15,or
Dopa<5,or
Cardiovascular•• EplS0.1,or Epi>0.1,or
<!:70mmHg<70mmHg lxw(ITTfcble) NEs0.1 NE>0.1
CNS{GlasgowComaScale) 15 13-14 10-12 6-9 <6
Renal
Creatinine(inmg/dl) <1.2 1.2-1.9 2.0 - 3.4 3.5-4.9 >5
Urineoutput(inml} <500 <200
Dopa:dopamine Epi:epinephrine "Withrespiratorysupport
NE:norepinephrine
Dobu:dobutamine ""Catecholamine doses are givenas mcg/kg/min
forat least 1 hour
Source:VincentJL,et al. The SOFAscore to describeorgandysfunction/failure.
IntensiveCare Med,1996
283
IV. MANAGEMENT

• For resuscitation of sepsis-induced hypo perfusion: at least 30 mL/kg of IV crystalloid


fluid (preferably balanced crystalloids over normal saline) within the first 3 hours, with
subsequent fluids guided by reassessment ofhemodynamics
• Using starches and gelatin for resuscitation is not recommended
• Avoid exceeding 5 liters of total !VF volume in the first 24 hours
• Initial target: MAP ;:65 mm Hg in septic shock requiring vasopressors (may consider MAP
target of 75-85mmHg if with pre-existing hypertension)
• Dynamic measures may be used (e.g., passive leg raising combined with CO measurement,
fluid challenges against stroke volume) to guide fluid resuscitation
• Serum lactate and capillary refill time as adjuncts to other measures of perfusion
• For patients who require ICU admission, admission is recommended within 6 hours

• Septic shock or high likelihood of sepsis: administer IV antimicrobials immediately, ideally


within I hour of recognition
• Possible sepsis without shock: rapid assessment for infectious vs. non-infectious causes;
administer IV antimicrobials within 3 hours of recognition if concern for infection persists
• Daily assessment for de-escalation of antimicrobials is recommended (based on cultures)
• Duration of7-10 days is adequate for most serious infections in sepsis & septic shock:
0 May be longer if with slow clinical response, undrainable foci of infection, bacteremia with
S. aureus,some fungal & viral infections, or immunologic deficiencies
0 May be shorter if with rapid clinical resolution following effective source control ofintra-
abdominal or urinary sepsis & those with uncomplicated pyelonephritis
• Acceptable regimens as the initial antimicrobial therapy for severe sepsis or septic shock
with no obvious source of infection in immunocompetent adults:
0 Piperacillin-tazobactam 3.375-4.5g IV q6, OR
° Cefepime 2 g IV 912,OR
0 Meropenem I g IV q8
0 Then add vancomycin (LD 25-30 mg/kg IV, then 15-20mg/kg IV q8-12)to each of the above
regimens to cover for MRSA

Vaso11ressors
• Norepinephrine (NE): first-line vasopressor in septic shock
• Vasopressin: may be added to NE with intent of raising MAP to target ;:65mmHg
• Epinephrine: third-line agent if still with inadequate MAP while on NE and vasopressin
• Septic shock with concomitant cardiac dysfunction: NE+ dobutamine, OR epinephrine alone
Source Contr.ol
• Identification of a specific anatomic diagnosis of infection requires emergent source control
• Includes surgery, resection or drainage of source; or evacuation of infectious material
Steroids
• Not routinely used if fluid resuscitation & vasopressors are able to restore stable hemodynamics
• IV hydrocortisone 200 mg/day (50 mg IV q6) may be given for septic shock unresponsive to
fluids and vasopressors (commenced once dose of norepinephrine or epinephrine ;co,25mcg/
kg/min at least 4 hours after initiation)

' Glucost Bimtroi


• Target upper serum glucose: ,,;180mg/dL
• Insulin infusion & frequent glucose monitoring with a target glucose range of 140-180mg/dL

Renal Replacement 'J,erapy-(w~enindieate.d)


• Intermittent hemodialysis for hemodynamically stable patients
• Consider continuous hemodialysis (CRRT) for hemodynamically unstable patients
284
Blood Transfusion
... .. -
• Restrictive transfusion strategy preferred over liberal transfusion
• Packed RBC: only when hemoglobin <7.0 g/dL (in the absence of myocardial ischemia, severe
hypoxemia, or acute hemorrhage)
• Prophylactic platelet transfusion:
° For platelet count <IO,ooo/mm' in the absence of bleeding
° For platelet count <20,ooo/mm' if patient has a significant risk of bleeding
• A plate.let count ~50,000/mm' is advised for active bleeding, surgery, or other invasive procedures
Mechanical Ventilation " -,
• Target tida] volume: 6 ml/kg of predicted body weight (compared with the usual 12ml/kg)
• Upper limit goal for plateau pressure: <30 cm H2O
• Moderate to severe sepsis-induced ARDS: use higher PEEP & utilize recruitment maneuvers

Prophylax~s ,
• Venous thromboembolism (e.g., DVT or PE): mechanica] compression devices and heparin/
low molecular weight heparin
• Stress ulcer: histamine blocker or proton pump inhibitor (PP!)
• Ventilator-associated pneumonia: maintain head of bed >30 degrees in intubated patients,
sedation holidays, daily assessment for weaning/extubation
Sources:EvansL,et al. Surviving sepsis campaign.IntensiveCare Med;2021
2020
BuensalidoJ, et al. CPGfor Sepsisand SepticShockinAdultsin the Philippines;

CARDIOGENIC SHOCK (CS)


I. ETIOPATHOGENESIS
A syndrome due to a primary cardiac disorder resulting in poor cardiac output, tissue
hypoperfusion and eventual multi-organ failure and death
• Most common etiology: LV failure from ACS/MI
Other causes include: · .
0 Mechanical complications of ACS (e.g., ventricular septa! or free wall rupture, acute · ·
MR from papillary muscle/chordal rupture)
° Cardiomyopathy
0 Myocarditis
° Critical valvular heart disease
0 Arrhythmias
0 Drug-induced (negative inotropes or vasodilators)

II. MANIFESTATIONS
A General Manifestations
• Cold, clammy extremities, narrow pulse pressure
Clinical evidence
• Oliguria
ofhypoperfusion
• Mental confusion, dizziness

Biochemical • Elevated serum creatinine


evidence of • Metabolic acidosis
hypoperfusion • Elevated serum lactate

Hemodynamic • Depressed cardiac index (<2.2L/min/m')


evidence of • Systolic BP <90 mmHg (or requires support to maintain ~90 mmHg)
hypoperfusion• • Elevated filling pressures (PCWP ~18 mmHg)

• Tachycardia and tachypnea


, Jugular venous distention (elevated )VP)
Other PE findings • Crackles from pulmonary congestion
• Soft Sr and audible S3 gallop
, Systolic murmurs of severe MR or ventricular septa) rupture (VSR)
'Hypoperfusionis not always accompaniedby hypotension,as BP may be preserved by compensatory
vasoconstriction(with/without
pressor agents)
285
B. Stae:es ofCardioe:enic Shock

• No signs or symptoms yet, but at risk for development (e.g.,


A At risk
patients with AMI, acute and/or chronic HF)
Beginning • With relative hypotension or tachycardia but without
B
cardiogenic shock hypoperfusion
Classic cardiogenic • With hypoperfusion requiring intervention beyond volume
C
shock resuscitation
Deteriorating or • Similar to Stage C but worsening (not responding to initial
D
doom interventions)
E Extremis • Circulatory collapse, refractory cardiac arrest, ongoing ACLS
TA,et al. 2021ESCGuidelines
Source:McDonagh forAcuteandChronicHeartFailure.EurHeartJ. 2021
of InternalMedicine20thedition,2018
Principles
JamesonJL,et al. Harrison's
III. DIAGNOSIS
DIAGNOSTIC I REMARKS

• Check for anemia or infection (leukocytosis), which may trigger


CBC
decompensation of HF
• Serum creatinine: marker of renal failure/hypoperfusion
Blood • Serum electrolytes: need to be corrected to minimize risk of arrhythmias
chemistry • Liver function tests: monitor for congestive hepatopathy or ischemic hepatitis
• Lactate: elevated values provide biochemical evidence of tissue hypoperfusion
Cardiac • Troponins: elevated absolute values or rise/fall in values suggest myocardial
biomarkers injury/infarction
• Measures ejection fraction, detects wall motion abnormalities, mechanical
2D Echo
complications of ACS, valvular lesions, tamponade physiology
ECG • To check for ischemia/infarction
Chest X-ray • To check for pulmonary congestion/edema and cardiomegaly
Angiography • To assess coronary anatomy to plan best option for revascularization

IV. MANAGEMENT
Since the vast majority of CS is caused by ACS, management strategy is initially aimed
towards working up this differential and early revascularization to improve myocardial
pump function

Classic wet • Stabilize with NE, then add an inotrope after revascularization (if indicated)
& cold profile • Dopamine (instead of NE) ifbradycardic or with low risk of arrhythmias

Euvolemic • Stabilize with NE, then add an inotrope afterrevascularization (if indicated)
cold&dry • Dopamine (instead of NE) if bradycardic or with low risk of arrhythmias
profile • May tolerate small fluid boluses (since LVEDP may be low)
Mixed
• NE
cardiogenic
• Invasive monitoring useful for guidance in multifactorial shock
& vasodilatory
• Fluid boluses
RV shock/ • NE, dopamine or vasopressin; then add or transition to inotrope after
failure revascularization (if indicated)
• Pulmonary vasodilators to decrease RV afterload
286
B. General Management Principles ofCardiogenic Shock
ASPECT I DETAILS IN MANAGEMENT
• No clear-cut first-line vasoactive medication for CS, but norepinephrine
Vasoactive is usually the vasopressor of choice due to lower risk of arrhythmias
drugs (compared to dopamine and dobutamine)
• Target MAP >65 mmHg
• Diuretics & nitrates for patients with pulmonary congestion/edema
• Anti-thrombotic therapy and statins as indicated in ACS patients
Other drugs • Avoid B-blockers& RAASinhibitors initially;may initiate at low doses once:
0 Euvolemic

, Off inotropic/vasopressor support ~24 hours


0 Renal function has returned to near baseline

• Admit to ICU or CCU (coronary care unit) with 1:1 nurse-patient ratio
• Arterial line for continuous BP monitoring & easier access for ABG
• Central venous line to monitor CVP & central venous 02 saturation and
Monitoring facilitates volume resuscitation & inotrope/vasopressor infusion
• Invasive hemodynarnic monitoring with a pulmonary arterial catheter (Swan-
Ganz catheter) can help guide fluid management & determine PA pressures
• Indwelling Foley catheter for more accurate urine output monitoring

Oxygen • Indicated if02 saturation on pulse oximetry <90%


support • Non-invasive ventilation or intubation/mechanical ventilation if needed
• Restores coronary blood flow and salvages ischemic myocardium
Revascularization
• Improves myocardial perfusion and performance
for ACS
• Fibrinolysis, early invasive PC!, or urgent CABG as needed
Mechanical
circulatory • IABP,LVAD and ECMOdiscussedat the end of this chapter
support (MCS)
• Ultrasound guidance when establishing central vascular access
Others
• Sedation protocols and holidays
bundles of!CU
• VTE prophylaxis with anticoagulation
care
• Stress ulcer prophylaxis with PP! or H2 blocker
Source:VanD1epen S, et al.AHAStatementon Management of Card1ogemc 2017
Shock.C1rculat1on.

SELECTING VASOPRESSORS AND INOTROPES IN SHOCK


Vasopressors improve MAP and tissue perfusion by an increase in vasomotor tone
Vasopressors are distinguished from inotropes by having no direct effect on the cardiac output (CO)
Drugs may have both vasopressor and inotropic properties
Concepts before starting vasoactive agents:
' Fluid responsiveness is assessed & volume resuscitation done if patient is a fluid-responder
Administer low initial doses and gradually titrate up
0

' Administer through central lines co avoid extravasation in peripheral lines & tissue necrosis

Inotropes and Vasopressors for Specific Clinical Situations


CONDITION I INOTROPE/PRESSOR OF CHOICE
• Norepinephrine (noradrenaline) is the first-line
Septic shock • Add vasopressin if needed to reduce norepinephrine dose
• Consider dobutamine if with persistent hypoperfusion

• Norepinephrine with/or dobutarnine


Cardiogenic
• Consider rnilrinone if with increased pulmonary venous resistance or PVR
shock
(i.e., RV failure-predominant shock)

Anaphylaxis • Intramuscular (IM) epinephrine

Cardiac arrest • Intravenous (IV) epinephrine per ACLS algorithm (seeACLS section)
287
Dosing of the Different Vasopressors and Inotropes
DRUG I DOSE
Norepinephrine • 0.oI-1.0 mcg/kg/min IV infusion
Vasopressin • 0.01-0.03 U/min IV infusion (up to 0.06 U/min in some studies)
Dobutamine • 5-20 mcg/kg/min IV infusion
Dopamine • 1-20 mcg/kg/min IV infusion

• 0.01-0.5 mcg/kglmin IV infusion


Epinephrine • 1 mg IV bolus (cardiac arrest)
• 0.5 mg IM (anaphylaxis)
Milrinone • 50 mcg/kg IV bolus then 0.25-0.75 mcg/kg/min IV infusion
Terlipressin • 1-2mg JV bolus
Sources:BejarRM.Manualof Perioperat1ve 2021
Carein AdultCardiacSurgery6th Ed1t1on,
PinskyMR,et al. Hemodynamic 2019& JentzerJC,et al. J Cardiovasc
Monitoring, Ther,2015
Pharmacol
2020
CPGfor SepsisandSepticShockin Adultsin thePhilippines,

I. NOREPINEPHRINE(NORADRENALINE)
Produces significant vasoconstriction with minimal effect on the heart rate (the beta-
agonist effect is counteracted by increase in afterload resulting in reflex bradycardia)
First-line agent for septic shock & used with dobutamine in cardiogenic shock
May have a role in critical hypotension from traumatic hemorrhagic shock while
ongoing volume replacement (diverts volume from splanchnic to arterial circulation)

A. Dosing ofNorepinephrine
0Usually started at a dose of 2 to 4 mcg/min (0.03 to 0.07 mcg/kglmin in a 60-kg patient)
and titrated upward as necessary
0If systemic perfusion or systolic pressure cannot be maintained at >90 mmHg with a
dose of 15mcglmin, it is unlikely that further increases in dose will be of benefit

B. Sample Cases for Norepinephrine Drip


Case 1: A 60-year-old male (50 kg) was admitted for septic shock. You plan to start
norepinephrine (NE) drip at a dose of 0.05 mcglkg/min. You decide to use 4 mg
norepinephrine in 250 mL D5W.
Step 1: Determine Drug concentration = 4 mg of norepinephrine divided by 250 mL
drug concentration =o.016mg/mL
Since the computed concentration is in mg/mL and the desired
Step 2: Convert dose is in mcglkglmin, there is a need to convert the units
units as needed
0.016 mg/mL = 16 mcg/mL

Desired Dose (mcg/kg/min) x Body weight (kg) x 60 (mins/hr)


=
Drug Concentration (mcg/mL)
Step 3: Compute
for drip rate 0.05 (mcg/kglmin) x 50 (kg) x 60 (mins/hr)
(mL/hr) =
16(mcg/mL)

= 9.38 or -10 mL/hr (or cc/hr or ugtt/min)


• Start NE drip: 4 mg norepinephrine + 250 cc D5W x 10 cc/hr
Order in the chart • This will achieve the desired dose of -0.05 mcglkglmin
(or 2.5 mcg/min for a 50-kg patient)

288
Case 2: Same case as above, but you plan to start NE drip at a dose of 2 mcg/min (note that dose
given is not based on weight). You decide to use 2 mg NE in 250 mL D5W. In this example, since
the desired drug dose is not based on weight, formula is simplified by omitting body weight.

Steps I and 2
Drug concentration =2 mg of norepinephrine divided by 250 mL
=0:008 mg/mL =8 mcg/mL
= Desired Dose (mcg/min) x 60 (mins/hr)
Step 3: Compute Drug Concentration (mcg/mL)
for drip rate
2 (mcg/min) x 60 (mins/hr) =15mL/hr
(mL/hr) = 8(mcg/mL)
(or cc/hr or ugtt/min)

Order in chart • Start NE drip: 2 mg norepinephrine + 250 cc D5W x 15cc/hr (2 mcg/min)

• I I

-
DripRate
(cc/hror
ml/hr)

5 cc/hr 0.7mcg/min 1.3mcg/min 2.7mcg/min


10cc/hr 1.3mcg/min 2.7mcg/min 5.3 mcg/min
15cc/hr 2.0 mcg/min 4.0 mcg/min 8.0 mcg/min
20 cc/hr 2.7mcg/min 5.3mcg/min 10.7mcg/min
25 cc/hr 3.3 mcg/min 6.7mcg/min 13.3mcg/min
30 cc/hr 4.0mcg/min 8.0 mcg/min 16.0mcg/min
40 cc/hr 5.3 mcg/min 10.7mcg/min 21.3mcg/min
50 cc/hr 6.7 mcg/min 13.3mcg/min 26.7mcg/min
60 cc/hr 8.0 mcg/min 16.0mcg/min 32.0mcg/min
70 cc/hr 9.3 mcg/min 18.7mcg/min 37.3mcg/min
80 cc/hr 10.7mcg/min 21.3mcg/min 42.7mcg/min
90 cc/hr 12.0mcg/min 24.0mcg/min 48.0mcg/min
100cc/hr 13.3mcg/min 26.7mcg/min 53.3mcg/min

II. DOPAMINE
Releases norepinephrine from nerve terminals, which then stimulates a1 and ~I receptors
• Generally used to augment BP & CO in patients with cardiogenic shock
May also be used to increase the HR in patients with symptomatic bradycardia (e.g.,AV block)
Use has become limited in the ICU setting
0 Higher mortality and tachyarrhythmia compared to norepinephrine
0 Renal-dose dopamine abandoned due to no proven benefit in renal function in AKI patients
0 Use confined to bradycardia with low risk of arrhythmia

A. Dosing of Dopamine
Dopamine demonstrates varying hemodynamic effects based on the dose
0

Usuall started at 2-5 me / /min & increased every 2-5 mins (maximum of 20 me /
0

<2 • ActivatesDAI& DA2 • Selective dilation of renal, splanchnic, & cerebral arteries
mcglkg/min receptors in kidney • Can increase renal blood flow & promote natriuresis
2-10 • Activates~!- • lnotropic effect due to norepinephrine release
mcg/kg/min receptors • Increase in CO with little or no change in HR or SVR.
• Effect on al-receptor
• Vasoconstriction from direct agonise effect on cxI-
>10 overwhelms the
mcg/kg/min dopaminergic receptors
• Increase in SVR, LV filling pressures, and HR
receptors
289
BS amp.e1 C ases £or D opamme o·np
Case 1: A 55-year-old female (45 kg) is in cardiogenic shock. You plan to start dopamine drip
at IO mcg/kg/min (chronotropic/inotropic dose). You prepared the dopamine drip as follows:
2 amps (400 mg) of dopamine in 250 mL D5W.

Step 1: Determine Drug concentration =400 mg· of dopamine divided by 250 mL


drug concentration =1.6 mg of dopamine/ml
Since the computed concentration is in mg/mL and the desired dose is in
Step 2: Convert mcg/kg/min, there is a need to convert the units
units as needed
1.6 mg/mL =1600 mcg/mL
= Desired Dose (mcg/kg/min) x Body weight (kg) x 60 (mins/hr)
Step 3: Compute Drug Concentration (mcg/mL)
for drip rate
(mL/hr) 1,= IO (mcg/kg/min) x 45 (kg) x 60 (mins/hr) =-17 mL/hr (or cc/hr or ugtt/min)
1600 (mcg/mL)

Order in the • Start dopamine drip: 400 mg (2 amps) dopamine+ 250 cc D5W x 17cc/hr
chart • This will achieve the desired dose of -IO mcg/kg/min

Case 2: Same scenario as above using the Shortcut Method


Desired Dose (mcg/kg/min) x Body weight (kg)
Dopamine Drip Rate (mL/hr) =
Dopamine factor

Dopamine Factors:
• For a formulation of I dopamine ampule (200 mg) in 250 cc D5W, factor used is 13.3
• For a formulation of 2 dopamine ampules (400 mg) in 250 cc D5W, factor used is 26.6

Computing the drip rate (same case as above):


• Since our desired dose is IO mcg/kg/min (chronotropic/inotropic dose), and we decided to use
400 mg (2 amps) of dopamine (factor is 26.6), the dopamine drip rate is computed as follows:
(mcg/kg/min) x 45 (kg)
Drip Rate (mL/hr) = IO

26.6
=16.9 or -17 mL/hr (or cc/hr or ugtt/min)

Case 3: Using Reverse Computation (computing for the dose of dopamine from the current drip rate).
The patient is a 55-year-old female (45 kg) being given 400 mg of dopamine in 250 mL PNSS at
a rate of19 ugtts/min (or 19cc/hr). How much dopamine (in mcg/kg/min) are you giving the patient?

Step 1: Determine
drug concentration
• 400 mg divided by 250 mL =1.6mg/mL
Step 2: Convert
units as needed
• 1.6 mg/mL =1600 mcg/mL

= Dopamine drip rate (mL/hr) x Drug concentration (mcg/mL)


Step 3: Compute Body weight (kg) x 60 (mins/hr)
for dopamine
19 (mL/hr) x 1600 (mcg/mL)
dose being given = =11.3mcg/kg/min of dopamine is being
45 (kg) x 60 (mins/hr) given to the patient

Since we are giving 11.3mcg/kg/min, we have a vasoconstrictive effect


which is beneficial in a patient with septic shock. If the patient is still
Titration hypotensive, we can increase the cc/hr (titrate) up to 34 cc/hr (20 mcg/kg/
min) for a 45-kg patient ("dopa max"). If still with no response to maximal
dopamine dose, we can start another vasopressor like norepinephrine._

290
C. Quick Reference for Dopamine
D , DWP ti
Drip Rate
(cc/hr or ml/hr)

2.5 7.5cc/hr 9.4cc/hr 11.3cc/hr 13.1cc/hr 15.0cc/hr 16.9cc/hr


C:
"§ 5.0 15.0cc/hr 18.8cc/hr 22.5cc/hr 26.3cc/hr 30.0cc/hr 33.8cc/hr
-"
oo 7.5 22.5cc/hr 28.2cc/hr 33.8cc/hr 39.4cc/hr 45.0cc/hr 50.7cc/hr
oo
u
_g 10.0 30.0cc/hr 37.6cc/hr 45.0cc/hr 52.6cc/hr 60.0cc/hr 67.6cc/hr

0
15.0 45.0cc/hr 56.4cc/hr 67.6cc/hr 78.8cc/hr 90.0cc/hr 101.5cc/hr
a
20.0 60.0cc/hr 75.2cc/hr 90.0cc/hr 105.2cc/hr 120.0cc/hr 135.3cc/hr

Dopamine 400 mg+ 250 cc D5W Preparation


Drip Rate
(cc/hr or ml/hr)

2.5 3.8cc/hr 4.7 cc/hr 5.6 cc/hr 6.6cc/hr 7.5cc/hr 8.4cc/hr


c
"§ 5.0 7.5cc/hr 9.4cc/hr 11.3cc/hr 13.1cc/hr 15.0cc/hr 16.9cc/hr
oo
-"
oo
u
7.5 11.3cc/hr 14.1cc/hr 16.9cc/hr 19.7cc/hr 22.5cc/hr 25.3cc/hr
_g 10.0 15.0cc/hr 18.8cc/hr 22.5cc/hr 26.3cc/hr 30.0cc/hr 33.8cc/hr
"' 15.0 22.5cc/hr 28.2cc/hr 33.8cc/hr 39.4cc/hr 45.0cc/hr 50.7cc/hr
Cl
20.0 30.0cc/hr 37.6cc/hr 45.0cc/hr 52.6cc/hr 60.0cc/hr 67.7cc/hr

Ill. DOBUTAMINE
A synthetic sympathomimetic amine with positive inotropic action
Effects are due to selective stimulation of B1adrenergic receptors
Drug of choice for myocardial dysfunction

A. Dosing ofDobutamine
0 Minimal positive chronotropic activity at low doses (2.5 mcg/kg/min) and moderate
positive chronotropic activity at higher doses
0 At therapeutic dose range (-IO mcg/kg/min), it has a peripheral vasodilatory effect which
can further worsen hypotension in those with decreased SVR & SBP <70 mm Hg (a
vasopressor should be added to counteract dobutamine's peripheral vasodilating effects)

BS amp 1e Computat1ons or D o b utamme D'np


Case 1: A 60-year-old-male (55kg) was admitted for cardiogenic shock. You plan to start dobutamine
at a dose of5 mcg/kg/min. You decide to use 500 mg (2 amps) of dobutamine in 250 cc D5W.

Step 1: Determine Drug concentration = 500 mg of dobutamine divided by 250 mL


drug concentration =2mg/mL

Since the computed concentration is in mg/mL and the desired dose is


Step 2: Convert in mcg/kg/min, there is a need to convert the units
units as needed
2 mg/mL = 2000 mcg/mL
= Desired Dose (mcg/kg/min) x Body weight (kg) x 60 (mins/hr)
Step 3: Compute Drug Concentration (mcg/mL)
for drip rate
(mL/hr) = 5 (mcg/kg/min) x 55 (kg) x 6o (mins/hr) = -8 mL/hr (or cc/hr or ugtt/min)
2000 (mcg/mL)
Order in the • Start dobutamine drip: 500 mg (2amps) dobutamine + 250cc D5W x 8 cdhr
chart • This will achieve the desired dose of -5 mcg/kg/min
291
Case 2: Same scenario as above using the Shortcut Method
Desired Dose (mcg/kg/min) x Body weight (kg)
Dobutamine Drip Rate (mL/hr) =
Dobutamine factor

Dobutamine Factors:
• For a formulation of1 dobutamine ampule (250 mg) in 250 cc D5W, factor used is 16.6
• For a formulation of2 dobutamine ampules (500 mg) in 250 cc D5W, factor used is 33.2

Computing the drip rate (same case as above):


• Since our desired dose is 5 mcg/kg/min and we decided to use 500 mg (2 amps) of
dobutamine (factor is 33.2),the dobutamine drip rate is computed as follows:
5 (mcg/kg/min) x 55 (kg)
Drip Rate (mL/hr) = = 8.3 or -8 mL/hr (or cc/hr or ugtt/min)
33.2

C. Quick Reference for Dobutamine


Dobutamine 250 m + 250 cc D5W Preparation
Drip Rate
(cc/hr or ml/hr)

c 2.5 6.0cc/hr 7.5cc/hr 9.0cc/hr 10.5cc/hr 12.0cc/hr 13.5cc/hr


E 5.0 12.0cc/hr 15.0cc/hr 18.0cc/hr 21.0cc/hr 24.0cc/hr 27.0cc/hr
OD
-"' 7.5 18.0cc/hr 22.5cc/hr 27.0cc/hr 31.5cc/hr 36.0cc/hr 40.5cc/hr
OD
u
..§, 10.0 24.0cc/hr 30.0cc/hr 36.0cc/hr 42.0cc/hr 48.0cc/hr 54.0cc/hr
15.0 36.0cc/hr 45.0cc/hr 54.0cc/hr 63.0cc/hr 72.0cc/hr 81.0cc/hr
0
Cl 20.0 48.0cc/hr 60.0cc/hr 72.0cc/hr 84.0cc/hr 96.0cc/hr 108.0cc/hr
Db t , DWP ,
DripRate
(cc/hror ml/hr)

c 2.5 3.0cc/hr 3.8 cc/hr 4.5 cc/hr 5.3cc/hr 6.0 cc/hr 6.8 cc/hr
.E
5.0 6.0cc/hr 7.5cc/hr 9.0 cc/hr 10.5cc/hr 12.0cc/hr 13.5cc/hr
OD
-"' 7.5 9.0cc/hr 11.3cc/hr 13.5cc/hr 15.8cc/hr 18.0cc/hr 20.3cc/hr
OD
u
..§, 10.0 12.0cc/hr 15.0cc/hr 18.0cc/hr 21.0cc/hr 24.0cc/hr 27.0cc/hr
15.0 18.0cc/hr 22.5cc/hr 27.0cc/hr 31.5cc/hr 36.0cc/hr 40.5cc/hr
0
Cl 20.0 24.0cc/hr 30.0cc/hr 36.0cc/hr 42.0cc/hr 48.0cc/hr 54.0cc/hr

IV. VASOPRESSIN
Acts on vascular vasopressin-1a receptors as a pure vasoconstrictor agent (no inotropic action)
• Adjunct to NE in distributive/septic shock if unresponsive or to reduce dose of NE
• Adverse effects: hyponatremia, pulmonary vasoconstriction, ischemia at higher doses

V. EPINEPHRINE (ADRENALINE)
Potent agonist of a, and ~I receptors, with stronger ~2 receptor activity than norepinephrine
Lower doses: ~1 agonist function with minimal effect on vascular tone
0

Higher doses: a1 agonist activity increases, resulting in vasoconstriction


0

First-line agent for anaphylactic shock


IV boluses used during resuscitation ofnon-shockable rhythms in ACLS
Disadvantages: arrhythmias and splanchnic vasoconstriction

VI. MILRINONE
Inodilator that inhibits phosphodiesterase 3 (PDE3),thus augmenting downstream !>-receptorsignaling
• Stimulates myocardial contractility and improves CO
Significantly lowers both SVR and PVR, with greater reduction in PVR
Preferred for chronic heart failure with pulmonary hypertension, RV failure, or on ~-blockers

VII. TERLIPRESSIN
A synthetic vasopressin analog with greater selectivity for the V1 receptor
Longer half-life, hence can be administered as IV boluses
292
OXYGEN DELIVERY
• Utilized to maintain adequate tissue oxygenation while minimizing cardiopulmonary work
• Indications: hypoxemia, increased working of breathing, & hemodynamic insufficiency
I. DEFINITION OF TERMS
TERM I DEFINITION/REMARKS

FiO2 • Fraction of inspired oxygen, in percent(%)


• Partial pressure of carbon dioxide (CO2) in arterial blood
PaCO2 • Used to assess adequacy of ventilation
• Normal value: 35-45mmHg
• Partial pressure of oxygen (02) in arterial blood
PaO2 • Used to assess adequacy of oxygenation
• Normal value: 80-IOo mm Hg (may be lower with increasing age)
SaO2 • Arterial oxygen saturation measured from blood specimen
SpO2 • Arterial oxygen saturation measured via pulse oximetry

II. DELIVERY METHODS


Choice of oxygen delivery devices depends on the patient's oxygen requirement, efficacy
of the device, reliability, ease of therapeutic application, and patient acceptance
Can be delivered via low-flow or high-flow systems:
0 Low-flow delivery: provides supplemental oxygen less than the patient's total minute
ventilation (i.e., patient's minute ventilation exceeds flow)
0 High-flow delivery: supplies O, at a flow equaling or exceeding the inspiratory flow demand
SYSTEM* I REMARKS I ILLUSTRATION I Flow Rate I FiO2

• FiOz increases by ~2-4%/L and


provides 23-45%of 02 11pm 24%
• Maximum flow rate of 6 1pm 2 lpm 28%
Nasal (rate ,6 lpm does not augment 3 lpm 32%
cannula the inspired gas) 4 lpm 36%
• High flowscan dry nasal mucosa 5 lpm 40%
• Humidification device is advised 6 lpm 44%
for flow rates ,4 lpm

• Higher potential FiO2 with flow


Simple rates between 5-IO1pm 5-6Ipm 40%
face • 5 1pm is needed to flush exhaled 6-?lpm 50%
mask .. CO2 from the mask (<5lpm is 7-8Ipm 60%
not recommended)

• Mask with 02 reservoir bag &


one-way valves which prevent
Non- exhaled gas from mixing with fresh
gas flow, & prevent entrainment
rebreather 10-15lpm 80-100%
of room air into the mask
mask ..
• Provides higher concentration
ofFiO2 (>60%) than a standard
face mask
*Low-flowoxygendeliverysystemsincludenasalcannula,simplefacemasksandnon-rebreather masks
(high-flow
systemsarediscussed in thenextsection)
'*Facemaskscannotdeliver100%oxygen,unlessthereis a tightseal.Non-rebreathermaskscandeliver
an FiO2upto -80%.AnFiO2of 100%canonlybe delivered witha ventilator,
wearinga tight-fitting
maskor
througha high-flownasalcannulawithveryhighflowrates
293
PRINCIPLES OF VENTILATION
I. FUNCTIONS OF MECHANICAL VENTILATION
Main objectives of mechanical ventilators are:
0 To increase Pa02
0 To decrease PaC02
• Ventilators also allow the respiratory muscles to rest & avoid/treat respiratory muscle fatigue

II. BASIC CONCEPTS IN MECHANICAL VENTILATION


CONCEPT I REMARKS
•Positive-Pressur,e :I,~rsus Negative-Pressure Ventilation
~-
• Raises the pressure at the level of airways to improve pressure
gradient from airway to alveoli
Positive-pressure
• Can be done invasively (e.g., endotracheal/tracheostomy tube) or
non-invasively (e.g., nasal/face mask)
• Lowers the pressure at the level of the alveolus to improve pressure
Negative-pressure
gradient from airway to alveoli (e.g.,normal tidal breathing)
Co~trolleaVersus ,lptient-InitiXte4 V_entilation .,.
Controlled • Ventilator delivers assistance independent of the patient's
ventilation spontaneous inspiratory efforts
• Ventilator delivers assistance in response to the patient's own
Patient-initiated inspiratory efforts
ventilation • lnspiratory effort of the patient may be sensed either by pressure
or flow-triggering mechanisms (see discussion below)
Pressure-Targeted, Y;~r,susVolume-Targeted Ventilation (in Positive-Pressure Ventilati_on)
Pressure-targeted • Airway pressure is set by physician (as the independent variable)
(pressure preset) • Tidal volume is the dependent variable
Volume-targeted • Tidal volume is set by the physician (as the independent variable)
(volume preset) • Airway pressure is the dependent variable

Ill. MODALITIES TO DELIVER VENTILATION


There are two basic methods of MV: noninvasive ventilation and invasive (or
conventional mechanical) ventilation
NON-INVASIVE VENTILATION I INVASIVE VENTILATION (MODES)
1. High-FlowNasalCannula(HFNC) 1.Assist/Control
Mode
2. Non-Invasive
Positive
Pressure Ventilation
(NIPPV) • Controlled
Mandatory Ventilation
(CMV)'
• CPAP(Continuous PositiveAirwayPressure) • Assisi/Control
(A/C)Ventilation
• BiPAP(BilevelPositiveAirwayPressure) A/C-VC
0 (Assist
Control,VolumeControlled)
NC-PC(AssistControl,Pressure
0 Controlled)
• Pressure-RegulatedVolumeControl(A/C-VC+)
2.Synchronized
lntenmittent
Mandatory
Ventilation
(SIMV)
• SIMV-VC (SIMV,VolumeControlled)
• SIMV-PC (SIMV,PressureControlled)
3. ContinuousSpontaneous Ventilation
• PressureSupportVentilation
(PSV)
• Continuous
PositiveAirwayPressure(CPAP)
4. AirwayPressureReleaseVentilation(APRV)
*Ventilatorswith pure ControlledMandatoryVentilation(CMV)are no longercommonlyencountered.
Instead,mostventilatorswill haveNC-VC as the basicmode.

294
HIGH-FLOW NASAL CANNULA (HFNC)
• HFNC oxygen delivery is an alternative means of respiratory support for critically ill patients
The apparatus comprises an air/oxygen blender, an active heated humidifier, a single
heated circuit, and a nasal cannula
It delivers maximal inspiratory flow without sacrificing dilution of oxygen in the inhaled gas
• Used extensively during the COVID-19 pandemic

I. SETTING UP THE HFNC


Nasal • An air/oxygen blender, allowing
Flow meter FiO2 from 0.21to 1.0,generates up to
6oL/minflow'
1';; Air-oxygen • Gas delivered is heated &
· blender humidified to allow laminar flow for
better tolerance to higher flow rates
• Washes out CO2 from the upper
airways, effectivelyreducing anatomic
dead space and work of breathing
• Patient breathes the adequately
Heated inspiratory circuit heated and humidified medical gas
through the nasal cannula
• Most effective when patient
breathes with mouth closed
'HFNCcandeliverupto 60 1pmfiow,unlikeothermodalities:
• Nasalprongs:upto 4 1pmonly
facemasks:upto 10 1pmonly
• Conventional
• Venturimasks:upto 15 1pmonly
Source:NishimuraM.Journalof IntensiveCare,2015

II. PHYSIOLOGIC EFFECTS OF HFNC


Reduction of anatomical dead space
• PEEP effect
• Constant fraction of inspired oxygen
Good humidification
I
III. SETTINGS
Initial device settings (e.g., inspiratory flow rate, FiO2%) are titrated qualitatively based
on patient's effort, use of accessory muscles & peripheral saturation
Initially thought to contribute to aerosolization ofSARS-CoV2, HFNC is now widely
used since its aerosolization profile is the same as regular face masks
Now advised to use a protective surgical mask over the HFNC prongs if using flow rates> 30 Umin
I
PARAMETER TITRATION I SETTING

• Titrate the flow rate according to patient's


Flow rate • Titrate from 15-60L/min
tolerance & comfort

• Titrate the FiOz based on the physician's


Fi02 • Titrate from ZI-100%
desired peripheral 02 saturation

JV.ADVANTAGES
Preconditioned gas enhances mucociliary function and improves airway clearance
Less anxiety & claustrophobia compared to noninvasive ventilation using masks/helmets
Can act as a CPAP providing PEEP as IO !pm offlow"' 1 cmH2O of PEEP
Associated with lower intubation rates for hypoxemic respiratory failure as compared to
regular 02 supplementation (but does not differ when compared to BiPAP/CPAP delivered
via facemask)
May have benefit for the following patients/conditions:
Acute hypoxemic (type 1) respiratory failure
0

Post-extubation
0

Post-surgery
0

Obese patients
0

295
V. THE ROX INDEX
Used in patients with pneumonia-related hypoxemic failure who are placed on HFNC
Useful clinical score in monitoring patients who might need eventual intubation
Computed at the 2nd, 6th and 12th hour after hooking the patient to HFNC
A deteriorating ROX index should alarm physician to prepare a patient for intubation
• Takes into consideration the RR & peripheral saturations with the level of02 support

Sp02 / Fi02 • ROX Index 2:4.88= low risk for intubation


ROX=
RR • ROX Index at high risk for intubation:
0<2.85 after 2 hours of HFNC
Sp0 2: peripheral 0 2 saturation on pulse oximetry(%) 0<3.47after 6 hours of HFNC
Fi02: fractionof inspired oxygen (e.g., for 21%, use 0.21) 0<3.85after 12hours of HFNC
RR: respiratoryrate (breaths per minute)
Sources: AmThorac
DrakeMG.Ann Soc,2018;RocaO ArnJ Respir
&CritCareMed,2019& FinkDL.AnnAmThoracSoc,2021

NON-INVASIVE POSITIVE PRESSURE VENTILATION (NIPPV)


Delivery of positive pressure ventilation to the lungs without endotracheal intubation
• Assisted ventilation is delivered via an interface (e.g., nasal or full-face mask or nasal pillow)

I. SETTING UP THE NIPPV


• Pressure support ventilation is delivered
through a noninvasive ventilator
Non-invasive ventilator • Gas flow is delivered via a single tubing
system with inspiratory & expiratory
flow sensors
• Different levels of PEEP and pressure
support can be applied
• A graphical waveform display provides
details about airway pressure, tidal
Gas delivery tubing volume, respiratory rate, and patient-
ventilator synchrony
Source: Neligan PJ. Anesthesiology Clin, 2012
II. MECHANISM
Provided by using a tight-fitting face or nasal mask or a helmet
Uses bilevel positive airway pressure ventilation ("BiPAP") & employs positive pressure
in two modes:
0 lnspiratory Positive Airway Pressure (IPAP): it is the set peak inspiratory pressure
during inhalation phase
0 Expiratory Positive Airway Pressure (EPAP}: it is the positive pressure that is set
during the exhalation phase
In contrast, Continuous Positive Airway Pressure (CPAP) uses only one set pressure
delivered throughout the respiratory cycle

III. INDICATIONS & CONTRAINDICATIONS


INDICATIONS/USES CONTRAINDICATIONS**

Cardiac or respiratory arrest


• COPD exacerbations'
Severe encephalopathy
• Acute cardiogenic pulmonary edema'
Severe GI bleed
• Immunocompromised patients'
Hemodynamic instability
• Facilitation of weaning in COPD patients'
Unstable angina and MI
• Asthma
Facial surgery, trauma, upper ainvay obstruction
• Cystic fibrosis
High-risk aspiration and/or those with
• Post-operative respiratory failure
inability to protect airways
• Advanced directives against intubation
Inability to clear secretions
'Conditions with strong evidence of benefit with NIPPV(COPD exacerbations & respiratory acidosis
pH <7.35 are those that benefit the most from NIPPV)
"If there are contraindicationsto NIPPV,invasiveventilationshould be used (e.g., may require intubation)
296
IV. SAMPLE CASE
A 61-year-old male COPD patient (weight 55 kg, height 167 cm), was admitted for progressive
dyspnea over the last 2 days. The patient has no fever, but with whitish sputum production. He
was seen in respiratory distress, tachypneic with diffuse expiratory wheezing. His vital signs
are as follows: BP 130/90, HR 106, RR 34, 02 sats 86% at 101pm via face mask.
Considerations Before Starting Non-Invasive Positive Pressure Ventilation (NIPPV):
Before starting NIPPY, make sure that there is an indication and the patient is a candidate for
NIPPY. First, establish the appropriate diagnosis with potential reversibility. Then, establish
need for ventilator assistance, and exclude patients with contraindications to NIPPY. The
patient has an indication forventilator assistance based on his presentation, and has a diagnosis
with good level of evidence for NIPPY (COPD exacerbation). NIPPY has its advantages, when
given appropriately, over invasive ventilation. This includes easy implementation, allowance
for intermittent application, improved patient comfort, and reduced need for sedation.

I. How will you initiate NIPPY?


Motivate & encourage patient, and explain thoroughly the risks/benefits of the procedure
• Place an appropriately fitting mask & start ventilation with the proper settings

2. What are the modes to choose from when setting up NIPPY?


MODE I DESCRIPTION
• Device triggers !PAP when flow sensors detect spontaneous inspiratory
S (Spontaneous)
effort and then cycles back to EPAP
T(Timed) • IPAP/EPAP cycling is purely machine-triggered at a set back-up rate
SIT • Like spontaneous mode, the device triggers to !PAP on inspiratory effort;
(Spontaneous/T=ed) but if patient fails to breathe, a machine-triggered breath will take over
For our case, spontaneous ventilationis an appropriate mode to initiatesince the patient is awake and has
intact breathing. SITor T modes may be used if a minimalset respiratoryrate is desired.

3. What are the parameters to be set when using NIPPY?


NIPPY is usually started at an !PAP of10 cmH2O, EPAP of5 cmH2O and an FiO2
equivalent to any supplemental oxygen the patient is already receiving
• Adjustments are made in the !PAP by increasing it in small increments based upon the
desired tidal volume (6-8 mL/kg), respiratory rate and ABG results
The !PAP is increased in small increments (keeping it at least 4 cm H2O above EPAP) to
the maximum pressure the patient can tolerate without discomfort and major air leaks
FiO2 and EPAP are titrated to achieve an oxygen saturation of >88-90%

4. How will you monitor patients on NIPPY?


PARAMETER I MONITOR FOR THE FOLLOWING

Subjective • Look for distress or discomfort


response • Synchrony of ventilation should be checked frequently

Physiologic • Drop in RR & HR in the first hour or two


response • Decrease in accessory muscle activity & abdominal paradox

• Continuous oximetry
Gas exchange
• ABG, ideally 30 mins after setting adjustment
Source: Jameson JL, et al. Harrison's Principlesof InternalMedicine20thedition,2018
Ambrosino N,et al. Eur Respir J 2008
Liesching T,et al. Chest 2003

297
INVASIVE MECHANICAL VENTILATION
Implemented once a cuffed tube is inserted into the trachea to allow conditioned gas
(warmed, oxygenated, and humidified) to be delivered to the airways and lungs at
pressures above atmospheric pressure
Process of using a device such as a ventilator to support, partially or totally, the delivery
of gas to the lungs
The ventilator exists to maintain the respiratory and metabolic functions of the lungs
until the patient recovers from illness
Breaths given by a ventilator are defined by four phases:
0 Trigger phase: how is the breath initiated?
0 Inspiratory phase (flow of gas into lungs): how the breaths get delivered?
° Cycle phase: how inspiration ends and expiration begins?
0 Expiratory phase: voluntary expiration of the patient

I. ENDOTRACHEAL INTUBATION
• The endotracheal tube provides an interface between the patient and the ventilator
Care must be made to ensure correct placement of the tube, maintenance of proper cuff
pressure, and suctioning to maintain a patent tube
After intubation, position of the tube must be confirmed by auscultation and a chest
radiograph (usually inserted to an average depth of 23 cm in men and 21cm in women)
• The ETT tip must be situated 2-3 cm proximal to the carina

II. GENERAL INDICATIONS FOR MECHANICAL VENTILATION


Primary indications for mechanical ventilation include:
0 Type I respiratory failure: peripheral saturation <90% or PaO2 <55 mmHg despite
Oz supplementation
0 Type II respiratory failure: high arterial pCO2 (usually >50 mm Hg) & arterial pH <7-25
Other indications include:
° Clinical deterioration, respiratory muscle fatigue, coma
Bradypnea or apnea with respiratory arrest
0 Inabiliry to protect airway from aspiration
0 Upper airway obstruction
0 Deteriorating neuromuscular disease
It is still the clinician's gestalt that is the best parameter for deciding to intubate and
initiate mechanical ventilation on a patient

Sources:BroaddusVC,et al. Murray& Nadel'sTextbookof RespiratoryMedicine7th Edition,2021


OwensW.The VentilatorBook2nd Edition,2018
Tobin,MJ. Principlesand Practiceof MechanicalVentilation3rd Edition,2013

III. DEFINITION OF TERMS


TERM I DEFINITION
Mode • Refers to manner in which ventilator breaths are triggered, cycled, & limited
• Defines what the ventilator senses to initiate an assisted breath
Trigger
• Either an inspiratory effort or a time-based signal

• Refers to the factors that determine the end of inspiration


• In volume-cycled ventilation, inspiration ends when a specific tidal volume
Cycle
(VT) is delivered
• Other rypes of cycling: pressure cycling and time cycling
• Operator-specified values (e.g., airway pressure) monitored by transducers
internal to the ventilator circuit
Limiting
• If the specified values are exceeded, inspiratory flow is terminated, and the
factors
ventilator circuit is vented to atmospheric pressure or the specified pressure
at the end of expiration (PEEP)
Source:JamesonJL,et al. Harrison's
Principles
of InternalMedicine
20thedition,2018
298
IV. SETTINGS IN MECHANICAL VENTILATION
REMARKS I USUAL VALUES/SETTINGS

Tidal Volume (V.,) ·;;

• "How much volume will the machine deliver?"in NC-VC • 8-10 ml/kg of ideal body weight
• It is the amount of gas that moves in and out of the • 6 ml/kg for ALI/ARDS
lungs with each breath, measured in ml • 10-12ml/kg for "conventionalset-up"
lnspiratory Pressure-(P,..,ro)
• "How much pressure will the machine deliver?"in NC-PC
• Set only in modes wherein pressure is controlled • Usual starting P""'"' is 14 cmH,O
a, ,..
Baak-up Rate (;BUR)or Respiratory Rate (RR)
• Minimum number of breaths per minute • 14-24breaths/minute
• Usually set 2-4 counts below the spontaneous rate • Faster RR is set to T exhalationof
(adjusted depending on the desired PaCO2 or pH) co,, leading to t Paco, & T pH
,. ,.
Oxygen Concentration (Fi02) ... '~
• Initially set at 100% unless there is evidence that a • Target values for 0 2 saturation:
lower Fi02 will provide adequate oxygenation 0 ~94%: if with heart disease/stroke

• Eventually down titrated based on desired P02 0 ~93%:for the general population

• Prolonged exposure to high 02 concentrations 0 ~88%:for patients with chronic

should be avoided (to avoid 02 toxicity) hypercarbia or COPD

Inspirator.y Elow Rate (lFR) " _,,.


• "How fast do we deliver the breath?" • Usuallyset at 40-60 L/minute
• This is the rate at which breath is delivered to the • Set at >60Uminin obstructivediseases
patient to achieve the set tidal volume (e.g., COPD,asthma) to allowlonBer
time for expiration(to regulate pC 2)
• The higher the flow rate, the lesser the inspiratory time • Target l:E ratio:
• Low !FR; most common cause of ventilator dyssynchrony 0 1:2 in the generalJo~ulation

(i.e., fighting the ventilator), so never set IFR <40 L/min 0 1:3 or longer in C P
Jnspiratory T,ime(111,.) ·'

• "How much time is spent in the inspiratory phase?" • Usual 1,m,:0.8-1sec


• Set only in modes wherein pressure is controlled • Deer.ease~ ,_ifwith CO, retention
• Too much I. can give you "inverse ratio ventilation" (l,m,1nCO B.0.5-0.7sec) .
(type of rec~ itment maneuver)
1 1
~ • Increase 16"" 1fyou are augmentingp0 2
--,
lnspirato~y Flow Pattern (IFP) ,.
• "How do you deliver the air?" • DeceleratinRwave: flowstarts at a
high level, t en tapers down
• How flow is distributed throughout the respiratory cycle • Square wave: keeps flowstable during
• A normal person has a sine wave pattern inspiratoryphase to open up atelectasis

-~
"'·
Positive End-Expiratory Pressure (PEEP)

• "Physiologic PEEP" of -5 cmH20 should be added, • Usual PEEP: 5 cmH20


regardless of Fi02, to prevent alveolar injury due to • Higher PEEP in ARDS
the shearing effect of opening & closing the alveoli
• LowerPEEP in hypotensive patients or
• Higher values (>to cmH20) can increase intrathoracic
pressures & impede venous return, reducing preload & CO withevidence of barotrauma

Trigger or Sensitivity ,
• Ranges from -5 to -0.5 cmH20 (pressure sensitivity) • Usuallyat -2.0 cmH,O (pressure
or r to 5 liters (flow sensitivity) sensitivity)or 2 L (flowsensitivity)
• Pressure versus flow sensitivity: • More sensitive (e.g., -0.5 cm or 1 L):
0 Pressure sensitivity: if set at -r, patient has to exert a easier for patient to trigger ventilator,
-r cm H20 pressure for ventilator to deliver the VT which may lead to hyperventilation
° Flow sensitivity: if set at t L, patient has to exert an • Less sensitive (e.g. -5 cm or 5 L):
air flow of at least t liter (better for COPD patients harder to trigger the ventilator,which
since it affords less work of breathing) may lead to increased workof breathing

299
MODES OF VENTILATION
• Modes differ based on trigger ("whatstartsinspiration?'),cycle ("whatendsinspiration?'),& the
controlled variables ('areyou goingtogiveafinite amountof volumeor afinite amount of pressure?')
Modes can also be differentiated by whether it facilitates controlled breaths, assisted
breaths, spontaneous breaths or a combination of each:
0 Assist/Control (NC) will deliver controlled breaths
0 SIMV has controlled & assisted breaths
0 Pressure Support Ventilation (PSV) (i.e., CPAP) will only have spontaneous breaths
• The other parameters optimize oxygenation (e.g., PEEP, FiO2) and augment ventilation
(RR, flow rate, or flow wave pattern)
Ventilator will only control the inspiratory limb of the respiratory cycle; the expiratory
limb can only be indirectly manipulated by adjusting inspiratory parameters (e.g., !FR,
inspiratory time, l:E ratio)
Source: Owens W.The VentilatorBook 2nd Edition,2018

I. ASSIST/CONTROL (A/C) MODES


Most widely used mode of ventilation & best used during initiation of ventilation
lnspiratory cycle is initiated by the patient's inspiratory effort or by a timer signal within
the ventilator (if no inspiratory effort is detected within a specified time window)
Patients receive a fixed tidal volume & inspiratory flow rate with each inspiratory effort,
regardless of their respiratory rate (RR)
A backup rate (BUR) is selected to guarantee that the patient receives a minimum
number of breaths per minute (i.e., if patient's RR falls below BUR, the ventilator
delivers the number of breaths set)

MODE
I REMARKS

• Patient breathes at his own rate & ventilator senses the


I Operator to
Set-up the ff:

inspiratory effort & delivers a controlled tidal volume ,VT


(volume controlled, volume-cycled) with each patient effort ,BUR
• If respiratory rate decreases below a preset rate, the • !FR
ventilator delivers tidal breaths at the preset rate • IFP
A/C-VC
• Since every breath is assisted, tachypnea may result in • PEEP
significant hypocapnia, respiratory alkalosis, and further • FiO2
hyperinflation can occur in CO PD/asthma, worsening • Sensitivity
the condition
• Often used as the initial mechanical ventilation settings
•p
• Patient breathes at his own rate & ventilator senses the controlled
•BUR
inspiratory effort & delivers a controlled inspiratory
•I ume orl:Eratio
pressure (pressure controlled, time-cycled) with each
A/C-PC .JFP
patient effort
• PEEP
• Often used if the patient has dyssynchrony with NC-VC
• FiO2
(e.g., obstructed patients) • Sensitivity
• A/C-VC+ is also known as pressure-regulated volume
control (PRVC) or pressure-targeted ventilation
• VT
• Patient breathes at his own rate & ventilator senses the
•BUR
inspiratory effort
• I.umc
A/C-VC+ • The ventilator calculates enough pressure it needs to
.JFP
orPRVC generate a preset volume
• PEEP
• The patient will receive different pressures breath-per-
• FiO2
breath depending on his respiratory effort
• Sensitivity
• May be maladaptive in patients with high inspiratory
drives as the ventilator will tend to give less assist
BUR:back-uprate I.,.:inspiratorytime
IFR:inspiratoryflowrate PEEP:positiveend-expiratorypressure
IFP:inspiratoryflowpattern
300
II. SYNCHRONIZED INTERMITTENT MANDATORY VENTILATION (SIMV)
Allows the patient to have spontaneous breaths in between the intermittent controlled
breaths beyond the set respiratory rate
Operator sets the number of mandatory breaths; between those breaths, the patient can
breathe spontaneously
IfBUR is set higher than spontaneous breaths, SIMV will work similar to A/C mode
The major difference between SIMV & NC is what happens when patient initiates a breath:
0In A/C, he gets the full tidal volume/pressure
0In SJMV, he gets whatever he can pull (usually with the help of pressure support)

MODE* I REMARKS
I Operator to Set-
up the ff:
• Patient has spontaneous breaths which are pressure-
supported in between mandatory volume-controlled • V./IFR
breaths set as backup rate •BUR
SJMV- • The spontaneous breaths work like PSV,while the • IFP
vc mandatory breaths work like A/C-VC • PEEP
• The spontaneous breath will have its unique volume/ • FiO2
flow rate and pressure breath-by-breath depending on • Sensitivity
the patient's lung mechanics and effort
• Patient has spontaneous breaths which are pressure-
supported in between mandatory pressure-controlled
• pcontrol /1.11me
breaths set as backup rate
•BUR
SIMV- • The spontaneous breaths work like,PSV, while the
• PEEP
PC mandatory breaths work like NC-PC
• FiO2
• The spontaneous breath will have its unique volume/
• Sensitivity,
flow rate and pressure breath-by-breath depending on
the patient's lung mechanics and effort
BUR:backup rate ltime:
inspiratorytime
IFR:inspiratoryflowrate PEEP: positiveend-expiratorypressure
IFP: inspiratoryflowpattern

'SIMVmode is usuallycoupled with PSV in modern ventilators.PSV ensures that even the
spontaneous breaths in SIMVmode are supported.
.. 2018
Source: Owens W.The VentilatorBook2nd Ed1t1on,

III. CONTINUOUS SPONTANEOUS VENTILATION


A. Pressure Support Ventilation (PSV)
0 Patient-triggered, flow-cycled, and pressure-limited
0 Patient receives ventilator assistance only when ventilator detects an inspiratory effort
0 Operator sets the pressure level (rather than the volume) to augment every

spontaneous respiratory effort


0 Inspiration is terminated when inspiratory airflow falls below a certain level
0 Operator to set-up: psupp,ort' EC)·de' PEEP, Fi02

B. Continuous Positive Airway Pressure (CPAP)


0 Not a true support mode of ventilation because all ventilation occurs through the
patient's spontaneous efforts
0Ventilator provides fresh gas to the breathing circuit with each inspiration
0Used to assess extubation potential in those who have been effectively weaned & in those
with intact respiratory function but require an endotracheal tube for airway protection
0Should not be confused with the non-invasive CPAP used for obstructive sleep apnea

301
IV. AIRWAY PRESSURE RELEASE VENTILATION (APRV)
A mode used to recruit collapsed lung units in very hypoxemic patients needing high
levels of PEEP (ARDS, pulmonary contusion, severe multilobar pneumonia)
• Works like a CPAP with intermittent discontinuation of pressure
• A problem with very high PEEP is one gets low alveolar ventilation & consequently high pCO2
0 If the lung is already fully inflated, it can no longer accommodate fresh gas coming in
0 The intermittent releases of this high PEEP will create a gush of air that enables
trapped CO2 to escape intermittently
Introduces new terminologies:
0 Ph; h: the pressure that is set high to recruit collapsed lung units (25-30 cmH2O)
0 P10:.: the airway pressure that the system drops to during the "pressure release" (0-5cmH2O)
0 Timeh;sh:time spent in Ph;h (4 secs)
Time 10w:time spent in P 10)0.4-o.8 secs) - the one adjusted depending on the
patient's flow rate
APRV uses longer periods of high pressure with intermittent, shorter drops to low pressure
Operator to set-up: ph;gh'plow'Timeh;gh'Timelow'FiO2

Source:Tobin,MJ.PrinciplesandPracticeof Mechanical
Ventilation
3rd Edition,2013
Samr,/e Case
A 45-year-old male (weight 50 kg, height 165 cm) was referred for a 3-day history of fever,
productive cough with yellowish sputum, and progressive dyspnea. Physical examination
showed a febrile patient with the following vital signs: BP 90/60, HR 120 bpm, RR 36/min,
T 39.0 °C, with 88% 02 saturation at IO 1pm via face mask. He had coarse crackles on the
right mid to lower lung fields, with diffuse expiratory wheezing on bilateral lower lung
fields. He also has a history of heart failure from hypertensive heart disease.
Diagnosis: Acute Respiratory Failure (ARF) Type 1 (Hypoxemic)
• From communiry acquired pneumonia (high risk) & decompensated heart failure
• V/Q mismatch is the most common cause ofhypoxemia
Settinq Up the Mechanical Ventilator
Initial mode of ventilation in acute respiratory failure is the A/C-VC mode. This ensures
adequate tidal volume & minute ventilation with a preset respiratory rate, and allows
room for optimal oxygenation.
• Tidal volume (VT)of 6-10 mL/kg: 300-500 mL of computed tidal volume means that the
patient will receive the preset volume in each respiratory cycle
•Back-uprate (BUR) of12-14 bpm: ensures a minimum respiratory rate of12-14.
Remember, in A/C mode, every breath delivered can be patient-triggered (patient
initiates the respiratory cycle) or machine-triggered (ventilator initiates the respiratory
cycle based on the set BUR if the patient does not initiate any breaths)
• Inspiratory Flow Rate (!FR) of 40-60 L/min: the !FR dictates how fast the preset tidal
volume is delivered during the inspiratory phase of the respiratory cycle. A value of 40-
60 L/min approximates the physiologic inspiratory-expiratory ratio.
• PEEP of 5-8 cm H2O: physiologic PEEP to keep the airways open after exhalation,
prevent atelectasis, & decrease shearing forces of alveolar distention during inspiration
• Fi Oz of woo/oinitially, then downtitrate depending on ABGs (desired Pa02, Fi Oz) and
peripheral 02 saturation

Principle of Protective Ventilatory Strategy•


•Seta target tidal volume close to 6 mL/kg of ideal body weight
• Prevent plateau pressure (static pressure in the airway at the end of inspiration)
exceeding 30 cmH2O
• Use the lowest possible fraction of inspired oxygen (FiO2) to keep the SaO2 at ~90%
• Adjust PEEP to maintain alveolar patency while preventing overdistention & closure/
reopening
*Basicgoalsof MVareto optimizeoxygenation
whileavoidingventilator-induced
lunginjurydueto
overstretch
andcollapse/re-recruitment
302
BASIC MONITORING AND TROUBLESHOOTING
I. THE VENTILATOR SCREEN
Modern ventilators are software-controlled and have interfaces used to generate useful
parameters for monitoring ventilated patients
Although ventilators may differ slightly in their interface, they invariably show three
panels: (1) monitored parameters, (2) scalars and graphs, and (3) ventilator settings
• The sample ventilator interface shown below is for a patient ventilated in A/C-VC mode

Real-timemeasurements are presentedonthe In thisexample,thesearethe settings


screenas patientbreathesthroughthe ventilator pluggedin bythe user:
• P.,.,.:peakpressure(highestpressureinsideairways) • A/C:assistcontrolmode
• P""": averagepressureattainedduringinspiration • V tidalvolume
• PEEP:positiveend-expiratory pressure • Vma,: flowrate
• 1:E:inspiratory:expiratoryratio(ratioof timespent • Te,:plateautime
duringinspiration/expirationper breathcycle) • P"": pressuresensitivity/trigger
• f,..:frequencytotal,theactualrespiratoryrategenerated • 0 2: fractionof inspiredoxygen(Fi02)
• VT£:exhaledtidalvolume,theactualvolumeof gas • PEEP:positiveend-expiratory pressure
expiredwitheachbreath
• v,10i minuteventilation(VTE x f101)
VC P·TRIG 85 kg
NC r-,;r.,., .. hop~

PeEAK
PMEAN
PEEP l:E fror Vre Veror f Vr Vw.x PSENS o,
20 ;;/. 58Q mL 50 ;;j;; 2.0 40 "
20 17 5 1:2 30 580 17.4 Te, .J'--.___ PEEP
Q.Q I RAMP 5

II. SCALARS & GRAPHS


Pressure, flow, and volume are plotted in real-time
• Useful in monitoring ventilated patients to check for leaks & obstructions at a glance

Normal Pressure, Flow, and Volume Scalars in AIC-VC Mode


PRESSURE SCALAR

• Breaths are triggered by patient's inspiratory


effort (negative pressure), as represented by
a miniscule downward deflection
Pc1Rc • This is followed by an increase in positive
-.-o pressure as a fixed volume of gas is
introduced in the lungs
FLOW SCALAR

• The inspiratory limb is above while the


~g..L'-:, _L', 1.41 expiratory limb is below the baseline
..,_
'vc1RC

CXP
fg
BO
V V • Must return back to baseline after each
BO breath cycle

VOLUME SCALAR

1000 • In A/C-VC, a fixed amount of tidal volume


is introduced into the lungs as shown
VrclRCffl~ by the identical volume scalars of each
.... 0
breath
VC,et al. Murray& Nadel'sTextbook
Source:Broaddus of Respiratory
Medicine
7thEdition,2021

303
III.ALARMS
A. High Pressure Alarms
0Occurs when too much pressure builds up at any time during the breathing cycle
0Must differentiate between obstructive and resistive causes of high pressures (the
pressure scalar can differentiate the two graphically)
0The pressure scalar is useful for monitoring airway and lung pressures, to check for
obstruction, and to monitor lung compliance

AIRWAY/OBSTRUCTIVE I RESTRICTIVE/
I PROBLEMS COMPLIANCE
PROBLEMS
• Tube problems: small ET size,
•ARDS
kinking/biting of the tube, right
• Multilobar pneumonia
Examples mainstem intubation
• Massive pleural effusions
• Airway problems: mucus plug,
• Tension pneumothorax
severe bronchospasm
• Check the corrugated tubes up
to the ET tube for kinks • Proning & diuresis for
• Suction secretions regularly ARDS
Troubleshooting • Administer bronchodilators • Taps for effusions
(e.g., salbutamol) • Needling for
• Decrease the back-up rate, tidal pneumothorax
volume or flow rate

I
rr=s=
p~ P,,a1

(Baseline/normal) (Airway resistance) (Decreased compliance)

• Elevated static P•'"


• Normal static P•'" (<30 cmHzO)
(>3ocmH2O)
• Wide gap between P•"' and PP'" • Normal gap between P.,,,
(>10-15cmHzO)
and P ,,,

B. Low Pressure Alarms


Occurs when there is a sudden drop in the circuit pressure
0

Almost always due to a problem with the tubes (e.g., interrupted corrugated cubes,
0

open device/suction ports, misplaced endotracheal tube, accidental extubation)


Troubleshooting:
0

• Check for tube connections and open ports


• Check for level of ET and tip of ET on chest radiographs (reposition as needed)
• Tube exchange and reintubation as a last resort

Source:MellemaMS.Ventilatorwaveforms.
TopCompanion
AnimMed.2013

304
SPONTANEOUS BREATHING TRIAL (SBT) & WEANING
It is important to consider discontinuation of MV once the underlying respiratory
disease begins to reverse
A simpleflowcharttoguidethe dailyapproachin weaningoff mechanicalventilationis shownbelow:

Daily wean screen


Resolving disease
Adequate gas exchange
Stable hemodynamics
Spontaneous breathing ability

Yes No

SBT
Continue MV
1----------Fa_,_·l_.i Treat reversible elements

Pass

Assess for extubation


Fail
Repeat daily screen

Pass

Consider
Extubate
tracheostomy

Source:JamesonJL,et al. Harrison's


Principles
of InternalMedicine
20thedition,2018 ·

I. WEANING FROM MECHANICAL VENTILATION


Weaning is the process of withdrawing MV support & transferring the work of breathing
from the ventilator back to the patient (weaning is not synonymous with excubation)
Need for MV is not the same as the need for an artificial airway

A. Definition of Terms
TERM I DEFINITION
Weaning • Patient is able to maintain spontaneous breathing for 48 hours
success following extubation
• Weaning failure is the inability to maintain adequate respiration
through an artificial airway
Weaning • Failure of a spontaneous breathing trial or the need for reintubation
failure within 48 hours following extubation
• Work of breathing cannot be sustained independently by patient, hence
not ready to be liberated from the ventilator (caused by a lot of factors)
• Inability to maintain adequate respiration after removal of artificial
airway
• Defined as the patient not being able to independently ventilate
Extubation
without an artificial airway (e.g., endotracheal tube, tracheostomy
failure
tube), which often leads to reintubation
• Common causes include upper airway obstruction, aspiration, or
inability to clear pulmonary secretions
30S
B. Prerequisites for Weaning
0 Improvement of respiratory failure & reversal of underlying cause
0 Absence of major organ system failure
0 Appropriate level of oxygenation
0 Adequate ventilatory status
0 Intact airway protective mechanism (needed for extubation)
0 Hemodynamic stability without vasopressor or inotropic support
° Clear or acceptable chest X-ray
0 Afebrile, normal WBC, or other indications of resolved or resolving infection
0 Requiring :;30% FiO2 to achieve target saturation set
0 :s20 cmH2O to achieve TV of7-15 mL/kg of pre-injury body weight
0 PEEP :;5cmH2O
0 Unsupported VC ;,150 mL
0 Rapid shallow breathing index or RSBI (ratio of respiratory rate to tidal volume) <105

Source:MacIntyre
NR,et al. Chest2001
C. Methods ofWeaning
METHOD I DESCRIPTION

• Abrupt discontinuation of MV and resumption of spontaneous breathing


through a T-tube system (simplest method of weaning)
• Best tolerated by patients who have undergone MV for brief periods and
T-piece
require little respiratory muscle reconditioning
Weaning
• Initiated for 5 minutes/hour followed by a 1-hour rest interval
• Trials are increased by 5 to IO min/hr increments until patient can remain
ventilator-independent for 60-120 ruins, & then extubation can be attempted

• Mandatory back-up rate is decreased by 2 to 4 breaths per minute while


monitoring blood gas parameters and respiratory rates until :;4-6 breaths per
minute are achieved
• Best for patients intubated for extended periods who are likely to require
SIMV
gradual respiratory muscle reconditioning
• Rares >25/min on withdrawal of mandatory ventilator breaths indicate
respiratory muscle fatigue & need to combine periods of exercise with rest
• Shifting to PSV,CPAP or T-piece trial can then be attempted before extubation
• Patient controls the rate, depth and timing of each breath (i.e., patient-
triggered, pressure-limited, flow-cycled)
CPAPor • Gradual reduction in the level of PSV (prevents activation of accessory muscles)
PSV • Gradual decrease on regular basis (hours or days) to minimum of 5-8 cmH2O
• Once the patient is capable of maintaining the target venti!atory pattern and
gas exchange at this level, MV is discontinued

D. Failure to Wean and Failure ofExtubation


CAUSES OF FAILURE TO WEAN I CRITERIA FOR FAILURE OF EXTUBATION
• Weaning to exhaustion • RR >25breaths/min for 2 hours
• Auto-PEEP • HR >140beats/min or sustained increase/
• Excessive work of breathing decrease of >20%
• Poor nutritional status • Clinical signs of respiratory muscle fatigue
• Overfeeding or increased work of breathing
• Left heart failure • SaO2 >90%; PaO2 <80 mmHg on FiO2 ;,0.50
• Decreased Mg'• & phosphate levels • Hypercapnia (PaCO2 >45 mmHg or ;,20%
• Infection/fever from pre-extubation), pH <7-33
• Major organ failure (i.e., CNS damage)
• Technical limitations
• Residual effects of sedatives
Sources:MacIntyre NR,et al. Chest2001
JamesonJL,et al. Harrison's
Principles
of InternalMedicine20thedition,2018
306
II. SPONTANEOUS BREATHING TRIAL (SBT)
Consists of a period of breathing through the endotracheal tube without (or with
minimal) ventilator support
Both CPAP of 5 cmH2O & an open T-piece breathing system can be used for 30-120 ruins

A. If the following are present, patient has passed the screening test & should undergo SBT:
0 Stable oxygenation (PaO2/FiO2 > 200) and PEEP :s5cmH2O
° Cough and airway reflexes intact
0 No vasopressor agents or sedatives being administered
0 Patient capable of spontaneous breaths

B. SBT is declared a failure and stopped if any of the following occur:


0 RR >35/min for >5 minutes
0 02 saturation <90%
0 HR >140/min or a 20% increase/decrease from baseline
0 Systolic BP <90 mmHg or >180 mmHg
0 Increased anxiety or diaphoresis

If at the end of the spontaneous breathing trial, the rapid shallow breathing index
(RSBI or ratio of the respiratory rate to the tidal volume in liters, f/VT) is <l05, the
patient has higher chances of successful extubation.

Source: Jameson JL, et al. Harrison'sPrinciplesof InternalMedicine20th edition,2018

III. CUFF-LEAK TEST


Assessing the presence of air movement around a deflated endotracheal tube cuff
Done when upper airway difficulty is suspected
If the "cuff-leak test" suggests a risk of post-extubation stridor, the administration of
systemic corticosteroids should be considered prior to extubation

Performing the Cuf /-Leak Test


• Set the ventilator to A/C-VC
• Suction tracheal and oral secretions for accurate measurements
• Take the mean of three exhaled tidal volume measurements when the cuff is inflated &
take the mean of three exhaled tidal volume measurements when the cuff is deflated

Leak volume = • The leak volumeshould be >110ml


Exhaled VT (inflated) - Exhaled VT (deflated) for extubationto proceed

307
SECTION THREE
. · l«DVANCED CARt>IAC UFE SUPPORT (ACLS)
ADULT CARDIAC ARREST ALGORITHM
ACLS is typically carried out in the hospital setting where a cardiac monitor and other equipment and drugs
are easily accessible. Asystole, PEA, ventricular fibrillation (VF), and pulseless ventricular tachycardia
(pVT) are all considered arrest rhythms for which ACLS should be done. Defibrillation shocks are only given
for VF and pVT. Every 2 minutes, chest compressions are paused to check the rhythm on the cardiac monitor.
The airway should also be secured via intubation. ACLS is continued until return of spontaneous circulation
(ROSC) or until patient is pronounced expired.

Start CPR
Give oxygen
Attach manitar/delibrillator

No
Rhythmshockable?

Asystole/PEA
Epinephrine
CPR2 min CPR 2 min
• IV/10 access IV/10 access
Epinephrineevery 3-5 min
Consider advanced airway, copnography

~------~------~No ~------~------~Ye,
Rhythmshockable? Rhythmshockable?

0 Yes - perform shock


Dl--------~-------,
No
CPR 2 min CPR2 min
Epinephrine every 3-5 min
Treat reversible causes"'
Consider advanced airway, copnography

~-------'-------~ Yes
Rhythmshockable? Rhythmshockable?
@ Yes - perform shock Na
Go ta A or B
CPR2 min
Amiodorone or lidocoine
Treat reversible causes"'

If no signsof return of spontaneouscirculation(ROSC)"'"'go to C or D


If ROSC, go to Post-CardiacArrest Core
Consider appropriateness of continued resuscitation

·Reversiblecausesinclude:hypovolemia,hypoxia,acidosis,hypo/hyperkalemia,
hypothermia,
tension
pneumothorax,tamponade, toxins,thrombosis
··Returnof Spontaneous Circulation(ROSC):pulse& bloodpressure;spontaneous
arterialpressurewaves
withintra-arterial
monitoring

CPRquality:
Pushhard(at least2 inches)andfast(100-120/min)andallowcompletechestrecoil
• If no advancedairway,30:2compression-ventilation
ratio(or 1 breathevery6 seconds)
• If withadvancedairway,give 1 breathevery6 seconds(1Obreaths/min) withcontinuouscompressions
Drugtherapy:
• EpinephrineIV dose:1 mgevery3-5 minutes
• AmiodaroneIV dose:300mg bolus(firstdose),then150mg(asseconddose)
• LidocaineIV dose:1-1.5mg/kg(firstdose),then0.5-0.75mg/kg(asseconddose)
Source:Panchal
AR,et al.2020AHAGuidelines
forCPRandEmergency
Cardiovascular
Care.Circulation.
2020
308
ADULT BRADYCARDIA ALGORITHM
Symptomatic bradycardia,with heart rate typically <50 bpm in the presenceof symptoms, is identified&
treated accordingto the underlyingcause. Reversibleetiologiesfor the bradyarrhythmiashould be worked
up. If bradycardiais unstable(hypotension,alteredsensorium,chestpain or acute heartfailure), intravenous
atropineis thefirst-linedrugfor intervention.If atropineis ineffectiveevenaftermaximumdose(3 mg)has been
reached,optionsincludedopamineorepinephrinedrips& transcutaneous pacing(orstandbytransvenouspacing).

Assess appropriateness for clinical condition.


Heart rate typically <50/min if bradyarrhythmio.
j

Identify and treat underlying cause


Maintain airway and assist breathing, as needed
Cardiac monitor and/or ECG to identify the rhythm
Obtain IV access

l
Persistent bradyarrhythmia causing
No Hypotension? lschemic chest discomfort?
Monitor and observe
I Acutely altered mental status? Acute heart failure?
Signs of shock?
! Yes
Atropine 1 mg IV bolus (first dose), then repeat every 3-5 minutes
(maximum of 3 mg)
If atropine ineffective, consider dopamine or epinephrine infusion, and/or
tronscutaneous pacing
Refer to Cardiology

ADULT TACHYCARDIA WITH A PULSE ALGORITHM


Usedfor patients who have marked tachycardia(usually >I50 bpm) and a palpablepulse.If the tachycardia
is unstable (hypotension, altered sensorium, chest pain or acute heart failure), synchronized electrical
cardioversion(ideally under light sedation) is necessary.If patient is stable, pharmacologiccardioversion · :
can be utilized.Forregularnarrow-complextachycardia,adenosinein escalatingdosesis thefirst-lineagent; ·
otherwise,IV beta-blockers,non-dihydropyridinecalciumchannelblockers,or amiodaroneshould be used.

Assessappropriatenessfor clinicalcondition.
Hearl role typically~150/min if lochyorrhythmio.
I
Identifyand treat underlyingcause
Oxygen as needed
Cardiac monitorand/or 12LECGlo identifythe rhythm
Obtain IV access

I
*
Synchronizedcardioversion Persistent
tachyorrhythmiacausing
Consideradenosinefor regular Hypotension? lschemicchestdiscomfort?
narrow QRS tachycardia Acutelyaltered mentalstatus? Acuteheart failure?
Referlo Cardiology Sians of shock?
j No
Antiarrhythmicinfusion(e.g., Yes
omiodorone) Wide QRS ~0.12 second
Referto Cardiology
I No
Vogel maneuvers(if regular)
Adenosine jfor SVT):6 mg rapid IVpush, followed by NSS flush
!second dose al 12 mg, if needed)
Beta blockeror calciumchannelblocker
Referlo Cordioloay

'Synchronizedcardioversion:
Forregular,narrowQRStachycardia:
50-100Joules
Forirregular,narrowQRStachycardia:
120-200Joules
Forregular,wideQRStachycardia:
100Joules
Forirregular,wideQRStachycardia:
considerdefibrillation
309
ADVANCED THERAPEUTIC MODALITIES
MODALITY I DESCRIPTION
• A long cylindrical balloon is inserted through the femoral artery &
positioned between the aortic arch vessels & the level of the renal arteries
• Counterpulsation: balloon inflates during diastole (improves coronary
perfusion) and deflates in early systole (decreases afterload)
• Primary goal: improve ventricular performance in the failing heart by
Intra-Aortic
increasing myocardial oxygen supply & decreasing oxygen demand
Balloon Pump
• Indications: AMI (especially if with mechanical complications like VSR or
(IABP)
acute MR), cardiogenic shock, high-risk PC!, or LV failure
• Contraindications: aortic regurgitation, aortic dissection, AAA, post-EVAR
or -TEVAR, severe PAD
• Complications: malpositioned balloon causing cerebral or renal compromise,
limb ischemia, thromboembolism, vascular injury, balloon rupture/entrapment
Left • A mechanical pump implanted in the LV to facilitate pumping of blood
Ventricular • Goals for LVAD implantation can be any of the following: bridge to
Assist Device transplant, bridge to candidacy, bridge to recovery, or destination therapy
(LVAD) • Complications: bleeding, device thrombosis, stroke, driveline infections
• Blood is drained from the patient, & undergoes oxygenation & removal of
CO2 via an artificial membrane system, before being returned to the patient
• Requires cannulating large vessels of the patient's body
• Veno-venous (VY) versus veno-arterial (VA) ECMO
0 VY ECMO: primarily just for respiratory support for hemodynamically
Extra corporeal stable patients with ARDS, pulmonary hemorrhage, other pulmonary
Membrane conditions needing lung rest, or as a bridge to lung transplant
Oxygenation 0 VA ECMO: provides both cardiac & pulmonary support, particularly for
(ECMO) unstable patients with refractory cardiogenic shock from any number of
etiologies (e.g., ACS, fulminant myocarditis, post-cardiac surgery), or as
a bridge to LVADor heart transplant
• Complications: hemorrhage (needs continuous anticoagulation to prevent
thrombosis of the ECMO circuit), infection, and need for frequent blood
transfusions (due to hemolysis and consumptive thrombocytopenia)
• Same indications as RRT: volume overload, acid-base or electrolyte
abnormalities, uremia with encephalopathy or pericarditis, persistent/
progressive AKI
Continuous • Better hemodynamic tolerance in unstable patients because of slower fluid
Renal removal & solute clearance over prolonged treatment (ideally 24 hrs/day)
Replacement • Easier control of volume status with less concern of fluid overload from
Therapy administration of medications or enteral/parenteral nutrition
(CRRT) • It is an inefficient form ofRRT, thus would not be considered optimal when
the goal is rapid removal of a dangerous solute (e.g., K·, ingested toxin)
• Efficientonly when applied through the 24-hr period with minimal interruption
• Requires continuous anticoagulation to prevent circuit thrombosis
• Hybrid modality that uses conventional intermittent HD machines with
Sustained an extension of therapy to 8 to 12hours, with the aim of achieving the
Low Efficiency hemodynamic benefits ofCRRT
Dialysis • Slow ultrafiltration maintains utility for unstable/critically ill patients, but
(SLED) principally makes use of diffusive solute clearance
• Less expensive and demanding on nursing staff (compared to CRRT)
• Adsorbent therapy where blood circulates through an extracorporeal circuit
equipped with a cartridge onto which the target molecule can be adsorbed
Hemoperfusion • Has been used in some cases of poisoning, cytokine storm of septic shock &
(HP) SARS-CoV-2 infection, removal of antibodies & antibody-antigen complexes
in autoimmune disorders, & removal of hepatic toxins in liver failure
• Can be done alone or in combination with HD or CRRT
310
INFECTIOUS
DISEAS
SECTION ONE
APPROACH TO COMMON ·coMPLAIN1S
FEVER
• The mean oral temperature in healthy adults is 36.8 °C +/- 0.4 °C, with lower levels in
the morning and higher levels towards the afternoon
The definition of fever is a temperature of:
0 >37,2°C in the morning
0 >37-7°C in the afternoon/evening
Rectal temperatures are generally 0.4 °C higher than oral temperature readings

I. GENERAL APPROACH TO FEVER

CHARACTERIZE FEVER
Temperature measurement, duration, occurrence, frequency and pattern,
oggrovoting end relieving factors, associated symptoms

l
ASSESS PATIENT
Age, presence of immunocompromised stole, any drug intake,
exposure to other people with the some illness, exposure to animals or vectors of disease,
physical examination suggesting potential focus of infection

BASIC LAB EXAMS


l
CBC with differential count is the most basic laboratory exam.
Other laboratories will depend on potential diagnostic clues based on history end PE.

Sources:Mackowiak PA,et al.JAMA.1992


JamesonJL,et al. Harrison's
Principles
of InternalMedicine
20thedition,2018

II. FEVER PATTERNS


PATTERN I ILLUSTRATION I DESCRIPTION I DIFFERENTIALS

Continuous/
sustained

Intermittent
~-I

rn=
-•-••-••

--•--
,_H_<
I

--+----•
------·-1 -----
I

-••-
---
• Minimal variation
(<0.3 °C per day)
• Does not remit

• Temperature returns
to normal between
• Viral infections
• Lobar pneumonia
• Drug fever

• Malaria (tertian)
• Tuberculosis*
episodes

• Temperature returns • Dengue (initial


Relapsing/
to normal for days phase)
biphasic
before rising again • Leptospirosis

• Variation in
temperature >0.3 °C • Typhoid fever
Remittent
without normalizing • Tuberculosis*
• Stepladder pattern

• Very wide
Hectic/
fluctuations of • Sepsis or bacteremia
septic
temperature per day

*Feverof tuberculosis
usuallypresentdaily,mainlyin theevening,butusuallynot>37.8 °C

313
FEVER OF UNKNOWN ORIGIN (FUO)
• Any febrile illness without an initially obvious etiology
The term FUO should be reserved for prolonged febrile illness without an established
etiology, despite intensive evaluation & diagnostic testing

FUO • Temperature 2:38.3°Con several occasions


(Petersdorf &
• Duration of fever >3 weeks on two occasions
Beeson Definition
in1961)
• Uncertain diagnosis despite I week of inpatient investigation

Recent·J)efinitions
• Fever >38.3 °Con at least two occasions
• Illness duration of 2:3weeks
• No known immunocompromised state
Recent • Diagnosis uncertain after a thorough history, physical exam, & the
definition of following obligatory investigations: ESR, CRP, platelet count, leukocyte
FUO count & differential, hemoglobin, electrolytes, creatinine, total protein,
alkaline phosphatase, ALT, AST, LOH, creatine kinase, ferritin, ANA,
RF, protein electrophoresis, urinalysis, blood culture (3 sites), urine
culture, CXR, abdominal ultrasound, and tuberculin skin test

• Fever >38.3 °C
Classic • Illness duration of 2:3weeks
• Evaluation for at least 3 outpatient visits, or 3 inpatient days

• Fever >38.3 °C
Nosocomial • Hospitalized for 2:24hours but with no fever during admission
• Evaluation for at least 3 days

• Fever >38.3 °C
Neutropenic • Neutrophil count,; 500 per mm'
• Evaluation for at least 3 days

• Fever >38.3 °C
HIV-
• Duration of >4 weeks as outpatient, or >3 days as inpatient
associated
• Confirmed HIV infection
Sources:Petersdorf RF,et al. Medicine;1961
JamesonJL,et al. Harrison'sPrinciples
of InternalMedicine20thedition,2018
BennetJE,et al. Principles
andPracticeof Infectious
Diseases,8thedition,2014

APPROACH TO FEVER AND RASH


Fever with a rash is a common presentation
• Skin manifestations may provide the only early clue to an underlying infection

I. BACTERIAL INFECTIONS PRESENTING AS FEVER & RASH


DISEASE I ETIOLOGY I DESCRIPTION

• Classic dermatologic manifestations include


Rheumatic • Group A
erythema marginatum and subcutaneous
fever Streptococcus
nodules over bony prominences

• Leptospira • Rash may be macular, maculopapular,


Leptospirosis
interrogans erythematous, petechial or ecchymotic

• Rose spots: faint, salmon-colored, blanching


Typhoid
fever
• Salmonellatyphi maculopapular rash usually seen on the trunk
and chest and lasting for 2-5 days
314
DISEASE I ETIOLOGY I DESCRIPTION
• Initially pale red or pink macules on the trunk
and proximal extremities which progress to
Secondary papular lesions usually on the palms and soles
• Treponemapallidum
syphilis • Condyloma lata: pa pules that enlarge to form
broad, moist, pink or gray-white highly infectious
lesions, usually in warm intertriginous areas

• Group A • Rash is made up of minute papules giving a


Scarlet fever Streptococcus "sandpaper" feel of the skin
(2nd disease) (pyrogenic • May also develop Forschheimer spots
exotoxin A, B, C) • Pastia's lines: accentuation ofrash in skin folds
• Group A Streptoaxcus
Streptococcal
(pyrogenic exotoxin
toxic shock • Scarlatiniform rash
A &/or B or certain
syndrome
M types)

• S. aureus
Staphylococcal • Erythroderma of variable intensity
(toxic shock
toxic shock • Desquamation of the skin occurs during
syndrome toxin 1,
syndrome ·convalescence
enterotoxin B or C)
Staphylococcal • Localized blister formation & exfoliation of skin
• S. aureus, phage
scalded-skin • Nikolsky's sign: rupture of the lesions with gentle
group II
syndrome pressure
Pseudomonas
• Pseudomonas • Pruritic erythematous follicular, papular,
"hot-tub"
aeruginosa vesicular, or pustular lesions
folliculitis

• Macular rash usually presents on the 5th day


Epidemic
• Rickettsiaprowazekii • Begins on the upper trunk and becomes
typhus
generalized, sparing the face, palms, and soles

• Initial macular rash usually detected on the


Endemic axilla or inner arm
• Rickettsiatyphi
(murine) typhus • Eventually presents as maculopapular rash later
in the course & involves the trunk & extremities
• Orientia • Maculopapular rash & presence of characteristic
Scrub typhus
tsutsugamushi eschar (site where the chigger has fed)
• Initially pink maculopapular lesions and later
evolves into non-blanching petechial rash
usually on the trunk and extremities
Acute • Purpura fulminans: large ecchymoses with
meningococcemia sharply irregular shapes evolving into
• Neisseria hemorrhagic bullae and then into black necrotic
meningitides lesions (reflects disseminated intravascular
coagulation)

• May have pink maculopapular, nodular,


Chronic
petechial and purpuric lesions with pale blue-
meningococcemia
gray centers; recurrent

• Typically begins as a papule which evolves to


Anthrax • Bacillusanthracis a painless vesicle, then later on to a coal-black,
necrotic eschar

315
II. VIRAL INFECTIONS PRESENTING AS FEVER & RASH
DISEASE
I ETIOLOGY
I DESCRIPTION

• Begins as erythematous macules behind the ears, neck


& hairline then progresses to face, trunk, & extremities
Rubeola: • Usually spares palms and soles
measles • Paramyxovirus • Koplik's spots (pathognomonic of measles): bluish
(1st disease) white dots surrounded by erythema on the buccal
mucosa opposite the lower molars during the first 2
days of symptoms
i
• Rash similar to measles, however the patient does
not appear to be as sick
Rubella: • Spreads from hairline downward, with clearing as
German measles • Togavirus it spreads
(3rd disease) • Usually with lymphadenopathy and arthritis
• Forschheimer spots: non-specific petechiae on the
soft palate in 20% of patients
• Erythematous macular rash that spreads to the
Erythema • Human
extremities in a lacy reticular pattern
infectiosum parvovirus
• May be accompanied by arthritis
(5th disease) B19
• "Slapped-cheeks" disease & "gloves-&-socks"syndrome
Roseola:
• Human
exanthem subitum • Diffuse maculopapular rash over trunk and neck
herpesvirus 6
(6th disease)

• Macules evolving into papules then vesicles on an


Varicella:
erythematous base (the classic "dew on a rose petal")
chickenpox • Varicella- • Lesions are in different stages of development
zoster virus
(VZV) • Caused by reactivation of latent VZV
Herpes zoster
• Vesicular lesions distributed along a dermatome and
(shingles)
ending at midline
• Morbilliform or papular rash usually on the arms
Infectious • Epstein-Barr
and trunk
mononucleosis Virus
• May also develop Forschheimer spots
• Herman's rash: erythematous, maculopapular rash
• Dengue virus beginning on the trunk & spreading to the extremities
Dengue fever (4 serotypes; and face, with islands of normal skin spared from the
flavivirus) rash and described as "isles of white in a sea of red"
• Rash usually presents near the time of defervescence
• Maculopapular rash on upper extremities and face,
Chikungunya • Chikungunya
appearing at the time of defervescence
fever virus
• With prominent arthritis
Zika virus • Itchy maculopapular rash with low-grade fever,
• Zika virus
disease headache and malaise
Primary herpes
• Painful grouped vesicles that may progress to
simplex virus • HSV
pustules and ulcerate
(HSV) infection

• Coxsackie
Hand-foot-and- • Painful vesicles in the mouth
virus A16
mouth disease • Papules on hands and feet
(mostcommon)
Source:JamesonJL,et al. Harrison's
Principles
of InternalMedicine20thedition,2018
LopezFA,et al. Feverandrashin the immunocompetent
patient.Availableonlinein Uptodate;
2021
316
SECTION TWO
OVERVIEW OF AVAILABLE ANTIMICROBIALS
This section summarizes the common antimicrobials available and their indications. Based on
the Philippine National Drug Formulary and the Antimicrobial Stewardship (AMS) Program
in Hospitals Manual of Procedures, the following antimicrobials will need prior authorization
by an infectious disease specialist or AMS physician (i.e., restricted antimicrobials): 4th
generation cepha/osporins & higher, aztreonam, carbapenems, polymyxins, vancomycin,
linezolid, and all intravenous antifungals (except fluconazole).

ANTIBACTERIAL AGENTS
I. BETA-LACTAMS
Broad-spectrum antibiotics (i.e., agents with a B-lactam ring) that work by inhibiting cell
wall synthesis
These antibiotics are generally bactericidal against susceptible organisms
Beta-lactamase inhibitors impede plasmid-encoded and chromosomal beta-lactamases,
which then restores the antibacterial activity of the antibiotic
EXAMPLES I ORGANISMS COVERED I COMMON DOSAGES
PENICILLINS:inhibit cell wall synthesis by binding to penicillin binding proteins (PBP)
• Primary syphilis:
• Highly effective against gram· Penicillin G 2.4 million
Penicillin G (IV) positive cocci, except penicillinase· units IM as single dose (SD)
Penicillin V (oral) producing bacteria, meningococci, • Prophylaxis for recurrent
spirochetes, anaerobic cocci rheumatic fever:
Penicillin V 250 mg PO BID
Anti-
staphylococcal
penicillins:
• Active against most penicillinase· • Nonpurulent cellulitis:
• Nafcillin
producing staphylococci Cloxacillin 500 mg PO QID
• Oxacillin
• Cloxacillin
• Dicloxacillin
Aminopenicillins: • Effective against gram-positive
• Nonpurulent cellulitis:
• Ampicillin cocci, enterococci and Listeria
Amoxicillin 500 mg PO TIO
• Amoxicillin monocytogenes
• Active against B-lactamase-
producing strains of staphylococci,
gonococci, H. influenzae,M. catarrhalis,
Beta-lactam/
Bacteroides,Klebsiellasp. & E. coli • Acute bacterial
beta-lactamase
• Ticarcillin-clavulanate and exacerbation of chronic
inhibitor
piperacillin-tazobactam are anti· bronchitis:
combination
pseudomonal penicillins Co-amoxiclav t g PO/IV BID
(BL/BLIC):
• Ticarcillin: active against • Bone & joint, skin & skin
• Amoxicillin-
P. aeruginosa,P. vulgaris,Providencia, structure infections:
clavulanate
Morganellaand Enterobactersp. Ticarcillin-clavulanate
• Ampicillin-
but less potent than the extended· 3,2g!Vq4-q6
sulbactam
spectrum penicillins against • Severe infections/
• Ticarcillin-
streptococci and enterococci nosocomial pneumonia:
clavulanate
• Piperacillin: combined Piperacillin-tazobactam
• Piperacillin-
characteristics of the extended- 4.5 g IV q6
tazobactam
spectrum penicillins and ticarcillin
• These BL/BLIC antibiotics also have
anaerobic coverage
317
• Active against most gram-positive • Surgical prophylaxis
cocci including penicillin-resistant (e.g., cardiovascular,
S. aureus but not against biliary tract, esophageal,
1st generation:
enterococcus, methicillin-resistant appendectomy or
• Cefazolin
S. aureus (MRSA) and methicillin- laparoscopic surgery):
• Cefalexin
resistant S. epidermidis(MRSE) Cefazolin 1-2 g IV pre-op SD
• Cefadroxil
• Also active against some gram- • Respiratory tract
negative bacilli: E. coli,K. pneumoniae infections:
and P. mirabilis Cephalexin 250 mg PO q6
• Cefoxitin: resistant to
beta-lactamase-producing
gram-negative bacilli
2nd generation: • Surgical prophylaxis
• Improved activity against
• Cefoxitin for non-perforated
H. influenzae,M. catarrhalis,Neisseria
• Cefaclor appendectomy:
meningitidesand N. gonorrhea
• Cefuroxime Cefoxitin 1-2 g IV pre-op
• Enhanced activity against
• Cefamandole SD
staphylococci, non-enterococci,
• Cefotetan • Pharyngitis/tonsillitis:
streptococci, & some Enterobacteriaceae
• Cefprozil Cefuroxime 250 mg PO q12
• Cefoxitin, cefotetan and cefmetazole
• Cefmetazole for IO days
are classified under the subgroup of
cephamycins - cephalosporins active
against anaerobic bacteria
• Effective against S. pneumoniae,
3rd generation:
S. pyogenes and other streptococci
• Ceftriaxone
(with the exception of ceftazidime) &
• Ceftazidime
modest activity against methicillin- • CAP, moderate risk:
• Cefixime
sensitive S. aureus (MSSA) Ceftriaxone 1-2 g IV q24
• Ceftizoxime
• Excellent activity against • CAP, high risk and with
• Cefpodoxime
N. gonorrhea,H. influenzae,M. risk for P. aeruginosa:
proxetil
catarrhalisand Enterobacteriaceae Ceftazidime 1-2 g IV q8
• Cefotaxime
• Ceftazidime has activity against
• Cefoperazone
P. aeruginosa,with poor activity
• Moxalactam
against gram-positive bacteria
• Active against both aerobic
gram-positive (but not MRSA) and
• CAP, high risk and with
gram-negative organisms, including
4th generation: risk for P. aeruginosa
P. aeruginosa
• Cefepime infection: Cefepime 1-2 g
• Inactive against MRSA, MRSE,
• Cefpirome IV q8-12 up to 21 days
Enterococcussp., B.fragilis, and ESBL-
producing bacteria

• Specifically developed to target


resistant strains
• Ceftobiprole: active against MRSA,
penicillin-resistant S. pneumoniae,
• Skin and soft tissue
P. aeruginosaand enterococci
5th generation: infections:
• Ceftaroline: for Acute Bacterial
• Ceftobiprole Ceftobiprole 500 mg IV
Skin and Skin Structure Infections
• Ceftaroline infusion q12
(ABSSSI) caused by MSSA, MRSA,
Ceftaroline 600 mg IV q12
S. pyogenes,E. coli,K. pneumoniae and
CAP caused by S. pneumoniae,
S. aureus, H. influenzae,K pneumoniae
and E.coli
318
EXAMPLES I ORGANISMS COVERED I COMMON DOSAGES

CEPHAI:.OSPORIN-BETA
LACTAMASEINHIBITOR{;OMBINATION
• Complicated UT!:
• Active against ESBL-producing gram-
Ceftolozane-tazobactam
negative bacteria & P. aeruginosa
1.5g!V q8
• Not active against anaerobic bacteria,
• Ceftolozane- Ceftazidime-avibactam
enterococci and staphylococci
tazobactam 2.5g!V q8
• FDA approved for complicated
• Ceftazidime- • HAP/VAP:
UT!, complicated intraabdominal
avibactam Ceftolozane-tazobactam
infections (with metronidazole) and
3 g IV q8
hospital acquired/ventilator-associated
Ceftazidime-avibactam
pneumonia
2.5g!V q8

inhibitcellwallsynthesis,& highly resistantto degra4ptionby B-lactamases


CARBAPENEMS:
• Nosocomial infections due to multiple-
resistant & complicated polymicrobial • Intraabdominal
infections caused by aerobic gram- infections:
• lmipenem- positive, gram-negative, anaerobic, and lmipenem-cilastatin
cilastatin ESBL-positive organisms 500 mg IV q6 or I g IV q6-8
• Meropenem • All are recommended for pseudomonal (to cover for P. aeruginosa)
• Doripenem infections, except ertapenem Meropenem I g IV q8
• Ertapenem • Cilastatin is combined with imipenem to Doripenem 500 mg IV q8
block enzymatic degradation of imipenem Ertapenem I g IV 924
by dehydropeptidase on the brush border
of the renal proximal convoluted tubule

MONOBACTAMS:inhibit cell wall synthesis (binds to PBP3)


'
• Activity limited to gram-negative bacilli • Pseudomonal infections:
• Aztreonam (Enterobacteriaceae,Aeromonassp., N. Aztreonam 2 g IV q6-8

I
gonorrhea,H. influenza and P. aeruginosa) (for P.aeruginosa)

II. AMINOCYCLITOLS AND AMINOGLYCOSIDES


• Inhibit protein synthesis & primarily used to treat aerobic gram-negative bacterial infections
• Aminoglycosides have concentration-dependent bactericidal activity
EXAMPLES I ORGANISMS COVERED I COMMON DOSAGES
AM.INOCYCLIT0LS:
inhibitformationof initiationcomplex& causemisreadingdf mRNA (30S inhibitor)
• Prophylaxis for colorectal
• Activeagainst aerobic gram-negative bacilli
surgery: Neomycin+
• Most are active against P. aeruginosa,
Erythromycin base I g each
E. coli,Klebsiellaand Proteussp.
• Neomycin PO at 1,2, & II pm a day
• Synergistic against staphylococcal,
before surgery or Neomycin
streptococcal and enterococcal
2 g + Metronidazole 2 g PO
endocarditis
at 7 & II pm day before surgery
AMIN<JGLYGOSIDES:
inhibitformationofinitiationcomplex& causemisreading
of mRNA(JoSinhibitor,)
• Activeagainst aerobic gram-negative bacilli
• In addition to an
• Amikacin • Most are active against P. aeruginosa
antipseudomonal ~-lactam
• Capreomycin (not to be used alone for Pseudomonas
or carbapenem in HAP:
• Gentamicin infection), E. coli,Klebsiella,Proteussp.
Amikacin 15mg/kg IV 924
• Kanamycin • Some activity against mycobacteria
• Tuberculosis:
• Netilmycin • Useful combination treatment for
Kanamycin 15mg/kg IM/IV
• Streptomycin serious gram-negative infections
q 24 or 3-5x/week
• Tobramcyin • Synergistic against staphylococcal,
streptococcal & enterococcal endocarditis

319
III. CHLORAMPHENICOL
Bacteriostatic by inhibiting peptide bond formation at the 50S ribosomal subunit
• Original indication was in the treatment of typhoid
EXAMPLES I ORGANISMS COVERED I COMMON DOSAGES
• Active against aerobes & anaerobes • Severe/complicated typhoid:
• Standard for fully susceptible typhoid JOOmg/kg IV for 14-21days
Chloramphenicol
fever, ampicillin-resistant • Uncomplicated typhoid:
H. influenzae& intraocular infections 50-75mg/kg IV for 14-21days

IV. MACROLIDES
Protein synthesis inhibitors which prevent translocation at the 50S ribosomal subunit
First-line indications for macrolides include atypical community acquired pneumonia,
H. pylori (as part of triple therapy), chlamydia, and acute non-specific urethritis
EXAMPLES I ORGANISMS COVERED I COMMON DOSAGES
• Acute bronchitis:
Azithromycin 500 mg PO LO day 1,
Azithromycin then 250 mg PO OD for days 2-5
• Cervicitis, chancroid:
Azithromycin 500 mg PO SD
• Active against aerobic
gram-positive cocci • Chancroid:
& bacilli, Legionella, Erythromycin 500 mg PO QID for 7 days
Mycoplasma,Chlamydia, • Alternative regimen for
Erythromycin and some gram-negative nongonococcal urethritis:
organisms including Erythromycin 500 mg PO QID for 7 days
Bordetel/apertussis, Erythromycin ethylsuccinate 800 mg
H. ducreyi & C.jejuni PO QID for 7 days
• Often used as alternatives
for penicillin-allergic • Acute bacterial exacerbation of
patients chronic bronchitis:
Newermacrolides:
Clarithromycin 500 mg PO BID for 7 days
• Clarithromycin
• H. pylori infection: (together
• Roxithromycin
with Bismuth and Amoxicillin)
• Josamycin
Clarithromycin 500 mg PO BID for
2 weeks

V. LINCOSAMIDES
Interfere with protein synthesis by blocking peptide bond formation at 50S ribosomal
subunit
EXAMPLES I ORGANISMS COVERED I COMMON DOSAGES
• Community-acquired MRSA infections:
Clindamycin 450 mg PO q6-8
• Bacterial vaginosis:
Clindamycin cream 2%, 1 full applicaJOr
• Useful for aerobic and (5g) intravaginally HS for 7 days
anaerobic gram-positive Clindamycin 300 mg PO BID for 7
• Clindamycin
cocci, some anaerobic days or Clindamycin ovules JOOmg
• Lincomycin
gram-negative bacilli, and intravaginally HS for 3 days
protozoans • Add-on therapy to aspiration
pneumonia regimen (except in
~-lactams that already have
anaerobic activity):
Clindamycin 450-900 mg IV q8

320
VI. GLYCOPEPTIDES AND GLYCOLIPOPEPTIDES
Some members of this drug class inhibit the synthesis of cell walls in susceptible
microbes by inhibiting peptidoglycan synthesis
Due to their toxicity, use is restricted to those who are critically ill, have hypersensitivity
to the ~-lactams, or infection from ~-lactam-resistant species
EXAMPLES I ORGANISMS COVERED I COMMON DOSAGES

GLYCOPEPTIDE:inhibit cellwall formation by bindingD-ala D-ala portionof cell wall precursors


• Active against MSSA,
MRSA, coagulase-negative • MRSA infections:
staphylococci, enterococci, Vancomycin 15-20 mg/kg IV q12
streptococci, C. diphtheria(JK (adjusted according to trough levels)•
• Vancomycin
Group), C. difficile& Listeria
• Teicoplanin Teicoplanin 6 mg/kg IV q12 x 3 doses
• Oral formulation of
vancomycin is available for LD then q24 (adjusted according to
treatment of C. difficileinfection trough levels)
• Teicoplanin can be given IM

GLYCOLIPOPEPTIDE: inhibits DNA, RNA and protein synthesis .,

• Active against gram-positive


• Daptomycin organisms - MSSA, MRSA, • Skin and soft tissue infections:
encerococci, VRE, streptococci Daptomycin 4 mg/kg IV q24
• Not recommended for pneumonia
'Vancomycin trough levels: use in monitoringvancomycintherapy is institutionalin nature
• Taken after steady state is achieved (e.g., before the 4th dose of vancomycin)
• Trough levels taken immediatelybefore next dose (reference: 5-15mcg/mL)
• Peak levels taken 1-2hours after completionof a dose (reference: 20-40mcg/mL)

VII. NITROIMIDAZOLES

I
Form toxic metabolites in the bacterial cell that damage DNA
• Possess both antiprotozoal and antibacterial activity

EXAMPLES I ORGANISMS COVERED I COMMON DOSAGES


• Bacterial vaginosis:
Metronidazole 500 mg PO BID for 7 days or
Metronidazole gel 0.75%, I full applicator
(5 g) intra vaginally OD for 7 days
• Metronidazole • Trichomoniasis:
• Excellent activity against
• Secnidazole Metronidazole 2 g PO SD
anaerobes and protozoans
• Tinidazole Metronidazole 500 mg BID for 7 days
• Acute gastroenteritis from B. hominis:
Metronidazole 750 mg PO TIO x 10 days
• C. difficilepseudomembranous colitis:
Metronidazole 500 mg PO TIO for 10 days

VIII. OXAZOLIDINONES
Protein synthesis inhibitors: disrupt mRNA translation
Possess a unique property that decreases the possibility of developing cross-resistance
with other antibiotics
EXAMPLES I ORGANISMS COVERED I COMMON DOSAGES

• Active against MSSA,


• Linezolid MRSA, S. epidermidis, • Soft tissue infections:
• Tedizolid S. pneumoniae,S. pyogenes, Linezolid 600 mg PO/IV q12
S. agalactiae,E.faecalis and Tedizolid 200 mg PO/IV q24
E.Jaecium (including VRE)
321
IX. QUINOLONES
Block bacterial DNA synthesis by inhibiting bacterial topoisomerase II (DNA gyrase) &
topoisomerase IV
EXAMPLES IORGANISMS COVERED I COMMON DOSAGES
1st gen~ration:
• Cinoxacin • Usually for UT! &
• Nalidixic acid diarrheal diseases among • Not available locally
• Oxolinic acid children
• Pipemidic acid
• Alternativefor acute uncomplicatedcystitis:
2nd generation: Ciprofloxacin 250 mg BID PO for 3 days
• Active against Norfloxacin 400 mg BID PO for 3 days
• Ciprofloxacin
Enterobacteriaceae and • Primary treatment for uncomplicated
• Norfloxacin
Haemophilus sp., acute pyelonephritis:
• Ofloxacin
N. gonorrhea & M. Ciprofloxacin 500 mg BID for 7-10days
• Lomefloxacin
catarrhalis, C. trachomatis • Salmonella gastroenteritis:
• Pefloxacin
and H. ducreyi Ciprofloxacin 500 mg PO OD for 7-10days
• Rufloxacin
• Shigellosis:
Ciprofloxacin 750 mg PO OD for 3 days
• Add-on therapy to IV non-
antipseudomonal ~-lactam in moderate-
or high-risk CAP:
• Broadened activity
Levofloxacin 500 mg PO/IV OD foq-14
against anaerobes,
days or 750 mg PO/IV OD for 5 days
3rd generation: intracellular pathogens
• Primary treatment for uncomplicated
• Levofloxacin and some gram-positive
acute pyelonephritis:
and gram-negative
Levofloxacin 250 mg OD PO for 7-10 days
aerobes
or 750 mg OD PO for 5 days
• Salmonella gastroenteritis:
Levofloxacin 500 mg PO OD for 7-10days
• Highly effective against
both typical & atypical
respiratory pathogens
• Moxifloxacin: most potent
4th generation: against S. pnewnoniae & • Post-necrotizing pancreatitis, infected
• Moxifloxacin greatest activity against pseudocyst or pancreatic abscess:
• Gatifloxacin anaerobic bacteria among Moxifloxacin 400 mg IV OD q24
all fluoroquinolones;
no activity against
P. aeruginosa; used as 2nd
line drug for M. tuberculosis

X. ANTIFOLATES
Folic acid synthesis inhibitors
Cotrimoxazole blocks two consecutive steps in synthesis of nucleic acids & proteins
essential to bacteria
0 Sulfamethoxazole: blocks bacterial synthesis of dihydrofolic acid
0 Trimethoprim: blocks production oftetrahydrofolic acid
EXAMPLES I ORGANISMS COVERED I COMMON DOSAGES

Cotrimoxazole:
• Excellent activity against • P.cariniipneumonia (PCP) prophylaxis:
• Trimethoprim-
S. typhi, some strains of TMP-SMX 160/800mg PO 3x weekly or
Sulfamethoxazole
(TMP-SMX) Shigella, V. cholera, TMP-SMX 80/400 mg PO OD
H. injluenzae • P.carinii pneumonia (PCP) treatment:
Cotrimazine:
• Used in Pneumocystis carinii TMP-SMX 15-20mg TMP/kg/day PO/
• Trimethoprim-
infection IV q6-8
Sulfadiazine

322
XI. TETRACYCLINES
Protein synthesis inhibitors which bind to 30S subunit
• Prevent aminoacyl-tRNA from binding to acceptor site ribosome-mRNA complex
EXAMPLES I ORGANISMS COVERED I COMMON DOSAGES

• Drug of choice for • Cervicitis/nongonococcal urethritis/


V. cholera,V. uulnificus, Chlamydia infections:
• Doxycycline B. burgdorferi,some Aeromonas Doxycycline 100 mg PO BID for 7 days
• Minocycline &Xanthomas sp., Mycoplasma • Donovanosis:
• Oxytetracycline • Penicillin-allergic patients Doxycycline JOO mg PO BID for 3-4
• Tetracycline with leptospirosis, syphilis, weeks until lesions completely healed
actinomycosis, melioidosis, • Lymphogranuloma venereum:
skin/soft tissue infections Doxycycline 100 mg PO BID for 21days
• For drug-resistant organisms,
including MRSA & • Complicated intraabdominal
• Tigecycline Acinetobacterbaumannii, infection: Tigecycline JOO mg IV LD
carbapenemase-producing then 50 mg IV 912
E. coli& K. pneumoniae

XII. POLYMYXINS
Causes destabilization of the lipopolysaccharide membrane of gram-negative
organisms, causing leakage of cell contents
Has "detergent effect," allowing restoration of activity of other drugs that were
previously resistant
EXAMPLES I ORGANISMS COVERED I COMMON DOSAGES

• For multi-drug resistant aerobic gram- • Polymyxin B 2.5 mg/kg IV LD


negativebacteria, including P.aeruginosa, over 2 hours then 1.5mg/kg IV
A. baumannii,& carbapenem-resistant 912 as 1-hour infusion
• Polymyxin B
Enterobacteriaceae • Colistin 4x body weight (kg)

I
• Polymyxin E
• Significant nephrotoxicity is observed IV LD then 300 mg/day (EMA
orColistin
among polymyxins, with polymyxin dosing) divided to 912 or 98
B having lower rates of renal failure & (computed according to colistin
no need for dose adjustment base activity dose)

XIII. URINARY TRACT AGENTS


EXAMPLES I ORGANISMS COVERED I COMMON DOSAGES

• Acute uncomplicated cystitis:


Nitrofurantoin monohydrate
• For the treatment of uncomplicated JOO mg BID PO for 5 days, or
• Nitrofurantoin
lower UT!
Nitrofurantoin macrocrystals
100 mg QID PO for 5 days

• With in vitro activity against gram-


positive bacteria, gram-negative
bacteria and Enterobacteriaceae • Uncomplicated UT!:
• Often used for uncomplicated UT! Fosfomycin 3 g in 50 mL water
• Fosfomycin
• Intravenous form can be given for POSD
complicated UT!, intraabdominal
infections, nosocomial pneumonia,
and osteomyelitis
• Nitrofurantoin:
inactivatesbacterial
ribosomalproteins
andothermacromolecules
resulting
to inhibition
of
DNA,RNA,proteinandcellwallsynthesis
• Fosfomycin:
inhibitssynthesis
of cellwallpeptidoglycan
Sources:JamesonJL, et al. Harrison'sPrinciplesof InternalMedicine20thedition,2018
GilbertDN,et al. TheSanfordGuidetoAntimicrobial Therapy,2021
323
• Candidasp. • Candidiasis:
• Cryptococcusneoformans Fluconazole 200-400 mg PO/JV 924
• Coccidioidesimmitis • Histoplasmosis:
• Fluconazole
• C. krusei& C.glabrataare resistant ltraconazole 200 mg PO 924
• Voriconazole
to fluconazole • Invasive aspergillosis:
• Posaconazole
• Voriconazole & posaconazole have Voriconazole 6 mg/kg IV 912 x 2
• ltraconazole
activity against Aspergillus doses LO then 4 mg/kg IV 912
• Itraconazole has activity against Posaconazole suspension 400
Blastomyces& Histoplasma mg PO BID with meals

ECH1NOGANDINS.inhibitsfungal cell wall synthesis by inhibiting /3-1,3-glucansynthase


• Micafungin 100-150 mg IV 924
• Candidasp.
• Caspofungin • Caspofungin 70 mg IV LO then
• Aspergilh1ssp.
• Micafungin 50 mg IV 924
• No activity against other molds 1
• Anidulafungin • Anidulafungin 200 mg IV LO
Cryptococcus,Trichosporon
then IOO mg IV 924

POLYENES: binds ce11membrane sterols of fungal cells


• Nystatin • Candidaalbicans • Nystatin 5 ml swish/swallow QID

• Amphotericin-B
• Candidasp.
deoxycholate
• Cryptococcusneoformans
(ABDC)•
• Mucora/es
• Amphotericin-B • ABDC: 0.3-1 mg/kg/day IV
• Blastomycesdermatitidis
lipid complex • ABLC: 5 mg/kg/day IV
(ABLC)..
• Coccidiodesimmitis
• LAm-B: 3-5 mg/kg/day IV
• Histoplasmacapsulatum
• Liposomal
• Sporothrix schenkii
AmphotericinB
(LAm-B)..
• Aspergillussp. (except A. terreus)

•ABDCis highlynephrotoxic
witha lot of infusionreactions
"ABLC andLAm-Barelessnephrotoxic butveryexpensive

ANTIVIRAL AGENTS
EXAMPLES I ORGANISMS I COMMON DOSAGES
• Oseltamivir • Oseltamivir 75 mg PO BID x 5 days
• Influenza A & B
• Zanamivir • Zanamivir 2 blisters (10 mg) BID x 5 days

• Herpes simplex virus


• Varicella (chickenpox):
• Acyclovir (HSY)
Acyclovir 20 mg/kg PO QID for 5 days
• Valacyclovir • Varicella zoster virus
Valacyclovir 20 mg/kg PO TIO for 5 days
(VZV)

• Ganciclovir: induction dose of 5 mg/kg IV


• Activity against HSV
912 for 14 days, then maintenance dose of
• Ganciclovir & VZV, but mainly
5 mg/kg IV 924
• Valganciclovir for cytomegalovirus
• Valganciclovir: 900 mg PO BID for 21 days,
(CMV)
then maintenance dose of 900 mg PO OD
Sources:
JamesonJL,et al. Harrison's
Principles
of InternalMedicine
20thedition,2018.
GilbertON,et al.TheSanfordGuidetoAntimicrobial Therapy,2021.

324
SECTION THREE
COMMON BACTERIAL INFECTIONS
LEPTOSPIROSIS
I. ETIOPATHOGENESIS
A.Etiology
° Caused by the spirochete Leptospirainterrogans
0 1st step is penetration of tissue barriers to gain entrance, then hematogenous dissemination
0 Rodents (especially rats): most important reservoir (others: wild mammals, dogs, fish, birds)
0 Incubation period is usually 5-14days (range of 2-30 days)

B. Transmission
0Leptospires are transmitted via:
• Direct contact with urine, blood or tissue of an infected animal
• Exposure to a contaminated environment
0 Human-to-human transmission is rare
0Leptospires may persist in water for many months

II. CLINICAL MANIFESTATIONS


A. Case Definition for Suspected Leptospirosis
1. Individual presenting with acute febrile illness of at least 2 days, AND
2. Either residing in flooded area or had high-risk exposure (e.g., wading in floods and
contaminated water, contact with animal fluids, swimming in flood water or ingestion of
contaminated water, with or without cuts or wounds), AND
3. At least two of the following:
• Myalgia (particularly calf muscle tenderness) • Abdominal pain
• Conjunctiva! suffusion (red eyes without exudate) • Headache
• Chills • Jaundice

I
B. Severity of Leptospirosis
MILD OR ANICTERIC LEPTOSPIROSIS I MODERATE-SEVERE LEPTOSPIROSIS

• Suspected case ofleptospirosis presenting • Suspected case ofleptospirosis presenting


with acute febrile illness but with: with acute febrile illness with:
Stable vital signs
0 , Unstable vital signs
No jaundice
0 0 Jaundice
Good urine output
0 0 Abdominal pain, nausea, vomiting, diarrhea
No meningismus/meningeal irritation
0 , Oliguria/anuria
No evidence of sepsis/septic shock
0 0 Meningismus/meningeal irritation/
No difficulty of breathing
0 altered sensorium
• Other nonspecific findings: nausea, 0 Sepsis/septic shock
vomiting, diarrhea, nonproductive cough 0 Difficulty of breathing and hemoptysis
and maculopapular rash • Weil's disease: triad of jaundice, renal
failure and hemorrhage

Leptospiremic • Lasts 5-7days: fever,chills, myalgia,conjunctiva! suffusion, nausea, vomiting


(acute) phase • Followedby a 1-3day period of improvement feverlyses& may be asymptomatic

• Occurs after a period of relative improvement & illness recurs (recurrence


Immune offever heralds the onset of this phase)
phase • Leptospires cannot be cultured from blood & antibiotics may not be
useful in this phase

Recovery • With proper care, most recover completely


Phase • Patients with renal failure on dialysis may regain most of their renal function
325
III. DIAGNOSIS
AB . Lb W kU •

I I
MARKERS OF SEVERE
DIAGNOSTIC REMARKS
LEPTOSPIROSIS

• WBC >12,000
Complete blood • May show leukocytosis,
• Neutrophilia
count(CBC) neutrophilia, thrombocytopenia
• Platelet <IO0,ooo
• Creatinine >3 mg/dL
• May be initially normal but can
Renal function • BUN >23 mg/dL
increase during the course (sign
(creatinine) • eGFR <20 rnL/min
of acute kidney injury)
• Serum potassium >4 mmol/L
Liver function • Bilirubins, ALT,AST, & ALP may , AST/ALT ratio >4
tests show slight elevation • Bilirubin >190umol/L
Bleeding • Prolonged prothrombin time (PT) &
• Prolonged PT <85% activity
parameters partial thromboplastin time (PTT)
• May show acidosis (especially • Acidosis: pH <7.2
Arterial blood if with renal failure) and • HCO3 <IOmeq/L
gas hypoxemia (especially if with • Hypoxemia (PaO2 <60 mmHg)
respiratory failure) • P/F ratio <250
• Abnormalities develop around
Chest
3-9 days in severe leptospirosis • CXR with extensive alveolar
radiograph
• Patchy alveolar pattern (suggests infiltrates
(CXR)
scattered alveolar hemorrhage)

• May show findings suggestive of • Heart block


Electrocardiogram
cardiac complications • Suggestive of myocarditis
• May show proteinuria, pyuria and often hematuria
Urinalysis
• Hyaline & granular casts may be present during the 1st week of illness
Creatine
• Elevated in patients with severe
phosphokinase ---
myalgia
(CPK-MM)

Direct Detection hf.ethods


• Dark-field microscopy, immunofluorescence, or light microscopy
after appropriate staining
Microscopic • Leptospires are thin, coiled, rapidly moving microorganisms
demonstration • Only of value during the first 7-ro days of acute illness during
leptospiremia
• Insensitive and lacks specificity

• Gold standard
• Time-consuming, labor-intensive, and requires 6-8 weeks for the result
Culture and
• Timing of culture (depends on the specimen):
isolation
° First week of illness: blood, CSF, peritoneal dialysate
0Second week of illness: urine

• Method of amplifying specific segments of leptospiral DNA


Polymerase
• Early confirmation of diagnosis especially during the acute
chain reaction
(PCR) leptospiremic phase, before the appearance of antibodies
• Limited clinical use due to cost (not generally available)

326
DIAGNOSTICS I COMMENTS/EXPECTED FINDINGS
Indirect Detection Methods
• Detects agglutinating antibodies in the serum of a patient by
mixing it with various dilutions of live or killed (formalinized)
leptospires
• Confirms the diagnosis of leptospirosis: fourfold rise in titer
from acute to convalescent sera
Microagglutination • Currently considered as the reference standard locally
test(MAT) • In endemic areas (e.g., Philippines), a single titer of at least
1:1600in symptomatic patients is indicative ofleptospirosis
• Highly sensitive and specific, but time-consuming and
hazardous to perform
• Cross-reacts with syphilis, viral hepatitis, HIV, relapsing fever,
Lyme disease, legionellosis and autoimmune disease
• Quick detection of Leptospiragenus-specific IgM antibodies in
human sera
Specific IgM
• Becomes more sensitive if samples are taken at >7 days of illness
rapid diagnostics
• False negative results may occur if performed during the early
stage of illness
• Detects leptospira antibodies in serum through agglutination
Non-specific
reaction, which may persist for years
rapid diagnostics
• A positive result should be confirmed with MAT
Sources:TheLeptospirosis
TaskForce.Leptospirosis
ClinicalPracticeGuidelines;
2010
HaakeDA,CurrTopMicrobiollmmunol2015
WHOandILS.HumanLeptospirosis; WHO2003

IV. MANAGEMENT
It is not necessary to wait for laboratory tests before starting treatment for leptospirosis;
antibiotics should be started as soon as the diagnosis is suspected
Benefit of antibiotics after 5th to 7th day of illness is controversial (but most still treat,
regardless of date of onset)
Jarisch-Herxheimer reaction: dramatic reaction consisting offever, chills, myalgia,
headache, tachycardia, tachypnea, neutrophilia, and vasodilation with mild hypotension
after initiation of antibiotic therapy for leptospirosis

A. Treatment ofLeptospirosis
INDICATION I DRUGS• I DOSE"
• Doxycycline (first-line) • 100 mg PO BID
Mild • Amoxicillin • 500 mg PO q6 or I g PO q8
Leptospirosis • Azithromycin • 1g initially, followed by 500 mg PO OD
for z more days
• Penicillin G (first-line) • 1.5million units IV q6-8
Moderate • Ampicillin • 0.5-1g IV q6
to severe • Ceftriaxone • I glV q24
leptospirosis' • Cefotaxime • I glV q6
• Azithromycin • 500 mg PO OD for 5 days
' Duration
of therapyis 7 days(exceptforazithromycin)
b Nodoseadjustment neededforceftriaxone,
doxycycline,
penicillin
G,& azithromycin
in renalfailure
'Parenteralroutemaybeshiftedto oraloncepatientis clinicallystable

327
• Leptospirosis damages the
Systemic
vascular system as a whole, • IV hydration for hypovolemic
Inflammatory
resulting in capillary leakage patients
Response
• May cause hypovolemia, shock, • Management depends on specific
Syndrome
disseminated intravascular injury (see respectivechapters)
(SIRS)
coagulation (DIC)

• Renal replacement therapy for:


• Presents as non-oliguric renal 0 Uremia

failure with mild hypokalemia 0Increasing creatinine >3 mg/dL


• Oliguria (UO <0.5 mL/kg/hr or 0Hyperkalemia with oliguria
Acute kidney <400 mL/day) with hyperkalemia ° Fluid overload
injury (AKI) indicates severe AKI 0 Pulmonary hemorrhage or
• Common findings: azotemia, ARDS
hyponatremia, hypokalemia, • Metabolic acidosis (pH <7.2)
pyuria, hematuria, proteinuria Intractable oliguria
0

• Refer to specialists

• Admit to ICU & refer to specialists


• Consider invasive ventilation
• Suspected when there is: • Consider steroids within the first
Pulmonary RR >30 breaths/min (first sign)
0 12hours of onset of respiratory
hemorrhage • Cough, hemoptysis, dyspnea involvement:
or ARDS • Usually appears on 4th to 6th Methylprednisolone I g IV/day
0

day of disease for 3 days, then


Oral prednisolone I mg/kg/day
0

for 7 days

Cardiac • May present with arrhythmias • Management depends on specific


complications and myocarditis manifestation

V. POST-EXPOSURE PROPHYLAXIS FOR LEPTOSPIROSIS


Most effective measure is avoidance of high-risk exposure & use of protective equipment
• There is no current recommended pre-exposure prophylaxis for pregnant/lactating women
Doxycycline: recommended post-exposure chemoprophylactic agent, but is not 100%
effective & is contraindicated in pregnancy (duration depends on degree of exposure)
LOW-RISK MODERATE-RISK HIGH-RISK
I EXPOSURE I EXPOSURE I EXPOSURE
• Continuous exposure 0
• Single exposure'
• With or without wounds,
• Single exposure• • With wounds, cuts,
cuts, or open skin lesions
• No wounds, cuts, or open skin lesions
Definition • Swimming in flooded
or open skin • Accidental ingestion
waters, especially if infested
lesions of contaminated
with sewer rats & ingestion
water
of contaminated water

• 200 mg single dose • 200 mg OD for


Doxycycline • Given within 3-5 days • 200 mg once weekly until
dose 24-72 hours from • Given within 24-72 the end of exposure
exposure hours from exposure
'Single exposure: historyof wading in fioodor contaminated water
"Continuousexposure:>1 exposureor severaldays (e.g., residinginfioodedareas, rescuers, reliefworkers)
Source:The Leptospirosis
TaskForce.Leptospirosis
ClinicalPracticeGuidelines;2010
WHOand ILS.HumanLeptospirosis; WHO2003
ShenoyW, et al. PostgradMed.82(971):602-6
328
TYPHOID FEVER (ENTERIC FEVER)
I. ETIOPATHOGENESIS
• Potentially fatal multisystemic illness caused by gram-negative Salmonellatyphi & paratyphi
Hallmark: penetration and multiplication within the mononuclear phagocytic cells in
the liver, spleen, lymph nodes and Peyer's patches of the ileum
Transmitted to humans orally by contaminated food or water (humans are the only known hosts)
• Incubation varies with amount of infecting dose (average of 10-20 days)

II. CLINICAL MANIFESTATIONS


In the 1st week of illness, patients complain of rising ("stepwise") fever pattern, which
may be associated with relative bradycardia
In 2nd week, abdominal pain and rose spots develop
In 3rd week, complications occur (hepatosplenomegaly, GI bleeding/perforation, bacteremia)

• Individual who lives in or who has a history of travel to tropical and


subtropical areas
Suspected
• Presents with fever of ~5 days
typhoid fever
• Fever documented at >38°C with any of the following: headache,
case
diarrhea, malaise/weakness, abdominal distention/pain, GI bleeding,
changes in orientation or confusion
• Prolonged fever
Acute
• Disturbance in bowel function (constipation in adults, diarrhea in children)
uncomplicated
disease • Non-specific: headache, malaise, nausea, vomiting, relative bradycardia
• Rose spots: faint salmon-colored macules appear primarily at the trunk area
• Occult or overt gastrointestinal bleeding
• Intestinal perforation & peritonitis
Complicated • Severe abdominal pain (usually at the right lower quadrant)
disease • Altered mental status (delirium, obtundation, psychosis, coma)
• Meningitis, encephalomyelitis, neuritis
• Others: hepatitis, pancreatitis, myocarditis, DIC, pneumonia
Source:TaskForceonTyphoidFever.PSMID;2017
III. DIAGNOSIS
AD' t'
DIAG~OSTIC I COMMENTS/EXPECTED FINDINGS
• May reveal neutropenia, leukopenia or leukocytosis, & thrombocytopenia
CBC&
• Normochromic anemia; hypochromia with blood loss
chemistry
• Abnormal liver function tests

• Gold standard
Blood
• Should be taken from at least 2 different sites
culture
• Taken anytime during illness, but yield is highest during the first 2 weeks
Stool culture • May be positive during the 3rd week of illness in untreated patients

Bone • Not usually done, but indicated in highly suspicious cases or if blood or
marrow stool cultures are negative (can be done anytime during the illness)
culture • Most sensitive diagnostic modality, but rarely indicated
• Not sufficiently sensitive or specific to replace culture-based methods for
diagnosis of enteric fever
, Salmonellacan be characterized by their somatic (0) & flagellar (H) antigens
• Felix-Wida]test measures agglutinating antibody levelsagainst O & H antigens
Serology
, Typhidot test: detects specific IgM and IgG antibodies to S. typhi
' Detection oflgM: reveals acute typhoid in the early phase of infection
' Detection of IgM & IgG:suggests acute typhoid in middle phase of infection
, Detection of IgG: can persist for >2 years after typhoid infection
Source:WHO.ThedIagnosIs,
treatmentandprevention
of typhoidfever.WHO;2003
329
B. Definition of Cases
CASE I DEFINITION

Confirmed • Fever (>38 °C) for at least 3 days, and


typhoid • Laboratory-confirmed positive culture for S. typhi or S. paratyphi (e.g.,
fever blood, bone marrow, stool)

Probable • Fever (>38 °C) for at least 3 days, and


typhoid • Positive serodiagnosis or antigen detection test, but without S. typhi or
fever S. paratyphi isolation
• Excretion of S. typhi or S. paratyphi in stool/urine for >l year after
onset of acute symptoms
Chronic • More common in women and in patients with biliary tract disease
carrier (e.g., cholelithiasis)
• Represent an infectious risk to others (e.g., in the setting of food
preparation)
Source:WHO.Thediagnosis,
treatmentandprevention
of typhoidfever.WHO;2003

IV. MANAGEMENT
A. Findings Necessitating Hospital Admission
0 Unable to take oral fluids, severe dehydration
0 Spontaneous bleeding
0 Persistent abdominal pain
° Changes in mental status and seizures
0 Signs of poor perfusion (e.g., weak/rapid pulse, cold/clammy skin, circumoral cyanosis,
and hypotension or narrow pulse pressure)
0 Typhoid in pregnancy

B. Antibiotic Therapy
0 Usually treated with a single antibacterial drug, but the optimal choice and duration
are uncertain
0 Treatment has been complicated by the development of organisms resistant to
ampicillin, trimethoprim-sulfamethoxazole, and chloramphenicol

INDICATION I ANTIBIOTIC
• Amoxicillin 1g PO 96 x 14 days
• Chloramphenicol 1 g PO 96 x 14days
• TMP-SMX 8oo/r6o mg PO 912 x 14days
Uncomplicated
• Cefixime 200 mg PO 912 x 7 days
typhoid fever
• Azithromycin 500-1000 mg PO q24 x 7 days
• Ciprofloxacin 500 mg PO 912 x 7 days
• Ofloxacin 400 mg PO 912 x 7 days

• Ceftriaxone 2-3 g IV q24, then step down to cefixime 200 mg


PO q12 x 14 days
• Azithromycin 1g IV q24, then step down to azithromycin 500 mg
Complicated PO q24 x 7 days
typhoid fever• • Ciprofloxacin 400 mg IV q12, then step down to
ciprofloxacin 500-750 mg PO 912 x 14 days
• Ofloxacin 400 mg IV 912, then step down to ofloxacin 400 mg
PO q12 x 14 days
*De-escalationto oralantibioticsshouldbebasedon resultsof culture& sensitivity.
Steppingdown
to anoralantibioticmaybedoneif afebrilefor 48 hours& patientis ableto takeoralmedications.
Source:TaskForceonTyphoidFever.PSMID;2017
330
TETANUS
I. ETIOPATHOGENESIS
Acute disease manifested by hypertonia or skeletal muscle spasms with or without
autonomic nervous disturbances
Caused by a powerful neurotoxin (tetanospasmin) produced by Clostridium tetani
(gram-positive spore-forming anaerobic rod) which can enter the skin through injury,
abrasions, or wounds
Suspected when there is a history of an antecedent tetanus-prone wound with typical
manifestations (describedbelow)
Tetanus is not transmitted from person to person
Incubation period is 3-21days (patients with shorter incubation periods have worse prognosis)

II. CLINICAL MANIFESTATIONS

Incubation period
Wound infection, multiplication ofC. tetani
In 7 to JO days (may range from 3-21 days)
Tetanus toxin uptake & synaptobrevin Initial symptoms:
cleavage in GABA inhibitory neurons muscle aches, trismus and myalgia
After 24 to 72 hours
Further toxin effects causing widespread Muscle spasm: local and generalized
disinhibition of motor & autonomic Cardiovascular instability: labile BP & HR
nervous system Pyrexia, increased respiratory & GI secretions
After 4 to 6 weeks
Cessation of spasms, restoration of normal muscle tone
Toxin degradation
Regains cardiovascular and autonomic control
Factorsassociated witha worseoutcomeinclude:
Age>70yearsold Incubationperiod<7days
Shorttimefromfirstsymptomto admission Puerperal,intravenous,
post-surgery,
& burnentrysite
Periodof onset<48hours Heartrate(HR)>140bpm
SystolicBP>140mmHg Severediseaseor spasms
Temperature>38.5•c

III. DIAGNOSISAND CLASSIFICATION


Diagnosis is usually based on typical clinical findings
Should be suspected when there is a history of a tetanus-prone injury & inadequate
immunization for tetanus

Grade I • Mild trismus and general spasticity


(Mild) • No respiratory compromise, no spasms, no dysphagia
• Moderate trismus and well-marked rigidity
Grade2
• Short spasms, mild dysphagia
(Moderate)
• Moderate respiratory involvement with respiratory rate >30 breaths/min
• Severe trismus and generalized rigidity
Grade3 • Prolonged spasms, severe dysphagia
(Severe) • With apneic spells and respiratory rate >40 breaths/min
• Pulse rate >120 beats per minute
Grade4 • Description for grade 3 with severe autonomic instability
(Very • Severe hypertension and tachycardia alternating with relative
Severe) hypotension and bradycardia
Source:AlbettJJL; LeedsGeneralInfirmary,1967
331
B. Classification of Tetanus Based on Extent oflnvolvement
1.Localized Cephalic Tetanus
• Only isolated areas of the body are involved and only localized muscle spasms occur
• Prognosis is poor if cranial nerves are involved, leading to respiratory/pharyngeal
muscle spasm with consequent aspiration or airway obstruction
2. Generalized Tetanus
• Most common and severe form of tetanus, presenting with a descending pattern
• Typical presentation involves face and jaw muscles first (e.g., trismus or lockjaw),
followed by generalized muscle spasm (described above)

IV. MANAGEMENT
A. Overview of the Management of Tetanus
OBJECTIVE I MANAGEMENT
• Wound should be cleaned & debrided of necrotic material to remove any
remaining source of anaerobic foci & prevent further toxin production
Stop toxin
• Antibiotic therapy for 7-10 days:
production 0 Metronidazole 500 mg IV q6 (preferred)
'Penicillin G 2-4 million units IV 94-6
• Since tetanus toxin is irreversibly bound to tissues, only unbound
Neutralize unbound
toxin can be neutralized
toxin in blood with
• Passive immunization for active tetanus is the standard of treatment
passive immunization
& improves survival:
(Tetanus Immune
0 Human tetanus immunoglobulin (HTIG) 3000-6000 units IM
Globulin or 0 Equine tetanus antitoxin 10,000-20,000 units IM single dose (skin
Antitoxin)
test needed)
• Since tetanus does not confer natural immunity after recovery from
Active acute illness, all should receive active immunization (total of three
immunization doses spaced at least 2 weeks apart)
(Tetanus Toxoid) • Should be administered at a different site than the HTIG
• Examples: Td vaccine, Tdap vaccine (tetanus-diphtheria-pertussis)
• Airway management & mechanical ventilation in severe cases
• Control of muscle spasms with sedatives (e.g., benzodiazepines) and
Supportive
muscle relaxants
management
• Management of dysautonomia (e.g., magnesium sulfate)
• Admit to calm, quiet, dark environments with close monitoring

For those with ~3 doses received

~10 years since most recent dose Yes No Yes No

5-9 years since most recent dose No No Yes No

<5 years since most recent dose No No No No


'TetanusToxoid(TT)-containing vaccine:tetanustoxoid(TT),Td (tetanus-diphtheria
toxoidsadsorbed), or
Tdap(tetanus-diphtheria-pertussis)
0 Tetanus lmmunoglobulin:doseof HTIGis 250unitsIMat differentsiteof TT
(Alternative:
TetanusAntitoxin1500unitsIM)
'All otherwoundsincludewounds>1cmin depth,incurred>6hoursearlier,or withstellateor avulsion
configuration; tissue;& contaminated
crushinjuriesor burninjuries;devitalized withdirt,fecesor saliva
Source:
CDCandPrevention.
Epidemiology
andPrevention
ofVaccine-Preventable
Diseases
332
INFECTIVE ENDOCARDITIS (IE)
I. ETIOPATHOGENESIS
Infection of the endocardial surface of the heart, which usually includes the heart valves
• Prototypic lesion: vegetation

II. CLINICAL PRESENTATION


Fever is the most common symptom of IE
• A murmur is common, which may be indicative of valvular damage in IE

A. Acute vs. Subacute Endocarditis


I ACUTE BACTERIAL IE
I SUBACUTE BACTERIAL IE

• S. aureus (sometimes)
• S. aureus • Streptococcusviridans
Usual • ~-hemolytic streptococci • Enterococci
etiology • Pneumococci • HACEK'
• Aerobic gram-negative bacilli • Coagulase-negative
staphylococcus (prosthetic valves)
Signs/ • High-grade fever (39.4- 40.0°C) • Low-grade fever (rarely >39.4°C)
symptoms • Decompensated heart failure common • Weight loss, abdominal symptoms
• Rapid
• Indolent, subtle, non-specific
Course • Hematogenouslyseeds extracardiacsites
• Rarely seeds other sites
• Complications are common
Prognosis • Poor (if untreated) • Better prognosis
'HACEK:Haemophilus sp.,Actinobacif/us
actinomycetemcomitans,
Cardiobacterium
hominis,Eikenefla
corrodens,
Kingeflasp.

B. Complications of IE
MANIFESTATION I DESCRIPTION

• Most common complication of IE and most frequent indication for


surgical intervention
Heart failure
• Most frequently caused by left-sided valvular regurgitant lesions,
such as severe aortic regurgitation
• Minor emboli: splinter hemorrhages, Janeway lesions, conjunctiva!
hemorrhages
• Major emboli: arterial emboli, intracranial hemorrhage,
Embolic events pulmonary infarcts, mycotic aneurysms
• Most common site of embolism is the brai~ (manifesting as stroke),
followed by the spleen (e.g., splenic abscess)
• Large mitral valve vegetations are more likely to embolize
• AV blocks or bundle branch blocks may occur due to perivalvular
Arrhythmias extension of infection
• Tachyarrhythmias may occur due to heart failure

• Caused by immune complex deposition at the glomerular


Glomerulonephritis
basement membrane

Mycotic • Focal dilations in the artery wall that have been weakened by
aneurysms infection or septic emboli

• Myocardial infarction from septic emboli, pericarditis, valvular


insufficiency
Others
• Abscess: renal, spleen, hepatic
• Uncontrolled infection

333
III. DUKE'S CRITERIA FOR IE
A. Major and Minor Criteria
0 Blood cultures: three 2-bott!e sets separated from one another by at least I hour and
obtained from different sites over 24 hours
0 If cultures remain negative after 48-72 hours, 2-3 additional culture sets should be obtained
0 Predisposing heart conditions include: valvular disease with stenosis or regurgitation,
prosthetic valves, congenital heart defects, prior IE, hypertrophic cardiomyopathy
MAJOR CRITERIA
1) Positive blood culture
• Typical organisms for IE from ?.2cultures [S. viridans,HACEK, S. gallolyticus(previously S.
bovis),S. aureus, Enterococcus]
• OR, persistently positive, defined as recovery of a microorganism consistent with IE from:
At least 2 blood cultures drawn >12hours apart, or
0

All of3 or a majority of?.4 separate cultures with first and last drawn at least I hour apart
0

• OR, single positive blood culture for Coxiel/aburnetiior phase I IgG antibody titer of?.1:800
2) Evidence of endocardial involvement
• Positive echocardiograrn for JE•:
0 Oscillating intracardiac mass on valves or supporting structures
0 Abscess
• New dehiscence of prosthetic valve
0 New valvular regurgitation (worseningor changingof preexistingmunnur is not sufficient)
MINOR CRITERIA

• Predisposing heart condition or injection drug use


• Fever ?.38.0°C
• Vascular phenomena: major arterial emboli, septic pulmonary infarcts, mycotic aneurysms,
intracranial hemorrhage, conjunctiva] hemorrhages, Janeway lesions
• Immunologic phenomena: glomerulonephritis, Osier's nodes, Roth's spots, rheumatoid factor
• Microbiologic evidence: positive blood culture but not meeting major criteria or serologic
evidence of active infection with organism consistent with IE
'Transesophageal echocardiography (TEE)is recommendedforprosthetic
valves,ratedat leastpossible
IEbyclinicalcriteria,or complicatedIE(e.g.,paravalvular
abscess).
Transthoracic
echocardiography
(TTE)is acceptable asa firsttestin others.

Somedefinitions:
• Janeway lesions:non-tender,
slightlyraisedhemorrhages onthepalmsandsoles
• Osle~snodes:tender,raisednodules onthepadsof thefingersor toes
• Splinterhemorrhages:painless,darkred,linearlesionsin theproximal
nailbedthatmaycoalesce
• Rothspots:retinalhemorrhages withsmall,clearcenters
Sources:JS Li et al: ClinInfectDis2000& BaddourLM,et al.AHAGuidelineson IE;Circulation;
2015

BC ·t . i o· :
DIAGNOSIS
I CLINICAL DEFINITION

• Two major criteria; OR


Definite
• One major criterion and three minor criteria; OR
endocarditis
• Five minor criteria

Possible • Imajor and I minor criteria; OR


endocarditis • 3 minor criteria

• Alternative diagnosis is established; OR


Diagnosis • Symptoms resolve & do not recur with ,q days of antibiotic therapy; OR
of!E is • Surgery or autopsy after ,;4 days of antimicrobial therapy yields no
rejected histologic evidence of endocarditis; OR
• Does not meet criteria for possible IE, as above
Sources:JS Li et al: ClinInfectDis2000;BaddourLM,et al.AHAGuidelineson IE;Circulation;
2015
334
IV. DIAGNOSIS
DIAGNOSTIC I REMARKS

Complete • Shows signs of infection and/or sepsis


blood count • Findings: anemia, thrombocytopenia, leukocytosis
• Baseline renal function (creatinine, BUN) to check for renal
involvement and prior to starting antibiotic therapy (e.g., vancomycin,
Chemistry& aminoglycosides)
other labs • Urinalysis may reveal microscopic hematuria as part of
glomerulonephritis
• Others: hlgh ESR, CRP,rheumatoid factor, circulating immune complexes
Blood • At least 3 sets of blood cultures obtained from different sites should be
cultures obtained, with the first and last samples drawn at least I hour apart
• Central to the diagnosis and management ofIE
• Transthoracic echocardiogram (TIE) is done as soon as possible in all
suspected IE patients
2D echo • Transesophageal echocardiogram (TEE) is done if:
• With poor windows on TIE (e.g., obese, COPD)
0 Initial TIE is non-diagnostic but with high suspicion for IE
• When considering complicated IE (e.g., abscess, prosthetic valve IE)

V. ANTIBIOTIC PROPHYLAXIS FOR IE


• Considered for those at highest risk for IE (e.g., before dental
procedures):
• With prosthetic valves (including transcatheter valve replacement)
• With prosthetic material for cardiac valve repair
0 With previous episode of IE

Prophylaxis • The following subpopulation with congenital heart disease (CHD) are
is given to also at high risk for IE, and must be given prophylaxis:
whom? • Unrepaired cyanotic CHD (includes palliative shunts or conduits)
• Repaired CHD with prosthetic material, especially during the 6
months after repair
• Incompletely repaired CHD with residual defects adjacent to
prosthetic material
• Prophylaxis not recommended in other forms ofCHD
• Dental procedures requiring manipulation of the gingival or periapical
Prophylaxis region of the teeth or perforation of the oral mucosa
is given • Prophylaxis is not recommended for respiratory tract procedures (e.g.,
forwhich bronchoscopy, laryngoscopy), transnasal or endotracheal intubation,
procedures? gastroscopy, colonoscopy, cystoscopy, vaginal or cesarean delivery,
transesophageal echocardiogram, or skin & soft tissue procedures
• Amoxicillin 2 g PO I hour before procedure (standard oral regimen)
• Ampicillin 2 g IV or IM I hour before procedure (if unable to take oral
medication)
Antibiotics • For penicillin allergy:
used • Clarithromycin or azithromycin 500 mg PO I hour before procedure
° Cephalexin 2 g PO I hour before procedure
• Clindamycin 600 mg PO I hour before procedure
• Cefazolin or ceftriaxone I g IV or IM 30 minutes before procedure
2015
on IE;Circulation;
Source:BaddourLM,et al.AHAGuidelines
fortheManagement
Taskforce of IEof theESC2015;European HeartJournal2015

335
VI. MANAGEMENT
Primary goal of antibiotics: to eradicate infection
IE is a multidisciplinary disease and referral to an infectious disease specialist, a
cardiologist, and a cardiac surgeon may be needed (consider referral to dental service for
oral care and hygiene)
Urgent or emergent surgery may be needed for severe complications such as heart
failure, severe valvular damage, uncontrolled infection, and prevention of embolism in
patients with large vegetations (see full guidelines)

ORGANISM

• Penicillin G (2-3 m U IV q4) x 4 weeks • Penicillin G (4 m U IV q4) x


• Ceftriaxone (2 g IV OD) x 4 weeks 6 weeks OR ceftriaxone (2 g IV
Streptococci • Penicillin G (2-3 mU IV q4) OR OD) for 6 weeks+/- gentamicin
(penicillin- ceftriaxone (2 g IV OD) x 2 weeks PLUS (3 mg/kg IV or IM OD or divided
sensitive) gentamicin (3 mg/kg IV or IM OD or into equal doses q8) x 2 weeks
divided into equal doses q8) x 2 weeks • Vancomycin (15mg/kg IV q12) x
• Vancomycin (15mg/kg IV q12) x 4 weeks• 6 weeks•

, Penicillin G (4 mU IV q4) OR
Streptococci
ceftriaxone (2 g IV OD) x 4 weeks PLUS
(relatively • Penicillin G (4 mU IV q4) OR
gentamicin (3 mg/kg IV or IM OD or
penicillin- ceftriaxone (2 g IV OD) x
divided into equal doses q8h) x 2 weeks
resistant) 6 weeks PLUS gentamicin (3 mg/
• Vancomycin (15mg/kg IV q12) x 4 weeks•
kg IV or IM OD or divided into
• Penicillin G (4-5 mU IV q4h) OR equal doses q8) x 6 weeks
Streptococci
ceftriaxone (2 g IV OD) x 6 weeks PLUS • Vancomycin (15mg/kg IV q12)x
(moderately
gentamicin (3 mg/kg IV or IM OD or 6 weeksb
penicillin-
divided into equal doses q8) x 2 weeks
resistant)
• Vancomycin (15mg/kg IV q12) x 4 weeks•

• Nafcillin OR oxacillin (2 g IV q4) x


Staphylococci
• Nafcillin OR oxacillin (2 g IV q4) x 6 weeks ;,6 weeks PLUSgentamicin (I mg/kg
(oxacillin-
• Cefazolin (2 g IV q8) x 6 weeks IV q8) x 2 weeks PLUS rifampicin
susceptible)
(300 mg PO q8) x ;,6 weeks

• Vancomycin (15mg/kg IV q12)x


Staphylococci
• Vancomycin (15mg/kg IV qS-12)x 6 weeks ;,6 wks PLUS gentamicin (I mg/kg
(oxacillin-
• Daptomycin ;,8 mg/kg/dose x 6 weeks IV q8) x 2 weeks PLUS rifampicin
resistant)
(300 mg PO q8) x ;,6 weeks

Enterococci • Ampicillin (2 g IV q4) OR penicillin G (4-5 mU IV q4) PLUS gentamicin (I mg/


(penicillin- & kg IV q8) all x 4-6 weeks
gentamicin- • Double P-lactam regimen: ampicillin (2 g IV q4) PLUS ceftriaxone (2 g IV 912)
susceptible) both x 6 weeks

• Ceftriaxone (2 g IV OD) x 4 weeks for NVE (6 weeks for PVE)


HACEK
• Ampicillin/sulbactam (3 g IV q6) x 4 weeks for NVE (6 weeks for PVE)
organisms
• Ciprofloxacin (400 mg IV BID) x 4 weeks for NVE (6 weeks for PVE)

• Individualize treatment
Culture- • For acute IE (days) NVE: cover for S. aureus, P-hemolytic streptococci, aerobic
negative gram-negative bacilli
• For subacute IE (weeks) NVE: cover for S. aureus,S. viridans,HACEK, enterococci

'Dosesrecommendedarefor patientswithnormalrenalfunction
• Consider
vancomycin
for Streptococcusonlyif penicillinor ceftriaxone
is nottolerated
Source:BaddourLM,et al.AHAGuidelines
on IE;Circulation;
2015
336
URINARY TRACT INFECTION (UTI)
I. ETIOPATHOGENESIS
UT! may be asymptomatic (subclinical infection) or symptomatic (disease)
Both UT! and asymptomatic bacteriuria connote the presence of bacteria in the urinary
tract, usually accompanied by white blood cells and inflammatory cytokines in the urine
• Common pathogens:£. coli, Staphylococcus saprophyticus, Klebsiella,Proteus, Enterococcus
• Usual pathophysiology: bacteria establish infection by ascending from urethra to bladder
TYPE I DEFINITION
• Bacteria in urine without signs/symptoms of UT! (it is a microbiologic diagnosis)
• Diagnosis of ASB should be based on urine culture:
0 In women: 2 consecutive voided specimens with same organism in
quantitative counts 2100,000 CFU/mL
Asymptomatic 0 In men: single, clean-catch voided specimen with one bacterial species
bacteriuria
(ASB)
in quantitative counts 2100,000 CFU/mL
0 Men/women: a single catheterized specimen with one bacterial species
isolated in a quantitative count 2100 CFU/mL
• Screening only recommended in pregnant women & those undergoing invasive
genitourinary tract (GU1) procedures (optimal screening test is a urine culture)

• Symptoms of urinary bladder infection (e.g.,acute dysuria, urgency,frequency,


gross hematuria) in a non-pregnant, healthy premenopausal female
Acute
• Urinalysis not necessary to confirm diagnosis if at least I symptom above is
uncomplicated
cystitis (AUC)
present (in theabsenceof vaginaldischarge& "complicating"conditionslistedbelow)
• The term "uncomplicated" means that structural or functional
abnormalities are absent
• Symptoms of kidney infection (e.g., fever, chills, flank pain, costovertebral
Acute
tenderness, nausea, vomiting) in otherwise healthy women with findings
uncomplicated
ofpyuria (25 WBC/HPF) and bacteriuria (210,000 CFU/mL on culture)
pyelonephritis
(AUP) • The term "uncomplicated" means that structural or functional
abnormalities are absent

• 23 episodes of AUC documented by urine culture in I year; or 22 episodes


Recurrent UT! in a 6-month period in a healthy non-pregnant woman with no known
in women urinary tract abnormalities
• Consider referral to specialist if with anatomical abnormalities
• Symptomatic UTI with functional or structural abnormalities of the GUT
• Significant bacteriuria (>IO0,ooo CFU/mL) & symptoms in the presence of:
0 Structural abnormalities: obstructive uropathy (e.g., calculi, outlet
obstructionL urologic abnormalities (e.g., strictures, masses)
° Functional abnormalities: neurogenic bladder, vesicoureteral reflux
Complicated 0 Metabolic abnormalities: diabetes, azotemia
UTI(CUTI)
0 Impaired host responses: renal transplant, AIDS, urosepsis, chemotherapy
0 Iatrogenic:indwellingcatheters/intem1ittentcatheterization,post-operativeUTI
0 Unusual pathogens (e.g., Candida)or multi-drug resistant organisms
0 UT! in males is considered complicated, except in the young exclusively
with lower urinary tract symptoms

• Signs/symptoms of UT! in a patient with indwelling urethral/suprapubic


Catheter-
catheter or intermittent catheterization & 210' CFU/mL 0£21bacterial species
associated UT!
in I catheter urine specimen or in a midstream voided urine specimen in
(CA-UT!)
patients whose catheter has been removed within previous 48 hours
Catheter-
associated • Significant bacteriuria (210' CFU/mL of21 bacterial species in a single
asymptomatic catheter urine specimen) without manifestations of UT! in a patient with an
bacteriuria indwelling urethral/supra pubic catheter (or intermittent catheterization)
(CA-ASB)

337
II. CLINICAL MANIFESTATIONS
PRESENTATION
I SIGNS & SYMPTOMS

• Dysuria, urinary frequency, nocturia, hesitancy, suprapubic


Cystitis discomfort, gross hematuria
• It is not associated with vaginal discharge
• Mild cases: low-grade fever, lower back or costovertebraI angle pain
Pyeionephritis
• Severe cases: high fever, rigors, nausea, vomiting, flank/loin pain

Symptoms ofUTI • Costovertebral tenderness, rigors, delirium


in a catheterized • Unusually manifests with dysuria, frequent urination, urgent
patient urination (may occur in CA-UT! after the catheter is removed)

• Severe form characterized by production of gas in renal and


Emphysematous
perinephric tissues
pyeionephritis
• Almost exclusively seen in OM patients

Xanthogranulomatous • Occurs when chronic urinary obstruction, together with chronic


pyelonephritis infection, leads to suppurative destruction of renal tissue

III. DIAGNOSIS
DIAGNOSTIC I COMMENTS/EXPECTED FIDINGS

• Midstream clean catch: patient voids first portion of urine, then


collects midstream urine and discards the latter portion
• In patients with short-term catheterization: obtain sample through
Collection of urine
the catheter port using aseptic technique or puncture catheter
specimens
tubing with a needle & syringe (if port not present)
• In patients with long-term indwelling cacheterization: replace the
catheter & collect a specimen from the freshly placed catheter

• Pyuria (-100%): increased numbers of polymorphonuclear


leukocytes in the urine
Urinalysis and
• Hematuria (-30%)
urine dipstick test
• Nitrite test: detects IO' CFU of Enterobacteriaceaeper mL of urine
• Leukocyte esterase test: used to detect >IOleukocytes/HPF
• Provides invaluable information for empiric antibiotic therapy
0 Gram-positive cocci in pairs & chains: Enterococcussp.,
Streptococcussp. (agalactiae,Group-B Streptococcus)
0 Gram-positive cocci in clusters: Staphylococcussp. (saprophyticus,
Urine gram stain
(GS)
aureus, epidermidis)
0 Gram-negative rods: not possible to distinguish chem based on
GS morphology (includes Escherichiacoli,Klebsiella)
• Detecting ~I organism in unspun gram-stained urine improves
predictive value that the urine will grow >IOO,oooCFU/mL

Urine culture and • Detection of bacteria in culture is the gold standard for diagnosis
sensitivity (CS) • Should be interpreted in conjunction with symptoms ofUTI
Blood cultures • Not routinely recommended unless with signs of sepsis

• Considered for patients with:


Radiologic 0 History of urolithiasis, urine pH ?.7.0,or renal insufficiency
imaging (CT scan, ° Fever despite 72 hours of treatment
KUB ultrasound) 0 Recurrent symptoms to rule out nephrolithiasis, obstruction, abscess
0 Suspicion of anatomic or structural abnormalities (in CUTI)
Sources:TaskForceon UTI,Philippine
PracticeGuidelineGroupInfectious
Disease, PPGG-1D PSMID;2013& 2014
TaskForceon UTI.Philippine
Practice
GuidelinesGroupin Infectious UTIinAdults2015Update.
Diseases. PSMID;2015
Infectious
DiseasesSocietyofAmerica (IDSA)Guidelines,
ClinicalInfectious 2005& 2010
Diseases;

338
IV. MANAGEMENT

• No screening & treatment recommended, except for pregnant


Asymptomatic patients or those undergoing invasive GUT tract procedures (also for
Bacteriuria CA-ASB)
• Treatment should be culture-guided with duration of? days
Acute • Empiric treatment is the most cost-effective (urinalysis & urine CS
Uncomplicated not usually needed prior to treatment)
Cystitis (AUC) • After treatment, urinalysis & CS are not needed if symptoms resolved
Acute • Urinalysis and urine CS recommended pre-treatment
Uncomplicated • Consider hospital admission for patients with severe illness
Pyelonephritis • Reassess patient within 72 hours of treatment (may refer to specialist
(AUP) if no improvement; consider urologic imaging)
• Urine GS & CS should be done before treatment (modify empiric
treatment based on culture results)
Complicated
• Consider admission for severe disease/sepsis
UTl(CUTI)
• Repeat urine CS after 1-2 weeks of treatment (may refer to infectious
disease, nephrology, or urology specialist if no improvement)
• Urine GS & CS should be done before treatment
Catheter
• Remove indwelling catheter as soon as it is no longer indicated
Associated UT!
• In catheterized patients, pyuria alone is not diagnostic of CA-UT!
(CA-UT!)
and is not an absolute indication to start empiric antibiotics

B. Oral Antibiotics

I AUC
I AUP I CUTI"

Nitrofurantoin 100mg QIDx 5 days --- ---


Fosfomycin 3 g singledosesachet -·· ...

Ciprofloxacin 250 mg BIDx 3 days 500mgBIDx 7-10days 500-750mg BIDx 7-14days

Ofloxacin 200 mg BIDx 3 days 400 mg BIDx 14days 200 mg BIDx 10-14days
Levofloxacin 250 mg ODx 3 days 750mg ODx 5 days 500-750mg ODx 7-14days
Cefuroxime 250 mg BIDx 7 days 500 mg BIDx 14 days ...
Cefixime 200 mg BIDx 7 days 400 mg ODx 14 days ...

625 mg TIOor
Co-Amoxiclav 625 mg BIDx 7 days 625 mg TIOx 14 days•
1 g BIDx 7-14days•

Norfloxacin 400 mg BIDx 3 days --- 400 mg BIDx 7-14days


160/800mg BID ... ...
TMP-SMX
x 3 days'
Those in bold& italicsare the preferredor first-lineantibiotics.Choicemust be correlatedclinically.

• Oralantibioticsfor CUTIreservedfor mild-to-moderate disease only(no signs of urosepsisor organ


dysfunction).Priorto use, determinepresence of riskfactorsfor drug resistance.
• WhenGS showsgram-positiveorganism
'Only ifwithTMP-SMX
susceptibility
Sources:TaskForceonUTI,Philippine
PracticeGuideline
GroupInfectious
Disease,PPGG-ID PSMID; 2013& 2014
TaskForceonUTI.Philippine
PracticeGuidelines
GroupinInfectious
Diseases.UTIinAdults2015.PSMID;2015
Infectious
DiseasesSocietyofAmerica(IDSA)
Guidelines,
Clinical
Infectious
Diseases;2005& 2010
339
C. Parenteral Antibiotics

I AUP•
I CUTlb
I CA-UTl 0

Ceftriaxone 1-2g q24 --- ---


Ciprofloxacin 400mg q12 --- ---
Levofloxacin 250-750mg q24 ... 750mgq24
Ofloxacin 200-400mg q12 -·- ---
Arnikacin 15mg/kgq24 15mg/kgq24 15mg/kg q24

Gentamicin +/-
ampicillin
3-5 mg/kgq24 3-5 mg/kgq24 ---
Ampicillin ... --- 1-2g q6-8

Ampicillin-sulbactam 1.5g q6 --- ---


Ertapenem 1 g q24(forESBL)• 1 g q24 1 g q24
---
I•
Meropenem i 1 g qB 1 g qB
Doripenem --- 500 mgqB 500mgqB
250-500mg
lmipenem-cilastatin --- q6-8
500 mgq6

Cefepime --- --- 1-2g qB-12

Ceftazidime --- --- 1-2g qB

Piperacillin- 2.25-4.5g q6-B• 2.25-4.5g q6-8 4.5 g q8


tazobactam
Vancomycin ",, --- --- 1 g q12
Colomycin
Colistin --- --- 240-480mg/kg/day
(in 2-4 doses)
100mg LO,then
Tigecycline --- --- 50 mgq12
'
Fluconazole
Antifungals --- --- Amphotericin
B
Thosein bold& italicsarethe preferred
or first-lineantibiotics.
Choicemustbe correlated
clinically.
• ParenteraldrugsforAUParegivenuntilpatientis afebrile(thenshiftto oral)
b Broad-spectrum parenteraldrugsareusedfortheseverelyill withCUTI(choicedependson
expectedpathogens, urineGS,riskfactors,andsusceptibility patterns).Maybeswitchedto oral
therapyuponclinicalimprovement. Durationof therapyis usually7-14days.
° Choiceof empiricantibioticsis institution-specific
sinceCA-UTIis oftenhealthcare-associated
(reviseit according
to cultureandsensitivity). Durationis usually7-14days.
d Formulti-drugresistantAUP
Sources:
TaskForceonUTI,PhilippinePracticeGuideline
GroupInfectious
Disease, PPGG-1D PSMID;2013& 2014
TaskForceon UTI.Philippine
Practice
GukJelines
Groupin Infectious
Diseases. UTIinAdults2015.PSMID;2015
Infectious
DiseasesSocietyofAmerica
(IDSA)Guidelines,ClinicalInfectious 2005& 2010
Diseases;

340
URINARY TRACT INFECTION IN PREGNANCY
I. DIAGNOSIS AND MANAGEMENT PRINCIPLES
SCREENING & DIAGNOSIS I MANAGEMENT PRINCIPLES

Asymptomatic Bacteriuria (ASB) -


• Screening of all pregnant women is done • Treatment is indicated to reduce risk of:
between 9-17weeks age of gestation 0 Acute cystitis & pyelonephritis in pregnancy
(AOG),preferably on the 16th week 0 Low birth weight neonates & preterm infants
0Urine CS for screening is recommended • Choice of antibiotic is based on urine CS
0If culture not available, pyuria • Follow-up urine culture should be done
(>IOWBC/HPF) or gram stain 1week after treatment
(>2organisms/OIF) in 2 consecutive • Monitoring should be done every trimester
midstream samples can be used until delivery
Acute UncompJicated Cystitis (AUC)
• Diagnosis made by urine CS • Treatment indicated to prevent spread of
• In the absence of urine CS: infection
0Urinalysis: significant pyuria (~8 pus • Start empiric antibiotics immediately, then
cells/mm' in uncentrifuged urine or adjust using urine GS/CS
~5 pus cells/HPF in centrifuged urine) • Post-treatment culture 1-2weeks after therapy
0Dipstick: positive leukocyte esterase is needed to document eradication of bacteria
and nitrite test • 7-day treatment with oral antimicrobial agent
• Pre-treatment urine culture is needed safe for pregnancy (I day for fosfomycin)
-
Aeute Uncomplicated Pyelonephritis (AUP)
• Urinalysis & urine GS/CS should be done • Consider admission if with signs of preterm labor
• Blood culture not routinely done, unless • Obtain post-treatment urine CS to confirm
septic resolution of infection
• Monitor with monthly urine cultures until delivery
II. ANTIBIOTICS FOR UTI IN PREGNANCY
I ASB in Pregnancy I AUC in Pregnancy I AUP in Pregnancy
I
0

Oral Antibiotics (FDA Risk Category)

Cefalexin (B) 500 mg BIDx 7 days 500 mg QIDx 7 days 500 mg QIDx 14 days

Cefuroxime (B) 500 mg BIDx 7 days 500 mg BIDx 7 days 500 mg BIDx 14 days

Co-Amoxiclav (B) 625 mg BIDx 7 days 625 mg BIDx 7 days 625 mg BIDx 14 days

Fosfomycin (B) 3 g sachet single dose 3 g sachet single dose ·--


Nitrofurantoin' (B) 100 mg QIDx 7 days 100 mg QIDx 7 days ...

TMP-SMX' (C) 160/800mg BIOx 7 days ... ...

Cefixime ... 200 mg BIDx 7 days 200 mg BIDx 14 days

ParenteralAntibiotics (for AUP)


Ceftriaxone ... . .. 1-2g IVq24 (first-line)

Ceftazidime ... ... 2 g IVq8 (first-line)

Ampicillin-sulbactam ... ... 1.5 g IVq6'


Warning:Beforeadministeringmedicationsin pregnantpatients,the clinicianmustbe familiarwiththe side
effects& contraindicationsof therapy(beyondthe scope of this section).
may be givenduring2nd trimesteruntil32 weeks.AvoidTMP-SMX
• Nitrofurantoin inthe 1st & 3rd trimesters.
with
• IVmay be shiftedto oral once patientis 48 hours afebrile, a recommended durationof 14 days (in the
patterns- primary
absenceof urineCS, empirictherapyshouldbe basedon susceptibility IV)
optionis ceftriaxone
'When urinegram stain shows gram-positive organisms
Source:Task Forceon UrinaryTract Infections,PPGG-IDPSMID;2013-2014
341
SECTION FOUR
COMMON VIRAL INEECTIONS
CORONAVIRUS DISEASE 2019 (COVID-19)
Due to the novelty of this disease entity, guidelinesare rapidly changing and adapting to the advances
in our knowledge about COV/D-19and its management. Information in this section was updated as of
September2021,but newer evidencemay further strengthen, alter,or refute the recommendationscovered
here. We encourageour readersto consult the referencesfor this sectionfor the most updated information.

I. ETIOPATHOGENESIS
A. Etiology
° Caused by the novel severe acute respiratory syndrome coronavirus-2 (SARS-Co V-2)
0 Originated from Wuhan, Hubei Province, China in December 2019
0 Declared as a global pandemic by WHO on March II, 2020
0 Prone to mutations with the appearance of several variants of concern
B. Transmission
0 Exposure to respiratory droplets from an infected person is the main mode of transmission
0 Aerosolization of the droplets can occur, which may lead to transmission at longer distances
0 Transmission through fomites is possible but risk is low
0 Isolation oflive virus in feces was demonstrated in laboratory studies, which may
imply fecal-oral transmission
0 Mother-to-child (vertical) transmission is also possible (but is uncommon)
C. Risk Factors for Severe Illness
0 Increasing age
° Comorbidities (table below lists some comorbiditiesbased on the CDC, as of November 2021)
• Cardiovascular disease (e.g., heart failure, CAD), cerebrovascular disease
• Chronic kidney disease, diabetes mellitus (types I and 2)
Established & • COPD and other lung diseases
probable risk • Cancer, HIV, use of corticosteroids or immunosuppressives
factors • Neurologic conditions, including dementia
• Obesity & overweight, current/former smoking, pregnancy
• Solid organ or blood stem cell transplantation

Possible risk • Asthma


factors (mixed • Hypertension
evidence) • Liver disease
Source:Centersfor diseasecontrol& prevention.AvailableOnline.AccessedSept2021

II. CLINICAL MANIFESTATIONS


A. Clinical Course
PHASE I REMARKS
• Generally within 14 days (mostly -4-5 days after exposure)
Incubation
• If patient remains asymptomatic 14days after exposure, symptoms are
period
unlikely to develop

• Common manifestations: cough, myalgia, headache, diarrhea, sore throat,


Initial smell/taste abnormalities (e.g., anosmia, dysgeusia)
presentation • Pneumonia is the most frequent serious manifestation (presents with
fever, cough, dyspnea)
Acute course
• Some with non-severe symptoms may progress over the course of a week
& onset of
complications • See next pagefor enumerationof organ-specificcomplications/manifestations

Recovery& • Time to recovery is highly variable (depends on age & preexisting comorbids)
long-term • Usual persistent symptoms include fatigue, dyspnea, chest pain, cough,
sequelae and cognitive deficits
Source:McIntoshK,et al. COVID-19
Clinicalfeatures.UptoDate
342
B. Severity of Disease
Patientswith COVID symptoms

Satisfies ony
Satisfies any of these criteriaf
of these criterial Impendingrespiratoryfailure
ARDS
breaths/min
RR;?:JO Sepsis
0 2 saturation <94% Shock
Requires 02
supplementation

Yes

Severo COVID-19 Critical C0VI0.19

Pneumoniaon chestima
;?:60years old
With co-morbidities•
Co-morbidities

Hypertension Bronchiedosis
Yes HIV
CAD
DM Chronic steroid use
COPD Malignancy

Source:NIHCOVID-19TreatmentGuidelines.2021
C. Mild Versus Severe COVID-19
IMild COV/D-19 Symptom Onset Viral load Symptom Relief
Infection

lI ~ti~
Recovery

Days -5 0 5 10 15 20+

ISevere COV/D-19 I
Infection Recovery

l or
Death

Days -5 0 5 10 15 20+

Incubation I Vlral responsephase ............--: Compllcillkil\ilf


Stage I Stage II Stage Ill

Clinical Pre-
~:::-;:z===:====;-;:====;:==:==:::;i
Symptoms symptomatic

events
Thromboemboffc

Clinicalstagesof COVID-19. canrangefromasymptomatic


Clinicalspectrum to flu-likeillnessto acuterespiratory
distresssyndrome andsepticshock.Symptoms withthediseasecoursewerescaledto theaverage
thatcorrelate
daysand timewindows. Afteran incubationperiodof 4-5 days,mostwill experience mildsymptoms during
stageI (correlateswithpeaksin viralload& inflammatory response),withno complications. Severeinfectionis
characterizedbya hyperinflammatory responsethatperpetuatesdiseaseprogression towards stagesIIandIll,with
ARDS,multiorgan failure,andthromboembolic eventsasmaincausesof death.
Stagesof Infection:
dominate
symptoms
• StageI: earlyinfectionor viralresponsephaseduringwhichrespiratory
• StageII: pulmonary phasewhenthe patientsdeveloppneumonia symptoms
andassociated
• StageIll: hyperinflammation phasewhenpatientssuddenlydevelopARDS,sepsis,andorganfailure
LaricciaV, et al. J Clin Med.2020
343
• Pneumonia, acute respiratory distress syndrome (ARDS), pulmonary
Pulmonary
embolism (PE), respiratory failure
• Acute Ml, myocarditis, decompensated heart failure, arrhythmias,
Cardiovascular Kawasaki-like syndrome, shock
• Thromboembolism: deep vein thrombosis (DVT), pulmonary embolism
Gastrointestinal • Anorexia, diarrhea, nausea & vomiting, abdominal pain, liver injury,
& hepatic elevated transaminases and total bilirubin
Genitourinary • Proteinuria, acute kidney injury, hematuria
• CNS: dizziness, headache, impaired consciousness, ischemic or
Neurologic hemorrhagic stroke, seizures, ataxia, encephalopathy
• PNS: ageusia & anosmia
Endocrine • Altered cortisol dynamics & HPA (hypothalamus-pituitary-adrenal) axis
Hematologic • Lymphopenia, thrombocytopenia, leukopenia, DIC
Immunologic • Cytokine release syndrome (CRS) or cytokine storm
Dermatologic • Purpuric eruptions, livedo reticularis, retiform purpura
Source:Gavriatopoulou
M,et al. ClinExpMed;2020

III. DIAGNOSIS
Most common laboratory abnormalities include lymphopenia, elevated CRP, elevated
cardiac enzymes, abnormal liver function tests
Patients with severe disease have also been reported to have higher viral RNA levels in
respiratory specimens (but data are mixed)

A. Diagnostics for Confirming the Diagnosis ofSARS-CoV-2 Infection


DIAGNOSTIC I DESCRIPTION

• Reference standard in the diagnosis of SARS-CoV-2 infection


• Detects the presence of SARS-CoV-2genetic material in submitted specimen
Nucleic acid
• May use nasopharyngeal swab, oropharyngeal swab, throat swab, saliva, or
amplification
test(NAAT) endotracheal aspirate as specimen
orRT-PCR • Cycle threshold (CT) refers to the number of cycles in an RT-PCR assay
needed to amplify viral RNA to reach a detectable level, indicating relative
viral RNA load in a specimen (lower CT values reflect a higher viral load)

• Detects viral antigens


• Lower sensitivity but comparable specificity to RT-PCR/NAAT
• Shorter turn-around-time than RT-PCR/NAAT & can be done as point-of-
Rapid
antigen test
care testing
• Use nasopharyngeal swab as specimen (use of saliva is not recommended)
• Recommended in patients who are symptomatic and within 7 days from onset
of symptoms (not recommended for use among asymptomatic individuals)

• Detects recent or past SARS-CoV-2infection by detecting IgM & IgG antibodies


• May take 21 days or longer for these tests to yield positive results
• Not recommended as the sole basis for diagnosing SARS-CoV-2infection
Serology or • Several methods are available:
antibody 0Enzyme-linked immunosorbent assays (ELISA)
testing ° Chemiluminescent immunoassays (CLIA)
0Lateral flow immunoassays (LFIA) or rapid antibody tests
• ELISA and CLIA have higher sensitivity and specificity compared to LFlA,
and may be used to determine seroprevalence among adults
Source:NIHCOVID-19
Treatment
Guidelines;
2021& Philippine
COVID-19 LivingRecommendations. APRIL27,2021
CascellaM, et al. Features,
StatPearls
Publishing;
2021
344
B. Laboratory Abnormalities Associated with Worse Outcomes
0 Studies have found that some laboratory features are associated with severe disease;
however, they have not been clearly demonstrated to have prognostic value:
• Elevated D-dimer (>I mcg/mL), lactate dehydrogenase (LDH), inflammatory
markers (ferritin and CRP), inflammatory cytokines (IL-6 and TNF-a), troponin
• Lymphopenia (low absolute lymphocyte count) and thrombocytopenia
C.Imaging
IMAGING I REMARKS

Chest • May be normal in early or mild disease


X-ray • Some findings:consolidationwith bilateral, peripheral, & lowerlung zone distributions
• More sensitive than a chest X-ray
Chest
• Common findings:ground-glass opacification,consolidation:often bilateral, with
CT scan
peripheral distribution, and involvesthe lower lobes . •i

IV. MANAGEMENT
The mainstay of management ofCOVID-19 is supportive respiratory care
• Nebulized medications are best avoided (because of the risk of aerosolization)
THERAPY
I RECOMMENDATIONS I DOSE
'.Antnnra lA. gents ., ., ,. ,:Y·- -
• Novel nucleotide analog with in vitro activity
against SARS-Co V-2 for patients who have oxygen
• 200 mg IV LD on
saturation <94% &Jor requiring 02 support
day 1,then 100 mg
• Given together with dexamethasone
IV (infused over 30
Remdesivir • Insufficient evidence if there is still benefit
min) once daily to
among patients who are already on mechanical
complete
ventilation or on ECMO
5-10 days
• Not recommended if eGFR <30mL/min/1.73m' and
ALT >!OX elevated
• RNA polymerase inhibitor initially used for • 1,800 mg PO BID LD
Favipiravir treatment of influenza on day 1,then 800 mg
• Insufficient evidence for its benefit PO BID up to 14days
• Recombinant monoclonal antibodies
• Non-competitively bind to epitopes of the spike • Administered
Casirivimab protein receptor-binding domain of SARS-CoV-2 together (casirivimab
+Imdevimab • Blocks viral entry into host cells 600 mg & imdevimab
(Ronapreve) • For mild to moderate COVID-19 age ;,12years, 600 mg) as a single IV
weight ;,40 kg, do not require supplemental 02, infusion
& high risk of progressing to severe disease
• Ribonucleoside analog that inhibits viral RNA
replication, leading to an accumulation of
• 800 mg PO BID for
Molnupiravir mutations known as viral error catastrophe
5 days
• Recommended for mild to moderate COVID-19
to prevent progression to severe disease
• Treatment for mild to moderate COVID-19 at
Nirmatreivir high risk of progression to severe disease • Nirmatrelvir 300 mg
+Ritonavir • Combination ofSARS-CoV2 main protease + ritonavir 100 mg
(Paxlovid) inhibitor (nirmatrelvir) boosted by HIV-1 BID x5 days
protease and CYP3A inhibitor (ritonavir)
• Monoclonal antibody expected to have retained
activity against the Omicron variant • Sotrovimab 500mg as
Sotrovimab
• Recommended for mild to moderate COVID-19 single IV infusion
at high risk of progression to severe disease
345
THERAPY I RECOMMENDATIONS I DOSE

Anti- I nif/ammatory !{lgents


• Dexamethasone 6 mg IV
• Recommended to be given among patients
daily for 10 days
Systemic requiring oxygen supplementation
• Methylprednisolone 32mg/day
corticosteroids • If dexamethasone is not available, it is
• Hydrocortisone 150 mg/day
reasonable to use other steroids
• Prednisone 40 mg/day

• IL-6 receptor blocker


• Recommended (along with systemic
corticosteroids) in patients with rapid
respiratory deterioration &/or requiring
high dose of oxygen supplementation
(HFNC or MV), with elevated
inflammatory markers (CRP, ferritin, IL-6)
• 4-8 mg/kg single dose with
• Ideally given in the absence of bacterial
recommended dose of 400
sepsis or chronic active hepatitis
mg IV diluted in pNSS to
• Avoided in those with:
IO0 mL, given as a 2-hour
0 Hypersensitivity to tocilizumab
Tocilizumab infusion
0 Immunocompromised or on other
• A second dose may be
immunosuppressive agents
given after 12hours at the
0 Uncontrolled serious infection other
discretion of the healthcare
than COVID-19
provider
0 Absolute neutrophil count (ANC) <500•
or <IO00 cells/uL b
0 Platelet count <50,000
i 0 ALT ,5x• or ,rox elevatedh
0 Active TB or herpes zoster infection
0 With risk for or active GI perforation, or
active diverticulitis

• )AK-inhibitor, originally used for


rheumatoid arthritis
• Recommended to be used together with
remdesivir in patients requiring oxygen
supplementation despite initiation of • 4 mg OD PO for 14 days or
Baricitinib
dexamethasone, although some data until hospital discharge
support baricitinib use independent of
remdesivir use
' • Not given for those who received tocilizumab
• Not recommended if eGFR <15mlJmin/1.73m'

Gr&ic;l Oare,and i/lspirator,y Mana:eme~t


- - "'
,._
---
Fluid • Conservative fluid management strategy preferred over liberal fluid
management management in mechanically ventilated patients with ARDS

High-flow
• Recommended for acute hypoxemic respiratory failure unresponsive to
nasal cannula
conventional oxygen support (see Critical Care chapterfor details)
(HFNC)

Mechanical • Recommended to use a lung protective ventilation strategy for ARDS


ventilation • Tidal volume of 4-8 mUkg predicted body weight & plateau pressure <30 cmH,O
(MV) • Individualize PEEP based on respiratory mechanics (e.g., lung compliance)

Proning • Self-proning recommended to improve lung recruitment & oxygenation

ECMO • May be used in judiciously selected patients with severe ARDS


• Philippine
COVID-19
LivingRecommendations.
APRIL27,2021
b NIHCOVID-19 TreatmentGuidelines;
2021
346
THERAPY I RECOMMENDATIONS I DOSE

Anticoagulation (for hospitalized COVID-19positive, suspect, or probable patients)


• Enoxaparin
0 1 mg/kg SC 912 (CrCI >30)

0 1 mg/kg SC OD (CrCI 15-29)

• Moderate or severe infection, not • Unfractionated heparin (UFH)


Therapeutic dose requiring ICU-level of care; or 0 80 u/kg IV bolus, then

anticoagulation • Confirmed VTE (DVT/PE) 0 18 u/kg/hr IV drip

• Fondaparinux
0 10 mg SC 924 (Wt>IO0 kg)

0 7.5 mg SC 924 (Wt 50-100 kg)

0 5 mg SC 924 (Wt<50 kg)

• Mild clinical syndrome with Padua • Enoxaparin


Prophylactic Prediction Score ~4; or 0 40 mg SC OD (CrCl >30)

dose • D-dimer ~1,500 ng/mL; or 0 30 mg SC OD (CrCl 15-29)

anticoagulation • Pregnant & postpartum; or • UFH 5000 units SC 912


• Critically-ill requiring ICU care • Fondaparinux 2.5 mg SC 924
:Ii
Supportive Treatment

• Utilized in those who continue • 3-4 sessions upon the


to deteriorate despite remdesivir, discretion of the healthcare
Hemoperfusion
steroids, & tocilizumab, or if there are provider
contraindications to these therapies • Insufficient evidence for use

Empiric • Recommend against empiric antibiotics unless with suspicion of


antimicrobials secondary bacterial infection

Pulmonary • Individualized pulmonary rehabilitation with pre-intervention medical


rehabilitation clearance for long COVID patienis who show residual respiratory symptoms

• Convalescent plasma, hydroxychloroquine, ivennectin, Lianhua, or


steam inhalation: recommend against the use of these interventions
Others
(very low quality of evidence to support their use, or with at least
moderate quality evidence showing no benefit or even harm)
Sources:NIHCOVID-19 TreatmentGuidelines;
2021
PhilippineSocietyof VascularMedicine.RoleofAnticoagulation
in COVID-19.
AccessedNov2021
PhilippineCOVID-19LivingRecommendations. APRIL27,2021

V. VACCINATION
VACCINE (Manufacturer) I TYPE I DOSING*

Covaxin (Bharar BioTech) • Inactivated virus • 2 doses, 14 days apart

Sputnik V Gam-COVID-Vac
(N.F.GamaleyaNationnlResearch
CenterofEpidemiology
& • Viral vector • 2 doses, 21 days apart
Microbiology
oftheMinistryofHealthoftheRussianFederation)

Ad26.COV2-S (recombinant)
• Viral vector • 1 dose only
(Johnson & Johnson/Janssen)

mRNA-1273(NJAID-Moderna) • mRNA • 2 doses, 28 days apart

NVX-CoV2373 (Novavax) • Protein subunit • 2 doses, 21 days apart

ChAdOx1-S (recombinant) • Viral vector • 2 doses, 4-12 weeks apart


(University of Oxford-AstraZeneca)

BNTI62b2 (Pfizer-BioNTech) •mRNA • 2 doses, 21 days apart

Coronavac (Sinovac) • Inactivated virus • 2 doses, 28 days apart

'Individualis consideredfullyvaccinated2 weeks afterreceivingthe 2nd dose (oronlydose inthe case ofJanssen)

347
DENGUE
I. ETIOPATHOGENESIS
Acute febrile illness of 2-7 days (sometimes biphasic), with no identifiable focus of infection
• Four (types I to 4) distinct flaviviruses (arboviruses whose transmission involves
mosquitoes)
• Principal vectors are Aedesaegypti&Aedesalbopictusmosquitoes, which breed near human
habitation
Macrophage/monocyte infection is central to the pathogenesis
• Second infection with a serotype different from that involved in the primary infection
leads to a more severe infection, which can progress to dengue hemorrhagic fever (DHF)
and/or dengue shock syndrome (DSS)

II. CLINICAL MANIFESTATIONS


• Classic symptoms ("break-bone fever"): sudden onset of fever, headache, retroorbital
pain, and back pain along with severe myalgia
• Initial symptoms: fever, macular rash, adenopathy, palatal vesicles and scleral injection
• Incubation period of 2-7 days (occurs prior to the three clinical phases of dengue)

A. Clinical Phases of Dengue


PHASE I REMARKS IWATCHOUT FOR
• Manifestations of dengue start to appear
(e.g., high fever, aches, rashes)
• Positive tourniquet test increases probability
of dengue infection
Febrile phase • Dehydration
• Need for close monitoring for development
(may last 2-7 • Neurologic
days) of warning signs
disturbance
• Earliest abnormality in the CBC is a
decreasing white blood cell (WBC) count
• Significant viremia during the early febrile
phase
• Defervescenc~ occurs (drop in temperature
to 37.5-38.0°Corless)
• During this time, patients may either improve
(dengue without warning signs) or deteriorate
(dengue with warning signs or severe dengue)
• Warning signs occur due to inflammation
• Shock from
& capillary fragility, which marks the
Critical phase plasma leakage
beginning of the critical phase
(days3-7of • Severe
illness) • Period of clinically significant plasma
hemorrhage
leakage usually lasts 24-48 hours
• Organ dysfunction
• There is progressive leukopenia and rapid
decrease in platelet count
• Degree of increase above the baseline
hematocrit (Hct) reflects the severity of
plasma leakage '
• Antibodies start to develop
• Gradual reabsorption of extravasated fluid
via lymphatics
• Improvement in well-being and
Recovery hemodynamic status (diuresis ensues) • Hypervolemia if
phase • Hct stabilizes or may be lower due to excessive IV fluids
(next 2-3 days) dilutional effect of reabsorbed fluid were given
• WBC starts to rise after defervescence, but
recovery of platelet count is later than that of
WBC
348
B. Course of Den ue

Course of dengue illness Febrile Critical Recovery

Days of illness 2 3 4 5 6 7 8 9 10

Temperature

40°

Potential clinical issues

Dehydration
: ....
, Reabsorption
Fluid overload

O,:gan impairment

Laboratory changes Platelet

Serology and virology

III. DIAGNOSIS
DIAGNOSTIC I COMMENTS/EXPECTED FINDINGS

• Leukopenia, thrombocytopenia, elevated hematocrit (Hct)


• Change in Hct is a useful guide to treatment in parallel with hemodynamics
0 Rising Hct + unstable patient: indicates active plasma leakage & need for

further intravenous fluids (!VF)


0 Decreasing Her+ unstable patient: may indicate hemorrhage and need for
CBC
blood transfusion (BT)
0 Rising Hct + stable patient: monitor closely & expect that Hct will fall

within 24 hours as plasma leakage stops


0 Decreasing Hct + stable patient: may indicate hemodilution (reabsorption of

extravasated fluid) & need to decrease/stop !VF to avoid pulmonary edema

• Immunoassay for the detection of non-structural protein I (NS1)antigens


NS1antigen • For screening and diagnosis as early as I day post-onset of symptoms (prior to
(rapid) test the detection of IgM or IgG antibodies)
• Best used during the first 5 days of illness

Dengue • After day 5, dengue antigens disappear (coincident with appearance of


IgM&IgG dengue antibodies)
(ELISA or 0 IgM antibody: usually detected after day 5 of illness (for acute infection)

rapid test) 0 lgG antibody: detects past dengue infection

• BP cuff is inflated midway between systolic & diastolic pressures for 5 mins
Tourniquet • Considered positive if;?:20petechiae per square inch, 1.5inches from the
test volar aspect of the antecubital fossa
• Least sensitive of all tests
Source:YipWCL.Denguehaemorrhagic
fever.MedicalProgress.
1980
WHODengueGuidelines,
NewEdition,WHOGenevaSwitzerland.2009
349
IV. REVISED DENGUE CASE CLASSIFICATION (WHO 2009; DOH 2011)
Dengue has different presentations & often unpredictable clinical evolution & outcome
• Classification into levels of severity is probably more practical clinically
DENGUE WITHOUT
WARNING SIGNS
I DENGUE WITH
WARNING SIGNS•
I SEVERE DENGUE

1. ProbableDengue: 1. ClinicalWarningSigns: Manifestationsof denguewith


• Livesin/travelsto dengue- • Livesin/travelsto dengue- or withoutwarningsigns,PLUS
endemicarea endemicarea anyof thefollowing:
• Withfever,PLUSanytwoof the • Feverfor 2-7days,PLUSany
following: of thefollowing: 1. SeverePlasmaLeakage'
0 Headache Abdominalpainor
0 • Shock(dengue shocksyndrome)
0 Bodymalaise tenderness • Fluidaccumulation with
0 Arthralgia Persistent
0 vomiting respiratory
distress
0 Retroorbital
pain
° Clinicalfluidaccumulation
0 Anorexia
Mucosalbleed
0 2. SevereBleeding
0 Nausea/vomiting
Lethargy,restlessness
0
0 Diarrhea
Liverenlargement
0 >2 cm 3. SevereOrganInvolvement:
° Flushedskin
0 Rash • Liver:ASTor ALT2!1000
0 Tourniquettestpositive 2. LaboratoryWarningSigns • Nervoussystem:impaired
• Andlaboratorytests:leukopenia • Increasein hematocrit consciousness,seizures
+/- dengueNS1 antigentestor • Rapiddecreasein platelet • Cardiac:impairedfunction
denguelgMantibodytest count (e.g.,myocarditis)
• Renal:impairedfunction(e.g.,
2. Laboratory-confirmed dengue' azotemia)
• Viralcultureisolation
• PCR
• Requires
strict
observ~~on
andmedical intervention
whenthereare
• Important nosignsofplasmirleakage ·
c Evidence
ofplasm.a
le11,kage~
hig'1
orrising·Hct,
pleural
effusion
orascites,
signs
ofshoe!<
Sources:WHODengueGuidelines,NewEdition,WHOGenevaSwitzerland;
2009
DOHRevisedDengueClinicalCaseManagement Guidelines,
2011

V. MANAGEMENT
A. Overview of Management (based on Group)

I GROUP A I GROUP B I GROUP C


• No warning • No warning
signs signs ,With
• Tolerates oral • With co-existing warning
• Severedengue (see
Criteria fluid complicating signs (see
definitionabove)
• Passesurine conditions' or classification
at least once unfavorable above)
every 6 hours ci.rcumstancesb

• Full blood count


• Full blood count
Work-up • Hematocrit (Hct)
• Hematocrit (Hct)
• Organfunctiontests

• Maybe sent • Admit and do


Disposition home (if with • Admit to hospital emergency
stable Hct) treatment

•Coexisting complicating
conditions:
pregnancy, oldage,obesity,diabetes,
renaldisease,
hematologic
disorder
'Social circumstances:livingalone,livesfar fromthe hospital
Sources:
WHODengue
Guidelines,
NewEdition,WHOGenevaSwitzerland;
2009
DOHRevised
DengueClinicalCaseManagement
Guidelines,
2011

350
, ,
'
GROUP A GROUP C
No Warning Severe
Signs Dengue

General Advice and Initial Management


• Bed rest • Encourage oral • Check baseline Hct before !VF • Get baseline
• Adequate fluid fluid intake (OF!) • Encourage OF!, if tolerated Hct before
intake (>5glasses) • Give IV fluids if IVF,if
• Paracetamol for OF! not tolerated possible
fever(max 4 g/day) • Immediate
• Close follow-up therapy
Intravenous Fluid (IVF)

• NIA • 0.9% NaCl or LR • Use isotonic fluids only (e.g., • Seenext


at maintenance 0.9% NaCl, LR) pagefor the
ratea • Initial rate: management
• Increase !VF • 5-7 mL/kg/hr x 1-2 hrs, then of shock
rate as necessary, • 3-5mUkg/hr x next 2-4 hrs,then • Goal of
based on clinical • 2-3 mL/kg/hr or less fluids:
assessment (e.g., • Reassess and repeat Hct: improve
dehydration) • If Hct the same or rises only circulation
• !VF rate may minimally, continue with 2-3 &end-organ
be decreased mL/kg/hr x next 2-4 hrs perfusion
anytime, as • If Hct rapidly rising, increase
necessary rate to 5-10mL/kg/hr x 1-2 hrs
• Give minimum !VF needed IO
maintain UO of 0.5 mL/kglhr
• !VF usually needed for 24-48
hours only
• Reduce !VF gradually towards
the end of critical phase until:
0 Adequate UO
• Adequate oral intake
• Hct decreases below baseline
Monitoring

• Daily review • Temperature • Vitals and peripheral perfusion (q1-4 hours,


ofWBC,Hct, pattern until out of critical phase)
platelet count, for • Fluid intake & • Urine output (UO)
defervescence, output • Hct before & after fluid replacement, then every
or warning signs • Warning signs 6-12 hours
• Consult if with • Hct,WBC, • Blood glucose
warning signs platelet count • Organ function: renal, hepatic, coagulation

Discharge Criteria
•NIA • All must be present:
• No fever for 48 hours
• Improved clinical status (e.g.,well-being, appetite, hemodynamics, UO)
• Increasing trend of platelet count
• Stable Hct without !VF
• Maintenance
fluidrate:4 mUkglhrforthefirst10kgbodyweight(BW),plus2 mUkglhrforthenext
10kgBW,plus1 mUkglhrfor subsequent kgBW(forobese,useidealBW)
Sources:
WHODengueGuidelines,
NewEdition,WHOGenevaSwitzerland;
2009
DOHRevisedDengueClinicalCaseManagement
Guidelines,
2011

351
• Clear and lucid patient • Changes in mental status (e.g.,
• Prolonged capillary refill time (CRT) restlessness)
• Cool extremities • Very prolonged capillary refill time (CRT)
• Weak & thready pulses • Mottled skin
• Tachycardia • Cold & clammy extremities
• Tachypnea • Feeble or absent pulses
• Normal SBP and rising DBP • Tachycardia (or bradycardia in late stage)
• Narrow pulse pressure • Tachypnea, Kussmaul breathing
• Hypotensive or unrecordable BP
• Narrow pulse pressure (<20 mmHg)
#I) Initial Intervention

• Fluid resuscitation: plain isotonic • Fluid resuscitation: plain isotonic


crystalloid at 5-10 mL/kg/hr over 1 hour crystalloid or colloid at 10-20 mL/kg over
• Reassess: vitals, CRT, Hct, UO 15minutes
• Reassess: vitals, CRT, Hct, UO
#2) If with improvement after initial intervention (#I)

• Give IV crystalloid at:


0 5-7 mL/kg/hr for 1-2hours, then reduce to:
0 3-5 mL/kg/hr for 2-4 hours, then reduce to:
0 2-3 mL/kg/hr for 2-4 hours, then may reduce further depending on hemodynamics
• Limit fluids to < 3L/day to avoid fluid overload
• Monitor Hct q6 & adjust !VF based on hemodynamics & Hct (e.g., ifHct increases,
consider increasing !VF rate)
• If patient stable for 48 hours, may stop !VF or give maintenance !VF or ORS
#3) If no improvement after initial intervention (#I), check Hct after 1st bolus

• IfHct increases or still high (>50%): • IfHct increases or still high (>50%):give
give 2nd bolus of colloid/crystalloid 2nd bolus of colloid at 10-20 mL/kg in
10-20 mL/kg in 1 hour 15-30mins
0If stable & Hct decreases: reduce to If stable & Hct decreases: reduce
0

7-10 mL/kg/hr for 1-2hrs, then go crystalloid !VF to 7-IOmL/kg/hr for


to #2 1-2hrs and go to #2
0If unstable & Hct increases: give 3rd If unstable & Hct increases: give 3rd
0

bolus of fluid at I0-20 mL/kg bolus offluid at I0-20 mL/kg for 1 hour
for I hour (start inotropes and (start inotropes and refer to tertiary
refer to tertiary center if still not center if still not improved)
improved)

• IfHct decreases compared to initial reference Hct (<45%):transfuse pRBC 5-10 mL/kg
or fresh whole blood (FWB) 10-20 mL/kg if with signs of bleeding
0If improved: go to #2
0If no improvement: start inotropes and refer to tertiary center
0Platelet concentrate and/or fresh frozen plasma for severe bleeding may be given
judiciously (although there is little evidence to support this practice)
Sources:WHODengueGuidelines,
NewEdition,WHOGenevaSwitzerland;
2009
DOHRevisedDengueClinicalCaseManagement
Guidelines,
2011

V. VACCINATION
A recombinant live attenuated tetravalent dengue vaccine is available for ages 9-45 years
Recommended schedule: o, 6, 12months (given 0.5 mL SC)
No efficacy study done in adults

352
RABIES
I. ETIOPATHOGENESIS
Rapidly progressive, acute disease of the CNS, caused by rabies virus (family
Rhabdoviridae)
Average incubation period is 20-90 days
Transmission is usually via bite of an infected animal:
0 In the Philippines, the most common vectors are dogs
Other animals that can transmit the virus are cats, carabaos, pigs, horses, goats
0

0Rodents and rabbits are not known to transmit rabies

II. CLINICAL MANIFESTATIONS


Rabies has the highest case fatality rate of any human infectious disease
• Once signs of disease are present, rabies usually leads to progressiveencephalopathy and death

A. ProdromaI Stage (Days 0-10)


0Occurs during viral replicationat the site of inoculation just before it enters the neurologicsystem
0Non-specific manifestations: fever, malaise, headache, paresthesia at bite site

B. Acute Neurologic Stage (Lasts for 2-7 days)


0 Virus reaches the nervous system and replicates within the gray matter
0 Two types: encephalitic and paralytic type
TYPE I MANIFESTATIONS

• Combativeness, seizures, autonomic dysfunction (hypersalivation,


gooseflesh, cardiac arrhythmia, priapism)
Encephalitic
• Hydrophobia: involuntary, painful contraction (spasm) of the
(furious) rabies
diaphragm and accessory respiratory/laryngeal/pharyngeal muscles
(80%)
in response to swallowing liquids
• Aerophobia: same features caused by stimulation from a draft of air
• Early and prominent flaccid muscle weakness predominates
Paralytic (dumb)
• Complete paraplegia eventually develops with fatal paralysis of
rabies (20%)
respiratory muscles

C. Coma and Death


° Coma begins within 10 days of onset
0Once disease develops, recovery is rare

III. DIAGNOSIS
Diagnosis is based on a history of exposure (e.g., bite) to a rabid animal & manifestations
Rabies laboratory confirmation may be needed in the paralytic type or atypical
presentations:
° Fluorescent antibody technique (gold standard)
Polymerase chain reaction (PCR)
0

0 Serum rapid fluorescent focus inhibition test (RFFIT)


Histopathology (biting animal should be examined for rabies, which shows the
0

pathognomonic Negri bodies)

IV. MANAGEMENT (FOR RABIES PREVENTION)


Post-exposure prophylaxis (PEP): refers to anti-rabies treatment after exposure to
potentially rabid animals
PEP usually includes local wound care and rabies vaccine with or without rabies
immunoglobulin (RIG)

Supportive Management for Animal Bites


Anti-tetanus immunization (see discussion on Tetanus)
0

Antibiotics for all category III cat bites & all other category III bites that are deep,
0

penetrating, or extensive or located on the hand, face, or genitals (e.g.,co-amoxiclav,


cloxacillin, cefuroxime)
Analgesics and/or anti-pyretics as needed
0

Postpone suturing if possible (if necessary, ensure that RIG has been applied locally)
0

353
Categorization (CAT) of Exposure and Respective Management (WHO Classification)
CAT I EXPOSURE I MANAGEMENT"
• Feeding/touching an animal • No vaccine or RIG
• Licking of intact skin needed
• Exposure to patient with signs of rabies by sharing utensils for • Consider
I eating/drinking pre-exposure
• Casual contact with patient with signs of rabies vaccinationh if
• Routine delivery of health care to a patient with likely to have
manifestations of rabies repeated exposure
• Nibbling/nipping of uncovered skin with or without bruising • Start rabies
II • Minor scratches/abrasions without bleeding (includes wounds vaccine
that are induced to bleed) immediately"

• Transdermal bites or scratches (includes puncture wounds,


lacerations and abrasions) with spontaneous bleeding
• Licks on broken skin or mucous membranes
• Exposure to rabies patient through bites, contamination of mucous
• Start rabies
membranes or open skin lesions with body fluids (except blood/
III feces) through splattering or mouth-to-mouth resuscitation
vaccine and RIG
immediately'
• Handling of infected carcass or ingestion of raw infected meat
• Suspected contact with bats
• All category II exposures on the head & neck are considered
as "category lll"
• Management beginsbywashingexposedareawithsoap& waterfor 10mins;applyalcoholor anyantiseptic
• Pre-exposure prophylaxis(vaccinationadministered beforean exposure) withPVRVor PCECmaybealso
considered for individuals
at highriskfor rabiesexposure(schedule is Day0, 7, and21/28)
c Forthe rabiesvaccine:

• Completeregimenuntilday28 if theanimal: Mayomitday28doseif theanimal:


1.Wasprovenin laboratory to haverabies,or 1. Is aliveandremainshealthyafter14-day
2. Hassigns& symptoms of rabies,or observation period,or
3. Diedor waskilledwithoutlaboratory testing,or 2. Diedwithin14daysbuthadno signsof
4. Cannotbeobservedfor 14days rabiesandwas!FATnegative
Sources:WHOGuidefor Post-Exposure
Prophylaxis.http://www.who.inVrabies/human/postexp/en/
DOHNationalRabiesPrevention
andControlProgram,Manualof Operations;
2018

A. Active Immunization (Rabies Vaccine)


Rabies vaccine induces protective immune response
0

In adults: given on the deltoids (vaccines should not be injected in gluteal region)
0

Human Diploid Cell Vaccine (HDCV) is the gold standard but it is not locally available
0

May be either:
0

• Purified verocell rabies vaccine or PVRV (Verorab), or


• Purified chick embryo cell vaccine or PCEC (Rabipur)

I INTRAMUSCULAR (IM) REGIMEN


I INTRADERMAL (ID) REGIMEN
• PVRV:0.5 mL
Dose • PVRV & PCEC: 0.1 mL
• PCEC: 1.omL

• Standard WHO intramuscular (IM) • Updated 2-site ID regimen:


regimen: one dose on days o, 3, 7, 14,28 one dose on each deltoid (right
• Zagreb regimen schedule (2-0-1-0-1):2 doses and left) on days o, 3, 7, and 28
Course
on day o, then I dose each on days 7 & 21 • May omit day 28 dose for WHO
• Shortened IM schedule (I-I-I-I-o): one pre-qualified vaccines
dose each on days o, 3, 7, 14

Given to • Immunocompromised state


• I:Iematologic conditions when
patients • Chronic liver disease
with IM injections are contraindicated
• Taking 1chloroquine or on systemic steroids

354
B. Passive Immunization: Rabies Immunoglobulin (RIG)
0 Given to those with Category III exposures to provide immediate antibodies at site of exposure
0 Should always be given in combination with rabies vaccine (i.e., day o)

Skin test • Not required • Required before administration


• Total dose infiltrated around & into the wound, administered at the same
time with the first dose ofrabies vaccine (Day o); remainder should be
Administration injected at an IM site distant from vaccine inoculation (e.g.,anterior thigh)
• IfRIG is unavailable on 1st visit (Day o), it may still be given until 7 days
after the 1st dose of the rabies vaccine
'HumanRIGis preferredif thereis historyof hypersensitivity
to equinesera,multiplesevereexposures
wherethedogis suspected to berabid,or symptomaticPLHIV
Sources:WHOGuideforPost-Exposure Prophylaxis.http://www.who.inVrabies/human/postexp/e
DOHNational RabiesPreventionandControlProgram,Manualof Operations;
2018

HUMAN IMMUNODEFICIENCY VIRUS: AIDS & RELATED DISORDERS


I. ETIOPATHOGENESIS
A. Etiologic Agent
° Four retroviruses known to cause human disease
• Human T lymphotropic viruses (HTLV)-1 and HTLV-11:transforming retroviruses
• Human immunodeficiency viruses, HIV-1 & HIV-2: cause cytopathic effects
0 HIV-1 is the most common cause of HIV disease throughout the world
0 Hallmark of HIV disease: profound immunodeficiency resulting primarily from a
progressive quantitative & qualitative deficiency of helper T cells

B. Transmission
0 Primarily by sexual contact: worldwide, heterosexual transmission is still the most
common mode
0 In the Philippines (as of July 2017),males who have sex with males (MSMs) have the
highest rate of infection among the high-risk groups
0 Blood and blood product transfusions
0 Occupational transmission of HIV (e.g., needlestick injuries)
0 Infected mothers to infants intrapartum, perinatally, or via breast milk
0 No evidence that HIV is transmitted by casual contact or that the virus can be spread by insects

II. CLINICAL MANIFESTATIONS

ACUTE HIV SYNDROME I ASYMPTOMATIC STAGE I


(CLINICAL LATENCY)
SYMPTOMATIC STAGE

• Occurs 3-6 weeks after • Median time: IO years (for • Symptoms can appear at
primary infection along untreated patients) anytime
with a burst of plasma • Ongoing and progressive • More severe and life-
viremia HIV disease with active threatening complications
• Symptoms: fever, skin rash, viral replication of HIV infection occur in
pharyngitis, myalgia • Rate of disease progression patients with CD4+ T cell
• Most patients recover is directly correlated with counts <200/uL
spontaneously from this HIV RNA levels
AIDS:
syndrome
• HIV infection+ CD4+ T cell
• Many have only a mildly
count <200/uL; or
depressed CD4+ T cell
• HIV infection + HIV-
count that remains stable
associated diseases
for a variable period before
indicative of a severe defect
beginning its decline
in cell-mediated immunity

355
III. DIAGNOSIS

• Standard blood screening test for HIV infection


ELISA,or
• Sensitivity of >99.5% (not optimal with regard to specificity)
enzyme
• Commercial EIA kit contains antigens from both HIV-1and HIV-2
immunoassay
• Scored as positive (highly reactive), negative (non-reactive), or
(EIA)
indeterminate (partially reactive)
• Most commonly used confirmatory test
Western blot • Western blot demonstrating antibodies to products of all three HIV genes
. (gag,pol, and env) is conclusive evidence of infection with HN
• EIA-type assay: consists of antibodies to the p24 antigen of HIV
p24 antigen • Detects the viral protein p24 in the blood of HIV-infected individuals
capture assay • Greatest use for screening HIV infection in patients suspected of having
acute HIV syndrome

HIV I nucleic • Used when specimens are initially reactive on the antigen/antibody
acid test combination assay, but non-reactive or indeterminate on the HIV-1/HIV-2
(NAT) immunodifferentiation assay
• Formats include agglutination tests, membrane immunoconcentration
Rapid devices, immunochromatographic strips
diagnostic • Usually detect antibodies against HIV-1& HlV-2 but do not differentiate
tests(RDT) between the two
• Results available in 5-30 minutes & allows same-visit post-test counselling

Rapid HIV • An algorithm that uses RDTs with comparable specificity and sensitivity,
diagnostic which will have a shorter turn-around time compared to Western blot
algorithm • Uses 3 rapid diagnostic tests in combination to confirm HIV infection;
(rHIVda) currently, there are 2 immunoassay tests & 3 RDTs selected for this algorithm

B. Laboratory Monitoring of Patients with HIV Infection


DIAGNOSTIC I DESCRIPTION

• Measured directly or calculated as the product of the percent of CD4+ T


cells and the total lymphocyte count
• Best indicator of & correlates with the level of immunologic competence;
important factor to determine if there is a need to initiate prophylaxis
CD4+ Tcell
° CD4+ T cell counts <200/uL: high-risk of disease from P.jiroueci
count
° CD4+ T cell counts <50/uL: high-risk of disease from CMV,
mycobacteria of the M. avium complex (MAC), and/or T gondii
• Measured at the time of diagnosis and every 3-6 months thereafter during
the first 2 years of ART, and for those who were not started on ART
• Number of copies of HIV RNA per milliliter of serum or plasma
• HIV RNA can be detected in virtually every patient with HIV infection
• Used to monitor ART effectiveness (can have viral suppression 8-24 weeks
after ART initiation)
HIVRNA
• CDC recommends monitoring of HIV viral load before initiation of ART,
determination
& 2-4 weeks (not later than 8 weeks) after initiation or modification
• Monitoring of viral load is done at 4-8 weeks until viral suppression is
achieved, then monitoring can be decreased to 3-4 months, or 6 months if
stable for 2 years or more

HN resistance • Drug resistance testing in the setting of virologic failure should be


testing performed while the patient is still on the failing regimen
Sources:CDC.Laboratorytestingforthe diagnosisof HIVinfection:updatedrecommendations. 2014
UnicefHIVDiagnosis.AguideforselectingROTkits.2008
DOHRapidHIVDiagnosticAlgorithm Brief.2018
information
US Departmentof Healthand HumanServices
356
IV. MANAGEMENT
A. Settings where Initial Antiretroviral Therapy (ART) is recommended in the Philippines:
• Presence of an AIDS-defining illness
• CD4 count s350 cells/mm'•
• All HIV-infected pregnant women
• HIV-associated nephropathy (HIVAN) and hepatitis B virus (HBV) or hepatitis C virus
(HCV) co-infection when treatment is indicated (regardless of the CD4 count)
• CDC criteria suggest initiation of ART during early HIV infection (the period up to 6
months after acquiring HIV) ..
*Cut-offset by the U.S. Department of Health and Human Services Guidelines is <500cells/mm3
**Thisis advocated to reduce the risk of transmission of the virus when the viral load is very high

B. Anti-Retroviral (ARV) Regimens in the Philippines

I ANTI-RETROVIRAL DRUGS
First-line NRTI
• Lamivudine (3TC) 300 mg OD+ tenofovir (TDF) 300 mg OD
Alternative first-line
NRTI • Lamivudine (3TC) 150mg BID + zidovudine (AZT) 300 mg BID

First-line NNRTI • Efavirenz (EFV) 600 mg OD HS


Alternative first-line
NNRTI • Nevirapine (NVP) 200 mg BID

Integrase Inhibitor • Dolutegravir (DTG) 50 mg OD

Regimen
.
Protease Inhibitor (Pl) • Ritonavir-boosted lopinavir (LPV/r)

..
w
\'

First-line regimen • 2 NRTI + NNRTI


Alternative regimen • 2 NRTI + lntegrase inhibitor

Second-line regimen
NRTI + Pl (use of 2nd-line regimen is based on adverse reactions
• 2
encountered with first-line regimen and/or resistance testing)
NRTI:Nucleoside Reverse Transcriptase Inhibitor
NNRTI:Non-Nucleoside Reverse Transcriptase Inhibitor
I
V. PRIMARY PROPHYLAXIS FOR OPPORTUNISTIC INFECTIONS
CD4COUNT
DISEASE
I TO START
PROPHYLAXIS I FIRST-LINE DRUG

• TMP-SMX 400/80 mg or 800/!60 mg


Pneumocystis
jiroveci pneumonia S200cells/u L daily
(PCP) • Alternative: Dapsone mo mg daily

Toxoplasmagondii <100cells/ul • TMP-SMX 800/160 mg daily


infection
Mycobacterium • Clarithromycin 500 mg once daily OR
<50cells/ul
avium complex • Azithromycin 1000 mg once weekly

Histoplasmosis <150cells/ul • Itraconazole 200 mg daily


Positive tuberculin
Latent TB skin test (>5 mm • See regimensin Tuberculosissectionof
Infection (LTBI) induration) or Pulmonologychapter
positive IGRA
Sources:Panelon Opportunistic
Infections
InHIV-Infected
AdultsandAdolescents.
CDC,NIH,IDSA;2017
HIVWorkingGroup,PSMID;2015
357
SECTION FIVE
COMMO>.NPARASITIC AND FUNGAL INFECTIONS
MALARIA
I. ETIOPATHOGENESIS
A. Pathogenesis
0 Most important parasitic disease in humans
° Caused by infection of red blood cells (RBC) with protozoan parasites (genus
Plasmodium) inoculated into the human host by a feeding female anopheline mosquito
0 In humans, the erythrocytic cycle is responsible for disease: rupture of schizonts and
release of merozoites present clinically as paroxysms of malaria
0 Hypnozoites are responsible for disease relapses

B. Plasmodium Species
° Five species: P.falciparum, P. vivax, P. ovale, P. malariae, P. knowlesi
0 Transmitted by bite of infected Anopheles mosquitoes (Anophelesjlavirostris in the
Philippines)
0 Severe malaria is often due to P.falciparum (causes nearly all deaths and neurological
complications)

IDuration of I
Can
SPECIES
I MORPHOLOGY

• Trophozoites are seen as ring forms


Erythrocytic
Cycle
Cause
Relapse?

P.falciparum • Banana-shaped gametocytes 48 hours No


• Maurer's dots
• Irregularly shaped large rings and trophozoites
P.vivax • Enlarged erythrocytes 48 hours Yes
• Schilffner's dots
• Infected erythrocytes, enlarged, & oval with
P. ovale tufted ends 48 hours Yes
• Schilffner's dots
• Band/rectangular forms of trophozoites common
P. malariae 72 hours No
• Erythrocytes are not enlarged
• Erythrocytes are not enlarged
• Difficult to accurately diagnose by microscopy
P. knowlesi 24 hours No
because of the morphologic similarity of the
trophozoites to P.falciparum and P. malariae
Sources:CDC.DPDxLaboratory
d1agnos1s diseasesof publichealthconcern2016
paras1t1c
Reviews;2013
SinghBet al. ClinicalMicrobiology

II. CLINICAL MANIFESTATIONS


First symptoms: non-specific (headache, fatigue, abdominal pain, aches & pains);
usually followed by fever, chills, perspiration, anorexia, vomiting, malaise
Disease progression to severe malaria may take a few hours to days (e.g., coma, acidosis,
severe anemia, hypoglycemia)

• P.vivax, ovale(benign tertian)


Tertian • Cyclic fever occurring
• P.falciparum (malignant tertian, but often
periodicity every 48 hours
with no fever pattern)
Quartan • Cyclic fever occurring
periodicity
• P.malariae
every 72 hours
358
B. Classification of Malaria According to Severity
UNCOMPLICATED MALARIA I SEVERE FALCIPARUM MALARIA

• Manifestations of malaria • Presence of P.falciparum asexual parasitemia, AND


• Positive parasitological test • One or more of the following:
(microscopy or ROT) 0 Impaired consciousness (GCS <II)
• No features of severe malaria 0 Prostration (generalized weakness)
(see next column) 0 Multiple convulsions: >2 episodes within 24 hours
0 Acidosis:arterial pH <7-25or plasma HCO3 <15mmol/L
0 Hypoglycemia: <40 mg/dL
0 Severe anemia (Hgb ~7 g/dL or Hct ~20%) +
parasite count >IO0,ooo/uL
0 Renal: creatinine >265umol/L or UO <400 mL/24 hrs
0 Jaundice (bilirubin >3g/dL)+ parasite count> 100,000/uL
0 Pulmonary edema or ARDS
0 Significant bleeding or DIC
0 Shock
0 Hyperparasitemia (P.falciparum parasitemia >10%)
Source:WHO.Guidelinesfor the Treatmentof Malaria,3rd edition,2015

III. DIAGNOSIS
Malaria should be suspected in patients with fever & relevant epidemiologic exposure
All cases of suspected malaria should have a parasitological test to confirm diagnosis
(microscopy or rapid diagnostic test [ROT)) ·
DIAGNOSTIC I COMMENTS/EXPECTED FINDINGS

• Gold standard (identifies parasites on thin and thick blood smears)


Thin and • Acute demonstration of the parasite in the smear:
thick smear 0Thick smear: for quantification of parasitemia
0 Thin smear: for species identification

Rapid • Usually detects Pan-PlasmodiumLOH (not species-specific) and PfHRPz


diagnostic (specific to P.falciparum)
test kits • Some RDTs can differentiate P, vivax
(RDTs) • Used if quality-assured microscopy is not available or during outbreaks
CBC • Normocytic normochromic anemia, thrombocytopenia

• May present with hypoglycemia, hyponatremia, hypocalcemia, elevated


Blood
BUN & creatinine, hypophosphatemia, hypoalbuminemia, hyperuricemia,
chemistry
elevated muscle/liver enzymes, & hyperbilirubinemia

Arterial • Metabolic acidosis (usually high-anion gap)


blood gas • Associated findings: L HCO3, j lactate
PT/PTT • Prolonged in severe cases

Lumbar tar • Mean CSF opening pressure -180 mmHg


(for cerebra
malaria) • Normal or slightly j total protein and cell count

IV. MANAGEMENT
Objective: prevent severe malaria & interrupt transmission via anopheline vectors (in endemic areas)
• Response to treatment must be monitored with daily blood-film microscopy until the end of
administration of first-line drugs, then weekly until the 28th day after initiation of treatment
A. First-Line Treatment Recommendations (according
to theNationalMalariaControlProgramof DOH)
SPECIES
I UNCOMPLICATED
I SEVERE

P.falciparum & Artemether + lumefantrine + primaquine Artesunate IM or IV+ primaquine


P. malariae

P.vivax & P.ovale Chloroquine+ primaquine Artesunate IM or JV+ primaquine

P.knowlesi Artemether + lumefantrine

359
B. Drugs for Treatment
REGIMEN
I CLINICAL USE & SIDE EFFECTS

• First-line drug for confirmed uncomplicated and severe (IV formulation) P.


Artemether-
falciparum malaria (replaced CQ + SP combination as recommended by both
Lumefantrine
DOH and WHO)
(Co-Artem" 1)
• Not recommended for pregnant & children <8 yrs old

Chloroquine+ • Previous first-line treatment in probable and confirmed non-severe falciparum


Sulfadoxine/ malaria
Pyrimethamine • Pyrimethamine can cause megaloblastic anemia, pancytopenia, pulmonary
(CQ+SP) infiltration

• Currently recommended as second-line treatment & in cases of treatment failure


Quinine+
• Quinine can be used for severe malaria if sensitive & iflV artemether is unavailable
Tetracycline/
• Quinine+ clindamycin for pregnant women & children <8 years old
Doxycycline
• Quinine can cause cinchonism, tinnitus, high-tone hearing loss, hypoglycemia

Artesunate • For treatment of uncomplicated (oral) or severe (IV) falciparum malaria

• For radical cure because it eradicates hepatic forms of P. vivax & P. ouale (to
prevent relapse)
Primaquine • Given as single low dose to P.falciparumcases to prevent onward transmission
• Not used to treat the erythrocytic stage of malaria
• Can cause massive hemolysis in G6PD deficiency

Chloroquine • Drug of choice for P. vivax, malariae,ovale and know/esicases

• Acts as a blood schizonticide


Mefloquine
• For prophylaxis or treatment of drug-resistant malaria

C. Drugs for Prophylaxis


0 Doxycycline, primaquine, atovaquone/proguanil, chloroquine, hydroxychloroquine
0 Mefloquine (only prophylaxis permitted in pregnancy)

SCHISTOSOMIASIS
I. ETIOPATHOGENESIS & MANIFESTATIONS
• Parasitic disease endemic in 24 provinces in the Philippines, with highest prevalence in
children 5-15 years of age
Major species involved in the Philippines: Schistosoma japonicum
Snail vector: Oncomelania hupensis quadrasi
Transmission requires skin penetration of cercaria (e.g., during swimming, rafting, boating)
Main pathology is caused by granulomatous reaction to eggs deposited in liver & other organs
• Most are asymptomatic and have a low parasite burden
ETlOPATHOGENESIS
I MANIFESTATIONS

Acute Schistosomiasis
• Systemic illness that develops • Cercarial dermatitis/swimmer's itch: localized
a few weeks after contact with dermatitis at the site of larval entry
infested water • Katayama fever: serum sickness-like syndrome with
• Corresponds to the first cycle of fever, lymphadenopathy and hepatosplenomegaly
egg deposition • Manifestations occur in those not living in endemic areas
• Symptoms usually resolve over a (i.e., travelers), because these are individuals who have not
few weeks, but may be fatal in some yet developed immunity associated with early exposure

Chronic Schistosomiasis
• Liver cirrhosis, portal hypertension, splenomegaly, ascites
• Results from egg-induced • Cor pulmonale, pulmonary granulomatosis and fibrosis
imn1une response, granuloma • Glomerulonephritis
formation, and fibrotic changes • Neuroschistosomiasis
• May present months to years • S. haematobium: associated with bladder cancer
after primary exposure • Manifestations usually seen in individuals with ongoing
exposure in endemic areas

360
III. DIAGNOSIS
DIAGNOSTIC I COMMENTS/EXPECTED FINDINGS

• High-level peripheral eosinophilia in Katayama fever


CBC
• Chronic anemia & thrombocytopenia from splenic sequestration

Blood • Elevated ALP & gamma-glutamyltransferase in hepatic granulomatosis


chemistry • Azotemia if with obstructive nephropathy
Ultrasound • Liver:clay-pipe stem fibrosis with lacelike pattern
• Demonstration of parasite eggs in stool or urine
Microscopic 0 Stool exam: S. japonicum,S. mansoni
identification
of eggs
0 Urine exam: S. haematobium
• Kato-Katz technique: quantifies egg output
Rectal imprint • Biopsy of rectal tissue with the aid of proctoscopy
Circumoval • Simple and inexpensive immunodiagnostic test
precipitin
test(COPT) • Detects circulating schistosome antigens

IV. MANAGEMENT
• Treatment should be supported by a positive result on Kato-Katz for S. japonicum ova by
stool exam or rectal imprint
Preferred regimen: praziquantel 40 mg/kg, divided into 3 doses in I day
For neuroschistosomiasis: dose is increased to praziquantel 60 mg/kg

2013
japonicumInfectionsin the Philippines:
Source:DOH.CPGfor Schistosoma

CANDIDIASIS
I. ETIOPATHOGENESIS
A. Candida
° Cause of Candida infections is mostly from C. albicans (sometimes by C. glabrata,
C. guilliermondii,C. krusei, C. parapsilosis,C. tropicalis,C. kefyr, C. lusitaniae,
C. dubliniensis,C. auris)
0 Inhabit the gastrointestinal tract, female genital tract, and skin
0 Small, thin-walled, ovoid yeasts which reproduce by budding
0 Among the most common nosocomial pathogens

B. Pathogenesis
0 Use of antibacterial agents can alter the normal human micro biota and allow non-
bacterial species to become more prevalent in the commensal flora
0 Most serious Candida infection is the disseminated (hematogenous) form, which
forms small abscesses in major organs
0 The innate immune system is the most important defense mechanism against
hematogenously disseminated candidiasis (neutrophils are the most important
component of this defense)

C. Risk Factors for Hematogenously Disseminated Candidiasis


0 Use of broad-spectrum antibacterial agents
0 Indwelling intravenous and urinary catheters
0 Hyperalimentation fluids or total parenteral nutrition (TPN)
0 Parenteral steroids, cytotoxic chemotherapy, and immunosuppression for organ
transplantation
0 Severe burns, high APACHE scores
0 Neutropenia, HIV with low CD4+ T cell counts
0 Prior abdominal and thoracic surgery
0 Diabetes mellitus

361
II. CLINICAL MANIFESTATIONS
Manifestations range from local mucocutaneous infections to widespread dissemination
with organ failure
Although they are part of the normal microbiota, they may invade and cause disease
when an imbalance is created

• White, adherent, painless, or confluent patches in the mouth, tongue, or


Thrush. esophagus
(oropharyngeal) • Organisms are identifiable in gram-stained scrapings from the lesions
• Thrush in young patients should prompt investigation for HIV
• Most common in HIV-infected patients (considered an AIDS-defining
Esophagi tis illness) and those with hematologic malignancies
• Hallmark is odynophagia (pain on swallowing in retrosternal area)
Vulvovaginal • Pruritus, pain, and vaginal discharge (thin with whitish curds)
candidiasis • Most common form of mucosa! candidiasis
Paronychia • Painful swelling at the nail-skin interface
Onychomycosis • Fungal nail infection
Intertrigo • Erythematous irritation with redness and pustules in the skin folds
• Erythematous-pustular infection of the glans penis
Balanitis
• Can also spread to thighs, gluteal folds, buttocks, scrotum
Folliculitis • Pustules, most frequently in the area of the beard
Deeply Invasive Cq,ndidiasis
• May or may not be due to hematogenous seeding
• More commonly due to hematogenous seeding of various organs due to candidemia (most
commonly the brain, chorioretina, heart, and kidneys)
• Candidemia refers to the presence of Candida species in the blood

III. DIAGNOSIS
Diagnosis is established by visualization of pseudohyphae or hyphae on wet mount (saline
& 10% KOH), tissue Gram's stain, periodic acid-Schiff stain, or methenamine silver stain

IV. MANAGEMENT
A. Mucocutaneous Candida Infection
DISEASE I PREFERRED TREATMENT I ALTERNATIVES
Thrush (Oral) • Clotrimazole troches • Fluconazole, nystatin
• Oral fluconazole
Vulvovaginal • Nystatin suppository
• Azole cream or suppository
Cutaneous • Topical azole • Topical nystatin

• Fluconazole tablets (100-200 mg/day) • Caspofungin, micafungin,


Esophageal
• ltraconazole solution (200 mg/day) amphotericin B

B. Candidemia and Hematogenously Disseminated Candidiasis


'All patients with candidemia are treated with a systemic antifungal agent
'No agent within a given class has been identified as superior to the others (but there is
a trend to treat with an echinocandin until sensitivity or speciation is determined)
0 Some agents include:

• Amphotericin B
• Azoles (fluconazole, voriconazole, posaconazole)
• Echinocandins (micafungin, caspofungin, anidulafungin)
362
SECTION SIX
SEXUALLY TRANSMITTED DISEASES
SEXUALLY TRANSMITTED DISEASES (STD)
Refer to a variety of syndromes caused by pathogens that can be acquired and
transmitted through sexual activity
Frequently asymptomatic but can lead to various complications
DISEASE
I FEATURES
I TREATMENT
Mucopurulent
Cervicitis (MPC) • Presence of yellow
• N.gonorrhea mucopurulent discharge Treat gonorrhea (unless excluded)
• C. trachomatis from the cervical os • Ceftriaxone 250 mg IM single dose (SD);or
• M. genita/ium • Cefpodoxime 400 mg PO SD; or
• Cefixime 400 mg PO SD
Urethritis in Men
• N. gonorrhea • Urethral discharge PLUS treatment for chlamydia! infection
• C. trachomatis • Dysuria • Azithromycin I g PO SD; or
• M. genita/ium • Usually without frequency • Doxycycline IOO mg BID PO for 7 days
• U. urealyticum
• T. vagina/is
Epididymitis • Ceftriaxone 250 mg IM SD+
• Unilateral pain
• C. tracliomatis Doxycycline IOO mg BID x IO days
• Swelling and tenderness of
• N. gonorrhea • Levofloxacin 500 mg OD x IO days (for
the epididymis
(less common) Enterobacteriaceae)

Outpatient:
• Ceftriaxone 250 mg IM SD; PLUS
• Doxycycline IOO mg PO BID
• Lower abdominal pain for 14 days; PLUS
Pelvic <3 weeks duration • Metronidazole 500 mg PO BID for 14days
Inflammatory • Pelvic tenderness
Disease on bimanual pelvic Inpatient (continue until 48 hrs of improvement,
• N. gonorrhea examination then change to outpatient therapy):
• C. trachomatis • Evidence of lower genital • Regimen A: Cefotetan 2 g IV q12 or
tract infection cefoxitin 2 g IV q6 PLUS doxycycline
IOO mg IV or PO q12
• Regimen B: Clindamycin 900 mg IV q8
PLUS gentamicin 1.5mg/kg IV q8
Vulvovaginal Infections
Vulvovaginal • Vulvar itching &/or irritation • Fluconazole 150mg PO single dose
candidiasis • Scanty, white, clumped • Azole cream, tab or suppository:
• C. a/bicans discharge miconazole, clotrimazole
• Vulvar itching
Trichomonal
• Often profuse white or • Metronidazole 500 mg PO BID for
vaginitis
yellow homogenous 7days
• T. vagina/is
discharge
• Malodorous
• Slightly increased discharge
Bacterial
• Clue cells: vaginal
vaginosis • Metronidazole 500 mg PO BID for 7 days
epithelial cells coated with
• Gardnerella • Topical: metronidazole gel, clindamycin
coccobacillary organisms
vagina/is
giving them a granular
appearance

363
• Sharply demarcated, elevated and
Primary syphilis
round ulcers
(syphilitic
• Lymph nodes are bilateral, firm
chancres)
and nontender

• Generalized, symmetric, non-


• Benzathine penicillin 2.4 million
Secondary pruritic mucocutaneous lesions
units IM as single dose
syphilis • Affects palms and soles
• Doxycycline IOO mg PO BID for
• Includes condyloma lata
14 days (alternative)
• Positive syphilis serology without
any signs and symptoms detected
Early latent
within 2 years (from untreated
syphilis
primary or secondary syphilis, or
from an initial negative serology)
• Positive syphilis serology without
any signs and symptoms detected • Benzathine penicillin 2.4 million
Late latent after ~2 years (from untreated units IM once weekly x3 weeks
syphilis primary or secondary syphilis, or • Doxycycline IOO mg PO BID for
from an initial negative serology, 30 days (alternative)
or unknown duration)

• Includes cardiosyphilis and • Benzathine penicillin 2.4 million


Tertiary syphilis
gumma units IM once weekly

rJlcerative G.enitalor Pel'ianal Lesions


• Acyclovir 400 mg PO TIO for
7-10 days or 200 mg PO 5x a day
• Erythematous ulcers for 7-IO days; or
Herpes
• Lymph nodes are firm, tender, • Valacyclovir I g PO BID for
• HSV
often bilateral 7-10 days; or
• Famciclovir 250 mg PO TIO for
7-10 days

• Undermined, ragged & irregular


• Ciprofloxacin 500 mg PO; or
ulcers
Chancroid • Ceftriaxone 250 mg IM; or
• Lymph nodes are tender, may
• H. ducreyi • Azithromycin I g PO
suppurate, loculated, usually
(all administeredas single dose)
unilateral
• Elevated, round or oval ulcers
Lymphogranuloma
• Lymph nodes are tender, may • Doxycycline IOO mg PO BID for
venereum
suppurate, loculated, usually 21 days
• C. trachomatis
unilateral

• Doxycycline IOO mg PO BID


Donovanosis • Elevated, irregular ulcers
for at least 3 weeks and until all
• K. granulomatis • Pseudobuboes
lesions have completely healed
Source:JamesonJL, et al. Harrison'sPrinciplesof InternalMedicine20thedition,2018.
WHO.Guidelines for thetreatmentofTreponemapallidum(syphilis).2016

364
SECTION SEVEN
IMMUNIZATION
VACCINES AND INDICATIONS

VACCINE TYPE/ROUTE I INDICATION I SCHEDULE

Influenza
• Trivalent inactivated
• Single dose annually
vaccine: IM or intradermal
(usually between February
• Quadrivalent vaccines: • Recommended for all adults
to June, but may be given
live attenuated or split-
anytime)
inactivated
-
Cholera
• Killed whole cell • Not routinely given • Killed whole cell monovalent
monovalent (01) vaccine • Vaccination is tailored to high (01) vaccine: 2 liquid doses
with cholera toxin B subunit risk areas for cholera infection with 2-6 weeks interval
(WC-BS or WC-rBS) • Specific age group and people , Modified killed bivalent
• Modified killed bivalent (e.g., travelers in areas with whole cell vaccine: 2 liquid
whole cell vaccine outbreaks) may be considered doses with 14-day interval
Pneumococcal
• Comorbidities (chronic lung • PCV-13followed by PPSV-23
disease, asthma, liver disease, at least I year after PCV-13
cardiovascular disease, DM) • Booster may be given to:
• Immunocompromised 0 ~65' (or ~50•) years old if
• Conjugate (PCV-13):IM
(asplenia, HIV) first dose >5 years ago &
• Polysaccharide (PPSV-23):
• Residents of nursing homes or before age 65' (50•) years
IM or SC
long-term care facilities 0 <65' (or <so•) years

• Smokers and alcoholics old who received


• Immunocompetent elderly vaccine >5 years ago &
adults (~65' or ~50• years) immunocompromised
Human Papillomavirus (HP.V) ...
• Females: 3 doses at age II or
12and those aged 13-26if not
previously vaccinated
• Bivalent (Types 16, 18)for
• Males: 3 doses at age II or
females only: IM • Three doses:
12and those aged 13-21,if 0 Bivalent: o, 1,6 months
• Quadrivalent (Types 6,
not previously vaccinated
11, 16,18) for females and 0 Quadrivalent o, 2, 6 months
(some aged 22-26may also be
males: IM
considered for vaccination - e.g.,
men who have sex with men,
immunocompromised, HIV)
Meningococcal
- ...

• Adults traveling to endemic • MPSV4 0.5 mL SC or


areas for meningococcal disease MenACWY-D 0.5 mL IM as
• Polysaccharide vaccine
• High-risk groups: asplenia, a single dose IO days before
(MPSV): IM or SC
immunocompromised (e.g., travel
• Quadrivalent diphtheria
PLHIV), immunodeficiencies; • For high-risk groups:
conjugate vaccine
personnel handling N.meningitidis 2 doses 2 months apart,
(MenACWY-D): IM
isolates; close contacts of with a booster vaccination
meningococcal disease patients every 5 years
a CDCRecommendation
CPG(PSMID)Recommendation
b Philippine
365
• ViPS: 1 dose should be
given ~2 weeks before
• Vi Polysaccharide • Travelers to outbreak areas exposure (booster dose:
(ViPS): IM • Health care workers and lab technicians 1 dose IM every 3 years
• Live attenuated: • Adults exposed to S. typhi cases for travelers)
oral • Those in refugee camps & disaster areas • Live attenuated: enteric-
coated capsules (oral)
every other day for 1 week

Hepatitis A
• MSM & illicit drug users
• Persons working with the virus in a • Single dose
• Inactivated research laboratory setting • Booster dose between
vaccine: IM • Chronic liver disease (CLO) & recipients 6-12 months after
of clotting factor concentrates primary course
• Travelers to endemic countries
Ifepatitis ,B
• Those sexually active not in monogamous
relationship & those with STD
• MSM & injection-drug users
• Exposed healthcare personnel
• Inactivated • Three doses: o, 1, 6
• Comorbids: ESRD, CLO, HIV
vaccine: IM months
• Household contacts & sex partners of
chronically-infected persons
• Clients & staff of institutions for
developmental disabilities
VaricellaZo§ter,(G'hickenpox)
• Close contact with those at high risk for
severe disease (e.g., health personnel &
family contacts of immunocompromised
• Two doses: o, 1-2 months
persons) or at high risk for exposure or
• Single dose post-
• Live attenuated transmission (e.g., teachers)
exposure prophylaxis:
vaccine: SC • Residents/staff of correctional institutions
within 72 hours of
• Military personnel
exposure
• Adolescents/adults living in households
with children
• Non-pregnant women of childbearing age

• Elderly ~60 years, regardless of whether


• Single dose
they had a prior episode of herpes zoster

• Measles & mumps: college students,


• Live attenuated health care workers, travelers
• Two doses: o, I month
vaccine: SC • Rubella: non-pregnant women of
childbearing age
'nengu11.,
• Individuals aged 9-45 years old
• Recombinant
• There are issues on safety for • Three doses:
live attenuated
seronegative individuals (e.g., those with o, 6, 12 months
tetravalent: SC
no previous dengue infection)

366
VACCINE I INDICATION I SCHEDULE

JapaneseEncephalitis
• Live attenuated • Individuals aged ~9 months
recombinant, • Travelers going to endemic areas: Bangladesh,
chimeric vaccine Bhutan, Brunei, Burma, Cambodia, China, India, • Single dose (for
(JE-CV):SC (only Indonesia, Japan, Korea, Laos, Malaysia, Nepal, adults >17years old)
FDA-approved Papua New Guinea, Philippines, Singapore,
vaccine locally) Taiwan, Thailand, Timar Leste, Vietnam
-
,Rabies II,

• Pre-exposure prophylaxis (PrEP): health


• Purified Vero • PrEP: Three doses
care workers caring for patients with rabies,
Cell(PVRV) on o, 7, and 21/28
personnel in rabies diagnostic centers, pet
• Purified Chick days
owners, animal handlers, veterinarians
Embryo Cell • PEP: see sectionon
• Post-exposure prophylaxis (PEP): see section
(PCEC) Rabies
on Rabies
Beforeadministering vaccines,particularly
in specialpopulations(e.g.,pregnant,immunocompromised),
the
clinicianmustbe familiarwiththecontraindications
to eachvaccine(beyondthescopeofthischapter).
TheMMR,
varicella,andherpeszostervaccinesarecontraindicated in pregnancyandin immunocompromisedpatients.

SUMMARY OF SCHEDULE FOR VACCINATION IN ADULTS


VACCINE I 19-21 yrs I 22-26 yrs I 27-59 yrs I 60-64 yrs I :?65Years
Influenza 1 dose annually

MMR 1 or 2 doses dependingon indication

VAR 2 doses
RZV 2 doses
HPV

PCV-13 1 dose

PPSV-23 1 dose
MMR:measles, mumps,
rubella 11 Recommended foradultswhomeettheagerequirement, lackof
VAR:varicella [.___J documentation
ofvaccination
orlackofevidenceofpastinfection
RZV:zosterrecombinant
HPV:humanpapillomavirus 1111 Recommended
conditions
foradultswithadditional
indications
or medical

PCV-13:
pneumococcalconjugate
PPSV-23:pneumococcalpolysaccharide C::JNorecommendation
'Depends on age at initial vaccinationor condition
Source:Recommendations
of theAdvisoryCommittee
for Immunization
Practices(ACIP).CDC;2021

REFERENCES
1• Sta~
~iri:!:~eG!~!~f
Altr!'aFBt~!i.t~~odn ~fi:::;.~~t~~
t~.e ~eds
Tetanus Unit In: Ellis M. ed. Symposium on
1 tetanus in
2. Baddour LM, Wilson WR. Bayer AS, Fowler VG Jr,Tleyjeh IM, Rybak MJ, et al. AHA Scientific Statement InfectiveEndocarditis
in Adults: Diagnosis, Antimicrobial Therapy, and Management of Complications: A Scientific Statement for Healthcare
Professionalsfrom the American Hean Association.Circulation. 2015;132(15): 1435-86.
3. Belizario VY and De Leon WU. Philippine Textbook of Medical Parasitology.Manila: The Publications Program, Information,
Publication and Public AffairsOffice,Universityof the Philippines-Manila;1998.
4. BeMen JE,Dolin R.BlaserMJ.Mandell, Douglas,and Bennen's Principlesand Practiceoflnfectious Diseases.8th Ed. USk Elsevier
Health Sciences. 2014-
5.Bongard E.Frimodt-Moller N, Gal M, Wootton M, Howe R.Francis N, e1al. Analytic laboratory performance of point of care urine
culture kit for diagnosis and antibiotic susceptibility testing. Eur J Clin Microbial Infect Dis. 2015;34(10):2111-2119.
6.Cascella M, Rajnik M, Aleem A, et al. Features, Evaluation, and Treatment of Coronavirus (COVID-19)!Updated 2021Apr 20). In:
StatPearls [Internet).Treasure Island (FL):StatPearls Publishing. 2021Jan. Availablefrom: www.ncbi.nlm.nih.gov/books/
7. Cemers for DiseaseControl and Preventionand Associationof PublicHealth LaOOratories. Lalx>ratoryTestingfor the Diagnosisof HIV
Infection:UpdatedRecommendations.Availableat hiqrJ/dx.doi.org/10.15620/cdc.23447. PublishedJuneT/,204- AccessedOctoberI, 2m7.
8.Center for Disease Control and Prevention. CDC Adult Immunization schedules. 2015.Availableonline: www.cdc.gov/vaccines/
schedules/hcp/adulthtml.
367
9.Centers for Disease Control and Prevention.DPDx- Laboratory identification orparasites of public health concern. 2016.Available
online: w,vw.cdc.gov/dpdx/malaria/
JO. and Prevention of Vaccine-Preventable Diseases. Tetanus. Available
II. Centers for Disease Conrro1 anS Prevention. Underll'lllg medical conditions associated with high risk for severe
COVID-19: Infonnation for healthcare providers. Avai able online: '"'"'v.cdc.gov/coronavirusl2019-ncov/hcp/clinical-care/
underlyingcondirions.html. Accessed September 13,2021.
12. Centers for Disease Control and Prevention. Vaccine Recommendations and Guidelines of the Advisory Comminee for
13. ~o~i~ ~;·a:~~~
1 6
iu~d~li~~~v\;~~1.ct~~~~~/t?:~;~11:ci~i:~;
Health. Availableonline: \\'Ww.covid19rreatmen1guidelines.nih.gov.
Treatment Guidelines. National Institutes of
AccessedSept 2021.
14 Coronel RF, Rosario MC, et al. Comminee on Adult Immunization 20!8. Philippine clinical practice guidelines for adult
immunization. Philippine Society for Microbiologyand Infectious Diseases.Zurbano Publishing &Prining Corp, Makari 2018
15. Departmem of Health, Philippines. Clinical Practice Guidelines for the Diagnosis, Treatment and Prevention of Schis1osoma
japonicum Infections in the Philippines:2013Update. DOH Manila; 2013.
16. Depanment of Health, Philippines. Interagency Committee on Antimicrobial Resistance. National Antibiotic Guidelines. DOH
Manila; 2017.
17. Depanment of Health, Philippines. Nationa1Malaria Control Progr.im:Manual of Operations, 5th edition. DOH Manila; 2014.
18. Department of Health, Philippines. Nationa1Rabies Preventionand Control Program, Manual of Operations. DOH Manila; 2012.
Availableonline: w,..,..v.doh.gov.ph.
19. Depanment of Health, Philippines. Revised Dengue Clinical Case Management Guidelines 2011.National Dengue Prevention
and Control Pro~; Nationa1Center for Disease Preventionand Control (DOH-NCDPC),Sta. Cruz, Manila; 2011.
8ilt~rfN~ihami!~HF:S:!i~S~~vi~ rt~Jhe~sf;W,fi~l:kn&1!~~~:~i;1i~t~f~i~ ~M(e~it~~~~·;is~:~r~~
22. o:nJd~~~J.t;grs;?~
~i;1n~~ 1 1
!~{s:~~~s~ A~1~b~?~~~~~rr~~~;;·s;~~\7.
23. Habib G, Lancellotti P, Antunes MJ, Bongiomi MG, Casafta JP, Del Zotti F, et al. Taskforce for the Management of Infective
Endocarditis of the European Society of Cardiology (ESC).2015ESC Guidelines for the management of infective endocarditis.
Eur Heart J.2015;36(44),3075-128.
24. HfV Working Group: Philippine Society for Microbiologyand Infectious Diseases.Clinical practice guidelines on the prevention,
diagnosis, and treatment of opportunistic infections in-human immunodeficiency virus-infected adults and adolescents in the
Philippines. PSMID; 2015.
25. Hooton TM, Bradley SF, Cardenas DD, C3olgan R.Geerlin~s SE, Rice JC, et al. Diagnosis, Prevention,and Treatment of Catheter
Associated Urinary Tract Infection in Adults: 2009 Intemauonal Clinical Practice Guidelines from the Infectious Disease Society
of America (IDSA).Clin Infect Dis. 2010;50(5):625-663.
26. Institute of Clinical Epidemiology,National Institutes of Health UP Manila, Philippine Society for Microbiologyand Infectious
Diseases. Philippine COVID-19Living Recommendations. 9 May 2021.Availabfe online: w,w1.psmid.org/philippine-covid-19-
living-recommendations/.AccessedSep 2021.
27. !to~ks~17a~tmrE~~c~~~~ 2~~.Fauci AS, Hauser SL,LoscalzoJ.Harrison's Principlesoflntema1 Medicine. 20th Edition. New

;:: ~XY:1c~dt t t~;kJi~~d~~~j~f~~~f;l!c:t~!:


£fS.~~;t~(~d~~~i~o!;:t~~:;r:~~
~sHl~~ 0
1
~!~.~~~:~?2
JO. ~~an~.n~~;1.-~~~lip~~eT,~~t~r-2h~ll!':!!:d;~~u~~·;~~~siri'!ss~~ ~eting inflammatory responses to SARS-CoV-2
infection:A narrative review:J Clin Med. 2020,9(12),4021.
JI. r:s~~fJ~;_f '!J~~~ t:c~:f ~~~~:~~~f.atient Availableon line:\V\\'\"uptodace.com/comems/fever-and-
32· Mackm\;ak PA,Wassennan SS, ~vine MM. A criticJap}raisal of 98.6degrees F,the upper limit of the nonnal body temperature,
33. Mj~t!sh fEfstC'ci~~~~~i:c~.li~f!~?~~~1~W!t1e online: \\'\\'W.Uptodate.com/contents/covid-19-
34. ~~~~u~-[E,tB~·1~;s:c~f~1R%~e JC, Schaeffer A, and Hooton TM. Infectious Diseases Society of America (IDSA)Guidelines
35. ~rttt~~~iw~~d ~d:~ie~~g,!i~:ao:t~i;i~~a!~~~e;~rth~nra;~u~f\:;~~~~~~~\~~~:~~~(50~;~· in the managed health
care era. Infect Dis Clin North Am 1997;u:593-
36. Petersdorf RF, Beeson PB. Feverof une.xplainedorigin: rep<>rton 100cases. Medicine(Baltimore);1961:40:1.
37. Philippine SocietyofVascular Medicine. Role of Anriocagulation in COVID-19:Recommendations on its Use.October 2021http://
W\.\l\\'.psvm.oi:g:.ph/PSVM_2021_A.i"\JTICOAGULATIONS:html. Accessed November 2021
38. Schnarr J and Smaill F.Asymptomatic bacteriuria and symptomatic urinary tract infections in pregnancy:Eur J Clin Invest 2008
39. Shenofi W. Narar VS, Chowdhury AA, Bha1gatPS, and Juvale NI. Pulmonary leptospirosis: an excellent response to bolus
1 9
40. Si~~h~g~~rB~n:~h~ E1f~'!:\;~i~~; 1~d~!;~~tion ofPlasmodium knowlesi.Clin Microbiol Rev.2013;26(2):165-184.
41. Smaill FM and Vazquez JC. Antibiotics for asymptomatic bacteriuria in pregnancy. Cochrane Daiabase Syst Rev 2015Aug.
(8),CD000490.
42. Task Forceon Typhoid Fever.The Philippine Clinical PracticeGuidelines on Typhoid Fever.Diagnosis.Treatment, and Prevention
of Typhoid Fever in Adults. Philippine Societ).'for Microbiologyand Infectious Disease. Quezon City,Philippines; 2017.
43. Task "Forceon Urinary Tract InTections,Philippine Practice Guideline Group Infectious Disease. Philippine Clinical Practice
Guidelines on the Diagnosis and Management of Urinary Tract Infections in Adults, 2013Update: Complicated Urinary Tract
Infections in Adults. Quezon Ci~ Philippines:PPGG-ID Philippine Society for Microbiologyand Infectious Disease; 2013.
44. Task Force on Urinary Tract Infections, Philippine Practice Guideline Group Infectious-Disease. Philippine Clinical Practice
Guidelines on the Diagnosis and Management of Urinary Tract Infections in Adults, 2013Update: Uncomplicated Urinary Tract
Infec1ion.Quezon City,Philippines: PPGG-ID Philippine Society for Microbiologyand Infectious Disease;2013.
45. Task Force on Urinary Tract Infections, Philippine Practice Guideline Grou~nfecrious Disease. Philippine Clinical Practice
Guidelines on the Diagnosis & Management ofUTI in Adults, 2014Update: mptomatic Bacteriuria and Recurrem Urinary
Tract Infection. Quezon Cit); Philippines: PPGG-ID Philippine Society for Micro iofogy& Infectious Disease;2014.
46. Task Force on Urinary Tract Infections, Philippine Pracnce Guidelines Group Infectious Disease. Urinary Tract Infections in
Adults 2015Update (Part 2): 05)'nlptomaric Bacteriuria, Recurrent Urinary Tract Infection, and Complicated Urinary Tract
Infection. PhilipJ?ineSociety of Microbiolow and Infectious Diseases Volume 3 No. 1Quezon City,Philippines
47. The Lep1ospiros1sTask Force:The Philippine Society of Microbiologyand Infectious Disease, Philippine Society of Nephrology,
and Philippine College ofChesc Physicians.LeptospirosisClinical Practice Guidelines; 2010.
48. Tropical Disease Foundation, Inc. Guidelines on Antimicrobial Therapy. 2012.
49. World Health Organization and International Leptospirosis Society. Human Leptospirosis: Guidance for Diagnosis,
Surveillance, and Control. \Vorld Health Organization, Geneva, Switzerland; 200J.
50. World Health Organization. Background document: The diagnosis, treatment and prevention of typhoid fever.World Health
Organization, Geneva, Switzerlana:;2003.
51. World Health Organization Communicable Diseases Working Group on Emergencies, Communicable Diseases Surveillance
and Response. Current Recommendations for Treatment of Tetanus during humanitarian emergencies. WHO Technical Note.
World Health Organization. Geneva Switzerland; 2010.
52. World Health Organization. Dengue Guidelines for diagnosis, treatment, prevention and control: New Edition 2009. World
~D~f; ~;ga,~~~t1~b1~~h~l ~~~;~;~h~.\~eu;~;i3eW:e~'9;8~~417~~~Ue~~1~~e~:!~i30~~;b~r 7:~~tg in Tropical Diseases
53. World Hearth Organization bepanment of Neglec1edTropical Diseases-Neglected Zoonotic Diseases Team. Rabies Pre and
Post-Exposure Prophylaxis in Humans. Revised June 2010.World Health Organization, Geneva, S,vitzerland; 2010.Available at
http://ww,v.who.int/rabies/human/postexp/. Accessed October 1,2017.
54. World Health Organization. Guidelines for the Treatment of Malaria, 3rd edition. World Health Organization (WHO) Geneva
Switzerland; 2015.
55. Yip WCL Dengue haemorrhagic fever:current approaches to management Medical Progress; 1980;7(13):201-209.
368
', GASTROENTER
APPROACH TO DISEASES IN GASTROENTEROLOGY
COMMON GASTROINTESTINAL TRACT (GIT) COMPLAINTS
TERM I DEFINITION
Anorexia • Loss or lack of appetite
Early satiety • Inability to eat a full meal
Heartburn • Retrosternal burning sensation due to excess gastroesophageal reflux
Dysphagia • Difficulty in swallowing
Odynophagia • Painful swallowing
Diarrhea • Feces are discharged from the bowels frequently and in liquid form
Constipation • Difficulty in emptying the bowels, usually with hardened feces
Obstipation • Complete constipation with no passage of either feces or gas
Tenesmus • Intense urge with straining but with little or no result
Hematemesis • Vomitus of red blood or coffee-ground material
• Black, tarry, foul-smelling stool which usually implies bleeding
proximal to the ligament ofTreitz (upper GI bleed), and that blood has
Melena
been in the GI tract for at least 14 hours
• Generally occurs when at least 50-100 mL of blood is present in the GIT
• Passage of bright red or maroon blood from the rectum which usually
implies bleeding from the colon (lower GI bleed) or anorectum
Hematochezia
• May also come from an upper GI source on a background of rapid
intestinal transit (e.g., massive hemorrhage)
Occult GI • Identified by a fecal occult blood test (FOBT) or fecal immunochemical
bleeding test (FIT) in the absence of overt bleeding
• Yellowing of the skin or sclerae, arising from obstruction in the bile
Jaundice
ducts, liver disease, hemolysis, etc.
Other • Indigestion, nausea, retching, regurgitation, vomiting, excessive gas,
complaints fullness, pain, weight loss

PHYSICAL EXAMINATION OF THE GASTROINTESTINAL TRACT (GIT)


I. INSPECTION
INSPECT I REMARKS I EXAMPLES OF PATHOLOGIES
• Dilated veins in portal hypertension
• Note presence of scars, striae, rashes, & in inferior vena caval obstruction
Skin
ecchyn1oses and dilated veins • Pink-purple/violaceous striae in
Cushing syndrome
• Umbilical bulge suggests ventral
Umbilicus • Observe contour and location
hernia
• Fla~ rounded, protuberant or scaphoid
Contour • Check for local bulges, asymmetry, • Bulging flanks in ascites
visible organs or masses
• Observe movements for several • Prominent peristaltic wavesin
Peristalsis
minutes patients with intestinal obstruction
371
II. AUSCULTATION
Done prior to percussion or palpation to prevent altering the frequency of bowel sounds
With the stethoscope's diaphragm placed gently on the abdomen (usually at the right
lower quadrant or RLQ), listen for bowel sounds and note frequency and character
Normal sounds consist of clicks and gurgles (5-34 per minute)
FINDINGS I REMARKS

Borborygmi • High-pitched rumbling sounds ofhyperperistalsis


• Generally suggest vascular occlusive disease
Bruits • Renal anery stenosis: bruits in midline between subxiphoid & umbilicus
• Hepatocellular carcinoma: bruits heard over right upper quadrant (RUQ)
• Present in HCCA, hepatic metastasis and abscess, gonococcal infection
Friction rubs
around the liver, splenic infarction and pancreatic carcinoma

III. PERCUSSION
• Percuss the abdomen lightly in all four quadrants
Assesses the amount and distribution of gas in the abdomen, characteristics of palpable
masses (whether solid or fluid-filled), and the size ofliver and spleen
Tympany: predominant percussion tone due to gas in the abdomen
Dullness may signify presence of underlying mass, organ, fluid or feces
ORGAN I PERCUSSION
I FINDINGS

• At the right midclavicular line:


• Locate lower border ofliver dullness: stan below
• Normal liver span is:
umbilicus in RLQ (start in an area of tympany
• 6-12cm at the right
then follow cranially until dullness is noted)
Liver midclavicular line
0 Locate upper border of liver dullness: start at
0 4-8 cm at midsternal line
the nipple line then lightly percuss from lung
resonance down toward liver dullness
• Measure the distance between the two points
• If tympany is prominent,
• As spleen enlarges, it expands anteriorly,
splenomegaly is unlikely
downward and medially and replaces the general
• Splenomegaly suggested by:
tympanitic tone with dullness
Splenic • Obliterated Traube's space
• Percuss the left lower anterior chest wall from
• Splenic percussion sign:
the border of cardiac dullness at 6th rib to the
change from tympany to
anterior axillary line & down to the costal margin
dullness on inspiration
IV. PALPATION (LIGHT & DEEP PALPATION)
• Light: with fingers together, palpate the abdomen with gentle dipping motion to elicit
tenderness, muscular resistance and presence of superficial organs and masses
Deep: using palmar surfaces of fingers, press down deeper in all quadrants to delineate masses &
characterize location, size, shape, consistency, tenderness, mobility & pulsations
Involuntary rigidity & rebound tenderness are associated with peritoneal inflammation
I ,

ORGAN I PALPATION

• Place the left hand behind the patient at the level of the right mh and 12thribs
• Place the right hand on the patient's right abdomen lateral to the rectus muscle
Liver with fingertips below the lower border of liver dullness
• Ask the patient to breathe deeply and then try to feel the liver edge as it comes in
contact (note its consistency, presence of contour irregularity and any tenderness)

• At the patient's right side, reach over and around the patient using the left hand, to
suppon and press forward the lower left rib cage
Splenic • Place the right hand below the left costal margin and press in toward the spleen
• Ask patient to take a deep breath: feel splenic edge & note contour/tenderness
• Measure the distance between the spleen's lowest point and left costal margin

372
s . · I Ti t 1s· . th Abd
FINDINGS I DESCRIPTION AND DIFFERENTIALS
Shifting
• Dullness shifting to the more dependent side is seen in ascites
dullness
• Palpable impulse on the side opposite the pressure (with hands
Fluid wave
pressed firmly on the midline of the abdomen) suggests ascites
• Pain in RLQ during left-sided pressure suggests appendicitis
Rovsing's sign • Also associated with referred rebound tenderness (RLQ pain on
withdrawal of pressure on left side)
• Abdominal pain on hip flexion and/or extension secondary to
Psoas sign
irritation of psoas muscle by an inflamed appendix
• Right hypogastric pain on internal rotation of right hip suggests
Obturator sign
irritation of obturator muscle by an inflamed appendix
• Extreme pain on light touch over the area bound by the umbilicus,
Cutaneous
pubic tubercle and anterior superior iliac spine
hyperesthesia
• May also be seen in appendicitis
• Sharp increase in RUQ tenderness with a sudden stop in inspiratory
effort (while pressure is applied under the costal margin lateral to
Murphy's sign the border of the rectus muscle) is seen in acute cholecystitis
• Same procedure may enhance hepatic tenderness (due to multiple
causes), but pain is usually less localized
Source:BickleyLS,et al. Bates'Guideto PhysicalExaminationandHistoryTaking.11thedition.2013
JamesonJL,et.al.Harrison's Principles
of InternalMedicine,
20thedition.2018

V. DIGITAL RECTAL EXAMINATION (DRE)


Part of a full examination to access key structures (e.g., rectum, anus, prostate, perineum)
• Done to detect several diseases in gastroenterology, urology, or gynecology (e.g.,
hemorrhoids, prostate disorders, anal/rectal neoplasms, anal fissures)
Procedure
• Perform handwashing, wear examination gloves, and lubricate index finger
• Ask patient to lie on his/her left side, flexing knees, with buttocks close to the edge of the table
"
• Spread buttocks apart and inspect the immediate area for skin lesions, ulcers, masses or
inflammation
• Ask patient to strain down while placing the pad of your lubricated gloved finger over the anus
• As the sphincter relaxes,gently insert your fingertip in the direction pointing towards umbilicus
• If severe tenderness is present, place fingers on both sides of the anus & gently spread the orifice
• Ask patient to strain down: assess anal sphincter tone, any tenderness, induration, nodules
or irregularities and presence of tender lesions such as anal fissures
• Insert the gloved finger as far as possible then rotate the hand clockwise and
counterclockwise to palpate as much of the rectal surface as possible
• In males: palpate the posterior surface of the prostate gland & note its size,shape & consistency

• Muscles of the anal sphincter tighten snugly around the finger


Normal • To check for external sphincter tone, ask patient to bear down & squeeze
anal
rectal muscles
sphincter
0 Sphincter tightness: may be noted in anxiety, inflammation or scarring
tone
0 Sphincter laxity: may be seen in some neurologic diseases
Induration • May be due to inflammation, scarring or malignancy
Skin tags • Soft, pliable tags of redundant skin at anal margin

373
COMMON DIAGNOSTICS IN GASTROENTEROLOGY
I. BASIC LABORATORY TESTS
DIAGNOSTIC I INDICATIONS AND POSSIBLE FINDINGS
• Microcytic hypochromic anemia in chronic blood loss or chronic disease
• Megaloblastic anemia in vitamin B12deficiency from small-intestinal,
Complete blood
gastric or pancreatic disease
count(CBC)
• Leukocytosis from inflammatory conditions
• Leukopenia in viremic illness
Electrolytes • Electrolyte abnormalities from severe diarrhea or vomiting
Hepatobiliary • Elevated transaminases in hepatic inflammation
system tests • Elevated bilirubin & alkaline phosphatase (ALP) in biliary obstruction
Pancreatic tests • Elevated lipase and amylase in pancreatic inflammation
Thyroid tests, • Obtained to exclude endocrinologic causes of GI symptoms
cortisol (especially chronic diarrhea and constipation)
• Women of reproductive age with abdominal (especially lower
Pregnancy test
abdominal) pain, unexplained nausea and abdominal enlargement
• Assess for celiac disease, autoimmune, inflammatory bowel disease,
Serologic testing
and rheumatologic conditions
Tumor markers
• Assess for intraabdominal malignancies
(CA19-9,AFP, CEA)

II. RADIOGRAPHY AND NUCLEAR MEDICINE


DIAGNOSTIC I REMARKS
• Provides mucosa! definition and assessment of gut transit and pelvic
floor dysfunction
Oral and rectal • Barium swallow: initial procedure for evaluation of dysphagia (subtle
contrast studies rings, strictures and achalasia)
• Small-bowel contrast radiography: for evaluation of small intestinal
tumors and Crohn's ileitis

• Evaluates regions not accessible by endoscopy or contrast studies


Ultrasound & CT
such as liver, pancreas, gallbladder, kidneys and retroperitoneum
scan
• Used for diagnosis of mass lesions, fluid collections, organ enlargement

Magnetic • Assesses pancreaticobiliary ducts to exclude neoplasm, stones and


resonance sclerosing cholangitis
imaging (MRI) • Assesses the liver to characterize benign and malignant tumors
• Evaluates mesenteric ischemia/occlusion
Angiography • Localizes sites of bleeding when the rate of arterial bleeding is at least
o.5mL/min

Positron emission • Facilitates differentiation of malignant from benign disease


tomography • Used for restaging after neoadjuvant therapy for certain tumors
(PET)scan (esophageal CA)

• Evaluates structural abnormalities and quantifies lurninal transit


• Radionuclide bleeding scans: localizes bleeding at a rate as low as
o.04mL/min
Scintigraphy • Radiolabeled leukocyte scans: searches for intraabdominal abscesses
not seen on CT scan
• Biliary scintigraphy: complements ultrasound in assessment of
cholecystitis (demonstrates patency of cystic duct)
Sources:JamesonJL,et.al.Harnson'sPnnc1ples of InternalMed1c1ne,
20thed11ion.
2018
FeldmanM,et al. SleisengerandFordtran'sGastrointestinal
andLiverDisease11thEdition.2021
374
III. ENDOSCOPIC PROCEDURES
DIAGNOSTIC I REMARKS I COMMON INDICATIONS
• Endoscope inserted through mouth
Esophago- • Dyspepsia despite
into esophagus, stomach, duodenal
gastroduodenoscopy treatment or with signs
bulb, & second part of duodenum
(EGO or upper of alarm (see discussion
• Best method of examining the upper
endoscopy) below)
gastrointestinal mucosa
• Scope inserted through anal canal
• Cancer screening
into the rectum and colon
Colonoscopy • LGIB or anemia
• Gold standard for diagnosis of
• Diarrhea, obstruction
colonic mucosa! disease
• Visualizes only the rectum and a • Mainly for evaluation of
Flexible
portion of the left colon, typically up diarrhea and rectal outlet
sigmoidoscopy
to 60 cm from the anal verge bleeding
• Obscure GI bleeding
• Capsule endoscopy
• Suspected small intestinal
Small bowel • Push enteroscopy
pathology (e.g., tumors,
endoscopy • Single- or double-balloon
Meckel'sdiverticulum)
enteroscopy or spiral enteroscopy
• Suspected Crohn's disease
Endoscopic • Scope passed through mouth to the
• Jaundice, cholangitis,
retrograde duodenum; the ampulla of Yater is
gallstone pancreatitis
cholangio- identified & cannulated; and contrast
• Pancreatic/biliary tumor
pancreatography material is injected into bile duct and
(ERCP) or strictures
pancreatic duct under fluoroscopy
• High-frequency ultrasound
transducers incorporated into the tip
Endoscopic
of a flexible endoscope • Staging of malignancy
ultrasound (EUS)
• Obtains images of gut wall & adjacent
organs, vessels, & lymph nodes

IV. FUNCTIONAL TESTING


DIAGNOSTIC I REMARKS .
Esophageal • Useful for suspected achalasia and other esophageal motility
manometry disorders (GERO, distal esophageal spasm)
Small-intestinal
• Useful for pseudoobstruction and other-intestinal motility disorders
and colonic
(enteric neuropathies, myopathies, irritable bowel syndrome)
manometry
Wireless motility • Measures transit and contractile activity in the stomach, small
capsule intestine and colon in a single test
Anorectal
manometry with • Useful for unexplained incontinence or constipation from outlet
balloon expulsion dysfunction
testing
Anorectal
• Assesses anal function in fecal incontinence and diagnosis of
manometry&
ultrashort-segment Hirschsprung's disease
electromyography

Biliary
• Tests for sphincter of Oddi dysfunction with unexplained biliary pain
manometry
Sources:JamesonJL,et.al.Harrison'sPrinciplesof InternalMedicine,20thedition.2018
FeldmanM,et al. SleisengerandFordtran'sGastrointestinal
andLiverDisease11thEdition.2021
375
SECTION TWO
D1~RRMEAAND CONSTIPAl'IO~
DIARRHEA
Passage of abnormally liquid or unformed stools at an increased frequency (three or
more bowel movements daily are considered abnormal)
Stool weight >200 g/day can be considered as diarrhea in an adult with a typical
Western diet
Must be differentiated from two common conditions
0 Pseudodiarrhea: frequent passage of small volumes of stool (associated with rectal
urgency, tenesmus, or a feeling of incomplete evacuation) seen in irritable bowel
syndrome or proctitis
° Fecal incontinence: involuntary discharge of rectal contents seen in neuromuscular
disorders or structural anorectal problems

Acute diarrhea • Diarrhea <2 weeks duration


Persistent diarrhea • Diarrhea 2-4 weeks duration
Chronic diarrhea • Diarrhea >4 weeks duration
Basedon Volumeof lndiviaua1Stools
• Typically involves the right colon or small intestine
Large-volume diarrhea (rectosigmoid reservoir is intact), less frequent large-
volume bowel movement, often painless
• Typically involves the left colon (loss of storage reservoir
Small-volume diarrhea from rectosigmoid), frequent small-volume bowel
movement, often painful
Based°cliStoolElectrrolY,tes
• Sodium (net inhibition of absorption), potassium (net
secretion), and other accompanying anions account for
Secretory diarrhea
stool osmolality
• Continues during fasting
• Poorly absorbable solutes (cations, anions, absorbed sugars,
Osmotic diarrhea sugar alcohols) account for osmotic activity of stool water
• Resolves with fasting

Watery diarrhea • Implies either secretory or osmotic diarrhea


• Implies defective absorption of fat and possibly other
Fatty diarrhea
nutrients in the small intestines

• Implies the presence of inflammatory or neoplastic disease


Inflammatory diarrhea
involving the gastrointestinal tract
Invasive diarrhea • Diarrhea with visible blood, commonly associated with
(dysentery) fever and abdominal pain
Source:JamesonJL,et.al.Harrison'sPrinciplesof InternalMedicine,20thedition.2018
FeldmanM, et al. Sleisenger
andFordtran's
Gastrointestinal
andLiverDisease11thEdition.2021
376
I. ACUTE DIARRHEA
A. Etiopathogenesis & Manifestations
0 Majority are caused by infectious agents acquired by ingestion of food or water
contaminated with pathogens from human or animal feces (i.e., fecal-oral transmission)
0 Side effects from medications are the most common noninfectious causes of acute
diarrhea (e.g., antibiotics, NSA!Ds, chemotherapeutic agents)
0 Manifests as watery and/or bloody diarrhea, abdominal pain, vomiting, and/or fever
B. Diagnosis
0 Most are mild & self-limited, and do not warrant diagnostic or pharmacologic interventions
0 Indications for evaluation:
• Profuse diarrhea with dehydration
• Grossly bloody stools
• Fever >38.5 °C
• Duration >48 hours without improvement
• Recent antibiotic use
• New community outbreaks
• Associated severe abdominal pain in patients >50 years
• Elderly (2:70years) or immunocompromised patients
• Weight loss (acute weight loss is a good marker of severity)
DIAGNOSTIC I REMARKS

• Cornerstone of diagnosis in suspected severe acute infectious diarrhea


Micro biologic ° Cultures for bacterial and viral pathogens
analysis of Direct inspection for ova and parasites
0

stool Immunoassays for bacterial toxins (Clostridiumdifficile),viral antigens


0

(rotavirus) and protozoa! antigens (Giardia,Entamoebahistolytica)


• Includes sigmoidoscopy, coionoscopy, abdominal CT scan
Structural
• For patients with uncharacterized persistent diarrhea
examination
• Done to rule out inflammatory bowel disease and intestinal neoplasia
• To detect the presence of infection caused by toxin-producing
Clostridium Clostridium difficilebacteria
difficiletoxin & • If both positive: diarrhea is secondary to toxin-producing C. difficile
antigen test • Positive antigen but negative toxin: C. difficileis present in GIT, but not
producing a detectable level of toxin

C. Management
"
THERAPY
I REMARKS

• Fluid repletion and oral sugar-electrolyte solutions


Supportive
• IV hydration for profoundly dehydrated patients
• For moderately severe nonfebrile and non-bloody diarrhea
Antisecretory • Avoided in febrile dysentery
agents • Dose: loperamide 4 mg initially, then 2 mg after each loose stool (not to
exceed 8 mg/day)

Bismuth • Help reduce fluidity of bowel movement & reduce vomiting


subsalicylate • Avoided in immunocompromised patients and those with renal failure
• Antibiotics indicated (whether or not a causative organism is identified) in:
• Elderly
0 Immunocompromised patients
0 Patients with mechanical heart valves or recent vascular grafts
Judicious use
• Commonly used antibiotics:
of antibiotics
° Ciprofloxacin 500 mg PO q12 for 5-7 days: empiric treatment for
moderately to severely ill patients with febrile dysentery
0 Metronidazole 500 mg PO q12 for 5-7 days: empiric treatment for
suspected giardiasis or other protozoa! infections
Source:Jameson JL,et.al.Harrison's
Principles
of InternalMedIcIne,
20thed1t1on.
2018
Feldman
M,et al.Sleisenger
andFordtran's Gastrointestinal
andLiverDisease11thEdition.2021
377
II. CHRONIC DIARRHEA
ETIOLOGY I PATHOGENESIS I EXAMPLES
• Medications: most common secretory causes of
chronic diarrhea (e.g., stimulant laxatives)
• Infections: Mycobacteriumtuberculosis,Aeromonas,
• Due to derangements Plesiomonas,HIV, Giardia,Cryptosporidium,CMV,
in fluid & electrolyte Tropherymawhippelii
Secretory
transport across the • Structural: bowel resection (short bowel
encerocolonic mucosa syndrome), post-vagotomy, mucosa! disease, fistula
• Chronic ethanol consumption
• Hormone-producing tumors: carcinoid, VIPoma,
medullary cancer of thyroid, gastrinoma, mastocytosis

• Poorly absorbable, • Osmotic laxatives


osmotically active solutes • Carbohydrate malabsorption
Osmotic draw enough fluid into • Nonceliac gluten intolerance
lumen, exceeding colonic • Intolerance of fermentable oligosaccharides,
reabsorptive capacity disaccharides, monosaccharides and polyols

• Accompanied by pain, • Inflammatory bowel disease


fever, bleeding or other • Primary or secondary forms of immunodeficiency
manifestations of • Eosinophilic gastroenteritis
inflammation • Radiation enterocolitis
Inflammatory
• Presence ofleukocytes • Chronic graft-versus-host disease
or leukocyte-derived • Diverticulitis
proteins (e.g., calproteccin • lschemic colitis
on stool analysis) • Infection: C.difficile,
E.histolytica,
Yersinia,
M. tuberculosis

• Disruption of fat • Intraluminal maldigestion from pancreatic


solubilization, digestion, exocrine insufficiency (e.g., clironic pancreatitis)
or absorption in the • Fecal fat concentration of >9-5g/100 g of stool
small intestine, which & suspected maldigestion strongly suggest a
Steatorrheal may lead to greasy pancreatic or biliary cause of steatorrhea
foul-smelling diarrhea • Mucosa! malabsorption from enteropathies: most
associated with weight commonly from celiac disease
loss and nutritional • Small intestinal bacterial overgrowth (SIBO)
deficiencies • Postmucosal lymphatic obstruction

• Hypermotility with resultant diarrhea:


• Diarrhea due to rapid
Dysmotile hyperthyroidism, carcinoid syndrome, drugs
colonic transit
• Irritable bowel syndrome (!BS)

Factitious • Unexplained diarrheas • Eating disorders, self-administration of laxatives


• Cholecystectomy, small intestinal or colonic
Iatrogenic • Usually post-surgery
resection, bariatric surgery, vagotomy, fundoplication

III. SUPPORTIVE MANAGEMENT OF DIARRHEA


DRUG
I MECHANISM/REMARKS
I DOSE

• Slows intestinal motility through opioid receptors • 4 mg initiallythen give


• Reduces fecal volume & increases viscosity 2 mg after each loose
Loperamide stool (not to exceed
• Contraindicated in bloody diarrhea, high fever,
infectious diarrhea, pseudomembranous colitis 8-16 mg/day)

• Diphenoxylate: acts on smooth muscle of GIT, • 5 mg/0.05 mg POq6h


Diphenoxylate/ inhibiting GI motility & excessive GI propulsion (not to exceed 20 mg
atropine • Atropine added to discourage deliberate diphenoxylate daily)
overdose of diphenoxylate

Bismuth • Bismuth: antimicrobial anti-inflammatory action • 262 mg/tab 2 tabs PO


subsalicylate • Salicylate: antisecretory effect q1/2-1h PRN

Alosetron • 5-HT3 receptor antagonist; for !BS • 0.5 mg POq12h (4 wks)

Cholestyramine • Forms a complex with bile acids that is not • 4 g with meals
resin absorbed in intestines; for !BS

378
CONSTIPATION
I. ETIOPATHOGENESIS
Unsatisfactory defecation characterized by infrequent stools, difficult stool passage (straining,
incomplete evacuation, hard/lumpy stools, need for manual maneuvers to pass stool), or both
Traditional medical definition is ,;3 bowel movements per week
Results from inadequate fiber/fluid intake or disordered colonic transit or anorectal function
Risk factors include advanced age, female gender, decreased food intake, reduced mobility,
obstetric/surgical history (e.g., rectal prolapse), chronic illness & drugs, psychological factors
RECENT ONSET CONSTIPATION I CHRONIC (:?3 MONTHS) CONSTIPATION

• Colonic obstruction • Irritable bowel syndrome


• Anal sphincter spasms • Medications'
• Medications' • Colonic pseudo-obstruction
• Endocrinopathies
• Psychiatric disorders
• Neurologic or generalized muscle disease
'Medications:
opioids,aspirin,someNSAIDs,calciumsupplements,
antispasmodics,
anticonvulsants

II. DIAGNOSIS
DIAGNOSTIC I INDICATIONS/REMARKS

• Identifies metabolic and other systemic disorders


CBC & chemistry
• Hemoglobin, ESR, TSH, serum calcium, serum glucose

• Allows direct visualization of the colonic mucosa


Flexible • Done if with warning signs (weight loss, rectal bleeding or anemia,
sigmoidoscopy or change in stool caliber, family history of colon cancer)
colonoscopy • Colonoscopy alone: most cost-effective because biopsy, polypectomy or
stricture dilation can also be performed
Imaging (CT scan • Excludes an obstructing lesion when colonoscopy is contraindicated
or MRI, barium • Allows assessment of extra-intestinal structures as well as staging of
enema) malignant lesions
• Measurement of colonic transit time (radiopaque markers, wireless
Other diagnostics
motility capsule, radioscintigraphy)
for intractable
• Ariorectal and pelvic floor tests
constipation
• Formal psychological evaluation '

III. MANAGEMENT
MANAGEMENT I REMARKS

For majority of • Ample hydration (1.5 to 2 L of water per day)


cases with no • Exercise
underlying cause • Supplementation of dietary fiber (15-25 g/day)
• Bulk or commercial fiber products (e.g., psyllium, methylcellulose)
• Osmotic laxatives (magnesium salts, polyethylene glycol, lactulose): for
Symptomatic
patients who do not improve with fiber supplementation
treatment
• Stimulant agents (bisacodyl, senna derivatives)
• Other agents (lubiprostone, linaclotide)

Directed • Biofeedback management for anismus or pelvic floor dysfunction


treatment for • Bowel regimen (rectal stimulation, enema therapy, timed laxative
certain conditions therapy) for spinal cord injuries and neurologic disorders

Failure of
• Laparoscopic colectomy with ileorectostomy (not done if with continued
medical therapy
evidence of evacuation disorder or generalized GI dysmotility)
(after3-6-mcmth
trial)
-
"
Source:JamesonJL,et.al.Harnson'sPrinciples
of InternalMedicine,
20thed1t1on.
2018
FeldmanM,et al. Sleisenger
andFordtran's
Gastrointestinal
andLiverDisease11thEdition.2021
379
~ERAL
-
DISEASESOF THE GI TRACT

GASTROESOPHAGEAL REFLUX DISEASE (GERD)


I. ETIOPATHOGENESIS
Condition by which gastric contents move retrogradely from the stomach to
the esophagus or beyond, producing a variety of troublesome esophageal and
extraesophageal manifestations

A. Pathophysiology
0 Results from anatomic and physiologic disruptions of the esophagogastric junction:
• Transient lower esophageal sphincter (LES) relaxation (vagovagal reflex elicited
by gastric distension): accounts for 90% of reflux
• LES hypotension
• Anatomic distortion of the esophagogastric junction inclusive of hiatal hernia

B. Factors that Exacerbate Reflux Regardless of Mechanism:


° Central obesity O Pregnancy

0 Gastric hypersecretory states O Delayed gastric emptying

0 Duodenogastric reflux O Disruption of esophageal peristalsis


Prolonged nasogastric intubation
0

cc .r
COMPLICATION I REMARKS
Chronic • Bleeding, ulcer perforation with severe esophagitis, stricture
esophagi tis formation

• Most severe histologic consequence of GERO


• Increased risk for development of esophageal adenocarcinoma
Barrett's • Endoscopic appearance: tongues of reddish mucosa extending
metaplasia proximally from gastroesophageal junction
• Histology: specialized columnar metaplasia replacing the
stratified squamous epithelium of the normal esophagus

II. CLINICAL MANIFESTATIONS


SYMPTOMS AND ASSOCIATED I ALARM FEATURES IN PATIENTS
CONDITIONS WITH UPPER GI SYMPTOMS
• Heartburn (classic symptom of GERO), • Age >55 years
regurgitation, dysphagia • Family history of upper GI cancer
• Extraesophageal symptoms/conditions: • Unintended weight loss
° Chronic cough • Occult or gross GI bleeding
0 Laryngitis (hoarseness, globus • Anemia
sensation, frequent throat clearing, • Jaundice
sore throat) • Left supraclavicular lymphadenopathy
° Chest pain • Palpable abdominal mass
0 Sleep disorders • Progressive dysphagia/odynophagia
0 Dental erosions • Persistent vomiting

380
III. DIAGNOSIS
DIAGNOSTIC I INDICATIONS/REMARKS
• For presumptive diagnosis in the setting of classic GERO symptoms of
heartburn & regurgitation
Empiric • Simplest method for diagnosing GERO & assessing its relationship to
trial of acid symptoms
suppression • Symptoms usually respond to PP! trial in 1-2 weeks
• Positive response: ~50% improvement in heartburn after ~2 weeks of PP!
• Non-response does not rule out GERO
• Major role for GERD complications especially peptic strictures
• Important in excluding other etiologies
• First-line for patients ~ith:
0 Oysphagia or other alarm symptoms • No response to PP!
• High risk for Barrett's esophagus 0 Non-cardiac chest pain

Endoscopy • Ideally performed after PPis have been discontinued for at least 2 weeks
• Findings: edema, erythema, friability, red streaks, erosions, ulcers
• Endoscopic screening in GERD patients with multiple risk factors
for esophageal adenocarcinoma is recommended (e.g., age >50 years,
male, hiatal hernia, nocturnal reflux, obesity, tobacco use, history of
Barrett's esophagus or adenocarcinoma in a first-degree relative)
Esophageal • Not routinely recommended for diagnosis
biopsy • Primary indications: Barrett's epithelium & exclude eosinophilic esophagitis

Ambulatory • Establishes pathologic acid reflux using wireless pH capsules or


reflux catheter-based pH probes
monitoring • Only test that can assess and correlate symptoms with reflux episodes
• Assess LES pressure and relaxation, peristalsis, contraction to rule out
Esophageal
achalasia or scleroderma especially before anti-reflux surgery
manometry
• Not recommended for diagnosis
• Useful in demonstrating subtle anatomic narrowing of esophagus &
assessing the presence & reducibility of a hiatal hernia, Schatzki rings,
Barium
webs, strictures
esophagograrn
• Not recommended for diagnosis of GERO unless evaluating for "
complications (stricture, ring)

IV. MANAGEMENT
A. Non-Pharmacologic Management
MANAGEMENT I REMARKS
• Selective elimination of substances that can trigger reflux and/or
reduce LES pressure (e.g., fatty food, alcohol, tobacco, spearmint,
Lifestyle peppermint, tomato-based food, and possibly coffee and tea)
modification • Weight reduction
• For nocturnal GERO: head of bed elevation, left lateral decubitus
positioning, and avoidance of meals 2-3 hours before bedtime
• Only intervention that corrects physiologic factors causing GERO
Surgical
• Laparoscopic antireflux surgery (LARS): now the standard
management
operative approach to fundoplication

Novel • Transoral incisionless fundoplication: recreation of a partial


endoscopic fundoplication using polypropylene fasteners placed
therapies endoscopically
Source:Jameson JL,et.al.Harrison's
Principles ''
of InternalMedicine, "
20thedition.
2018
FeldmanM,et al. Sleisenger
andFordtran's Gastrointestinal
andLiverDisease11thEdition.2021
KatzPO,et al.AmJ Gastroenterol. 2021
381
• More effective mode of therapy initiated at once-daily dosing
• 8-week course of PP! given 30-60 minutes before meals for symptom
relief and healing of erosive esophagi tis
• If with partial response: adjustment of dose timing, twice-daily
Proton
dosing, or switching to a different PP! may be considered
pump
• Maintenance PP! therapy in the lowest effective dose may be
inhibitors
(PP!) administered for patients with GERO complications, including severe
erosive esophagitis (EE) and Barrett's esophagus
• After symptom resolution, attempts should be made to discontinue
PP Is or to switch to on-demand therapy for those who do not have
severe EE or Barrett's esophagus
• Bedtime H2RAs, given on an as-needed basis, may be an option
Histamine for patients with nocturnal symptoms & for those with objective
(H2) receptor
evidence of nocturnal acid reflux despite PP! therapy
antagonists
• May develop tachyphylaxis after chronic use
• Role is limited to as-needed use for relief of mild GERO symptoms
• Neutralizes gastric pH (decreases exposure of esophageal mucosa to acid)
Antacids
• Usually contain a combination of magnesium trisilicate, aluminum
hydroxide, or calcium carbonate
• Pro kinetic medications & reflux inhibitors for mild symptoms
0 Metoclopramide 10 mg IV/IM/PO q6h 30 minutes before meals
Other drugs 0 Domperidone IO mg PO TID
• Transient LES relaxation inhibitors (e.g., baclofen)
• Mucosa! protective agents (sucralfate in GERO during pregnancy)
FeldmanM,et al. SleisengerandFordtran'sGastrointestinal
andLiverDisease11thEdition.2021
KatzPO,et al.AmJ Gastroenterol.
2021

PEPTIC ULCER DISEASE (PUD)


Disruption of the mucosa! integrity of the stomach and/or duodenum, leading to a local
defect or excavation
Ulcer: circumscribed mucosa! defect having a diameter of 5 mm with a perceivable
depth or flat mucosa! breaks with a diameter of3 mm or more
Ulcers occur within stomach and/or duodenum and are usually chronic in nature

I. ETIOPATHOGENESIS
A. Helicobacterpylori
0 S-shaped gram-negative microaerophilic rod with multiple sheathed flagella which
can transform into coccoid form (dormant state)
0 Bacterial urease aids in infection by producing ammonia from urea, which then
alkalinizes the surrounding pH
0 Gastro-oral & fecal-oral routes are the dominant mechanisms of transmission (vomitus
has rno-fold higher bacterial load than saliva and stool)
0 Risk factors for higher colonization rates: poor socioeconomic status, low educational
attainment, unsanitary conditions
0 Plays a role in development of PUD, gastric mucosa-associated lymphoid tissue
(MALT) lymphoma, and gastric adenocarcinoma
B. NSAID-Induced Disease
• Interruption of prostaglandin synthesis can impair mucosa! defense & repair
0Established GI risk factors for NSAID ulcers:
• History of ulcer (especially complicated ulcer)
• High-dose/multiple NSA!Ds (including aspirin, COX-2 inhibitors)
• Advanced age (>70 years)
• Concomitant glucocorticoid, anticoagulant, or anti platelet use
• H. pylori infection
• Serious/multisystem disease
382
II. MANIFESTATIONS

I GASTRIC ULCER (GU) I DUODENAL ULCER (DU)


• Distal to the junction between • >95% occur in the first portion of the
Usual location
antrum & acid secretory mucosa duodenum (within 3 cm of pylorus)
Malignancy risk • Common (should be biopsied) • Extremely rare
Usual etiology • H. pylori, NSAID-induced injury
• Gastric acid output normal or • Gastric acid secretion increased
Pathophysiology
decreased • Bicarbonate secretion decreased
• Burning or gnawing discomfort
Abdominal • Pain that wakes the patient from sleep
• Burning or gnawing discomfort
pain (between midnight and 3 A.M.) is the
most discriminating symptom
Pain in relation • Occurs 90 mins to 3 hrs after a meal
• Precipitated by food
to food intake • Relieved by antacids or food
• GI bleeding (melena or coffee-ground emesis): most common complication
• Perforation: sudden severe, generalized abdominal pain (second most
common complication of PUD)
• Penetrating ulcer: form of perforation in which the ulcer bed tunnels into
Complications
an adjacent organ
ofPUD
• GU: tends to penetrate into the left hepatic lobe
• DU: tends to penetrate posteriorly into the pancreas, leading to pancreatitis
• Gastric outlet obstruction: pain worsening with meals, vomiting of
undigested food, early satiety, abdominal fullness
Source:Jameson JL,et.al.Harrison's
Principles
of InternalMedicine,
20thedition.2018
FeldmanM, et al. Sleisenger
andFordtran's Gastrointestinal
andLiverDisease11thEdition.2021

III. DIAGNOSTICS
A. Diagnostics for Ulcer Detection
DIAGNOSTIC I REMARKS

• Previously used as a first test for documenting an ulcer


Barium studies
• GU: discrete crater with radiating mucosa! folds originating from
of proximal
the ulcer margin, commonly located in the antrum '
GIT
• DU: appears as a well-demarcated crater, most often seen in the bulb

• Allows direct visualization of mucosa


Upper • Most sensitive and specific approach for examining the upper GIT
endoscopy
(EGO) • Facilitates documentation of a mucosa) defect and tissue biopsy to
rule out malignancy (GU) or H. pylori

B. Indications for H. pylori Infection Testing


ESTABLISHED INDICATIONS FOR I CONTROVERSIAL INDICATIONS FOR
TESTING FOR H. PYLORI INFECTION TESTING FOR H. PYLORI INFECTION

• Active PUD • Long-term aspirin and NSAID intake


• History of PUD without previous • Unexplained iron deficiency anemia
treatment of H. pylori despite evaluation
• Low-grade gastric MALT lymphoma • Idiopathic thrombocytopenic purpura
• History of endoscopic resection of early (ITP)
gastric cancer • Asymptomatic individuals with a family
• Uninvestigated dyspepsia if the history of gastric cancer
population prevalence is >20% • Functional dyspepsia if the population
prevalence is <20%
Source:CheyWD, el al. ACGclinicalguideline.Am J Gastroenterol
2017
383
• Simple to perform; rapid results
• Performed during gastroscopy: biopsy of mucosa is taken & the
urease produced by H. pylori raises the pH of the medium and
Rapid urease
changes the color of the specimen from yellow to red (positive)
test
• Prone to false negatives with blood in the stomach, recent
antibiotics, bismuth or PP! use (PP!s should be discontinued at least
1week and up to 4 weeks prior to testing)
• Provides histologic information
Histology
• Requires pathology processing
• Allows determination of antibiotic susceptibility
• Time-consuming and expensive
Culture
• Difficult culture protocol (fastidious, slow-growing organisms)
• Not widely available
• PCR testing of stool & biopsy specimen provides rapid & accurate
results
PCRassay
• Detects antibiotic resistance genes
• Not widely available; useful for research purposes
Non-Invasive ifesis

• Simple and rapid


• Patient swallows a capsule containing urea: the detection of isotope-
labeled CO2 in exhaled breath indicates the presence of urease from
Urea breath
H. pylori
test
• Test of choice for documenting eradication (done 4-8 weeks after
completion of treatment); useful both before and after treatment
• Associated with low-dose radiation exposure
• Inexpensive and convenient
Serology • Not useful for early follow-up ("serologic scar," remains positive for
months to years after successful treatment)
• Inexpensive and convenient
Stool Antigen
• Useful both before and after treatment
Source:JamesonJL, et.al.Harrison'sPrinciplesof InternalMedicine,20thedition.2018
FeldmanM, et al. SleisengerandFordtran'sGastrointestinal
andLiverDisease11thEdition.2021

D. Documentation of Eradication of H. pylori


0 Testing successful eradication is recommended at least 4 weeks after completion of
antibiotics with 1-2weeks of withholding PP! therapy, using:
• Urea breath test
• Fecal antigen test
• Biopsy-based test
0 Serologic testing may not be useful for documenting eradication because antibody
titers fall slowly

384
IV. MANAGEMENT OF H. PYLORI-POSITIVE ULCERS
All patients who test positive for active H. pylori infection should be offered treatment
• Leads to relief of symptoms, ulcer healing, and prevents ulcer recurrence & complications

• Aluminum hydroxide
• Constipation
5-30 mL between meals & HS
• Neutralize gastric
Antacids • Magnesium hydroxide • Diarrhea
acidity
400 mg PO q4 (maximum of • Avoid in those
4 doses per day) withCKD
• Competitive
• Headache,
Hi Receptor • Cimetidine 400 mg BID inhibition at the
fatigue,
Antagonists • Ranitidine 300 mg HS parietal cell H2-
myalgias
(H2RAs) • Famotidine 40 mg HS receptor, suppress
• Relatively safe
acid secretion
• Headache,
• Covalently bind abdominal
and irreversibly pain, diarrhea,
• Omeprazole 20 mg/day
inhibit H+,K•- flatulence,
Proton • Esomeprazole 40 mg/day
ATPase dermatitis,
Pump • Rabeprazole 20 mg/day
• Most potent acid pruritus,
Inhibitors • Pantoprazole 40 mg/day
inhibitory agent dry mouth,
(PPis) • Lansoprazole 30 mg/day
• Maximum efficacy blurred vision,
• Dexlansoprazole 30 mg/day
if taken 30-60 mins angioedema,
before a meal elevated liver
enzymes

B. Cytoprotective Agents
EXAMPLES I MECHANISM I SIDE EFFECTS
• Becomes a viscous paste within the
• Constipation, aluminum '
Sucralfate stomach and duodenum, binding
toxicity (in renal
1gQID primarily to sites of active ulceration
insufficiency)
• Serves as a physicochemical barrier
• Black stools
Bismuth • Mechanism is unclear
• Constipation
Subsalicylate • Postulated to have antibacterial,
• Darkening of the tongue
2 tabsQID anti-secretory, and anti-
• Neurotoxicity (if with long-
(BSS) inflammatory properties
term use)
• Prostaglandin analogue • Diarrhea
Misoprostol
• Enhancement of mucosa! defense • Contraindicated in
2oomcgQID
and repair pregnancy
• Stimulates prostaglandin
production in gastric mucosa
• Improves quality & speed of ulcer • Dizziness
Rebamipide healing: • Drowsiness
10omgTID 0 Increases epidermal growth factor • Constipation
expression • Nausea/vomiting
0 Scavenges oxygen free radicals

0 Increases gastric blood flow

38S
C. First-Line Therapies for H. pylori Eradication
0 Antibiotic resistant strains: most common cause for treatment failure in compliant patients
° Choice of regimen is based on resistance patterns, local recommendations, & availability

1. Triple Therapy for 14 days


• For low clarithromycin resistance areas (<15% resistance rates)

Th h .
• Also for patients with no history of macrolide exposure for any reason
I d
Any PPI
(standard or double dose)
I Pl
us. · ·
I Plus either of the
following . ..

Omeprazole 20 mg BID
Lansoprazole 30 mg BID
Dexlansoprazole 30 mg BID Amoxicillin IOOO mg BID
Esomeprazole 40 mg BID Clarithromycin 500 mg BID
Metronidazole 500 mg TIO
Pantoprazole 40 mg BID
Rabeprazole 20 mg BID

2. Bismuth-Based Therapy for I0-14 days


• Used for areas with high clarithromycin & metronidazole resistance or those with
penicillin allergy
• Includes:
• PP! (standard dose) BID, plus
• Bismuth subsalicylate 300 mg (or Bismuth subcitrate 120-300 mg) QID, plus
• Metronidazole 250 mg QID (or 500 mg TID to QID), plus
• Tetracycline 500 mg QID

• PP! plus Amoxicillin IOOO mg BID for 5-7 days, then


Sequential
• PP! plus Clarithromycin 500 mg BID and Nitroimidazole• 500 mg
therapy
BID for the next 5-7 days
• PP! plus
Concomitant ° Clarithromycin 500 mg BID plus
therapy 0 Amoxicillin moo mg BID plus
0 Nitroimidazole• 500 mg BID

• PP! plus Amoxicillin moo mg BID for 7 days, then


Hybrid therapy • PP! plus Amoxicillin plus 2 other antibiotics
(Clarithromycin 500 mg & Nitroimidazole• 500 mg) BID for 7 days
*Nitroimidazoles:
Metronidazole
or Tinidazole

Triple/
• PP!, plus
conventional
0 Amoxicillin 1000 mg BID, plus
therapy
0 Levofloxacin 500 mg OD
(for 10-14 days)
• PPI plus Amoxicillin moo mg BID for 5-7 days, then
Sequential
• PP! plus Amoxicillin moo mg BID plus Nitroimidazole 500 mg BID
therapy
plus Levofloxacin 500 mg OD for 5-7 days
• Levofloxacin 250 mg OD, plus
L-0-A-D • PP! (double dose) OD, plus
(for 7-10 days) • Nitazoxanide 500 mg BID, plus
• Doxycycline 100 mg OD
Source:
CheyWD,et al.ACG.AmJ Gastroenterol
2017& ShahS,etal.AGAPractice
Update.
Gastroenterology
2021
386
V. MANAGEMENT OF NSAID-INDUCED ULCERS
Intervention for NSAID-related mucosa! injury includes:
0Treatment of active ulcer
0Prevention offuture injury
DRUGS I REMARKS
Proton pump • Superior to standard-dose H2RAs in healing NSAID-induced peptic ulcers
inhibitors • Only PP!s can heal ulcers, independent of whether NSA!Ds are discontinued

• More effective in healing DUs than GUs


Hz-receptor
• Not preferred in those who require continuous NSAID therapy (limited data
antagonists
on efficacy in healing NSAID-induced ulcers if patients continue NSAID use)
Misoprostol • Not as effective as PP!s in NSAID-induced ulcer healing
(zoomcgQID) • Only gastroprotective agent which can reduce risk of complications

DIVERTICULAR DISEASE
I. ETIOPATHOGENESIS
Diverticular disease: general term for diverticula (i.e.,sacs) that form from the wall of the colon
• Diverticulosis is the "presence of" and diverticulitis is the "inflammation/infection of"
one or more diverticula
A. Pathophysiology
0 Herniation of mucosa through the colon at sites where arterioles penetrate the muscular wall
0 Diverticula commonly affect the left and sigmoid colon (rectum is always spared); but
in Asians, 70% are seen in the right colon and cecum
0Attributed to a low-fiber diet, constipation, high-fat content in stool
0 Diverticulitis would usually occur acutely when the diverticula are obstructed
B. Two Types ofDiverticula
0 True diverticulum: saclike herniation of the entire bowel wall
° False (pseudo) diverticulum: involves only a protrusion of the mucosa & submucosa
through the muscularis propria of the colon (type that most commonly affects the colon)

II. MANIFESTATIONS & DIAGNOSIS


A. Diverticular Bleeding
0 Hemorrhage from a colonic diverticulum is the most common cause ofhematochezia in
patients >60 years old (but only 20% of patients with diverticulosis will have GI bleeding)
'
0 Most bleeding is self-limited and stops spontaneously with bowel rest
0 Diagnosis and management of LGIB is discussed below

B. Diverticulitis

Manifestations

• CT scan shows: sigmoid diverticula, thickened colonic wall >4 mm,


and/or inflammation within the pericolic fat
Diagnosis
• Colonoscopy (to rule out cancer) may be done -6 weeks after an attack
of diverticular disease

III MANAGEMENT
• Asymptomatic diverticular disease: managed by lifestyle changes (e.g., fiber-
Medical rich diet, supplementary fiber, smoking cessation)
management • Diverticulitis: bowel rest, antibiotics for 7-10 days (3rd generation
cephalosporin or ciprofloxacin plus metronidazole)

Surgical • Indicated for all low-surgical risk patients with complicated diverticular
management disease to control sepsis & correct complications (e.g., fistula, obstruction)

387
SECTION FOUR
GASTROINTESTINAL GI BLEEDING
UPPER GASTROINTESTINAL BLEEDING (UGIB)
I. COMMON CAUSES OF UPPER GASTROINTESTINAL BLEEDING (UGIB)
Usually presents with hematemesis or melena
• Massive UGIB can also present with hematochezia
CONDITION I REMARKS
Bleeding PUD • Most common cause ofUGIB
(BPUD) • Usually secondary to NSAID use or H. pylori infection
Bleeding • Second most common cause ofUGIB
esophageal • Usually arises due to portal hypertension from liver cirrhosis
(BEV) or gastric • Poorer outcomes compared to patients with other sources ofUGIB, especially
varices for patients with higher MELD scores or Child-Pugh Class-C cirrhosis
Hemorrhagic
• Endoscopically visualized subepithelial hemorrhages and erosions
&erosive
• Usually with NSAIDs, alcohol intake & stress (serious trauma, major
gastropathy
surgery, burns, major intracranial disease, or severe medical illness)
("gastritis")
• A linear mucosa) rent near or across the gastroesophageal junction that is
Mallory-Weiss often associated with retching, vomiting, or incessant coughing
tear • Results from an increased intra-abdominal pressure in combination with the
shearing effect of the negative intrathoracic pressure above the diaphragm
• Large-caliber submucosal arteriole that runs immediately beneath the GI
Dieulafoy
mucosa and bleeds via a pinpoint mucosa! erosion
lesion
• Seen most commonly on the lesser curvature of the proximal stomach
Upper GI • Large, ulcerated masses in the esophagus, stomach, or proximal duodenum
malignancy • Presents with weight loss, pain, vomiting, dysphagia, bleeding, early satiety

II. MANAGEMENT OF UGIB


A. Initial Resuscitation
ASPECT
I REMARKS

• Consider intubation to prevent aspiration in the following:


Airway • Decreased sensorium (shock, hepatic encephalopathy)
protection 0 Massive hematemesis
• Active variceal bleeding
Restorationof • IV fluids to target SBP >I00 mmHg & heart rate <JOObpm
intravascular • Transfusion with packed RBCs or whole blood as necessary (target
volume hemoglobin >7g/dL)
• Discontinuation of offending anti thrombotic, followed by infusion ofFFP
Correction of
• Parenteral vitamin K for prolonged PT (INR) from warfarin, liver disease
coagulopathy
(target INR value <2.5)
(if present)
• Platelet infusion if with thrombocytopenia (target platelet count >50,000/mm')
• In non-variceal upper GI bleeding: consider high dose IV PP! therapy to
Initial
downstage endoscopic stigmata & reduce need for endoscopic therapy
medical
• In suspected BEV from portal hypertension: give empiric IV vasoactive drugs
therapy
(e.g., somatostatin, octreotide) to reduce rebleeding risk
• Patients with active hemorrhage should undergo emergency endoscopy
Referral for • To allow adequate endoscopic visualization: prokinetic agents (IV
endoscopy erythromycin or metoclopramide 30-90 minutes before upper endoscopy) help
visualization & decrease need for second-look endoscopy

388
8. Management of Bleeding Peptic Ulcer Disease
1. Forrest Classification
• Classification of gastroduodenal ulcers used for selecting patients for endoscopic treatment
• Instrumental when stratifying patients with UGIB into high and low risk categories
of mortality and in predicting the risk of rebleeding

TYPE I DESCRIPTION
(Risk of Rebleed)
I MANAGEMENT
I DISPOSITION

Acute He'!'orrh_age
• Arterial spurting • Intravenous (IV) PP!, plus • Clear liquids for
IA (90%) • Combination endoscopic treatment ~2 days

• Arterial oozing (epinephrine injection+ second • Hospitalize for


1B (10%) hemostasis modality) 3 days

Signs of Rec_entHemorrhage .3 ~· ,._ _.,'il

• Non-bleeding • IV PP!, plus


IIA visible vessel • Single endoscopic treatment,+/- • Clear liquids for
(50%) • Epinephrine injection ~2 days

• IV PP!, or • Hospitalize for


• Adherent clot 3 days
IIB (33%)
• Endoscopic clot removal, then
treatment of underlying stigmata
• Hospitalize ~1-2
• Pigmented flat • Oral PP!
IIC spot (7%) • No need for endoscopic treatment
days or expedited
discharge
Lesions without Active Bleeding "'
• Clean ulcer base • Oral PP! • Discharge after
III (3%) • No need for endoscopic treatment endoscopy
'IV PPl (80mg IV bolus + infusionof 8 mg/hr or minimumtwice-dailyIVbolus dosing for72 hours)
"Endoscopic trealment: epinephrine injection,thermal (e.g., multipolarelectrocoagulation, MPEC),
mechanical (hemostatic clipping)or sclerosant injection(absolute alcohol, polidocanol,or ethanolamine)
Source: Forrest, JA. Lancet; 1974& Gralnek IM,et al. ESGE Gu1del1ne.Endoscopy 2021 1-=
2. Other Aspects in Management · "
• H. pylori eradication if with evidence of infection "
• Avoidance of NSA!Ds if feasible
• Surgery for intractable or recurrent bleeding
• PP! for gastroduodenal prophylaxis is recommended in patients on anticoagulation

A spmn, Cl OPI'd Ol(reri·m Pat1entsw1t


3. Mana11:emento fAn.up Iate et e.11:., · . h a H'!Story 0 fBPUD*

I Patient on ontiplotelet therapy with history of BPUD I


I
I Whot is the indication of ontiplotelet therapy; I
I
l l

I P,;ma,y pme"1;0"al co,d;ovo,cula, d;,ea,o


(i.e., prevent first episode of Ml/stroke)
ISe,oodo,y p,ovo"tio" ofco,d;ovomla,di,oo,eI
(i.e., prevent another episode of Ml/stroke)

I
For /hose on 1npirin For thout on duo/
monotheropy onliplclelot therapy
I I
Aspirin should not be interrupted Aspirin should not be interrupted
Temporarily interrupt ontiplotelet therapy
(may resume eventually) Second ontiplotelet should be
I If interrupted for any reason, resume
aspirin as soon as possible
(within 3-5 days)
interrupted, resume as soon as possible
(within 5 days)

'PPI should be given as co-lherapy in patients who require DAPT


Sources: Gralnek IM,et al. ESGE Guideline- Update 2021.Endoscopy 2021
389
• Somatostatin 250 mcg IV bolus (repeated in
the 1st hour if ongoing bleeding) then 3 mg • Should be started as soon as
infusion over 12hours (250-500 mcg/hr) possible (before endoscopy)
Vasoactive • Octreotide 50 mcg IV bolus (repeated in 1sthour to reduce portal venous
agents if ongoing bleeding) then 50 mcg/hr infusion pressures acutely through a
• Vasopressin IV infusion: 0.2-0.4 U/min, up to splanchnic vasoconstrictive
o.8 U/min (with IV nitroglycerin at 40 mcg/min, effect
adjusted to maintain SBP 90 mmHg)
• Decreases recurrent bleeding
• .R-blockadeallows unopposed
vasoconstriction(froma-receptor)
Non- of splanchnic vessels
• Propranolol 20-40 mg PO BID
selective • Adjust every 2-3 days until
• Nadolol 20-40 mg PO OD
beta goals are achieved:
blockers • Carvedilol 6.25 mg PO BID 0 Resting HR of 55-60 bpm
or 25% below baseline
0 SBP should not decrease to
<90mmHg
• Ceftriaxone I g IV OD (antibiotic of choice) • Integral part of therapy
• Ciprofloxacin 400 mg IV 912 for cirrhosis & should be
Antibiotics • Levofloxacin 500 mg IV OD instituted upon admission
• Cefotaxime 2 g IV q8 • Decreases bacterial infections
• Norfloxacin 400 mg PO BID & mortality in cirrhotics

• Ligation or banding: endoscopic therapy


of choice as it controls active hemorrhage
(lower rates of success for gastric compared
to esophageal varices) • EGD should be performed
Endoscopic/ • Cyanoacrylate "glue" injection within 12 hours of admission
surgical • Sclerotherapy
and once the patient is
options • Balloon tamponade with a Sengstaken-
Blakemore tube or Minnesota tube hemodynamically stable
• Transjugular lntrahepatic Portosystemic
Shunt (TIPS)
• Liver transplantation
• Transfuse pRBCs only when hemoglobin drops <7 g/dL (improved survival
Judicious
with restrictive strategy in Child-Pugh classes A and B)
blood
• Overtransfusion may further increase portal pressures
transfusion
• Goal is to maintain hematocrit at -25%, hemoglobin between 7 and 8 g/dL
Source:Karstensen JG,et al. ESGE.CascadeGuideline. Endoscopy International
Open.2020
Garcia-Tsao
G.et al. 2016PracticeGuidance.
Hepatology.
2017
FeldmanM,et al. Sleisenger
andFordtran's Gastrointestinal
andLiverDisease11thEdition.2021

D. Management of Other Causes ofUGIB


DISEASE I TREATMENT
• Mild tears are often self-limited and heal spontaneously within 48 hours
Mallory-
• Active and recurrent bleeding: may perform epinephrine injection, MPEC,
Weiss Tear
hemoclip placement, or band ligation

• Difficultto see in endoscopy because overlying mucosa often appears to be normal


• Once seen, endoscopic hemostasis (epinephrine injection, thermal probe,
Dieulafoy
hemoclip placement) should be performed
lesion
• Transcatheter angiographic embolization (TAE) or surgery may be
considered if endoscopic treatment fails or is not technically feasible

Upper GI • Surgical resection, angiography with embolization, external beam radiation,


malignancy or endoscopic hemostatic spray
390
LOWER GASTROINTESTINAL BLEEDING (LGIB)
I. CAUSES OF LOWER GASTROINTESTINAL BLEEDING (LGIB)
Common causes include hemorrhoids, diverticula, anal fissures, neoplasms
(adenocarcinoma), ischemic colitis, arteriovenous malformations
Usually presents with hematochezia
ETIOLOGY I REMARKS
• Common presentations: bleeding & protruding rectal mass
• Severe pain may indicate a thrombosed hemorrhoid
• Grading of internal hemorrhoids
Hemorrhoids 0 I: enlargement with bleeding and does not prolapse
0 II: protrusion with spontaneous reduction
0 III: protrusion requiring manual reduction
0 IV: protrusion that cannot be reduced (at risk for strangulation)
• Linear tear of the anoderm from trauma to anal canal following
defecation
• Presents with severe pain (like "razor blades") during defecation
Anal fissure • Triad of chronic anal fissure: fissure, hypertrophied papilla, sentinel
skin tag
• More common in the posterior, midline anal canal (due to decreased
blood supply and elevated resting anal canal pressures)
• lschemic necrosis from insufficient blood supply to a segment or the
entire colon
• Presents as hematochezia, mild left-sided abdominal discomfort, or
Ischemic urgent desire to defecate
colitis • Risk factors: age, shock, cardiac disease, surgery, laxative & OCP use
• Most at risk for ischemia is the splenic flexure: the watershed area
between superior & inferior mesenteric arteries (area with the fewest
collateral vessels)
• Outpouching of mucosa & submucosa through muscular coat of the colon
• Hemorrhage from a colonic diverticulum: most common cause of
hematochezia in patients >60 years old
Diverticular
• Bleeding more often from the right colon, usually abrupt & painless
disease
• Localization of bleeding done by colonoscopy (diagnostic &
therapeutic), nuclear scintigraphy (tagged RBC scan), angiography+/-
embolization, CT scan
• Should be excluded in any patient with severe LGIB and colitis
Infectious • Etiology: Salmonella,Shigella, Campylobacterjejuni, C. difficile,
colitis enterohemorrhagic E. coli (O157:H7),cytomegalovirus
• Investigate using stool cultures, flexible sigmoidoscopy or colonoscopy
• Formation of aberrant blood vessels found throughout the GI tract that
often develop with advancing age
Angioectasia • Associated conditions: chronic kidney disease, cirrhosis, rheumatoid
disorders, heart disease, antiplatelet/anticoagulant use
• Investigate using endoscopy, radionuclide scintigraphy, & angiography
• Iron deficiency anemia before occurrence of more overt bleeding
Colonic (acute hematochezia), weight loss, bowel movement changes, crampy
polyps and abdominal pain or intermittent bloating
cancer • Investigate using FOBT, barium enema, CT scan, sigmoidoscopy or
colonoscopy
Source:JamesonJL,et.al.Harrison's Principles
of InternalMedicine,
20thedition.2018
FeldmanM,et al. Sleisenger
andFordtran's
GastrointestinalandLiverDisease11thEdition.2021
WaldA, et al.ACGclinicalguideline;AmJ Gastroenterol;2014
391
II. MANAGEMENT OF LGIB

Acute lower GI bleeding

Yes No

Age 2:40 years old, or


Age <40 years old family history of colon CA, or Upper
Minimal bleeding iron deficiency anemia, or endoscopy
voluminous bleedin

Flexible sigmoidoscopy Colonoscopy

'If siteis notidentifiedaftercolonoscopy,


proceedwithobscurebleedingwork-up.If siteis identifiedafter
colonoscopy andthereis persistentbleeding,performangiography
and/orreferfor possiblesurgery.

A. Initial Resuscitation
° Consists of restoration of intravascular volume, correction of coagulopathy (if present),
and referral for endoscopy (see management of UGIB for details)
0 To allow adequate endoscopic visualization: may use polyethylene glycol purge (6-8 L)
prior to urgent colonoscopy in patients with severe hematochezia & suspected colonic
source of bleeding

B. Therapy for Specific Lesions


DISEASE I TREATMENT
• Initial treatment is medical (e.g., fluid intake of 6-8 glasses daily, high-fiber
diet or 20-30 g of fiber supplementation, stool softeners)
• Rubber band ligation (RBL):most popular and effective office treatment for
Hemorrhoids
internal hemorrhoids
• Sclerotherapy: favored treatment for immunocompromised patients
• Hemorrhoidectomy: for grade III and IV internal hemorrhoids
• Sitz baths, high-fiber diet (20-35 g/day) especially unprocessed bran, stool
softeners, topical anesthetics
Anal fissures • Chronic fissures (>6 weeks): topical ointments/creams (0.2-0.4% nitroglycerin
ointment, 2% diltiazem cream, topical nifedipine), botulinum toxin A injection
• Surgery: anal dilatation and lateral internal sphincterotomy
Ischemic • If without gangrene or perforation, expectant management is done
colitis (parenteral fluids, bowel rest, antibiotics, manage underlying cause)
• Endoscopy allows identification of stigmata of recent bleed (visible vessel,
adherent clot) & intervention (epinephrine injection, thermal probe
Colonic
coagulation, endoclips, band ligation)
diverticulum
• Mesenteric angiography with coiling (if with active bleeding)
• Segmental resection of the colon
• Medical: hormonal therapy (e.g., estrogen), thalidomide, erythropoietin
• Avoid/minimize use ofNSAIDs, anti platelets, & anticoagulants if possible
Angioectasia • Endoscopic management: injection therapy with epinephrine, thermal
probe coagulation, argon plasma coagulation, band ligation
• Surgery (intestinal resection) for refractory lesions
Colonic • Endoscopic: epinephrine injection or hemoclipping to treat ulcerated lesions
polyp and that cannot be resected completely through polypectomy, EMR, or ESD
cancer • Surgical resection
392
SECTION FIVE
DISEASES OF THE LIVER
OVERVIEW OF LIVER DISEASES
I. MAJOR RISK FACTORS FOR LIVER DISEASE
Alcohol use Medications (e.g., herbal, birth control pills)
Personal habits Sexual activity
Travel Exposure to jaundiced or other high-risk persons
Injection drug use Recent surgery
History of blood transfusion Occupation
Familial history of liver disease Accidental exposure to blood or needlestick injury

II. CLINICAL MANIFESTATIONS


Symptoms of Hepatic Disease ,,,
- -\ '
• Fatigue, poor appetite, weakness, nausea, and malaise
Overview of • Fatigue: most common & most characteristic symptom of liver disease
symptoms • Liver-specific symptoms include jaundice, dark urine, light stools,
itching, abdominal pain, & bloating

• Hallmark ofliver disease & most reliable marker of severity


Jaundice • Yellowish discoloration of body tissues due to deposition of bilirubin,
which occurs in the presence of serum hyperbilirubinemia
Signs of Hepatic Disease
Icterus • Check sclerae, skin, mucous membrane below the tongue

• Occurs in acute and chronic liver disease


Palmar erythema
• Prominent in persons with cirrhosis

• Superficial tortuous arterioles seen on the arms, face, upper torso; fill
Spider angiomata
outwards from the center (unlike telangiectasias)

• Seen in veno-occlusive disease, infiltrative disorders, hepatic


Hepatomegaiy malignancy, alcoholic hepatitis
• Hepatic tenderness: most reliable physical finding in examining liver
Splenomegaly • Subtle significant finding in liver disease & portal hypertension '

• Best appreciated by percussing for shifting dullness (-1500 mL of


Ascites
ascitic fluid should be present before dullness is appreciated)

• Contributing factors: hypoalbuminemia, venous insufficiency, heart


Peripheral edema
failure, and medications

• First signs: change in sleep patterns, change in personality, irritability,


Hepatic mental dullness
encephalopathy • Confusion, disorientation, stupor, and eventually coma supervene
• Acute liver failure: excitability, mania
• Slightly sweet, ammonia-like odor in patients, especially if there is
Fetor hepaticus
portovenous shunting of blood

Umbilical hernia • Secondary to increased intraabdominal pressures from ascites

• Results from recannulation of the umbilical vein: collateral veins


Caput medusa
seen radiating from the umbilicus

• For males: gynecomastia, testicular atrophy, loss of male-pattern hair


Hyperestrogenemia
distribution
, , ,,
Source:Jameson JL,etaL Hamsons Principlesof InternalMed1cIne,
20thed1t1on,
2018
FeldmanM,et aLSleisenger andFordtran's
Gastrointestinal
andLiverDisease11thEdition,2021
393
• Presence of icterus: total serum bilirubin is at least 3.0 mg/dL (2.5mg/dL in
acute viral hepatitis)
Serum
• Unconjugated (indirect) bilirubinemia: hemolytic disorders, Crigler-Najjar and
bilirubin
Gilbert's syndrome
• Conjugated (direct) bilirubinemia: almost always implies liver or biliary disease

• Enzymes that reflect damage to hepatocytes


• ALT is more specific as an indicator of liver injury than AST
• Patterns of Liver Injury:
0 AST:ALT <I: chronic viral hepatitis, non-alcoholic fatty liver disease
0 AST:ALT>2: alcoholic liver disease, cirrhosis, Wilson's disease

• Acute ALT elevation >IO00U/L (>20-25x): • Acute AST elevation >IO00 U/L
Amino- 0 Acute viral hepatitis (20-25xnormal):
transferases 0 Ischemic liver (prolonged hypotension) Toxin- or drug-induced
0
(AST,ALT) 0 Toxin- or drug-induced liver injury injury in a patient with
0 Acute phase of biliary obstruction underlying alcoholic liver
caused by passage of gallstone into disease
the common bile duct 0 Acute rhabdomyolysis

0 Autoimmune hepatitis
0 Acute Budd-Chiari syndrome
0 Wilson's disease

Enzymes that Refle t,Cholestasis


• Principal clinical value of measuring serum ALP lies in the diagnosis of
cholestatic liver disease
Alkaline • ALP >4XULN: cholestatic liver disease, infiltrative diseases, rapid bone turnover
phosphatase • ALP elevation more than elevation in aminotransferases: cholestatic
(ALP) conditions like primary biliary cirrhosis or PBC (ALP:AST >3)
• ALP elevation less than elevation in aminotransferases: autoimmune
hepatitis (ALP:AST <!.5)

5' Nucleotidase • Elevated in cholestatic liver disease, hepatitis, biliary cirrhosis, hepatotoxic
(5NT) drugs, and metastasis

y-Glutl!myl
• Primary use of GGT: identify the source of an isolated elevation in ALP
transpeptidase
(GGT is not elevated in bone disease)
(GGT
Tests that Measure Biosynthetic Function of Liver
• Synthesized exclusively by the hepatocytes
• Half-life of 18-20 days (therefore not a good indicator of acute or mild
Serum
hepatic dysfunction)
albumin
• Hypoalbuminemia (<3g/dL) is more common in chronic liver disorders (e.g.,
cirrhosis), reflecting severe liver damage & decreased albumin synthesis

Serum • y-globulins are increased in chronic liver disease (increased antibody


globulins synthesis to fight off intestinal bacteria that the cirrhotic liver failed to clear)

• Single best acute measure of hepatic synthetic function


• All clotting factors are synthesized in the liver except for factor VIII
(produced by endothelial cells)
• Prothrombin time (PT): measures factors II, V, VII, X
Clotting factors
• Prolongation of PT >5 secs above control not corrected by IV vitamin K
portends poor prognosis
• Prognostic value in patients with alcoholic hepatitis and acute paracetamol-
and non-paracetamol-related liver injury

394
Approach to Jaundice

History and exominotion


Bilirubins (total, direct, indirect)
I AST, ALT,ALP, PT, Albumin I
l l
Isolated hyperbilirubinemia
- (high bilirubins + other liver - Bilirubin and other liver tests
elevated
tests normal)

Hepatocellular pattern (ALT/AST


Direct hyperbilirubinemia elevated out of proportion to ALP)
(DB >15%) Viral hepatitis: A, B, C, D, E

-
Inherited: Alcoholic hepatitis
Dubin-Johnson syndrome, lschemic hepatitis
Rotor syndrome Chronic liver disease ond
cirrhosis
Drugs: porocetamol, isoniozid
Indirect hyperbilirubinemia Wilson disease
(DB< 15%) Autoimmune hepatitis
Drugs: rifampicin,
probenecid Cholestatic pattern (ALP
Hemolytic disorders out of proportion to ALT/AST)
Ineffective erythropoiesis Obstructive biliary disease
Inherited: (e.g., choledocholithiasis)
Gilbert syndrome, Cholangitis
Crigler-Najjar syndrome Drugs: steroids, chlorpromazine,
erythromycin, TPN
Infections: leptospirosis, malaria
Infiltrative: TB, lymphoma,
amyloidosis
Molignoncy: poncreatic,
cholangiocarcinoma

• Hepatocellularpattern:features of liverinjury,inflammationand necrosis predominate "


• Cholestaticpattern:features of bileflowinhibitionpredominate
Source:Jameson JL, et.al. Harrison'sPrinciplesof InternalMedicine,20th edition.2018
IV.IMAGING
DIAGNOSTIC
I REMARKS
• Most commonly employed for imaging of the liver
Ultrasonography
• First-line ifinitial tests suggest cholestasis (check for dilated ducts/stones)
(US)
• Bile duct may be obscured by bowel gas (operator-dependent)
CT • Higher resolution than US and is not operator-dependent
MRCP,ERCP • Procedures of choice for visualization of the biliary tree
Doppler US and • Assessment of hepatic vasculature and hemodynamics
MRI • Doppler US is first test ordered if suspecting Budd-Chiari syndrome

Percutaneous • Direct imaging of the bile ducts


transhepatic • Allows simultaneous therapeutic intervention especially for lesions
cholangiography proximal to the common hepatic duct

Endoscopic • Superior to ultrasound and CT


ultrasound • Allows needle aspiration/biopsy of lesions

395
V. OTHER DIAGNOSTICS
DIAGNOSTIC
I REMARKS
• The criterion standard in the evaluation of patients with liver
disease
• Subject to errors in focal infiltrative disorders (e.g., hepatic
Liver biopsy
metastasis) & in the estimation of the extent of fibrosis
• Contraindications to percutaneous approach: significant ascites
and prolonged INR (may use transjugular approach instead)

Newer Imaging Te~~ni~ues to Identify Hepatic Fibrosis


Transient • Uses low amplitude shear wave that propagates through the liver
elastography parenchyma (the speed at which the wave moves is correlated
(fibroeiastography) with liver stiffness)
• Combines MRI with elastography
Magnetic resonance
• Better than transient elastography in differentiating mild from
elastography (MRE)
moderate to severe fibrosis
Source:Jameson JL, et.al. Harrison'sPnnc1plesof InternalMed1c1ne,
..
20thed1t1on.
2018
FeldmanM,et al. Sleisengerand Fordtran'sGastrointestinaland LiverDisease 11thEdition.2021

VI. RISK STRATIFICATION OF LIVER DISEASES


A. Child-Pugh Score
0 Initial staging system used once cirrhosis develops to assess compensated versus
decompensated disease
0 Stratifies patients into risk groups
0 Predicts likelihood of major complications of cirrhosis (e.g., variceal bleeding,
spontaneous bacterial peritonitis)
PARAMETER I 1 I 2 I 3
umol/L < 34 34-51 > 51
Serum Bilirubin < 2.0 2.0-3.0
mgldL > 3.0

g/L > 35 30-35 < 30


Serum Albumin
g/dL > 3.5 3.0-3.5 < 3.0

Prothrombin Seconds 1-3 4-6 >6


Time INR < 1.7 1.7 - 2.3 > 2.3

Moderate or
Ascites
Slight/easily
None severe/poorly
controlled
controlled
Hepatic Encephalopathy None Minimal Advanced
Interpretation:
• Class A= scores of 5-6 (compensated cirrhosis)
• Class B = scores of 7-9
• Class C = scores of 10-15

B. Model for End-Stage Liver Disease (MELD) Score


0Scoring system to predict prognosis of patients with liver disease and portal
hypertension
• Provides a more objective means to assess disease severity
0Score ranges from 6 to 40
° Calculated using three non-invasive variables:
• PT-INR
• Serum bilirubin
• Serum creatinine
396
VIRAL HEPATITIS
Hepatitis is a general term referring to inflammation of the liver
Acute infection with a hepatitis virus may result in subclinical to self-limited
symptomatic disease to fulminant hepatic failure
Acute viral hepatitis may evolve into chronic hepatitis, and these patients may develop
cirrhosis and hepatocellular carcinoma (HCC)

s
0 verview o1 ero ogy f,or v·,ra I H epat1t1s
- -
.!fepatitis A (HAV)
• Diagnosis of hepatitis A during acute illness & persists for several months
Anti-HAV
• Detected when aminotransferase activity is elevated and fecal HAV
(lgM)
shedding is still occurring
• After acute illness, anti-HAV of the IgG class remains detectable indefinitely
Anti-HAV
• Predominates during convalescence
(lgG)
• Marker of immunity to reinfection
Hepatitis B (HBV)
HBsAg
• First virologic marker detectable in serum within 1-12 weeks after
(Hepatitis
infection with HBV
B Surface
• Chronic HBV Infection: HBsAg remains detectable beyond six months
Antigen)
• After HBsAg disappears, anti-HBs becomes detectable and remains
detectable indefinitely thereafter (protective antibody)
Anti-HBs
• Also seen after immunization with hepatitis B vaccine (only serologic
marker to appear in immunization)
• IgM anti-HBc: predominates during the first six months after acute
Anti-HBc infection, including anti-HBc window period
(lgMor • IgG anti-HBc: predominant class of anti-HBc beyond six months
IgG) • An isolated reactive anti-HBc can be seen in: gap or window period of acute
hepatitis B, occult infection, resolved or remote infection, false positive result
• Appears concurrently with or shortly after HBsAg
• Qualitative marker of HBV replication and relative infectivity
HBeAg
• HBsAg-positive serum with HBeAg is more likely to be highly infectious
(Hepatitis B
• Its disappearance may be a harbingerofimprovement & resolution of infection
'e' Antigen)
• Persistence beyond first 3 months of acute infection predicts development
of chronic infection
Anti-HBe • Its appearance coincides with a period of relatively lower infectivity
• More sensitive and quantitative indicator ofHBV replication
HBVDNA
• Predictor of future progression to cirrhosis & hepatocellular CA in HBV
Hepatitis C (HCV) -
Anti-HCV • Diagnosis of hepatitis C
• Most sensitive test for HCV infection
HCVRNA
• "Gold standard" in establishing a diagnosis ofHCV
- -.a .. ,.
Hepatitis D fHDV) and Hepatitis E (HEV)

Anti-HDV • Testing for anti-HDV is useful in those with hepatitis B & severe disease
Anti-HEY
• Not routinely available
(IgM/IgG)
Source:JamesonJL,et.al.Harrison'sPrinciples
of InternalMedicine,20thedition.2018
FeldmanM, et al. Sleisenger
andFordtran's
Gastrointestinal
andLiverDisease11thEdition.2021
397
Hepatitis A • Anti-HAV lgM
Hepatitis B
• Acute infection • HBsAg, IgM anti-HBc
• Chronic infection • HBsAg, IgG anti-HBc
• Markers of replication • HBeAg, HBV DNA
Hepatitis C • Anti-HCV and HCV RNA

Hepatitis D (Delta) • HBsAg and anti-HDV


• HBV/HDV coinfection • IgM anti-HBc and anti-HDV
• HOV superinfection • IgG anti-HBc and anti-HDV
Hepatitis E • Anti-HEV

I. ACUTE VIRAL HEPATITIS


A. Etiopathogenesis

I
TRANSMISSION I ONSET &
SEVERITY
I PROGRESSION TO
CHRONIC DISEASE
I PROGNOSIS

• Acute&mild
HAV • Fecal-oral • More symptomatic • None • Excellent
in adults
• Occasional (defined
• Percutaneous
• Insidious or acute as persistence of • Variable
• Perinatal/
HBV• • Occasionally HBeAg for >3mos or (worse with
vertical
severe HBsAg for >6 mos) age, debility)
• Sexual
• Common if perinatal
• Common (CLO from
• Insidious or acute HCV is the most
HCV • Percutaneous • Moderate
• Moderate frequent indication
for transplant)
• Percutaneous • Insidious or acute • Good ifacute
HDV .. • Common
• Sexual • Occasionallysevere • Poorifchronic
• None (except in the
HEV • Fecal-oral • Acute &mild • Good
immunosuppressed)
'OnlyHBVhasa DNAgenome;othertypeshavean RNAgenome
"HOV is a defectivevirusthatrequireshelperfunctionof HBVfor replication
andexpression

Bo· . fA t H .
lgM Anti- lgM Anti-
INTERPRETATION
I HBsAg
I HAV
I HBc
I Anti-HCV

Acute hepatitis B + - + -
Acute hepatitis A
superimposed on + + - -
chronic hepatitis B

Acute hepatitis A and B + + + -


Acute hepatitis A - + - -
Acute hepatitis C - - - +
Source:Prev1sam
N, et al. WorldHealthOrganization.
HepatitisB;2002
398
CG enera am estat10ns &D"iagnost1cs
Symptoms and Signs
• Systemic & variable: anorexia, nausea/vomiting, fatigue, malaise,
arthralgias, myalgias,headache, photophobia, pharyngitis, cough, & coryza
Prodromal • May precede the onset of jaundice by 1-2weeks
symptoms • Dark urine & clay-colored stools noted 1-5days before onset of jaundice
• Low-grade fever between 38-39 °C (100-102°F) is more common in
hepatitis A and E than hepatitis B or C
• With the onset of jaundice, the constitutional prodromal symptoms
Jaundice usually diminish
• May have RUQ pain from liver enlargement and tenderness
• Complete clinical and biochemical recovery occurs:
0 1-2months after hepatitis A and E
Recovery phase 0 3-4 months after the onset of jaundice in hepatitis Band C (among
healthy adults, acute hepatitis Bis self-limited in 95-99% while
hepatitis C is self-limited in only 15-20%)
• Most feared complication of viral hepatitis (massive hepatic
necrosis), primarily seen in hepatitis B, D, and E
• Presents as encephalopathy (confusion, disorientation, somnolence)
Fulminant that may evolve to deep coma
hepatitis • Prothrombin time (PT) is prolonged, bilirubin is rapidly rising, liver
is small on imaging
• Terminal events: brainstem compression, GI bleeding, sepsis,
respiratory failure, cardiovascular collapse, renal failure
Laboratory Features
'
• Increase during the prodromal phase of acute viral hepatitis and
precede the rise in bilirubin level
AST and ALT
• Peak levels vary from :?:400-4000 JU
• Levels do not correlate well with degree of liver damage
Bilirubin • When jaundice appears, bilirubin typically rises
• Prolonged values reflect severe hepatic synthetic defect and extensive "
Prothrombin time
hepatocellular necrosis, and may indicate a worse prognosis
ALP • Normal or mildly elevated
Hypoalbuminemia • Uncommon in uncomplicated acute viral hepatitis
Source:JamesonJL,et.al.Harrison's
Principles
of InternalMedicine,
20thedition.2018

D. Management of Acute He

• Immunoglobulin (lg)
HAV • None
• Inactivated vaccine
• HB-Ig • Entecavir, Tenofovir, Adefovir, Interferon, PEG-
HBV
• Recombinant vaccine IFN, Lamivudine, Telbivudine

• PEG-IFN + Ribavirin
HCV • None • Direct-Acting Antiviral agents (DAAs): Daclatasvir,
Sofobuvir, Telaprevir, Boceprevir

HDV • HBV vaccine • PEG-IFN

HEV • Vaccine • None


399
II. CHRONIC VIRAL HEPATITIS
Syndrome of persisting hepatotropic viral infection
• Associated with chronic inflammation and progressive hepatocyte injury

A. New Nomenclature of Chronic Hepatitis B (CHB)


0 Natural history is divided into 5 phases, taking into account the presence ofHBeAg,

HBV DNA levels, ALT values, and presence or absence of liver inflammation (seebelow)
0 New nomenclature is basedon 2 main characteristicsof chronicity: infection versushepatitis

eHASES•~

HBeAg Positive Negative


Intermediate
HBsAg High Low Intermediate Negative
to high
HBVDNA >107 IU/mL 10' -10 7 IU/mL <2000 IU/mlb >2000IU/mL Undetectable

ALT Normal Elevated Normal Elevated Normal

Liver Moderate/ Moderate/


None/minimal None None
disease severe severe
Immune HBeAg (-) HBsAg-
Old Immune
reactive Inactivecarrier chronic negative
terminology tolerant
HBeAg (+) hepatitis phase
• Serialmonitoringof HBVmarkers(HBsAg,HBeAg/anti-HBe, HBVDNA)andliverdiseasemarkers(ALT,
fibrosismarkerssuchas elastography or biomarkers,
liverbiopsyin selectedcases)maybe requiredin
mostinstancesfor classification
intooneof the phasesabove
• HBVDNAlevelscanbe between2,000& 20,000IU/ml in somewithoutsignsof chronichepatitis

B. Natural Histo of Chronic He atitis B CHB


Prodrome Convalescence
Incubation Acute Disease Earty Late
Important HBsAg HBsAg (Anti-HBC) Antl-HBc Antl-HBs (AnU-HBc)
diagnostic
tests Symptoms
SGPT(ALT)

-------..--.............
..
DNA pclymerase
HBV particle
,,
,
,,-
Core windows
--- • • • • Anti·HB

Detection
level

Months
post-exposure
OnceinfectedwithHBV,thefirstvirologicmarkerdetectable in serumis HBsAgandthis precedes elevations
of ALT.AfterHBsAgdisappears, anti-HBsbecomesdetectable in serumand remainsdetectable indefinitely
thereafter.
Anti-HBc is detectedwithinthefirst1-2 weeksaftertheappearance of HBsAgandpreceding detectable
levelsof anti-HBsbyweeksto months.HBeAg, appearstogether
withor shortlyafterHBsAg,coinciding withhigh
levelsof virusreplication.
In self-limitedinfections,
HBeAgbecomes undetectableshortlyafterpeakelevationsin
aminotransferase activity.
Anti-HBe thenbecomes detectable,
coincidingwitha periodof relatively
lowerinfectivity.
400
C. Sequelae of Chronic Hepatitis
0 Liver cirrhosis
• Hepatocellular carcinoma (HBV & HCV), especially ifHBY acquired perinatally

Factors that Affect Decision to Treat and Duration of Treatment


• Clinical status (presence of cirrhosis, compensated • Detectable HCV RNA in
vs. decompensated, liver mass/HCC) serum
• HBeAg status • HCV genotype
• HBV DNA titers • Presence of significant
• ALT levels fibrosis, cirrhosis
• Family history of hepatocellular carcinoma
• Presence of extrahepatic manifestations
Treaiment

• First-line nucleos(t)ide analogs: • PEG IFN-based treatments


0Entecavir 0.5 mg PO OD have been supplanted by
• Tenofovir disoproxil fumarate (TDF) 300 mg PO OD Direct-Acting Antiviral (DAA)
combinations
0Tenofovir alafenamide (TAF) 25 mg PO OD
• Antivirals recommended for
• Pegylated interferon (PEG IFN) SC once weekly all adults with acute & chronic
• Lamivudine (first successful oral antiviral agent, but HCV infection, except those
no longer recommended as first-line) with a short life expectancy
Souroo:
AASLD-IDSA
Hepatitis
C GuidancePanel.AASLD-1D SARecommendations. 2020
Hepatology
NA.etal.AASLD2018Hepatitis
Tenault B Guidance.
Hepatology 2018;EASL2017CPG.Journal 2017
ofHepatciogy
EASL2017CPG.European Association 2017
for theStudyof the Liver.Journalof Hepatology

ALCOHOLIC LIVER DISEASE


I. ETIOPATHOGENESIS
• Acetaldehyde: compound responsible for many of the systemic effects of alcohol &
postulated to play an etiologic role in alcoholic liver disease
Abstinence of 4 to 6 weeks reverses simple uncomplicated fatty liver
Threshold for Deve/opinq
.-----'--_____.____.____ ___
Alcoholic Liver Disease: ___, '
• Men: >40-80 g/day of alcohol for 10-12 years•
• Women: >20-40 g/day of alcohol for 10-12 years•
• 15standard drinks/day
• >160 g/day for I0-20 years causes hepatitis or cirrhosis

'Thefollowingallcontain-12 g of alcohol:1 bottleof beer,4 ouncesofwine,or 1 ounceof 80%spirits

• Initial & most common histologic response to excessive alcohol ingestion


Alcoholic
fatty liver • Fat accumulation within perivenular hepatocytes
• Cessation of alcohol intake results in normalization of hepatic architecture
• Results from progressive alcohol-associated li,;er injury (transition from
Alcohol- alcoholic fatty liver to alcoholic hepatitis is blurred)
associated • Hallmark: hepatocyte injury characterized by ballooning degeneration,
hepatitis spotty necrosis, PMN infiltrate & perivenular and perisinusoidal fibrosis
• Still potentially reversible with cessation of alcohol intake
Alcohol- • Presents with clinical features identical to other causes of cirrhosis
associated
cirrhosis • Regression is uncertain even after cessation of alcohol intake

401
II. MANIFESTATIONS & DIAGNOSIS
A. Overview of Manifestations
PATHOLOGY MANIFESTATIONS DIAGNOSIS

• AST or ALT increased 2- to


• RUQ pain, nausea, fatigue
Alcoholic 7-fold (<400 IU/L)
• Hepatomegaly: most common
fatty liver • AST/ALT ratio >I
physical finding
• Bilirubin: usually >3.0 mg/dL
(may be markedly increased
• Fever, spider nevi, jaundice, and in alcoholic hepatitis despite
abdominal pain modest elevation in ALP)
• Portal hypertension, ascites, or • Classic histopathologic
variceal bleeding can occur in the features of alcoholic hepatitis:
Alcohol-
absence of cirrhosis ° Centrilobular & perivenular
associated
• Dupuytren's contracture (abnormal fatty infiltration
hepatitis
thickening of palm or base of the 0 Ballooning degeneration of
fingers, causing finger/s to be pulled hepatocytes
or curled inward towards the palm) 0 Alcoholic hyaline (Mallory-

• Parotid gland enlargement Denk bodies)


• RUQ pain, fever, nausea & vomiting,
diarrhea, anorexia, and malaise
• More specific complications: ascites,
• Hypoalbuminemia
edema, or upper gastrointestinal
• Prolonged prothrombin time
hemorrhage
(PT)
• Jaundice, encephalopathy,
• Elevated bilirubin
Alcohol- hepatomegaly, splenomegaly
• Anemia & thrombocytopenia
associated • Liver edge may be firm & nodular
(from portal HPN &
cirrhosis • Sciera! icterus, palmar erythema,
hypersplenism)
spider angiomas, parotid gland
• Nodular liver, splenomegaly,
enlargement, digital clubbing,
and venous collaterals on
muscle wasting, edema and ascites
imaging
• Males: decreased body hair,
testicular atrophy, gynecomastia
• Females: menstrual abnormalities
Source:JamesonJL, et.al.Harrison'sPrinciplesof InternalMedicine,20thedition.2018
FeldmanM, et al. SleisengerandFordtran'sGastrointestinal
andLiverDisease11thEdition.2021

B. Determining Disease Severity


0 The Maddrey discriminant function (DF) and the Model for End-stage Liver Disease
(MELD) score are the most commonly used to identify patients who are more likely to
benefit from pharmacologic therapy
• The MELD score was discussed above, while the Maddrey score is discussed below:

Maddrey Discriminant Function (DF)

OF= (4.6 x [PT in sec - Control PT in sec]) + serum bilirubin in mg/dL

OR

OF= (4.6 x [PT in sec - Control PT in sec])+ (serum bilirubin in mmol/L)


17.1

• DF ~32 have high short-term mortality & may benefit from glucocorticoids
• Those with lower scores may not benefit from glucocorticoids

402
III. MANAGEMENT
MANAGEMENT I REMARKS

• Cornerstone in the management of alcoholic liver disease


Complete
• Screen for alcohol abuse (screening tools: AUDIT-C, CAGE, MAST)
abstinence
from alcohol • FDA-approved medications to treat alcohol dependence: disulfiram,
acamprosate, naltrexone
• Recommended caloric intake: 2500 kcal/day (oral or NGT feeding)
Nutritional • Nighttime snack and morning feeding to improve nitrogen balance
support • Vitamin supplementation: folate, vitamin B, fat-soluble vitamins (A, D,
E), minerals (magnesium, selenium, zinc)

• Eligibility for treatment: Maddrey OF ~32 or MELD score >20


• Dose: Prednisolone 40 mg or equivalent+/- IV N-acetylcysteine
• For patients with no active GI bleeding, systemic infection, renal
insufficiency (creatinine >2.5 mg/dL), and concomitant disease (HBV,
Glucocorticoids HCV, acute pancreatitis, HIV, TB)
• Use Lille model score' after 7 days to determine response to steroids:
0 If Lille <0.45: may continue prednisolone for 28 days total
0 If Lille ~0.45: suggests patient is not responding to glucocorticoids
(stop prednisolone and consider early liver transplant)
• Inhibits TNF synthesis (which is increased in alcoholic hepatitis)
• Alternative to glucocorticoids for those who have contraindications to
Pentoxifylline steroids or at risk for sepsis
• Dose: 400 mg TID (400 mg OD if creatinine clearance <30 mL/min)
• Stop therapy when bilirubin <5 mg/dL
• Alcohol-associated cirrhosis is the second most common indication for
liver transplantation
Liver
• A 6-month period of abstinence was initially recommended as a
transplantation
minimum listing criterion to help ensure maximal hepatic recovery off
alcohol; however, this has not been shown to affect long-term survival
*Lillemodel score is based on patient's age, serum albumin,bilirubin,creatinine,& prothrombinlime
Source:SingalAK,et al.ACGClinicalGuideline.
TheAmericanJournalof Gastroenterology. 2017
JamesonJL,et.al.Harrison's
Principles
of InternalMedicine,20thedition.2018 : :
FeldmanM,et al. Sleisenger
andFordtran'sGastrointestinal
andLiverDisease11thEdition.2021 . :;

NON-ALCOHOLIC FATTY LIVER DISEASE (NAFLD)


I. ETIOPATHOGENESIS
Most common chronic liver disease in industrialized nations
Results when hepatocyte mechanisms for triglyceride synthesis overwhelm mechanisms
for triglyceride disposal, leading to accumulation of fat within hepatocytes
Triglycerides per se are not hepatotoxic but their precursors and metabolic by-products
may lead to hepatocyte lipotoxicity

II. MANIFESTATIONS
Most are asymptomatic (diagnosis made during incidental findings of abnormal liver
aminotransferases or features of fatty liver)
May present with vague RUQ pain or hepatomegaly
Most have features of metabolic syndrome
Associated with:
0 Overweight/obesity and insulin resistance (but can also occur in lean individuals)
0 Diabetes
0 Hypertriglyceridemia
0 Hypertension, cardiovascular disease
° Chronic fatigue, mood alterations
0 Obstructive sleep apnea
0 Thyroid dysfunction
403
III. DIAGNOSIS
Requires demonstration of increased liver fat in the absence of hazardous levels of
alcohol consumption (<I drink/day in women and <2 drinks/day in men)
Also requires exclusion of other causes of liver fat accumulation (e.g., drugs) and liver
injury (e.g., viral hepatitis, autoimmune liver disease, iron/copper overload)
Advanced imaging techniques, laboratory tests and scoring systems may be used as
noninvasive markers of either steatohepatitis or fibrosis:
DIAGNOSTIC I REMARKS
• Most widely used and validated technique (e.g., FibroScan)
• Sonography-based noninvasive and rapid bedside method for the
diagnosis and quantification of hepatic fibrosis
Transient
• Results are expressed in kilopascals (kPa): normal value is ~5 kPa
elastography
• Optimal cutoff for advanced fibrosis in NAFLD is 9.9 kPa (i.e., the stiffer the
(TE)
tissue, the faster the shear wave propagates)
• False positive in acute hepatitis, liver congestion, extrahepatic cholestasis,
excessive alcohol, food intake (should be performed at least 2 hrs of fasting)
Acoustic • Involves excitation of tissue using shon-duration acoustic pulses that
radiation propagate shear waves and generate localized displacements in tissue
force impulse • Higher applicability in ascites and obesity
(ARFJ) • Performance similar to TE for cirrhosis and significant fibrosis
MR • Uses modified phase-contrast method to image the propagation
elastography characteristics of the shear wave in the liver
(MRE) • Able to analyze entire liver, with higher applicability in ascites and obesity

JV. MANAGEMENT
Divided into three components:
0 Specific therapy of NAFLD-related liver disease
0 Treatment of NAFLD-associated comorbidities
0 Treatment of advanced NAFLD complications
At present, there are no FDA-approved therapies for treatment ofNAFLD
Only patients with NASH or those with features of hepatic fibrosis on liver biopsy
are considered for targeted pharmacologic therapies (metformin, thiazolidinediones,
vitamin E, ursodeoxycholic acid, omega-3 fatty acids)
NAFLD management focuses on treatment to improve risk factors for NASH
ASPECT I MANAGEMENT
• Moderate calorie restriction: aim to decrease daily calories by 500-750 kcal
Diet and
• Achieve 3-5% weight loss (improves steatohepatitis) or up to 10% weight loss
exercise
• Aerobic and/or resistance exercise 3-4 times week
• Vitamin E (most studied antioxidant) showed modest improvement
• Pioglitazone improves NASH and possibly fibrosis
• Omega-3 fatty acids improve triglycerides, decrease hepatic steatosis
Pharmacologic
• Stalins: no evidence to suggest that it can cause liver failure in patients
therapies
with any chronic liver disease, including NAFLD
• Future therapies: farnesoid X receptor agonists (obeticholic acid, aldafermin),
anti-fibrotics (cenicriviroc, selonsenib), PPAR-a/6 agonist (elafibrinor)
• Beneficial for metabolic syndrome & refractory obesity
Bariatric
• Not contraindicated in otherwise eligible patients with NAFLD/NASH
surgery
• Reduces liver fat and likely to reduce NASH progression
Liver • Patients with NAFLD in whom end-stage liver disease and/or HCC
transplantation develops (NAFLD is the 3rd most common indication for transplantation)
Monitoring & • NASH with fibrosis should receive closer monitoring because of a higher
surveillance risk of disease progression (HCC may develop in the pre-cirrhotic stage)
Source:Chalasam 2018& EASL,EASL-ALEH
N,et al. Hepatology; CPG:Journalof Hepatology
2015
FeldmanM,et al. SleisengerandFordtran's
Gastrointestinal
andLiverDisease11thEdition.2021
404
LIVER CIRRHOSIS
Condition defined histopathologically & has a variety of manifestations & complications
Represents a late stage of progressive hepatic fibrosis characterized by distortion of the
hepatic architecture and formation ofregenerative nodules
Generally considered to be irreversible in its advanced stages, at which point the only
option is liver transplant
The presence of complications differentiates decompensated versus compensated cirrhosis

Etiologies and Laboratory Features of Cirrhosis


ETIOLOGY I LABORATORY FINDINGS

• Alcoholic cirrhosis • Elevated AST and ALT' (but normal values do


• Chronic viral hepatitis B or C not preclude a diagnosis of cirrhosis)
• Autoimmune hepatitis • Elevated alkaline phosphatase (usually <2-3x the
• Nonalcoholic steatohepatitis upper normal limit")
• Biliary cirrhosis • Normal to elevated bilirubin levels
• Cardiac cirrhosis • Hypoalbuminemia
• Inherited metabolic diseases (e.g., • Abnormal prothrombin time (PT)
hemochromatosis, Wilson's disease) • Thrombocytopenia (most common hematologic
• Cryptogenic abnormality), leukopenia, anemia
• Shrunken, irregular/nodular liver on imaging
*ASTmoreoftenelevated thanALT.Mostchronic
hepatitis
(except
alcoholic
liverdisease)
haveAST/ALT ratio<1
**Higherlevelsof alkalinephosphatase
areseenin thosewithunderlyingcholestatic
liverdisease

Complications of Cirrhosis (discussed in detail below)


• Portal hypertension (varices, gastropathy, ascites, splenomegaly, hypersplenism)
• Hepatic encephalopathy
• Hepatorenal syndrome
• Portopulmonary hypertension
• Hepatopulmonary syndrome
• Cirrhotic cardiomyopathy
• Bone disease (osteopenia, osteoporosis, osteomalacia)
• Malnutrition
• Coagulopathy (factor deficiency, librinolysis, thrombocytopenia)
• Hematologic (anemia, hemolysis, thrombocytopenia, neutropenia)

I. PORTAL HYPERTENSION "


Elevation of hepatic venous pressure gradient >5 mmHg
Cirrhosis is the most common cause of portal hypertension
Development of portal hypertension is usually revealed by thrombocytopenia, splenomegaly &
development of ascites, encephalopathy, &Jor esophageal/gasnic varices with/without bleeding
Classification of Portal Hypertension

I PRE-HEPATIC I HEPATIC I POST-HEPATIC


• Affects portal • Most common; can be • Affects hepatic
Description system before it pre-sinusoidal, sinusoidal, veins & venous
enters the liver post-sinusoidal drainage to heart
• Pre-sinusoidal
0 Schistosomiasis

• Portal vein ° Congenital hepatic fibrosis


thrombosis 0 Primary biliary cirrhosis , Budd-Chiari
• Splenic vein 0 Sarcoidoisis syndrome
Examples thrombosis • Sinusoidal • Inferior vena
• Massive ° Cirrhosis caval webs
0 Alcohol-associated hepatitis
splenomegaly • Cardiac causes
(Banti syndrome) • Post-sinusoidal
0 Hepatic sinusoidal obstruction

(veno-occlusive syndrome)
405
A. Ascites
I. Etiopathogenesis and Manifestations
• Accumulation offluid within peritoneal cavity, presenting as progressive abdominal distention
• Ascites is due to vasodilation of the splanchnic arterial system due to portal
hypertension, resulting from:
• Increased splanchnic pressure due to increased portal venous flow
• Underfilling of arterial system leads to RAAS activation and Na· & H2O retention
• Decreased synthetic function of the liver causes hypoalbuminemia, leading to
decreased oncotic pressure
2. Diagnosis
• Diagnostic paracentesis should be performed in all patients with new-onset ascites
that is accessible for sampling
• Initial tests should include (see Chapter 2 for details):
• Ascitic fluid neutrophil count
• Ascitic fluid total protein, albumin
• Simultaneously obtained serum albumin
M ;
GRADE I DESCRIPTION I MANAGEMENT
• Mild ascites only
I • No treatment
detectable by US
• Moderate ascites with • Sodium (Na•) & fluid management'
2 moderate symmetrical • Diureticsb
abdominal distension • Consider referral for liver transplant evaluation
• Initial treatment of choice: large-volume
• Tense or gross ascites paracentesis (LVP)' + hyperoncotic human albumin
• Oral diuretics after LVP (spironolactone 100 mg/
3 with marked abdominal
distension day & furosemide 40 mg/day)
• Consider referral for liver transplant evaluation
• First-line treatment: LVP with albumin infusion
• Ascites that cannot
• Hold diuretics
be mobilized by
• Consider TIPS procedure
maximum diuretics, or
• Non-selective beta-blockers (not necessarily
the development of a
Refractory contraindicated)
complication related
• Others: midodrine, IV albumin, vaptans,
to diuretic therapy that
alfapump system (implantable pump that
precludes its effective
transfers ascitic fluid to urinary bladder)
dosage
• Referral for liver transplantation

a Na· & fluidmanagement are important in all grades. Moderate Na' restrictionis 2 g (90 mmol)/dayto
achieve negative Na' balance & net fluidloss. Fluid restrictionis not necessary unless Na' ~125 mmol/L.
b Diuretics:aldosterone antagonists (spironolactone)& loop diuretics (furosemide, torsemide, bumetanide)
are mainstays of diuretictreatment. Initialdose of spironolactone is 100 mg/day, increased by 100 mg
every 72 hours to a maximumof 400 mg/day. Initialdose of furosemide is 40 mg/day, increased
progressivelytowards 160 mg/day.
c LVPis arbitrarilydefined in the latest guidelines as paracentesis of >5 liters.

B. Hypersplenism
0 Hypersplenism with the development of thrombocytopenia is a common feature of
patients with cirrhosis and is usually the first indication of portal hypertension
0 Splenomegaly itself usually requires no specific treatment
0 Supportive transfusion with platelet concentrate as needed during episodes of bleeding
Source: BigginsSW,et al. Hepatology.2021
AithalGP,et al. Guidelineson the Managementof ascites in cirrhosis.Gut 2021
Jameson JL, et.al. Harrison'sPrinciplesof InternalMedicine,20th edition.2018
FeldmanM,et al. Sleisengerand Fordtran'sGastrointestinaland LiverDisease 11thEdition.2021
406
C. Gastrointestinal Varices
1. Etiopathogenesis and Manifestations
• Resistance to portal venous flow leads to increased resistance in portal pressure
• Decreased splanchnic arteriolar resistance leads to increased splanchnic flow
(increased portal blood flow)
• May present as bleeding (i.e., bleeding esophageal varices may cause hematemesis)
2. Management (see details in the Manaqement of GI bleedinq):
Primary • Routine screening by endoscopy, nonselective beta-blockers,
prophylaxis endoscopic variceal band ligation
Treatment of an • Vasoconstrictors(somatostatin,octreotide),balloon tamponade, endoscopic
acute bleed intervention (sclerotherapy, variceal band ligation), TIPS, surgery
Prevention of • Repeated variceal band ligation
rebleeding after • Beta-blockers
an initial bleed • Portosystemic shunt surgery

II. HEPATIC ENCEPHALOPATHY (HE)


Alteration in mental status and cognitive function occurring in the presence ofliver failure
• Encephalopathy is more commonly seen in chronic liver disease
In acute liver injury with fulminant hepatic failure, encephalopathy is a requirement for diagnosis

A. Etiopathogenesis
0 Symptoms due to gut-derived neurotoxins not removed by liver because of vascular shunting
0 Precipitating events:
• Hypokalemia: causes increased ammoniagenesis (alkaline tide)
• Infection
• Increased dietary protein load
• GI bleeding
0Ammonia levels are usually elevated but poorly correlate with severity of liver disease

:
B. Manifestations
° Confusion, changes in behavior, violent, difficult to manage, sleepy, and difficult to rouse
0 Asterixis or liver flap: sudden forward movement of the wrist after it is bent back on an
extended arm (cannot be elicited if already comatose) .
° Cerebral herniation is a feared complication of brain edema ."
0 Diagnosis is clinical

0 • Normal

• Minor abnormalities of visual perception


Minimal • Subtle changes in work or driving
or on psychometric or number tests
• Personality changes, attention
1 deficits, irritability, depressed state
• Tremor, incoordination

• Changes in sleep-wake cycle,


2 lethargy, mood and behavioral
changes, cognitive dysfunction
• Asterixis, ataxic gait, speech
abnormalities (slow and slurred)

• Altered level of consciousness


• Muscular rigidity, nystagmus, clonus,
3 (somnolence), confusion,
disorientation, amnesia
Babinski sign, hyporeflexia

• Oculocephalic reflex,
4 • Stupor and coma
unresponsiveness to noxious stimuli
Source:FerenciPet al. Hepatology;
2002
407
C. Management
THERAPY I REMARKS
• Protein restriction is discouraged as it has negative impact on overall nutrition
• Replace animal-based protein with vegetable-based protein in the diet
Nutrition
because of more calorie-to-nitrogen ratio
• Zinc supplementation and L-ornithine-L-aspartate may be helpful
• Mainstay of treatment for encephalopathy
• Dose of 30-45 cc BID to QID per orem to promote 2-3 soft stools per day
Lactulose
• Mechanisms of action: colonic acidification and catharsis
• May be administered per rectum (as enema) to those at risk of aspiration
• Adjunctive to lactulose
• Poorly absorbed antibiotics: neomycin & metronidazole (given
Antibiotics
alternately to reduce individual side effects)
• Rifamixin 550 mg BID: very effective with a better safety profile
• Acarbose: inhibits intestinal absorption of carbohydrates and glucose,
resulting in their enhanced delivery to the colon (therefore, increased
Other ratio of saccharolytic to proteolytic bacterial flora)
agents • Probiotics: modify intestinal flora and decrease ammonia generation
• Sodium benzoate, sodium phenylbutyrate, sodium phenylacetate:
increase ammonia clearance in urine
• Correction of aforementioned precipitating factors (e.g., hypokalemia)
• Hydration and correction of electrolyte imbalance
Supportive
• MARS (molecular adsorbent recirculating system): extracorporeal
albumin dialysis improves severe HE in acute-on-chronic liver failure
Source:Vilstrupet al; Hepatology 2014
JamesonJL, et.al.Harrison'sPrinciplesof InternalMedicine,20thedition.2018
FeldmanM,et al. SleisengerandFordtran'sGastrointestinalandLiverDisease11thEdition.2021

III. SPONTANEOUS BACTERIAL PERITONITIS (SBP)


Spontaneous infection of ascitic fluid without an intra-abdominal source, usually in the
setting of liver cirrhosis
Low-protein ascitic fluid (<I g/dL or IO g/L) is particularly susceptible to SBP because the
endogenous antimicrobial (opsonic) activity of the ascitic fluid correlates directly with
its protein concentration

A. Etiopathogenesis
1. Etiology
• Most common organisms are Escherichiacoliand other gut bacteria
• Isolation/growth of >2 organisms should raise suspicion for perforated viscus
(secondary bacterial peritonitis)

2. Pathogenesis
• Alterations in the gut-liver axis (gut dysbiosis, increased intestinal permeability,
bacterial translocation)
• Cirrhosis-associated immune dysfunction
• Local peritoneal factors (low ascitic fluid protein <I.Oto 1.5g/dL)

B. Manifestations & Diagnosis


0Presents as fever, altered mental status, elevated WBC, and abdominal pain/discomfort
0Ascitic fluid sample analysis:
• Absolute neutrophil count >250/uL
• Positive culture (but may be negative in up to 60% of patients with SBP)
• No evidence of an intra-abdominal surgically treatable source of infection
408
CM
ASPE~T I SPECIFIC MANAGEMENT
• Community-acquired SBP: third generation cephalosporins (empiric
antibiotic of choice) such as cefotaxime 2 g IV every 12 hours,
amoxicillin-clavulanic acid, fluoroquinolone
• Health-care associated or nosocomial SBP:
0 Piperacillin/tazobactam (first-line in areas with low prevalence ofMDR)
Antibiotics 0 Meropenem +/- vancomycin/daptomycin/linezolid (in areas with high
prevalence of MOR bacteria)
• Repeat paracentesis 48 hours after antibiotics should be done to
monitor efficacy: if ascitic fluid neutrophil count did not drop by at
least 25% from pretreatment value, there is high probability of failure to
respond to therapy
• IV albumin 1.5 g/kg of body weight at the time the infection is detected
IV albumin
(preferably within 6 hours), and I g/kg on day 3
Sodium • Diuretic-sensitive patients should preferably be treated with sodium
restriction restriction and oral diuretics rather than with serial paracentesis
& diuretics (paracentesis removes opsonins while diuresis concentrates opsonins)
Source:BigginsSW.et al. Hepatology.2021

D. Prevention ofSBP
0Prophylaxis is aimed at achieving selective intestinal decontamination by reducing
Gram-negative bacteria
0Oral norfloxacin is the treatment of choice
INDICATION
I DRUG

Primary prophylaxis (if without UGIB) • Norfloxacin 400 mg/day


• Cirrhosis with ascites & ascitic fluid • Ciprofloxacin 500 mg/day
protein <I.5 g/dL • Trimethoprim-sulfamethoxazole I double-
• Advanced cirrhosis (Child-Pugh "'9 strength tablet OD
or serum bilirubin ;;:3 mg/dL)
• Impaired renal function
(creatinine ;;:1.2 mg/dL, BUN ;;:25 mg/
..
dL, and/or Na· :;;130 mEq/L)
Primary prophylaxis in cirrhosis • Ceftriaxone I g IV OD is the antibiotic of
with UGIB choice, for a maximum of 7 days
• Norfloxacin 400 mg/day (if not available,
Secondary prophylaxis (previous oral ciprofloxacin is acceptable)
episode ofSBP) • Trimethoprim-sulfamethoxazole I double-
strength tablet OD
Source:B1gg1ns SW,et al. Hepatology.2021
FeldmanM, et al. Sleisengerand Fordtran'sGastrointestinaland LiverDisease11thEdition.2021

409
GALLSTONE DISEASE
DISORDER I DESCRIPTION
• Gallstone anywhere in the biliary tree
Cholelithiasis • Divided into two major types:
("Gallstones") ° Cholesterol stones (80%)
0 Pigment stones (<20%)

• Passage of stones in the common bile duct (CBD)


• CBD stones should be suspected in gallstone patients who have any of
Choledo- the following risk factors:
cholithiasis 0 History of jaundice or pancreatitis

0 Abnormal tests ofliver function


0 Ultrasonographic/MRCP evidence of a dilated CBD or stones in the duct

Acute • Acute inflammation of the gallbladder wall usually follows


Cholecystitis obstruction of the cystic duct by a stone
• Bacterial infection superimposed on an obstruction of the biliary tree,
Ascending
most commonly from a gallstone
Cholangitis
• Medical +/- surgical emergency

I. ETIOPATHOGENESIS
Big Four risk factors for gallstones: Female, Fat (obesity), Forty, and Fertile (multiparity)
• Others: pregnancy,Crohn's disease, gastric/terminal ilea!surgery,hemolytic disorders, biliary stasis

II. CLINICAL MANIFESTATIONS


A. Biliary Colic
0 Most specific and characteristic symptom of gallstone disease
0 Steady epigastric or RUQ pain, radiating to interscapular area, right scapula or shoulder
0 Begins quite suddenly and may persist with severe intensity for 15minutes to 5 hours
0 Subsides gradually or rapidly
0 Persistence of pain beyond 5 hours should raise the suspicion of acute cholecystitis
0 May be precipitated by eating a fatty meal or consumption of a large meal following a
period of prolonged fasting

B. Other Symptoms
0 Nausea and vomiting
0 Jaundice usually if with CBD obstruction
° Fever/chills with biliary pain imply a complication: cholecystitis, pancreatitis, cholangitis
C . I mportant 1gns to El'lClt.
• Deep inspiration or cough during subcostal palpation of the
Murphy's sign RUQ usually produces increased pain and inspiratory arrest
• Usually in acute cholecystitis
• Sudden RUQ tenderness
Triad of Acute
• Fever
Cholecystitis
• Leukocytosis "
• Biliary (RUQ) pain
Charcot's Triad of
• Jaundice
Acute Cholangitis
• Spiking fevers with chills
Reynolds' • Biliary (RUQ) pain • Shock
Pentad of Acute • Jaundice • Altered mental status
Cholangitis • Fever
410
III. DIAGNOSIS
DIAGNOSTIC I REMARKS
• Leukocytosis if with inflammation (e.g., cholecystitis, cholangitis)
Basic tests • Bilirubin, alkaline phosphatase: elevated levels suggest biliary
obstruction (e.g., common bile duct stones)
• Rapid, accurate identification of gallstones (>95%)
Transabdominal
• Procedure of choice for detection of stones
gallbladder
• Findings: thickening of wall, pericholecystic fluid, dilated bile duct
ultrasound (US)
• Limitations: bowel gas, massive obesity, ascites
Endoscopic • Permits gallbladder visualization without interference from bowel
gallbladder gas, subcutaneous tissue, or the liver
ultrasound (US) • More sensitive than transabdominal US for detection of gallstones
• Pathognomonic findings in:
° Calcified gallstones
Plain abdominal
0 Limey bile, porcelain GB
X-ray
0 Emphysematous cholecystitis
0 Gallstone ileus
MR cholangio-
• Useful modality for visualizing pancreatic and biliary ducts
pancreatography
• Cannot offer therapeutic intervention
(MRCP)
Endoscopic • Best visualization of distal biliary tract
retrograde • Cholangiogram of choice in the absence of dilated ducts,
cholangio- pancreatic or ampullary disease, prior biliary surgery, & when
pancreatography endoscopic sphincterotomy is a treatment possibility
(ERCP) • Can be complicated by pancreatitis, cholangitis, or perforation
Percutaneous • Best visualization of proximal biliary tract
transhepatic • Can provide biliary drainage
cholangiogram • Indicated when ERCP is contraindicated or has failed

IV. MANAGEMENT
CONDITION I MANAGEMENT "

• Laparoscopic cholecystectomy: gold standard for symptomatic stones


• Ursodeoxycholic acid (UDCA) 10-15 mg/kg per day: used for cholesterol
Cholelithiasis
stones, but therapy should be limited to radiolucent stones :5:IOmm in
(gallstones)
diameter (<6 mm optimal)
• Pigment stones are not responsive to UDCA therapy
• Endoscopic biliary sphincterotomy (EBS)followed by spontaneous passage
or stone extraction is the treatment of choice, especially in the elderly
Choledo-
• Laparoscopic cholecystectomy & ERCP has decreased the incidence of
cholithiasis
complicated biliary tract disease & need for choledocholithotomy and
T-tube drainage of the bile ducts
• Cholecystectomy:mainstay of therapy for acute cholecystitis& complications
• Meperidine or NSAIDs: usually employed for analgesia because they may
Acute
produce less spasm of the sphincter of Oddi than drugs such as morphine
cholecystitis
• Antibiotics in severe cholecystitis should be guided by the most common
organisms likely to be present (e.g.,E. coli,Klebsiellaspp.,Streptococcus
spp.)
• ERCPwith endoscopic sphincterotomy: preferred initial procedure for
Ascending
both establishing a definitive diagnosis and providing effective therapy
cholangitis
• Endoscopic management of cholangitis is as effective as surgery

411
ACUTE PANCREATITIS
I. ETIOPATHOGENESIS
• Inflammation of the pancreas due to activation of enzymes within the pancreas
• Mechanisms for the pancreatic inflammation have not been well-established

A. Pathophysiology: Autodigestion
° Currently accepted pathogenic theory
0Proteolytic enzymes (e.g., trypsinogen, chymotrypsinogen, proelastase) are activated in
the pancreatic acinar cells rather than in the intestinal lumen

B. Common Etiologies of Pancreatitis (G-A-T-E-D)


• Gallstones: most common cause'
• Alcohol: 2nd most common cause•
• Hypeririglyceridemia (usually with serum triglycerides >1000 mg/dL)
• .Endoscopic retrograde cholangiopancreatography (ERCP)
• Drugs
•Trauma
• Post-operative
• Sphincter of Oddi dysfunction
*Forrecurrentattacksof pancreatitis,
thetwo mostcommonetiologiesare alcohol& cholelithiasis

II. CLINICAL MANIFESTATIONS


A. Signs and Symptoms of Acute Pancreatitis
Sy,mptoms 11 Iii'-

• Major symptom of acute pancreatitis


• Quality: steady and boring in character
Abdominal • Location: epigastrium and periumbilical region
pain • Radiation: back, chest, flanks, lower abdomen
• Effects of position changes: more intense when supine; relieved upon
sitting with the trunk flexed and knees drawn up
Others • Nausea, vomiting, and abdominal distention
•.
"
Signs !I
• Distressed and anxious patient
General
• Low-grade fever, tachycardia, and hypotension are fairly common
• Hypovolemia secondary to exudation of blood and plasma proteins
Shock into the retroperitoneum
• Systemic effects of proteolytic & lipolytic enzymes released into circulation
Abdominal • Compared with the intense pain, these signs may be unimpressive
tenderness • Guarding more marked in the upper abdomen
Bowel·sounds • Decreased or absent
• Due to edema of the pancreatic head with compression of the
Jaundice intrapancreatic portion of the CBD
(infrequent) • Possible choledocholithiasis (gallstone pancreatitis)
• Co-existent liver disease
Cullen's sign • Blue discoloration around umbilicus (results from hemoperitoneum)
Turner's • Blue-red-purple or green-brown discoloration of the flanks (reflects
sign tissue catabolism of hemoglobin)
Source:JamesonJL,et.al.Harrison's
Principles
of InternalMedicine,
20thedition.2018
FeldmanM,et al.SleisengerandFordtran'sGastrointestinal
andLiverDisease11thEdition.
2021
412
B. Revised Atlanta Definitions of Morphologic Features of Acute Pancreatitis
FEATURE I DEFINITION

Categories of Acute Pancreatitis


Interstitial • Acute inflammation of pancreatic parenchyma & peripancreatic tissues
pancreatitis • No recognizable tissue necrosis
• Inflammation associated with parenchymal &Jorperipancreatic necrosis
Necrotizing
• Higher mortality in necrotizing pancreatitis (compared to interstitial
pancreatitis
pancreatitis)
,.·-
local Complications of Acute Pancreatitis '
Acute • Peripancreatic fluid associated with interstitial edematous pancreatitis
pancreatic • No associated necrosis
fluid • Applies only to areas of fluid seen within the first 4 weeks after onset of
collection interstitial edematous pancreatitis and without features of a pseudocyst
• Encapsulated collection of fluid with a well-defined inflammatory
Pancreatic
wall usually outside the pancreas with minimal or no necrosis
pseudo cyst
• Usually occurs >4 weeks after onset of interstitial edematous pancreatitis
Acutenecrotic
• Collection containing variable amounts of both fluid and necrosis
collection
associated with necrotizing pancreatitis
(ANC)
Walled-off • Mature, encapsulated collection of pancreatic and/or peripancreatic
necrosis necrosis that has developed a well-defined inflammatory wall
(WON) • Usually occurs >4 weeks after onset of necrotizing pancreatitis
Source:JamesonJL, et.al.Harrison'sPrinciplesof InternalMedicine,20thedition.2018
BanksP,et al. Gut62:102,2013

C. The Revised Atlanta Criteria


0 Defines the phases of pancreatitis, outlines severity of the disease, and clarifies

I
imaging definitions
0 Note that organ failure is defined as a score of ,!2 for any one of three organ systems
(respiratory, cardiovascular, or renal) using the modified Marshall scoring system "

1. Phases of Acute Pancreatitis


PHASE I REMARKS
• Severity is defined by clinical parameters, rather than morphologic
findings
• Most exhibit SIRS and are predisposed to organ failure
Early • Three organs should be assessed to define organ failure: respiratory,
(<2 weeks) cardiovascular, and renal
• Persistent organ failure (>48 hours): most important clinical finding
with regard to severity of the acute pancreatitis episode
• Lasts 1-2 weeks
• Characterized by a protracted course of illness and may require
imaging to evaluate for local complications
Late • Important clinical parameter of severity: persistent organ failure
(>2weeks) • Radiographic feature of greatest importance is necrotizing
pancreatitis on CT
• May require supportive measures (dialysis, ventilator support, TPN)

413
• No local complications or organ failure•
• Self-limited & subsides within 3-7 days after treatment is instituted
Mild
• Oral intake may be resumed if patient is hungry, has normal bowel
function, and has no nausea/vomiting

Moderately • Transient organ failure• (resolves in <48 hours), or local or systemic


Severe complications in the absence of persistent organ failure
• Persistent organ failure• (>48 hours)
Severe
• CT or MRI should be obtained to assess for necrosis &/or complications
*Organfailure:acuterespiratory
failure,shock,renalfailure
Source:JamesonJL,et.al.Harrison'sPrinciplesof InternalMedicine,20thedition.2018
BanksP,et al. Gut62:102,2013

D. Complications
LOCAL COMPLICATIONS I SYSTEMIC COMPLICATIONS
• Necrosis (sterile or infected) • Pulmonary: ARDS, effusion, atelectasis,
• Walled-off necrosis pneumonitis, mediastinal fluid, unrecognized
• Pancreatic fluid collections hypoxemia
(pseudocyst, abscess) • Cardiovascular: hypotension, pericardia! effusion,
• Pancreatic duct disruption ST-T wave changes on ECG, hypovolemia
• Pancreatic ascites • Hematologic: DIC
• Involvement of contiguous • GI hemorrhage: ulcer formation, erosive gastritis,
organs by necrotizing hemorrhagic pancreatic necrosis with erosion into
pancreatitis major vessels, portal vein thrombosis, splenic vein
• Thrombosis of blood vessels thrombosis, variceal hemorrhage
(splenic vein, portal vein) • Renal: oliguria, azotemia, renal artery/vein
• Bowel infarction thrombosis, acute tubular necrosis
• Pancreatic enteric fistulization • Metabolic: hyperglycemia, hypertriglyceridemia,
(usually to the left colon) hypocalcemia, encephalopathy, sudden blindness
• Obstructive jaundice (Purtscher's retinopathy)
• CNS: psychosis, fat emboli
• Fat necrosis: subcutaneous (erythematous nodules),
bone, miscellaneous (mediastinum, pleura)
Source:JamesonJL,et.al.Harrison'sPrinciplesof InternalMedicine,20thedition.2018
FeldmanM,et al. SleisengerandFordtran'sGastrointestinal
andLiverDisease11thEdition.2021

III. DIAGNOSIS
A. Criteria for Pancreatitis
0 Any severe/acute abdominal or back pain should suggest acute pancreatitis
0 Other symptoms include nausea, emesis, fever, tachycardia, and abnormal findings on
abdominal exam
0 Although not required for diagnosis, markers of severity may include:
• Hemoconcentration (hematocrit >44%)
• Azotemia on admission (BUN >22 mg/dL)
• SIRS and signs of organ failure

Diaanosis is established b112 of the followinq 3 criteria:


• Typical abdominal pain in the epigastrium that radiates to the back
• Threefold or greater elevation in serum lipase and/or amylase
• Confirmatory findings of acute pancreatitis on cross-sectional abdominal imaging

414
B. Diagnostics for Pancreati tis
DIAGNOSTIC I REMARKS/EXPECTED FINDINGS
• Increased levels (more than 3-fold) within 6-12hours in acute pancreatitis
• Returns to normal after 3-7 days
Amylase• • Differentials for elevated amylase: acidemia (arterial pH s7.32)as in OKA,
macroamylasemia, papillary cystadenocarcinoma of the ovary, benign
ovarian cyst, lung CA, intestinal infarction, perforated viscus
• Increased levels (more than 3-fold) within 4-8 hours and peaks at 24
hours in acute pancreatitis
• Elevated for 7-14days
Lipase•
• Preferred test (more specific than amylase)
• Can be instrumental in differentiating a pancreatic or nonpancreatic
cause ofhyperamylasemia
• Leukocytosis (15,000-20,000/µL)
CBC • Hemoconcentration with hematocrit >44% & failure to decrease levels in
24 hours from admission are predictors of necrotizing pancreatitis

Renal • Azotemia with BUN >22 mg/dL (associated with increased mortality) due
function to loss of plasma into the retroperitoneal space and peritoneal cavity
• Hyperglycemia: due to decreased insulin release, increased glucagon
release, increased output of adrenal glucocorticoids and catecholamines
• Hypocalcemia: due to decreased albumin (calcium is normally bound to
albumin, which is lost as albumin-rich intravascular fluid extravasates
into the peritoneum or retroperitoneum)
• Hyperbilirubinemia (>4.0 mg/dL), transiently elevated serum ALP and
Serum
AST: acute biliary obstruction from choledocholithiasis or inflammation
chemistry
in the pancreatic head (gallstone pancreatitis)
• Hypertriglyceridemia >IO00 mg/dL may precipitate attacks of acute
pancreatitis (serum amylase in this setting may be spuriously normal)
• ALT concentration 150 IU/L (-3-fold elevation) may distinguish gallstone
pancreatitis from other causes
• Markedly elevated serum LDH levels: poor prognosis
ABG • Hypoxemia (arterial PO2 s6o mmHg) may herald the onset of ARDS '

12-LECG • ST-segment & T-wave abnormalities may simulate myocardial ischemia


Chest • A pleural effusion documented within 72 hours of admission by chest
radiograph X-ray (or CT) correlates with severe disease
• Initial diagnostic imaging to evaluate for gallstone disease and the
Sonography
pancreatic head
• Best evaluated 3-5days into hospitalization when patients are unresponsive
to supportive care to look for local complications such as necrosis
• Two types of pancreatitis on imaging: interstitial & necrotizing
Abdominal
• Interstitial pancreatitis (90-95%):diffuse enlargement, homogenous
CT scan
contrast enhancement, mild inflammation or peripancreatic stranding
• Necrotizing pancreatitis (5-10%):lack of pancreatic parenchymal
enhancement and/or presence of pancreatic necrosis
'There is no correlationbetween severityof pancreatitis& degree of serum amylase & lipase elevations
Source:Jameson JL, et.al. Harrison'sPrinciplesof InternalMedicine,20thedition.2018
FeldmanM,et al. Sleisengerand Fordtran'sGastrointestinaland LiverDisease 11thEdition.2021
WorkingGroupIAP/APAAcute PancreatitisGuidelines.Pancreatology13. 2013

415
C. Defining Severe Acute Pancreatitis
Risk Factors for ~everity
• Age >60 years old
• Obesity (BM! >30 kg/m')
• Comorbid disease (Charlson Comorbidity Index)
•F -
Markers of Seve~ity (lt Admission or Within 24 Hours
• SIRS criteria (2 or more)
° Core temperature <36 or >38 °C
0 Heart rate >90 bpm
0 Respiration >20/min or PCO2 <32mmHg
0 WBC >12,000/uL, <4,000/uL, or wo/o bands
• APACHE II
• Hemoconcentration (hematocrit >44%)
• Admission BUN (>22 mg/dL)
• BISAP (?!3of these factors: associated with increased risk for in-hospital mortality)
0 B: BUN >25mg/dL

0 I: Impaired mental status (GCS <15)


0 S: SIRS criteria ?!2of 4 present

0 A: Age >60 years

0 P: Pleural effusion
• Organ Failure (Modified Marshall Score)
• Cardiovascular: SBP <90 mmHg, HR >130bpm
• Pulmonary: PaO2 <60 mmHg
• Renal: serum creatinine >2.0 mg/dL
.-
Markers of Seve~ity Dqring Ho,spitalization --
• Persistent organ failure (>48 hours)
• Pancreatic necrosis
Source:JamesonJL, et.al.Harrison'sPrinciplesof InternalMedicine,20thed1t1on.
2018
WuBU,et al. Theearlypredictionof mortalityin acutepancreatitis.
Gut.2008
IV. MANAGEMENT
Usually the disease is self-limited and subsides spontaneously
• Resolution occurs within 3-7 days after treatment is instituted
MANAGEMENT I REMARKS
• Analgesics for pain
• Oxygen via nasal cannula (2 1pm)
Conventional
• No oral alimentation (NPO) until nausea and vomiting have subsided
measures
(but there has been a major change in this concept and currently, gut
rousing and not gut resting is the key management)
• The most important intervention: safe, aggressive IV fluid
resuscitation
• Initial !VF: Lactated Ringer (LR) solution at a rate of5-10 mL/kg/hr or
250 to 500 mL/hr, until resuscitation goals are reached
• Assessment of response should be based on I or more of the following:
Fluid 0 Noninvasive clinical targets: heart rate <120bpm, mean arterial

resuscitation pressure between 65-85 mmHg, urinary output >0.5-1mL/kg/hr


0 Invasive clinical targets: stroke volume variation, intrathoracic
blood volume determination
0 Biochemical target: hematocrit 35-44%
• Measure hematocrit and BUN every 8-12hours and serum
electrolytes daily to ensure adequacy of fluid resuscitation
416
MANAGEMENT I REMARKS

Antibiotics/ • Prophylactic antibiotics have no role in either interstitial or


probiotics necrotizing pancreatitis
• To evaluate for necrosis and other local complications if the
CT scan patient still exhibits evidence of severe disease and/or organ
failure >72 hours despite adequate resuscjtation
• For gallstone pancreatitis (transient or sustained impaction of a
stone or sludge within the common channel of the ampulla of Yater)
• Patients with suspected biliary obstruction, based on
hyperbilirubinemia and evidence of clinical cholangitis, should
undergo ERCP (with or without sphincterotomy) within
ERCP 24 to 48 hours of admission
• Biliary sphincterotomy refers to cutting the biliary sphincter and
the intraduodenal segment of the CBD using a high-frequency
current applied with a special knife (sphincterotome)
• Biliary sphincterotomy can reduce the risk of recurrent acute
pancreatitis and cholangitis prior to cholecystectomy

• Early refeeding improved outcomes and allowed early discharge


• Time to reinitiate oral feeding depends on the severity of
pancreatitis:
• Mild acute pancreatitis: oral feedings (low-fat solid diet > clear
Resumption of liquids) can be started immediately if there is no nausea &
diet vomiting and abdominal pain has subsided
• Severe acute pancreatitis: early enteral nutrition is
recommended to prevent infectious complications
• Parenteral nutrition should be generally avoided unless enteral route
is not tolerated, not available, or not meeting caloric requirements

II
• In patients with mild gallstone pancreatitis, cholecystectomy
during index admission appears safe and is recommended
• In patients with peripancreatic collections, cholecystectomy should
Role of surgery be delayed until the collections either resolve or if they persist beyond ·_'.
6 weeks, at which time cholecystectomy can be performed safely ..
• Alternatively, for patients who are not surgical candidates,
endoscopic biliary sphincterotomy may be done before discharge
Source:JamesonJL,et.al.Harrison'sPrinciples of InternalMedicine,
20thedition.2018
FeldmanM, et al. Sleisengerand Fordtran'sGastrointestinal
and LiverDisease11thEdition.
2021
WorkingGroupIAP/APAAcute PancreatitisGuidelines.
Pancreatology13.2013

417
SECTION SEVEN
OVERVIEW OF THE BASICS IN NUTRITION·

BASIC DEFINITIONS

TERMS I DESCRIPTION
Nutrition • Intake of food, considered in relation to the body's dietary needs
• Use of nutrition services to manage a certain condition or illness
Medical nutrition
• Encompasses oral nutritional supplements, enteral nutrition,
therapy (MNT)
and parenteral nutrition
Enteral nutrition • Administration of food and nutrients by oral route and/or by a
(EN) tube directly into the gastrointestinal tract
• Administration of nutrients via the venous route
Parenteral nutrition
• Indicated for individuals who can neither accept nor assimilate
(PN)
nutrients given via the enteral route
• Quantity of essential nutrients needed to meet the minimal
Recommended requirement to maintain health
nutrient intake
(RN!) • Provides adequate reserves, plus additional amount of
nutrients for incomplete digestion
• Synonymous to reference, ideal, expected or standard body weight
Desirable body
• Weight for height found statistically to be the most compatible
weight (DBW)
with health and longevity
Actual body weight • Weight measured during hospitalization or reported just
(BW) before the hospitalization
• Applicable in the obese patient
Adjusted body
weight(BW) • Calculated as:
(Actual BW - Desirable BW) x_o.33+ Desirable BW
Isocaloric diet • Energy administration around the defined target
Hypocaloric diet • Energy administration of <70% of the defined target
Hypercaloric diet • Energy administration of>noo/o of the defined target
Low protein diet • Protein administration of <0.5 g/kg/day
• Energy expended at rest, during physical activity, and as a
Total energy result of thermogenesis
expenditure • Measured using calorimetry (the determination of energy
expenditure by measuring the heat produced by the body)
Indirect • Measures the amount of oxygen consumed and the amount of
calorimetry carbon dioxide produced by the body over a period of time
• Severe disruption in electrolyte or fluid balance that is
Refeeding precipitated in malnourished patients when feeding (oral, EN,
syndrome PN) is started too aggressively after a period of inadequate
nutrition
Source:Cederholm T,et al. ESPENguidelines.
ClinicalNutrition2017
TanchocoCC,Jamorabo-Ruiz A. DietManual5thEdition2010
MtawehH,et al. IndirectCalorimetry:
History,Technology,
andApplication.
Front.Pediatr2018

418
DIETARY CALCULATIONS
A thorough nutritional assessment is the basis of a nutritional care plan
• In arriving at a nutritional plan for patients, a number of simple steps can be used:

STEP 1: Determine/Compute the Desirable Body Weight (DBW)


• DBW is the optimal weight associated with maximum life expectancy for a given height
• May be derived by any of the following methods:

A W . : h 6 H . : h T: bl ( : 6 )b F d&N R hi (FNRI)
MALES FEMALES

4' 6" 28-35kg 5'4" 53-65kg 4' 6" 28-35kg 5' 4" 49-60kg
4' 7" 30-39kg 5' 5" 55-68kg 4' 7" 30-37kg 5' 5" 51-62kg
4' 8" 33-40kg 5' 6" 58-70kg 4' 8" 32-40kg 5' 6" 53-65kg
4' 9" 35-44kg 5' 7" 60-74kg 4' 9" 35-42kg 5' 7" 55-67kg
4' 10" 38-46kg s·8" 63-76kg 4' 10" 36-45kg 5' 8" 57-70kg
4' 11" 40-50kg 5' 9" 65-80kg 4' 11" 39-47kg 59
1 11
59-72kg
5' 0" 43-53kg 5' 10" 67-83kg 5' 0" 40-50kg 5' 10" 61-75kg
5' 1" 45-55kg 5' 11' 70-85kg 5' 1" 43-52kg 5' 11" 63-77kg
5' 2" 48-59kg 6' 0" 72-89kg 5' 2" 45-55kg 6' 0" 65-80kg
5' 3" 50-61kg 5' 3" 47-57kg

B. Derived Formula based on Body Mass Index (BM!)


0 BM! is widely used to identify lean, overweight, or obese individuals
0 The generally considered normal BM! range is 20 to 24.9 kg/m'

DBW(kg)
= Desirable BMI x (Height in m)'
• DesirableBMI:
° For men = 22 kg/m2
° For women= 21 kg/m2
Source:NuttallFQ.CriticalReview.NutrToday.2015
I '

C. Modified Tannhauser's Method


• Measureheightin centimeters
DBW(kg) • Deduct100& theansweris the DBW
• DBWobtainedappliesto Filipinos'
= [height (cm) - 100] - 0.10 [height (cm) - 100]
statureby takingoff another10%
Source:TanchocoCC,et al. DietManual5thEd1t1on
2010
Sam le Case:
Compute the Desired Body Weight of a 23-year-old male who is 5'3" (or 1.6 m) tall

Weight for height


• Desired body weight is 50 to 61 kg (seetable above)
(FNRI)
• DBW (kg)= Desirable BM! x (Height in m)'
Based on BMI
= 22 kg/m' x (1.6 m)' = 56.32 or 56 kg

Modified Tannhauser's • DBW (kg)= [Height (cm) -100] - o.10[Height (cm) - 100]
method = (160 - 100) -0.10(160-100) = 60-6 = 54 kg
419
STEP 2: COMPUTE FOR TOTAL CALORIE REQUIREMENT (TCR) PER DAY
TCR or total energy requirement (TER) is the level of dietary energy intake predicted to
maintain energy balance in healthy, normal-weight individuals of a defined age, gender,
body size, composition, & level of physical activity consistent with good health
There are several methods to measure total energy expenditure (TEE, also known as
TCRorTER)
The two most commonly used prediction equations are the following:

A. Harris-Benedict Energy Expenditure


More complex comiiputation
0

Takes into account the basal energy expenditure (BEE), injury factor, & activity factor
0

• TER:total energyrequirementper day


• BEE:see computationbelow
TER = BEE x Injury factor x Activity Factor
• Injuryfactor:see tablebelow
• Activityfactor:see table below
Source:Tanchoco CC,et al. DietManual5thEdition2010
HarrisJA, et al. Biometricstudyof basalmetabolism in man.1919

Computing BEE (Basal Energy Expenditure) or Basal Metabolic Rate (BMR)


Amount of energy the body requires to carry out fundamental metabolic functions
0

such as circulation, breathing, thermoregulation


SEX I FORMULA FOR BEE

Males • BEE= 66.5 + (13.75x weight in kg)+ (5.003 x height in cm) - (6,775x age)

Females • BEE= 655.1+ (9.563x weight in kg)+ (1.850x height in cm) - (4.676 x age)

I . 1' t
INJURY I FACTOR
No illness/no stress 1.0 Confined to bed 1.2
Mild malnutrition, 1.1 Outofbed 1.3
post-operative
Normal, healthy, active 1.5
Mild illness, non-catabolic
• Confined to bed 1.0-1.1
• Ambulatory 1.1-1.2
Infection and stress
•Mild 1.1-1.2
• Moderate 1.3-1.4
• Severe, hypercatabolic 1.5-1.7
• Sepsis 1.8-2.0
Burns
• <20%BSA 1.2-1.4
• 20-40% BSA 1.5-1.8
• >40% BSA 1.8-2.0

420
B. Krause Method (based on physical activity)
0Simpler computation
0Takes into account the physical activity level depending on nutritional goals

• TER:totalenergyrequirement
perday
TER=
• DBW:desiredbodyweight(computed in step1)
DBW {kg) x Physical Activity (kcal/kg)
• Physicalactivity:seetablebelow
Source:TanchocoCC,et al. DietManual5th Edition201O

1.Values for Physical Activity (kcal/kg)

PHYSICAL I kcal/kg
DBW/ EXAMPLES
ACTIVITY
day I
Bed rest 27.5 • Patients confined in hospital
Sedentary 30 • Secretary, clerk, typist, administrator, cashier, bank teller
Light 35 • Teacher, nurse, student, lab technician, housewife with maids
Moderate 40 • Medical student, housewife, vendor, mechanic, driver
• Farmer, laborer, coal miner, fishermen, construction
Heavy 45
worker, heavy equipment operator

2. Computation ofTER depending on Goals


GOALS I REMARKS
• Calculate TER based on the actual weight
0 Add 500 kcal to TER to gain I lb per week
For 0 Add woo kcal to TER to gain 2 lbs per week
upbuilding
• Use an activity above the actual activity of the individual in
computing for the TER in order to gain 1-2lbs per week
• Calculate TER based on the actual weight
0 Subtract 500 kcal from TER to lose I lb per week "
For weight 0 Subtract woo kcal from TER to lose 2 lbs per week
reduction
• Use an activity below the actual activity of the individual in
computing for the TER in order to lose 1-2lbs per week

Sample Case:
Compute the Total Energy Requirement of a 23-year-old male student who is 5'3'' (or
1.6 m) tall with a weight of 53 kg. The computed DBW is 54 kg (by Modified Tannhauser's
Method in Step 1)
METHOD I CALCULATION
• TER = BEE x Injury Factor x Activity Factor
BEE (male) = 66.5 + (13.75x 53 kg)+ (5.003 x 160cm)- (6.775x 23y/o)
Harris-Benedict
BEE (male)= 66.5 + 728.75+ 800.48 -155.83
Energy
BEE (male)= 1439.91
Expenditure
Method
• TER= 1440 X!.0 x 1.5
• TER = 2,160kcal/day
• TER = DBW (kg) x Physical Activity (kcal/kg)
Krause Method
= 54 kg x 35 kcal/kg= 1,890 kcal/day
421
STEP 3: COMPUTE FOR THE DISTRIBUTION OFTER INTO CHO, CHON, AND FATS:
• This step calculates the macronutrient distribution of the total daily energy requirement
(TER) for carbohydrates, fats, and proteins
The percentage method is a simpler computation for macronutrient distribution,
wherein the TER is divided into carbohydrates, proteins, and fat as follows (for adults):
MACRONUTRIENT I % ofTER
Carbohydrate (CHO) 50-70%(60%)
Protein (CHON) 10-15%(15%)
Fat 20-25(25%)
2010
Source:TanchocoCC,et al. DietManual5th Ed1t1on

Sam le Case:
Determine the distribution of the TER of a 23 year old male student with a height of 5'3",
weight of53 kg, DBW of54 kg (by Modified Tannhauser's Method in Step 1)and TER/day
of -1900 kcal/day (by Krause method in Step 2)
METHOD I CALCULATION
• CHO= 1900 x 0.60 = II40 kcal
Percentage
• CHON= 1900 x 0.15= 285 kcal
method
• Fats = 1900 x 0.25 = 475 kcal

STEP 4: MAKE THE DIETARY PRESCRIPTION


Final step in drafting a nutritional care plan includes the breakdown of macro nutrients
(converted to grams) based on the total energy requirement (in kcal)
Convert calorie allotment for CHO, CHON, and fats to grams
• CHO (g) = CHO (kcal)/ 4
• CHON (g) = CHON (kcal)/ 4
• Fats (g) = Fats (kcal)/ 9
Write the dietary prescription expressed in whole numbers, rounded to the nearest 5
0 TER(kcal)
° CHO (g)
° CHON (g)
0 Fats (g)

Sample Case:
Determine the distribution of the TER of a 23-year old male student with a height of5'3'',
weight of 53kg, DBW of 54 kg (by Modified Tannhauser's Method in Step 1)and TER/day of
-1900 kcal/day (by Krause method in Step 2)

TER distribution in kcal: CHO n40 kcal, CHON 285 kcal, Fats 475 kcal
TER distribution in grams:

CHO = (n40 kcal)/ 4 = 285 g CHO


CHON = (285 kcal)/ 4 = 71 g CHON
Fats = (475 kcal)/ 9 = 53 g Fats

Dietary Prescription: Full diet, 1900 kcal, CHO (285 g), CHON (70 g), Fats (55 g)

422
ENTERAL NUTRITION
I. TYPES OF TUBES
TYPE I REMARKS
• Nasogastric tube (NGT) feeding is ideal for short-term enteral
access (4-6 weeks)
Nasal feeding
• Fine-bore NGT may be used when long-term nasal feeding is
needed, especially when gastrostomy options are not suitable
• Preferred access device when long-term enteral nutrition is
needed
Percutaneous
• Preferred over surgical gastrostomy (lower complication rate,
endoscopic
cost, operating time)
gastrostomy (PEG)
• Less intervention failure (e.g., feeding interruption, blockage
or jejunostomy (PEJ)
or leakage of the tube), better adherence to treatment,
improvement in nutritional status, and better quality of life

II. DELIVERING NUTRITION


• Bolus, intermittent continuous, or continuous infusion through a pump may be used
depending on clinical need, safety and level of precision required
INFUSION I REMARKS
• Division of total feed volume into 4-6 feeds throughout the day
• Volume is typically between 200-400 mL of feed administered
Bolus infusion
over a 15-60 minute period, depending on the patient's needs
and tolerance
• Uses enteral feeding pumps (allows patients to get

I
uninterrupted sleep without the need to adjust flow rates)
Continuous • Safely allows infusion of small volume of solutions for variable
n
infusion periods
• Ideal in jejuna! feeding as the jejunum relies on controlled
delivery of isotonic substrates

III. CARING FOR TUBES


A. Water Flushing
° Feeding tubes are prone to blockages due to the chemistry of the protein-rich
solutions, the viscosity of the fluid and the small diameter of the tube lumen
(especially if medications are given through the tube)
° Flush at least 30 mL of water before and after feeding (in bolus feeding) or every 4
hours (in continuous feeding) to prevent tube obstruction

B. Drug Administration
° Crushing medicines should be avoided whenever possible because of the potential
risks of exposure to the drug and inaccuracies of drug dosing
0 Necessity & appropriateness for a drug to be administered through an enteral tube should

be confirmed (effect of the site of drug delivery, drug interactions with enteral formula)
0 The site of an enteral tube tip is an important factor when establishing drug efficacy
(e.g., administration into the stomach versus into the duodenum or jejunum)

423
IV. STANDARD COMMERCIAL FORMULAS
Commercial formula tube feed can be used unless there is justification for blended tube feed
Standard polymeric (i.e., macronutrient components are intact and not predigested)
formulations are lactose-free and gluten-free
Fiber-containing feeds may be used for patients with diarrhea or constipation

AS . "fi T b F d
TYPE I DESCRIPTION
• Contains protein in the form of free amino acids and is nearly fat-free
Elemental
• Affords more efficient protein absorption
formula
• Designed for patients with limited digestive capacity
• Supplemented with arginine (needed for cell growth & proliferation, wound
Immune healing, nitric oxide production, & lymphocyte differentiation), glutamine,
modulating omega-3 fatty acids (decrease systemic inflammation), antioxidants
formula • Reduces infectious complications, antibiotic needs, duration of
mechanical ventilation, multiple organ dysfunction, and hospital stay
Organ- or • Renal formulas: low potassium and phosphorus
disease- • Hepatic formulas: low sodium
specific • Diabetic formulas: lower sugar content containing slowly digestible
formula carbohydrates & a fat content enriched in unsaturated fatty acids (MUFA)

B. Common Commercial Formula

AlitraQ 76 glsachet 302 49.3 15.8 4.6 300 360 220

Beneprotein 1 (7 g) 25 0 6 0 15 30

Boost Optimum 7 (55 g) 250 29.3 9.9 9.6 115.5 376.8 146.9

Diabetasol 4(609) 260 39 10 7 110 240 200

EnPlus 6 (54 g) 230 33.8 8.6 7.7 200 275 227

Ensure Gold HMB 6 (60.6g) 262 34.19 10.5 8.48 194 406 158

Glucerna Triple Care 5 (52.1g) 228 26.11 10.16 8.7 211 370 168

Glucobest 4 (52 g) 229 29.6 10 8.5 208 343 138


Neo-mune 7 (60 g) 250 31.26 15.66 7.26 200 262 149

NeproLP 237 ml/can 425 46.4 10.6 22.7 190 270 170
NeproHP 237 ml/can 427 37.9 19.1 22.7 250 250 170
Novasource Renal 237 ml/pack 475 43.8 21.6 23.7 223 194 166
Nutren Diabetes 7 (55 g) 253 24.5 11.3 11 237 240 138

Nutren Fibre 8 (58 g) 251 31.3 10 9.5 241 320 116

Nutribest 4 (57 g) 258 35.4 9.4 9.4 171 371 143

Oral Impact 74 glsachet 303 40.2 16.8 8.3 320 402 216

Oxepa 500 ml/bottle 759 53 31.25 46.9 655 980 500


Peptamen 7 (55 g) 251 31.5 10.2 9.9 217 286 179
Peptamen AF 500 ml/bottle 761 70 47 32.5 650 1350 300

Prosure 9 (75 g) 302 43.24 15.97 6.14 288 432 160


Source: MIMSPhilippines,2021
424
PARENTERAL NUTRITION (PN)
Delivery of nutrients directly into the venous system, categorized as:
0 Total (or exclusive) PN: nutritional need is provided in entirety
0 Partial (or complementary) PN: nutrition is also provided via the oral/enteral route
PN generally consists of 1000 to 1200 kcal/Land 30 to 50 g/L of protein
Delivered in a solution comprising macronutrients (CHO, CHON, lipids), micronutrients
(vitamins, minerals, trace elements), water, electrolytes, and medications
° CHO is delivered as dextrose
° CHON is delivered as amino acids
0 Lipids are delivered as JV fat emulsions (IVFEs)

I. ADMINISTRATION
Ideally infused via large central vein (e.g., subclavian, internal jugular veins)
Peripheral venous access may be considered for low osmolarity (<850 mOsmol/L) mixtures
Delivery of PN is usually over 12 to 24 hours

II. METABOLIC COMPLICATIONS


COMPLICATION I REMARKS
Hyperglycemia • Directly related to PN dextrose content, patient's insulin
(most common) sensitivity, and rate of PN infusion
• Results from sudden provision of a large amount of glucose
calories to a patient who was previously malnourished
• Insulin production in increased, pushing potassium, phosphorus,
Refeeding syndrome magnesium, & thiamine into the intracellular space resulting in
hypokalemia, hypophosphatemia, & hypomagnesemia
• In patients at high risk for refeeding syndrome, feeding should
be increased slowly over 3-4 days while monitoring electrolytes
Abnormal liver
enzymes & associated • Steatosis, cholestasis, and gallbladder sludge/stones
hepatobiliary disease

• Contains branched-chain
Aminoleban • 500ml (6.11g nitrogen)
Infusion amino acids for hepatic
encephalopathy
• For mild hypoproteinemia or • Double-chamber
bag:500ml (210kCal),1000ml
BFluid (420kCal)
malnutrition
• For pre· & post-op
Moriavit hypoproteinemia/ • 20 ml (-10 g aminoacids)
malnutrition
Celemin • For acute & chronic renal • 500ml (70g aminoacids/1000ml, 10.8g
Nephro insufficiency, hemofiltration, nitrogen/1000
ml)
Nephrosteril peritoneal & hemodialysis • 7% infusion
in250ml, 500ml
• Central:493ml (550kCal),986ml (1100kCal),
Smof- 1477ml (1600kCal},1970ml (2200kCal)
Kabiven • Peripheral:1206ml (800kCal},1448ml (1000
kCal),1904ml (1300kCal}
• Three-chamber bag system
Kabiven • Peripheral:
1440ml (1000kCal),1920ml (1400kCal)
(glucose, amino acids, lipid
emulsion) • Central:1540ml (1400kCal),2053ml (1900kCal)
Combiflex • PeripheraJ:
1440ml (900kCal),1920ml (1400kCal)
MGTNA- • 360ml (250kCal),480 ml (350kCal),960ml (700
PERI kCal),1440ml (1000kCal},1920ml (1400kCal)
425
REFERENCES
1. AASLD-IDSAHepatitis C Guidance Panel Hepatitis C Guidance 2019Update: American Associationfor the Study ofLi,..erDiseases lnfectiou.sDiseases Society
of America Rt.'COmmendations for Testing,Managing,and Treating HepatitisC Virus Infection.Hepatology.2020;71{2);688-721.
2. Aithai GP, P::ilaniy:ippan N, China L, Harmala S, Macken L. R)'an Ji\-1. WUkcs EA, Moore K. Leithead JA,Hayes PC, Obrien AJ, Venna S. Guidelines on the
management of ascites in cirrhosis. Gut 2021:70:9-29.
J- Banks P.Bollen TI.. Den-enis C. Gooszen HG, Johnson CD, Sarr MG, et al Classification of acuie pancreatitis - 2012: revision of the Atlanta classificationand
definitions by in1emational consensus. GuL 2013:62:102-111.
4- Bickk-yLS, SzilagyiPG,and Bates BB.Bates'Guide to PhysicalExamination and HismryTaking. 11thedition. Philadelphia: Lippincott Williams & Wilkins;201)-
5- Biggins SW, Angeli P. Garcia-Tsao G. Gines P, Ling S. Nadim MK. Wong F, Kim WR Diagnosis, Evaluation, and Management of AsciteS and Hepa1orenal
Syndrome. Hepamlogy.2021May 3,
6. BleiAT and Cordoba J.Hepatic Encephalopathy.Am J Gastroenterol. 2001;96(7):1968-;6.
7. Bischoff SC, Austin P, Boeykens K, Chourdakis M, Cuerda C, Jonkers-Schuitema C. Lichota M, Nyulasi I, Schenider SM, Stanga Z, Pironi L ESPEN
guidline on home enteral nutrition 2020;39:5-22
8. Bi.5choffSC.Bemal W,OasarathyS,MerliM, PlankLD,SchutzT,PlauthM. ESPEN practicalguidlinc:Clinicalnutrition in livercfueasc.ClinicalNutrition2020;J9:3533-3_562
9. Boxhoom L,Voem1:msRP, Bouwcnse SAW, Bruno M. Acute P.:mcreatitis.The Lancet 202.0Sep.396(10252).
JO. Cederholm T. Baraz.zoni R. Austin P, Ballmer P, Biolo G, Bischoff SC, Compher C, Correia I, Higashiguchi T, Holst M, et al. ESPEN guidelines on
definitions and 1enninology of clinical nutrition. Clinical Nuirition 2017,36=49·64
11.Chalasani N,YounossiZ, La\ine )E, Charlton M, Cusi J,Rinella M, et al. The Diagnosis and Managemenl of Nonalcoholic Faity LiverDisease: PracticeGuidance
From the American Association for the Study of Liver Diseases.Hepatology 2018;6j(1):32S-3,7.
12.ChanFK.Primer.Man.,ging:NSAID-induced UkttComplications - BalancingGastrointestinaland Carriim~ Risks.Nat ClinPractGastrocnterolHepacol2006
13,ChcyWO, Leontiadis GI, Howden CW,and Moss SF.ACG clinicalguideline: treaunenr of Helirobacter pylori infection.Arn J Gasnoenterol 2017:112.:212-2}8.
1+ EASL2017Clinical PracticeGuidelines on the management ofhcpatitis B virus infection. Journal ofHepa1olog}:2017:6n70-3g8.
l),EASL EASL-ALEHClinical PractiL"C Guidelines: Non-in..-asivetests forc-\'aluationofli..-erdisea..eSC\"erity
and prognosis.Journal of Hepatology.2015;6):237-264-
16. Feldman M, Friedman L,and Brandt L (editors).Sleisenger and FordrranSGastrointestinal and LiverDisease 11thEdi1ion.Philadelphia: Saunders; 2021.
17.Ferenci Pet al. Hepatic cncephalopath}'-definition. nomenclature, diagnosis. and quantification: final report of the working party at the 11thWorld Congresses
ofGasuoemerology, Vienna, 199,.~. Hepatology 2002;35-716-21.
18. Fiaccadori E, Sabatino A, Barazzoni R. Can-cro JJ,Cupisti A, De Waele E, Jonckhcer J, Singer P. Cuerda C. ESPEN guideline on clinical nutrition in
hospitalized patients with acut~ or chronic kidney disease. Clinical Nutrition 2021.
19. FNRI-DOST, PDRI 2015Philippine Die1ary Reference Intakes Summary of Recommendations, 2015edition
FNRI-OOST, RENI - Rccomm~nded Energy and Nutrient Intakes 2002 edition
21.ForrestJA, Finlayson ND, and Shcamian DJ. Endoscopy in gastrointestinal bleeding. Lancet 1974;2(78n>:3~-7.
22. FostcrC The Washington Manual of Medical Therapeutics. Philadelphia:Wolters Kluwcr/LlppincoflWilliams & Wilkins;2010.
23- Garcia-TsaoG,Abraldes JG,BerzigoniA. BoschJ.Portalhypertensr.-ebleeding in cirrhosis:risk stratification,diagnosis,and management: 2016practiceguidance
by the American Association for the Study ofU.-cr Diseases. Hcpa1ology.2017:65(1):301-335-
24, Gisbert JPand Calvet X. Non-bismuth Quadruple (Concomitant) Therapy for Eradication of Helicobacter Pylori.Aliment Pharma col Thcr. 2011,34(6):6<:4-617.
25- Gralnek IM. Dumonceau JM, Kuipers EJ,Lanas A, Sanders OS, Kurien M, ct al. Diagnosis and management of nonvariceal uppergasrrointestinal hemorrhage:
European Socicl)·of Gastrointestinal (ESGE)Guideline. Endoscopy.2015;47:ai-.46.
26. Gralnek IM, Stanley AJ, Morris AJ, Camus M, Lau J, Llanas A. Laursen SB. Radaelli F. Papanikolaou JS, Goncalbes TC, et al. Endoscopic diagnosis and
management of nonvariceal upper gastrointestinal hemorrhage (f',,'Vl.JGJH): European SocietyofGastrointestinal Endoscopy (ESGE)Guideline - Update 2021.
Endoscopy 2021;)}: 300-332.
Gomes F, Schuetz P, Bounoure L, Austin P, Ballesteros-Pomar M, Cederholm T. FlecchcrJ, Laviano A, Nonnal K. Poulia Ket al. ESPEN guidelines on
nutritional support for polymobid intemaJ medicine patients. Clinical Nutrition 2018:3;:336-353
28. Harris JA, lknedic1 FG. Biometric StudyofBasal Meiabolism in Man. Washington. DC:Camegie Institution, 1919
29. JamesonJL KasperDL.LongoDL.FauciAS,Hauser SL.LoscalzoJ.Harrison'sPrinciplesoflntemalMedicine.20th E.dition.NewYorlcMcGrawHill Education,2018.
JO, Karstensen JG, Ebigbo A, Bhat P,Dinis-RibeiroM, Gralnek I, GuyC, Le Moine 0, Vilmann P,Antonelli G, ljoma U et al Endoscopic treatment ofvariceaJ upper
gastroimestinal bleeding: European Sociecyof Gastrointestinal Endoscopy(ESGE)Cascade Guideline. Endoscopy International Open 2020:o8: E990-E997.
31. K.1t1. PO, Dunbar KB,Schnoll-Sussman FH, Greer KB,YadlapatiR.Spcchler SJ.ACG Clinical guideline for the diagnosis and management of gastroesophageal
renux disease.Am JGastrocnterol 2021;00:1-30
J2. Kornbluth A and SacharD.ACGpracticeguidelines:ulceraili-ecolitispracticeguidelines in adults. Am J Gastroenterol 2010:105(3):5()1-23,
)J. Laine L,Jensen D. Management of patients \\ith ulcer bleeding. ACG PracticeGuide.lines.Am JGastrocnterol 2012;10,:345-)60.
J4. LichtensteinG, HanauerS,3nd Sandborn \V.ACGpracti<."cguidelines management of crohnsdiscasc in adults.Am J Gastroentcrol2009
35. Mahan LK.Stump SE. Krause's Food, Nutrition and Diet Therapy 111hedition, 2004
36. McCla\'e SA. DiBaise JK,Mullin GE, Martindale RG. ACG Clinical Guideline: Nutrition Therapy in the Aduh Hospitalized Patient The American
Journal ofGas1rocnterology 2016;111:315-333
37. MIMS Philippines. 2021.Jn MIMS Online. hrtps://www.mims.com/philippines
38. Mtawch H, Tuira L. Floh A, Parshuram C. Jndirect C.1.lorimeu1~History, Technology, and Application. Front Pediatr 2018:6:257.
39. Nuttall FQ. Body Mass Index: Obcsiry, BMI, and Health: A Critical Review. NutrToday. 2015Ma>~So{J):117•128
40- 0Connor A.Moliro·lnfunteJ,GisbcnJP,andOMor.un C Trmtn'ICJltofHelio::mcterP)ion infoaio112C113,
Hclicoooaer.Scp201~8Suppl L-s&6;.
41. o·shca RS,0as.'lr.lthyS, and McCullough AJ.AASLO prncticeguidelines Alcoholic liverdisease. Hcpatology.2010:51(1);307-p8 .
.µ. Po-.,~11 OW,YamadaT (Ed).Te.xtbookof gastroentero!ogyand hepatology,4th Ed Philadelphia, Llppinco1Williams & WBkins;2003,
43- Pn..._isaniN and L.a..-anchy 0. World Health Organization. Hepatitis B {WHO/COS/CSR/LY0!2002.2). 2002.
44 Pugh RN,Murray-LyonIM. Dawson Jl. Pienoni MC. and Williams R.Transection of the oesophagus for bleeding oesophageal ,.uices. Br I Surg. 1973;60:646-
45- Runyon BA. The American Association for the Siudy of Liver Diseases.Practice Guideline: Man.,gemcnt of adult patients \vith ascites due to cirrhosis: update
2012.Availablefrom:ww\\:.aas[d.org/sites/defaulllfiles/guideline_doruments/adultascitescnhanced.pdf
,¢. ShahS,l)erPG,MossS.AGAClinica!PractireUpd3IeontheManagcmentofRclractoryHclicobacrerPyloriWcction:E.xpcrtRcvie-...:Gasuocnter0logy2021;16ai83t•1B41.
47. Singer P. Blaser AR. Berger MM. Alhazzani W, Calder PC, Casaer MP. Hiesmayr M, Mayer K. Montejo JC, Casaer MP,cl al. ES PEN guideline on clinical
nutrition in che in1esr.·ecare unit Clinical Nutri1ion 2019;38:48-79
48. SingaJAK.BatallerR.AhnJ, Karnath PS.Shah VH.ACG ClinicalGuideline:AlcoholicLivcrDisease.Am JGasu-ocnterol2018;113=175-194
49- TennerS. BaillieJ,DeWinJ,andVegcSS.AmericanCcllegeofGasnoentero\ogyguidclinemanagementofaa.nepancreatitis.AmJGastroeruerol2013.1o8{9)'.f4.oo-15;1416.
50. Terrault NA.BZO\\-ej NH, Chang KM, H\\ang JP,Jonas MM, and Murad MH. AASLDguidelines for tre-atmentof chronic hepatitis B.Hepatolog>:2016;63(1):261
51. Tanchoco CC, Jamorabo-RuizA. Diec Manual 51hEdition. The Nutritionis1-Dietitian's Assocation of the Philippines(NDAP) 2010
52. Terra.ultNA.l.okASF, McMahon BJ.Chang K. Hwang JP.Jonas MM, Brown RS, BzowcjNH, Wong JB. Update on Pre.""encion, Diagnosis, and Treatment of
Chronic Hepatitis B:AASLD2018Hepatitis B Guidance. Hcpatology 2018;6j(4):156o-1599.
53- ½.kil N. Revie,,,•article: how valuable are PPIs in establishing a diagnosis ofGERD. Alimem Pharmacol Thcr 2005;22(Suppl1):64-9.
5+ Vil:.1:IUp H, 1\modio P. BaiaiJ, Cordoba J.Ferenci P. Mullen KD. Weksenbom K. \Vong P. Hepatic l:n«phalopathy in Chronic 11-'=rDtS<.!aSC: 204 Practice
Guideline by1heAmerican Associationfor the Study of LiverDi~ases and the European Association for the Study of the LJ.,.er.Hepatology 204;6o(1)7t5-735-
5), Wald A. Bharucha AE. Cosman BC,and Whitehead WE ACG clinicalguideline: management of benign anorectal disorders. Arn J Gastroenterol. 2014;
56. Wang KK, Prasad G, and Tian J. Endoscopic mucosal resection and endoscopic submucosal dissection in esophageal and gastric cancers. Curr Opin
Gastroenterol. 2010September: 26<5): 453-4;8.
)7. \Vasil'}·
A. Gl)1dal S, and Gallagher K.Surveillancefor acute \iral hepatitis-United States,2006.MMWRSur,-eillSumm. 2ooS;,7(2}'.1-24
58,. Working Group IAP/APAAcute PancreatitisGuidelines. Pancre.1tology132013=e1-c15-
59. Wu BU,Johannes RS.Sun X.Tabak Y,Conwell DL,and BanksPA.The early pn."<.liction of mortality in acute pancreatitis:a large population-based study. Gut
2ooS Dec;;J(12):1698-703

426
NEPHROL
QJAPPROACH TO DISEASES IN NEPHROLOGY
1. Approach to Complaints in Nephrology
2. Common Formulas in Nephrology
3. Clues for Diagnosis of Major Syndromes

0 ACUTE KIDNEY INJURY & CHRONIC KIDNEY DISEASE


1. Acute Kidney Injury
2. Chronic Kidney Disease
3. Renal Replacement Therapy

0 OTHER DISORDERS IN NEPHROLOGY


1. Glomerular Diseases
2. Nephrolithiasis
3. Renal Tubular Defects
4. Urinary Tract Obstruction

0 OVERVIEW OF EXTRACORPOREAL THERAPY


SECTION ONE
APPROACH TO DISEASES IN NEPHROLOGY

APPROACH TO COMPLAINTS IN NEPHROLOGY


I. PERTINENT MANIFESTATIONS OF KIDNEY DISEASE
ASPECT I MANIFESTATIONS
• Disturbances in urine volume
• Manifestations of decreased glomerular filtration rate or GFR (e.g.,
Alterations in hypervolemia) and/or electrolyte abnormalities
renal & urinary • Hypertension and/or expanded total body volume
tract functions • Fever or pain (e.g., urinary tract infection or UT!, inflammation)
• Abnormalities of urine sediment
• Abnormal excretion of serum proteins
• Skin: pruritus, soft-tissue calcification, uremic frost, edema
• Pulmonary: pulmonary edema, pleural effusion, pleuritis
• CVS: hypertension, accelerated atherosclerosis, cardiomyopathy,
pericarditis
Manifestations
• GIT: metallic or NH3 taste, bleeding
of chronic renal
• Skeletal (renal osteodystrophy): osteomalacia, osteosclerosis, osteitis
failure
fibrosa (PTH related)
• Neurologic: encephalopathy (e.g., fatigue, drowsiness, insomnia,
cognitive dysfunction, asterixis, hiccups, disorientation, coma),
cramps, neuropathy, autonomic dysfunction
Uremic • Nausea, anorexia, vomiting, weight loss
symptoms • Weakness, easy fatigability, lethargy, drowsiness

II. PERTINENT DATA FROM THE MEDICAL HISTORY


ASPECT I PERTINENT DATA
• High blood urea nitrogen (BUN) or creatinine in the past
• History of albuminuria or foamy urine, hematuria or dark urine,
edema, or UT!
Previous • Previous medical exams (clinic, private MD, insurance, employment)
evidence of
renal disease • History of hypertension
• Lower urinary tract symptoms: urgency, burning, frequency, hesitancy
• Oliguria, polyuria, nocturia
• History of infections (throat, skin)
• Hypertension, diabetes
History of • Autoimmune: SLE, scleroderma, PAN, Sjogren's, GPA, HSP
systemic
diseases • Malignancy: multiple myeloma, breast, lung, colon, lymphoma
• Others: sickle cell disease, primary or secondary amyloidosis
• NSAID, penicillin, aminoglycosides, chemotherapeutic drugs,
History of drug narcotic abuse
exposure • Recent intake of any new drug temporarily associated with renal
failure (e.g., ACE-inhibitors)
• Heart failure
• Diuretic use
History • Nausea with vomiting, diarrhea, high fever, GI bleeding
of factors
• Salt-restricted diet
contributing to
azotemia • Liver cirrhosis
• Lower urinary tract symptoms
• Pelvic disease
429
III. DEFINITION OF COMMON MANIFESTATIONS IN NEPHROLOGY
TERM I DEFINITION
• Normal GFR in young adults: -125 mL/min/1.73 m'
Normal Values • Normal urine volume: -1000-2000 mL/24 hours
(for reference) • Protein excretion in normal individuals: total protein <150mg/z4h
and albumin <30 mg/24h
Azotemia • Reduction in GFR
• General term for the presence of increased amounts of protein in the
urine, with protein excretion rate (PER) >150mg/24h
• Proteins can be in the form of albumin (glomerular proteinuria), low
Proteinuria molecular weight proteins (overflow proteinuria), and Bence-Jones
proteins (tubular proteinuria)
• Nephrotic range proteinuria: PER of >3500 mg/z4h or protein-to-
creatinine ratio (PCR) >3500 mg/g
• Abnormal loss of albumin in the urine (e.g., glomerular proteinuria)
0Microalbuminuria: albumin excretion rate (AER) 30-300 mg/z4h or
albumin-to-creatinine ratio (ACR) 30-300 mg/g
Albuminuria
0Macroalbuminuria: AER >300 mg/z4h or ACR >300 mg/g
0Nephrotic range: usually with AER >2200 mg/24h or
ACR >2220 mg/g (or 220 mg/mmol)
• Clinical syndrome associated with fluid, electrolyte, and hormone
Uremia imbalances and metabolic abnormalities, which develop in parallel
with deterioration of renal function
Polyuria • 24 hour-urine output >3000 mL
Oliguria • 24 hour-urine output <400 mL
Anuria • Complete absence of urine formation (<50-100 mL in 24 hours)
• Pain that occurs during urination; perceived as burning or stinging
Dysuria
sensation in the urethra
Hematuria • Presence of 2-5 RBCs per high power field
• Waking at night to void, where each micturition is preceded and
Nocturia
followed by sleep, clinically meaningful if2:2 per night
Lithuresis • Passage of small calculi in the urine
Source:JamesonJL, et al. Harrison's
Principles
of InternalMedicine20thedition,2018
KDIGOCKDWorkGroup.KDIGO2012CPGfortheEvaluation andManagement ofCKD.Kidneyinter;.2013

430
IV. FORMS OF INJURY IN RENAL AND URINARY TRACT DISEASE
FORM OF
INJURY I POSSIBLE CAUSE
I RESULT OF INJURY

• Casts, glomerular proteinuria, leading to


• Urinary obstruction foamy urine & edema
• Reflux • Decreased GFR
• Hereditary immune • Tubule injury, leading to tubular
Nephron disease proteinuria
injury/loss • Vascular disease • Anemia
• Neoplasm • Small kidneys
• Infections • ! NH3 production
• Toxins • Metabolic acidosis
• Osteomalacia and osteitis fibrosa
• Nephron injury
• Volume overload, hypertension, edema
• Hypertension, CHF,
Reduction of • Hyperuricemia
edematous states
GFR • Az9temia, oliguria, uremia
• Urinary tract
• Hyperkalemia, metabolic acidosis
obstruction
• Infection • Acute obstruction leads to decreased
Urinary tract • Stones, neoplasia GFR
obstruction • Surgery • Chronic obstruction leads to nephron
• Inherited injury
Urothelial • Urothelial erosion or
• Leukocyturia and hematuria
damage inflammation

PHYSICAL EXAMINATION IN NEPHROLOGY

ASPECT I EXAMINATION FINDINGS


Volume • Volume depleted: orthostatic hypotension or tachycardia, skin tenting
status • Fluid overload: l JVP, rales, ascites, edema
• Skin: malar rash, purpura, macular rash, scleroderma
• HEENT: alopecia, uveitis, papilledema, icterus, funduscopic exam,
Evidence throat infections, hearing loss
suggesting • Lungs: consolidation
systemic • Heart: murmurs
illness • Abdomen: bruits, palpable organs
• Extremities: livedo reticularis, blue-toe syndrome, Janeway lesions,
Osier's nodes, angiokeratomas
• Skin: uremic frost, ecchymoses, xerosis, excoriation
• Lungs: Kussmaul respirations (metabolic acidosis)
Signs of
• Heart: pericardia! rub
uremia
• Extremities: asterixis, Trousseau or Chvostek signs (calcium disorders)
• Neurologic: depressed sensorium, slurred speech
Signs of
• Percussible bladder, enlarged prostate, phimosis
obstruction

431
COMMON FORMULAS IN NEPHROLOGY
I. ESTIMATION OF GLOMERULAR FILTRATION RATE (GFR)
Best overall index of kidney function, but is difficult to measure (necessary to estimate
GFR rather than relying on serum creatinine)
Can use various formulas using serum creatinine (SCr) to predict estimated GFR (eGFR):
° Cockcroft-Gault creatinine clearance (CrCl)
0 24-hr urine creatinine clearance
0 Modification of Diet in Renal Disease (MORD) eGFR
° Chronic Kidney Disease Epidemiology Collaboration (CKD-EPI)
Can also be predicted utilizing serum cystatin C measurement alone (CKD-EPI cystatin
C) or using both SCr and serum cystatin C (CKD-EPI creatinine-cystatin C)
• Normal GFR in young adults: -125 mL/min/1.73 m'
TEST I REMARKS I LIMITATIONS*
• Not normalized to body surface
area (BSA)
• Less accurate in obese patients
Crea ti nine
and in those with normal or
Clearance
• Measures estimated creatinine near normal GFR
(CrCl) from
clearance (CrCl) rather than eGFR • Overestimates the true GFR
Cockcroft-
since creatinine is secreted
Gault
in the proximal tubule in
addition to being filtered in the
glomerulus
II • Tends to exceed true GFR by
2:10-20%due to presence of
24hrUrine tubular secretion of creatinine
• Measures CrCl instead of eGFR
CrCI • Relies on timed urine collection,
which may be often difficult and
impractical
• Measures eGFR using 6-variable equation
• Less accurate in obese patients,
• Normalized to body surface area
those with normal/near-normal
eGFRfrom • Based on MORD study (included only
GFR, children, elderly, pregnant
MORD Caucasian, non-diabetic patients with
women
lowGFR)
• Not validated in Asians
• Performs better in patients with low GFR
• Superior in measuring eGFR in
patients with normal or mildly-
eGFRfrom reduced GFR (>60 mL/min/I.73 m')
• Not validated in children and
CKD-EPI • Based on pooled studies of people with
pregnant patients
creatinine and without kidney disease who had a
wide range of GFR
• Validated in Filipino population
• Combination of creatinine and
cystatin-C equation is more accurate
eGFR from
than using either one alone
CKD-EPI • Steroids may affect cystatin-C
• Cystatin C-based equations may be
cystatin C levels, making it unreliable to
more accurate in populations with
&CKD-EPI use in chronic steroid users (e.g.,
lower creatinine production, such as in
creatinine- transplant patients)
children, the elderly, or cirrhotic patients
cystatin C
• Validated in various populations & less
subject to the effects of age, sex & race
*Equationsare valid only if patient is in steady state, that is, creatinine is neither rising nor fallingover
several days, before SCr levels accurately reflect kidney function
Source:Cockcroft DW,et al. Nephron;1976;LeveyAS,et al.Annalsof InternalMed1c1ne; 1999
MatsushitaK,et al.AmericanJournalof KidneyDiseases;201O;SkoreckiK,et al. Brenner& Rector'sTheKidney10e;2016
432
A. Estimated Creatinine Clearance (Cockcroft-Gault Formula)
• Formula was developed before the use of standardized creatinine assays and has not
been revised since the adoption of newer standardized assays
0 Despite its limitations, this formula is still used regularly mainly for drug research

purposes and for drug dose adjustments


CrCI:creatinineclearanceinmUmin
(140- age)x BW BW:leanbodyweightin kg
CrCl= SCr:serumcreatininein mg/dl
72 x SCr
'For females,multiplyresultby 0.85

B. Measured Creatinine Clearance (24-hour Urine Collection)


• Adequacy of urine collection should be assessed by computing daily creatinine excretion
• In adults <50 years old (oflean body weight), 24-hour creatinine should be:
• Men: 20-25 mg/kg (177-221umol/kg)
• Women: 15-20mg/kg (133-177umol/kg)
UCr:urinecreatininein mmol/L
UCrxUvol SCr:serumcreatininein mmol/L
CrCI= Time:collectiontimein mins(1440mins)
timexSCr
Uvol:urinevolumein ml

C. Normalized Creatinine Clearance


• Since both estimated and measured CrCl are not normalized, adjusting it with the
body surface area (BSA) will allow comparison with normal values.
• Adjustment with BSA also increases accuracy ofCrCl equations particularly among
those with reduced kidney function
CrCI:creatinineclearanceusingformulasabove
(e.g.,Cockcroft-Gault,
24 hoururine)
CrClx1.73
Normalized CrCl = BSA:bodysurfacearea in m'
BSA Normalized CrCI:CrCIcorrectedaccordingto BSA
(unitis mUmin/1.73 m')

II. RENAL FAILURE INDICES


A. BUN-Creatinine Ratio (BCR)
• BUN:serumBUNin mmol/L
• Crea:serumcreatininein umol/L
BCR = BUN x 247 Interpretation:
Crea BCR>20:1is seen in pre-renalazotemia
BCR10-15:1is seen in oliguricrenalfailure

B. Fractional Excretion of Sodium (FENa)


• Fraction of the filtered sodium load that is reabsorbed by the tubules
• Measure of kidney's ability to reabsorb sodium and endogenously & exogenously
administered factors that affect tubular reabsorption
0 Depends on sodium intake, effective intravascular volume, GFR, diuretic intake, and

tubular reabsorptive mechanisms


UNa:urinesodiumin mmol/L
SNa:serumsodiumin mmol/L
SCr:serumcreatinineinmmol/L
UCr:urinecreatininein mmol/L
UNaxSCr
FE Na(%)= ------ x mo Interpretation:
SNa x UCr FENa<1%:seen in pre-renalazotemia(intravascular
volumedepletion)
• FENa>2%:seen inoliguricacute renalfailure&
I!
ischemicAKI
433
C. Fractional Excretion of Urea
0 Unlike FENa, FEUrea can be used in patients who are on diuretics
° Convert blood urea nitrogen (BUN) & urine urea nitrogen (UUN) first to "serum Urea" &
"urine Urea", respectively, as BUN & UUN reflect only the nitrogen component of urea

UUr:urineurea in mmolll
UUrxSCr SUr:serumurea in mmolll
FE Urea(%)= XIOO
SUrxUCr UCr:urinecreatininein mmolll
SCr:serumcreatininein mmolll
SerumUrea(mmol/L)= BUN(mgldl)x 0.3571 Interpretation:
= UUN(mgldl)x 0.3571
UrineUrea(mmol/L) ° FEUrea<35%:suggestspre-renalazotemia
regardlessof diureticuse

D. Urine Albumin-to-Creatinine Ratio (Urine ACR)


An alternative to 24-hour urine collection, used to quantify albuminuria
0

° Can use random urine sample, preferably the first or second morning void after
avoiding exercise
ACR:albuminto creatinineratioin mglg
UAlb:urinealbuminin mgldl
' UCr:urinecreatininein mgldl
: UrineACR=
UA!b
XIOOO Interpretation:
ii UCr <30mglg:normal
II 30 to 300 mglg:microalbuminuria
1, >300mglg:macroalbuminuria

S ampeI Case
A 48-year-old male was admitted due to difficulty of breathing. He started having
productive cough with greenish sputum three days ago, associated with fever. He
was recently diagnosed to have hypertension and is maintained on losartan and
hydrochlorothiazide. He was seen at the ER in respiratory distress, with BP of 90160
mmHg, HR 104 bpm, RR 32 cpm, Temp 39.2°C, weight of78 kg and height of 163cm. He
has pink palpebral conjunctivae and flat neck veins, with coarse crackles over the upper
and middle lung fields. He has good S1 and S2, is tachycardic with regular rhythm, has
soft non-tender abdomen and no peripheral edema with dry extremities. He was noted
to have poor urine output since the day before.

His initial laboratory tests showed the following:


CBC Hgb 126, Hct 0.41, WBC26.5, Neut 95%, Lymph5%, Pit 324
Serum Crea 185.6 umol/L(2.1 mg/dl)
Serum BUN48 mg/dl
Na• 132 mmolll
Random UrineCrea 25,636 umol/L(290 mg/dl)
UrineNa• 216 mmol/L
Random Urine Urine Urea 168 mmol/L
Urinealbumin25 mg/dl
UrineCrea 1585 mg/24h
24-hour Urine Volume500 ml
Collection Protein 81.4 mg/24h

434
1. What is the estimated creatinine clearance of the patient?

Using the Cockcroft-Gault Formula:


CrCl = [(140 - 48) x 78] / (72 x 2.1)
= 47.5 mL/min

Using 24-hour urine collection:

First check for adequacy of collection: 1585mg Creatinine / 78 kg= 20.3 mg/kg, hence
the collection is adequate (adequate collection in males is 20-25 mg/kg in 24 hours)

Urine Crea = Convert 1585mg in 500 mL to umol/L


= 1585mg/500 mL = 3.17mg/mL 317mg/dL x 88.4 = 28,022 umol/L

CrCl = (UCr x Uvol) / (time x SCr)


= (28,022 umol/L x 500 mL) / (1440 minx 185.6umol/L)
= 52.4 mL/min

If normalized to BSA (1.84 m'):

CrCl = 52.42 X I.73 / 1.84


= 49.3 mL/min/1.73 m'

2. Using renal failure indices, what type of acute kidney injury does the patient have?

BCR = (17.1mmol/L* / 185.6 umol/L) X 247


= 22.75, indicative of pre-renal azotemia

FENa = [(216mmol/L x 185.6umol/L) / (132mmol/L x 28,022 umol/L)] x 100%


= 1.08 %, inconclusive and may be erroneous since the patient has
been taking hydrochlorothiazide causing high urine Na

FEUrea = [(168mmol/L x 185.6umol/L) / (17.1mmol/L* x 28,022 umol/L)] x 100%


= 6.5 %, indicative of pre-renal azotemia

*Note: BUN of 48 mg/dL = serum urea 17.1mmol/L (computed as 48 mg/dL x 0.3571)

The patient may be having pre-renal azotemia from renal hypoperfusion in early phase
of sepsis and volume depletion from diuretic use (hydrochlorothiazide).

3. Does the patient have albuminuria?


Urine ACR = (25 mg/dL) / (290 mg/dL)
= 0.086 mg/mg, multiply by 1000 to convert mg of creatinine tog
= 86 mg/g

Hence, patient has microalbuminuria (between 30 to 300 mg/g)

435
CLINICAL CLUES FOR DIAGNOSIS OF MAJOR NEPHROLOGIC SYNDROMES

SYNDROME

• Documented recent (rapid)


Acute Kidney Injury • Oligo-anuria
decline in GFR
• Azotemia >3 months
• Manifestations of uremia • Kidneys reduced in size
Chronic Kidney
• Hypertension, edema • Broad (WBC) casts in urinary
Disease
• Renal osteodystrophy sediment due to dilated
tubules
Nephritic Syndrome • Oliguria, edema, • Hematuria, RBC casts,
(Acute Nephritis) hypertension azotemia
• Massive proteinuria >3-5g/day
Nephrotic Syndrome • Edema (or anasarca) • Hypoalbuminemia,
hyperlipidemia
• Frequency, urgency, • Bacteriuria >Io' CFU/mL
Urinary Tract hematuria • Infectious agent documented
Infection • Fever in urine
• Bladder/flank tenderness • Pyuria, leukocyte casts
• Stone may be seen on x-ray or
other imaging
• Previous history of stone • Other findings:
Nephrolithiasis passage hydronephrosis, dilated
• Renal colic calyces
• Abnormalities in urinalysis
common
'
• Electrolyte disorders (from
renal transport defects)
Tubular Syndromes • Polyuria, nocturia
• Renal calcification or large
kidneys on imaging
• Oligo-anuria or polyuria, • Azotemia
Urinary Tract nocturia, urinary retention • Hematuria, pyuria
Obstruction • Weak urine stream, bladder • Imaging: large prostate, large
fullness kidneys
• Subnephrotic range
proteinuria (i.e., proteinuria
Asymptomatic Urine below nephrotic range)
Abnormalities • No symptoms
• Hematuria, sterile pyuria,
casts (RBC, WBC, tubular
cells)
• With or without proteinuria,
Hypertension from • Systolic and/or diastolic
hematuria, abnormalities on
Renal Disease hypertension
urinalysis
Source:
Jameson
JL,et al. Harrison's
Principles
of InternalMedicine
20thedition,2018

436
Simplified Approach to Azotemia
Azotemia of unknown duration
increase in SCr or eGFR < 60

KUBultrasound

No

Urinary tract
kidney size? obstruction

Renal artery
Asymmetric Urinalysis Normal
stenosis

Renal artery or
RBCs
vein occlusion

Crystals,
Nephrolithiasis
pyurio, RBCs

Pyurio,
bocteriuria, Pyelonephritis
RBCs

Small

Yes Chronic
glomerulanephritis

No
Chronic
nephrosclerosis

Normal

RBC, Minimal Coarse, granular casts, Pyuria,WBC casts,


RBCcosts sediments mudd brown costs eosino hils

Acute Nephrolic Acute interstitial


Acute tubular necrosis
ne hritis s ndrome ne hritis

Source:JamesonJL, et al. Harrison'sPrinciplesof InternalMedicine20thedition,2018


Schrier,RW.Manualof Nephrology, 8th Edition,2015
437
SECTION TWO
ACUTE KIDNEY INJURY & CHRONIC KIDNEY DISEASE
ACUTE KIDNEY INJURY (AKI)
I. ETIOPATHOGENESIS
• Sudden impairment of kidney function resulting in the retention of nitrogenous and
other waste products normally cleared by the kidneys
• Not a single disease but a designation for a heterogeneous group of conditions that
share common diagnostic features:
Increase in BUN
0

0 Increase in the plasma or serum creatinine (SCr) concentration


Associated with reduction in urine volume
0

A. KDIGO 2012Definition of AKI


AKI is defined as any of the following:
0

• Increase in SCr by ~0.3 mg/dL (~26.5mmol/L) within 48 hours; or


• Increase in SCr to ~1.5xthe baseline, which is known or presumed to have
occurred within the prior 7 days; or
• Urine volume <0.5 mL/kg/h for 6 hours
0Limitation: focuses on SCr and urine output only and does not take into consideration
other abnormalities such as urine sediments (e.g., hematuria, pyuria, passage of
stones) or imaging (e.g. obstruction) and acute glomerulonephritis

B KDIGO s . . fAKl(AKIN S . .
STAGE* I INCR~A~E IN SERUM C~E~TININE I URINE OUTPUT (UO)
CRITERIA
1.5-1.9times from baseline
I OR UO <0.5 mL/kg/h x 6-12 hours
~0.3 mg/dL (~26.5 mmol/L) increase••

2 2.0-2.9 times from baseline UO <0.5 mL/kg/h x ~12hours

>3.0 times from baseline


OR
~4.0 mg/dL (~354mmol/L) increase'*; UO <0.3 mL/kg/h x ~24 hours
OR (oliguria);
3 Initiation of renal replacement therapy; OR
OR Anuria ~12 hours
In patients <18years: decrease in eGFR to
<35mL/min/1.73 m'
*Onlyone criterion(creatininerise or urine output decline) needs to be fulfilled
**Increasein creatinine must occur in <48 hours
Source:KDIGOAcuteKidneyInjuryWorkGroup.KDIGOClinicalPracticeGuideline
forAKI

Example 1: A 56-year-old male has a baseline creatinine of1.o mg/dL. Upon consult (day 1),his
creatinine was 1.3mg/dL. Monitoring was done as follows: day 2: 1.5mg/dL; day 3: 2.0 mg/dL;
day 7: 1.0 mg/dL.

Diagnosis: AKI - he fulfilled criterion #I (~0.3 mg/dL rise in 48 hours) and criterion #2
(50% increase from baseline)
AKIN Stage: Stage 2 (on day 3, there was a 2x increase from baseline)
Example2: A45-year-old-malewith diarrhea has a baseline creatinine of 0.4 mg/dL. Monitoring
was done as follows: Day 1:0.5 mg/dL; day 2: o.6 mg/dL; day 3: 0.7 mg/dL; day 7: 0.5 mg/dL

Diagnosis: AKI - he fulfilled criterion #2 (50% increase from baseline), but not criterion #I.
AKIN Stage: Stage 1(1.5-1.9xincrease from baseline)
438
Estimating Baseline Serum Creatinine
0 Many patients will present with AKI without a reliable baseline SCr - if so, an

estimated SCr can be used, provided there is no evidence of CKD


° Can be estimated using the MORD Study equation, assuming that baseline eGFR is
75 mL/min/1.73m' (table below is the estimated baselineSCrfor non-blackpatients)

AGE

20-24 years 1.3 115 1.0 88


25-29 years 1.2 106 1.0 88

30-39years 1.2 106 0.9 80

40-54 years 1.1 97 0.9 80

55-65 years 1.1 97 0.8 71

>65 years 1.0 88 0.8 71


Source:BellomoR, et al.ADQIGroup.CritCare2004

C. RIFLE Criteria for AKI


0 System for diagnosis/classification of a broad range of acute impairment of renal function
0 The acronym R-I-F-L-E stands for the increasing severity classes
CLASS I GFR CRITERIA I URINE OUTPUT CRITERIA
• Serum creatinine increased x 1.5;OR • Urine output <0.5 mL/kg/h x
RISK
• GFR decreased >25% 6-12hours
• Serum creatinine increased x 2; OR • Urine output <0.5mL/kg/h x
INJURY
• GFR decreased >50% ;:,12hours
• Serum creatinine increased x 3; OR
• Urine output <0.3 mL/kg/h x
• GFR decreased >75%;OR
FAILURE ;:,24hours (oliguria); OR
• Serum creatinine ;:,4mg/dL
• Anuria x 12hours
(Acute rise ;:,0.5mg/dL)
LOSS • Persistent acute renal failure: complete Joss of kidney function >4 weeks
ESRD • Complete loss of kidney function >3 months
.. Classis basedon theworstof eitherGFRor urineoutputcriteria
GFRdecreaseis calculated fromthe increasein serumcreatinineabovebaseline

. AKIshouldbe bothabrupt(within1-7 days)andsustained(>24 hours)


Whenbaselinecreatinineis elevated,an abruptriseof at least0.5 mg/dL(44 mg/dL)is
sufficientfor RIFLEclass'Failure"
Source:BellomoR, et al.ADQIGroup.CritCare2004

439
D. Classification of the Major Forms of AKI
TYPE I PATHOPHYSIOLOGY
Pre-renal • Due to inadequate renal plasma flow and intraglomerular hydrostatic
AKI pressure (i.e., decreased renal perfusion)
(most • Involves no parenchymal damage to the kidney
common) • Prolonged hypoperfusion leads to ischemic injury (acute tubular necrosis)
Post-renal • Occurs when there is an obstruction to the passage of urine, leading to
AKI increased retrograde hydrostatic pressure & interference with GFR

• Sepsis: causes generalized vasodilation leading to endothelial damage


resulting in microvascular thrombosis, activation of ROS and leukocyte
adhesion and migration
• Ischemia: hypoxia in the renal medulla leads to impaired
autoregulation, endothelial vascular smooth muscle damage, leukocyte-
endothelial adhesion, vascular obstruction, and inflammation
Intrinsic • Further categorized according to major site of parenchymal damage:
AKI 0 Tubular damage: major causes of acute tubular necrosis are ischemic or
nephrotoxic (either exogenous compounds such as drugs & contrast agents
or endogenous toxins such as rhabdomyolysis, myeloma, or hemolysis)
0 Glomerular damage: occurs in severe cases of acute glomerulonephritis
0 Interstitial damage: can result from acute interstitial nephritis (e.g.,
allergic reaction, infections)
0 Vascular damage (e.g., vasculitis, TIP-HUS, malignant hypertension)
Source:BasileDP,et al. Pathophys1ology
of acutekidneyInJury.ComprPhys1ol;2012

II. CLINICAL MANIFESTATIONS


CATEGORY I ETIOLOGY I MANIFESTATIONS
• Evidence of volume depletion (tachycardia,
• Fluid depletion
Decreased hypotension, dry mucous membranes, low )VP)
• Heart failure
Kidney • Most diagnostic feature: azotemia is rapidly
• Impaired
Perfusion reversed once hemodynamics are restored
(Pre-Renal) autoregulation (e.g.,
• Renal indices: BCR >20 and FENa <I%
NSA!Ds, ACEi, ARBs)
• May show hyaline cases in urine sediment
• Bladder neck obstruction • May have history of kidney stones, prostate
(most common) disease, neurogenic/obstructed bladder
Obstruction
• Others: prostatic disease, catheter, retroperitoneal or pelvic neoplasm
(Post-Renal)
neurogenic bladder, • No specific findings other than AKI
ureteral calculi, strictures • Often needs imaging with ultrasound or CT
• Systemic hypotension, often superimposed
on sepsis and those with limjted renal
• Ischemia-associated reserve such as old age and CKD
AKI • Granular casts & renal tubular epithelial
cell casts on urinalysis
• FENa>I%
Non-Specific • Overt hypotension not always seen in mild
AKI to moderate AKI
• Sepsis is one of the most common
• Sepsis-associated AKI contributing factors in AKI
(SA-AKI) • Granular & renal tubular epithelial cell
casts seen on urinalysis
• FENa usually>!% (but it may be low early
in the course)
Source:KDIGOAcuteKidneyInjuryWorkGroup.KDIGOClinicalPracticeGuideline forAKI
JamesonJL,et al. Harrison's
Principles
of InternalMedicine20thedition,2018
440
ETIOLOGY I MANIFESTATIONS

Specific Diagnosis ·
• Recent skin infection or pharyngitis, edema, rashes
Glomerulonephritis
• Variable features of arthralgias, sinusitis (anti-GBM disease), lung
(GN)
hemorrhage (anti-GBM, ANCA-associated vasculitis, lupus nephritis)
• Ask about recent medication exposure (e.g., penicillins,
Interstitial cephalosporins, sulfonamides) or recent infections (e.g., leptospirosis)
nephritis • Presents with fever, rash, arthralgia, eosinophilia, sterile pyuria
• Kidney biopsy may be helpful
• Recent exposure to ischemia or nephrotoxins (e.g., aminoglycosides,
Acute tubular cisplatin, zoledronate, contrast agents, amphotericin,
necrosis rhabdomyolysis)
• Granular casts, renal tubular epithelial cell casts on urinalysis, FENa >I%
Thrombotic • Recent GI infection or use of calcineurin inhibitors
thrombocytopenic • Hematologic work-up: schistocytes on PBS, elevated LDH, anemia,
purpura (TTP) or
hemolytic-uremic thrombocytopenia
syndrome (HUS) • Kidney biopsy may be helpful

• Elderly patient with constitutional symptoms, anemia, bone pain


Myeloma • Work-up reveals monoclonal spike in urine or serum electrophoresis
• Bone marrow or renal biopsy can be diagnostic
• History of traumatic crush injuries, seizures, immobilization
Rhabdomyolysis • Elevated myoglobin & CK & heme-positive with few RBC on urinalysis
• FENa may be low
• History of recent exposure to iodinated contrast
Contrast
• Rise in serum creatinine in 1-2 days; peak within 3-5 days; and
nephropathy
recovery within 7 days

III. DIAGNOSIS
A. Overview of Common Diagnostics
DIAGNOSTIC
I REMARKS
• Determine presence of anemia, signs of infection (e.g., leukocytosis)
CBC
• Eosinophilia in interstitial nephritis, atheroembolic disease, some vasculitis
Renal function • Serum creatinine and blood urea nitrogen (BUN)
tests • Compute for GFR and renal indices discussed above
Serum • Derangements in AKI may include hyperkalemia,
electrolytes hyperphosphatemia, hypocalcemia
• Neutrophil gelatinase associated lipocalin (NGAL): novel biomarker
for AKI, increased after inflammation and kidney injury
Biomarkers
• Kidney injury molecule-I ~KIM-1):detected in the urine shortly after
ischemic or nephrotoxic injury
Urinalysis • Discussedin the next page
• Simple way to determine obstruction (post-obstructive AKI) in the
Ultrasound urinary tract (e.g., stones, BPH)
• Evaluate the renal parenchyma and overall renal structure
Angiography • Evaluate the renal vasculature, as in renal artery stenosis
• Peripheral smear if entertaining HUS or TTP
• Serology if considering lupus, vasculitis, anti-GBM disease
Others
• Renal biopsy if considering GN, vasculitis, interstitial nephritis,
myeloma, HUS/TTP, or allograft dysfunction
441
Leukocyte (-) (-) (-) (-) (-)
esterase
Heme (-) (-) (-) (-) or trace (+)

Protein (-) (-) (-) or trace (+) {+)

Specific gravity I.020 1.010 1.010 Variable Variable


Microscopy
RBCs (-) (-) (-) (-) or few (+)

WBCs (-) (-) (-) (-) (-)

RBC casts (-) (-) {-) (-) (+)

WBCcasts (-) (-) (-) (-) (-)

Granular casts (-)or few (-) (+) (-) (-)

Renal tubular
(-) (-) (+) (-) (-)
epithelial cells
Microscopy
Osmolality(mOsm/L) >500 ,;350 ,;350 Variable Variable
Protein (g/d) (-) (-) <I >3 1-2

*If a superimposedinfectionis presentdueto urinestasis,theleukocyteesterase,heme,protein,


RBCs,andWBCsmaybe positive.
**If acutetubularnecrosis(ATN)is secondary to rhabdomyolysis,hemewill be positiveon dipstick
andRBCswill beabsenton microscopy.

C. Renal Failure Indices in Differentiating Prerenal AKI from Acute Tubular Necrosis (ATN)
Index I PRERENAL AKI I ATN
Urine sodium (UNa) <20mEq/L >40mEq/L
Urine osmolality >500mOsm/kgH,O <350mOsm/kgH20
UCrto PCr >40 <20
Serum BUN to Creatinine ratio >20 :S10
Fractional excretion of Na• (FENa) <1%* >1%''
Fractional excretion of urea (FEUrea) <35 >50
*Exceptin thoseon diureticsor thosewithunderlying
CKD(FENamaybe >1% despitehypovolemia
becauseof highurineNa·excretion)
'*Exceptin thosewithglomerulonephritis,earlysepsis,rhabdomyolysis,
contrast-induced
nephropathy(FENamaybe <1%)
Sourceforbothtables:Schrier,
RW.Manual
of Nephrology,
8thEdition,
2015

442
IV. MANAGEMENT

Discontinue all nephrotoxic agents

Ensure ~olume status & perfusion pressure

Monitor serum creatinine & UO

Correct hyperglycemia

Avoid radiocontrast procedures

Non-invasive diagnostic work up


(consider invasive work-up)

Check for changes in drug doses

Consider renal replacement therapy


Source:KDIGOAcuteKidneyInjuryWorkGroup.KDIGOClinicalPracticeGuidelineforAKI
B. General Aspects of Management
ASPECT I THERAPY
Prevention and Treatment of.Complications -- .. ,. - s

Intravascular • Salt & H, 0 restriction


volume overload • Diuretics, ultrafiltration
Electrolyte • To manage hyponatremia, hyperkalemia, hyperphosphatemia,
abnormalities hypocalcemia, & hypermagnesemia - see Chapter 8
Metabolic • Sodium HCO3 (maintain serum HCO3 >15mmol/L or arterial pH >7.2)
acidosis • Dialysis in severe cases
• Treatment usually not necessary if <890 umol/L or <15mg/dL
Hyperuricemia
• Allopurinol for tumor lysis, forced alkaline diuresis for rhabdomyolysis
• Total energy intake recommended: 25-30kcal/kg/day(enteral route preferred)
• Recommended protein/amino acid intake:
Nutrition 0 0.8-1.0 g/kg/day in AKI without need for dialysis

0 1.0-1.5g/kg/day in AKI on renal replacement therapy


0 1.5-1.7
g/kg/day in AKI on continuous renal replacement therapy
• Indicated when medical management fails to control volume overload,
Dialysis hyperkalemia, or acidosis & when there are severe complications of uremia
(asterixis, pericardia! rub or effusion, encephalopathy, coagulopathy)
• Avoid other nephrotoxins: ACE inhibitors/ARBs, aminoglycosides,
Choice of agents
NSAIDs, radiocontrast
Drug dosing • Adjust dose/frequency of administration for degree of renal impairment
Glucose • For critically ill patients, insulin therapy may be used to target plasma
monitoring glucose 110-149mg/dL
Reversal of 'l{enal Insult .. .'
""'
.,
"'
,d
'
• Restore systemic hemodynamics and renal perfusion through volume
resuscitation and use of vasopressors
Ischemic AKI
• Isotonic crystalloids (rather than colloids) as initial management for
expansion of intravascular volume in AKI (or those at risk for AKI)
• Eliminate nephrotoxic agents
Nephrotoxic AKI
• Consider toxin-specific measures
., ..
Source:JamesonJL,et al. Harrison's
Principles
of InternalMedicine20thedition,2018
443
CHRONIC KIDNEY DISEASE (CKD)
I. ETIOPATHOGENESJS
Spectrum of different pathophysiologic processes associated with abnormal kidney
function and a progressive decline in GFR
Pathophysiology involves two sets of mechanisms:
0 Initiating mechanisms specific to the underlying etiology
0 Progressive mechanisms, involving hyperfiltration & hypertrophy of remaining
viable nephrons
Leading etiologies: diabetic nephropathy, glomerulonephritis, hypertension-associated
CKD (vascular and ischemic kidney disease)

A. KDIGO Definition ofCKD


Abnormalities o

• Albuminuria:
0Urinary albumin excretion rate (AER) ;,30 mg/z4h
0Urinary albumin-to-creatinine ratio (ACR);,30 mg/g (or 3 mg/mmol)
Markers of
• Urine sediment abnormalities
kidney damage
• Electrolyte and other abnormalities due to tubular disorders
(one or more)
• Abnormalities detected by histology
• Structural abnormalities detected by imaging
• History of kidney transplantation
Decreased GFR • GFR <60 mL/min/1.73 m' (GFR categories G3a-G5)

B. Staging and Prognosis ofCKD based on Cause, GFR and Albuminuria: KDIGO 2012
0Stages ofCKD are stratified by both estimated GFR (derived from the CKD-EPI
equation) and degree of albuminuria
0Used to predict progression of CKD
PersistentAlbuminuriaCategories
(DescriptionandRange)

A1 A2 A3
Normalto Moderately Severely
mildlyincreased increased increased
AER <30mg/ 30-300mg/ >300mg/
24 hrs 24 hrs 24 hrs
ACR<30mg/g 30-300mg/gor >300mg/gor
or <3 mg/mmol 3-30 mg/mmol >30mg/mmol

G1 Normalor high ,,go Lowrisk Moderately


increasedrisk

G2 Mildlydecreased 60-89 Lowrisk

Mildlyto moderately
G3a decreased
45-59

Moderatelyto
G3b severelydecreased
30-44

G4 Severelydecreased 15-29

GS Kidneyfailure <15

Sourcefor bothtables:KDIGOCKDWorkGroup.KDIGO2012CPG.2013
444
Examvle on how to write CKD Diaanosis:
Case 1:A 58-year-old Filipino male has been diagnosed with CKD 2 years ago from type 2
diabetes mellitus. His present serum creatinine is 120 umol/L (1.36mg/dL). His spot urine
creatinine is 26,520 umol/L (300 mg/dL) & urine albumin is 79 mg/dL. Using the CKD-
EPI formula, his eGFR is 57.1mL/min/1.73 m'. His spot urine ACR is 263 mg/g.
Diagnosis: Chronic Kidney Disease Stage G3a-A2 from diabetic kidney disease
Case 2: A 23-year-old female (weight 56 kg) is diagnosed with autosomal dominant
polycystic kidney disease (ADPKD). Her current serum creatinine is 217 umol/L (2.45
mg/dL). Using the CKD-EPI formula, her eGFR is 26.8 mL/min/1.73 m'. Her 24-hour urine
collection revealed urine creatinine ofI036 mg/24h and urine albumin of2.52 g/24h. Her
24h urine collection is deemed adequate (18.5mg/kg urine creatinine).
Diagnosis: Chronic Kidney Disease Stage G4-A3 from ADPKD

C. Definition ofCKD Progression


0 Regardless of etiology, CKD will eventually progress to end stage renal disease (ESRD)
° CKD progression is defined as progressively declining eGFR & increasing albuminuria
• CKD progression: decline in GFR category (e.g., from G1 to G2) accompanied by
~25%drop in eGFR from baseline
• Rapid CKD progression: sustained decline >5 mL/min/1.73 m' per year
0 Factors affecting the likelihood & rate of CKD progression
• Baseline eGFR and degree of albuminuria
• Primary renal disease
• Ongoing exposure to nephrotoxic agents
• Others: obesity, hypertension, age, ethnicity, and laboratory parameters
0 The normal annual decline in GFR with age from peak GFR (-120 mL/min/1.73 m') is
-1 mL/min/1.73 m' per year, reaching a mean value of?o mL/min/1.73 m' at age 70
D. End-Stage Renal Disease (ESRD)
0Stage ofCKD where accumulation of toxins, fluid, and electrolytes normally excreted
by kidneys results in the uremic syndrome
0There is marked disturbance in the activities of daily living, well-being, nutritional
status and water and electrolyte homeostasis
0Uremia leads to disturbances in the function of virtually every organ system

II. CLINICALMANIFESTATIONS
FINDINGS I MANIFESTATIONS
Fluid, Electrolyte, and Acid-Base Disorders
• Total body content of Na• & H2O are increased, leading to hypertension
& peripheral edema
Sodium • Hyponatremia not commonly seen
and water • Impaired renal conservation of sodium and water
homeostasis • Signs of volume overload indicates CKD associated with CHF or cirrhosis
• Signs of volume depletion may suggest long-standing prerenal azotemia
with recurrent AKI, leading to CKD
• Hyperkalemia may be due to decline in urinary K· excretion, leading to
K· retention
• Hyperkalemia is precipitated by increased dietary K• intake, protein
Potassium
catabolism, hemolysis, hemorrhage, transfusion of RBC, metabolic
homeostasis
acidosis, medications (e.g., RAAS inhibitors, spironolactone, NSAIDs)
• Hypokalemia is uncommon (e.g., reduced dietary intake, excessive
diuretics, GI loss)
• Common disturbance in advanced CKD
Metabolic
• Due to impaired ammoniagenesis, leading to impaired excretion of protons
acidosis
• NAGMA in early stages, then HAGMA in later stages
445
• High bone turnover with increased PTH: osteitis librosa cystica (classic
lesion of secondary hyperparathyroidism)
• Low bone turnover with low/normal PTH: adynamic bone disease &
Bone
osteomalacia
manifestations
• FGF-23: produced by osteocytes, which promotes phosphate excretion
(earliest to increase during the course of CKD, even before PTH, calcium
and phosphorus levels rise, and vitamin D levels fall)
• Strong association between hyperphosphatemia & increased CV mortality
Calcium,
• Hyperphosphatemia & hypercalcemia are associated with increased
phosphorus, &
vascular calcification
cardiovascular
• CKD patients have calcification in the arterial media in contrast to the
system
usual atherosclerosis (which involves arterial intima layer)
Calciphylaxis • Almost exclusive to advanced CKD
(calcific uremic • Livedo reticularis: patches of ischemic necrosis especially on legs, thighs,
arteriolopathy) abdomen, & breasts (warfarin treatment is a risk factor for its development)
Hemato.logi1(>4.bno.r,nalities.
• Normocytic, normochromic anemia observed as early as CKD 3 &
universal in CKD 4
• Causes: relative EPO deficiency, diminished RBC survival, bleeding
Anemia diathesis, iron deficiency, hyperparathyroidism/marrow fibrosis, chronic
inflammation, folate, or vitamin B12deficiency, hemoglobinopathy
• Other causes: comorbid conditions such as hypo/hyperthyroidism,
pregnancy, HIV, autoimmune disease, and immunosuppressive drugs
• Prolonged bleeding time, decreased activity of platelet factor III, abnormal
Abnormal platelet aggregation & adhesion, & impaired prothrombin consumption
hemostasis • Greater susceptibility to thromboembolism especially if with nephrotic-
range proteinuria
CardiovascularA:bnormalities(leading cause of morbidity and·mortality in CKD patients)
• CKD is a major risk factor for ischemic CVD
• Risks:
0 Presence of traditional risk factors in CKD patients
Ischemic
° CKD-related nontraditional risk factors: anemia, hyperphosphatemia,
vascular
increased FGF-23, sleep apnea, and generalized inflammation
disease
• Cardiac troponins frequently elevated in CKD patients without evidence
of acute ischemia
• Abnormal bruits may indicate renal artery stenosis
• Abnormal cardiac function from ischemia, left ventricular hypertrophy
Heart failure
(LVH), and frank cardiomyopathy
• Hypertension develops early during CKD
• LVH and dilated cardiomyopathy are the strongest risk factors for death
& morbidity in CKD patients
Hypertension • Absence of hypertension (worse prognosis) may signify a salt-wasting
andLVH form of CKD, effect of anti-HPN therapy, volume depletion or poor left
ventricular function
• Fundoscopic abnormalities (e.g., retinopathy, arteriovenous nicking)
may be seen in microvascular disease (e.g., hypertension, diabetes)
• Chest pain with respiratory accentuation accompanied by friction rub
Pericardia!
• Pericarditis observed in advanced uremia (more often seen in
disease
underdialyzed, non-adherent patients)
446
FINDINGS I MANIFESTATIONS
Other Systemic Manifestations
• Due to retained nitrogenous metabolites and middle molecules
• Early signs seen at CKD 3; usually clinically evident at CKD 4
CNS, peripheral
• Early: mild disturbances in memory, concentration & sleep
and autonomic
• Late: hiccups, cramps, twitching, restless leg syndrome
neuropathy
• Advanced untreated CKD: asterixis, myoclonus, seizures & coma
• May be associated with microvascular diseases or dysautonomia (e.g.,DM)

• Uremic fetor: urine-like breath odor with dysgeusia (unpleasant metallic taste)
Gastrointestinal
• Gastritis, peptic disease, or mucosa! ulcerations at any level of the GIT
manifestations
• Anorexia due to retention of uremic toxins
• Consequence of low protein and caloric intake, metabolic acidosis,
Protein-energy
inflammatory cytokines
malnutrition
• Resistance to anabolic effects of insulin and growth factors
(PEM)
• Assessment for PEM starts at Stage G3

• Abnormal glucose metabolism: slowed response to glucose loading, FBS


Endocrine/ normal or slightly elevated, and diminished renal degradation of insulin
metabolic • Sexual dysfunction
disturbances • Low estrogen, menstrual abnormalities, inability to carry pregnancies to term
• Reduced plasma testosterone, oligospermia
• Skin changes: pruritus, hyperpigmentation (due to urochromes)
Dermatologic
• Nephrogenic fibrosing dermopathy: progressive subcutaneous induration
manifestations
especially on the arms and legs associated with exposure to gadolinium
Source:JamesonJL, et al. Harrison'sPrinciplesof InternalMedIcIne
20thed1t1on,
2018
FatehiP,et al. Chronickidneydisease(newlyidentified);UpToDate; 2021

III. DIAGNOSIS
A A t Kid D' (AKO) Ch ' K'd o· (CKD)
ACUTE KIDNEY DISEASE CHRONIC KIDNEY DISEASE
I (AKD) I (CKD)
• AKI•
• GFR <60 mL/min/1.73 m' for
Functional <3 months; OR • GFR <60 mL/min/1.73 m' for
Criteria • Decrease in GFR by ~35% or >3 months
increase in SCr by >50% for
<3 months

Kidney
• Present <3 months • Present >3 months
damage ..

• Small (except in diabetic nephropathy,


HIV-associated nephropathy,
Kidney infiltrative disease)
• Normal or large
size • Size discrepancy may be present
• Loss of renal cortex (e.g., cortical
thinning)
*Notall withAKIwillmeettheeGFRcriteriaforthedefinition ofAKO.Kidneydamageis nota
criterionforAKI,however;
it maybepresent(e.g.,in a patientwitha baselineGFRof >60,an
increasein SCrof only<1.5xduring7 consecutivedays& a GFRof <60duringthenext3 months
hasAKO,butnotAKI)
**Kidney damageis determinedby urinarymarkers(e.g.,casts,proteinuria)
andimagingstudies
Source:KOIGO
AcuteKidneyInjuryWorkGroup.KOIGOClinicalPracticeGuideline
forAKI
447
B. Diagnostics
0 Most common laboratory finding in CKD include increased serum creatinine and
BUN, proteinuria (albuminuria), and/or abnormal cells on urine microscopy
0 Other findings include anemia, hyperkalemia, hyperphosphatemia, metabolic
acidosis, hypocalcemia, and elevated parathyroid hormone (PTH)
DIAGNOSTIC I COMMENTS/EXPECTED FIDINGS

Basic • CBC: to check for anemia, infection, thrombocytopenia


laboratory • BUN, creatinine: to estimate eGFR
tests • Electrolytes: to determine abnormalities from deranged renal function
• 24-hour urine collection: standard for measurement of albuminuria
Measurement
• Protein-to-creatinine ratio in spot first-morning urine: more practical
of albuminuria
and correlates well with 24-hour urine collection
• Microscopic hematuria with abnormal RBC morphology (anisocytosis):
GBM disorders
Urinary • RBC casts: proliferative glomerulonephritis
sediment • WBC casts: pyelonephritis, interstitial nephritis
abnormalities • Oval fat bodies or fatty casts: diseases with proteinuria
• Granular casts & renal tubular epithelial cells: non-specific for
parenchymal diseases
• Verifies presence of 2 kidneys, determines symmetry, estimates size, and
rules out masses/obstruction
• Discrepancy >I cm in kidney length suggests either:
Renal 0 Unilateral developmental abnormality
ultrasound
0 Renovascular disease causing hypoperfusion to smaller kidney
• ·Finding of bilaterally small kidneys supports CKD (except for early DM
nephropathy, amyloidosis, HIV nephropathy, polycystic kidney disease)
• Not advised for bilaterally small kidneys
Renal biopsy • Other contraindications: uncontrolled hypertension, active UT!, bleeding
diathesis (including ongoing anticoagulation), & severe obesity

V. MANAGEMENT
A. Interventions and Goals in CKD
INTERVENOON I GOALS

• If obese, aim for 5% weight loss


Exercise & • Moderate intensity physical activity for at least 150minutes per week, or
lifestyle to a level compatible with their cardiovascular and physical tolerance
• Complete smoking cessation
• In DM: diet high in vegetables, fruits, whole grains, fiber, legumes,
plant-.based proteins, unsaturated fats, and nuts; and low in processed
Dietary meats, refined carbohydrates, and sweetened beverages
modification • Salt restriction: <5g/day (equivalent to <2g/day or 90 mEq of Na• per day)
• Energy requirements: 25-35kcal/kg/day in CKD G1-5D&
·, posttransplantation who are metabolically stable
• No CKD but at risk (solitary kidney, polycystic kidney, DM, HPN):
moderately low protein diet (0.8-1.0g/kg/day)
• CKD G3-5, metabolically stable, nondiabetic, not on dialysis:
Protein 0Low protein diet (LPD): 0.55-0.60 g/kg/day, OR
intake 0Supplemented very low protein diet (sVLPD): 0.28-0.43 g/kg/day with
ketoanalogues of amino acid (KAA) supplementation
• CKD G3-5, diabetic, not on dialysis: o.6-o.8 g/kg/day
• CKD G5 on maintenance HD/PD, with or without DM: 1-1.2g/kg/day

Avoid and • Manage AKI risk during intercurrent illness or during procedures that
prevent AKI are likely to increase AKI risk
448
B. Blood Pressure Control with RAAS Blockade
0 ACE inhibitors or ARBs (to reduce intraglomerular hypertension & proteinuria) to target:
• Urine protein level <0.5 g/day
• Slow progression ofCKD and GFR decline to <I ml/min/year

Ti ' t Bl d P
SCENARIO I TARGET BP
CKD +/- DM not on dialysis • Systolic BP <120 mmHg as tolerated
CKD transplant • Systolic BP <130 mmHg and diastolic BP <80 mmHg
• Predialysis BP <140/90 mmHg
CKDonHD
• Postdialysis BP <130/80 mmHg

CKD transplant • CCBs or ARBs


Source: KDIGOBlood Pressure WorkGroup. 2021

C. Glycemic Control
0 Individualized

° F' st I' th t~ . & d' 1 t .


HbA1c target of <6.5 to <8.0% & FBS 90-130 mg/dL in those not on dialysis
t 2 · h'b't (SGLT2')
eGFR I CHOIC~ OF THERAPY
• Start metformin &/or SGLT2i, plus additional therapy to achieve target
eGFR ,30 • May use GLP-1 receptor agonises (preferred), DPP4-inhibitors, insulin,
sulfonylurea, TZD or alpha-glucosidase inhibitors

• Discontinue or do not initiate metformin and/or SGLT2i


eGFR<3oor
• May use GLP-1 receptor agonists (preferred), DPP 4-inhibitors, insulin,
on dialysis
sulfonylurea, TZD or alpha-glucosidase inhibitors
Source: KDIGODiabetes Work Group. KDIGO2020Diabetes. Kidney Inter.2020

D. Commonly Used Drugs in Management of CKD Patients


1. Renin-Angiotensin System Blockers
• ACE-inhibitors and ARBs are first line anti-hypertensive agents
• Optimal renoprotective doses:
• Losartan 100 mg daily • Valsartan 320-640 mg daily
• Candesartan 16 mg daily • Lisinopril 40 mg daily
• lrbesartan 900 mg daily
• Caution: may cause reversible hyperkalemia & AKI especially in advanced CKD &
renovascular hypertension

2. Diuretics
• Useful in CKD because there is decrease in filtered load of salt & fluid when there is
reduced GFR
• Loop diuretics: preferred in dissipating edema & in treating HPN, acidosis, & hyperkalemia
• Avoid exceeding ceiling doses ofloop diuretics to prevent AKI, ototoxiciry, & electrolyte
imbalance

GFR

GFR 20-50 mL/min • 80-160mg IV or 160mg PO • 6 mg IVor PO

GFR <20 mL/min • 200 mg IV or 200mg PO • 10 mg IVor PO


Source: Skorecki K, et al. Brenner and Rector's The Kidney;Elsevier.2016
449
3. Drugs for Anemia
CLASS I REMARKS I EXAMPLES
• To provide -200 mg/day of
Oral iron • Ferrous sulfate 325 mg/tab
elemental iron in non-dialytic
Supplement (elemental iron 65 mg/tab) TID
CKD patients
• Can be used in both non-dialytic • Iron sucrose 1000 mg IV in 10
Intravenous and dialytic CKD patients doses (100 mg/dose) is equivalent
iron • Better tolerated compared to oral to iron content in one bag of
iron (but more expensive) packedRBC
• Erythropoietin-alpha/beta 20-50
Erythropoiesis • Preferred initial form of therapy
IU/kg body weight 3x/wk SC/IV
stimulating • Used when Hgb <100 g/L despite
• Darbopoietin-alpha 0.45 ug/kg
agents (ESAs) correction of iron deficiency
body weight 1x/wk SC/IV
Source:KDIGOWorkGroup.KDIGO2012Chrncal
PracticeGuideline
forAnemia.KidneyInter;2012

4. Mineral-Bone Disorders
CLASS I REMARKS I EXAMPLES
• Calcium carbonate 500 mg/tab
• Calcium-based: more contains 200 mg of elemental
commonly used but dose is calcium, given I tab TIO with meals
restricted (<2000 mg/day) to • Calcium acetate 667 mg/cap contains
Phosphate prevent hypercalcemia 169mg of elemental calcium, given 2
binders caps with each meal
(taken with • Sevelamer 800 mg/tab, I tab TID
meals) with meals
• Non-calcium-based: less • Lanthanum carbonate: effective
calcium exposure to patients binder but no long-term studies
• Aluminum hydroxide: now avoided
due to potential for aluminum toxicity
• Used in CKD stage 5 (on dialysis)
with hyperparathyroidism • Cinacalcet 30 mg OD
Calcimimetics
• Used in combination with (downregulates PTH levels)
calcitriol or vitamin D analogs
Calcitriol & • Not routinely used • Calcitriol 0.25mcg OD-BID,
VitaminD • Reserved for CKD stage 4-5 with while carefully monitoring for
analogues severe hyperparathyroidism hypercalcemia & hyperphosphatemia

5. Bicarbonate Therapy
• Start oral bicarbonate supplementation when serum bicarbonate levels fall <22 mmol/L
• Dose of oral Na- bicarbonate: 0.5 to I mEq/kg/day (target HCO3 level within normal range)
• Oral sodium bicarbonate 650 mg contains 7.7 mEqs of bicarbonate
• Example: 60-kg male - initial dose can be I tab TID-QID depending on baseline
bicarbonate level
• Can slow down progression ofCKD

6. Ketoanalogues of Amino Acids (KAA) Supplement


• Mixture of essential amino acids and nitrogen-free ketoanalogues
• Given as supplement to a very low protein diet (0.3 g/kg/day)
• Shown to delay onset of uremia and initiation of dialysis in nondiabetic CKD patients
• Dose: ketoanalogue 600 mg/tab, given as I tablet per 5-10 kg of body weight
• Example: a 60-kg male on very low protein diet will need 12 tablets per day (given as
4 tabsTID)
• Carefully monitor serum calcium levels as KAA preparations contain different calcium salts
450
E. Management of Uremic Complications
ASPECT I MANAGEMENT
• Salt restriction: if with evidence of volume expansion
• Loop diuretics+/- metolazone to maintain euvolemia (thiazides have
Fluid
limited utility in CKD stages 3-5)
disturbances
• Intractable edema, hypertension and hyperkalemia in advanced CKD
are indications for initiating dialysis
Hyponatremia • Responds to water restriction
• Dietary restriction of K+ & avoidance of K+ supplements
• Kaliuretic diuretics: promote urinary K+ excretion
Hyperkalemia • Potassium binding resins (e.g., calcium resonium,
sodium polystyrene): promote K+ loss through the GIT
• Dialysis for intractable hyperkalemia

Electrolyte • If asymptomatic: oral Ca" supplementation taken in


abnormalities between meals, correct underlying hyperphosphatemia
Hypocalcemia
• If symptomatic: may give IV calcium to target serum Ca'•
levels within normal range (2.1-2.5mmoUL)
• Low-phosphate diet
• Phosphate-binding agents taken with meals to limit GI
Hyper-
absorption of dietary phosphate
phosphatemia
• Target phosphate within normal range (0.81-1.45mmol/L)
• Target intact PTH 2-9 times the upper normal limit
• Alkali supplementation (e.g., NaHCO3): may slow down CKD
Metabolic progression (started when HCO3 <20-23mmol/L)
acidosis • Concomitant sodium load in NaHCO3 needs careful attention to volume
status and possible need for diuretics

• Hypertension: control BP based on current guidelines (seeprevious)


Cardiovascular • Uremic pericarditis: absolute indication for urgent initiation or
intensification of dialysis (heparin-free)
• Recombinant human EPO: initiated once with adequate bone marrow
iron stores
• Iron supplementation: to ensure adequate bone marrow iron stores
Anemia
(avoid when ferritin is >500 ng/mL)
• Vitamin B12and folate supplementation
• Target hemoglobin 100-II5 g/L

• Most abnormalities resolve with dialysis and successful renal


Neuromuscular
transplantation

• Protein energy malnutrition (PEM) is an indication to do renal


replacement therapy
Nutritional
• Protein restriction may slow renal decline at earlier stages (may increase
protein intake if with or at risk for PEM)
• Local moisturizers, mild topical steroids, UV radiation
Dermatologic • Minimize exposure to gadolinium in CKD stage 3 & avoidance in CKD
stage 4-5

Source:Source:Jameson JL,et al. Harrison's


Principles
of InternalMedicine20thedition,2018
SkoreckiK,et al. BrennerandRector'sTheKidney,10thEdition.Elsevier. 2016
KDIGOCKDWorkGroup.KDIGO2012CPG.2013
Singh,AK.Proteinrestriction andprogression of chronickidneydisease.In:UpToDate, 2013
lkizlerTA,et al. KDOQINutritionin CKOGuideline WorkGroup.2020
KDOQIclinicalpracticeguidelines
for cardiovasculardiseasein dialysispatients.
AmJ KidneyDis.2005

451
RENAL REPLACEMENT THERAPY (RRT)
• Done via kidney transplantation (KT), hemodialysis (HD) or peritoneal dialysis (PD)
KT offers best potential for complete rehabilitation; as HD/PD replaces only a small
fraction of the kidneys' filtration function and none of the other renal functions such as
endocrine and anti-inflammatory effects
Dialysis relies on the principles of solute diffusion across a semipermeable membrane,
where movement of metabolic waste products takes place down a concentration
gradient from the circulation into the dialysate

ABSOLUTE INDICATIONS FOR RRT I RELATIVE INDICATIONS FOR RRT


• Pericarditis or pleuritis' • Anorexia and nausea
• Progressive uremic encephalopathy or • Impaired nutritional status
neuropathy, with signs such as confusion, • Increased sleepiness
asterixis, myoclonus, wrist or foot drop, • Decreased energy level, attentiveness,
or seizures* and cognitive tasking
• Clinically significant bleeding diathesis
attributable to uremia'
• Persistent metabolic disturbances
that are refractory to medical therapy
(hyperkalemia, metabolic acidosis,
hypercalcemia, hypocalcemia, and
hyperphosphatemia)
• Fluid overload refractory to diuretics
• Hypertension poorly responsive to
antihypertensive medications
• Persistent nausea and vomiting
• Evidence of malnutrition
*Considered for RRT
as urgentindications
Source:Bleyer,A. Indications
for initiationof dialysisin chronickidneydisease.In: UpToDate,
2013
SkoreckiK, et al. Brennerand Rector'sThe Kidney,10thEdition.Elsevier.2016

452
OTHER DISORDERS ENCOUNTERED IN NEPHROLOGY

GLOMERULAR DISEASES (CLINICAL SYNDROMES)


Acute Nephritic Pulmonary-Renal
Syndrome I Syndromes I Nephrotic Syndromes
• 1-2g/24h proteinuria • RPGN with lung • >3 g/24 h proteinuria
• Hematuria with RBC casts hemorrhage • Hypertension
• Pyuria • Hypercholesterolemia
• Hypertension • Hypoalbuminemia
• Fluid retention • Edema/anasarca
• Rise in SCr • Microscopic hematuria

Soff!.ePrototype Diseases
'"
• Post-streptococcal • Goodpasture's syndrome • Minimal change disease
glomerulonephritis • ANCA vasculitis: • Focal segmental
• Subacute bacterial IE granulomatosis with glomerulosclerosis
• Lupus nephritis polyangiitis (GPA), • Membranous
• Anti-GBM eosinophilic granulomatosis glomerulonephritis
• lgA nephropathy with polyangiitis (EGPA), • Diabetic nephropathy
• ANCA vasculitis microscopic polyangiitis • AL and AA amyloidosis
• Henoch-Schonlein purpura • Henoch-Schonlein • Light-chain deposition
• Cryoglobulinemia purpura (HSP) disease
•MPGN • Cryoglobulinemia • Fabry's disease

Basement Membrane Glomerular Vascular IAssociated


Infectious Disease-
Syndromes I Syndromes Syndromes
• Microscopic hematuria • Vascular injury • Inflammatory reactions in
• Mild to heavy proteinuria • Hematuria glomerular capillaries
• Hypertension with • Moderate proteinuria • Combination of
variable elevations in SCr hematuria & proteinuria

Some Prototype Diseases


• Anti-GBM disease • Atherosclerotic nephropathy • Post-streptococcal GN
• Alport's syndrome • Hypertensive nephropathy • Subacute bacterial IE
• Thin basement membrane • Cholesterol emboli • HIV
.
disease • Sickle cell disease • Hepatitis Band C
• Nail-patella syndrome • Thrombotic • Syphilis
microangiopathies • Leprosy
• APAS • Malaria
• ANCA vasculitis • Schistosomiasis
• HSP
• Cryoglobulinemia

NEPHROLITHIASIS
I. ETIOPATHOGENESIS
Refers to calculi in the kidneys or renal calculi (note that ureteral calculi are called
ureterolithiasis)
Urine citrate is the most clinically important inhibitor of calcium-containing stones

A. Formation of Stones
0 Supersaturation of urine with respect to stone-forming crystals (calcium, oxalate, struvite,
cysteine, and uric acid) accompanied by a decrease in inhibitors of crystallization
0 Deposition of stone material on a Randall's plaque (calcium phosphate deposits at the
tip of renal papillae), which is where majoriry of calcium oxalate stones grow
453
B. Risk Factors for Nephrolithiasis
FACTOR
I INCREASED RISK
• Low dietary calcium and/or fluid intake
Dietary • High supplemental calcium intake, oxalate diet, animal protein diet,
factors sodium, and sucrose intake
• Others: ascorbic acid supplements, sugar-sweetened carbonated beverages
• Middle-aged
Non-
• Weight gain
dietary
• Hot environment
factors
• Lack of ready access to water or bathroom
• Low urine volume (<IL/day)
• Hypercalciuria (most common metabolic abnormality), hyperoxaluria,
Urinary hyperuricosuria
factors • Hypocitraturia
• Urine pH s5.5 for uric acid stones
• Urine pH;;:6.5for calcium phosphate stones
Genetic • Primary hyperoxaluria
factors • Cystinuria

II. MANIFESTATIONS
May be asymptomatic and incidentally noted during radiographic studies undertaken
for unrelated reasons
Classic presentation of acute renal colic (as stone traverses the ureter):
0 Sudden onset of severe pain in the flank
0 Begins in the lateral upper-midback over the costovertebral angle
0 Radiates inferiorly and anteriorly
Pain is due to dilation and spasm caused by acute ureteral obstruction
• May be associated with microscopic hematuria, painless gross hematuria, pain radiating
to lumbar or groin area
On examination: costovertebral angle tenderness may be observed
III. DIAGNOSIS
DIAGNOSTIC I REMARKS
CBC • May show leukocyrosis, especially in febrile patients
Chemistry • Serum creatinine (to measure renal function) and electrolytes
• Dipstick test: to demonstrate hematuria, bacteriuria, or crystals
Urinalysis
• Urinary pH (and serum pH) may give a clue as to the type of calculus
• Gold standard diagnostic procedure
Helical CT • Can visualize almost all types of stones (except indinavir-related
scan without stones) as small as I mm
radiocontrast • Can determine stone density using Hounsfield units (HU) to guide
identification of stones & prognosticate success of shock wave treatment
Plain • Can identify radiopaque stones (calcium oxalate and phosphate,
radiograph of struvite, cysteine) that are ;,2 mm in size
the abdomen • Can still miss a stone in the ureter or kidney
• Avoids radiation and provides information on hydronephrosis
Abdominal • Less sensitive than CT scan
ultrasound • Images only the kidney and proximal segment of the ureter, hence
distal ureteral stones are not detectable
• Includes intravenous pyelography
Urologic
• Plain renal tomography
imaging
• Retrograde pyelography
454
IV. MANAGEMENT
A. Acute Management of Renal Colic
0 Medical treatment has been shown to facilitate spontaneous passage of ureteral stones
0 Necessary to manage renal colic until spontaneous passage of stone within 48 hours,
which usually occurs in patients with:
• Smaller stones (<4 mm)
• More distal ureteral stones
ASPECT I MANAGEMENT
• IV hydration
• Analgesics (e.g., ketorolac, ibuprofen, morphine)
Supportive • Anti-emetics
, Hospital admission for pain warranting IV analgesics, presence of UT!/
sepsis, or obstruction in a solitary or transplanted kidney
• For pain retie~ improvement of quality of life, & reduction of stone transit time
• Includes:
Medical
° Calcium channel blockers (e.g., nifedipine): suppress smooth muscle
expulsive
contraction and reduce ureteral spasm
therapy 0 Alpha-I adrenergic blockers (e.g., tamsulosin, terazosin): decrease

ureteral smooth muscle tone and the frequency and force of peristalsis
• 50% of patients experiencing acute upper urinary tract stones will require
Surgical surgery (e.g., percutaneous nephrostomy, extracorporeal shockwave
consult lithotripsy, scent placement)
• Stones ~7 mm are unlikely to pass spontaneously

B. Chronic Management (according to type of stone)


0 Prevention of stone formation & recurrence depends on stone type & metabolic evaluation
0 Stone analysis & 24-h urine profiling (e.g., volume, pH, Ca'-, Na-, oxalate, citrate levels)
and fasting blood chemistries (e.g., metabolic panel, blood gas, PTH, vitamin D assay)
TYPE I COMMON
ETIOLOGIES I MANAGEMENT
• Increase water intake to produce urine at
• Hypercalciuria least 2 L/day
Calcium • Hypocitraturia • Low-sodium and low-protein diet
oxalate • Dietary hyperoxaluria • Thiazides (for idiopathic hypercalciuria)
stones • Hyperuricosuria • Alkali supplements such as potassium
(75%) • Low urine volume & pH citrate (for hypocitraturia, hyperuricosuria)
• Medullary sponge kidney • Low-oxalate diet (for hyperoxaluria)
• Allopurinol (for hyperuricosuria)
Calcium • Increase water intake
phosphate • High urine pH
stones(15%) • Avoid excessive urinary alkalinization

• Low urine pH (metabolic


syndrome, primary gout, high
• Alkali supplements to raise urine pH to 6.5
Uric acid animal protein, diarrhea)
• Increase water intake to 2-3 Lid
stones • Low urine volume (diarrhea)
• Low purine diet & allopurinol if daily
(8%) • Hyperuricosuria (e.g.,
urine uric acid >JOOO mg
uricosuric drugs, congenital
enzyme disorders)
• Fluids to produce urine of at least 4 L/day
Cystine
• Hereditary cystinuria • Alkalinize urine to pH 7.5
stones(!%)
• D-penicillamine ifneeded
• Percutaneous nephrolithotomy &
Struvite • Infections (Proteus, extracorporeal shock wave lithotripsy
stones Morganella,Providencia, • Antibiotic prophylaxis postoperatively
(1%) Pseudomonas,Klebsiel/a) • Urease inhibitors (acetohydroxamic acid)
for those with recurrent upper UTis
455
RENAL TUBULAR DEFECTS
I. POLYCYSTIC KIDNEY DISEASE
A. Etiopathogenesis
0 Group of genetically heterogenous disorders and a leading cause of kidney failure
0 Autosomal dominant polycystic kidney disease (ADPKD) is the most common life-
threatening monogenic disease
0 The recessive form (ARPKD) is rarer but affects the pediatric population

B. Clinical Manifestations
° Focal cysts are typically present before the age of 30, while hundreds to thousands of
cysts are already present in most patients by the 5th decade
0Variable clinical presentation:
• Asymptomatic until 4th-5th decade of life
• Only diagnosed by incidental findings of hypertension, abdominal mass, or back/
flank pain (most common symptom) which may result from renal cyst infection,
hemorrhage, nephrolithiasis
0Other complications:
• Renal insufficiency
• Renal cell carcinoma (rare)
• Cardiovascular complications (major cause of mortality)
• Liver cysts (most common extrarenal complication)
• lntracranial aneurysms (4-5x more frequent in ADPKD)
• Mitra! valve prolapse (30% of ADPKD)

C. Diagnosis
0 Positive family history consistent with AD inheritance and multiple kidney cysts bilaterally
0 Renal ultrasound: used for pre-symptomatic screening of at-risk subjects and
evaluation of potentially related kidney donors from ADPKD families
° CT scan & T2-weighted MRI more sensitive but exposes patient to radiation & radiocontrast
0 Genetic testing for ambiguous cases
AGE (YR) I CRITERIA FOR POSITIVE DIAGNOSIS
• 2:3cysts, unilateral or bilateral
40-59 • 2:2cysts in each kidney
2:60 • 2:4cysts in each kidney

• 2:2cysts
Source:SkoreckiK, et al. Brennerand Rector'sThe Kidney,10thEdition.Philadelphia,
PA:Elsevier.2016

D. Management
0No specific treatment to prevent cyst growth or decline of renal function has been
approved
0High-dose tolvaptan (vasopressin-2 receptor antagonist): approved in slowing renal
decline (Europe, Canada, and Japan)
0BP control to target <140/90 mmHg is recommended: ACEi or ARBs may be superior to
diuretics and CCBs
0Antibiotics (TMP-SMX, quinolones, chloramphenicol) for cyst infection, usually 4-6 weeks
0Standard treatment for kidney stones including pain relief and hydration
° Chronic flank, back or abdominal pain may need pharmacologic (analgesics), non-
pharmacologic, or surgical measures
0More than half will eventually require renal replacement therapy (KT, PD, or HD)
456
11.RENALTUBULAR ACIDOSIS
Disorder of renal acidification out of proportion to the reduction in GFR
• Characterized by hyperchloremic metabolic acidosis with normal anion gap
ETlOPATHOGENESIS I FEATURES I MANAGEMENT
Type r: Distal (Secretory Defect)
• Kidneys unable to acidify • Hypokalemia • Alkalireplacement 1-3mmol/
the urine to pH <5.5 in • Hypocitraturia, kg/day in divided doses
the presence of systemic hypercakiuria • Citrate tolerated better
metabolic acidosis or after • Nephrocalcinosis, and/ than NaHCO3
acid loading or nephrolithiasis • Large fluid intake &
• Due to impaired hydrogen ion sufficient alkali to
secretion (H·-ATPase defect) correct hypocitraturia &
or HCO3 reabsorption in hypercakiuria in patients
distal nephron with kidney stones
Type 2: Proximal (Reabsorptive Defect)
• Result of impaired HCO3 • Hypokalemia • Alkali supplementation
reabsorption in the proximal • Hyperphosphaturia, 5-15mmol/kg/day with
tubule where the bulk of hyperuricosuria, supplemental potassium
filtered HCO3 is recovered hypercakiuria, non- • Prefer citrate over sodium
• Usually due to autoimmune, selective aminoaciduria bicarbonate to avoid renal
drug-induced, infiltrative, or & glycosuria, potassium wasting
other tubulopathies encompassing Fanconi
Syndrome
Type 4 (Hypoaldosteronism)
.. ''" -
• Distal tubule secretion of K· • Associated with • Correction of
& H· impaired, resulting in moderate renal hyperkalemia
hyperchloremic acidosis with dysfunction • Management ofrenal
hyperkalemia dysfunction and
• Most common form ofRTA, underlying disease
often seen in diabetic kidney
disease, SLE, NSAID use, sickle
cell anemia & amyloidosis

URINARY TRACT OBSTRUCTION (UTO)


I. ETIOPATHOGENESIS
Obstruction of urine flow impairs renal and urinary conduit functions due to associated
stasis and elevation of urinary tract pressure
UTO is a common cause of AKI and CKD (i.e., obstructive nephropathy)

• Abnormal insertion of ureter into bladder (most common cause)


Congenital
• Vesicoureteral reflux
causes
• Posterior urethral valves (most common cause of bilateral hydronephrosis in boys)

• Nephrolithiasis: most common cause ofureteral obstruction in younger men


• Benign prostatic hyperplasia: most common cause ofUTO in men
Acquired
• Malignancies: cervical and bladder cancer are the first and second most
causes
common cause of malignant ureteral obstruction, respectively
. • Abdominal aortic aneurysm: most common vascular cause ofUTO

Prognosis
Recovery of renal function depends largely on whether irreversible damage has
occurred (course will depend on whether obstruction is complete or incomplete,
bilateral or unilateral, or if infection is present)
0 1·2 weeks of complete obstruction: may still have partial return of GFR
0 After 8 weeks of obstruction: recovery is unlikely
457
II. CLINICAL MANIFESTATIONS
Most commonly presents with pain (capsule distention)
Polyuria and nocturia commonly accompany partial UTO
HPN is frequent in acute or subacute unilateral UTO, or in CKD from bilateral UTO
Evidence of distention of the kidney or bladder by palpation & percussion of abdomen
Genital & rectal examination should be done to check for prostate, pelvic or rectal masses
CATEGORY I REMARKS
• Usually due to intraluminal obstruction such as calculi
Acute
Duration • Severe renal colic with flank pain, hematuria, nausea
Chronic • Usually due to ureteral strictures, malignancy, iatrogenic causes
• Unilateral dilatation of ureter (hydroureter) & renal pyelocalyceal
Unilateral
Dilatation system (hydronephrosis):occurs if obstruction is above level of bladder
Bilateral • Occurs iflesion is at or below the level of bladder
Extrinsic • From compressive or restrictive force
Location
Intrinsic • From intraluminal obstruction such as calculi, tumors, papilla

III. DIAGNOSIS
DIAGNOSTIC I FINDINGS
• Azotemia +/- uremia occurs when overall excretory function is
impaired (e.g., bladder outlet obstruction, bilateral obstruction, or
Chemistry
unilateral obstruction in a solitary kidney)
• In some, distal RTA, hyperkalemia, and renal salt wasting may occur
Urinalysis • Hematuria, pyuria, bacteriuria, normal urine sediments
Abdominal • To evaluate for hydronephrosis, renal/bladder size, pyelocalyceal contour,
ultrasound or possible causes of obstruction (e.g., radiopaque stones, masses)
Noncontrast high • Advantageous in visualizing retroperitoneum and identifying
resolution CT intrinsic & extrinsic sites of obstruction
scan • Safe for patients with renal impairment
Retrograde/
• For visualization of a suspected lesion in the ureter or renal pelvis
anterograde
without the risk of contrast nephropathy
urography
Voiding
• For vesicoureteral reflux & bladder neck and urethral obstructions
cystourethrography

IV. MANAGEMENT
ASPECT I MANAGEMENT
Initial • Insertion of bladder catheter is both diagnostic and therapeutic
intervention • Analgesics
• Need for immediate relief (to prevent development of sepsis
and progressive renal damage) with urologic intervention (e.g.,
If complicated by nephrostomy, ureterostomy, or urethral or suprapubic catheterization)
infection • May need prolonged antibiotics
• Chronic or recurrent infections in a poorly functioning and
obstructed kidney may necessitate nephrectomy
If without infections • May delay surgery until acid-base & fluid/electrolyte status is restored
• Polyuria (may be massive) after relief of bilateral complete obstruction,
where urine is hypotonic with large amounts of NaCl, K\ Mg", & phosphate
Post-obstructive
• If stable: replacement with IV fluids in amounts less than urinary losses
diuresis
• If with hypematremia due to loss of electrolyte-free water: give half
saline IV fluid while monitoring serum and urine sodium
458
'
SECTION FOUR
OVERVIEW OF EXTRACORPOREAL THERAPY FOR ESRD
MULTIDISCIPLINARY CARE & PATIENT EDUCATION
Patients should ideally be under care of nephrologist as early as CKD stage 4 (eGFR <30 ml/
min/1.73m'). Social, psychological, and physical preparation for renal replacement therapy (RRT)
are best accomplished by involving a multidisciplinary team, including but not limited to the
nephrologist, vascular or transplant surgeon, nutritionist, and social workers. Patient and family
education are very important especially in choosing the treatment modality that is best.

HEMODIALYSIS (HD)
Form ofRRT that utilizes the principles of solute diffusion across a semipermeable
membrane
• Metabolic waste products move via diffusive clearance through a concentration gradient
from the blood circulation into the dialysate
In addition, convective clearance also occurs because of solvent drag, with solutes being
swept along with water across the membrane during ultrafiltration (UF)

I. HEMODIALYSIS CIRCUIT

Dialyzerinflow
pressure monitor

Airtrap
and air
detector

Blood Filtered blood


pump

removed for
dialysis
The hemodialysis circuitstartsat the vascularaccessfromwherethe bloodis pumpedout fromthe
arterialblood line of the vascularaccessinto the dialyzer.Afterwhich,bloodis returnedfrom the
dialyzerbackintothevenousbloodlineof the vascularaccess.Alongthiscircuit,chambers,sideports
and variousmonitorsare attached,and are usedto infusesalineor anticoagulant, and to measure
pressuresanddetectanyleakageof air intothe system.

A. Hemodialyzer Membrane
0 A plastic chamber with the ability to perfuse blood and dialysate compartments
simultaneously at very high flow rates
0 Usually, a hollow fiber dialyzer composed of bundles of capillary tubes through which
blood circulates while dialysate travels on the outside of the fiber bundles
° Can be reprocessed for reuse (using reprocessing agents/machines) to decrease cost
and wastage
459
B. Blood Delivery System
1. Vascular Access

ACCESS I DESCRIPTION
• Inserted into internal jugular vein, subclavian or femoral vein
Central venous • Can be tunneled or non-tunneled type
catheter • Easiest to create and can be used immediately
• At risk for bleeding, thrombosis, infection
• Longest patency rate
Arteriovenous
• Need to be created early on to allow maturation period (6-8 weeks)
fistula
• Will need large bore needles for cannulation once mature for HD use
Arteriovenous • Can be created if with poor vascular anatomy
graft • Very prone to thrombosis

2. Extracorporeal circuit
• HD machine is composed of blood pumps and tubings, various safety monitors, and
dialysate solution delivery system
• Blood pumps move blood through the vascular access, into the dialyzer membrane,
and back again into the patient
• Blood flow rate (QB or BFR): rate at which blood is pumped into the circuit, which
typically ranges from 250-450 ml/hr during usual HD treatment
• Ultrafi!tration rate can be programmed & adjusted depending on goals of treatment

C. Dialysis Circuit
0Dialysate delivery system dilutes concentrated dialysis solution with purified water, & can
be adjusted in terms of temperature, HCO3 concentration, Na- conductivity, & flow rate
0Dialysate flow rate (QD or DFR), the rate at which dialysate solution is delivered into
the dialyzer membrane, is usually 500 mL/min
0Standard dialysate solution composition range is typically as follows:
Component I Concentration (mM) Component I Concentration (mM)

Sodium 135-145 Citrate 0.8-1.0 (2.4-3.0 mEq/L)

Potassium 2-3 Bicarbonate 25-35

Calcium 1.25-1.75 (2.5-3.5 mEq/L) Glucose 0-11

Magnesium 0.25-0.375 (0.5-0.75 mEq/L) pC02 40-110 mmHg


Chloride 98-124 pH 7.1-7.3
Acetate 3-8 Source:Daugirdas,JT et al. Handbookof Dialysis,2015

II. HEMODIALYSIS PRESCRIPTION


Every patient needing HD is unique and their circumstances vary, hence the need to
individualize the HD prescription
Parameters to consider are summarized below
PARAMETER I SIGNIFICANCE I PRESCRIPTION
Duration of • Most important determinant of • Usually lasts for 4 hours every session
session the amount of dialysis to be given • Rarely exceeds 6 hours
• Conventional frequency is 3 times
Frequency of • The number of treatment
weekly for 4 hours duration
treatment sessions done per week
• Less frequent (2x/week) in some
• Determine best dialyzer to use • For acute settings or initiation HD:
Dialyzer based on its capacity for solute use low flux & low efficiency dialyzers
membrane clearance, ultrafiltration capacity, • For maintenance (chronic) HD: high
biocompatibility, & reusabiliry flux & high efficiency dialyzers
460
PARAMETER I SIGNIFICANCE I PRESCRIPTION
• Rate at which blood from patient
is being pumped out & passed
through the dialyzer membrane • Usually set at 200-500 mL/min
• BFR influences the efficiency of • May use lower BFR in hypotensive
Blood Flow solute removal: patients
Rate (BFR) 0 Urea clearance increases steeply • During initial treatments, set BFR
as BFR reaches 300 mL/min at 150to 200 mL/min only to avoid
0 Urea clearance becomes less disequilibrium syndrome
steep as BFR goes up to
400-500 mL/min
• Rate at which the dialysate
solution passes through the
Dialysate • The usual DFR is at 500 mL/min
dialyzer membrane
Flow Rate • Optimum value for DFR is
• Increase in DFR also leads to
(DFR) 1.5-2times the BFR
increased urea clearance but
smaller effect in total
• Standard composition is designed • Adjust accordingly based on baseline
Dialysate
for acidotic, hyperphosphatemic, levels of electrolytes & acid-base
Solution
hyperkalemic chronic HD patients balance
• Target UF of <IO mL/kg/hr, but fluid
• UF is set to achieve the dry
removal rates of up to 10 mL/kg/hr
weight, defined as the lowest
are usually well tolerated
body weight a patient can
• Those with pulmonary edema or
tolerate without becoming
are edematous rarely require >4 L of
Ultrafiltration hypotensive
fluid during initial HD session
(UF)rate • Best to avoid large UF rates
• Patients without pulmonary
• May target lower fluid removal
congestion, edema or anasarca will
rates in the first session, then
rarely require >2-3L ofUF per session
target the excess during the
• Mayset UF to zeroifthere is hypovolemia
succeeding session the next day
or in normovolemic patients
• Unfractionated heparin (UFH) at
start of HD (fixed dose 1000-5000
U or weight-based dose 50 U/
kg bolus) followed by continuous
• Used to prevent blood clotting infusion (1000-1500 U/hr) until 15-60 '
during HD treatment minutes before the end of dialysis
• Blood clotting during HD or as intermittent boluses as needed
Anti-
results in blood loss and during dialysis
coagulation
decreases solute clearance by • In patients at risk of bleeding, low-
decreasing available dialyzer dose heparin (500-1000 U bolus
surface area followed by 500-750 U/hr), regional
anticoagulation, dialyzers coated
with heparin, or no anticoagulation
may be appropriate
• May also use LMWH (e.g.,enoxaparin)
• Cooling of dialysate solution
Dialysate
is associated with decreased • Usually at 35 'C to 37 'C
temperature
mortality in HD patients
Intradialytic
• May include glucose solutions (D50-50) for hypoglycemia, iron sucrose
& post-dialysis
and other IV iron preparations, or erythropoietin stimulating agents
medications

461
Sample Prescription for Chronic Hemodialysis:
Frequency • 3xweekly
Duration • 4 hours
Access • AV fistula, left radiocephalic
Dialyzer • High flux dialyzer
BFR • 300 ml/min
DFR • 6ooml/min
UFR • Target dry weight of 60 kg, max at 10 mL/kg/hr
Dialysate • Standard bicarbonate bath
'
Anticoagulation • Routine heparinization (2ooou bolus then IOOO units per hour)
Dialysate temp • 36°C
• Erythropoietin alpha 4000 units SC after HD twice weekly
Medications
• Iron sucrose IOO mg IV infusion after HD once weekly

REFERENCES
I. Basile OP, Anderson MD, and Sutton TA. Pathophysiology of acute kidney injury. Compr Physiol. 2012; 2(2):1303-1351.
2. Bellomo R, Ronco C, Kellum JA, Mehta RL and Palevsky P. Acute renal failure-definition, outcome measures, animal
models, fluid therapy and information technology needs: Second International Consensus Conference of the Acute
Dialysis Quality Initiative (ADQI) Group. Crit Care. 2004; 8(4): R.204-212.
3. Bleyer, A. Indications for initiation of dialysis in chronic kidney disease. In: UpToDate, 2013.
4.Cockcroft OW and Gault MH. Prediction of creatinine clearance from serum creatinine. Nephron. 1976;16(1):31-4.
5. Daugirdas JT, Blake PG, Ing TS. Handbook of Dialysis, Fifth Edition. USA:Wolters Kluwer Health, 2015.
6.Fatehi P, Hsu C. Chronic kidney disease (newly identified): clinical presentation and diagnostic approach in adulcs.
UpToDate. Accessed online August 25, 2021
7. Gameata, L, Stancu, A, Dragomir, D, Stefan G, and Mircescu G. Ketoanalogue-Supplemenced Vegetarian Very Low-
Protein Diet and CKD Progression. J Amer Soc Nephrol. 2016. 27(7):S:2164-2176.
8.lkizlerTA, Burrowes JD, Byham-Gray LD, et al; KDOQI Nutrition in CKD Guideline Work Group .. KDOQI clinical practice
guideline for nutrition in CKD: 2020 update. Am J Kidney Dis. 2020;76(3) (suppl 1):S1-S107.
9.Kidney Disease: Improving Global Outcomes (KDIGO) Acute Kidney Injury Work Group. KDIGO Clinical Practice
Guideline for Acute Kidne}· Injury. Kidney Inter., Suppl. 2012; 2:1-138.
10. Kidney Disease: Improving Global Outcomes (KDIGO) Blood Pressure Work Group. KDIGO 2021 Clinical Practice
Guideline for the Management of Blood Pressure in Chronic Kidney Disease 2021. Kidney Inter. 2021;99:559-569.
11. Kidney Disease: Improving Global Outcomes (KDIGO) CKD Work Group. KDIGO Clinical Practice Guideline for the
Evaluation and Management of Chronic Kidney Disease. Kidney Inter., Suppl. 2013; 3:1-150
12. Kidney Disease: Improving Global Outcomes (KDIGO) Diabetes Work Group. KDIGO 2020 Diabetes Management in
CKD Guideline. Kidney Inter. 2020; 98: 839-848.
13. Kidney Disease: Improving Global Outcomes (K.DIGO) Work Group. KDIGO Clinical Practice Guideline for Anemia in
Chronic Kidney Disease. Kidney Inter., Suppl. 2012. 2:1-335.
14. Kidney Disease: Improving Global Outcomes (KDIGO) Work Group. KDIGO clinical practice guideline for the diagnosis,
evaluation, prevention and treatment of chronic kidney disease mineral and bone disorder (CKD-MBD). Kidney Inter.,
Suppl. 2017; 76: S1-128.
15. Kumar S, Berl T. Diseases of water metabolism. In: Atlas of Diseases of the Kidney, RW Schrier [ed]. Philadelphia:
Current Med. 1999.
16. Levey AS, Bosch JP, Lev,:isJB, Greene T, Rogers N, and Roth D. A more accurate method to estimate glomerular filtration
rare from serum creatinine: a new prediction equation. Modification of Diet in Renal Disease Study Group. Ann Int Med.
1999; 130(6): 461-70.
17. Matsushita K, Selvin E, Bash LO, Astor BC, and Coresh J.Risk implications of the new CK.D Epidemiology Collaboration
(CKD-EPI) equation compared with the MORD Study equation for estimated GFR: the Atherosclerosis Risk in
Communi1ies (ARIC) Study. Am J Kidney Dis. 2010; 55 (4): 648-59.
18. National Kidney Foundation: K/DOQI Clinical Practice Guidelines for Chronic Kidney Disease: Evaluation, Classification
and Stratification. Am J Kidney Dis. 2002; 39(2 Suppl 1):S1-266.
19. Philippine Practice Guidelines Group: Task Force on Urinary Tract Infections. Philippine Clinical Practice Guidelines on
Diagnosis and Management of Urinary Tract Infections in Adults 2013 Update.
20. Schrier RW. Manual ofNephrology, 8th Edition. Philadelphia: Lippincott Williams & Wilkins. 2015.
21. Singh, AK. Protein restriction and progression of chronic kidney disease. In: UpToDate, 2013.
22. Skorecki K, Chertow GM. Marsden PA. Ta.al MW, Yu ASL (ed). Brenner n.nd Rector's The Kidney, 10th ed. Philadelphia:
Elsevier. 2016.
23. Turk C, Knoll T, Petrik A, Sarica K, Skolarikos A, Straub M, et al. EAU Guidelines on Diagnosis and Conservative
Management ofUrolithiasis. Eur Urol. 2016; 69(3): 468-74.
24. Workgroup KD. K/DOQl clinical practice guidelines for cardiovascular disease in dialysis patiencs. Am J Kidney Dis.
2005;45:S1-S153.

462
ENDOCRINOL
SECTION ONE
APPROACH TO DISEASES IN ENDOCRINOLOGY
APPROACH TO COMMON COMPLAINTS IN ENDOCRINOLOGY
It is important to evaluate patients in the context of their presenting symptoms, review of systems,
family and social history, and exposure to medications that may affect the endocrine system.

Common Endocrine and Metabolic Disorders in the Adult


DISORDER I SCREENING/TESTING RECOMMENDATION*
Type 2 diabetes • Fasting plasma glucose (FPG), oral glucose tolerance test (OGTT),
mellitus or HbA1c every 3 years starting at age 45 (earlier if high-risk)
• Cholesterol levels at least every 5 years (more often if high-risk)
Hyperlipidemia
• Lipoprotein (LDL, HDL) and triglyceride analysis
Metabolic
• Waist circumference, FPG, blood pressure (BP), lipid profile
syndrome
• TSH (confirm with free T4)
Hypothyroidism
• Screen women after age 35 and every 5 years thereafter
Graves' disease • TSH, free T 4
• Physical examination of the thyroid
Thyroid nodules
• Fine-needle aspiration biopsy, as indicated
• Bone mineral density measurements in postmenopausal or women
Osteoporosis
>65 years or in men at risk
Hyper- • Serum calcium
parathyroidism • Parathyroid hormone (PTH) if calcium is elevated
VitaminD
• Serum 25-OH vitamin D
deficiency
Infertility • Semen analysis in males & assess ovulatory cycles in females
Hyper- • Prolactin (PRL) level
prolactinemia • Cranial MRI with contrast, if not medication-related
Male
• Testosterone, luteinizing hormone (LH)
hypogonadism
• Review medications, hypogonadism, liver disease, possible
Gynecomastia
syndromic associations such as Klinefelter's

Generalprinciplein thework-upof disorders


in endocrinology
• Disorders
withhormone hypersecretion=dosuppression testing
•-Disorders
withhormone deficiencies=dostimulationtesting
Source:JamesonJL,et al. Harrison'sPrinciples 20thed1t1on,
of InternalMed1c1ne 2018

PHYSICAL EXAMINATION
I OVERVIEW OF EXAMINATION TECHNIQUES
• Appropriateness of appearance for age and sex
Overall
• Growth & maturation, including height, weight & body mass index (BM!)
appearance
• Vital signs
• Pay particular attention to the body habitus, face, skin and
Inspection
appendages, eyes, genitalia, breasts
• Focus on thyroid, breasts, testes
Palpation
• Other glands may need imaging (not accessible by physical examination)
465
II. EXAMINATION OF THE THYROID GLAND
• Inspect the seated patient from the front and side
Inspection
• Note any surgical scars, obvious masses, or distended veins
• The patient's neck should be slightly flexed to relax the neck muscles
• Identify the cricoid cartilage to locate the isthmus, then follow
laterally to locate either lobe
• Palpate the thyroid using both thumbs, with both hands from behind
Palpation
or while facing the patient
• Check thyroid consistency by asking the patient to swallow
• Palpate for lymphadenopathies in the supraclavicular and cervical
regions of the neck
• Auscultate for bruits over the insertion of the superior and inferior
Auscultation
thyroid arteries (supero- or inferolaterally)
• Pemberton's sign: symptoms of faintness with facial congestion &
Other
external jugular venous obstruction when arms are raised above the head
maneuvers
• Thyroglossal duct cysts: move upward when the tongue is extended
• Drawing is the best way to record findings
Recording • Make an estimate of thyroid size
findings • Note thyroid size, consistency, nodularity, & any tenderness or fixation
• Note size, location, and consistency of any nodules

III. ASSESSMENT OF OTHER GLANDS (IN TERMS OF FUNCTION)


Adrenal • BP & pulse (recumbent & upright), weight, skin pigmentation, color
function • Presence of body hair, body fat distribution
• Individual target gland function (adrenal cortex, thyroid, gonads)
Pituitary
• Growth parameters
function
• Visual fields (pituitary enlargement may compress optic chiasm)
Parathyroid
• Musculoskeletal irritability, hydration, mental status
function
Pancreatic
• Vital signs, mental status, point-of-care CBG testing
islet function

IV. GENERAL FORMULAS


B 0 dfl,/ M ass I n d ex (BMI) an d W.aist-Hiv Ratio (WHR)
Wei~htink~ • Calculated by dividing the weight (in kg) by the
BMI=
(Heie;ht in m)' height (in meters) squared
• Measure waist circumference at the midpoint
between the lower margin of the last palpable
rib and the top of the iliac crest
WHR = Waist circumference (cm) • Hip circumference should be measured around
Hip circumference (cm) the widest portion of the buttocks
• Abdominal obesity is defined as a WHR:
0 >0.90 for males

0 >0.85 for females

Ideal Body Weiqht (IBW)

In males = ro6 lbs + (6 lbs per inch over 5 feet) • Estimates the ideal weight of a
In females = 100 lbs + (5 lbs per inch over 5 feet) person based on sex and height
• Does not reflect fat or muscle
(divide by 2.2 to convert to kilograms) percentage in one's body
2000
FP.AnnPharTTI
Source:PaiMPandPaloucek
466
METABOLIC SYNDROME & DIABETES MELUTUS
THE METABOLIC SYNDROME
I. ETIOPATHOGENESIS
Metabolic abnormalities that confer an increased risk of cardiovascular disease & DM
Insulin resistance: most accepted and unifying hypothesis in metabolic syndrome
Central adiposity: key feature of metabolic syndrome
Hypertriglyceridemia is an excellent marker of insulin resistance
Other associated conditions: non-alcoholic fatty liver, hyperuricemia, polycystic
ovarian syndrome (PCOS), and obstructive sleep apnea (OSA)

II. DIAGNOSIS
A. BM! Classification
CLASSIFICATION I WHO (kg/m 2 ) I ASIANS (kg/m')

Underweight <18.5 <18.5


Normal 18.5-24.99 18.5-22.9
Overweight ;,:25 ;,:23
Obese I ;,:30 ;,:25
Obese II ;,:35 ;,:30
Obese III ;,:40 ---
Source:RegionalOfficefor theWesternPacificof theWHO.HealthCommunications
Australia.2000
B. Diagnostic Criteria (requires <!3of the following)

I NCEP: ATP 1112001 I HARMONIZING DEFINITION

• <!90cm (M) or <!80cm (F) in South


Central obesity Asian, Chinese, and ethnic South and
• >102cm (M)
(Waist Central American
• >88 cm (F) • <!94cm (M) or 280 cm (F) in Europid,
Circumference)
Sub-Saharan African, Middle Eastern
Hypertriglyceridemia • TG 2:150mg/dL or use of specific medication
Low HDL-cholesterol • <40 mg/dL (M) or <50 mg/dL (F) or use of specific medication
Hypertension • BP <!130systolic or ;,85 diastolic or use of specific medication
Fasting Glucose • ;,100 mg/dL or use of medication or previously diagnosed T2DM
EducationProgramandAdultTreatmentPanel111,
Source:NationalCholesterol 2001

III. MANAGEMENT OF OBESITY


AO fM

THERAPY

Diet&
exerciseh If withct.morbids If withct.morbids + + +
Drug
therapy - +
If withco-morbids + + +

Surgery' - - Considered +
Ifwithco-morbids If withco-morbids +
• Cut-offforAsians
'Dietarytherapy:reductionin overallcalorieconsumption (mosteffective:dietarymodification
+ exercise)
'Surgery:classifiedintothree(restrictive,
restrictive-malabsorptive,
andmalabsorptive)
467
B. US-FDA Approved Medications for Obesity•
DRUG I MECHANISM I ADVERSE EFFECTS
Short· r.erm 7iTeqt'7!ent (s12 wee k)s
Phentermine • Sympathomimetic • Dry mouth, insomnia, dizziness,
8-37.5mg OD amine irritability, increased BP, tachycardia
Long-Term Trea~ment (>I-;?."!eeks)
• Abdominal discomfort, oily stool,
Orlistat
• Lipase inhibitor flatulence, fecal urgency
60-120 mg TID
• Malabsorption of fat-soluble vitamins

Phentermine/ • Sympathomimetic • Paresthesia, constipation, insomnia,


topiramate ER amine/anticonvulsant nasopharyngitis, dry mouth,
7.5 mg/46 mg OD combination increased blood pressure

Naltrexone/ • Opioid antagonist/


• Nausea, constipation, headache, dry
bupropion aminoketone
mouth, insomnia, increased blood
8mg/90mg, antidepressant
2 tabs BID pressure, tachycardia
combination
• Glucagon-like • Hypoglycemia
Liraglutide
peptide I receptor • Nausea, bowel movement changes
3mgSCOD
agonist • Injection site reactions
•Lorcaserinhas been withdrawnby the FDAlast 2020for reviewdue to possibleincreasedcancer risk

DIABETES MELLITUS (DM)


I. ETIOPATHOGENESIS
• Group of metabolic disorders that share the common phenotype of hyperglycemia
• DM is defined as the level of glycemia at which diabetes-specific complications occur
A. Classification (based on the pathogenic process that leads to hyperglycemia)
TYPE I REMARKS
Type1DM • Due to autoimmune ~-cell destruction leading to absolute insulin deficiency
Type2DM • Due to progressive loss of ~-cell insulin secretion with insulin resistance
Gestational • Diabetes diagnosed in 2nd or 3rd trimester of pregnancy that was not clearly
DM overt diabetes prior to gestation
• Monogenic diabetes syndromes (e.g., neonatal diabetes, maturity-onset
Other diabetes of the young or MODY)
types • Diseases of exocrine pancreas (e.g., cystic fibrosis)
• Drug- or chemical-induced diabetes (e.g., glucocorticoids, treatment of HIV)

B. Classification of Glucose Tolerance


GLUCOSE
TOLERANCE
I FASTING PLASMA
GLUCOSE (FPG) I
GLUCOSE AFTER ORAL
GLUCOSE CHALLENGE I HbA1C

<100mg/dl <140mg/dl
Normal <5.7%
(5.6mmol/L) (7.8mmol/L)
Impaired 100-125mg/dl 140-199mg/dl
glucose (5.6-6.9mmol/L) (7.8-11mmol/L) 5.7-6.4%
homeostasis (Impaired
FastingGlucose) (Impaired
GlucoseTolerance)
Diabetes ~126mg/dl ~200 mgldl
2:6.5%
mellitus (DM) (7.0mmol/L) (11.1mmol/L)
Source of tables above:AmericanDiabetesAssociation.DiabetesCare; 2021
468
II. MANIFESTATIONS
• Classic symptoms: polyuria, polydipsia, polyphagia, nocturia, weight loss
• Fatigue, weakness, blurred vision, frequent superficial infections, & poor wound healing
A. Acute Complications ofDM
Diabetic ketoacidosis
0

Hyperglycemic hyperosmolar state


0

Eye Disease • Coronary artery disease • Gastrointestinal


• Retinopathy (CAD) (gastroparesis, diarrhea)
(nonproliferative/proliferative) • Peripheral arterial • Genitourinary (uropathy,
• Macular edema disease (PAD) sexual dysfunction)
Neuropathy • Cerebrovascular • Dermatologic
• Sensory and motor (mono-/ disease (CVD) • Cataracts and glaucoma
polyneuropathy) • Periodontal disease
• Autonomic • Hearing loss
• Increased risk for infection
Nephropathy • Cheiroarthropathy (thick
• Albuminuria skin+ reducedjoint mobility)
• Declining renal function
Source:JamesonJL,et al. Harrison'sPrinciples
of InternalMedicine
20thedition,2018

III. DIAGNOSIS
A. Criteria for the Diagnosis ofDM
Any of the following ful/fi/s the criteria for OM:
HbA1c' • ;;:6,5%
Fasting Plasma Glucose (FPG)• • ;;:126mg/dl (7.0 mmol/L}
z-hour plasma glucose during 75-g OGTT • ;;:200mg/dl (11.1mmol/L)
• ;;:200mg/dl (11.1mmol/L)withclassic
Random Blood Sugar symptomsof hyperglycemia
'Perform HbA1C withan NGSPstandardizedmethod
• Fasting:definedas no caloricintakeforat least 8 hours
'Random: definedas withoutregardto timesince last meal
For FPG,2-hourPG, or A1C criteria:in the absence of unequivocalhyperglycemia,
these criteria
shouldbe confirmedby repeat testingon a differentday

• Begin at age 45 years, then every 3 years


• Screen at earlier age if they are overweight (BM! 2:23 kg/m' in Asians)+ one additional
risk factor for DM•
• May use HbA1C,FPG, or 2 hour plasma glucose after 75g OGTT for screening
*Riskfactorsfor DMincludethe following:
HbA1C;;:5.7%,IGT,or IFGon previoustesting
0

° First-degreerelativewithdiabetes
High-risk
0 ethnicity(e.g.,African
American,Latino,NativeAmerican,AsianAmerican,PacificIslander)
GestationalDM(GDM)
0

Hypertensionor historyof CVD


0

HDL<35 mg/dl and/ortriglycerides>250mg/dl


0

Physicalinactivity
0

Polycysticovarysyndrome
0

Otherconditionswithinsulinresistance(e.g., severe obesity,acanthosisnigricans)


0

469
MANAGEMENT OF DIABETES MELLITUS
I. PREVENTION OR DELAY OF T2DM (for the Prediabetics: !GT, IFG, or HbA1C 5.7-6.4%)
ASPECT I MANAGEMENT

• Annual monitoring for the development ofDM among those with prediabetes
• Diabetes Prevention Program (OPP) could reduce incidence ofT2DM - goals are:
Lifestyle 0 Achieve and maintain a minimum of?% weight loss
0 150 mins of physical activity per week similar in intensity to brisk walking
• Screen & treat modifiable risk factors for cardiovascular disease
• Metforrnin for prevention ofT2DM considered in prediabetes, especially if:
• BM! 2:35kg/m'
Drug
• Age <60 years
therapy
0 Women with prior GDM
• Rising HbA1C despite lifestyle modification
Source:AmericanDiabetes
Association.
DiabetesCare;2021
II. OVERVIEWOF THE MANAGEMENT OF OM
TYPE I MANAGEMENT

• Most should be treated with multiple daily injections of prandial & basal
insulin (or continuous subcutaneous insulin infusion)
• Insulin is the mainstay of therapy for T1DM:
Type1 0 Starting insulin dose: calculate total insulin requirement usually at
OM 0.4-1.0 units/kg/day
0 50% of computed value given as basal insulin
• Pramlintide (amylin analog): FDA-approved for T1DM (induces weight loss &
lowers insulin doses)
• Metforrnin is the preferred initial pharrnacologic agent for T2DM
• Consider insulin (with or without additional agents) in newly diagnosed T2DM
Type2
who are symptomatic and/or have HbA1c >IO%&/or blood glucose 2:300mg/dL
OM
• If patients have ASCVD, CKD, or CHF, use an SGLT2 inhibitor or a GLP-1
receptor agonist with CV benefit as part of regimen, independent of HbA1c

III. HYPOGLYCEMIC AGENTS

• Gliclazide 30-120mg/d PO
Sulfonylureas • Glibenclarnide 2.5-20mg/d PO
(SU) • Glimepiride 1-8mg/d PO • Increases insulin • Hypoglycemia,
• Glipizide 5-40 mg/d PO secretion weight gain
Non- • Repaglinide 0.5-16 mg/d PO
sulfonylureas • Nateglinide 120 mg/d PO
Insulin Sensitizers
• Decreases • Weight loss, GI
hepatic glucose upset, vitamin
• Metformin production & B12deficiency,
Biguanides
500-2000 mg/d PO slightly improves metallic taste,
peripheral glucose lactic acidosis
utilization (rare)
• Decreases insulin
• Edema,
resistance &
Thiazolidinediones • Pioglitazone weight gain,
increases glucose
(TZD) 15-45mg OD PO osteoporosis,
utilization in
anemia
peripheral tissues
470
TYPES I EXAMPLES I MECHANISM I SIDE EFFECTS
Intestinal Absorption Inhibitors
Alpha•
• Acarbose 25-100 mg TIO PO • Inhibits
glucosidase • Weight loss,
• Miglitol 25-100 mg TIO PO intestinal
inhibitors diarrhea,
absorption of
Lipase flatulence
• Orlistat 120 mg TIO PO sugars
inhibitors
Incretin-Related Drugs
"
• Sitagliptin 25-100 mg OD PO
• Saxagliptin 2.5·5 mg OD PO
DPP-IV • Linagliptin 5 mg OD PO
• Headache,
inhibitors • Vildagliptin 50-100 mg BID PO
nasopharyngitis
(orDPP4i) • Gemigliptin 50 mg OD PO
• Tenegliptin 20-40 mg OD PO
• Prolongs
• Alogliptin 25 mg OD PO
endogenous
• Exenatide 5-10 mcg BID SC GLP-1 action
GLP-1 • Liraglutide 0.6·1.8 mg OD SC
• Skin irritation
receptor • Albiglutide 30-50 mg weekly SC
after injection,
agonists • Dulaglutide 0-75-1.5
mg weekly SC
nausea
(GLP-1RA)" • Lixisenatide l0·20 mcg OD SC
• Semaglutide 0.25-1mg weekly SC

Others
"' -
• Urinary & vaginal
Sodium- infections, risk
• Dapagiiflozin 5-10 mg OD PO
Glucose Co· • Increases of amputation
• Canagliflozin 100-300mg OD PO
Transporter•2 urinary glucose (canagliflozin),
• Empagiiflozin 10-25mg OD PO
Inhibitors excretion dehydration,
• Ertugliflozin 5-15mg OD PO
(SGLT2-i) diabetic
ketoacidosis (rare)
• Slows gastric
Amylin • Pramlintide 15-120mcg OD emptying • Nausea,
agonists• SC • Decreases hypoglycemia
glucagon
• Binds bile acids
in intestinal • Constipation,
tract, increasing hyperoiglyceridemia,
Bile acid
• Colesevelam 3.75 g/d PO hepatic bile acid decreased
sequestrants
and decreasing absorption of
hepatic glucose other medications
production
• Activates
dopaminergic
receptors and
Dopamine-2 • Bromocriptine 0.8-4.8 mg/d • Dizziness, nausea,
modulates
agonists PO fatigue, rhinitis
hypothalamic
regulation of
metabolism
'Parenteral drugs
Source:Jameson JL, et al. Harrison'sPrinciplesof InternalMedicine20th edition,2018
AmericanDiabetesAssociation.DiabetesCare;2021
471
IV. INSULIN THERAPY
Provided to T1DM patients (insulin-deficient) and T2DM patients who are symptomatic
and/or have severe hyperglycemia or uncontrolled with oral hypoglycemic agents alone
Common side effects are hypoglycemia and weight gain
Adjust doses in renal insufficiency

• Required to regulate metabolic processes even in the absence of meals


• Usual "basal insulin": given as intermediate or long-acting insulin
Basal insulin
• Intermediate-acting insulin usually given in portions of 2/3 in AM
& 1/3in PM

• Required to cover glycemic excursions following a meal


• Usual "bolus insulin": given as short- or rapid-acting insulin
Bolus insulin • Rapid-acting insulin given within 15-20mins or immediately before
meals
• Short-acting insulin given within 30-45 minutes before meals
• Supplemental doses of short- or rapid-acting insulin given to
Correctional
correct elevations in glucose that occur despite the use of basal and
insulin
bolus insulin

Lispro
Aspart <15mins 30-90 mins 2-4 hrs

Glulisine
Short-Acting I11:slin
Human Regular 30-60 mins 2-3 hours 3-6 hours

1ntermediate-Actir&9
Insulin
Isophane/Human NPH 2-4 hours 4-10 hours 10-16hours

Basal Insulin Analogs


Glargine 2-4 hours
Minimalpeak Up to 24 hours
Detemir 1-4 hours
activity
Degludec 30-90 mins >24hours
Source:Jameson
JL,et al. Harrison's
Principles
of InternalMedicine
20thedition,2018
AmericanDiabetes Association.
Diabetes
Care;2021

472
V. INITIATING ANTIHYPERGLYCEMIC THERAPY
A. Monotherapy
° Consider if baseline HbA1C ~1.5-2% above target, and patient not markedly
symptomatic (e.g., polyuria or polydipsia)
0 Start with metformin + lifestyle modification
0 Metformin has high efficacy, low hypoglycemic risk, may have neutral effect or
decrease in weight, and low cost

B. Combination Non-Insulin Therapy


0Defined as metformin & lifestyle modification PLUS one or more of the drugs below
° Consider if:
• Baseline HbA1C >I.5-2% above target, AND patient not markedly symptomatic, OR
• HbA1C target not achieved even after 3 months of monotherapy
Insulin
I SU
I I
TZD DPP4i
I SGLT2i I GLP-1 RA I (Basal)
Efficacy High High Intermediate Intermediate High Highest
Hypoglycemia
Moderate Low Low Low Low High
risk
Weight Gain Gain Neutral Loss Loss Gain
Costs Low Low High High High High

First-linetherapy with
metforminand lifestylemanagement I
1
.

No
1 1 l
With HF(especiallyLVEF<45%)
With ASCVDor
high risk for ASCVO- or CKD(eGFR30-60 or If HbA1c above target
UACR>30 mg/dl)
Add either regimen Add either regimen And with hypoglycemia
(independent of HbA1c (independent of HbAl c level) risk, add:
level) Preferred: SGLT2iii eGFR DPP4i,or
GLP-1RA (liraglutide, adequate (empaglillozin, GLP-1RA, or
dulaglutide, dapaglillozin, canaglillozin) SGLT2i,or
semaglutide) or GLP-1RA (liraglutide, TZD
SGLT2iii eGFR dulaglutide, semaglutide) ii
adequate SGLT2inot tolerated, And need to minimize
(empaglillozin, contraindicated, or eGFR not
weight gain:
dapaglillozin, adequate If need to promote
canaglillozin) weight loss: add GLP-1
RAor SGLT2i
If need to be weight
neutral or GLP-1
RA/SGLT2inot
tolerated, add DPP4i
And cost is an issue, add:
SU, or
TZD

ASCVD:alherosclerotic cardiovascular disease HF: heart failure


CKD:chronic kidney disease
'High risk for ASCVD:~55 years old with LVHor ~50% coronary,carolid, or lowerextremitystenosis)
Source:AmericanDiabetesAssoc1at1on.
DiabetesCare; 2021
473
C. Combination Non-Insulin and Insulin Therapy
° Consider this if:
• Baseline HbA1C ;,10%
• FBS ;,300 mg/dL

.
• Patient markedly symptomatic
HbA I C target not ach'1eved even aft er 3 mont hs o f com b'mat1on non-msu I'mt h erapy
Initiate basal insulin
(usually with metlormin ± other OHA)

Initial dosing: 10 u/doy or 0.1 - 0.2 u/kg/day


Adjust: 10-15% or 2-4 u once or twice weekly
until target fasting plasma glucose is reached

I
If HbA 1c still uncontrolled:
Consider combination insulin therapy

I
I I
Add 1 rapid acting insulin Change to premixedinsulinBID
before largest meal before breakfast and supper

Initial dosing: 4 u/day or 0.1 u/kg or Initial dosing: Divide current basal dose to either:
10%of basal dose 2/3 AM and 1/3 PM, or
1/2 AM and 1/2 PM

Adjust: increase dose by 1-2 u or Adjust: increase dose by 1-2 u or

y i---J
10-15% once or twice weekly to achieve target 10.15% once or twice weekly to achieve target

If goals not met, consider changing to


alternative regimen or refer to specialist

• Possibleregimens:
0 If alreadyon oralcombination:addbasalinsulinor GLP-1RA
0 If alreadyon GLP-1RA:addbasalinsulin
0 If alreadyon optimally-titrated
basalinsulin:addGLP-1RAor mealtimeinsulin
• Metformin shouldbe maintained whileotheroralagentsmaybediscontinuedonan individualbasisto
avoidunnecessarily complexor costlyregimens
Source:AmericanDiabetes
Assoc1at1on.
DiabetesCare;2021& lnzucch1,
SE,et al. DiabetesCare2015

VI. DRUGS AND THEIR PRIMARY AREAS OF GLYCEMIC CONTROL


Use of various insulin & non-insulin therapies, whether as monotherapy or
combination, have differing effects on glycemic control
Their primary areas of glycemic control are as follows:
MONOTHERAPY
I COMBINATION THERAPY I INSULIN

Sulfonylureas Sulfonylurea + Lispro


Metformin Metformin
FPG FPG Aspart PPG
Thiazolidinediones Sulfonylurea +
Regular
Thiazolidinediones
Meglitinides NPH
Sulfonylurea +
GLP-1agonists GLP-1agonist Glargine
PPG FPG FPG
Acarbose Metformin+ PPG Detemir
OPP-IV Inhibitors Meglitinide Degludec

FPG:FastingPlasmaGlucose;PPG:PostprandialGlucose
'Priorityof glycemictargetgoalsmaydependon the patient'sHbA1C levels:
• If HbA1Cis <7% ControlPPGfirst
• If HbA1Cis 7-9% ControlbothFPGand PPG
• If HbA1Cis >9% ControlFPGfirst
V.CurrMedResOpin.2003
Source:Fonseca,
474
VII. MONITORING

Sulfonylureas FPG at 2 weeks, HbA1C at 3 months


1-2 weeks
Meglitinides FPG at 2 weeks, HbA1Cat 3 months, PPG at initiation
Metformin 2-3 weeks FPG at 2 weeks, HbA1C at 3 months
Acarbose 2-4 weeks HbA1C at 3 months, PPG at initiation
Thiazolidinediones 1-2 months FPG at 4 weeks, HbA1C at 3-6 months
OPP-IV Inhibitors 2 weeks FPG at 2 weeks, HbA1Cat 3 months, PPG at initiation
Source:Fonseca,V.CurrMedRes Opin.2003& NathanDM,et al. DiabetesCare. 2009

B. Self-Monitoring of Blood Glucose (SMBG)


0Should be performed by patients on multiple-dose insulin or insulin pump
• Prior to meals and snacks, occasionally postprandially, and at bedtime
• Prior to exercise or critical tasks such as driving
• When they suspect low blood glucose
• After treating low blood glucose until they are normoglycemic

C. Goals of Treatment
GLYCEMIC CONTROL INDEX I GOAL
HbA1C(Primary Goal)' <7.0%"

Preprandial capillary plasma glucose 80-130 mg/dl (4.4-7.2 mmol/L)

Peak postprandial capillary plasma glucose <180 mg/dl (<10.0 mmol/L)


'HbA1ctestingshouldbe done:
At least twotimesa year in patientswhoare meetingtreatmentgoals & stableglycemiccontrol
0

Quarterlyin patientswhose therapyhas changedor whoare not meetingglycemicgoals


0

"Target HbA1c maychange dependingon population:


Morestringenttarget HbA1c <6.5%:considerfor individualswithshort durationof diabetes,T2DM
0

treatedwithlifestylechanges or metforminonly,longlifeexpectancy,or no cardiovasculardisease


Source:AmericanDiabetesAssociation.DiabetesCare;2021
SamveI Case
48/F (weight of8o kg) with polyuria and polydipsia sought consult.
• HbAIC12% • FBS 320 mg/dL
• eGFR 92 mL/min/I.73 m' • ASCVD with clinically-diagnosed HF & LVEF40% on 2D Echo
• Since the patient is symptomatic, HbA1Cis 12%and FBS is 320 mg/dL, we start the following:
0Metformin 500 mg 2 tabs BID (max dose) to optimize weight loss (since eGFR is normal), AND
0Basal insulin at a starting dose of 16units (0.2 U/kg) q24h (e.g., insulin Glargine 16units
q24h), with titration of 2-4 units weekly to reach FPG target
• Additionally, since the patient has ASCVD (specifically HF with reduced EF <45%),we add an
SGLT2 inhibitor (dapagliflozin, empagliflozin, canagliflozin) independent of the HbA1C level
After 3 months, her HbA1C decreased to 8.5% (still did not meet target of <7%) ...
• Following the algorithm, we consider combination injectable therapy:
0Option #1:Add 4 units regular- or rapid-acting insulin before largest meal (e.g., insulin
Aspart 4 units before lunch)
0Option #2: Shift basal insulin to premixed insulin BID: 1/, pre-breakfast and 1/, pre-supper,
or 2/3 pre-breakfast and 1/3pre-supper (e.g.,shift current Glargine 16units q24h to Premix
Aspart + Protaminated Aspart 30/70 at IO units pre-breakfast and IO units pre-supper)
IfHbA1cstilluncontrolled after3 months,see fullalgorithmin the ADAguidelinesforfurthermanagement.Atthis
point,considerreferralto specialistDisdaimer.Management ofeverypatientshouldstillbeon a case-to-casebasis.
475
VIII. ANTICIPATORY CARE
ASPECT I MANAGEMENT
• Minimum weight loss of 5% body weight for those who are overweight/obese
• 150min/week moderate-to-vigorous intensity aerobic exercise, spread over
at least 3 days/week, with no more than 2 consecutive days without exercise
• 2-3 sessions/week of resistance exercise on non-consecutive days if no
Lifestyle contraindications
changes • Prolonged sitting should be interrupted every 30 mins with bouts of
physical activity
• Diet acceptable for T2DM & prediabetes (e.g., Mediterranean, DASH)
• Smoking cessation and moderation of alcohol intake
• Referral to a dietitian or nutritionist as necessary
• Best treated with ACE inhibitor (ARB for ACEi-intolerant patients)
Hypertension
• DASH-style diet by reducing sodium & increasing potassium intake
• Maximally-tolerated statins regardless of baseline lipid levels for all
diabetics
Dyslipidemia
• Check lipid profile at diabetes diagnosis, at initiation of statins, at 4-12
weeks after initiation of statins or change in dose, & annually thereafter
• Consider for primary prevention in patients with increased cardiovascular
Antiplatelet
risk: those .:50 years with at least r additional major risk factor (family
agents
history of premature disease, HPN, dyslipidemia, smoker, albuminuria)
(aspirin)
• Should be given to all for secondary prevention
• Recommended daily protein allowance of o.8 g/kg/day
Nephropathy
• Monitor renal function annually (more frequent if with CKD)
• Assessment for peripheral neuropathy
0 Type I diabetes: within 5 years after diagnosis
0 Type 2 diabetes: shortly after diagnosis
Neuropathy • Include assessment of temperature or pinprick sensation, vibration
sensation, and ro-g monofilament test
• Examinations at least annually
• Pregabalin, duloxetine, gabapentin: initial therapy for neuropathic pain
• Screening (comprehensive eye examination by an ophthalmologist)
0 Type I diabetes: within 5 years after diagnosis
Retinopathy 0 Type 2 diabetes: shortly after diagnosis
• Exams every 1-2years
• Influenza vaccine to all diabetics ;,6 months of age
• Pneumococcal vaccine (PPSV23) if;,2 years old, revaccination once
2'65years old
Immunization • Hepatitis B vaccine in unvaccinated adult diabetics 18-59years old
(consider administering if 2'60 years old)
• Tetanus, diphtheria, and pertussis (TDAP) vaccine every IO years
• Varicella zoster vaccine if .:50 years of age, even if previously vaccinated
• Regular assessment for symptoms of diabetes distress, depression,
Psychosocial
anxiety, and disordered eating behaviors
screening
• Adults ;,65 years old should be screened for cognitive impairment
Family
• For women of reproductive age
planning
Oral care • Referral to a dentist for comprehensive dental and periodontal care
Autoimmune • Patients with T1DM should be screened for autoimmune thyroid
screening disease soon after diagnosis
Source:ArnencanDiabetes DiabetesCare;2021
Assoc1at1on.
476
HYPERGLYCEMIC CRISES IN DIABETES
I. ETIOPATHOGENESIS & MANIFESTATIONS
Associated with absolute or relative insulin deficiency combined with counterregulatory
hormone excess, volume depletion, and acid-base abnormalities
Decreased insulin-glucagon ratio promotes gluconeogenesis, glycogenolysis & ketogenesis
Infection is the most common precipitating factor
Others include: discontinuation of or inadequate insulin therapy, comorbidities (e.g.,
pancreatitis, ACS, stroke), restricted water intake (bedridden, altered thirst response of the
elderly), and drugs (e.g., steroids, thiazides, sympathomimetics, pentamidine, antipsychotics)
DIABETIC KETOACIDOSIS HYPEROSMOTIC
I (OKA)
• Due to increased gluconeogenesis
I HYPERGLYCEMIC STATE (HHS)
• Greater degree of dehydration & higher
& glycogenolysis, and impaired endogenous insulin secretion (vs OKA)
peripheral glucose utilization • Primarily in older patients with T2DM
Pathogenesis • Formerly a hallmark o( type I OM • Insulin levels inadequate to facilitate
• Serum ketones (indicator of OKA) glucose utilization by insulin-sensitive
should be measured in type I OM tissues but adequate to prevent
when glucose is >JOOmg/dL lipolysis and ketogenesis
• Nausea, vomiting • Polyuria, weight loss
Symptoms • Thirst, polyuria • Diminished oral intake
• Abdominal pain, dyspnea • Confusion, lethargy, coma
• Tachycardia, tachypnea, dehydration • Profound dehydration +/- hypotension
• Kussmaul respirations • Altered mental status
Signs
• Abdominal tenderness • No nausea, vomiting, abdominal pain,
• Decreased sensorium Kussmaul respiration (unlike OKA)
• Triad develops over 24 hours • Develops within several weeks
• Uncontrolled hyperglycemia • Characterized by severe hyperglycemia,
Course
0 Metabolic acidosis hyperosmolality, dehydration
0 Increased total body ketones • Absence of ketoacidosis

II. DIAGNOSIS

PARAMETER HHS

Plasma glucose (mg/dL) >250 >250 >250 >600


Arterial pH 7.25-7.30 7.00-7.24 <7.00 >7.30
Serum bicarbonate (mEq/L) 15-18 10to <15 <10 >15
Urine & serum (plasma) ketones Positive Small
Serum osmolality (mOsm/kg)• Variable >320
Anion gap .. >10 >12 >12 Variable
Alteration in sensorium Alert Alert/drowsy Stupor/coma Stupor/coma
Serum Creatinine Slightlyi Moderatelyj
Electrolytes
Na•(meq/L) 125-135 135-145
K• Normalto increased Normal
Mg,. Normal Normal
CJ· Normal Normal
Phosphate Decreased Normal
Arterial PCO (mmHg) 20-30 Normal
'Formulaforeffective
serumosmolality:
2[Na]+ glucose(mg/dl)/18
**Formulaforaniongap:(Na- [Cl+ HCOJ)
Source:K1tabch1
et al. Hyperglycem1c
Crisesin AdultPatientswithDiabetes.DiabetesCare;2009
JamesonJL,et al. Harrison'sPrinciples
of InternalMedicine20thedition,2018
477
III. MANAGEMENT
Admit to ICU & measure capillary blood glucose (CBG) every 1-2 hours
Monitor blood pressure, pulse, respirations, mental status, and fluid intake & output
every 1-4 hours
Assess serum electrolytes, ABG, and renal function
Main components of management: IV fluids+ potassium+ bicarbonate+ insulin

A. Intravenous (IV) Fluid Therapy

Determine hydration status


I I
I
! I !
Severe dehydration Mild dehydration Cardiogenic shock

Give IVF 0.9% Compute corrected Hemodynamic


NaCl 1l/hr Na monitoring/pressors

I
! !
Normal or increased Na I I Decreased Na

Give IVF 0.45% NaCl Give IVF 0.9% NaCl


250-500 ml/hr depending 250-500 ml/hr depending
on hydration status on hydration status

I I
!
When serum glucose ~200 mg/dl for DKA, or ~300 mg/dl for HHS,
shift IVF to 0.45% NaCl with 5% dextrose 150-250 ml/hr

• 15-20ml/kg/hror 1-1.5literspNSSduringthefirst hour(unlesswithriskof congestion)


• OnceCBGis ~200-250mg/dl, shiftfluidsto D5-IVF(glucose-containing)
• IVFreplacement shouldcorrectestimateddeficitswithinthefirst24 hours
patients,monitorserumosmolality& cardiac,renal,andmentalstatusto avoid
• In renal/cardiac
iatrogenicoverload
Source:Kitabchi
et al. Hyperglycemic
CrisesinAdultPatients
withDiabetes.
Diabetes
Care;2009

B. Potassium
0Insulin therapy, correction of acidosis, and volume expansion decrease serum K·
concentration
0Determine serum K· and establish adequate renal function & urine output -50 cc/hr
(before correcting K-)
• lfK- <3.3 mEq/L: hold insulin & give 20-30 mEq/hr until K· >3-3 mEq/L
• lfK- 3.3-5.2 mEq/L: add 20-30 mEq K- in each liter of!VF to keep K· between
4-5 mEq/L
• IfK· >5.2 mEq/L: do not give K· (monitor every 2 hours)

C. Bicarbonate
0If pH <6.9: start 100 mmol HC03- + 400 mL sterile water+ 20 mEq KC! to infuse for
2 hrs (repeat every 2 hours until venous pH >7.0 and monitor serum K+ every 2 hours)
0If pH 2'.6.9: no need to give HC03-

Source:Kitabchiet al. Hyperglycemic


Crisesin AdultPatientswithDiabetes.DiabetesCare;2009

478
D. Insulin Therapy
Regular insulin
I IV route: DKA and HHS I
I
1 1
Bolus: 0.1 u/kg, then
Infusion:0.14 u/kg/hr
Infusion: 0.1 u/kg/hr I I
1 l
If serum glucose does not decrease by 10% in the first hour,
give bolus: 0.14 u/kg, then continue previous infusion
l l
DKA HHS
If serum glucose :S2OOmg/ dL If serum glucose :S3OOmg/ dL
Decrease infusion to Decrease infusionto
0.02 - 0.05 u/kg/hr, or 0.02 - 0.05 u/kg/hr
Shift to rapid acting insulin,
0.1/kg SC q2 hours

Target serum glucose: Target serum glucose:


150-200 mg/dl until DKA resolves 200 - 300 mg/dL until mentally alert

I I
1
Monitor creatinine, BUN, electrolytes, venous pH, and glucose q2-4 hrs until stable
After resolution of DKA or HHS and patient is able to eat, start SC multidose insulin
regimen
To transfer from IV to SC: overlap IV infusion for 1-2 hrs after SC insulin is begun to
ensure adequate plasma insulin levels and prevent recurrence of DKA/HHS
Dose of insulin
Insulin no'ive patients: start at 0.5- 0.8 u/kg/doy and adjust PRN
Known diabetics: give at the dose they were receiving before the onset of
DKA/HHS, as long as it was controlling sugars adequately

Regularinsulinpreferablyby IV route(shorthalf-lifeandeasytitration)is mainstayof therapy


InitialIV bolus(0.1units/kg)is given,then infusionis startedat 0.1units/kg/hr(seealgorithm)
MonitorCBGsQ1-2hourlyand titrateinsulindrip as follows:
0 If CBGincreases:re-boluswith 0.1units/kgintravenously anddoubledrip rate
0 If CBGdecreases0-49mg/dl fromprevioushour:increasedrip rateby 50%
0 If CBGdecreasesby 50-75mg/dl: maintaincurrentdrip rate
0 If CBGdecreasesby>75 mg/dl: holddripfor 30 minutesand resumeafterat 50%of
previousrate
WhenCBGis -200 mg/dl in DKAor -300 mg/dl in HHS,maydecreaseinsulininfusionrate
to 0.02-0.05units/kg/hrto maintainCBGbetween150-200mg/dl (in DKA)or 250-300mg/dl
(in HHS)untiltheyare resolved
Source:K1tabch1
et al. Hyperglycem1cCrisesInAdultPatientswithDiabetes. DiabetesCare;2009

E. Criteria for Resolution of DK.Aand HHS


DKA I HHS
Plasma glucose <200 mg/dL & two of • Normal serum osmolality
the following: • Improvement of mental status
• Serum bicarbonate level ~15mEq/L
• Venous pH >7.3
• Calculated anion gap s12 mEq/L
479
E , . U. ,I r D.

SAMPLE FORMULATION
(DEPENDS ON PHYSICIAN)
'
I SAMPLE ORDER:
Let us say we want to give our patient 2
units of insulin per hour (via msulm drip)
Drip I: Add 20 units of insulin (HR) in 100 mL
pNSS: this gives a concentration of 0.2 unit/mL For Drip 1:Give 10 mL per hour (IO mL/hr)
(20 units/100 mL)

Drip 2: Add 50 units of insulin (HR) in 100 mL


pNSS: this gives a concentration of 0.5 unit/mL For Drip 2: Give 4 mL per hour (4 mL/hr)
(50 unhs/100 mL)
Drip 3: Add 100 units of insulin (HR) in 100 mL
pNSS: this gives a concentration of1 unit/mL For Drip 3: Give 2 mL per hour (2 mL/hr)
(100 units/100 mL)

Sample Order for Insulin Drip: Start insulin drip at 0.1 unit/kg/hr & titrate to desired blood q/ucose
If the patient weighs 50 kg, start insulin drip at 5 units/hr. Ifwe decide to use Drip #3 from
the example above, our order will be: insulin drip 100 units HR+ 100 mL pNSS at a rate of
5 mL/hr (to deliver 5 units/hr).

IV. COMPLICATIONS
COMPLICATION I REMARKS
Hypoglycemia
• Due to overzealous treatment ofDKA with insulin and bicarbonate
& hypokalemia

Hyperchloremic • Usually seen during the recovery phase of OKA


non-anion gap • Caused by loss ofketoanions plus excess infusion of chloride-
acidosis containing fluids during therapy
• Occurs in -0.3-1% of OKA in children but rare in adults
• Associated with 20-40% mortality rate
Cerebral • Presents with headache, gradual deterioration in level of
edema consciousness, seizures, sphincter incontinence, pupillary changes,
papilledema, bradycardia, elevation in BP & respiratory arrest
• Treated with mannitol and mechanical ventilation

S amp,eI Case
History and Physic~/ Examination
19/F was brought to the ER for unresponsiveness. She was noted to have I-month history of
unexplained weight loss, nocturia, and increased thirst and a 1-week history of abdominal
pain and recurrent vomiting. She had non-sustained wakefulness.
BP: 120/80 mm Hg, HR: 110bpm, RR: 22 breaths/min, Temp: 38°C
Height: 158cm, Weight: 45 kg, Waist Circumference: 24 inches, Hip Circumference: 34 inches
Dry oral mucosa, deep and rapid breathing, soft and non-tender abdomen
.,
-Laboratory Results II -
Random blood sugar 420 mg/dl (23.3 mmol/L)

ABG pH 6.9, PO, 98 mmHg,PC0 2 25 mmHg, HC03 8 mEq/L,0 2 sat 98%

CBC Hgb 140 g/L, Hct 45%, WBC 13,000/µL

Electrolytes Na 138 mEq/L, K 4.2 mEq/L, PO, 2 mEq/L (0.64 mmol/L),Cl 108 mEq/L

Renal Function BUN23.8 mgldl (8.5 mmol/L),Creatinine 1.15 mg/dl (102 µmol/L)

Urinalysis WBC 30-40/hpf, RBC4-5/hpf, ketone +++,protein +, glucose +++

480
A. Compute for Body Mass Index, Waist-Hip-Ratio, and Serum Osmolalitv
= weight (in kg)/ [height (in m)]'
Body Mass = 45 / (1.58)'
Index(BMI) = 18 kg/m'
Interpretation: UNDERWEIGHT
= waist circumference/ hip circumference
Waist-Hip- = 24 / 34
Ratio(WHR) =0.71
Interpretation: NORMAL
= 2 x (Na+ K) + (BUN in mg/dL)/2.8 + (RBS in mg/dL)/18
Serum
= 2 X (138+ 4.2) + 23.8/2.8 + 420/18
Osmolality
= 316 mOsm/kg

B. Interpret the Arterial Blood Gas (ABG)


• Check the pH, HCO3, and pCO2; since pH is <7-4,it is ACIDOTIC
• Check the 6HCO3 and 6pCO2:
Step 1: Primary
Disturbance • 6HCO3 = (24 - 8)/24 = 0.67
• 6pCO2 = (40 - 25)/40 = 0.38
• Since 6HCO3 > 6pCO2, the acidosis is METABOLIC in origin
• In metabolic acidosis, the expected compensatory response is a 1.25
mmHg decrease in pCO2 per 1 mEq/L decrease in HC03
• Compensation = (24 - 8) X 1.25
Step 2: Predict = 16X 1,25
Compensation = 20 mmHg expected decrease in pCO2
• Expected pCO2 = 40 - expected decrease in pCO2
=40-20
=2ommHg
Step 3: • Since the patient's actual pCO, of25 mmHg is higher than
Secondary the expected compensatory level (20 mmHg), she also has
Disturbance CONCOMITANT RESPIRATORY ACIDOSIS

•AG= Corrected Na - (Cl+ HC~),)


Step 4: = [Actual Na+ 0.016 (RBS m mg/dL- 100)] - (Cl+ HCO,)
Compute
Anion Gap (AG) = [138+ 0.016 (420 - 100)] - (108 + 8)
= 27.12(normal anion gap is 8-12)

I
• Our patient has PRIMARY HIGH-ANION GAP METABOLIC
ACIDOSIS with CONCOMITANT RESPIRATORY ACIDOSIS
Final
• This is actually consistent with our case. Our patient is likely in
Interpretation
OKA (causing the metabolic acidosis) and her comatose state is
causing CO2 retention, leading to respiratory acidosis.

C Wh . ?
:
DIAGNOSIS
I BASIS
• RBS >250 (420 mg/dL)
• Ketone-positive urine(+++)
Diabetic
• Arterial pH <7-00 (6.90)
Ketoacidosis,
• Non-sustained wakefulness
Severe
• High anion gap (27.12)
• Serum osmolality of316 mosm/kg

Complicated • CBC: WBC 13,000


UT! • Urine: WBC 30-40 cells/hp£
Underweight • BM!: 18 kg/m'
481
...
DWh at 1syour m1t1aI management
• Since patient has moderate to severe dehydration (dry mucosa, high
Fluid
hematocrit), start with 900 mL PNSS x I hour, reassess hydration status
therapy
through urine output monitoring & readjust rate accordingly
• Give 5 units (0.1 units/kg) regular insulin IV bolus, then start insulin drip
5 units/hr (0.1 units/kg/hr) via infusion pump (may mix 50 to 100 units in
Insulin
100 cc PNSS in a soluset)
therapy
• Check CBG q1, if CBG does not decrease -50-75 mg/dL/hour, give 6 units
regular insulin IV bolus (0.14 units/kg)
• Since pH <6.9, start 100 mmol HCO3 in 400 mL sterile water at 200 mL/h
Bicarbonate for 2 hrs
• Repeat every 2 hours until pH reaches >7.0
• Since K is within 3.3-5.2(4.2 mEq/L) and bicarbonate drip will be started,
Potassium
add 30 mEqs KC! per I L ofIVF
• CBG every hour
Monitoring • RBS, BUN, Na, K, Cl, VBG every 2-4 hours
• BP, HR, respirations, mental status, fluid intake & output every 1-4 hours
• Admit to ICU & place on NPO
• Antibiotics for urinary tract infection (e.g., ceftriaxone 1g IV q24)
Others • Paracetamol 3.00mg/IV q4 for fever
• F-PIfor stress ulcer prophylaxis (e.g., omeprazole 40 mg IV OD)
• Anti-embolic stockings and/or enoxaparin for DVT prophylaxis
. ·" " - '
II ,.,"' "-' -~

.A:Jt(!r
8 hours . .
• Hematocrit 35%
• Patient awake, coherent, moist
• RBS 180 mg/dL, BUN 16.8 (6 mmol/L)
mucosa, stable vitals
• Serum osmolality 299 mOsm/kg
• Urine output 50 cc/hr
• Anion gap 16,Venous pH 7.2, HCO, 12mEq/L
• 02 saturation 98%
• Na 136mEq/L, K 5.3 mEq/L, Cl 108 mEq/L
. a:','
.
Adjust Man.agement1!s
follows . .. ""
&! -"- -
• Decrease insulin drip to 2-3 units/hr (half the previous drip rate) to maintain CBG 150-200
• May decrease monitoring of CBG to 92-4 hours
• Since K is 5.3 mEq/L, discontinue KC! incorporation & repeat serum K after 4-6 hrs
• Since pH is already 7-2, discontinue bicarbonate drip

After 24 h(!urs.. u - ".? .,; a

• Patient awake, coherent, moist mucosa, stable vitals


• RBS 160 mg/dL x 4 hours
• Serum osmolality 298 mosm/kg
• Anion gap 12,Venous pH 7.4, HCO3 18 mEq/L
• Electrolytes: Na 138mmol/L, K4.5 mEq/L, Cl 108 mmol/L
11 - - - - ~'
"' ...'•'-·
A.djustManagement gsJlollows. .. __,
"'~-a
Fluids • Shift !VF to pNSS 1Lto run for 12-16hours
• Start SC insulin 24 units/day (0.5-0.8 units/kg/day for an insulin nai've
patient) with 50% as basal & 50% as prandial insulin; adjust accordingly
Insulin
• Continue insulin drip for 1-2hours after starting the SC dose
• Decrease CBG monitoring to q4 then to TID pre-meals once feeding
Potassium • Continue monitoring while on insulin drip and correct accordingly
,.
Bicarbonate • May decrease monitoring to q12 hours
• Decrease monitoring of RBS, BUN, Na, K, Cl, VBG to every 12hours
Monitoring
• Decrease monitoring of VS & fluid intake/output to every 4 hours
482
DIABETIC FOOT ULCER
I. ETIOPATHOGENESIS
• Diabetics are prone to foot ulcers
Development is attributed to: neuropathy, ischemia, infection, and immune impairment
Neuropathy: most common underlying etiology of foot ulceration

II. CLASSIFICATION OF DIABETIC FOOT ULCERS


A. Wagner Classification System (to assessulcerdepth & presenceof osteomyelitisorgangrene)
GRADE I DESCRIPTION
Gradeo • Pre- or post-ulcerative lesion, completely epithelialized
Grade I • Partial/full-thickness ulcer; superficial wound
Grade2 • Penetrates the tendon or capsule
Grade3 • Deep with osteitis
Grade4 • Partial foot gangrene
Grade5 • Whole foot gangrene

B. University of Texas System


0Assesses ulcer depth, presence of wound infection, and the presence of clinical signs
oflower-extremity ischemia
0Better predictor of outcome than the Wagner system
0There are four stages within each wound grade
0E.g., A superficial wound that is ischemic but not infected is classified as Grade I, Stage C
DEPTH I PRESENCE OF INFECTION/ISCHEMIA

• Pre- or post-ulcerative lesion, • Clean wound:


0
completely epithelialized A (-) ischemia, (-) infection
• Non-ischemic, infected wound:
I • Superficial wound B (-) ischemia, (+) infection
• Wound penetrates tendon or • lschemic, non-infected wound:
2
capsule C (+) ischemia, (-) infection

• Wound penetrates bone and • lschemic & infected wound:


3 joint D (+) ischemia, (+) infection

C. IWGDF Classification System

Uninfected • No local or systemic symptoms or signs of infection


I
• Infection with no systemic manifestations, and involving only
Mild
2 skin or subcutaneous tissue; any erythema does not extend >2
infection
cm around the wound
• Infection with no systemic manifestations; and involving tissue
Moderate
3 deeper than skin or subcutaneous tissue, or erythema extending
infection•
~2 cm around the wound
• Any foot infection with ~2 manifestations of systemic
Severe
4 inflammatory response syndrome (SIRS) or sepsis (See Infectious
infection•
Diseaseschapterfor details)
•Add"O":if wilhassociated
osteomyelitis
Source:International
WorkingGroupon the DiabeticFoot.DiabMetabResRev;2020.
483
III. DIFFERENTIALS FOR A FOOT ULCER

ULCER
I USUAL
ETIOLOGY
• Diabetic
I DESCRIPTION

• Located at sites of trauma, such as areas of callus


Neuropathic
neuropathy ("OM formation, bony prominence, or parts of the foot
ulcer
foot") exposed to mild chronic trauma
• Small, annular, pale, tender, circumscribed, &
Arterial • Peripheral arterial desiccated
(ischemic) occlusive disease • Located on distal areas oflimbs (e.g., toes, heels)
ulcer (PAOD) • May progress to tissue necrosis & gangrene
• May co-exist in diabetic patients with PAOD
• Large with irregular borders, erythematous and
moist (shiny appearance)
• Chronic venous
Venous • Located near the medial or lateral malleolus
insufficiency
ulcer • Chronic edema may impart hemosiderin
(CVI)
deposition in the skin, giving rise to a "brawny
appearance "

IV. DIAGNOSIS
Plain radiographs to assess possibility of osteomyelitis
Culture from debrided ulcer base or from purulent discharge/wound aspiration
(culture from wound surface is not helpful)
Others: MRI (most specific modality), bone scan of the foot, indium-m labeled WBC scan

V. MANAGEMENT
A. Screening & Surveillance
0 Screen for distal polyneuropathy at diagnosis and at least annually thereafter
0 Annual comprehensive foot examination:
• Inspection for calluses, discolorations, and deformities
• Assessment of foot pulses and ankle-brachia! index (ABI)
• Tests for loss of protective sensation using IO-gmonofilament plus: vibration using 128-
Hz tuning fork, pinprick sensation, ankle reflexes, & vibration perception threshold

B. Genera Management
• Pressure avoidance on the wound and immediate surrounding area
0 Total contact cast (TCC):gold standard for off-loading of a foot wound

0 Removable cast walker (RCW)

0 Pneumatic cast boots


Non-
• Hydrocolloid wound dressings
pharmacologic
• Use of multi-disciplinary approach
management
0 Referral to foot care specialists for preventive care & surveillance
0 Referral to vascular specialists if with significant claudication or
positive ABI for further vascular assessment
0 Referral to orthotists (shoe fitters) for proper footwear for high-risk feet

• Antibiotics if with prominent discharge, local erythema, or cellulitis


• Symptom relief from painful neuropathy
Pharmacologic 0 Tricyclic antidepressants (e.g., norrriptyline)
management 0 Anti-convulsant (e.g., pregabalin, gabapentin)
0 Selective serotonin reuptake inhioitors (e.g., duloxetine)
0 Opioid and opioid-like drugs (e.g., tramadol)

Surgical • Debridement and removal of all dead and macerated tissue


management • Limited amputation (may be followed by vacuum-assisted closure)

484
DIABETES MELLITUS AND PREGNANCY
I. DEFINITIONS
TERM*
I DEFINITION

Pregestational
• DM diagnosed in a woman even before pregnancy
DM (pre-GDM)
• DM diagnosed during pregnancy (usually 2nd and 3rd trimesters) that is
Gestational not clearly overt DM
DM(GDM) • The word "gestational" implies that diabetes is induced by pregnancy
• Most important perinatal correlate ofGDM is fetal macrosomia

OvertDM • Pregnant woman who meets the standard non-pregnant criteria for OM
*Bothpre-GDM& overtDMpossesshigherratesof maternal(chronicHPN,eclampsia, thyroiddysfunction,
renaldisease)and fetal (CNSmalformation,
fetaldemise)comorbiditiesduringpregnancyas comparedto
GDMalone,andalsopossesshigherratesof dystocia,failedlaborinduction,andcaesareandelivery.

II. DIAGNOSIS (ONE-STEP STRATEGY)


A. General Screenin for GDM

• All pregnant patients with any risk factors should be screenedat


High risk for the soonest possible time with an FBS, HbA1C,or RBS
GDM' • Those with negative screening result should be screenedagain at
24-28weeks AOG using a 2-hour 75gram OGTT

No risk factors for • Women with no risk factors should be screenedat 24-28weeks
GDM AOG using a 2-hour 75gram OGTT
'Risk factorsinclude:priorGDM,glucosuria, familyhistoryor first-degree
relativewithT1DMandT2DM,
PCOS,age>25yearsold,overweightor obesitybeforepregnancy, macrosomia in previousor current
pregnancy, polyhydramnios in currentpregnancy
andintakeof drugsaffectingcarbohydrate metabolism
Screeningfor Filipino Gravidas:
• Sinceall Filipinogravidasareconsidered "highrisk"by raceor ethnicgroup(PacificIslander),all should
be screenedfor T2DMduringthefirst prenatalvisit(FBS,HbA1C, or RBS)
• If the initialtest(FBS,HbA1C or RBS)is normal,screeningfor GDMshouldbedoneat 24-28weeks
usinga 2-hour75gramOGTT

BS C ·c .
I
I
DIAGNOSIS OF GDM* DIAGNOSIS OF OVERT DM IN
PREGNANCY**
Any of the following: Any of the following in their first visit:
• FPG: ~92 mg/dL (5.1mmol/L) • FPG ~126mg/dL (7 mmol/L)
• 1-hr PPG: ~180mg/dL (10.0mmol/L) • RBS ~200 mg/dL (II.I mg/dL)
, 2-hr PPG:~153mg/dL (8.5mmol/L) • HbA1c ~6.5%
'Screeningfor GDMshouldbe basedon the IADPSGcriteriausing75 g OGTT(seebelow)
"Appliesto womenwithoutknowndiabetesantedatingpregnancy
Procedure forthe75 g OGTT:
• Askthe patientto haveat least3 daysof unrestricted
carbohydrateintake(>150g carbohydratedaily)
• Performthetestin the morning(7-9am)afteran 8-14hourovernightfast (wateris allowed;smokingor
physicalactivityarenot permittedduringthetest)
• Extractthe initialfastingbloodglucose(FPG)sample
• Givepatienta standard75-gglucosesolutionin 250-300ml of water,to be ingestedwithin5 minutes
• Extractbloodsamplesafter1 hour(1-hrPPG)and2 hours(2-hrPPG)of theglucoseload
• Thetestshouldnotbe doneduringacuteillness
Source.
Jimenoetal.UNITEforDiabetes Ph1hpp1nes.
2011
IADPSG Recommendations. Diabetes
Care2010
Blumeretal.J ClinEndocrMetab.2013
485
Ill. MONITORING
Do SMBG before and I hour or 2 hours after the start of each meal
• Glycemic targets in pregnancy:
GDM I PREEXISTING TYPE 1 AND TYPE 2 DM *
• FPG ,;95 mg/dL (5.3 mmol/L) • FPG 70-95 mg/dL (3.9-5.3 mmol/L)
• 1-hr PPG s140 mg/dL (7.8 mmol/L) • 1-hr PPG 110-140mg/dL (6.1-7.8mmol/L)
, 2-hr PPG s120 mg/dL (6.7 mmol/L) , 2-hr PPG 100-120 mg/dL (5.6-6.7 mmol/L)
• HbAIC <6% optimal if no significant hypoglycemia
'Lower limitsdo not apply to diet-controlledT2DMin pregnancy
DiabetesCare; 2021
Source:AmericanDiabetesAssoc1at1on.
IV. MANAGEMENT
Medical nutrition therapy (MNT), with 3 meals & 3 snacks per day
Insulin therapy implemented if glycemic goals are not met after 1 week ofMNT or if
with fetal macrosomia (may use NPH, detemir, regular insulin, lispro, or aspart)
CBG targets during labor and delivery: 72-126 mg/dL (4.0-7.0 mmol/L)

HYPOGLYCEMIA
I. ETIOPATHOGENESIS
Glucose <70 mg/dL with symptoms that are relieved promptly after the glucose level is raised
Hepatic glycogen stores usually only last for 8 hours
Physiologic response to hypoglycemia:
0 1st line of defense: decreased insulin (primary glucose regulatory factor)
0 2nd line of defense: increased glucagon (primary glucose counter-regulatory factor)
0 3rd line of defense: increased epinephrine (critical when glucagon is deficient)
Some causes include drugs to treat OM, alcohol, critical illness, hormone deficiencies
(e.g., cortisol, glucagon, epinephrine), endogenous hyperinsulinism (e.g., tumors)
LEVELS I GLUCOSE I DESCRIPTION
• Sufficiently low for treatment with fast-acting
• :570mg/dl
I Glucose alert value carbohydrate & dose adjustment of glucose-
(3.9 mmol/L)
lowering therapy
Clinically significant • <54 mg/dl , Sufficiently low to indicate serious, clinically
2
hypoglycemia p.o mmol/L) important hypoglycemia
Severe • No specific • Associated with severe cognitive impairment
3 threshold
hypoglycemia requiring external assistance for recovery
Source:International
HypoglycaemiaStudyGroup.DiabetesCare2017
AmericanDiabetesAssociation.
DiabetesCare:2021
II. MANIFESTATIONS & DIAGNOSIS (Whipple's Triad)
Symptoms consistent with hypoglycemia
0 Neuroglycopenic symptoms: behavioral changes, confusion, fatigue, seizures, loss
of consciousness
0 Adrenergic symptoms: palpitations, tremors, anxiety, sweating
Low plasma glucose measured with a precise method (not a glucose monitor)
Relief of symptoms after the plasma glucose level is raised

Ill. MANAGEMENT
Major limiting factor in glycemic management of diabetes
If awake & conscious: initial dose of 15-20g oral glucose (preferred treatment), then
repeat SMBG after 15minutes
If unconscious or unwilling:
0 Parenteral glucose 25 g, if not practical SC or IM glucagon (1.0 mg in adults)
0 Glucagon will not work in the absence of glycogen stores (e.g., after binge drinking)
Manage primary reason for hypoglycemia
486
SECTION THREE
THYROID DISORDERS
HYPERTHYROIDISM
I. ETIOPATHOGENESIS
Consequence of excessive thyroid function
Classification:
0 Thyrotoxicosis: clinical state that results from excessive thyroid hormone (may be
exogenous or endogenous)
0Hyperthyroidism: form of thyrotoxicosis due to inappropriately high production of
thyroid hormone from the thyroid gland itself(endogenous)

PRIMARY I SECONDARY I THYROTOXICOSIS


WITHOUT
THYROTOXICOSIS THYROTOXICOSIS HYPERTHYROIDISM

• Graves' disease • TSH-secreting pituitary • Subacute thyroiditis


• Toxic multinodular goiter adenoma • Silent thyroiditis
• Toxic adenoma • Thyroid hormone • Thyroid destruction:
• Functioning thyroid CA resistance syndrome amiodarone, radiation,
metastasis • Chorionic gonadotropin- infarction of adenoma
• Activating mutation of secreting tumors • Ingestion of excess thyroid
TSH receptor • Gestational hormone (thyrotoxicosis
• McCune-Albright thyrotoxicosis factitia) or thyroid tissue
syndrome
• Struma ovarii
• Drugs: iodine excess
(Jod-Basedow)
Source:JamesonJL, et al. Harrison'sPrinciplesof InternalMedicine20thedition,2018

A. Graves' Disease
0 Accounts for 60-80% ofthyrotoxicosis (autoimmune disorder)
0 Typically occurs between 20 and 50 years of age; also occurs in the elderly
° Caused by thyroid-stimulating immunoglobulins (TSI), which are antibodies that
chronically stimulate TSH receptor
0 Diagnosis is straightforward in a patient with:
• Biochemically-confirmed thyrotoxicosis
• Diffuse goiter on palpation
• Ophthalmopathy
• Dermopathy

B. Toxic Multinodular Goiter (MNG)


0 Presence offunctional autonomy (in contrast to nontoxic MNG)
0 Includes subclinical hyperthyroidism or mild thyrotoxicosis

C. Hyperfunctioning Solitary Nodule (Toxic Adenoma)


0 Mutations lead to enhanced thyroid follicular cell proliferation and function
0 Thyrotoxicosis is usually mild
0 Disorder is suggested by:
• Thyroid nodule: generally large enough to be palpable
• Absence of clinical features suggestive of Graves' or other causes of thyrotoxicosis
0 Radioiodine Ablation (RA!): treatment of choice

487
II. CLINICAL MANIFESTATIONS
Cellular actions of thyroid hormone are mediated by T3 (active form of thyroid hormone)
Some of the most profound effects of increased levels occur in the cardiovascular system
Only moderate correlation exists between the degree of thyroid hormone elevation and
clinical manifestations
SYMPTOMS* I SIGNS*
• Hyperactivity, irritability, dysphoria • Tachycardia
• Heat intolerance and sweating • Atrial fibrillation in the elderly
• Palpitations • Tremor
• Fatigue and weakness • Goiter
• Weight loss with increased appetite • Warm, moist skin
• Diarrhea • Muscle weakness, proximal myopathy
• Polyuria • Lid retraction or lag
• Oligomenorrhea, loss of libido • Gynecomastia
*Arranged
in decreasing
orderof frequency.

III. DIAGNOSIS
AC D' : U d
DIAGNOSTIC
I COMMENTS/EXPECTED FINDINGS
• Single best screening test for hyperthyroidism (TSH is suppressed)
Sensitive TSH
• When thyrotoxicosis strongly suspected, it should be taken with
analysis
free T 4 and total T3
• Elevated (isolated T 4 toxicosis is occasionally seen when
FreeT4 RIA
hyperthyroidism is induced by excess iodine)
• Elevated (may be the only thyroid hormone elevated in 2-5%,
FreeT3 RIA
which is referred to as T3 toxicosis)
Thyroid-Stimulating
• Elevated (not routinely necessary)
Antibodies
• Thyrotoxicosis with elevated RAJ uptake:
0 Graves' disease: enlarged gland & increased uptake distributed

homogeneously
Radioactive Iodine 0 Toxic adenoma: focal areas of increased uptake with
(RA!) uptake & suppressed tracer uptake in the rest of the gland
thyroid scan 0 Toxic MNG: gland enlarged with distorted architecture &
multiple areas of relatively increased or decreased tracer uptake
(measures RA!
trapped into thyroid • Thyrotoxicosis with low RAJ uptake:
0 Painless (silent) thyroiditis
tissue)
0 Amiodarone-induced thyroiditis
0 Subacute thyroiditis (granulomatous, de Quervain's)
0 Others: Thyrotoxicosis factitia, struma ovarii

• Primary thyrotoxicosis: Graves' disease, multinodular toxic goiter,


Low High toxic adenoma
• Destructive thyroiditis, excess iodine intake, excess thyroid hormone
• Subclinical hyperthyroidism (ifnormal FT3)
Low Normal
• T3 toxicosis (if high FT3)
Normal • Secondary thyrotoxicosis: TSH secreting pituitary adenoma or thyroid
High
or High hormone resistance
Sources:JamesonJL,et al. Harrison's
Principles
of InternalMedicine
20thedition,2018
RossDS,et al. 2016ATAGuidelines; Thyroid.2016
488
IV. MANAGEMENT

Propylthiouracil , Inhibit thyroid • Common side


(PTU): peroxidase (TPO), effects: rash,
, wo-200 mg PO q6-8h reducing oxidation urticaria,
(initiation) & organification of arthralgia
• 50-100 mg PO/day iodide • Rare but major
(maintenance) • Reduce thyroid side effects:
Thionamides
antibody hepatitis, SLE-
Methimazole/ levels (unclear like syndrome,
Carbimazole mechanism) agranulocytosis
• w-20 mg PO q8-12h • PTU has added (e.g., sore throat,
(initiation) benefit in thyroid fever, mouth
• 2.5-IOmg PO/day storm (see below) ulcers)
• Propranolol • Control adrenergic
w-40 mgq6h symptoms, • Bradycardia,
Beta- • Atenolol especially in early AV block,
Blockers 25-wo mg OD-BID stages before anti- bronchospasm,
, Metoprolol thyroid drugs take hypotension
25-50 mg BID-TID effect
Source:RossDS,et al. 2016ATAGuidelines.Thyroid.
2016

B. Radioactive Iodine Therapy (RA!)


0 Damages gland through cytotoxic effects which yields the quickest resolution of
hyperthyroidism
0 Leads to post-procedural hypothyroidism and requires lifelong thyroid hormone
replacement therapy
° Carbimazole or methimazole must be stopped at least 2 days before RAJ
0 PTU has prolonged radioprotective effect & should be stopped several weeks before RAJ
° Contraindications:
• Pregnancy or breast-feeding
• Coexisting or suspicion of thyroid cancer

C. Surgical Management
0 Now uncommonly performed, unless with coexistent thyroid cancer
0 Surgical candidates:
• Pregnant patients who are intolerant to medications
• Non-pregnant patients who refuse RA!
• Patients with very large goiters
• Pediatric patients
° Complications include hypoparathyroidism and vocal cord paralysis (recurrent
laryngeal nerve injury)

489
THYROID STORM
I. ETIOPATHOGENESIS
Extreme accentuation of hyperthyroidism, usually with Graves' disease or toxic
multinodular goiter
<IO%of hospital admissions for thyrotoxicosis but reaches mortality rate of 20-30%
A. Pathophysiology
The point at which thyrotoxicosis transforms to storm is controversial
0

• No evidence that there is an increased production ofT3 or T4 causing the storm


• Magnitude of increase in thyroid hormones does not appear to be critical
Increased catecholamine receptors have been noted
0

Decreased binding to thyroid-binding globulin (increased FT3/FT4) is a possible mechanism


0

B. Precipitants of Thyroid Storm


Pre-existing thyrotoxicosis, untreated or partially treated
0

Surgery (e.g., poorly prepared patient with diffuse toxic goiter for thyroidectomy)
0

Other conditions associated with a rapid rise in hormone levels


0

• Withdrawal of anti-thyroid drug therapy


• Radioiodine therapy
• Vigorous thyroid palpation
• iodinated contrast dyes
• Salicylates (compete with albumin binding thus increasing free thyroid hormone levels)
° Conditions associated with a non-thyroidal illness: infection, stroke, trauma, OKA

II. MANIFESTATIONS
• Suspect thyroid storm in patients with fever and atrial fibrillation (AF)
• Apathy and coma are rare manifestations of storm
Some laboratory findings:
Increased FT 4 & FT3 and decreased TSH
0

Leukocytosis (shift to the left if with infection)


0

Mild hypercalcemia (increased bone turnover)


0

Liver function test abnormalities


0

Mild-moderate hyperglycemia
0

99 -109 bpm 5 Absent 0 Negative 0


uo-u9bpm 10 Mild (pedal edema) 5 Positive 10
120- 129 bpm 15 Moderate (bibasilar rales) 10
130-139 bpm 20 Severe (pulmonary edema) 15
~140 bpm 25 Atrial fibrillation 10
2. fhermon·gulatory I
J. CNS ljfect, 14-Ga~1ro111testi1wl-Hepatic
37,2-37.7 "C 5 Absent 0 Absent 0
Moderate (diarrhea,
37-8 - 38.2 "C 10 Mild (agitation) 10 10
vomiting, abdominal pain)

Moderate (delirium, Severe (unexplained


38.3 - 38.8 "C 15 20 20
psychosis, lethargy) jaundice)

38.9 - 39.3 "C 20 Severe(seizure,coma) 30


39-4 - 39-9 "C 25 ---
?:40.0°C 30
INTERPRETATION:
• < 25 points: UNLIKELY • 25-44points: Impending stonm • ~45 points: HIGHLYsuggestive
Source:Burch,HB;Wartofsky.
"Life-threatening
thyrotox1cos1s.
Thyroidstorm'
490
IV. MANAGEMENT
Goals of Management
0 Stop synthesis of new thyroid hormones
0 Halt release of preformed thyroid hormones
0 Prevent peripheral conversion ofT 4 to T3
° Control adrenergic symptoms associated with thyrotoxicosis
° Control systemic decompensation
0 Treat underlying cause
DRUG I DOSE I ACTION
Inhibition of new hormone production "

• 500-1000 mg LD • Inhibits thyroid peroxidase (TPO)


& 250 mg q4h PO , In high doses, it inhibits peripheral
PTU
or per rectum/per conversion ofT 4 to T3
NGT • Decreases circulating TS!
Methimazole • 60-80 mg/day PO • Inhibits thyroid peroxidase (TPO)
Inhibits preformed hormone r.elease .
Supersaturated
• 5 drops PO q6h I hr
Solution of
after PTU (Wolff-
Potassium Iodide
Chaikoff Effect) • Blocks thyroid hormone release
(SSKI)
• Decreases fractional turnover of thyroid
Lugol's solution • 4-8 drops PO q6-8h iodine and T 4 secretion rate
Sodium ipodate • 1-3g PO QID
lopanoic acid • 500 mg PO q12h
... ,,
Control of adrenergic symptoms ..
• Reduces sympathetic overdrive
• 60-80 mg PO q4h or
Propranolol • High doses decrease peripheral conversion
2 mg IV q4h
ofT4 to T3
Atenolol • 50-200 mg PO QID
• Cardio-selective (may be used in COPD &
Metoprolol • 100-200 mg PO q6-12
asthma)
Esmolol • 50-100mcg/kg/min IV
.,
Supportive management ,.·
• 325-650 mg PO
Paracetamol • For fever & pain
q4-6h as needed
• 300 mg IV bolus, • Decreases peripheral conversion ofT 4
Hydrocortisone
then 100 mg IV q8h • Addresses relative adrenal insufficiency
• Dextrose solutions are the preferred IV
Glucose • 5-10%solution
fluids to cope with high metabolic demand
Alternatives ....
• Mimics iodine
Lithium • 300 mg PO q8h • Inhibits coupling of iodotyrosinases &
peripheral conversion ofT 4
Potassium • Competitively inhibits the active iodide
•tgPOQID
perchlorate transport mechanism in the thyroid
Cholestyramine • 4gPOQID • Hastens removal ofT 4 and T3 from serum

491
HYPOTHYROIDISM
I. ETIOPATHOGENESIS
Results from under-secretion of thyroid hormone
• Iodine deficiency remains the most common cause worldwide
ETIOLOGY
I EXAMPLES
• Iodine deficiency
• Autoimmune thyroiditis (Hashimoto's thyroiditis)'
Primary
• Iatrogenic hypothyroidism• (e.g., previous thyroidectomy, RAJ or
hypothyroidism
neck radiotherapy, lithium/amiodarone-induced)
• Subacute lymphocytic thyroiditis
• Lesions compressing the pituitary (e.g., adenoma,
Central craniopharyngioma, meningioma, empty sella)
(secondary) • Sheehan syndrome
hypothyroidism • Autoimmune diseases (e.g., polyglandular disorders)
• Infectious (e.g., tuberculosis, syphilis)
*Inareas of iodine sufficiency,autoimmune (Hashimoto's thyroiditis)& iatrogenic causes are the most
common etiologies
Source: Garber JR, et al. Clinicalpractice guidelines for hypothyroidismin adults. Thyroid,2012

II. CLINICAL FEATURES


SYMPTOMS* I SIGNS*
• Tiredness, weakness • Dry coarse skin
• Dry skin • Cool peripheral extremities
• Cold intolerance • Puffy face, hands, and feet (myxedema)
• Hair loss • Diffuse alopecia
• Difficulty concentrating and poor memory • Bradycardia
• Constipation • Peripheral edema
• Weight gain with poor appetite • Delayed tendon reflex relaxation
• Dyspnea • Carpal tunnel syndrome
• Hoarse voice • Serous cavity effusions
• Menorrhagia (later oligomenorrhagia or
amenorrhea)
• Paresthesia
• Impaired hearing
*Arranged in decreasing order of frequency.
Source: Jameson JL, et al. Harrison's Principlesof Internal Medicine20th edition, 2018

492
III. DIAGNOSTICS
A. Common Diagnostics Used
DIAGNOSTIC I EXPECTED FINDINGS

• Elevated in primary hypothyroidism


Sensitive TSH
• May be normal or decreased in secondary hypothyroidism
analysis
(pituitary disease)
• Low in primary hypothyroidism
FreeT4
• May be normal in mild hypothyroidism ,.

Thyroid
• May be noted in autoimmune etiologies
autoantibodies
Thyroid scan and
• To determine the specific cause of hypothyroidism
ultrasound

B. Interpretation of Data
TSH I FT4 I DIFFERENTIALS
High Normal • Mild (subclinical) hypothyroidism
• Primary (overt) hypothyroidism
High Low • Autoimmune hypothyroidism (if thyroid autoantibodies are
positive)

• Drug effects, sick euthyroid syndrome


Normal Low
• Central hypothyroidism (e.g., pituitary disease)
Source:JamesonJL,et al. Harrison'sPrinciplesof InternalMedicine20thedition,2018
GarberJR,et al. Clinicalpracticeguidelines
for hypothyroidism
in adults.Thyroid,2012

JV. MANAGEMENT
• Levothyroxine (LT4): preparation of choice for treatment of hypothyroidism due to its:
0Efficacy in resolving symptoms
0Long-term experience of benefits
° Favorable side effect profile
0Ease of administration
0 Good intestinal absorption
0 Long serum half-life and low cost

L evot h,yroxzne (LT.4 ) S upp. ementatwn


• Start usually with 25-50 mcg/day (1.6mcg/kg/day)
Dosage • Use lower dosages of12.5-25 mcg for patients >60 years old and those
with cardiac disease
• Symptoms improve in weeks
• Increase dose by 25-50 mcg every 4 weeks until patient is clinically and
Duration
biochemically euthyroid
• Lifelong treatment is necessary
• Monitor plasma TSH 93-4 months (maintain in normal range)
Monitoring • For secondary hypothyroidism, monitor serum T 4 and other pituitary
hormones and give steroid replacement prior to LT4

493
GOITER AND NODULAR THYROID DISEASE
I. ETIOPATHOGENESIS
Goiter is defined as an enlarged thyroid gland
• Causes: biosynthetic defects, iodine deficiency, autoimmune disease, & nodular diseases

II. CLASSIFICATION
CLASSIFICATION I REMARKS

• Diffuse enlargement of the thyroid occurs in the absence of


Diffuse Non- nodules and hyperthyroidism
Toxic (Simple or • Thyroid function is preserved & most patients are asymptomatic
Colloid) Goiter • Levothyroxine can be started to suppress the TSH into the low-
normal, but detectable, range

Multinodular • Most are asymptomatic


Non-Toxic Goiter • Thyroid architecture is distorted and multiple nodules can be
(MNTG) appreciated
• Presence of functional autonomy in contrast to nontoxic MNG
Toxic • TSH is low, T 4 level is normal or minimally increased, and T3 is
Multinodular often elevated to greater degree than T 4
Goiter (TMNG) • Antithyroid drugs often given with beta-blockers can normalize
thyroid function
I I
TSH level
Normal/high I I
low

I I
Thyroid/neck sonogrophy Radionuclide scan
I I Non-functioning 1 I
nodule Hyperfunctioning
nodule

I Do fine needle aspiration I


(FNA) biopsy bosed on Treat as hyperthyroidism
ultrasound features and size I I

4 Non-diagnostic Repeat FNA

"'
.,§ Benign Follow-up
0
Cl.

""
-
>- Repeat ultrasound-
"'
0 Atypia or follicular lesion
guided FNA or
lu of undetermined
significance (AUS/FLUS)
1--
consider molecular
testing
11
E
t I Surgery
indicated
if I

"'
--1
0
al Consider molecular
Follicular neoplasm
testing
;;
"'
Suspicious for PTC

I Surgery
I
I
Malignant

TheBethesdaSystemfor ReportingThyroidCytopathology
established
a standardized,
category-
basedreportingsystemfor thyroidFNABspecimens

PTC,papillarythyroidcancer TSH,thyroidstimulatinghormone
Source:Jameson
JL,et al. Harrison's
Principles
of InternalMedicine
20thedition,2018
494
SECTION FOUR
DISORDERS OF THE ADRENAL GLANDS
CUSHING SYNDROME
I. ETIOPATHOGENESIS
Constellation of features due to chronic exposure to excess glucocorticoids of any etiology
• Iatrogenic use of glucocorticoids (for immunosuppression or treatment of inflammatory
disorders) is the most common cause
Cushing "disease" refers to Cushing syndrome caused by a pituitary corticotrope adenoma
GROUP I COMMENTS*
• Means that Cushing syndrome is due to excess ACTH
• Possible sources of ACTH excess:
ACTH-
0 Pituitary corticotrope adenoma (Cushing disease)
Dependent
0 Ectopic secretion of ACTH by nonpituitary tumor (paraneoplastic
syndrome)
• Means that Cushing syndrome is not due to excess ACTH
ACTH- • Majority of patients with ACTH-independent cortisol excess harbor a
Independent cortisol-producing adrenal adenoma
• Other causes: adrenocortical carcinoma, nodular adrenal hyperplasia
*Skinhyperpigmentation
mayserveas a cluetowardsanACTH-dependent
source.

II. MANIFESTATIONS
SYSTEM
I SYMPTOMS AND SIGNS
• Brittle and thin skin, facial plethora, purple and broad stretch
Integumentary
marks, hirsutism, acne, easy bruisability
• Proximal myopathy (gluteal and proximal leg muscle atrophy)
Musculoskeletal
• Weakness, osteopenia and osteoporosis
Cardiovascular
• Atherosclerosis, hypertension, hypokalemia, edema
and renal
• Central obesity, rounded face ("moon facies")
Endocrine • Prominent fat pad at the nape and upper back ("buffalo hump")

Reproductive
Central nervous
• Glucose intolerance, diabetes, and dyslipidemia
• Decreased libido, amenorrhea
• Emotional !ability, irritability, depression, cognitive defects
I
system (CNS) (occasional), psychosis (severe cases)
Hematologic and • Hypercoagulability, leukocytosis, lymphopenia, eosinopenia
immune • Increased susceptibility to infection

III. DIAGNOSIS
When to Suspect Cushing Syndrome?
Proximal muscle weakness • Buffalo hump
Purple reddish striae Hypokalemia
Truncal obesity , Thinning of skin
Hirsutism • Impaired glucose tolerance
Diastolic hypertension Osteoporosis

495
Alpproac h to patients wit h Suspecte d Cushinq Sundrome
Screening
1 mg dexomethasone suppressiontest
(positive if plasma cortisol >50 nmol/l at 8-9am after 1 mg dexamethosone at 11 pm)
24-h urinary free cortisol (3x increased above normal)
Midnight salivary cortisol >5 nmol/l or midnight plasma cortisol >130 nmol/L

l Positive

PlasmaACTH level

ACTH normal or I ACTH suppressed


high >15 pg/ml
I [ to <5 pg/ml

ACTH-dependentCushing ACTH-independentCushing

I
MRI with pituitary protocol
CRH test {positive if >40% ACTH increase at 15-30 min+ CTwith adrenalprotocol
>20% cortisol increase at 45-60 min ofter 100 µg CRH IV)
High dose dexamethasone test (positive if >50% cortisol I
suppression after 2 mg dexomethasone q6 for 2 days) Bilateral micronodulor or
macronodular adrenal hyperplasia
CRHlest and
high dose DEXpositive I CRH testand
high dose DEX negative
Unilateral adrenal mass

l l
Cushing disease Ectopic ACTH production
I I
ACTH,adrenocorticotropic
hormone;CRH,corticotropin-releasing
hormone;DEX,dexamethasone
Source:JamesonJL,et al. Harnson's
Principlesof InternalMed1c1ne
20thedItIon,2018
MelmedS, et al. WilliamsTextbook of Endocrinology,
12thedition

24-h urinary cortisol • Needs 2-3 complete consecutive collections


excretion • Positive (abnormal) test: >3X increase from upper limit
• Overnight suppression test: dexamethasone I mg given at
u:ooPM; then cortisol taken the next morning at 8-9 AM
Overnight and low-
• Low-dose suppression test: dexamethasone given q6 for 2 days;
dose dexamethasone
then cortisol taken the next morning at 8-9 AM
suppression test
• Suppression of cortisol is absent in Cushing syndrome
• Positive (abnormal) test: plasma cortisol >50 nmol/L
Midnight salivary cortisol • Positive (abnormal) test: >5 nmol/L
Determination of the Cause of Cushing ~yndrome
• Distinguishes between ACTH-dependent and ACTH-
Plasma ACTH
independent Cushing syndrome
• In Cushing disease, high-dose (8 mg/d) dexamethasone can
suppress pituitary adenomas (hence lower ACTH and cortisol)
High-dose dexamethasone
• In ectopic (non-pituitary) ACTH production, ACTH & cortisol
suppression test
levels remain high (unsuppressed)
• Positive test: >50% suppression of cortisol
Localization Tests
Pituitary MRI • To localize suspected Cushing disease
Unenhanced adrenal CT • To localize suspected adrenal mass

496
IV MANAGEMENT OF CUSHING SYNDROME
• Transsphenoidal pituitary surgery (Cushing disease)
Surgery
• Unilateral adrenalectomy (adrenal adenoma)
• Usually used to rapidly control cortisol excess in the period leading up
Medical to surgery
management • Includes drugs that inhibit certain pathways in cortisol synthesis such
as metyrapone and ketoconazole

ADRENAL INSUFFICIENCY (Al)


I. ETIOPATHOGENESIS
PRIMARY Al
I (ADDISON DISEASE) I SECONDARY Al

Etiology • Primary adrenal abnormality • HPA axis dysfunction


• No mineralocorticoid
• Glucocorticoid,
deficiency since the adrenal
mineralocorticoid and adrenal
itself is intact & still amenable
Pathophysiology androgen deficiency
to RAAS regulation
• ACTH secretion leads to
• ACTH deficiency leads to
hyperpigmentation
pale, alabaster-like skin
Most common
• Autoimmune adrenalitis • Iatrogenic suppression
cause
• X-linked adrenoleukodystrophy • Surgery
• Infection • Irradiation
Examples • Hemorrhage • Pituitary apoplexy or
• Infiltration Sheehan's syndrome
• Congenital adrenal hyperplasia

II. MANIFESTATIONS
CATEGORY I MANIFESTATIONS
• Fatigue, lack of energy, weight loss, anorexia, myalgia, joint pain
• Normochromic anemia, lymphocytosis, eosinophilia
Glucocorticoid • Slightly high TSH (due to loss offeedback inhibition ofTSH release)
deficiency • Hypoglycemia

I
• Low blood pressure, postural hypotension
• Hyponatremia (due to loss of feedback inhibition of AVP release)
• Abdominal pain, nausea, vomiting
• Dizziness, postural hypotension
• Salt craving
Mineralocorticoid
• Low blood pressure, postural hypotension
deficiency
• Increased serum creatinine (due to volume depletion)
• Hyponatremia
• Hyperkalemia
• Lack of energy
Adrenal
• Dry and itchy skin (in women)
androgen
• Loss of libido (in women)
deficiency
• Loss of axillary and pubic hair (in women)
• Hyperpigmentation (primary adrenal insufficiency only) due to
excess proopiomelanocortin (POMC)-derived peptides
Others
• Alabaster-colored pale skin (secondary adrenal insufficiency only)
due to deficiency of POMC-derived peptides
497
III. ACUTE AI (ADRENAL CRISIS)
Most frequently observed in patients with primary Al
Postural hypotension may progress to hypovolemic shock
May mimic acute abdomen (abdominal tenderness, nausea, vomiting and fever)
Can resemble neurologic disease with decreased responsiveness progressing to stupor/coma
Can be triggered by intercurrent illness, surgery, stress or increased glucocorticoid
inactivation (e.g., hyperthyroidism)

IV. DIAGNOSIS AND MANAGEMENT


When to Susvect Adrenal Insufficiencu?
Weight loss, fatigue
Postural hypotension
Hyperpigmentation
Hyponatremia

Approach to Patients with Suspected Adrenal Insufficiency

Screening
Plasma cortisol <450-500 nmol/L after 30-60 min of
cosyntropin 250 µg IM or IV (assay-specific)

High plasma ~------....L.------~ Low plasma


ACTH l lACTH

Primary adrenal insufficiency Secondary adrenal insufficiency

Glucocorticoid and Glucocorticoid


mineralocorticoid replacement replacement alone

ACTH,adrenocorticotropic
hormone
Source:JamesonJL,et al. Harrison'sPrinciplesof InternalMedicine20thedition,2018

MINERALOCORTICOID EXCESS
I. EPIDEMIOLOGY
The prevalence ranges from 5 to 12% in studies systematically screening all patients
with hypertension
Higher when patients are preselected for hypokalemic hypertension

II. ETIOLOGY
Excess activation of mineralocorticoid receptor leads to:
0 Potassium depletion (hypokalemia)
0 Increased sodium retention (expansion of extracellular and plasma volume leading
to hypertension)
Most common cause is primary hyperaldosteronism (adrenal [Conn's) adenoma,
bilateral [micronodular] hyperplasia, glucocorticoid-remediable hyperaldosteronism
[dexamethasone-suppressible hyperaldosteronism])
• Other causes: syndrome of apparent mineralocorticoid excess (SAME), Cushing's
syndrome, glucocorticoid resistance, adrenocortical carcinoma, congenital adrenal
hyperplasia, progesterone-induced hypertension, Liddle's syndrome

498
Ill. CLINICAL MANIFESTATIONS
Clinical hallmark is hypokalemic hypertension
K· depletion & Na· retention which can lead to expansion of extracellular & plasma volume
Hypokalemia can lead to muscle weakness, overt proximal myopathy or even paralysis
• Serum Na· tends to be normal due to concurrent fluid retention (can lead to edema)
Metabolic alkalosis secondary to hydrogen depletion from increased epithelial sodium
channel (ENaC) activity
Severe alkalosis can lead to muscle cramps and in severe cases, tetany
Aldosterone excess can cause direct damage to the myocardium and kidney glomeruli
in addition to secondary damage due to systemic hypertension

IV. DIAGNOSIS
When to Consider Testing for Primary Hypera/dosteronism?
Patients with hypertension, AND
Severe hypertension (>3 anti-hypertensives, drug resistant), OR
Spontaneous hypokalemia, OR
Adrenal incidentaloma, OR
Early-onset hypertension s20 years old or cerebrovascular events at <40 years old

Approach to Patients with Suspected Primary Hyperaldosteronism

Screening
Morning blood sample in a seated ambulant patient (hypokalemiashould be corrected and mustbe
off spironolactonefor 4 weeks)
Increased plasma aldosterone concentration (>450 pmol/L),
Decreased plasma renin activity(<l .Ong/ml/hr) or plasma renin concentration< lower limitof
detection for the assay, and
Aldosterone-reninratio (ARR)>750 pmol/L/ng/ml/hr (ii resultsare equivocal, consider repeat
off beta-blockersfor 2 weeks)

I Positive

Confirmatory
Saline infusiontest, or
Oral sodiumloading test, or
Fludrocortisonesuppressiontest, or
Captopril challenge test

j Positive
Adrenal CTscan

l
Unilateral
I
I

Bilateralmicronodular
hyperplasia
I
Normal
adrenal glands
I
adrenal mass
I I I
I Adrenal adenoma
I Screen for glucocorticoid-
remediable aldosteronism if
withfamilyhistory of early
onset hypertension

Source:Jameson
JL,et al. Harrison's
Principles
of InternalMedicine
20thedition,2018

499
SECTION FIVE
OTHER GENERAL ENDOCRINOLOGIC DISORDERS
PITUITARY DISEASES

• Growth hormone • Frontal bossing • Transsphenoidal surgery (TSS)


excess • Increased hand & mandibular • Somatostatin analogues
enlargement with prognathism (octreotide, lanreotide)
• Widened space between lower • GH receptor antagonist
incisors (pegvisomant)
• Gigantism in children & • Dopamine agonists
adolescents (cabergoline, bromocriptine)
• Radiation
2) Hyperprolactine"Jia
• Prolactin-producing • Galactorrhea • Dopamine agonises
pituitary tumor • Amenorrhea and infertility (cabergoline, bromocriptine)
(prolactinoma) • Surgery (TSS)
• Stalk compression
3) Gushing Disease
• ACTH-producing • Obesity, thin skin, moon facies, • Surgery (selective TSS)
adenoma hypertension, purple skin striae, • Pituitary irradiation
hirsutism, menstrual disorders, • Ketoconazole
acne, bruising, truncal obesity, • Metyrapone
proximal muscle weakness • Mitotane

4) Secondary hypert~yroidism
• TSH-secreting • Signs and symptoms of • Surgery (TSS)
pituitary hyperthyroidism • Somatostatin analogues
adenoma • Antithyroid drugs
5) Hypogonadism
• Gonadotropin • Oligomenorrhea, amenorrhea, • Hormone replacement
deficiency infertility, decreased vaginal (testosterone, estrogen,
secretions, decreased libido, progesterone)
decreased muscle mass,
reduced body hair growth, soft
testes, fine facial wrinkles
6) Diabetes Insipiduf,_
• Decreased • Large volumes of dilute urine • Desmopressin (ADH
secretion or action (24-hour urine >50 mL/kg BW analogue)
of antidiuretic & osmolarity <300 mOsm/L) • Thiazide diuretics
hormone (ADH) • Indomethacin
7) Syndrome of Inap ropriate ADH (SIADH)
• Excess secretion • Euvolemic hyponatremia • Water restriction to less
or action of ADH than the sum of urinary and
insensible losses
• Demeclocycline
• AVP receptor antagonists
(tolvaptan, conivaptan)

500
II. TESTS OF PITUITARY SUFFICIENCY
HORMONE I TEST I INTERPRETATION

• Insulin-induced hypoglycemia should normally


• Insulin
induce a rise in GH
tolerance test
Growth • Lack thereof suggests GH deficiency
hormone • GHRH test
• Normal respons~ is a rise in GH
• L-arginine test
• Lack thereof suggests GH deficiency
• L-dopa test
Prolactin • TRH test • TRH should normally induce a rise in prolactin levels
• Insulin-induced hypoglycemia induces a rise in
• Insulin ACTH and subsequently cortisol
tolerance test • Lack of rise in both ACTH and cortisol suggests
ACTH deficiency
• Normal response is an increase in basal ACTH
• CRH test
• Lack thereof suggests ACTH deficiency
ACTH
• Metyrapone blocks cortisol synthesis causing
• Metyrapone test decreased cortisol and increased ACTH
• Lack of rise in ACTH suggests ACTH deficiency
• Standard ACTH • Normal response is an increase in cortisol
stimulation • Lack of rise in cortisol plus low ACTH suggests
(cosyntropin) test ACTH deficiency
• Basal thyroid • TSH normally induces a rise in T3 and T 4 levels
function tests: • Low TSH with low free thyroid hormones indicate
T4, T3, TSH TSH deficiency
TSH
• Normal response is rise in TSH (unless thyroid
• TRH test hormone levels are increased)
• Lack thereof suggests TSH deficiency
•LH • Basal LH and FSH should normally be increased in
•FSH postmenopausal women
LH,FSH
• Low sex hormones in setting oflow LH and FSH

I
• Testosterone
• Estrogen indicate pituitary insufficiency
..
Source:JamesonJL, et al. Harnson'sPrinciplesof InternalMedicine20thed1t1on,
2018
MSmedS, et al."'momsTe,l.book
ofEodoc,iool.ogy,
121.h
editioo

OSTEOPOROSIS
I. ETIOPATHOGENESIS
A reduction in the strength of bone that leads to an increased fracture risk
• Results from bone loss due to age-related changes in bone remodeling and alteration of
skeletal microarchitecture, which may be superimposed on a low peak bone mass
May be affected by other factors such as inadequate calcium intake during growth,
vitamin D deficiency, and estrogen deficiency
Activation of RANK by RANK ligand (RANKL) represents a common pathway in
osteoclast activation

II. CLINICAL MANIFESTATIONS


• Asymptomatic in the majority of affected individuals
Fragility fracture
Height loss
Back pain
Postural change (anterior compression of vertebrae lead to kyphosis - "dowager's hump")
501
III. DIAGNOSIS
A. Criteria for Diagnosis
0 Based on WHO classification criteria for bone mass using dual-energy x-ray
absorptiometry (DXA) as gold standard
0 Presence of vertebral fractures on either radiograph or VFA examination confirms
clinical diagnosis of osteoporosis
CLASSIFICATION
I BONE MINERAL DENSITY (BMD) T-SCORE
Normal • 2! -I SD

Osteopenia (low bone mass) • Between -1 and -2.5 SD


Osteoporosis • s -2.5 SD
Severe Osteoporosis • S -2.5 SD and 2!1 fragility fracture/s
• Recommended DXAsites: femoral neck or total femur and/or lumbarspine (distalthird of radius if cannot
evaluate spine/hips)
• Peripheral BMDtechnologies (quantitativeUTZ,CT scan, single x-ray absorptiometry)done in sites like
the calcaneus, wrist & metatarsals; and bone turnover markers should not be used in the diagnosis of
osteoporosis (but can be used in fractureriskassessment & assessing adherence/effectivenessof therapy)
Source: Camacho PM, et al. Endocrine Practice. 2016
B 0th o·
DIAGNOS;IC I REMARKS & FINDINGS
• Reduced Ca•· levels reflect malnutrition and osteomalacia
Serum Ca•·&
• Elevated Ca•· with low phosphorus may suggest hyperparathyroidism or
phosphorus
hypercakemia of malignancy as the cause of osteoporosis
Serum 25-0HD • .Level >20-30 ng/mL usually suggests Vitamin D sufficiency
• Low urine Ca'· excretion suggests osteomalacia, malnutrition, malabsorption
24-h urine
• High urine Ca•· excretion may be due to renal calcium leak, absorptive
calcium
hypercalciuria, or excessive bone turnover
• Increased lucency of bone
Plain X-rays
• Check for neglected fractures
CT/MRI • May provide architectural information of affected bones but are costly
CBC& • To check for secondary causes of osteoporosis such as multiple
chemistry rnyeloma, hyperthyroidism, renal failure, and hepatic failure
Biochemical • Bone-specific alkaline phosphatase, osteocakin (bone formation)
markers • C- and N-telopeptide (bone resorption)

IV. SCREENING
A. Osteoporosis Screening Tool for Asian (OSTA)
0 Identifies risk for osteoporosis in areas where DXA is not widely available

45

55.59
60-64
65-69
70-74
75-79
11)-&1

95-99
Source: Koh LKH,et al. Osteoporosis International.2001
502
B. Fracture Risk Assessment Tool (FRAX)
0 Used to assess fracture probability in all postmenopausal women with at least one
WHO risk factor, prior to undergoing central DXA:
•LowBMI
• Previous fragility fracture
• Parental history of hip fracture
• Glucocorticoid treatment
• Current smoking
• Alcohol intake (at least 3 units/day)
• Rheumatoid arthritis
0 Other secondary causes: CKD, liver disease, malignancies, COPD, malabsorption, IBD,
hypogonadism, hyperparathyroidism, Cushing's disease, androgen deprivation therapy

C. Indications for Bone Density Testing (consider BMD testing):


0 Women :265years and men ?.70years (regardless of risk factors)
0 Younger postmenopausal women, women in menopausal transition and men aged
50-69 years old with risk factors for fracture
0 Adults who have a fracture after the age of 50
0 Adults with a condition associated with low bone mass or bone loss (e.g., RA, on
steroids :23mos)

V. MANAGEMENT
A. Indications to Treat
IF WITH BMD MEASUREMENT,
TREAT IF:
I IF WITHOUT BMD MEASUREMENT,
TREAT IF:

• Osteopenia plus history of fragility • Belongs to high-risk category based on


fracture of the hip or spine OSTA tool where central BMD cannot be
• BMD T-score of ~-2.5 SD done or not available, and
• IO-year probability of hip fracture :23%or
any major osteoporosis-related fracture
:220%based on FRAX estimates

B. Pharmacologic Therapies
0 Bisphosphonates (alendronate, risedronate, zoledronic acid, ibandronic acid)
0 Hormonal replacement therapy
0 Selective estrogen receptor modulators (SERM) such as tamoxifen and raloxifene
0 Strontium ranelate
0 Teriparatide (1-34hPTH)
° Calcitonin
0 Tibolone

0 RANKL inhibitor (denosumab)


0 Vitamin Dor its analogue, in combination with calcium, is used as mandatory adjunct

C. Duration of Treatment
THERAPY I DURATION

Oral • 5 years for moderate-risk patients, 6-10 years for high-risk


bisphosphonates patients
IV • 3 annual doses for moderate-risk patients, 6 annual doses for
bisphosphonates
(zoledronic acid) high-risk patients

Teriparatide • 2 years
Principles
Source:JamesonJL,et al. Harrison's 20thedItIon,2018
of InternalMed1c1ne
MelmedS, et al.WilliamsTextbook 12thedition
of Endocrinology,
2014NationalOsteoporosis Foundation Guide
Clinician's
503
D Non Pharmacolo . ic Ma a : ement
THERAPY I REMARKS

• Daily calcium intake ofmoo-1200 mg (supplement doses should be taken


,;600 mg at a time)
Nutrition • Daily vitamin D3 1000-2000 JU to maintain serum 25-OHD levels 230 ng/mL
• Magnesium supplementation only if with inflammatory bowel disease,
chemotherapy, severe diarrhea, alcoholism, or malnutrition
• Supervised high-intensity resistance exercise (8-12repetitions at least 2-3
days/week)
Exercise
• Moderate levels of walking (24 METs/week or 8 h/week of walking at an
average pace)
• Smoking cessation & limiting alcohol consumption
• Hip protectors for individuals with a high risk of falling
Lifestyle
• Environmental risk reduction includes good home lighting, avoiding
stocking feet on wood floor, and elimination of exposed wires
Surgery • Kyphoplasty & vertebroplasty may be usecl for painful vertebral fractures

REFERENCES
I. Alvarez JR, Fechner AJ, Williams SF, Ganesh VL, and Apuzzio JJ.Asymptomatic bacteriuria in pregestational diabetic
pregnancies and the role of group B streptococcus. Am J Perinatal 2010; 27:231.
2.BIUmer I, Hadar E, Hadden DR, Jovanovic 1, Mestman JH, Murad MH et al. Diabetes and Pregnancy: An Endocrine Society
Clinical Practice Guideline. J Ciin Endocr Metab. 2013; 98(II): 4227-49.
3. Burch, HB and Wartofsky, L. "Life-threatening thyrotoxicosis. Thyroid storm.", Endocrinology and metabolism clinics of
4.C~~~~t~PM'.ci:?itS~\ 2)Bt:t-eV· N, Clarke BL, Harris ST, Hurley DL, et al. 2016 American Association of Clinical
Endocrinologists and American College of Endocrinology Clinical Practice Guidelines for the Diagnosis and Treatment
5. c~far~i~(ed~f~!~r~~~di~b ~fc~r~
~A~fed~
0est~~~~~i~i~n~:s~~~d~rd~ 1 ~: 2 D~betes 2017.Diabetes Care. 2017Jan; 40
6.Fong A, Serra A, Herrero T, Pan D, and Ogunyemi D. Pre-s-estational versus gestational diabetes: a population-based study
on clinical and demographic differences. J Diab Complications. 2014; 28(1):10.1016/jdiacornp.2013.08.009.
7. Fonseca, V. Clinical significance of targeting postprandial and fasting hyperglycemia in managing type z diabetes mellitus.
Curr Med Res Opin. 2003; 19(7}:635-41
8. Furnary AP, Wu YX and Boo kin SO. Effect of hxperglycemia and continuous intravenous insulin infusions on outcomes of
9.Garber, JR; Cobin, RH; Gharib, H; Hennessey, lV;
cardiac surgical procedures: the Portland Dia0etic Project. Endocr Pract. 2004; IQ Suppl 2:21-33.
Klein, I; Mechanick, JI; Pessah-Pollack, R; Singer, PA; et al. (December
G
0 2
10. ~1~:~~i~~:l!rs~1~i~tt~~d~snfD.
J Obstet Gynecol Reprod Biol 1989;33:101.
0
~XS~a!id ~~-A!~:g~~:~ic~~~t!~~~?~1~ ~~~~;i~~d
2 1
hlih-risk pregnancy. Eur
11. Hone, J and Jovanovic L. Approach to the patient with diabetes during pregnancy. J Clin Endocrine! Metab. 2010; 95:
12. International Association of Diabetes and Pregnancy Study GrouJ?S Consensus Panel. IADPSG Recommendations on
the Diagnosis and Classifica1ion of Hyperglycemia in Pregnancy. Diabetes Care 2010; 33(3):676-82.
13. International Hypoglycaemia Study Group. Glucose concentrations of less than 3.0 mmol/L (54 mg/dL) should be
f~f~h~egr~l 1t/Dlab~~~~-adf~b~fe~s(~~~ !~~~~:i:~; 5~;;~e American Diabetes Association and the European Association
14. lnzucchi, s? Bergenstal RM, Buse JB, Diamant M, Ferrannini E, Nauck M, et al. Management of hyperglycemia in type
2 diabetes, 2015:a patient-centered approach: update co a position statement of the American Diabetes Association and
the European Association for the stucfy of Diabetes. Diabetes Care 2015;38:140-149
15. Jameson JL, Fauci AS, Kasper DL, Hauser SL, Longo DL, and Loscalzo J (editors). Harrison's Principles of Internal
Medicine, 20th ed. McGraw Hill Education, 2018.
16. Jimeno CA, Abad L, Andag-Silva A, Cunanan E, Fernando RE, Fojas M, et al. Philippine Practice Guidelines for the
Diag_nosisand Management of Diabetes. UNITE for Diabetes Philippines. 2011.
17. Jonklaas J, Bianco AC, Bauer AJ, Burman KO, Ca~pola AR, Celi FS, et al. American Thyroid Association Task Force on
18. k~~b~i~i '1t.muo~;i!~ie 1 :G;Mli~J
Care. 2009 July; 32(7):1335-1343.
t{'!~el $?;here J~~i-\;;;r~ltch/J~ttri~~~ii::\.J~rp~~i!~\~~~,f
2 thbiaberes. Diabetes

19. Koh LKH, Sedrine WB, Torralba TP, Kung A, Fujiwara S, Chan SP, et al. A simple tool to identify Asian women at
increased risk of osteoporosis. Osteoporosis International 200!; 12:699-705
20. Li-Yu J, Perez EC, Canete A, Bonifacio L, Llamado LQ, Martinez R, et al. Consensus Statements on Osteoporosis
Diagnosis, Prevention and Management in the Philippines. Int J Rheum Dis Aug 2011;14:223-238.
21. Litonjua AD, Boedisantoso R, Serirat S, and Zaini A. AFES Study Group on Diabetes in Pregnancy (ASGODIP). Phil J
Int Med. 1996;34(2),67-68.
22. Melmed S, Polansky KS, Larsen PR, Kronenberg HM (editors). Williams Textbook of Endocrinology, 12th ed. Elsevier
Saunders, 2011.
23. Nathan DM, Buse JB, Davidson MB, Ferrannini E, Holman RR, Sherwin R et al. Medical management of hyperglycemia
in Type 2 diabetes: a consensus algorithm for the initiation and adjustment of therapy. Diabetes Care. 2009; 32(1):193-203-
24. Pai MP and Paloucek FP. The origin of the ideal body weight equations. Ann Pharm 2000; 34(9):1066-9.
25. Fatal. P; Hamin, J: Bautista, A; Jimeno, CA; Acampado, LT; Hipolito, M; et al. "Trimester Specific Reference Interval for
Thyroid Function Tests in Pregnant Filipino Women." JAFES 2016; 31(1).
26. Paulweber B, Valensi P, Lindstrom J, Lahc NM, Greaves CJ,McKee M, et al. A European evidence-based guideline for the
prevention of tyre 2 diabetes. Horm Me tab Res. 2010; 42 Suppl 1:S3-6.
27. Re~ional Office for the Western Pacific of the World Health Organization, the International Association for the Study
ofubesity and the International Obesity Task Force. The Asia-Pacific perspective: Redefining obesity and its treatment.
Health Communications Australia Pty. Limited, February 2000.
28. Ross OS, Burch HS, Cooper DS, Greenlee MC, Laurberg P, Maia AL, et al. 2016 American Thyroid Association Guidelines
for Dia~nosis and Management of Hyperthyroidism and Other Causes ofThyrotoxicosis. Thyroid. 2m6; 26(10):1343-1421.
29. Stone NJ, Robinson/• Lichtenstein A, ·Merz CNB, Blum CB, Eckel RH et al. 2013ACC/AHA Guideline on the Treatment
of Blood Cholestero to Reduce Atherosclerotic Cardiovascular Risk in Adults. DOI: 10.1161/01.cir.0000437738.63853.7a
30. Wells SA Jr, Asa SL, Dralle H, Elisei R, Evans DB, Gagel RF et al. Revised American Thyroid Association guidelines for
the management of medullary thyroid carcinoma. Thyroid 2015;25:567.
504
RHEUMATOL
APPROACH TO RHEUMATOLOGIC COMPLAINTS
1. History of a Patient with a Rheumatologic Complaint
2. Physical Examination
3. Diagnostics in Rheumatology

0 RHEUMATOLOGIC DISORDERS
1. Osteoarthritis
2. Gouty Arthritis
3. Rheumatoid Arthritis
4. Infectious Arthritis
5. Systemic Lupus Erythematosus (SLE)
6. Antiphospholipid Syndrome (APS)
SECTION ONE
APPROACH TO RHEUMATOLOGIC COMPLAINTS

HISTORY OF A PATIENT WITH A RHEUMATOLOGIC COMPLAINT


Rheumatology is a branch of medicine devoted to the management of rheumatic diseases. Patients
seen by rheumatologists encompass those with pain and disorders of the joints, muscles, tendons,
bones, and other connective tissues, including immune-mediated diseases.

I. JOINT PAIN
QUESTIONS
TO ASK
I REMARKS

• Monoarthritis (1 joint involved): gout, septic arthritis, knee


osteoarthritis (OA), reactive arthritis (ReA)
What is the • Oligoarthritis (2-3 joints involved): OA, ReA, gout
number of joints • Polyarthritis (>3joints involved): rheumatoid arthritis (RA),
affected? systemic lupus erythematosus (SLE), hand OA, chronic gout
• Psoriatic arthritis has five types, and can therefore be mono- to
polyarticular
• Acute versus chronic
0 Acute onset: ,,;6weeks: gout, septic arthritis
° Chronic onset: >6 weeks: OA, RA, TB arthritis, connective tissue
What is the mode
disease (CTD)-related arthritis like SLE
of onset?
• Chronic diseases may have acute onset or "flares" (e.g., OA flare),
while some acute diseases can become chronic (e.g., chronic
tophaceous gout)
• Inflammation usually presents with at least one of the following:
dolor (pain), ca/or (warmth), rubor (redness), tumor (swelling), and
functio laesa (loss of function)
Is there presence
• Inflammatory arthritis: pain and stiffness in involved joints; worse
or absence of joint
in the morning or after periods of inactivity ("gel phenomenon");
inflammation?
improves with mild to moderate activity
• Non-inflammatory arthritis: pain that worsens with activity and
improves with rest

• Non-articular sites: extraarticular ligaments, tendons, bursae,


Is it articular or muscles, fascia, bones, nerves, overlying skin
non-articular? • Articular sites: synovium, synovial fluid, articular cartilage,
intraarticular ligaments, joint capsule, juxtaarticular bone
• Pattern of onset: migratory, additive, intermittent
0 Migratory: joints are sequentially affected where, as one joint
improves or quiesces, another becomes inflamed (e.g., acute
rheumatic fever)
What is the 0 Additive (most common pattern but least specific): subsequent
pattern of joint
joints become involved while preceding ones are still inflamed
involvement?
(e.g.,RA)
0 Intermittent: the same joint is involved in different episodes
of inflammation, but the joint is quiescent during intervening
periods (e.g., gout)

• Symptoms include fever, weight loss, fatigue


Is there systemic
• May also involve the eyes and skin (e.g., uveitis, scleritis, malar
involvement?
rash, oral ulcers, photosensitivity)

507
Clinical A roach to Arthritis
Gout
Pseudogout
Acute (S6 weeks) Chronic arthritis, initial presentation
Infectious/septic arthritis
Reactive arthritis

Articular arthritis

Osteoarthritts
Non-inflammatory (DIP,CMCl, hip, knee)

Chronic (>6 weeks)

Inflammatory

Mono/oligoarthritis Polyarthritis
(1-3 joints) (>3 joints)

Psorioticarthritis
Reactive arthritis
Pauciarticularjuvenile
idiopathic arthritis
Indolent infection Asymmetric

Psoriatic arthritis
Reactive arthritis

Yes No
PIP:proximalinterphalangeal
DIP:distalinterphalangeal SLE
CMC:carpometacarpal • Rheumatoidarthritis Scleroderma
MCP:metacarpophalangeal Polymyositis

Firstdetermine if disorderis articular


(characterized
by deep/diffuse
painor limitedROMduringpassive& active
movement; associated withswelling, crepitation,
instability,
deformity).
Thevariousarticular
disorders
canthenbe
differentiated
viadisease duration, presenceofinflammation,symmeby ofinvolvement,
andlocationofaffected
joints.
Source: Jameson JL, et al. Harrison'sPrinciplesof InternalMedicine,20th edition;2018
Cush JJ, et al. Polyarticulararthritis.MusculoskeletalKey.Online:musculoskeletalkey.com
II. LOW BACK PAIN
QUESTION I REMARKS
• Local pain: site of pain is near the affected part of the back
• Pain referred to the back: described as primarily abdominal or visceral, with
back pain & unaffected by posture
Wliatistl,e
• Pain of spinal origin: upper lumbar spine conditions produce pain referred to
the lumbar region, groin, or anterior thighs while lower lumbar spine conditions
type of back
produce pain referred to the buttocks, posterior thighs, calves, or feet
pain?
• Radicular pain: typically sharp & radiates down a leg within the territory of the nerve
root; exacerbated by coughing, sneezing, or contraction of abdominal muscles
• Pain associated with muscle spasm: usually accompanied by abnormal posture,
tense paraspinal muscles, and dull or achy pain in the paraspinal region

Are tliere • Age <40 or >70 years old


risksfor • Pain worse at rest and improves with activity, or pain at night
a possibly • History of trauma, cancer, or infection (especially oflungs, skin, & urinary tract)
serious or • Rapidly progressive neurologic deficit, bowel or bladder incontinence
systemic • IV drug or glucocorticoid use, unexplained fever/weight loss, tenderness over spine
cause? • Positive straight-leg or reverse straight-leg raising tests

508
PHYSICAL EXAMINATION
I. DEFINITION OF TERMS
TERM I DEFINITION
Crepitus • Palpable vibratory or crackling sensation produced by joint movement
• A change in joint alignment that results in incomplete approximation of
Subluxation
articulating surfaces
• An abnormal displacement of articulating surfaces that results in the
Dislocation
surfaces not being in contact
Range of • The arc of measurable movement for diarthrodial joints, through which
motion the joint moves in a single plane
• Loss offull movement that results from a fixed resistance caused by tonic
Contracture
muscle spasm or fibrosed periarticular structures

• Abnormal shape or size that results from bony hypertrophy, malalignment of


Deformity
articulating structures, or damaged periarticular structures that provide support
Enthesitis • Inflammation of en theses (tendinous or ligamentous insertions in bone)
Epicondylitis • Infection or inflammation of the epicondyles

Inspection • Check for erythema, subluxation, dislocation, contractures, deformities, atrophy

Palpation • Palpate each joint for swelling, warmth, crepitus, and tenderness

Range of • Perform range of motion maneuvers (seenext page)


motion • If with pain, differentiate whether it is reproduced with active or passive ROM
• Grading of muscle strength (5-point scale)
0 Grade o: no movement
0 Grade 1: trace movement or twitch
Muscle 0 Grade 2: movement with gravity eliminated
Strength 0 Grade 3: movement against gravity only
0 Grade 4: movement against gravity and resistance
0 Grade 5: normal strength

Specific Maneuversfor Low BackPain

Heel • Compression of the heel towards the hip with the


percussion examiner's palm while the leg is extended reproduces
1-si_g_n
___ -l-_h.:_ip_p:_a_i_n
________________
• Bedside test for nerve root disease
St raight leg • With the patient supine, passive flexion of the
raise (SLR) extended leg at the hip with dorsiflexion of the foot
sign
reproduces the pain
• Flexion of the hip with knee extended reproduces the
Crossed usual pain in the opposite leg or buttocks
SLR sign • Less sensitive but more specific than the SLR sign for
disc herniation

• With the patient prone or standing, passive extension


Reverse
of each hip with the knee fully extended reproduces
SLR sign
the usual pain

509
RIGHT LEFT

There are 42 easily examined joints


• Distal interphalangeal joints (D!Ps)
• Proximal interphalangeal joints
(P!Ps)
• Metacarpophalangeal joints
Wrist (MCPs)
• Wrists
• Elbows
~'"''""" • Shoulders
• Hips
• Knees
KneeJoint
• Ankles

Anklejoint

~Too)olnts

IV. RANGE OF MOTION MANEUVERS


JOINT I MANEUVER

Temporo- • Open and close mouth


mandibular • Jut jaw forwards and backwards
joint • Move jaw to the right and left

• Flexion (look down until the chin reaches the sternum)


• Extension (return to neutral position)
Cervical
• Hyperextension (look at the ceiling)
spine
• Lateral flexion (let your ear touch your shoulder without the shoulder moving up)
• Rotation (look to the right and to the left)
• Trunk flexion (bend down without flexing knees)
Thoraco- • Trunk extension (return to neutral position)
lumbar • Hyperextension (bend backwards with both hands on the hips)
spine • Lateral bending (bend trunk to the right and left)
• Rotation (rotate trunk to the right and left)

• Place hands behind the head (elbows out)


• Place hands by the side (elbows straight}
Upper
• Place hands behind (with the back of hands touching your back)
extremities
• Place hands in front (palms down)
(patient is
seated) • Turn hands over (palms up)
• Make a fist
• Touch the tip of each finger to the tip of the ipsilateral thumb
• Flexion, extension, internal rotation, & external rotation of hip (patient is supine)
Lower • Abduction and adduction of the hip (patient is lying to one side)
extremities • Flexion and extension of the knee (patient is seated or supine)
• Inversion,eversion,dorsiflexion& plantar flexionof ankles (patient seated or supine)
S10
DIAGNOSTICS IN RHEUMATOLOGY
I. BLOOD TESTS
DIAGNOSTIC I COMMENTS/EXPECTED FINDINGS

• Anemia is common in chronic inflammatory conditions


• Leukocytosis is common in septic arthritis, acute gout, & juvenile arthritis
• Leukopenia and lymphopenia in a patient presenting with polyarthritis
Complete
can be suggestive of SLE
blood count
• Reactive thrombocytosis is common with active chronic inflammatory
arthritis, e.g., RA and juvenile idiopathic arthritis
• Thrombocytopenia can be a presenting feature of SLE
• Inflammatory marker (usually elevated in inflammatory arthritis or
Erythrocyte
vasculitis)
sedimentation
rate(ESR) • Can be elevated due to many other factors, e.g., menstruation,
pregnancy, infection
• Inflammatory marker (usually elevated in inflammatory arthritis or
C-reactive vasculitis)
protein (CRP) • Generally lacks specificity, but when very high, septic arthritis or gout
should be excluded
• Values range from:
0 Men: 4.0-8.6 mg/dL (238-516umol/L)
0 Women: 3.0-5.9 mg/dL (178-351umol/L)
• Hyperuricemia is associated with gout and nephrolithiasis, but levels
Serum Uric
may not correlate with articular disease severity
Acid(SUA)
• 50% of patients with an acute gouty attack will have normal serum uric
acid levels
• Monitoring may be useful in assessing response to therapy
(target: <6 mg/dL)

II. AUTOANTIBODY TESTS


ANTIBODY I REMARKS
Antinuclear • ANA-immunofluorescence (IF) with pattern & titer is the preferred test
antibody (ANA) • Screening test for SLE, but is not diagnostic without clinical features
ds-DNA • 50% of SLE (high specificity)
U1-RNP • >90% of mixed connective tissue disease (MCTD)
Smith(Sm) • 30% of SLE (high specificity)
Histones • Present in drug-induced lupus (e.g., procainamide, hydralazine)
• 60% of Sjogren's syndrome, subacute cutaneous lupus erythematosus,
Ro(SS-A)
neonatal lupus
La(SS-B) • 50% ofSjogren's syndrome, 15%of lupus
PM-r • Polymyositis, dermatomyositis

Jo-I • Polymyositis with pneumonitis + arthritis

Scl-70 • 40% of diffuse scleroderma

RNA
• 40% of progressive systemic sclerosis
polymerase I
Centro mere/
• 75% of CREST (limited scleroderma)
kine to chore

511
III. IMAGING TESTS
DIAGNOSTIC I COMMENTS/EXPECTED FINDINGS
• Add little to the evaluation of acute presentations of arthritis
Radio graphs (except in cases of suspected trauma) but often critical for the
assessment of chronic arthritis
• Useful in detection of fluid in joints
• Can differentiate between synovitis (pre-clinical rheumatoid
arthritis) versus tendinitis (spondyloarthritis)
Ultrasonography • Assessment of soft tissue abnormalities (e.g., tendinitis,
tenosynovitis, enthesitis, bursitis)
• Good method for evaluation of synovial (Baker's) cysts, rotator
cuff tears, tendinitis, and crystal deposition in cartilage
• Provides detailed visualization of the axial skeleton & articulations
• Superior to conventional X-rays for fractures of the posterior
Computed spine, craniocervical and cervicothoracic junctions, C1 and
tomography (CT) Cz vertebrae, bone fragments within the spinal canal, or
scan malalignment
• CT myelography provides optimal imaging of the spinal canal
lateral recess and is better for claustrophobic patients

Magnetic • Provides superior definition of musculoskeletal soft tissue


resonance structures such as fascia, vessels, nerves, muscle, cartilage,
imaging (MRI) ligaments, tendons, pannus, synovial effusions, and bone marrow

IV. MINIMALLY INVASIVE TESTS


DIAGNOSTIC I COMMENTS/EXPECTED FINDINGS
• Most reliable means of distinguishing between inflammatory
and non-inflammatory arthritis is analysis of the white blood cell
(WBC) count in the synovial fluid
Arthrocentesis
• Important for consideration of septic arthritis and gout
and synovial
fluid analysis • Determine cell count; detect bacteria by gram stain; look for uric
acid crystals under polarized light
• Synovial fluid WBC count is <2000/mm' in non-inflammatory
arthritis
• Normal if focal sensory loss is due to nerve root damage (i.e.,
Sensory nerve proximal to the dorsal root ganglion)
conduction
studies • Reduced signals seen in nerve tissue injuries distal to the dorsal
root ganglion
• Complements nerve conduction studies by detecting denervation
Needle or reinnervation changes in a myotomal distribution
electromyography
(EMG) • Provides information about motor nerve fiber injury when clinical
evaluation of weakness is limited by pain or poor effort

512
SECTION TWO
RHEUMATOLOGIC DISORDERS
OSTEOARTHRITIS (OA)
I. ETIOPATHOGENESIS
• Most common joint disease and a leading cause of disability in the elderly
Characterized by hyaline articular cartilage loss
Initially present in a focal and non-uniform manner, progressing with increased
thickness and sclerosis of the subchondral bone, osteophyte outgrowth, stretching
of the joint capsule, and weakness of muscles surrounding the joint - all of which
ultimately lead to joint failure
Risk factors include age (most important), obesity, repeated joint use

II. MANIFESTATIONS
Joint pain is activity-related, starting as episodic and progressing continuously with
accompanying brief morning stiffness (<30 min) that gradually resolves
• Commonly affected joints:
° Cervical and lumbosacral spine, hip, knee and first metatarsophalangeal (MTP) joints
In the hands, the distal and proximal !Ps and base of the thumb are often affected
0

Wrist, elbow, and ankle joints are less commonly affected


0

Consider OA over other inflammatory arthritic disorders (such as rheumatoid arthritis) if:
With morning stiffness <30 mins
0

With involvement oflarge weight-bearing joints and/or distal and proximal !Ps
0

Non-involvement of MCPs
0

III. DIAGNOSTICS
No blood tests are routinely indicated
Synovial fluid analysis reveals a non-inflammatory pattern (WBC count <2000/mm')
Joint imaging correlates poorly with presence and severity of pain
IMAGING I REMARKS
• May be normal in very early stages
• Advanced stages may show joint space narrowing, subchondral
sclerosis, osteophytes
Radiographs • May be indicated for:
0 Evaluation of chronic hand and hip pain thought to be due to OA, if
diagnosis is unclear
° Knee pain that persists after initiation of effective treatment
• May reveal the extent of pathology in osteoarthritis but is almost never
MRI
indicated as part of diagnostic work-up

IV. MANAGEMENT
A. Overview of Management
0 Involves both pharmacologic and non-pharmacologic/adjunctive measures
0 Goal is relief of pain and prevention of disability

B. Non-Pharmacologic and Adjunctive Management


0Mainstay is alteration of joint loading and improving function of joint protectors
0Aerobic± resistance exercises (preferably low-impact) with brief periods of rest for the
involved joint
0Weight management (weight loss of5 kg translates to 50% reduction in pain)
° Correction of possible malalignment (knee braces, orthotics)
0 Acupuncture has been shown to offer some pain relief
0Oral standardized ginger preparation
0Surgery (arthroscopic debridement and lavage, meniscectomy, arthroplasty)
513
C. Pharmacologic Management (tailoredto avoid aggravationof patients' comorbidities)
DRUG CLASS & ADVERSE EFFECTS/
EXAMPLE I MECHANISM
I COMMENTS
Non- 0 pw1 dA na 9f*ICS -
• Paracetamol • Modulates endogenous • Initial pharmacologic treatment
(max: 3 g per day) cannabinoid system in the of choice
brain • Can prolong half-life ofwarfarin
-
Opioid-Like Analge1ics
• Tramadol 50-100 mg • Binds to the u-opioid • Dizziness, sedation, nausea/
94-6 receptor vomiting, urinary retention,
constipation, dry mouth

Traditional NSAIDV
• Naproxen (oral) • Suppress inflammation by • GI ulceration and bleeding,
375-500 mg BID, inhibiting both COX-1 and edema, renal insufficiency
taken with food COX-2 activity
• Rubbed onto affected joint
• Diclofenac 1%gel
• Skin irritation common
(topical), 4 g QID
• Avoid if with renal disease
• Topical formulations less efficacious
than oral, but less adverse effects

Oral CGX-2InhibitQrs -,·


• Celecoxib 100-200 mg • Inhibits COX-2 activity only • Increased risk for stroke and
OD-BID myocardial infarction
• Etoricoxib30-6omg OD • Avoid if with renal disease
fntraarticular Inje~tions ·-

• Intraarticular steroids • Suppress inflammation • Pain at injection site


• Useful for acute pain flares or if
with coexistent gout
• Hyaluronic acid • Nonsulfated • Pain at injection site
glycosaminoglycan • Evidence for effectiveness is
distributed in joints mixed
"symptomatfr: Slow:!cting Drugs for Osteoarthritis (SYSADOA)
• Hexosamines • Precursors of glycosylated • Evidence for effectiveness is
(glucosamine sulfate, lipids and proteins, thereby mixed
chondroitin sulfate) supporting joint structure
and function
• Avocado-soybean • May inhibit breakdown of
unsaponifiables cartilage, promote cartilage
repair & stimulate synthesis of
collagen& aggrecanby inhibiting
inflammatory cytokines
• Diacerein • Anthraquinone derivative • GI adverse effects (diarrhea,
with anti-inflammatory activity abdominal pain)
Others u
• Capsaicin • Binds to the vanilloid receptor • Can irritate mucous membranes
0.025-0.075% cream subtype 1(TRPV1),thereby
3-4 times per day modulating pain sensations
Source:Bruyere,et al. ESCEO;2019& Fernandes
L, et al. EULAR.AnnRheumDis;2013

514
GOUTY ARTHRITIS
I. ETIOPATHOGENESIS
Metabolic disease that usually affects men (from late 20s onwards) & postmenopausal women
Results from increased body urate pool with hyperuricemia, which may be a result of
either urate overproduction or urate underexcretion
Hyperuricemia is a central feature of gout, but does not inevitably and absolutely cause it
Genetics play a big role in hyperuricemia, but precipitants include:
0 Dietary excess
0 Trauma/surgery
0 Excessive ethanol ingestion
0 Hypouricemic therapy
0 Serious comorbid illnesses such as stroke and myocardial infarction

II. CLINICAL MANIFESTATIONS


Initially affects the MTP of the first toe (podagra), or any joint in the foot, ankle, or
knee, followed by recurring episodes of acute mono- or oligoarthritis, & finally chronic
polyarticular nonsymmetric synovitis with tophaceous deposits
Inflamed Heberden's or Bouchard's nodes may also be the initial manifestation
Initial attacks mimic cellulitis, subside spontaneously over 3-IO days, and have varying
asymptomatic periods until the next attack
Consider gout over other inflammatory arthritic disorders if initially presenting with
classic podagra or if already with chronic tophaceous deposits
PHASE
I DEFINITION/DESCRIPTION
• Hyperuricemia in the absence of gouty arthritis & uric acid nephrolithiasis
Asymptomatic • Hyperuricemia: defined as serum uric acid (SUA) level:
hyperuricemia 0 >7 mg/dL (417umol/L) in men

0 >6 mg/dL (357umol/L) in pre-menopausal women

• Presence of pain, swelling, erythema of one to several joints


0 Maximum inflammation develops within I day
0 May have more than I attack of acute arthritis

Acute gouty 0 Asymmetric swelling within a joint or subcortical cysts without erosions on x-ray

arthritis 0 Hyperuricemia usually (but not always) present, especially during a gout attack

• Septic arthritis may coexist; ifin doubt, establish absence of organisms on


joint fluid culture during attack
• Characteristic urate crystals seen in joint fluid or from pre-existing tophi

Intercritical • Asymptomatic periods between gouty attacks


(interval) gout • Urate crystals may still be seen in the joint fluid

Chronic • Occurs in untreated gouty arthritis


tophaceous • Low grade inflammation of joints with sporadic flares
gout(CTG) • Joint deformities occur due to deposition of crystals, forming visible tophi
Source:L1-Yu
J, et al. Ph1l1pp1ne
ClinicalPracticeGuidelines
for the Management
of Gout.PRA;2008.

lll. COMMON MYTHS AND FACTS ABOUT GOUT ,


MYTH
I FACT

"Gout only affects • Women tend to be relatively protected by the uricosuric effect of estrogen.
nten" Hence, gout cases in women tend to increase only after menopause.

"Goutonly affects • Although gout attacks often initially involve the MTP of the first toe
the big toe" (podagra), it can eventually progress to involve other joints if left untreated.

"Goutonly affects • While beer is very high in purines, other food substances such as organ
peop!,ewho drink meats and certain fish products (anchovies, sardines) are also high in
beer purines and can precipitate a gout attack.

"Peoplewith gout • Vegetable protein sources (e.g., peas, beans, and legumes) contain much
cannot eat beans lower levels of purines compared to animal protein, and are actually
and legumes" healthy choices for people with gout

515
IV. DIAGNOSIS
DIAGNOSTICS I COMMENTS/EXPECTED FINDINGS

• Indicated to rule out pseudogout or infectious arthritis


• Strongly negative birefringent needle-shaped monosodium urate
Diagnostic
(MSU) crystals both intra- & extracellularly
arthrocentesis
• Findings on synovial fluid analysis:
(synovial fluid
• Thick chalky/opaque fluid
analysis)
• WBC 2,000-60,000/uL
0 May not be as viscous as normal

• Soft tissue swelling only early in the disease


• Cystic changes with well-defined erosions, sclerotic margins, and
Radiograph
overhanging bony edges ("rat-bitten" appearance) in late disease
• Soft tissue masses (tophi) in advanced disease

• Double contour sign (echogenic line on outer surface of joint cartilage


Ultrasound
parallel to the subchondral bone due to deposition ofMSU crystals)

Serum uric acid (SUA) • May be low or normal at the time of attacks

24-hour uric acid • >Boo mg/24h (over-producers)


urine collection • <600 mg/24h (under-excretors)

V. MANAGEMENT
Treatment goal is to reduce uric acid to <6 mg/dL (<5 mg/dL if with chronic tophaceous gout)
• For asymptomatic hyperuricemia: treatment with urate-lowering medicine is not routinely advised

A. Medical Management of Acute Attacks


• Acute flares of gout should be treated as early as possible
• Recommended first-line options for acute flares: colchicine, NSAIDs, oral
corticosteroid, or ·oint as iration & steroid in ·ection

mg loading dose
• 1 • Inhibits microtubule
• Avoid in patients receiving
within 12 hours of flare polymerization &
CYP3A4inhibitors
Colchicine onset, followed by neutrophil activity&
• Renaldosing ifwith lowGFR
0.5 mg after I hour, OR motility
• Stop if diarrhea occurs
• TIO until flare subsides • Anti-inflammatory

• Indomethacin25-50mgTID • Avoid in patients with


• Naproxen 500 mg BID • Suppress severe renal impairment
• Ibuprofen 800 mgTID inflammation by • GI ulcers/bleeding (with
NSA!Ds
• Diclofenac 50 mg TID inhibiting COX-1or NSAIDs;most with naproxen)
• Celecoxib400 mg BID COX-2 activity • Increased risk for stroke &
• Etoricoxib 120mg OD Ml (least with naproxen)

• Anti-inflammatory
• Side effects from acute use
• Give intra-articularly
• Prednisolone 30-35 mg/ are relatively minor
for monoarticular
Steroids day or equivalent for • Mild to moderate
gout or systemically
3-5days pain at injection site if
for oligo-/
intraarticular
polyarticular gout

• Considered if with frequent


flares & contraindications
mcg/day SC foq-5
• 100 to colchicine & NSAIDs
Anakinra • IL-1blocker
days • Current infection is a
contraindication
• Not available locally
• Indications include:
Therapeutic • Unable to take oral medications
• Have only 1-2actively inflamed joints
arthrocentesis 0 Ready access to a clinician with expertise in arthrocentesis
° Can be performed immediately prior to injecting intraarticular medications

516
B. Flare Prophylaxis
0 Recommended until serum uric acid is <6 mg/dL or the patient is without attacks for 6
months, or for as long as tophi are present
0 Dose: colchicine 0.5 mg OD-BID (reduce dose in renal impairment)

C. Urate-Lowering Therapy (ULT)


I COMMENTS
• Recurrent flares (>2/year)
• Presence of top hi and urate arthropathy
Indications
• Nephrolithiasis
for ULT
• First diagnosed with gout <40 years old, SUA >9 mg/dL, & with
comorbidities (renal impairment, hypertension, CAD, heart failure)

• Lifelong SUA: <6 mg/dL (360 umol/L)


Targets of
• Lower target SUA <5 mg/dL (300 umol/L) for those with tophi until
ULT
complete dissolution is achieved

Initialdose is • Inhibits • First-linefor those with normal renal function


100 mg/day xanthine • Most commonly used dose: 300 mg/day
Increased oxidase th us • Adjust dose based on creatinine clearance
Allopurinol
at 100-mg blocking • Adverse effects: increased liver enzymes,
increments uric acid hypersensitivity reactions, bone marrow
every2-4 wks production depression, interstitial nephritis

• Inhibits • Indicated if SU A target cannot be


xanthine reached by allopurinol or if allopurinol
oxidase, not tolerated
Febuxostat • 40-80mg/d
thus blocking • Ideal for patients with renal insufficiency
uric acid • Adverse effects: GI intolerance,
production headache, increased liver enzymes

• Increases
, Potent inhibitor of CYP2C9
Benzbromarone • 50-200mg/d uric acid
• Not widely available locally
excretion

• Indicated in crystal-proven, severe/


debilitating chronic tophaceous gout &
• Pegylated poor quality of life, in whom SUA target
•8mgIVq2 recombinant cannot be reached with any other drugs
Pegloticase
weeks porcine at maximal dosage
uricase • Contraindicated in G6PD deficiency
• May induce immunogenicity :
• Not yet available locally _
Sources.Fitzgerald
et al 2020ACRGuideline for the Management of Gout.Arthnt1s
CareandResearch,
2020
RichelleP,et al. 2016EULARRecommendations forGout.AnnRheumDis;2017

D. Non-Pharmacologic/Adjunctive Management
° Control of body weight (low-impact & aerobic exercises lasting 45 minutes 4x/week)
0 Increased oral fluid intake (>8 glasses or >I.5 L of water per day unless contraindicated)
0 If possible, substitute loop diuretics or thiazides with ARBs or CCBs
0 Avoidance of heavy meals and excessive intake of meat and seafood
0 Limitation or avoidance of ethanol consumption, especially beer
0 Decreased intake of fructose corn syrup-containing food and beverage
0 Low-fat dairy products should be encouraged
0 Screen for associated comorbidities (e.g., renal impairment, CAD, heart failure, stroke,
PAD, obesity, hyperlipidemia, hypertension, and diabetes)
517
RHEUMATOID ARTHRITIS (RA)
I. ETIOPATHOGENESJS
Chronic inflammatory disease of unknown etiology marked by symmetric, peripheral
polyarthritis
Pathologic hallmark: pannus
Result in a variety of extraarticular manifestations (e.g., fatigue, subcutaneous nodules,
lung involvement, pericarditis, peripheral neuropathy, vasculitis, and hematologic
abnormalities)
Risk factors: genetics, female sex, and smoking (most important environmental factor)

II. CLINICAL MANIFESTATIONS


MANIFESTATIONS I REMARKS
• Initially involves small joints of hands and feet
• Early morning stiffness >I hour easing with physical activity
• Most frequently involved joints: wrists, MCP, PIP (DIP
involvement usually a sign of coexistent osteoarthritis)
• Swan neck deformity: hyperextension of the PIP with flexion of
the DIP joint
Joint involvement • Boutonniere deformity: flexion of the PIP with hyperextension
of the DIP joint
• Z-line deformity: subluxation of the first MCP with
hyperextension of the 1st IP joint
• Piano-key movement: tenosynovitis of extensor carpi ulnaris
with subluxation of distal ulna
• Flexor tendon tenosynovitis
Subcutaneous • Seen in 30-40%; usually benign, firm, nontender and adherent
nodules to periosteum, tendons, or bursae
• Most common cause of death in RA
Cardiovascular
• Pericarditis: most common site of cardiac involvement
disease
• Mitra! regurgitation: most common valvular abnormality
Pulmonary disease • Pleuritis: most common pulmonary manifestation
• Fever of >38.3°Cduring the clinical course should raise
Vasculitis
suspicion of systemic vasculitis
Constitutional • Weight loss, fever, fatigue, malaise, depression, and cachexia in
symptoms more severe cases

III. DIAGNOSIS
A. Diagnostic Tests
DIAGNOSTIC I COMMENTS/EXPECTED FINDINGS
•(+)RF or anti-CCP
Serum markers
• Elevated acute phase reactants
CBC • Normocytic normochromk anemia
Synovial fluid
• Consistent with inflammatory arthritis
analysis

• Juxtaarticular osteopenia (initial finding), soft tissue swelling,


Joint imaging joint effusions, symmetric joint space narrowing, joint
subluxation and collapse in severe cases

518
B. Classification Criteria for RA: Score 2:6 fulfills the requirements for Definite RA
CRITERIA* I SCORE
• 1 large joint (shoulder, elbow, hip, knee, ankle) 0
• 2-10 large joints 1
Joint 2
• 1-3 small joints (MCP, PIP, thumb IP, MTP, wrists)
involvement
• 4-10 small joints 3
• >IO joints (at least I small joint) 5
• Negative RF and negative anti-CCP antibodies 0
• Low-positive RF or low-positive anti-CCP antibodies
2
Serology (s3x ULN)

• High-positive RF or high-positive anti-CCP antibodies


3
(>3xULN)

Acute-phase • Normal CRP and normal ESR 0


reactants • Abnormal CRP or abnormal ESR 1

Duration of • <6 weeks 0


symptoms • 2:6weeks 1
MCP: Metacarpophalangeal RF: Rheumatoid Factor
PIP: Proximal lnterphalangeal Anti-CCP:Anti-CyclicCitrullinatedProtein Antibodies
MTP: Metatarsophalangeal CRP: C-Reactive Protein
ESR: Erythrocyte Sedimentation Rate

*Aimedat classifying newly presenting patients who have at least 1 joint with definite clinicalsynovi-
tis not better explained by another disease
Source: 2010Revised ACR-EULARClassification Criteriafor Rheumatoid Arthritis
D Aletaha et al. ArthritisRheum, 2010

IV. MANAGEMENT
Aim is to reach a target of sustained remission or low disease activity
Monitoring should be done every 1-3 months in active disease, with treatment adjusted
if no improvement seen by 3 months or target not reached by 6 months
Remission is defined as having all of the following:
0 Tender joint count SI
0
Swollen joint count SI
° CRP SI mg/dL
0 Patient global assessment scale SI
0 At any time point, patient must have a clinical disease activity index (CDAI) score of <2.8
Non-pharmacologic measures include dynamic strength training exercises & physical
therapy

A. NSAIDS
° Considered as adjunctive therapy

B. Glucocorticoids
0 Low-moderate doses for rapid disease control before the onset offully effective
DMARDs
0 1-10-2 week burst of glucocorticoids for acute disease flares

519
C. Conventional Synthetic Disease-Modifying Anti-Rheumatic Agents (csDMARDs)
0 Slow or prevent structural progression of RA
° Cornerstone of therapy which should be started as soon as diagnosis of RA is made
0 Exhibit delayed onset of action of around 6-12weeks
csDMARD I REMARKS I ADVERSE EFFECTS
• csDMARD of choice
• Folic acid I mg/day (or • Hepatotoxicity,
Methotrexate
5 mg/week) given to myelosuppression, infection,
10-25 mg/week
reduce toxicities nausea, diarrhea, interstitial
PO or SC
• Contraindicated in pneumonitis
pregnancy
• If with contraindication • Irreversible retinal damage,
Hydroxychloroquine
or early intolerance to rash, cardiotoxicity, blood
200-400 mg/day PO
methotrexate dyscrasia, nausea, diarrhea
• If with contraindication • Granulocytopenia, hemolytic
Sulfasalazine
or early intolerance to anemia in persons with G6PD
500-1500 mg BID PO
methotrexate deficiency, nausea, diarrhea
• Hepatotoxicity,
Leflunomide • Contraindicated in
myelosuppression, infection,
10-20 mg/day pregnancy
alopecia, diarrhea

D. Biologic or Small Molecule Inhibitor DMARDs


0Protein therapeutics designed mostly to target cytokines and cell-surface molecules
0Share the common adverse effect of a potentially increased risk for infection
BIOLOGIC
DMARD I MECHANISM

• Chimeric (part-mouse, part-human) anti-TNF monoclonal


Infliximab
antibody
Adalimumab;
• Fully humanized anti-TNF monoclonal antibody
golimumab

Certolizumab • Pegylated Fe-free fragment of a humanized monoclonal


pegol antibody with binding specificity for TNF
• Soluble fusion protein comprising the TNF receptor-2 in a
Etanercept
covalent linkage with the Fe portion of IgG1
Anakinra • Recombinant form of!L-1 receptor antagonist
• Soluble fusion protein consisting of a domain of human
Abatacept
CTLA-4 linked to a portion of human IgG
Rituximab • Chimeric monoclonal antibody against CD-20
Tocilizumab • Humanized monoclonal antibody against the IL-6 receptor
Tofacitinib • Small molecule inhibitor that targets )AKI and JAK3
Baricitinib • Small molecule inhibitor that targets )AKI and JAK2

520
INFECTIOUS ARTHRITIS
I. ETIOPATHOGENESIS
A. Pathogenesis
0 Hematogenous route is the most common route in all age groups
0The knee is the most commonly involved joint
0 Acute bacterial infection typically involves a single joint or a few joints
0 Subacute or chronic monoarthritis or oligoarthritis suggests mycobacterial or fungal
infection
0 Polyarticular involvement may be seen in RA

B. Etiologic Agents
0 Infants: Group B Streptococci, Gram(-) enteric bacilli, and Staphylococcus aureus
0 Staphylococcus aureus accounts for most non-gonococcal isolates in adults of all ages
0 N. gonorrhea if with high-risk behaviors

II. CLINICAL MANIFESTATIONS


Fever (may be absent in immunosuppressed patients)
• Moderate-severe pain that is uniform around the joint
Musculoskeletal: joint effusion, muscle spasm, decreased range of motion

III. DIAGNOSTICS
Definitive diagnosis of an infectious process relies on:
0 Identification of the pathogen in stained smears of synovial fluid, OR
0 Isolation of the pathogen from cultures of synovial fluid and blood, OR
0 Detection of microbial nucleic acids & proteins by nucleic acid amplification (NAA)-
based assays and immunologic techniques
DIAGNOSTIC I COMMENTS/EXPECTED FINDINGS
Acute phase
• Elevated ESR, CRP
reactants
CBC • Elevated WBC counts with neutrophilia
• Aspiration of synovial fluid (i.e., arthrocentesis) is essential
• Synovial fluid cell counts:
0 Normal: <180/uL (predominantly mononuclear cells)
0 Acute bacterial infecton: JOo,ooo/uL (range: 25,000-250,000/uL),
predominantly (>90%) neutrophils or PMNs
Synovial fluid 0 TB or fungal infection: 10,000-30,000/uL, with 50-70%
analysis neutrophils (remainder are lymphocytes)
° Crystal-induced, rheumatoid, or non-infectious inflammatory
arthritides: <30,000-50,000/uL
• Usually opaque, cloudy, with poor string sign (drops like water)
• Elevated LOH and total protein
• Decreased glucose
• Early findings: soft tissue swelling, joint space widening,
displacement of tissue planes by distended capsule
Joint imaging
• Late findings: effusions, joint space narrowing, joint subluxation and
collapse in severe cases
• Blood CS will be positive for Staphyloccoccus aureus in 50% of cases
Cultures
• Synovial fluid CS is positive for S. aureus in 90% of cases
Source:Jameson
JL,et al. Harrison's
Principles
of InternalMedicine,
20thedition;201B

521
IV. MANAGEMENT
A. Non-Pharmacologic
0Repeated arthrocentesis
0Surgical drainage/arthroscopic lavage usually indicated especially for:
• Septic hip
• Concomitant osteomyelitis
• Prosthetic joint infection

B. Pharmacologic (Antibiotics)
I. Recommen d e dE mp1ricAnti b"1otics

• Cefazolin 2 g IV q8; OR
Gram-positive smear,
• Oxacillin 2 g IV q4; OR
MRSA unlikely
• Nafcillin 2 g IV q4
Gram-positive smear,
• Vancomycin I g IV q12
MRSA!ikely
Gram-negative smear or no • Cefotaxime I g IV q8; OR
organisms on smear • Ceftriaxone 1-2 g IV q24
• In addition to the above, add an aminoglycoside or 3rd
Pseudomonas suspect
generation anti-pseudomonal cephalosporin
Source:JamesonJL,et al. Harrison'sPnncIples
of InternalMedicine,20thedition;2018

2. Duration o fS,peci fic Treatment Accor d ing to 0 rgamsm


• 4 weeks of cefazolin, oxacillin or nafcillin (if not MRSA)
Staphylococcus aureus
• 4 weeks ofvancomycin (ifMRSA)
Pneumococcus/Penicillin- • 2 weeks penicillin-G 2M units q4 (penicillin-sensitive)
sensitive streptococcus or cefotaxime
Haemophilus influenzae/
• 2 weeks cefotaxime or ceftriaxone
Penicillin-resistant streptococcus

• 3-4 weeks 2nd/3rd generation cephalosporin IV or


Enteric gram-negative bacilli
fluoroquinolone IV/PO (e.g., levofloxacin)
• At least 2 weeks of aminoglycoside plus extended
Pseudomonas aeruginosa spectrum penicillin or anti-pseudomonal cephalosporin;
continued for another 2 weeks if tolerated
Source:JamesonJL,et al. Hamsons Principlesof InternalMed1c1ne,
20thedItIon;2018

SPONDYLOARTHRITIDES
DISEASE I COMMENTS/EXPECTED FINDINGS
• Autoimmune disorder of the axial skeleton mostly affecting young males
Ankylosing
• Associated with the HLA-B27 gene; earliest manifestation is sacroiliitis
spondylitis
• Characteristic "bamboo spine" appearance on imaging in advanced stages

• Inflammatory autoimmune musculoskeletal disease characteristically


seen in patients with psoriasis
Psoriatic
• In 60-70% of cases, psoriasis precedes joint disease
arthritis
• Can have both axial (asymmetric sacroiliitis) and peripheral (DIP
arthritis, nail involvement) manifestations

• Acute non purulent arthritis complicating an infection elsewhere in the body


Reactive
• Associated pathogens include Salmonella spp., Shigella spp., Yersinia
arthritis
spp., Campylobacter jejuni and Chlamydia trachomatis
522
SYSTEMIC LUPUS ERYTHEMATOSUS (SLE)
I. ETIOPATHOGENESIS
Autoimmune disease in which organs and cells undergo damage mediated by tissue-
binding autoantibodies and immune complexes
Mostly affects women of child-bearing age; can still affect children, elderly, males
Diagnosis is based on characteristic clinical features and autoantibodies

II. CLINICAL MANIFESTATIONS AND DIAGNOSIS

2019 EULARIACR Classification Criteria for Systemic Lupus Erythematosus (SLE)

Entry criterion
Antinuclear antibodies (ANA) at a titer of ;,:J:80 on HEp-2 cells or
an equivalent positive test (ever)

Present l Absent I Do not classify as SLE

Apply additive criteria:


Do not count a criterion if there is a more likely explanation than SLE.
Occurrence of a criterion on at least one occasion is sufficient.
SLEclassification requires at least one clinical criterion and ;?:10points.
Criteria need not occur simultaneously.
Within each domain, only the highest weighted criterion is counted toward the total score.

CLINICAL DOMAIN SCORE IMMUNOLOGICAL DOMAIN SCORE


Constitutional Antiphospho/ipid antibodies
Fever 2 Anticardiolipin antibodies
Hematologic OR
Leukopenio 3 Anti-~2GP1 antibodies 2
Thrombocytopenia A OR
Autoimmune hemolysis A Lupus anticoagulant
Neuropsychiatric Complement proteins
Delirium 2 • Low C3 OR low CA 3
Psychosis 3 • Low C3 AND low CA 4
Seizure 5 SLE-specific antibodies
Mucocutaneous Anti-dsDNA antibody OR
• Anti-Smithantibody 6
Non-scarring alopecio 2
Oral ulcers 2
Subacute cutaneous
OR discoid lupus A
Acute cutaneous lupus 6
Serosal
Pleural or pericardial 5
effusion
Acute pericarditis 6
Musculoskeletal
Joint involvement 6
Renal
Proteinuria>0.5 g/2A hr A
Class ti or V lupus
nephritis on biopsy 8
Class Ill or IV lupus 10
nephritis on biopsy

Classify as SLE with a score of 10 or more, if entry criterion fulfilled


Source:Aringeret al:Ann RheumDis.2019

523
Sample Case:
A 30/F was referred for consideration ofSLE. Physical examination revealed pallor with oral ulcers
and discoid rash behind the ears. Blood tests revealed ANA titer of 1:80and Hgb of 105mg/dL with(+)
direct Coombs' test, while 24-hour urine collection revealed proteinuria of 1g.
• Since the patient fulfillsthe entry criterion with an ANA titer oh1:80, apply additive criieria for SLE:
0 4 points: autoimmune hemolysis
0 4 points: discoid lupus (this is the higher weighted criterion for the mucocutaneous category, so
we do not count the 2 points for oral ulcers)
• 4 points: proteinuria >0.5g/24h
• Since patient has a total of12 points & has at least I clinical criterion, she is classified as having SLE

III. MANAGEMENT

• Risk for aseptic meningitis, renal insufficiency,


NSAIDS • Arthritis
elevated transaminases & cardiovascular events
Topical steroids • Dermatitis • Skin atrophy, hypopigmentation, dermatitis
Sunscreens • Photosensitivity • At least SPF 30; SPF 50 preferred
Calcium&
• Bone health • None when taken in appropriate doses
vitaminD
• Hepatotoxicity, bone marrow suppression,
pulmonary fibrosis
Methotrexate
• Contraindicated in pregnancy; stop 3-6 months
before trying to conceive
• Dermatitis, • A mainstay that helps prevent flares,
arthritis thrombosis, and morbidity
• Potential retinal toxicity at high & prolonged
Hydroxychloroquine
doses
• QT prolongation on ECG increases risk for
cardiac arrhythmias in predisposed patients
• lmmunosuppression, HPN, hyperglycemia,
Systemic steroids/ hypokalemia, acne, aseptic necrosis of bone/
Methylprednisolone osteoporosis, fragile skin, mood swings/
psychosis, Cushing syndrome
• Predisposition to infection, bone marrow (BM)
Cyclophosphamide suppression, hemorrhagic cystitis, bladder CA
• Contraindicated in pregnancy
• Control of • Predisposition to infection, BM suppression,
Azathioprine disease activity alopecia, hepatotoxicity, flu-like illness
• May use in pregnancy if benefits outweigh risks
• Predisposition to infection, BM suppression,
Mycophenolate lymphoproliferative disorders, GI symptoms,
mofetil tremors, rash
• Contraindicated in pregnancy
Biologics (Belimumab; • Predisposition to infections, TB reactivation,
Rituximab - off-label) infusion reactions

B. Methylprednisolone Pulse Therapy (MPPT)


• Allows rapid delivery of large IV corticosteroid doses during severe life-threatening
SLE flares such as life-threatening nephritis and neuropsychiatric SLE
0 Dose: Methylprednisolone 500 mg - 1 g per day x 3 days
0 Side effects: infections (most common), neuropsychiatric complications (seizures, mania,
psychoses), arthralgia and myalgia, arrhythmias, increased risk for osteoporotic fractures
and avascular necrosis, and suppression of the hypothalamic-pituitary-adrenal axis
524
ANTIPHOSPHOLIPID SYNDROME (APS or APAS)
I. ETIOPATHOGENESIS
Autoimmune disorder characterized by venous or arterial thrombosis &Jor pregnancy
morbidity with persistent laboratory evidence of antiphospholipid antibodies (aPL)
It may occur as a primary condition or may occur in the presence of SLE or other
systemic autoimmune diseases
Presence of aPL-binding plasma protein antibodies places patients in a prothrombotic state
Trigger for induction of antibodies to PL-binding proteins is unknown but infections,
oxidative stress, surgery & discontinuation of anticoagulants may induce exacerbation

II. CLINICAL MANIFESTATIONS


APS should be suspected in the following situations:
0 Occurrence of one or more unexplained venous or arterial thrombotic events,
especially in young patients (e.g., stroke in the young)
0 Occurrence of I or more specific adverse outcomes related to pregnancy (e.g., fetal death
after IOweeks AOG, premature birth due to severe eclampsia, or placental insufficiency)
0 Otherwise unexplained thrombocytopenia or prolongation of coagulation parameters
MANIFESTATIONS
I SPECIFIC EXAMPLES

Venous thrombosis and • Deep vein thrombosis, livedo reticularis, pulmonary embolism,
related consequences superficial thrombophlebitis & thrombosis in various other sites
• Stroke, transient ischemic attack, myocardial ischemia, leg ulcers/
Arterial thrombosis and
digital gangrene, retinal artery thrombosis/amaurosis fugax,
related consequences
avascular necrosis of bone, multi-infarct dementia
Neurologic • Migraine, epilepsy, chorea, cerebellar ataxia, transverse myelopathy
Renal • Renal artery/vein thrombosis, fibrous intima hyperplasia
Osteoarticular • Arthralgia, arthritis
Obstetric & fetal • Pre-eclampsia, eclampsia, fetal loss, premature birth
Hematologic • Thrombocytopenia, autoimmune hemolytic anemia

III. DIAGNOSIS: REVISED SAPPORO APS CLASSIFICATION (SYDNEY) CRITERIA


The Sapporo Criteria were revised during the 11th International Congress on aPL
• Diagnosis of APS requires at least I clinical criteria and I laboraIOry criteria
CLINICAL CRITERIA I LABORATORY CRITERIA

Vascular Thrombosis Lupus Anticoagulant (LA)


• ;:,1episode/s of arterial, venous or small • Present in plasma, on ;:,2occasions at least
vessel thrombosis, in any tissue or organ 12weeks apart

Pregnancy Morbidity Anticardiolipin Antibody (aCL)


• l!I unexplained death/sofa • IgG and/or IgM isotype in serum or
morphologically normal fetus at or beyond plasma, present in medium/high titer, on
10th week AOG, OR l!2 occasions, at least 12weeks apart
• l!I premature birth/sofa morphologically
normal neonate before 34th week
AOG because of eclampsia or severe
preeclampsia or recognized features of Anti-Beta-2 Glycoprotein-I Antibody
placental insufficiency, OR • lgG and/or lgM isotype in serum or
• ;,3 unexplained consecutive spontaneous plasma, present on ;:,2occasions, at least 12
abortions before IOth week AOG, weeks apart
with maternal anatomic or hormonal
abnormalities and paternal/maternal
chromosomal causes excluded
ource: ameson , et a . arrisons

525
IV. MANAGEMENT OF APAS
• Thrombotic event: indefinite anticoagulation with warfarin with INR target of 2.5-3.5,
alone or in combination with 80 mg aspirin
Prevention of pregnancy morbidity:
0Heparin in combination with 80 mg aspirin
0IVIg 400 mg/kg daily for 5 days
0Glucocorticoids are ineffective
Recurrent thrombosis despite anticoagulation: IV!g
Direct oral anticoagulants (DOA Cs) are not recommended due to the risk of recurrent
thrombotic events

REFERENCES
I. Aringer M, Costenbader K, Daikh D, Brinks R, Mosca M, Ramsey-Goldman R, et al. 2019 European League Against Rheumatism/
American College of Rheumacology classification criteria for systemic lupus erythematosus. Ann Rheum Dis. 2019
Sep;78(9J,1151-1159.
2. Badsha H. and Edwards CJ.Intravenous pulses of mechylprednisolone for systemic lupus erythematosus. Seminars in Arthritis
and Rheumarism. 2003; 32(6),370-3n-
J. Barton JL,CriS\vell LA, Kaiser R, Chen YH, and Schillinger D. Systematic review and meta-analysis of patient self-repon versus
trained assessor joinc counts in rheumatoid anhritis. The Journal ofRheumatology. 2009 Dec; 36(12):2635-41.
4. Bruyere 0, Honvo G, Veronese N, Arden NK, Branco J, Curtis EM, et al. An updated algorithm recommendation for the
management of knee osteoanhritis from the European Society for Clinical and Economic Aspects of Osteoporosis,
Osteoanhritis and Musculoskeleral Diseases (ESCEO). Semin Arthritis Rheum. 2019 Dec;.49(3):337-350.
5. Cush JJ,Dao KH. Polyanicular anhritis. Musculoskeletal Key.Online: musculoskeletalkey.com. Accessed on August 29, 2021.
6. Fernandes L, Hagen KB, Bijlsma JWJ, Andreassen 0, Christensen P, Conaghan PG, et al. EULAR recommendations for the non-
pharmacological core management of hip and knee osteoanhritis. Ann Rheum Dis. 2013;72(7):1125-35.
7. FitzGerald, JD, Dalbeth N, Mikuls T, Brignardello-Petersen R, Guyan G, Abeles AM, et al. 2020 American College of
Rheumacology Guideline for the Management of Gout. Arthritis Care Res (Hoboken). 2020 Jun;72(6):744-760.
8.Greenhalgh T. Facts About Gout: Debunking Common Myths. Available online: http://www.rheumatologyadvisor.com
9. Imboden JB, Hellmann DB, and Stone JH. CURRENT Diagnosis & Treatment; Rheumatology, 3rd edition. USA: The McGraw-
Hill Companies, Inc.; 2013
10. Jameson JL. Kasper DL. Longo DL. Fauci AS, Hauser SL. Loscalzo J.Harrison's Principles of Internal Medicine. 20th Edition.
New York: McGraw Hill Education, 2018.
11. Kasper DL. Fauci AS, Hauser SL. Longo DL. Jameson JL and Loscalzo J (editors). Harrison's Principles of Internal Medicine,
20th ed. McGraw Hill Education.
12. Li RR. Yu K, and Li CW. Dietary factors and risk of gout and hyperuricemia: a meta-analysis and systematic review. Asia Pac J
Clin Nutr. 2018; 27(6):1344-56.
13. Li-YuJ, Salido EO, Manahan S, Lichauco JJ,Lorenzo JP, Torralba KT, et al. Philippine Rheumacology Association. Philippine
Clinical Practice Guidelines for the Management of Gout PRA; 2008.
14. Lim W. Antiphospholipid syndrome. Hematology Am Soc Hematol Educ Program2013;2013:675.
15. Miyakis S, Lockshin MD, Atsumi T, Branch DW, Brey RI..,Cervera R, er al. International consensus statement on an update of
the classification criteria for definite antiphospholipid syndrome (APS). J Thromb Haemost 2006; 4:295.
16. Penserga EZ, Salido EO, Del Rosario AG, Reyes HM, Perez E, Dytan A, e1 al. Knee Os1eoarthric.is Clinical Practice Guidelines
Technical Working Committee. Philippine Rheumatology Association. Philippine Clinical Practice Guidelines for the Medical
Management of Knee Osteoanhriris. PRA; 2009.
17. Petri M, Orbai AM, Alarcon GS, Gordon C, Merrill JT, Fortin PR, et al. Derivation and validation of the Systemic Lupus
International Collaborating Clinics classification criteria for systemic lupus etythematosus. Anhricis Rheum. 2012 Aug;
64(8J,z6n-s6.
18. Richeue P, Doherty M, Pascual E, Barskova V, Becce F, Castaneda-Sanabria J. et al. zo16 Updated EULAR Evidence-Based
Recommendations for the Management of Gout Annals of the Rheumatic Diseases. 2017;76(1):29-42.
19. Smolen JS, Landewe R. Bijlsma J, Burmester G, Chatzidionysiou K, Dougados M, et al 2016 Update of the EULAR
Recommendations for the Management of Rheumatoid Arthritis with Synthetic and Biological Disease Modifying Anti-
Rheumatic Drugs. Annals of the Rheumatic Diseases. 2017;76(6):96o-9n
20. Tikly M and Makda M. A diagnostic approach to the common anhritic conditions. South African Family Practice 2009; 51:88-
193.
21. Wallace SL Robinson H, Masi AT, Decker JL,McCarty DJ, and Yu TF. Preliminary criteria for the classification of the acute
arthritis of primary gout Arthritis Rheum. 19n: 20(3):895-900.

526
IMMUNOL
[I] BASIC CONCEPTS IN IMMUNOLOGY

0 COMMON CONDITIONS IN ALLERGOLOGY & IMMUNOLOGY


1. Urticaria & Angioedema
2. Allergic Rhinitis
3. Atopic Dermatitis
4. Anaphylaxis
THE IMMUNE SYSTEM
The human immune system is a highly evolved network of processeswhose primary goal is to
protect the host from pathogenic microbesand eliminate toxic or allergenicsubstances while at
the same time, avoiding responsesthat produce excessivedamage to "self"tissues.

I. KEY PROPERTIES OF THE IMMUNE SYSTEM


• Specificity: highly diverse repertoire of antigen receptors which enables recognition of a
nearly infinite range of pathogens
Memory: immune memory to enable rapid recall immune responses
Self-discrimination: immunologic tolerance to prevent immune damage to normal "self" tissues

II. INNATE AND ADAPTIVE IMMUNE SYSTEM


Human immunity has two primary constituents: the innate and adaptive immune systems
First line of defense against non-self pathogens: innate, or non-specific, immune response
Second line of defense: adaptive (acquired) immune response, hallmark of which is
clonal expansion oflymphocytes

III. IMMUNOPATHOLOGY
Immunopathology is defined as inappropriate immune responses associated with disease
It can take various forms such as:
0 Hypersensitivity or allergy: collateral damage to host tissues by immune system
0 Autoimmunity: loss of self-discrimination such that the host produces an abnormal
response to its own tissue
0 Lymphoproliferative disease: uncontrolled production oflymphocytes that causes
monoclonal lymphocytosis, lymphadenopathy, and bone marrow infiltration
0 Immunodeficiency: immune system's ability to fight microbes is compromised or absent

, 5ueDama e

MECHANISM PHY
••
I •
• Extravascular
Type! recruitment of • Anaphylaxis
• Immediate, immunological • Allergic asthma • Chronic urticaria
IgE-mediated components • Allergic rhinitis
• Parasite expulsion
• Incompatible blood • He1nolytic anen1ia
transfusion • Thrombocytopenia
Type II • Lysis of pathogens
• Hemolytic disease of • Pemphigoid
• IgG,IgAor by extracellular or
the newborn • Good pasture's disease
IgM-mediated intracellular events
• Hyperacute graft • Myasthenia gravis
rejection • Thyrotoxicosis
• Local: arthus
• Neutralization of • Rheumatoid arthritis
Type III reaction, dermatitis
patho1~en-derived • Systemic lupus
• Immune herpetiformis, allergic
factors (e.g., toxins) erythematosus (SLE)
complex- alveoli tis
• Transport of antigen • Widespread
mediated • Systemic: serum
to germinal centers vasculitis
sickness, vasculitis
• Tuberculosis • Thyroiditis
• Defense against
Type IV • Leprosy • Adrenali tis
intracellular
• Cell-mediated • Contact dermatitis • Pernicious anen1ia
parasites
• Graft rejection • Diabetes
Source:ToddI, et al. LectureNotes,n Immunology
6thed1t1on. USA:WileyBlackwell; 2010
JamesonJL,et al. Harrison'sPrinciples of InternalMedicine,20thedition;2018
MarshallJS,et al.Allergy& ClinicalImmunology.2018
529
COMMON C©NDITIONS IN ALLERGOLOGY & IMMUNOLOGY
URTICARIA AND ANGIOEDEMA
I. ETIOPATHOGENESIS
A. Urticaria versus Angioedema
° Fundamentally similar pathophysiologies but occurring at different levels of the skin
• Urticaria: dilation of vascular structures in superficial dermis
• Angioedema: involves deeper dermis and subcutaneous tissues
° Can occur at any age but chronic form most commonly occurs from 3rd-5th decade oflife
0More common in women; slight predominance for those with history of atopy

B. Acute versus Chronic Urticaria

I DURATION I ETIOLOGY
Acute • <6 weeks • Often results from exposure to food, environmental
urticaria duration allergens, drugs, or infection (especially viral)

• Often idiopathic
Chronic • >6 weeks
• Collagen vascular disease, physical stimuli, and
urticaria duration
hereditary conditions (e.g., hereditary angioedema)

II. CLINICAL MANIFESTATIONS


FEATURE I URTICARIA I ANGIOEDEMA
• Any body area
• Most common sites: periorbital
Location • Frequently migrates around the
and perioral
body

• Well-circumscribed wheals with • Characterized by dramatic swelling


erythematous raised serpiginous • Presents with more pain than
borders & blanched centers pruritus; may develop with a
Presentation
• May coalesce to become giant wheals pruritic prodrome
• Old lesions fade as new ones appear • Minimal erythema
• Intensely pruritic • Lasts for hours to days
• May be life-threatening if involving
Prognosis • Leaves no bruising or scarring upper respiratory tract
• Usually leaves no residual scarring

Ill.DIAGNOSIS
Diagnosis is usually clinical for self-limited episodes (history alone may be enough) &
usually followed by confirmatory skin testing or assaying for serum allergen-specific IgE
DIAGNOSTIC I USUAL INDICATIONS
Direct reproduction of • Physical urticarias (e.g., dermographism, pressure urticaria, cold
lesions urticaria, cholinergic urticaria)
CBC (for eosinophilia),
• Chronic urticaria
ESR,TSH
• Indications:
0 Lesions that last for >36 hours

Skin biopsy 0 Those that result in scarring

0 Those reported to be painful and not pruritic (to check for

urticaria! vasculitis)
Complement levels • Chronic angioedema without urticaria
Evaluation of parasites • Appropriate travel history
530
IV. MANAGEMENT
I •
I REMARKS
H1 antihistamines (long-
• Usually used first and then titrated up
acting, non-sedating)
H2 antihistamines • May be added if response to H1 antihistamines is inadequate
CysLl) receptor antagonists • May be used as add-on therapy
Monoclonal anti-IgE
• For refractory chronic urticaria
antibodies (e.g., omalizumab)
• Generally avoided in idiopathic, allergen-induced, or
physical urticarias due to long-term toxicity (no benefit)
• Systemic glucocorticoids useful for those with:
Topical glucocorticoids &
0 Pressure urticaria, vasculitic urticaria, idiopathic
systemic glucocorticoids
angioedema with or without urticaria
° Chronic urticaria unresponsive to conventional treatment
0 Any patient with debilitating disease
Hydroxychloroquine,
• Persistent vasculitic urticaria
dapsone, or colchicine
Cyclosporine • Severe and refractory chronic idiopathic urticaria

ALLERGIC RHINITIS (AR)


I. ETIOPATHOGENESIS
Characterized by a constellation of symptoms (sneezing, rhinorrhea, obstruction of the
nasal passages, conjunctival/nasal/pharyngeal itching, and lacrimation) all occurring in
a temporal relationship to allergen exposure
Has perennial (PAR) and seasonal (SAR) forms depending on allergen exposure temporality

.. . ...
Manifestations caused by sensitization of lgE-rich intraepithelial mast cells with allergens
DEFINITION
Seasonal allergic • Usually caused by seasonal exposure to airborne pollens
rhinitis (SAR) (e.g., weeds, grasses, trees)

• Chronic exposure to house dust mites, animal dander, or


Perennial allergic
insect (cockroach) products
rhinitis (PAR)
• Commonly develop in adult life

JI. CLINICAL MANIFESTATIONS


Hallmarks of allergic rhinitis: episodic rhinorrhea, sneezing, obstruction of the nasal
passages with lacrimation, pruritus of the conjunctiva, nasal mucosa, and oropharynx
Nasal mucosa becomes pale and boggy
Conjunctivae become congested and edematous

FREQUENCY OF . . • .
A. Classification of Allergic Rhinitis {based 011AllergicRhinitis & its Impact on Asthma; ARIA}
.
• Normal sleep
• <4 daysa week,OR • No impairment of daily
INTERMITTENT MILD
• <4 weeks activities,work and school
• No troublesome symptoms
• Abnormal sleep
• 2:4daysa week,AND MODERATE-
• Impairment of daily
PERSISTENT SEVERE
• 2:4weeks activities,work and school
(any of theff):
• Troublesome symptoms
Source:Bousquet
J, et al.ARIA2008Update.Allergy2008
531
B. Visual Analogue Scales (VAS)
0Visual analogue scales are now used in to classify symptom severity and disease control
0These are psychometric response scales used to assess for subjective characteristics or
attitudes of disease-related symptom severity in each patient
0VAS score <!5suggests moderate-severe allergic rhinitis
0AR patients mark a point in a horizontal line that best corresponds to the severity of
their symptoms (or current status of disease control)

No.al qmptonu
• Congestion

,.
0 2 3 4
How are you
6 7 8 9
,.
10
Very
• Itching
• Secretion
• Sneezing
Nottroubl11som11
otoll
feeling today? troublesome Oeulor J)'l'lptom1
· Redness
Think about how troublesome your symptoms • Woteryey111
hove been for the last 24 h • Itching

...
- W 9 8 7 6 5 4 3 2 0
Not 1ro!1,1ome Classification of v:ry
0 cit oU allergic rhinitis control troubl,some

~--============--"'"'_:~_'·-'---,
Source:KlimekL, et al.AllergoJournalInternational;
2017
SybilskiAJ. PediatrMedRodz.2018
III. DIAGNOSIS
Diagnosis depends on a history revealing characteristic symptoms coincident with triggers
(e.g.,pollen, animal dander, house dust mite, work-related allergens) & findings on examination

A. Clinical Diagnosis
• Diagnosis usually established if 2 or more of the following are identified, lasting >I hour/
day for >2 weeks/year with an allergen-mediated cause of these symptoms
Nasal obstruction
0

Rhinorrhea
0

Sneezing
0

Nasal itching
0

Source:KlimekL, et al.AllergoJournalInternational; 2017

Nasal secretions • Secretions are rich in eosinophils

CBC • May display modest eosinophilia


• lntracutaneous route (puncture or prick) with allergens of interest
provides a rapid and reliable approach to identifying allergen-specific
IgE that has sensitized cutaneous mast cells
• Positive intracutaneous skin test with 1:10-1:20 weight/volume of extract
Skin test
has a high predictive value for the presence of allergy
• lntradermal test ¼ith 1:500-1:1000 dilution of 0.05 mL may follow if indicated
by history when intracutaneous test is negative (more sensitive, but less
reliable due to reactivity of some asymptomatic individuals with test dose)

• Newer method for detecting IgE using enzyme-linked immunosorbent


Total and
assays (ELISA)
specific IgE
• Total lgE frequently elevated
532
IV. MANAGEMENT
Allergen avoidance: most effective form of management (not always feasible)
• Pharmacologic agents represent standard approach to seasonal or perennial allergic rhinitis

A. Choice of Pharmacotherapy

..
0 Selection of pharmacotherapy now based on visual analogue scale (VAS) & patient preference
0 Usual initial re imens:
INITIAL REGIMEN I OTHER OPTIONS
Seasonal allergic • lNCS + lNAH (might act faster
rhinitis (SAR) • lNCS + OAH or lNCS alone than INCS alone)
Perennial allergic
rhinitis (PAR) • INCS alone • INCS+INAH

INCS:intranasalcorticosteroids 0AH:oral antihistamines INAH:intranasalantihistamines


Importantcharacteristics of certain pharmacologicagents and regimens
• 0AH are less potent than INCS (but many patients prefer oral drugs)
• INAHare less effectivethan INCS
• INAHare effectivewithinminutes, while onset of action of INCStakes a few hours to a few days
• INCS should continue being prescribed as first-linetherapy in patients with moderate-to-severe rhinitis
• INCS+ 0AH combinationgenerally offers no advantages over INCS
• INCS+ INAHcombinationmore effectivethan INCS alone and is effective withinminutes

B. Classification of Treatment Used in Patients with Allergic Rhinitis


0 ARIA algorithm for AR was revised and a proposal was made to classify treatments
0 Presented in tiers/steps and used in conjunction with the ARIA algorithm (e.g., a patient in
T2 treatment requiring STEP UP is recommended to use T3 drugs)
• Patients with mild symptoms are usually started on TI drugs
• Patients with moderate-severe symptoms and/or persistent AR are started on T2 drugs

• Non-sedating H1 antihistamines (oral, intranasal, and ocular), leukotriene receptor


TI
antagonists, or cromones (intranasal and ocular)
T2 • Intranasal corticosteroids
• Intranasal corticosteroids+ intranasal azelastine
• Oral corticosteroids (as short course and add-on treatment)
• Consider referral to a specialist and allergen immunotherapy

C Ph "M aement
CLASS I
• Oral: cetirizine, desloratadine,
• Blocks H1 receptors (onset is 15-
fexofenadine, levocetirizine, loratadine
Antihistamine 30 mins for oral agents, 15mins
• Intranasal: azelastine, olopatadine
for intranasal agents)
• Ocular: azelastine, olopatadine
• Intranasal: budesonide, ciclesonide,
flunisolide, fluticasone furoate,
• Inhibits inflammatory cells
Corticosteroid fluticasone proprionate, mometasone,
• INCS: onset of action <30 mins
triamcinolone, beclomethasone
• Oral: prednisone
Cromone • Intranasal cromolyn • Inhibits histamine release
Leukotriene • Blocks leukotriene receptors
receptor • Montelukast, zafirlukast, pranlukast
• Less potent than INCS
antagonist
• Oral: pseudoephedrine, • Acts on adrenergic receptors
phenylephrine causing vasoconstriction
Decongestant
• Topical: oxymetazoline, • Prolonged use (>3-5 days) is not
phenylephrine, xylometazoline recommended
533
V. ASSESSMENT OF CONTROL
An algorithm based on the VAS was devised by ARIA expert group for selection of pharmaco-
therapy for patients with AR and to step up or step down treatment based on control

A. Assessment of Control in Untreated Symptomatic Patients

VAS <5 VAS;;e5

Initiate treatment
Initiate treatment
Intermittent rhinitis: Any first-line agent
Any first-line agent
Persistent rhinitis: INCS or INCS + AZE

VAS <5 Re-assess VAS oi y VAS ;;,5


~-------l up to Day 3 of f--------~
s m toms
Step up treatment and
Check for symptoms 1--------------, re.assess VAS doily up to Doy 7
VAS <5
VAS~
Symptomatic Asymptomatic

Consider step
Continue
treatment
down from Consider SIT
current regimen

First-lineagents:H1antihistamines
or intranasalcorticosteroids
(INCS)or INCS+ azelastine(AZE)
SIT:specificimmunotherapy
ConsiderINCS+ AZEif previoustreatmentis historicallyineffective

B. Assessment of Control in Treated Symptomatic Patients

VAS <5 VAS ;;,5

First-linetreatment Step up treatment


Anti-HJ or /NCS or /NCS + AZE INCS or INCS + AZE

Intermittent rhinitis, Persistentrhinitis,


no allergen exposure or allergen exposure
Step down treatment or STOP Maintain or step up treatment

~V_A_S_<_S
__ 5______
--< Re.assess VAS daily up >-V_A_S_"
to Doy 3 of symptoms

VAS <S
Check for symptoms

Sympfomafic
Step up treatment and
Continue treatment
re-assess VAS doily

Asymptomatic
Consider step down
Consider SIT
from current regimen

First-lineagents:H1 antihistamines
or intranasalcorticosteroids
(INCS)or INCS+ azelastine(AZE)
SIT:specificimmunotherapy
Source:BousquetJ, Schunemann
HJ,TogiasA, et al. J AllergyClinlmmunol.2020

534
ATOPIC DERMATITIS {AD)
I. ETIOPATHOGENESIS
AD is the cutaneous expression of the atopic state (i.e., family history of asthma, allergic
rhinitis, or eczema)
Endogenous eczema: "the itch that rashes"
Though its etiology is still not fully elucidated, there is a clear genetic predisposition
Begins in infancy with many outgrowing AD as they develop allergic respiratory symptoms
Pathogenesis: genetics, decreased skin barrier function, altered immunology

II. CLINICALFEATURES
A chronic or relapsing disease characterized by pruritus and eczematous lesions with a
distinctive morphology
It has an age-specific distribution and has three stages
S,pec1'fi C D'1stn'b ut1on
A A,11:e-
• Face (prominent on cheeks with sparing of central face), scalp,
Infantile
extensors with sparing of diaper area
(<2 years)
• Features are more acute

• Flexural areas
Childhood
• More chronic lesions
(2-12years)
• More pronounced and widespread xerosis
• Also flexural with chronic lesions but can also present as chronic
Adult
hand dermatitis, facial dermatitis with severe eyelid involvement,
(>12years)
extensive or erythrodermic diseases

BT empora !Ch aractenst1cs o fSk' m L es1ons


• Erythematous papules with excoriations
Acute lesions
• Vesicles over erythematous skin and serous exudate
Subacute lesions • Erythematous, excoriated, scaling papules
Chronic lesions • Thickened plaques of skin, lichenification, prurigo nodularis

III. DIAGNOSIS: BASED ON THE CONSTELLATION OF CLINICAL FINDINGS

• Xerosis • Dennie-Morgan infraorbital fold


• lchthyosis, palmar hyperlinearity, keratosis • Keratoconus
pilaris • Anterior subcapsular cataract
• Immediate (type I) skin test reactivity • Orbital darkening
• Elevated serum lgE • Facial pallor/erythema
• Early age of onset • Pityriasis alba
• Tendency toward cutaneous infections/ • Anterior neck folds
impaired cell-mediated immunity • Pruritus when sweating
• Tendency toward non-specific hand or foot • Intolerance to wool and lipid solvents
dermatitis • Perifollicular accentuation
• Nipple eczema • Food intolerance
• Cheilitis • Course affected by environmental factors
• Recurrent conjunctivitis • White dermographism/delayed blanch
Source:HanifinJM,et al.ActaDermVenerealSuppl.1980
535
IV. MANAGEMENT
Appropriate skin hydration and use of emollients: mainstay of management
Avoidance of irritants
Identification and avoidance of proven allergens
• Identification/treatment of complicated/superimposed bacterial, viral or fungal infections
Anti-inflammatory therapy: topical glucocorticoids, topical calcineurin inhibitors
• May give sedating antihistamines (hydroxyzine 25-50 mg tab OD HS)

ANAPHYLAXIS
I. ETIOPATHOGENESIS
Most severe clinical presentation of acute systemic allergic reactions
Hallmark is the onset of some manifestation within seconds to minutes after
introduction of the antigen (with the exception of alpha-galactose allergy)
• Angioedematous & urticaria! manifestations are attributed to the release of endogenous
histamine

A. Risk Factors for Poor Outcome


0 Age-related factors: infants, elderly, pregnancy
° Concomitant disease: asthma and other respiratory diseases, cardiovascular disease,
mastocytosis, allergic rhinitis, eczema, psychiatric illness
° Concurrent drug intake: beta-blockers, ACE-inhibitors, ethanol, sedatives, hypnotics,
antidepressants
° Co-factors that amplify anaphylaxis: exercise, acute infection, emotional stress,
premenstrual status

B. Mechanisms
lgE-mediated Anaphylaxis I Non-lgE-mediated Anaphylaxis
• Anaphylaxis is triggered by interaction of an • May be immunologic or
allergen with the allergen-specific IgE/high-affinity non-immunologic
receptor (FcERI)complex expressed on effector • Immunologic: may involve
cells (predominantly mast cells and basophils) complement system
leading to intracellular signaling and release of (anaphylatoxins: C3a and C5a),
preformed and de nouosynthesis of mediators contact and coagulation system,
• Most frequent mechanism orlgG

II. CLINICAL MANIFESTATIONS


SYSTEM I MANIFESTATIONS
Cutaneous/ • Pruritus, urticaria, angioedema, flushing
mucosa) • Swelling of the conjunctiva, lips, tongue, throat
• Laryngeal edema: hoarseness, "lump in the throat"
• Intense bronchospasm: dyspnea, stridor, wheezing
Respiratory
• Reduced peak expiratory flow
• Hypoxemia
Gastrointestinal • Nausea, vomiting, crampy abdominal pain, diarrhea
• Hypotension, tachycardia, palpitations
Cardiovascular
• Cardiac arrest
• Aura of impending doom, uneasiness, throbbing headache, altered
CNS
mental status, dizziness, confusion
• Metallic taste in mouth
Others • Cramps and bleeding due to uterine contractions in females
• Loss of bladder control
Source:CardonaV,AnsoteguiIJ, EbisawaM,et al. WAOJournal.2020
536
III. DIAGNOSIS
Diagnosis depends on a history revealing the onset of symptoms and signs within
minutes after the antigen is encountered
Appropriate to rule out: complement-mediated immune complex reaction, idiosyncratic
response to NSAID, direct effect of certain drugs or diagnostic agents on mast cells

Acute onset of an illness (minutes to several hours) with simultaneous involvement of


the skin, mucosa! tissue, or both AND at least one of the following:
• Respiratory compromise
• Reduced BP or associated symptoms of end-organ dysfunction (e.g., hypotonia,
syncope, incontinence)
• Severe gastrointestinal symptoms, especially after exposure to non-food allergens

Acute onset ofhypotension• or bronchospasmb or laryngeal involvement' after


exposure to a known or highly probable allergend for that patient (minutes to several
hours), even in the absence of typical skin involvement
• Hypotension:decreasein SBP>30%frombaselineor SBP<90mmHg
• Excludinglowerrespiratorysymptoms triggeredby commoninhalantallergensor foodallergensperceived to
cause"inhalational"
reactionin theabsenceof ingestion
'Strider,vocalchanges,odynophagia
'Allergen:substance (usuallya protein)capableof triggering
animmuneresponse thatcanresultin anallergic
reaction(mostactthroughan lgE-mediated pathway, butsomenon-allergen
triggerscanactindependent of
lgElikedirectmastcellactivation)

B. Other Laboratory Findings


• Acute emphysema and lung hyperinflation on chest radiography
• Eosinophilia
• ECG abnormalities reflecting a primary cardiovascular event mediated by mast cells
(which are prominent near the coronary vessels) or secondary to blood volume reduction
• Elevation of serum tryptase
• Elevation of serum histamine

IV. MANAGEMENT
A. Acute Management ofAnaphylaxis (WAO)
Have a written emergency protocol for recognition and treatment of anaphylaxis and
rehearse it regularly
Remove exposure to trigger
Assess the patient: airway, breathing, circulation, mental status, skin, and body weight
Call for help: resuscitation team (hospital) or EMS (community)'
Epinephrine':
• Inject intramuscularly (IM) at the mid-anterolateral aspect of the thigh
• Dose: 0.01 mg/kg of a 1:t000 (1 mg/mL) solution (maximum of 0.5 mg in adults or 0.3 mg
in children per single dose)
• RMecordti_meof dose adndrepedatevery 5-t5 minutes, as needed
• ost patients respon to 1-2 oses
Place patient on the back or in a position of comfort if there is respiratory distress and/
or vomiting; elevate the lower extremities (standing or sitting suddenly may be fatal)'
When indicated, give high-flow supplemental oxygen (6-8 L/min) by face mask or
oropharyngeal airway
Establish intravenous access (wide-bore; gauge 14-16) & consider giving 1-2 liters of plain
saline rapidly (e.g., 5-IOmL/kg in the first 5-10 minutes to an adult; IO mL/kg to a child)
Perform cardiopulmonary resuscitation with continuous chest compression, if indicated
At frequent, regular intervals, monitor the patient's blood pressure, cardiac rate and
function, respiratory status, and oxygenation (continuously, if possible)
'Thesestepsshouldbedonepromptlyandsimultaneously
Sourcefor bothtables:CardonaV,Ansotegu1
IJ, Eb1sawa
M,et al. WAOJournal.2020
537
B. Specific Management
ASPECT I MANAGEMENT
• Epinephrine 0.3-0.5 mL ofI:IOOOsolution (1 mg/mL) IM (repeat q5-15
First-line
min for severe reactions)
drug
• Dose is 0.01 mg/kg (maximum of 0.5 mg)
• Diphenhydramine 25·IO0 mg IM/IV for urticaria/angioedema
Ancillary • Ranitidine 50 mg IV
agents • Salbutamol nebulization
• Hydrocortisone 200 mg IV
For injected • Application of tourniquet proximal to the site
material & • Epinephrine 0.2 mL of1:IOOOsolution IM
insect stings • Removal without compression of insect stinger (if present)

Intractable • IV infusion of 2.5 mL of 1:10,000 epinephrine at 5· to IO·min intervals


hypotension • Volume expanders and vasopressors
• Fluids (1-2Lor 5-IOmL/kg of 0.9% isotonic saline in the first 5·!0
minutes) and pressors, as needed
• Oxygen supplementation and intubation as needed
Supportive , IV glucocorticoids may alleviate later recurrence ofbronchospasm,
hypotension or urticaria
0 Methylprednisolone 0.5·1.o mg/kg
0 Hydrocortisone 200 mg

C. Post-Discharge and Anticipatory Management


0Epinephrine auto-injector (or prefilled syringe)
0Medical identification (e.g., bracelet or wallet card)
0Anaphylaxis emergency action plan; education
° Confirm anaphylaxis trigger:
• Allergen skin tests at follow-up visit to allergologist (e.g.,3·4 weeks after acute episode)
• Allergen-specific serum IgE levels
• Avoidance and/or immunomodulation (e.g., avoidance of known triggers,
desensitization)
0Optimal management of concomitant conditions

Sources:CardonaV,AnsoteguiIJ, EbisawaM,et al. WAOJournal.2020


JamesonJL,et al. Harrison'sPrinciplesof InternalMedicine,20thedition.2018

REFERENCES
I. ARIA in the pharmacy:management of allergic rhinitis symptoms in the pharmacy.Allergic rhinitis and its impact on asthma.
Allergy 2004;59,373-387
2. Bousquet J,VanCauwenbcrge P, Khahaev N. Allergic rhinitis and its impact on asthma (ARIA) - executive summary. Allergy
2002;57,841-855
3. Bousquet J,KhaltaevN, Cruz AA, Denburg J,FokkensWJ,Togias A, et al. Allergic Rhinitis and its Impact on Asthma (ARIA)
2008 Update. Allergy 2008; 63,8-160
4. BrozekJL,Bousquet J,Agache I, Agarwal A, BachertC, Bosnic-AnticevichS, et al. Allergic Rhinitis and its Impact on Asthma
(ARJA)guidelines - 2016revision,JAllergyClin Immunol. 2017.
5. HanifinJM,RajkaG. Diagnostic featuresof atopic dermatitis.Acta Denn VenerealSuppl. 1980;92:44-47.
6.Jameson JL.KasperDL,Longo DL,FauciAS, Hauser SL,LoscalzoJ.Harrison'sPrinciplesoflmemal Medicine.20th Edition.New
York:McGrawHill Education,2018.
7. Klimek L. Bergmann KC, Biedennann T, Bousquet J, Hellings P, Jung K, et al. Visual analogue scales (VAS):measuring
instruments for the documentation or symptoms and therapy monitoring in cases or allergic rhinitis in everyday health care:
position paper of the German Society of Allergology (AeDA) and the Gennan Society of Allergy and Clinical Immunology
(DGAKI),ENT Section, in collaboration with the working group on Clinical Immunology, Allergology and Environmental
Medicine of the German Society ofO1orhinolaryngology, Head and Neck Surgery (DGHNOKHC). Allergo J Im 20li;26,i6-24
8. MarshallJS,WarringtonR, Watson W, Kim L An introductionto immunology and immunopathology,2018.14(Suppl2)49.
9.Simons FER, Ardusso LR, Bilo MB, El-Gama! YM, Ledford DK, Ring J,e1 al. World Allergy Organization Guidelines for the
Assessment and Managementor Anaphylaxis.WorldAllergyOrganizationJournal.2011;4:13-37
10. Todd I, Spicken G. LectureNotes in Immunology 6th edition. USA:Wiley Blacb.-.•ell;
2010.
538
SECTION ONE
APPROACH TO COMMON HEMATOLOGIC COMPLAINTS

• Weakness
.
APPROACH TO HEMATOLOGIC COMPLAINTS
- SIGNS I HEMATOLOGIC FINDINGS

• Pallor
• Fatigue • Anemia
• Jaundice
• Exertional dyspnea

• Recurrent infection • Hepatomegaly • Leukopenia


• Fever of unknown • Splenomegaly • Leukocytosis
origin • Lymph node enlargement • Abnormal WBC morphology

• Petechiae' • Thrombocytopenia
• Bleeding • Ecchymosisb • Abnormal prothrombin time
• Easy bruising • Hemarthrosis' (PT)+/- activated partial
• Mucosal bleeding thromboplastin time (aPTT)
• Petechiae (<3 mm diameter):tiny hemorrhageswithindermal/submucosallayers (due to rupturedvessels)
• Ecchymosis(>3 mm diameter):reddish-purplediscoloration(due to extravasationof bloodfromtrauma)
progressingto a bluish-greenishtinge,eventuallyturningyellowishover time
' Hemarthrosis:extravasationof bloodintoa joint

BLOOD COMPONENTS
• •• DESCRIPTION
Red blood
• Contain hemoglobin
cells (RBC) or
• Transport oxygen to tissues
erythrocytes

White blood • Integral pan of the immune system


cells (WBC) or • RE:sponsible for removing senescent and aberrant cells and attacking
leukocytes infectious and foreign substances
Neutrophils or Poly-
• First responders to infection (especially bacterial infections)
morphonuclear
• Predominant WBC during first few days of birth & after 4-5 years old
cells(PMN)
• Three major types
0 Natural killer [NK] cells (cytotoxic innate immunity)
Lymphocytes 0 T cells (cell-mediated adaptive immunity)
0 B cells (humoral adaptive immunity)

• Migrate from the bloodstream to other tissues and differentiate


Monocytes into resident macrophages
• Possess phagocytic potential

• Active in parasitic infections & modulate allergic responses


Eosinophils
• Involved in cytokine production forother processes (e.g.,wound healing)

• Important effector cells in asthma & other allergic disorders


Basophils
• Also express positive and negative immunoregulatory functions

• Contribute to hemostasis
Platelets or
• Store certain molecules involved in hemostasis, inflammation, innate
thrombocytes
inununity. cell proliferation, vascular tone, fibrinolysis, & wound healing
541
DIAGNOSTICS IN HEMATOLOGY
I. THE COMPLETE BLOOD COUNT
Tests for amount of blood cells & relative percentage of each cell type, among others
Normal values for the different parameters vary by age, gender, and laboratory
(seeChapter 2)
Interpreted in context with other parameters and with the patient's clinical picture

A. Red Blood Cell Parameters

PARAMETER I DEFINITION

• Number ofRBCs • Blood loss • Polycytbemia vera


Red blood
per mm' of blood • Iron deficiency • Prolonged stay in high-
cell (RBC)
• Refers to the actual • Hemolysis altitude areas
count
number of red cells • Bone marrow failure • Smoking, hypoxia

• RBC protein • Anemia (WHO): • Erythrocytosis:


Hemoglobin
essential in oxygen 0 <130 g/L in males 0 >l70 g/L in males
(Hgb)
transport 0 <120 g/L in females 0 >150g/L in females
• Dehydration
• Fractional volume • Anemia
Hematocrit • Erythrocytosis:
of blood occupied • Excessive IV fluid 0 >0.50 in males
(Hct)
byRBCs infusion 0 >0.45 in females

• Index by which
• Blood loss
RBC production is • Hypoproliferative
• Hemolysis
Reticulocyte assessed anemias
• -Erythrocyte membrane
count • Reticulocytes: newly • RBC maturation
disorders
released anucleate disorders
red cells
• Macrocytic anemias
Mean cell (e.g., myelodysplastic
• Measures average
volume • Microcytic anemias syndrome, hemolysis,
RBCsize
(MCV) hemorrhage, folate or
vitamin B12 deficiency)

• Measures average • Folate and vitamin B12


Mean cell
Hgb amount per RBC • Iron deficiency deficiency
hemoglobin
• Increases/decreases • Thalassemia • Chemotherapy-
(MCH)
in parallel with MCV induced

• Measures average
Hgb weight per unit
Mean cell • Hereditary
RBCvolume
hemoglobin spherocytosis
concentration
• Used for quality
• Autoimmune
---
control purposes
(MCHC) hemolytic anemia
(e.g., detecting
sample turbidity)

• Estimate of variance
Red cell in volume within • Iron deficiency anemia
distribution the RBC population ·-- • Various inflammatory
width(RDW) • Surrogate for systemic conditions
inflammation

• Immature RBCs • Hypoxia


• Usually reported • Severe hemolytic
Nucleated as number of anemia
red cells nucleated RBCs per
--- • Primary myelofibrosis
100WBCs • Marrow infiltration/
• Present in newborns myelophthisis
"
542
B. White Blood Cell Parameters
0White blood cell (WBC) count: number ofWBCs per uL of blood
0Differential count gives the percentage of each WBC type & reveals abnormal WBC populations
0 Differential count is also used to generate absolute value for each cell type (more
meaningful as compared to relative percentage)

COMPONENT

• Neutrophilia: increase in ANC to >2 SD


above normal population mean
• Inflammation (e.g.,bacterial or fungal)
• Neutropenia: ANC<2 SD below • Malignancy (especially if metastatic)
mean of normal population or • Drugs: glucocorticoids, hematopoietic
<I,500 cells/uL growth factors
• Accelerated neutrophil • Leukemoid reaction: neutrophilia with
Neutrophils
destruction (e.g., autoimmune >50,000 cells/uL
neutropenia) 0Inflammatory (pancreatitis),
• Decreased production (e.g., infectious or neoplastic (lung CA)
marrow failure, drug-induced) ° Composed of mature neutrophils
with low proportion of bands &
myelocytes (in contrast to leukemia,
where cells are more immature)
• Lymphopenia: ALC <1,000 cells/uL
(reflects depletion ofT-cells)
• Lymphocytosis: ALC >4,000 cells/uL
• Aplastic anemia
• Primary or clonal lymphocytosis
• Infections (AIDS, hepatitis, HSY,
(leukemia, lymphoma)
Lymphocytes TB, typhoid)
• Reactive lymphocytosis (pertussis,
• iatrogenic (e.g.,immunosuppressive)
mononucleosis, heart failure,
• Systemic (SLE, cancer, pancreatitis)
malignancy, major surgery)
• Nutritional (alcohol, zinc deficiency)
• Congenital immunodeficiency
• Monocytosis: AMC >800 cells/uL
• Monocytopenia: AMC <400 cells/uL • Neoplasms, drug-induced
• Aplastic anemia • Connective tissue disorders (SLE, RA)
Monocytes
• Leukemia/lymphoma • Infections (TB, subacute bacterial
• Glucocorticoids endocarditis, dengue, syphilis)
• Alcoholic liver disease
• Eosinophilia: AEC >500 cells/uL
• Bacterial infections • Hypereosinophilia: AEC >I,500cells/uL
• Cushing syndrome • Infections (e.g., parasitic, fungal)
Eosinophils
• Glucocorticoid use • Allergic disease, drug reactions
• Burns • Neoplasms (leukemia, lymphoma)
• Systemic diseases (RA, IBO)
• Inflammation
• Glucocorticoid use • Allergic disease
Basophils • Hyperthyroidism • Infection (varicella, influenza, TB)
• Ovulation • Endocrinopathy (DM, myxedema)
• Neoplasms (leukemia, carcinoma)
• Includes metamyelocytes, myelocytes, promyelocytes, myeloblasts (these
cells are not normally seen in the CBC)
Immature • Blasts: earliest precursors in the evolution ofWBCs (a differential count of
white blood
>20% is a clue for acute leukemia)
cells
• Presence of any of these cells warrants a hematologic evaluation (differentials
include severe sepsis, primary myelofibrosis, marrow infiltration, leukemia)
ANG:absoluteneutrophil
count ALC:absolutelymphocyte
count
AMC:absolutemonocytecount AEC:absoluteeosinophil
count

543
C. Platelet Count (PC)
0 Number of platelets per uL of blood
0 Primary function is hemostasis, thrombosis, & wound heaJjng through a complex process
THROMBOCYTOPENIA I THROMBOCYTOSIS
PC <150,000/uL PC >450,000/uL

• Impaired platelet production: aplastic • Reactive thrombocytosis (blood loss,


anemia, MDS, leukemia inflammation)
• Increased platelet destruction: ITP,TIP, DIC • Iron deficiency
• Abnormal platelet distribution: • Post-splenectomy
hypersplenism • Malignancies
• Drug-induced
Sources:JamesonJL, et al. Harrison'sPrinciplesof InternalMedicine20thedition,2018.
KaushanskyK, et al. WilliamsHematology,9thedition.McGraw-Hill Education;2016
TefferiA,et al. Howto InterpretandPursueandAbnormalCompleteBloodCountin Adults.MayoClinProc.2005

II. OTHER LABORATORY WORK-UP


A. Coagulation Tests
0 Used to assess efficiency of clotting function, evaluate bleeding, & adjust anticoagulation
DIAGNOSTIC I REMARKS
• Evaluates deficiencies or inhibitors of extrinsic and common
Prothrombin time (PT) coagulation pathways
• Note that PT activity has no proven clinical utility
Partial thromboplastin • Evaluates deficiencies or inhibitors of intrinsic and common
time(PTT) coagulation pathways
International • Standardizes PT to allow monitoring for those on oral vitamin
Normalized Ratio (INR) K antagonists

• Evaluates deficiencies or dysfunction of fibrinogen or presence


Thrombin time (TT)
of an inhibitor to thrombin
• Terminal product offibrinolysis used in excluding diagnosis
D-dimer of venous thromboembolism and assessing presence of
disseminated intravascular coagulation (DIC)
Fibrinogen • Evaluates DIC and other various bleeding conditions

• Used to determine if prolonged PT or PTT is due to factor


Mixing studies
deficiency or an inhibitor

• Used to monitor response to therapy with low molecular


Anti-Xa levels weight heparins (LMWH) and other newer anticoagulants (or
when PTT cannot be reliably used for UFH)

B. Tests for Iron Stores


0 Used to evaluate for (absolute and functional) iron deficiency
DIAGNOSTIC
I REMARKS

Ferritin • Useful indicator of body iron stores


Serum iron • Measures iron bound to transferrin

Total iron binding • Measures all proteins available for binding mobile iron
capacity (TIBC) (transferrin accounts for majority)
Transferrin saturation
(TSAT) • Parameter to assess iron status (reference range: 20-50%)

544
III. BONE MARROW ASPIRATION (BMA) AND BIOPSY
Used in diagnosis & staging of hematologic disorders & assessment of marrow cellularity
May also play a role in assessment of fever of unknown origin (FUO) and in the
diagnosis of storage and infiltrative disorders

A. Indications and Contraindications

USUAL INDICATIONS ICONTRAINDICATIONS*


ABSOLUTE

• Unexplained cytopenia/pancytopenia or cytoses • Severe hemophilia


• Lymphoma & other solid tumors (diagnosis & staging) • Severe DIC
• Plasma cell disorders and leukemias • Other related severe
• Evaluation of iron metabolism & stores (when routine bleeding disorders
testing is inadequate)
• Suspected storage & deposition disorders (e.g.,
amyloidosis)
• Fever of unknown origin
• Suspected fungal, mycobacterial, or parasitic
infections; or granulomatous diseases
• Unexplained splenomegaly
*Thrombocytopenia,
regardless
of severity,is NOTa contraindication!

B. Some Conditions and Usual BMA Findings


CONDITION I USUAL BMA FINDINGS
• Erythroid hyperplasia with predominance of early erythroblasts
Megaloblastic
with cells displaying nuclear-cytoplasmic dyssynchrony &
anemia
dysplasia
• Markedly hypocellular marrow with numerous empty/fat-filled
Aplastic anemia
spaces
Pure red cell • Cellular to hypocellular marrow with few/absent erythroid
aplasia precursors
Myelophthisic
• Presence of islands or clusters of non-hematopoietic cells
marrow
Myelodysplastic • Cellular to hypercellular marrow with erythroid hyperplasia and
syndrome (MDS) dysplastic changes in various cell lines
• Cellular to hypercellular marrow with medium/large-sized
Acute myeloid
mononuclear cells with abundant granular basophilic cytoplasm,
leukemia (AML)
fine chromatin, & prominent nucleoli
• Cellular to hypercellular marrow with presence of small-sized
Acute
mononuclear cells with round to slightly indented nuclei, scanty
lymphoblastic
basophilic cytoplasm, and fine to slightly coarse and clumped
leukemia (ALL)
chromatin (inconspicuous nucleoli)
Chronic myeloid • Markedly hypercellular marrow with granulocytic hyperplasia
leukemia (CML) with cells in various stages of maturation
Chronic • Cellular to hypercellular marrow with presence of small
lymphocytic mononuclear cells with scanty deeply basophilic cytoplasm and
leukemia (CLL) coarse hyperclumped chromatin
• Cellular marrow with presence of variably sized cells with
Multiple eccentrically located nuclei, perinuclear clearing, abundant
myeloma (MM) basophilic cytoplasm and coarse clumped chromatin (some cells
multinucleated)
545
IV. SPECIAL TESTS IN HEMATOLOGY
DIAGNOSTIC
I REMARKS
I EXAMPLES

Karyotyping/ • Detects numerical & large structural • Translocation 9;22 in


conventional chromosomal aberrations in sampled CML
cytogenetics tissue • Deletion sq in MOS
• Used to detect, identify, & count specific
cells in a sample
• Uses monoclonal antibodies treated
with a fluorochrome to detect presence/
absence of specific cellular components
• Leukemia panel
Flow seen in various conditions (e.g.,
• Lymphoma panel
cytometry hematologic malignancies)
• Some uses in hematology:
0 Immunophenotyping in malignancies
l , HLAtyping
0 Quantification of stem cells for
transplant use
Fluorescence • Utilizes hybridization of fluoresce in- • FISH for MOS
in-situ labeled DNA probes to detect specific • FISH for AML/ALL
hybridization chromosomal abnormalities which may • FISH for myelo-
(FISH) not be detected in karyotyping proliferative neoplasms
Polymerase
• Used to examine target gene expression • BCR-ABL1
mutation in CML
chain reaction
in various hematologic conditions • PML-RARAmutationinAPL
(PCR)

Next-generation • Uses technologies to sequence large segments of genome or the entire


sequencing genome in high throughput settings to detect genetic aberrations in a
(NGS) faster and more reliable manner

COMMON COMPUTATIONS AND FORMULAS IN HEMATOLOGY


I. RED BLOOD CELL INDICES
INDEX I FORMULA I NORMAL VALUE I INTERPRETATION

Mean Corpuscular
Volume MCV= ----
Hct x
RBC
JO
• N: 80-100fl • Micro-/Normo-/
Macrocytic

Mean Corpuscular Hgb • Hypo-/Normo-/


MCH= • N:27-31pg
Hemoglobin RBC Hyperchromic

Mean Corpuscular • Mostly used for


Hemoglobin MCHC= • N:330-390g/L quality control
Concentration Hct purposes

II. MENTZER INDEX


Values <13signify that microcytic hypochromic
MCV anemia is most likely from thalassemia
Mentzer index = XIOO
RBC Values >13signify that microcytic hypochromic
anemia is most likely from iron deficiency

III. TRANSFERRINSATURATION(TSAT)
The serum iron and TIBC are used to compute for the transferrin saturation (TSA T)
which is a useful parameter to assess iron status (reference range: 20-50%)
Serum Iron TSAT <20%: suggestive of iron deficiency
TSAT = ------- x 100
TIBC TSAT >50%: suggestive of iron overload
546
IV. RETICULOCYTE COUNT AND INDEX
Reticulocytes are immature red blood cells that reflect the erythropoietic function of
bone marrow and are used to distinguish various etiologies of anemia
Since reticulocytes are released prematurely by the bone marrow, we calculate for the
reticulocyte production index or reticulocyte index to adjust for the degree of anemia
and maturation time of reticulocytes
STEPS I FORMULA

Step 1: Compute for absolute Hct


reticulocyte count (ARC) ARC= Retie count x IOOO x Expected Hct for age and gender

Maturation Time (MT) Hematocrit of Patient

Step 2: Choose maturation time 1.0 45%


(most clinicians choose 2.0 as the 1.5 35%
MT regardless of the patient's
hematocrit for simplicity) 2.0 25%
2.5 15%

Step 3: Compute for ARC


Rl%=---
reticulocyte index (RI) MT
Interpretation:
• If RI<2.5:thinkof hypoproliferative
anemiasor maturation
disorders
• If RI.:2.5:thinkof hemolytic
anemia(hemolysis)or hemorrhage

Sample Case
A 34/F with warm autoimmune hemolytic anemia presents with pallor, jaundice,
Hgb 73 g/L, Hct 25%, retie count ofo.045. Compute for ARC & RI.

ARC = 0.045 x IOOO x (25/45) = 25.0


MT =2.0
RI =ARC/ MT= 25.0 / 2.0 = 12.5%

The RI is >2.5, compatible with autoimmune hemolytic anemia causing the drop in Hgb

V. ABSOLUTE NEUTROPHIL COUNT (ANC) & LYMPHOCYTE COUNT (ALC)


Evaluation of neutropenia or lymphopenia is best done using the ANC or ALC
(respectively) rather than laboratory-designated reference ranges for percentage of
neutrophils or lymphocytes (also true for the other WBC subtypes)
Usually computed in immunocompromised individuals

• Neutropenia: absolute neutrophil count <2 SD


below mean of normal population
ANC = WBC x (PMNs + stabs) x IOOO 0 Mild: 1,000-1,500cells/uL

0 Moderate: 500-1,000 cells/uL


ANG:absoluteneutrophil
count
0 Severe: <500 cells/uL
WBC:whitebloodcellcount
PMNs:neutrophils/segmenters • Risk of infections is related to severity of
neutropenia
• Antibiotic prophylaxis is usually given if ANC <500

ALC = WBC x (lymphocytes) x 1000 • Interpretation:


0 ALC <1,000/uL: lymphopenia (typically reflects

ALC:absolutelymphocyte
count depletion ofT-cells)
WBC:whitebloodcellcount 0 ALC >4,000/uL: lymphocytosis

547
Sample Case
A 21/F with aplastic anemia presents with pallor, recurrent fever, and petechiae on her
lower extremities. Her latest CBC results showed: Hgb 55 g/L, Hct 18.3%, RBC of 2.0
x10"/L, WBC of 1.4 x10'/L (neutrophils 37%, lymphocytes 51%, monocytes 12%),platelet
count of 30,000/uL and reticulocyte count of 0.005.
1. Calculating RBC indices 2. Calculating reticulocyte count & index

MCV= 18·3 x 10 = 91.5fL = normocytic red cells


2.0
ARC= 0.005 x l000 x --18.3
45
=2.03

2.03
MCH= - 5-5 - = 27.5pg= norrnochromic red cells RI= --=1.015
2.0 2.0

3. Calculating ANC and ALC


ANC = 1.4x (0.37) x 1000=518
ALC= 1.4 x (0.51) x 1000=714
Thispatientpresentswithfindingstypicallyseenin aplasticanemia.Hersymptomspointto clinical
manifestationsof bone marrowfailure(anemia,neutropenia, and thrombocytopenia).Anemiain
aplasticanemiais usuallynormocytic
andnormochromic. Reticulocytopenia
isanotherclinicalhallmark
of aplasticanemiaas seenin thiscase(notethatthe RI is <1.5%).Shealsopresentswithmoderate
neutropenia andlymphopenia whicharealsoseenin aplasticanemia.

FINDINGS IN PERIPHERAL BLOOD SMEAR (PBS)


I. VARIATIONS IN RBC SIZE (ANISTOCYTOSIS)
SIZE I CHARACTERISTICS AND CLINICAL CORRELATES

Normal • -7.5 um in diameter (similar in size to the nucleus of a small lymphocyte


erythrocyte which is used as a size reference when assessing blood smears)
size • MCV is a more sensitive measure of red cell volume than red cell diameter

• Mnemonic: TAILS
0 T: Thalassemia

0 A: Anemia of chronic disease


Microcytic
0 I: Iron deficiency anemia

0 L: Lead poisoning

0 S: Sideroblastic anemia
• Seen in megaloblastic anemia, chronic liver disease, chronic
Macrocytic
alcoholism, reticulocytosis, myelodysplastic syndrome

II. VARIATIONS IN RBC COLOR


COLOR I CHARACTERISTICS AND CLINICAL CORRELATES

• Concentration of hemoglobin in the RBC is within normal range


Normochromic • Reflected by the MCH
(RBC) anemia • Seen in aplastic anemia, hemolytic anemia, post-hemorrhagic
anemia, anemia of chronic disease
Hypochromic • RBCs are paler than normal
(RBC) anemia • Most common causes are iron deficiency and thalassemia
• Reflects RBCs which are slightly larger than normal and grayish blue
in Wright-Giemsa stain (represents reticulocytes)
Polychromasia • Color represents residual amounts of ribosomal RNA
• Index ofRBC production: polychromasia signifies increase in
production ofRBCs

548
III. VARIATION IN RBC SHAPE (POIKILOCYTOSIS)
SHAPE I CHARACTERISTICS I CLINICAL CORRELATES
• Disc with a rim of hemoglobin & clear central area (central pallor) which
Normal shape
normally occupies less than one-half of the cell diameter
• Renal/liver disease, malnutrition
• Spiculated RBCs with short,
Burr cells • Stomach carcinoma
equally spaced projections over
(echinocytes) • Bleeding peptic ulcers
the entire surface
• Common in vitroartifact after storage

• Irregularly spiculated RBCs with • Alcoholic liver disease


Spur cells
projections of varying length & • Abetalipoproteinemia
(acanthocytes)
position • Post-splenectomy & malabsorptive states

• Myelofibrosis
Teardrop cells • RBCs with a single elongated or
• Myelophthisislmarrow infiltration
(dacryocytes) pointed extremity
• Thalassemia

Fragmented • Broken, fragmented RBCs (often • Microangiopathic hemolytic anemia


cells half-disc shaped with 2-3 pointed • Heart-valve hemolysis
(schistocytes) extremities) • Carcinomatosis, severe burns

• Thalassemia
• Hemoglobinopathies
Target cells • Bell-shaped RBCs that assume a
• Iron deficiency
(codocytes) target shape on dried blood films
• Post-splenectomy
• Obstructive liver disease

• Hereditary elliptocytosis & ovalocytosis


• Thalassemia
Elliptocytes
• Oval to elongated ellipsoid RBCs • Iron deficiency
(ovaiocytes)
• Myelophthisic anemias
• Megaloblastic anemias
• Hereditary spherocytosis (increased
osmotic fragility)
• Sphere-shaped RBCs with dense
Spherocytes • Immune hemolyticanemia
hemoglobin content
• Posttransfusion
• Water dilution & fragmentation hemolysis

Sickle cells • Crescent-shaped& fragilemembranes • Sickle cell anemia


(drepanocytes) • Contain polymerized hemoglobin S • Other hemoglobinopathies

Mouth cells • Bowl-shaped RBCs with a single • Hereditary stomatocytosis


(stomatocytes) concavity (slit-like central pallor) • Alcoholism, cirrhosis

IV. COMMON ERYTHROCYTE INCLUSIONS


INCLUSION I CHARACTERISTICS I CORRELATES
• Small, fine to coarse, dark blue granules • Lead poisoning
Basophilic
• Represents aggregated ribosomes and • Megaloblastic anemia
stippling
degenerating mitochondria and siderosomes • Thalassemia

• Small, round, uniform black inclusions


• Arise from nuclear fragmentation or
Howell-Jolly • Megaloblastic anemia
incomplete expulsion of the RBC nucleus
bodies • Hyposplenic states
• Pitted from reticulocytes during their transit
through the spleen

• Irregular, refractile, peripheral granules • Thalassemia


• Composed of denatured proteins (primarily • Unstable Hgb syndromes
Heinz bodies
hemoglobin) that form in red cells as a result • Erythrocyte enzyme
of chemical insult disorders
MW raw-HillEducation;2UL1
~ource:Kaushansk K,et al. WilliamsHematology, 1uth ed1t1on.
549
COMMON ANTIPLATELETS, ANTICOAGULANTS AND FIBRINOLYTICS
I. THE COAGULATION CASCADE & SITE OF ACTION OF COMMONLY USED DRUGS

t Antithrombin Factor Xa

r- - - ---- -------- - - - ---- --- -


: Antip1atelets :
: 1 Aspirin :
[ 2 Clopidogrel, prasugrel, ticagrelor l
1 3 GP llb/llla inhibitors{e.g., tirofiban) :
'
: •
'
j
Fibrin
: Ant1coagul~nts :
!4 Fondapannux
L:·····
:
I s LMWH, heparin :
: 6 Blvalirudin _________________ : Thrombus

II. OVERVIEW OF SOME DRUGS USED


DRUG I MECHANISM OF ACTION I SIDE EFFECTS
• Irreversibly acetylazes platelet • GI complaints (dyspepsia to
Aspirin
cyclooxygenase (COX) bleeding and perforation)
• Inhibits ADP-induced platelet • GI complaints
Thienopyridines aggregation by irreversiblyblocking P2Y12 • Most serious: hematologic
(Clopidogrel, • Prasugrel: more rapid onset of action (neutropenia, thrombocytopenia)
Prasugrel) and more predictable inhibition • Prasugrel is contraindicated in
compared to clopidogrel patients with prior stroke or TIA
Non- • Reversibly binds P2Y12receptor on the • Major side effect is bleeding
thienopyridine platelet surface, which prevents ADP- • Doses of aspirin >IOO mg reduces
(Ticagrelor) mediated platelet aggregation effectiveness of ticagrelor
• Bleeding (most common)
Unfractionated • Acts by activating antithrombin
• Thrombocytopenia,
heparin (UFH) • Monitored via aPTT levels
osteoporosis & elevated LFTs
• Similar mechanism as UFH but has
Low molecular • Bleeding (lower with LMWH)
better bioavailability, longer half-life
weight heparin • Risk for thrombocytopenia &
and more predictable response
(LMWH) osteoporosis lower with LMWH
• No need for aPTT monitoring
• Binds antithrombin & accelerates its
Fondaparinux • Major effect is bleeding
inhibition of factor Xa
• Oral anticoagulant which interferes • Most common hematologic
with synthesis ofVitamin-K dependent side effect is bleeding
Warfarin
clotting factors (11,VII, IX, X) • Most common non-hematologic
• Monitored via PT/INR levels side effect is alopecia
• New generation anticoagulants which
Direct oral • Major side effect is bleeding
act by direct thrombin inhibition
anticoagulants (lower than warfarin)
(dabigatran) or factor Xa inhibition
(DOACs) • No need for PT/INR monitoring
(rivaroxaban and apixaban)
• Degrade thrombi by converting
• Bleeding
plasminogen to plasmin indirectly
Fibrinolytics • Rare allergic reactions and
(streptokinase) or directly (alteplase,
transient hypotension
tenecteplase, reteplase)
550
APPROACH TO ANEMIA
I. PHYSIOLOGIC CLASSIFICATION OF ANEMIA
1. Hypoproliferative anemia: marrow production defects
2. Ineffective erythropoiesis: red cell maturation defects
3. Decreased red cell survival: blood loss/hemolysis

II APPROACH TO CLASSIFYING ANEMIA


Patient with anemia
I
!
Review RSCindicesand I
reticulocvte count I
I Reticulocyte index <2.5 Reticulocyte index ~2.5

1
Normocytic,
1 1 Hemolysis/
Microcytic Macrocytic
normochromic hemorrha•e
I I I I
Hypoproliferative
Maturation disorders
anemia Bloodloss
lntravascular
Primary
hemolysis
hematologic
Metabolicdefects
diseases(aplasia,
Folate deficiency (e.g.,RBCenzyme
infiltration,
lron deficiency Vitamin B12 defect)
fibrosis)
Thalassemia deficiency RBCmembrane
Inflammation
Sideroblastic Drugeffect abnormalities
Metabolicdefects
anemia Myelodysplastic Hemoglobinopathy
(e.g.,thyroid
syndrome Immune
diseases)
destruction
Iron deficiency
RBCfragmentation
Kidney disease

MAJOR CLASSIFICATIONS OF ANEMIA

I. MICROCYTIC ANEMIA
Group of etiologies of anemia morphologically characterized by circulating RBCs that
are smaller (low MCV) and have decreased red color (low MCH)

I INFLAMMATION ITHALASSEMIA I SIDEROBLASTIC


PARAMETER

RBC morphology/
I IRON
DEFICIENCY

Micro/hypo(may Normalto
ANEMIA

Micro/hypowith
Variable
smear findings benormal) micro/hypo targetcells

Serum Iron (SI) Low Low Normalto high Normalto high

Total Iron Binding Highto Lowto


Normal Normal
Capacity (TIBC) high-normal low-normal

Transferrin
<10% 10-20% 30-80% 30-80%
saturation(TSAT)

Ferritin <15g/L 30-200 g/L 50-300 g/L 50-300 g/L

Hemoglobin Abnormal(may
pattern on Normal Normal be normalin Normal
electrophoresis a-thalassemia)

Source:JamesonJL,et al. Harrison'sPrinciplesof InternalMedicine20thedition,2018.


KaushanskyK, et al. WilliamsHematology,10thedition.McGraw-Hill Education;2021
551
II. HYPOPROLIFERATIVEANEMIA
Group of anemias characterized by inadequate marrow production of erythrocytes
• Hallmark is a low reticulocyte count

A. Differential Diagnoses ofHypoproliferative Anemias

PARAMETER IDEFICIENCY
IRON IINFLAMMATION I RENAL
DISEASE
I
HYPO-
METABOLIC
STATES
Severity of Mildto severe Mild Mildto severe Mild
Anemia
Normocytic
Norma-
Morphology (microcyticif Normocytic Normocylic
microcytic
prolonged)
Serum iron <30ug/dL <50 ug/dL Normal Normal
Highto Lowto Normal
TIBC high-normal low-normal
Normal

Transferrin Lowto
saturation Low Normal Normal
low-normal
(TSAT)
Serum <15g/L 30-200 g/L 115-150g/L Normal
ferritin
Iron stores Absent Lowto normal Lowto normal Normal
Source:JamesonJL,et al. Harrison's
Principles
of InternalMedicine20thedition,2018
Kaushansky
K, et al. WilliamsHematology,10thedition.McGraw-HillEducation; 2021

• Most common form of anemia


• Oral iron suffices for most
• Causes include:
• IV iron can be given to those
0 Increased iron demand
Iron- who cannot tolerate oral iron
(growth, pregnancy)
deficiency • Blood transfusion reserved for
0 Increased loss (bleeding,
anemia symptomatic and/or unstable
menstruation, phlebotomy)
patients and those with
0 Decreased intake or
continued/excessive blood loss
absorption
• 2nd most common form of • Central to management is
anemia treatment of underlying
• Caused by inadequate iron condition
delivery to the marrow despite • Blood transfusion for
Anemia of
normal or increased stores symptomatic and/or unstable
inflammation
(functional iron deficiency) patients or for those with
• Distinguished from iron hemoglobin <7-8 g/dL
deficiency by high ferritin • Erythropoietin may be
levels beneficial
'Not all typesof anemiarequireironsupplementation. Onlythosewithdocumented irondeficiency
or thosewithdiseasemechanisms involvingirondeficiencyshouldbe givenironreplacement.Iron
overloadhasseriousconsequences (e.g.,cardiacandendocrinedysfunction).

552
III. MEGALOBLASTIC ANEMIA
A. Etiopathogenesis
0 Group of disorders characterized by distinctive morphologic appearances of
developing red cells due to defects in DNA synthesis
0 Anemia is due to ineffective erythropoiesis and hemolysis
0 Usually due to vitamin B12deficiency or folic acid deficiency

B. Clinical Manifestations
• All cell lines are affected
• Red cells vary markedly in size and shape; may show basophilic
stippling and nuclear remnants (Howell-Jolly bodies and Cabot rings)
Peripheral 0 Slight macrocytosis may be the earliest sign
blood 0 Increase in red cell distribution width (ROW) may be the earliest
observable change in red cell indices
• Neutrophils often are hypersegmented (more than the usual 3-5 lobes)
• Platelets often reduced and slightly smaller in size
• Marrow is usually cellular and shows megaloblastic changes especially
Bone
in the erythroid lineage
marrow
• Macrophage iron is increased
• Increased plasma bilirubin, iron, and ferritin
Other
• Increased LOH: due to rapid bone marrow erythroid turnover
changes
• Elevated erythropoietin

Vitamin B12 (Cobalamin) Deficiency


Major Causes • Low plasma or serum • Lifelong cobalamin
• Malabsorption cobalamin (in most cases) injections (woo mcg
(pernicious anemia) • Low plasma or serum hydroxycobalamin IM
• Inadequate dietary intake holotranscobalamin daily for I week, once
(vegans) • Elevated urine weekly for 4 weeks,
and plasma/serum then monthly)
Minor Causes methylmalonic acid • Oral supplementation
• Total/partial gastrectomy (1,000-2,000 ug/day)
• Ilea! resection may be as effective
• Abnormalities of
cobalamin metabolism
Folic Acid Deficiency

• Dietary deficiencies • Low serum or plasma • Oral doses of1-5 mg


• Malabsorption folate folic acid daily for
• Excess utilization/ 1-4months or until
loss (pregnancy, co- complete hematologic
morbidities) recovery
• Use of antifolate drugs • Cobalamin deficiency
must be excluded/
treated first before
large doses of folic acid
are given

553
IV. HEMOGLOBINOPATHIES/THALASSEMIA SYNDROMES
A. Etiopathogenesis
0 Inherited disorders of alpha- or beta-globin synthesis leading to diminished
production of hemoglobin tetramers and peripheral blood findings ofhypochromia
and microcytosis
0 Unbalanced chain accumulation dominates the clinical phenotype
• Severity of disease greatly varies (depends on degree of globin synthesis
impairment, altered synthesis of other globin chains & coinheritance of other
abnormal globin alleles)
• Beta thalassemias are usually more severe than alpha thalassemias (unlike
beta globin chains, excess alpha globin chains precipitate in red cell precursors
leading to ineffective erythropoiesis)

• Homozygous or compound heterozygous


state for beta-thalassemia
Beta-
• Presents in early childhood (e.g., failure to
Thalassemia
thrive, skull bossing, maxillary overgrowth) • Regular
Major
• Hepatosplenomegaly transfusions &
• Hypermetabolic state chelation
Beta- • Presents with anemia later in life
Thalassemia • May still present with growth/
Intermedia developmental retardation
Beta- • Heterozygous state for beta-thalassemia
• Transfusions as
Thalassemia • Usually diagnosed incidentally (e.g.,
necessary
Minor abnormal CBC results)
• Four-gene deletion alpha-thalassemia • Prenatal
Hemoglobin • Hydrops fetalis syndrome detection &
Barts • Associated with toxemia of pregnancy and exchange
delivery difficulties due to massive placenta transfusion
• Three-gene deletion alpha-thalassemia • Regular
Hemoglobin H • Variable clinical findings transfusions &
• Lifelong anemia with variable splenomegaly chelation
• One- to two-gene deletion alpha-
Alpha-
thalassemia • Transfusions as
Thalassemia
• Variable clinical findings but milder necessary
Trait
compared to hemoglobin H
Source:JamesonJL, et al. Harrison'sPrinciplesof InternalMedicine20thedition,2018
KaushanskyK, et al. WilliamsHematology, 10thedition.McGraw-HillEducation;2021

554
V. HEMOLYTIC ANEMIA
A. Etiopathogenesis
• Anemia due to increased destruction of RBCs leading to decreased red cell survival
• Destruction of red cells is termed hemolysis
1.General Classification
CLASSIFICATION I EXAMPLES

Based on • Inherited , Sickle cell disease


inheritance • Acquired • Autoimmune hemolytic anemia
• ABO incompatibility
Based on site • Intravascular • Paroxysmal nocturnal hemoglobinuria (PNH)
ofred cell • G6PD deficiency
destruction
• Extravascular • Warm autoimmune hemolytic anemia
Based on • Intrinsic • Membrane defects
origin ofRBC
damage • Extrinsic • Immune vs non-immune

2. Common Hereditary Hemolytic Anemias


DISEASE
I FEATURES I MANAGEMENT
• Autosomal dominant inheritance
Hereditary • Deficiency of spectrin &Jar ankyrin
• Splenectomy is the
• Increased MCHC on PBS
Spherocytosis definitive treatment
• Diagnosed by osmotic fragility test
& eosin-5'-maleimide binding test
• X-linked inheritance
Glucose-6- • Avoidance of exposure to
• Usual triggers include fava
phosphate triggering agents
beans, infection and drugs
dehydrogenase • No specific treatment for
• Hemighosts, bite cells and Heinz
(G6PD) most cases; blood transfusion
bodies on PBS
deficiency for emergency cases
• Diagnosed by G6PD assays

• Warm type autoimmune hemolytic


anemia (wAIHA):involves IgG,
• Steroids and steroid-sparing
reacts at body temperature,
agents (e.g.,azathioprine)
primarily extravascular
Autoimmune • Rituximab and splenectomy
• Cold type (i.e.,cold agglutinin
hemolysis also useful
disease):involveslgM, reacts at cold
• Some with spontaneous
temperature, primarily intravascular
resolution
• Diagnosed by direct antiglobulin test
(i.e.,direct Coombs)
• Clinical triad • Hematopoietic stem cell
Hemolytic anemia
0 transplant (HSCT): definitive
• Venous thrombosis treatment
Paroxysmal
• Deficient hematopoiesis/ • Washed pRBC transfusions
Nocturnal
pancytopenia • Steroids
Hemoglobinuria
• Due to complementhypersusceptibility • Anticoagulation for thrombosis
(PNH)
from CD55 & CD59 deficiency • Eculizumab: anti-C5
• Diagnosedby flowcytometry(FIAER) (complement component)
• Coombs-negative hemolytic anemia antibody

B. Manifestations
0 Presents with variable degrees of fatigue, pallor and jaundice
• Splenomegaly may be seen in those with extravascular hemolysis
555
SECTION THREE
BLEEDING
APPROACH TO BLEEDING
I. GENERAL APPROACH TO BLEEDING
APPROACH I REMARKS
• Bleeding after hemostatic challenge (e.g., dental procedures, surgery)
Obtain a detailed • Systemic diseases
• Family history of genetic bleeding disorder
history & physical
• Use of antiplatelets, anticoagulants, or supplements that affect hemostasis
exam
• lntraarticular or intramuscular bleeding: suggests hemarthrosis,
retroperitoneal hemorrhage, hematoma (especially in absence of trauma)

Multiple positive • Excessive vaginal bleeding or postpartum hemorrhage


responses to • Brisk or prolonged bleeding after epistaxis
• Exaggerated bleeding after minor trauma
questions that
• Spontaneous bleeding, poor wound healing, surgical wound dehiscence
relate to excessive • Severe bleeding requiring blood transfusion support
bleeding/bruising • Unusual rashes or easy bruising
Basic screening • CBC with platelet count
tests for patients • Peripheral blood smear
with hemorrhagic • Prothrombin time (PT) & partial thromboplastin time (PTT)
complications • Platelet function assay (PFA) or bleeding time

II. USUAL ETIOLOGIES FOR BLEEDING


ACQUIRED I INHERITED

• Thrombocytopenic disorders • Deficiencies of coagulation factors:


• Liver & kidney disease hemophilia, von Willebrand disease
• Vitamin K deficiency • Qualitative platelet defects: Glanzmann
• Hematologic: leukemia, myelodysplasia, etc. chrombasthenia, Bernard-Soulier
• Acquired inhibitors of coagulation factors • Fibrinolytic disorders
• Disseminated intravascular coagulation • Vascular disorders: hemorrhagic
• Drugs: anti platelets, anticoagulants, hepatotoxic telangiectasias
& nephrocoxic agents, corticosteroids • Connective tissue disorders: Ehlers-
• Vascular disorders, senile purpura, Danlos syndrome
vitamin C deficiency

III. MANIFESTATIONS ASSOCIATED WITH SPECIFIC HEMOSTATIC DISORDERS


MANIFESTATIONS I BLEEDING DISORDER
Mucocutaneous bleeding • Thrombocytopenias
(e.g., epistaxis, petechiae, • Platelet dysfunction
gastrointestinal bleeding) • von Willebrand disease
• Severe hemophilia NB
Hemarthroses, hematuria,
• Severe deficiencies of factor VII, X, or XIII
intramuscular, intracerebral, &
• Severe type 3 von Willebrand disease
retroperitoneal hemorrhages
• Afibrinogenemia

• Mild-to-moderate hemophilia A/B


Injury-related bleeding and
• Severe factor XI deficiency
mild spontaneous bleeding
• Moderate deficiencies offibrinogen & factors II, V, VII, or X

Bleeding from umbilical cord • Dysfibrinogenemia, hypofibrinogenemia, afibrinogenemia


stump & habitual abortions • Factor XIII deficiency
Impaired wound healing • Factor XIII deficiency
Recurrent severe epistaxis &
chronic iron deficiency anemia • Hereditary hemorrhagic telangiectasias

556
THROMBOCYTOPENIA
Most common cause of thrombocytopenia is drug-induced
Most common non-iatrogenic cause is infection
May present with mucocutaneous bleeding (e.g., epistaxis, petechiae)

ETIOPATHOGENESIS I FEATURES I MANAGEMENT


Immune Thrombocytopenia (ITP)

• Immune-mediated • Mucocutaneous • Corticosteroids (mainstay of


platelet destruction bleeding treatment)
• Termed secondary • Low platelet count • Splenectomy (second-line)
if associated with but normal peripheral • High-dose !Vig
other conditions (e.g., blood cells and smear • Immunosuppressants (e.g.,
autoimmune disorders, rituximab)
infections) • Thrombopoietin receptor
agonists (e.g., eltrombopag)

Thronibotic Thrombocytopenic Purpura (TTP)

• Inherited and • Classic pentad (FAT RN) • Plasma exchange (mainstay of


idiopathic cases not often seen treatment)
due to deficiency 0 Eever • Corticosteroids
of ADAMTS13 that 0 Microangiopathic • Immunomodulatory therapies
normally cleaves van Hemolytic Anemia (rituximab, vincristine,
Willebrand factor 0 Ihrombocytopenia cyclophosphamide)
(vWF) 0 Renal failure • Splenectomy
0 Neurologic decline
.
Hemolytic Uremic Syndrome (HUS)

• Predominantly seen in • Triad • Primarily supportive


children Microangiopathic
0 • Some patients may require
• Most frequently hemolytic anemia short-term dialysis
caused by£. coli Thrombocytopenia
0

0157:H7 Renal failure


0

• ADAMTS13 levels are


generally normal
. -
Heparin-Induced Thr,om_~ocytopenia (HIT)
• Occurs after exposure • Thrombocytopenia not • Prompt discontinuation of
to UFH (more usually severe (rarely heparin
common) or LMWH <20,000/L) • Direct thrombin inhibitors
• Usually develops after • Not associated with • Non-heparin-based
exposure to heparin bleeding (in fact, anticoagulation
for 5-14 days (may be increases risk of
earlier if with previous thrombosis)
exposure to heparin)
Source:JamesonJL, et al. Harrison'sPrinciplesof InternalMedicine20thedition,2018
KaushanskyK, et al. WilliamsHematology,10thedition.McGraw-HillEducation;2021 :

557
COAGULOPATHIES AND RELATED DISORDERS
Coagulopathyis broadlydefinedas any derangementof hemostasisresultingin either excessivebleedingor
clotting,although usually is used in the contextof impairedclotformation (i.e.,bleeding).

I. HEMOPHILIA
A. Etiopathogenesis
0 Hemophilia is a congenital disorder of secondary hemostasis
• Hemophilia A: deficiency in factor VIII
• Hemophilia B: deficiency in factor IX
0 Results from genetic mutations in F8 & F9 genes in the long arm of the X chromosome
0 This leads to reduced thrombin generation on the surface of activated platelets and
sites of injury, and thus, bleeding

SEVERITY I ACTIVITY
FACTOR I MANIFESTATIONS

• Presents later in childhood or during teenage or adult years with


Mild 5-40%
intercurrent injury or surgery (rather than spontaneous bleeding)

• Bleeding associated with intercurrent injury/invasive procedure


Moderate 1-5% • Deep tissue, muscle, or joint bleeding
• Mucocutaneous bleeding
• Usually present in newborns with intracranial bleeding or bleeding
Severe <1% from puncture sites
• Bleeding in joint or muscle related to activity or injury

C. Diagnosis
0 PT and PTT (PTT is almost always abnormal)

0 Specific factor assays are then used to identify the deficient factor

D. Management (mainstay is replacement of deficient coagulation factor)


0 Recombinant and plasma-derived products
° Fresh frozen plasma
° Cryoprecipitate (for hemophilia A)
° Cryosupernate (for hemophilia B)

II. DISSEMINATED INTRAVASCULAR COAGULATION (DIC)


• Consumption coagulopathy is a systemic process with potential for both bleeding & thrombosis
• State of abnormally activated coagulation & fibrinolysis within the vasculature

I ACUTE DIC I CHRONIC DIC

• Trauma • Malignancy: pancreatic, gastric,


Common • Sepsis ovarian, brain
etiologies • Malignancy
• ABO incompatibility

• Bleeding • Venous/arterial thromboembolism


• Thrombocytopenia • Mild or no thrombocytopenia
• Prolonged PT and PTT • Normal or mildly prolonged PT/
• Low plasma librinogen PTT
Manifestations
• Elevated D-dimer • Normal or slightly elevated
&diagnosis
• Microangiopathic changes librinogen
(thrombocytopenia with large • Elevated cl-dimer
platelets, fragmented red cells, • Microangiopathic changes
microspherocytes)

• Early identification of DIC and addressing its underlying cause are the most
important pillars for DIC management
Management
• Supportive management: hemodynamic and/or ventilatory support,
hydration, transfusion as needed
558
III. RELATED DISORDERS ASSOCIATED WITH COAGULOPATHY
ETIOPATHOGENESIS I FEATURES I MANAGEMENT

Coagulopathy of Liver Disease


• Individuals with • Prolonged PT/INR, • Intervention in the setting of
liver disease have PTT asymptomatic laboratory changes
hemostatic changes •Mild is usually not warranted
which increase risk thrombocytopenia • Cases that require further
ofboih bleeding and • Elevated D-dimer investigation or intervention:
thrombosis • To differentiate from 0 Large or unexpected changes
• State of rebalanced DIC, liver disease (e.g., newly prolonged PT/PTT,
hemostasis since has the following: new decline in platelet counts)
decreased hepatic 0 Increased/normal 0 Severe thrombocytopenia
function leads to factor VIII levels ° Complications (e.g., portal vein
both procoagulant 0 Normal or slightly thrombosis, DIC, HIT)
and anticoagulant high D-dimer 0 Bleeding and/or need for an
effects (old notion of invasive procedure
patient being "auto- 0 Vitamin Kif with deficiency
anticoagulated" is no • Avoid medications that increase
longer applicable) bleeding and/or thrombotic risk
Von Willebrand Disease (VWD)
• Most common • Clinical features • Cornerstone of management
congenital bleeding depend on subtype is replacement ofvWF with
disorder • Most present only recombinant or plasma-derived
• Caused by quantitative with mild bleeding concentrates or increasing its
deficiency or • Typical features: release (from endogenous stores
qualitative defect of excessive bleeding using desmopressin)
von Willebrand factor post-injury/ • Antifibrinolytics are useful
(vWF) surgery, epistaxis, adjuncts
menorrhagia,
hematuria/
hematochezia, easy
bruisability
Source:JamesonJL, et al. Harrison'sPrinciplesof InternalMedicine20thedition,2018
KaushanskyK, et al. WilliamsHematology, 10thedition.McGraw-Hill Education;2021

559
SECTION FOUR
BONE MARROW FAILURE AND MALIGNANCIES

BONE MARROW FAILURE


ETIOPATHOGENESIS I FEATURES I MANAGEMENT
Aplastic Anemia ,(A11\)
• Hematopoietic • Seronegative hepatitis • Allogenic hematopoietic stem
stem cell disorder is the most common cell transplant (Allo-HSCT): best
characterized preceding infection treatment for young patients
by bone marrow • Diagnosed by a with a fully compatible sibling
hypocellularity combination of donor
and decreased pancytopenia with a • lmmunosuppressive therapy
hematopoiesis fatty (hypocellular) (1ST): antithymocyte globulin
• Usually due to bone marrow (ATG) + cyclosporine
T-cell-mediated • Bleeding is the • Supportive care: blood
autoimmune most common early transfusions, androgens (e.g.,
destruction ofCD34- symptom danazol), vigilance and proactive
positive stem cells use of prophylactic antibiotics
• Can be inherited or
acquired
• Biphasic peak (teens
and older adults)

Myelodysplastic Sy~drome (MDS) '


• Heterogenous • Cytopenia associated • Only stem cell transplantation is
group of clonal, with a dysmorphic curative
acquired disorders & usually cellular , Hematopoietic growth factors
characterized bone marrow, and (e.g., erythropoietin)
by ineffective by consequent • Hypomethylating agents (e.g.,
hematopoiesis ineffective blood cell decitabine, azacitidine)
resulting in production • Immunomodulators:
cytopenias and • Anemia dominates lenalidomide (for those with
variable risk of the early course isolated 5q-deletion)
progression to AML • 1ST similar with a plastic anemia
• Usually seen in older in those with hypocellular bone
individuals marrow

Marrow lnfiltr.atio~kMyelophthisis)
• Marrow infiltration • Presents with • Goal of management is treatment
of metastatic cancer teardrop cells and of underlying disease
(usually portends leukoerythroblastic • Transfusion as necessary
a poor prognosis) picture on PBS
& any other non- (presence of
hematopoietic nucleated RBCs and
conditions (e.g., immature myeloid
granulomatous cells)
disease, fibrosis) • Diagnosed using bone
marrow biopsy

560
HEMATOLOGIC MALIGNANCIES
ETIOPATHOGENESIS I FEATURES I MANAGEMENT
Acute Myeloid Leukemia (AML)
• Clonal, malignant disease • Presents with symptoms of • Goal is to quickly induce
of hematopoietic tissues bone marrow failure clinical remission
characterized by • Most important prognostic • Divided into two phases:
accumulation of abnormal factors: age & cytogenetic/ 0 Induction (cytarabine +

(leukemic) myeloid blast cells, molecular features anrhracycline)


principally in the marrow, • Auer rods usually point to ° Consolidation (allogeneic
& impaired production of myeloid lineage hematopoietic stem
normal blood cells cell transplantation or
• Predisposing factors: high-dose cytarabine
hereditary, radiation, chemotherapy)
benzene exposure, drugs • All-trans retinoic acid
(e.g., chemotherapy) (ATRA): effective for APL
• Acute promyelocytic (APL is the most curable
leukemia (APL): specific formofAML)
subrype characterized by • Targeted therapy for
balanced translocation of specific mutations:
PML gene in chromosome ° FLT3 inhibitors
t5 with RARA gene in 0 !DH inhibitors

chromosome t7 & presents 0 Hedgehog inhibitors

with distinct coagulopathy 0 Bcl-2 inhibitors

Acute Lymphoblastic Leukemia (ALL)


• Accumulation of abnormal • Hematologic malignancies • Treatment typically longer
leukemic lymphoid blast oflymphoid precursor cells thanAML
cells • Presents with symptoms of • Four aspects of treatment:
• More common in childhood bone marrow failure induction, intensification
but can occur at any age • Important prognostic (or consolidation),
• Broadly divided into B cell factors: age, leukocyte CNS prophylaxis and
and T cell lineages (and count on presentation, maintenance
rarely, NK cell) immunophenotype, • Multi-agent chemotherapy
genorype & minimal residual in rapid succession is key to
disease after induction achieve disease control
-
Chronic Myeloid Leukemia (CML)
• Usually found in older • Mild to moderate • First-line therapy:
individuals splenomegaly: most tyrosine kinase inhibitors
• Caused by reciprocal common PE finding (e.g., imatinib, nilotinib,
translocation of dasatinib)
chromosomes 9 & 22 • Chemotherapy and/or
forming a BCR-ABL hematopoietic stem cell
fusion gene (Philadelphia transplantation for advanced
chromosome) disease (e.g., blast phase)

I
Chronic Lymphocytic Leukemia (CLL)
• CLL is an indolent • Characterized by peripheral • Require treatment if
malignancy of mature B lymphocytosis and may symptomatic or with rapid
cells present with cytopenias, progression or marrow
constitutional symptoms failure
and hepatomegaly/ • Chemoimmunotherapy
splenomegaly/ • Targeted therapies
lymphadenopathy
• Variable clinical course
561
ETIOPATHOGENESIS I FEATURES I MANAGEMENT
Hodgkin I;ymphome (HLJ
• Monoclonal lymphoid • The malignant cell in • Usual frontline
neoplasm usually classic HL (cHL) is the chemotherapy regimen:
derived from B cells Hodgkin Reed-Sternberg ABVD (doxorubicin,
• Most common subtype (HRS) cell, a large bi-lobed bleomycin, vinblastine,
is nodular sclerosing cell with two or more dacarbazine)
nuclei with eosinophilic • Radiotherapy may be
nucleoli (like owl's eyes) used depending on
• Characteristic B symptoms: disease extent
'Fever (Pel-Ebstein fever)
0 Night sweats
0 Weight loss
• Most common
presentation is palpable
lymphadenopathy
• Staging is done using the
Lugano system

• Encompasses clonal • Burkitt's lymphoma: most • Usual frontline option


lymphoid malignancies rapidly progressive human for DLBCL: CHOP
ofB/T/NK cells tumor (cyclophosphamide,
presenting primarily in • Staging is done using the doxorubicin, vincristine,
solid tissue Lugano system prednisone) plus
• Diffuse large B cell rituximab (R-CHOP)
lymphoma (DLBCL):
most common subtype
Multiple My~oma II
• Malignant proliferation • Diagnosis requires • High-dose chemotherapy
of plasma cells marrow plasmacytosis followed by autologous
(>IO%) or biopsy-proven HSCT for eligible patients
plasmacytoma and any • Combination therapy
CRAB feature or myeloma- (usually triplets) using
defining events IMiDs (thalidomide,
' H yper_Galcemia lenalidomide),
'Renal insufficiency proteasome inhibitors
0 Anemia (bortezomib), steroids and
0 B.one lesions alkylating agents
Source:JamesonJL,et al. Harrison'sPrinciplesof InternalMedicine20thedition,2018
KaushanskyK, et al. WilliamsHematology,10thedition.McGraw-Hill Education;2021

REFERENCES
J. Jameson JL,KasperDL, LongoDL, FauciAS, HauserSL,LoscalzoJ.Harrison'sPrinciplesoflntemal Medicine.20th Edition.New
York!McGrawHill Education,2018.
2. Kaushansky LichtmanMA, PrchalJT,LeviMM, Bums LJ,Linch DC (editors).Williams Hem::ttology, IOthedition. Ne,.,;York
McGraw-HillEducation;2021.
3. KitchensCS, KesslerCM, KonkJeBM.ConsultativeHemostasisand Thrombosis,3rdedition. USA:ElsevierSaunders,2013.
4-TefferiA, Hanson CA,and InwardsDJ.How to Interpretand Pursueand AbnormalCompiece BloodCount in Adults.MayoClin
Proc. 2005;80(7),923-936.
5.Zehnder, J.(2020, Jan).Bone marrowaspiratjonand biopsy:indications and technique. Retrievedfrom https://www.uptodate.
com/contents/bone-marrow-aspiracion-and-biopsy-indicarions-and-
technique

562
DERMATOL
APPROACH TO PATIENTS IN DERMATOLOGY
1. Approach to the Patient with a Skin Disorder
2. Morphology of Skin Lesions
3. Common Diagnostic Modalities

0 COMMON CASES IN DERMATOLOGY


1. Acne Vulgaris
2. Contact Dermatitis
3. Psoriasis Vulgaris
4. Hansen Disease (Leprosy)
5. Stevens-Johnson Syndrome & Toxic Epidermal Necrolysis
6. Other Cases in Dermatology
SECTION ONE
APPROACH TO PATIENTS IN DERMATOLOGY

APPROACH TO THE PATIENT WITH A SKIN DISORDER


Dermatology is the branch of medicine concerned with conditions of the skin, hair, & nails
• The skin is the largest organ in the body

I. GENERALAPPROACH
Challenge of examining the skin lies in distinguishing the following:
0 Normal versus abnormal
0 Significant versus trivial findings
0 Integrating pertinent signs & symptoms into an appropriate differential diagnosis
The observer can be misled by a variety of stimuli and overlook important, subtle signs
of skin or systemic disease (e.g., minor differences in color/shape that distinguish a
melanoma from a benign nevomelanocytic nevus)
It is advisable to assess the patient first before taking an extensive history to ensure that
the entire cutaneous surface is evaluated, and objective findings can be integrated with
relevant historical data

II. HISTORYOF PRESENT ILLNESS


Evolution of lesions: site of onset, manner in which the eruption progressed or spread,
duration, and periods of resolution or improvement in chronic eruptions
Symptoms associated with the eruption: itching, burning, pain, numbness
Current or recent medications (prescribed as well as over-the-counter)
Associated systemic symptoms (e.g., malaise, fever, arthralgias)
Ongoing or previous illnesses
History of allergies
Presence of photosensitivity
Review of systems
• Family history (relevant for patients with melanoma, atopy, psoriasis, or acne)
• Social, sexual, or travel history

III. PHYSICALEXAMINATION
Complete skin examination includes evaluation of the skin, hair, nails, and mucous
membranes of the mouth, eyes, nose, nasopharynx, and anogenital region
Disrobe patient as completely as possible to minimize chances of missing important
lesions and permit accurate assessment of the distribution
Palpation of skin lesions can yield insight into the character of an eruption

MORPHOLOGY OF SKIN LESIONS


A variety of descriptive terms have been developed to characterize cutaneous lesions,
towards their interpretation and in the formulation of differential diagnoses

Four Basic Features of a Skin Lesion must be noted during the examination:
• Arrangement of the lesions
• Types of primary and secondary lesions
• Shape of individual lesions
• Distribution of the eruption

I. ARRANGEMENT
Erythematous pa pules & vesicles can occur in many conditions, but their arrangement
in a specific linear array suggests an external etiology (e.g., allergic contact dermatitis or
primary irritant dermatitis)
Lesions with a generalized arrangement are more suggestive of a systemic etiology
565
II. PRIMARY VERSUS SECONDARY SKIN LESIONS
Primary lesions appear as a direct result of a disease process
Secondary lesions may develop from primary lesions or as a result of external trauma
(e.g., infections, scratching)
LESION I DESCRIPTION I USUALLY SEEN IN
Primary Skin Lesio,is
• Flat circumscribed area of skin color • Solar lentigo
change • Idiopathic guttate
Macule
• Non-palpable, can be ill-defined or well- hypomelanosis
defined • Macular exanthem
• Melasma
• Similar to a macule, but 2:1 cm in
Patch • Vitiligo
diameter
• Mongolian spot
• Solid, elevated lesion in which a • Acrochordon
Papule significant portion projects above the • Keloid
plane of the surrounding skin • Lichen planus
• Psoriasis vulgaris
Plaque • Similar to a papule, but 2:1 cm in diameter
• Lichen simplex chronicus
• Solid elevated lesion in which a significant • Lipoma
portion is beneath the skin surface • Nodular basal cell
Nodule
• Usually 2:1 cm (depth of involvement carcinoma
differentiates it from a papule/plaque) • Gumma of tertiary syphilis
• Encapsulated cavity or sac lined by true
Cyst • Epidermoid cyst
epithelium
• Skin swelling that is characteristically • Dermatographism
Wheal
evanescent (disappearing within hours) • Urticaria
• Superficial elevated cavity containing • Dyshidrotic dermatitis
Vesicle
clear, serous, or hemorrhagic fluid • Herpes simplex
Bulla • Bullous pemphigoid
• Similar to a vesicle, but 2:1 cm in diameter
(Bullae) • Bullous drug eruption
Pustule • Similar to a vesicle, but with purulent fluid • Folliculitis
Secondqry Skin Lesjbns
• Dried serum (yellow-brown), blood
(reddish-black), or pus (yellow-green) on • Classic honey-colored
Crust
the skin surface crusts of impetigo
• Discontinuous epithelial cell layer
• Psoriasis vulgaris
• Flakes of stratum corneum
Scale • Pityriasis rosea
• Represents excessive epidermis
• Ichthyosis vulgaris
Excoriation • Surface excavations of the epidermis due to scratching
• Linear loss of skin surface continuity, usually from excessive tension or
Fissure
decreased elasticity
Erosion • Sharply defined, red, moist lesion resulting from partial epidermal loss
Ulceration • Deeper defect, extending to at least the dermis
Scar • Fibrous tissue replacement of a previous ulcer
Atrophy • Diminution of some or all layers of the skin
• Reactive skin thickening with accentuation of markings due to repeated
Lichenification
rubbing or scratching
566
III. SHAPE OR CONFIGURATION OF SKIN LESIONS
Configuration pertains to the shape of single lesions & arrangement of clusters of lesions
• Flat, round, erythematous papules and plaques are common in many cutaneous diseases
SHAPE I DESCRIPTION
Annular • Ring-shaped
Round/nummular • Discoid/coin-shaped
Polycyclic • Coalescing circles or incomplete rings
Arcuate • Arc-shaped
Linear • May imply Koebner phenomenon
Reticular • Net-like, lacy
Serpiginous • Serpentine, snake-like
• Target-like with at least three distinct zones
Targetoid • Target-shaped lesions that consist in part of erythematous plaques
are specific for erythema multiforme
Whorled • Like a marble cake with 2 distinct colors interspersed in a wavy pattern

JV. DISTRIBUTION OF SKIN LESIONS


Patient should first be viewed from a distance of about 4-6 feet so that the general character of
the skin and the distribution oflesions can be evaluated
Ex.A hospitalizedpatient with a generalizederythematousexanthem is more likely to have a
drug eruptionthan a patient with similarlesionslimitedto the sun-exposedareasof theface
SHAPE DESCRIPTION
Dermatomal/
• Occurring along a single spinal afferent nerve root
zosteriform
Blaschkoid • Following lines of skin cell migration during embryogenesis
Lymphangitic,
• Implies an infectious agent spreading centrally from an acral site
sporotrichoid
Sun-exposed • Face, dorsal hands, V of the neck
Sun-protected • In areas covered by layers of clothing
Acral • Wrists, hands, ankles, feet, ears, penis
Truncal • Occurring on trunk or central body
Extensor • Occurring on dorsal extremities, elbows, knees
Flexor • Overlying flexor muscles of extremities, antecubital, popliteal fossae
Intertriginous • In skin folds where 2 surfaces are in contact (e.g., axillae, inguinal)
• Hair-bearing locations: scalp, eyebrows, beard, central chest, axillae,
Seborrheic
genitals, nasolabial and postauricular areas
Follicular • Papules centered around hair follicles
Localized • Occurring on a single body location
Generalized • Widespread
Bilateral • Occurring with mirror image symmetry
symmetric
Universal • Involves the entire cutaneous surface
Source:KangS, et al. Fitzpatrick'sDermatology, 9thEdition.McGraw-Hill
Education;
2019
WolffK, et al. Fitzpatrick's
ColorAtlasandSynopsisof ClinicalDermatology,6thEdition;2013
BologniaJ, et al. Dermatology,
3rdEdition.Elsevier;
2012
567
COMMON DIAGNOSTIC MODALITIES IN SKIN DISORDERS
No special instrumentation usually necessary to examine the skin, although it can be
biopsied with little morbidity
DIAGNOSTIC I DESCRIPTION
I PROCEDURE

• Lesion can be excised or


saucerized with a scalpel or
removed by punch biopsy (after
anesthetizing area)
• Minor surgical procedure done • A punch is pressed against surface
Skin biopsy to a lesion that is most likely to of the skin & rotated with downward
yield diagnostic findings pressure until it penetrates to the
subcutaneous tissue
• Circular biopsy is then lifted with
forceps, and the bottom is cut with
iris scissors
• Performed on scaling skin
• Edge oflesion is scraped gently
lesions where a fungal infection
with a blade
is suspected
• Removed scale is collected on a
• Used to identify hyphae in
KOH microscope slide & treated with
dermatophyte infections,
preparation 10-20% KOH solution
pseudohyphae and budding
• KOH dissolves keratin and allows
yeasts in Candidainfections, and
easier visualization of fungal
"spaghetti and meatballs" yeast
elements
forms in tinea versicolor
• Most often used in the diagnosis
• An early vesicle is unroofed and
of herpesvirus infections
the base of the lesion is scraped
(herpes simplex virus [HSV] or
Tzanck gently with a scalpel blade
varicella zoster virus [VZV])
smear • Material is placed on a glass slide,
• Multinucleated epithelial giant
air-dried, and stained with Giemsa
cells suggest the presence of
or Wright's stain
HSVorVZV
• Designed to assess whether a • Done by pressing a microscope
skin lesion will blanch with slide or magnifying lens against a
pressure lesion and noting the amount of
Diascopy • Urticaria will blanch with blanching that occurs
pressure, whereas a purpuric • Granulomas often have an opaque
lesion caused by necrotizing to transparent, brown-pink "apple
vasculitis will not blanch jelly" appearance on diascopy
• To identify extent of pigmented
or depigmented patches and to • Skin to be examined should not
detect fluorescence have been recently washed
Wood'slight
0 Pale blue: Pseudomonas • Skin is then examined with
0 Yellow:tinea capitis Wood's lamp for a few seconds
0 White: vitiligo
• Suspected allergens are applied to
Patch • Designed to document the back under occlusive dressings
testing sensitivity to a specific antigen and allowed to remain in contact
with the skin for 48 hours
Source:Jamesonet al. Harrison's
Principles
of InternalMedicine,
20thEdition.2018

568
SECTION TWO
COMMON CASES IN DERMATOLOGY

ACNE VULGARIS
I. ETIOPATHOGENESIS
Common chronic skin disease involving blockage and/or inflammation of pilosebaceous
units (e.g., hair follicles & accompanying sebaceous gland)
Occurs most frequently in adolescents and young adults, with onset at puberty and
resolution by the third decade
More common in males in the adolescent period and more common in females in the
post-adolescent period
A. Four Main Pathogenic Factors•:
• Follicular epidermal hyperproliferation
• Excess sebum production
• Inflammation and immune response
• Cutibacterium acnes: gram-positive, anaerobic, normal flora of sebaceous glands
*Lesionsbeginwithmicrocomedone
formationdueto hairfollicleblockage.Cutibacterium
acnes
withinthecomedonescauseinflammation,
resultingin extrusionof oily andkeratinous
debris.

B. Other Suspected Contributory Factors


0Skin trauma
0 Dairy consumption

0Glucocorticoids (topical or systemic)


0High glycemic load diets, insulin resistance
0Stress

II. CLINICAL FEATURES


Clinical hallmark of acne vulgaris is the comedone, which may be closed (whitehead) or
open (blackhead)
Closed comedones appear as 1- to 2-mm pebbly white papules, which are accentuated
when the skin is stretched
Most common location for acne is the face, but involvement of the chest & back is common
Most disease remains mild and does not lead to scarring
Lesions are polymorphous:
0Non-inflammatory lesions: closed comedones (whiteheads), open comedones
(blackheads)
0Inflammatory lesions: erythematous papules, pustules, fluctuant nodules
Common sequelae would include postinflammatory erythema, postinflammatory
hyperpigmentation, atrophic scars (ice pick scars, rolling scars, and boxcar scars),
hypertrophic scars, and keloids

A. Common Acne Variants


VARIANTS I REMARKS
• Common in teenage males
Acne conglobata • Severe form of nodular acne more prominent on the trunk
• Often results in scarring and requires systemic treatment

• Common in teenage males


Acne fulminans
• Severe form of nodular acne on the back (sparing the
(acute febrile
face) but more explosive in onset, with lesions becoming
ulcerative acne)
ulcerative, and associated with systemic findings
Source:KangS, et al. Fitzpatrick's
Dermatology,
9th Edition.McGraw-Hill
Education;
2019
569
B. Common Acneiform Eruptions
ERUPTIONS I REMARKS
• May appear as early as 2 weeks after oral/topical steroids are started
Steroid
• Monomorphous pustules and red papules usually on the trunk,
folliculitis
shoulders and upper arms
• Phenytoin, lithium, isoniazid, high-dose vitamin B complex,
Drug- halogenated compounds (bromides, iodides in cold remedies,
induced acne sedatives, contrast material, diet pills), epidermal growth factor
receptor inhibitors, some vitamin-mineral combinations
• Coal tar derivatives, cutting oils, chlorinated hydrocarbons (chloracne)
• Malar, retroauricular, mandibular regions of the head and neck and
Occupational areas in close contact with clothing saturated with the offending
acne chemical (axillae, scrotum)
• Large comedones, pa pules, pustules, nodules, cysts, hyperpigmentation
• May take up to 2 years to clear
• Seen in patients with acne vulgaris treated with long-term
Gram-
antibiotics especially tetracyclines
negative
• Papulopustules or deep-seated nodules prominent around the nose
folliculitis
• Treat with antibiotic with gram-negative coverage &/or isotretinoin
• Small inflammatory pa pules and pustules, scaling, and dryness
Periorificial
around the mouth sparing the lip margin
dermatitis
• May involve nose, eye, groin regions
Source:KangS, et al. Fitzpatrick's
Dermatology,
9th Edition.McGraw-Hill
Education;2019

III. MANAGEMENT
Normalizing follicular keratinization
Decreasing sebaceous gland activity
Decreasing population of C. acnes
Decreasing inflammation

• Initiate treatment early and aggressively to prevent permanent sequelae


Supportive
• Use of a gentle cleanser twice daily
• Retinoids (e.g., adapalene, tretinoin)
• Benzoyl peroxide (BPO)
Topical • Topical antibiotics (e.g., erythromycin and clindamycin)
agents • Clindamycin
• Salicylic acid
• Azelaic acid (safe in pregnancy)
• Antibiotics (e.g., doxycycline 50-100 mg BID, clindamycin 150-300 mg QID)
• lsotretinoin 0.5-1 mg/kg/day with a total cumulative total dose of
Systemic 120-150 mg/kg± prednisone 40-60 mg OD for 1-2 weeks (take CBC, AST,
therapy ALT, and lipid profile at baseline, week 3-6 and week 6-8)
• In hormonal acne: oral contraceptives, prednisone 2.5-5 mg HS,
spironolactone 50-100 mg BID, flutamide 250 mg BID
• Acne surgery
• Intralesional injection of corticos.teroids
Procedures • Chemical peels
• Phototherapy and lasers
• Subcision, dermabrasion, microneedling, fillers
Source:KangS, et al. Fitzpatrick's
Dermatology,
9th Edition.McGraw-HillEducation;2019
Jamesonet al. Harrison's Principles
of InternalMedicine,20thEdition.2018
570
CONTACT DERMATITIS
Acute or chronic inflammatory reactions (exogenous eczemas) to substances that come
in contact with the skin
There are two main types: irritant and allergic contact dermatitis

I. IRRITANT CONTACT DERMATITIS (!CD)


• Localized, inflammatory skin response to a wide range of chemical or physical agents
Accounts for 80% of all contact dermatitis which is due to direct cytotoxic effect of irritants
Unlike allergic contact dermatitis, is not immune-mediated
Most common occupational skin disease (affects anyone exposed to the irritant substance)
A E.t10pat h ogenes1s
Risk • Infants, women, athletes, wet work, low ambient humidity
factors • Atopic dermatitis, stasis dermatitis

• Soaps, detergents, waterless hand cleaners


Common • Acids and alkalis
causative • Industrial solvents
agents • Plants
• Fiberglass, wool, rough synthetic clothing, fire-retardant fabrics, "NCR" paper

B. Clinical Features
• Erythema occurs a few hours after exposure, often resulting from a single
exposure to an irritant or caustic chemical
• Lesions evolve from vesicle or blister formation, erosion, crusting, shedding
Acute
of crusts and scaling or (in chemical burn) erythema, necrosis, shedding of
!CD
necrotic tissue, ulceration, and then healing
• No papules
• Strictly confined to site of exposure
• From repeated exposures to mild irritants or low concentrations of strong irritants
Chronic
• Dryness eventually leads to chapping, erythema, hyperkeratosis and scaling,
!CD
fissures and crusting, then finally to lichenification

CD' :
I MAJOR CRITERIA
I MINOR CRITERIA

• Symptom onset within minutes to


• Onset within 2 weeks of exposure
hours of exposure: pain, burning,
Subjective • Many people in the area affected
stinging, or discomfort exceeding
similarly
itching early in the clinical course
• Macular erythema, hyperkeratosis,
• Sharp circumspection of dermatitis
or fissuring predominating over
• Evidence of gravitational influence
vesiculation
such as dripping effect, lack of
• Glazed, parched, or scalded
tendency of the dermatitis to spread
Objective appearance of the epidermis
• Morphologic changes suggesting
• Healing process begins promptly
small concentration differences
on withdrawal of exposure to the
or contact time produce large
offending agent
differences in skin damage
• Negative patch testing'
*Patchtestingis oftennecessary to excludeallergiccontactdermatitis.
A seriesof allergensis
usuallyusedfor initialscreening.
..
Source:FowlerJF,et al. Fisher'sContactDermat1t1s.7thed;2019
D. Management
0 Avoid irritants
• Emollients or occlusive dressings (petroleum jelly, ceramide-containing lotions)
0 In severe/chronic cases: potent topical steroids, topical calcineurin inhibitors, phototherapy
571
II. ALLERGIC CONTACT DERMATITIS (ACD)
Classic presentation of a T cell-mediated, delayed-type hypersensitivity response to
exogenous agents
Occurs in sensitized individuals (i.e., prior exposure to the offending agent is necessary)

A. Etiopat h ogenesis
Risk • Occupation (healthcare worker,chemical industry,hairdressers,construction workers)
factors • Stasis dermatitis, venous ulcers

• Nickel (metals, jewelry)


• Neomycin sulfate (creams, ointments)
• Balsam of Peru (topical medications)
• Fragrance mix (fragrances, cosmetics)
• Thimerosal (antiseptics)
Common • Sodium gold thiosulfate (medication)
causative • Formaldehyde (disinfectant, curing agents, plastics)
agents • Quatemium-15 (disinfectant)
• Bacitracin (ointments, powder)
• Cobalt chloride (cement, galvanization, industrial oils, cooling agents, eyeshades)
• Carba mix (rubber, latex)
• Paraphenylenediamine (PPD) (hair dye, textile dye, printer's ink)
• Thiuram (rubber)

8. Clinical Features
0 Localized to the skin areas that come in contact with the allergen
0 Involvement of hands, face, or eyelids (most commonly come in contact with the
environment) occurs most frequently in ACD
0 Dominant symptom of ACD is itch or pruritus

Acute • Erythema occurs 12-72 hours after exposure


• Consist of erythematous, indurated, scaly plaques
ACD • Vesiculation and bullae may be seen in severe cases
• Repeated/continued exposure to allergens results in chronic ACD
Chronic • Skin becomes dry, scaly, and thicker due to acanthosis, hyperkeratosis, edema,
ACD and cellular infiltration in the dermis
• Lichenification and fissuring may develop later

C. Diagnosis
TEST
I REMARKS
• Gold standard (to identify causal allergens)
Patch testing • Indicated for recurrent or persistent dermatitis in whom ACD is suspected
• Standard test contains 35 allergens and I control
Repeat open • Can be used for personal care products that are suspected allergens
application
test or use test • Apply substance to the cubital fossa twice a day for I to 3 weeks

• Based on the distribution of the dermatitis


• Allergens to consider based on topography of skin lesion:
Topographic 0 Face: cosmetics, nail lacquer, gold, topical antibiotics, fragrances, hair dye
approach 0 Umbilicus: nickel
0 Dorsum of feet: rubber-related chemicals, glues, adhesives
Allergen-
specific • Based on trends in dermatitis specific to allergens
approach

D. Management
0 Identify and remove allergens
0 Potent topical steroids, topical calcineurin inhibitors
° For severe cases, systemic steroids for 1-2 weeks
Source:KangS, et al. Fitzpatrick's
Dermatology,
91hEdition.McGraw-Hill
Education; 2019
MaanoCC.Eczemasin UP-PGHSectionof Dermatology. CommonDermatoses in theAmbulatory
Setting,4th Edition
572
PSORIASIS VULGARIS
Chronic inflammatory skin disease that may exhibit a variety of clinical manifestations
• More likely to appear between ages 15-30 years

I. ETIOPATHOGENESIS
• T-cell driven disease
Pathogenesis
• Production of epidermal cells is increased 28x
• Koebner phenomenon or isomorphic response
• Infections (streptococci in guttate psoriasis, HIV in severe exacerbations)
• Hypocalcemia
Triggers
• Pregnancy
• Drugs (e.g., lithium, beta blockers, interferons, antimalarials)
• Alcohol consumption, smoking, obesity, psychogenic stress

II. CLINICAL FEATURES


A. Common Forms
FORMS I REMARKS
Chronic plaque • Classic lesion: well-demarcated red plaque with silvery white scales
type (most over the scalp, extensor aspect of extremities especially elbows and
common) knees, lumbosacral area, buttocks, and genitalia
Guttate
• Presents as the acute onset of numerous small, inflammatory plaques
(eruptive)
• Primary sites: trunk and proximal extremities
psoriasis
Pustular
• Presents as an acute, subacute, or chronic pustular eruption
psoriasis
Erythroderrnic/ • From worsening of chronic plaque type or non-tolerated treatment
pustular • Exhibits cutaneous erythema and scales involving most or all body
psoriasis surface areas
Inverse • Involves the intertriginous areas, including the inguinal, perinea!,
psoriasis genital, intergluteal, axillary, or inframammary regions
Source:FeldmanSR.Uptodate;2021

Koebner • Describes the development of skin disease in sites of skin trauma


phenomenon • Traumatic induction of psoriasis on non-lesional skin, usually 7-14days later
Auspitz sign • Pinpoint bleeding after removal of scale overlying a psoriatic plaque
• Nail pitting, oil spots, onychodystrophy, metabolic syndrome
Other related
• Arthritis
findings
• Geographic tongue (map-like pattern in the tongue)

Ill. DIAGNOSIS
• Psoriasis is a clinical diagnosis
• A biopsy is indicated if the five classic features are not present:
0 Symmetry oflesions

Skin biopsy 0 Extensor distribution

0 Auspitz sign (pinpoint bleeding when scale is gently removed)

0 Sharply demarcated lesions


0 Silvery scale

Characteristic • Microabscess of Munro: collection of neutrophils in stratum comeum


histopathology • Spongiform pustule ofKogoj: collection of neutrophils in stratum spinosum
573
IV. MANAGEMENT
Patient education
All should avoid excessdrying/irritation of their skin to maintain adequate cutaneous hydration
Lifestyle modificaton (i.e., control obesity, infection, smoking, intake of alcohol/drugs)

A. Chronic Plaque Psoriasis


MODALITY I FIRST-LINE I SECOND-LINE
, Emollients
Topical • Glucocorticoids • Salicylic acid, dithranol, tazarotene, tar
• Vitamin D3 analogs
• Methotrexate • Fumaric acid esters, cyclosporine-A,
Systemic • Acitretin hydroxyurea, 6-thioguanine,
• Biologicals mycophenolate, sulfasalazine

• NB UVB
Phototherapy • PUVA, excimer, climatotherapy
• BB UVB
*Mostcasesof localized,plaque-type
psoriasiscanbe managedwithmid-potency
topicalsteroids

B. Other Forms of Psoriasis


• Oral retinoids are the treatment of choice in non-pregnant patients
Erythrodermic/
with pustular psoriasis
pustular
• Others: acitretin, cyclosporine-A, PUVA/NB UVB, methotrexate,
psoriasis
anti-TNF agents, systemic steroids

• No treatment
Guttate
• NB UVB
psoriasis
• Topical vitamin D3 analogs, topical steroids
Source:KangS, et al. Fitzpatrick's
Dermatology,
9th Edition.McGraw-Hill
Education;
2019

HANSEN DISEASE (LEPROSY)


I. ETIOPATHOGENESIS
WHO reported 2.77new cases per 100,000 population globally in 2017with the
Philippines remaining as one of the global priority countries
Adult male to female ratio 3:1
Disease onset common between 10-20 years and 30-60 years

A. Mycobacterium leprae
0 Non-cultivable, gram-positive, obligate intracellular, acid-fast bacilli that requires cool
temperatures for growth (e.g., skin, superficial nerves, anterior eye chamber, testes)
0 Transmission: respiratory (nasal droplet) and hypothetically, skin-to-skin
0 Reservoir: humans and armadillos
0 Incubation: average of 2-5 years and as long as 20 years

B. Risk Factors
0 Genetics, immunodeficiency

0 Poor nutrition
0 Multi bacillary source of infection, close contact or repeated exposure
0 Low socioeconomic status, poor housing and sanitation, lack of education

574
II. CLINICAL FEATURES

• Symmetric,poorly
marginated,
infiltrated
• Sharplydefined
nodules/plaques
annularasymmetric • Ill-definedplaqueswith
ordiffuseinfiltration
Skin lesions macules/plaques with occasionalsharpmargins
• Xanthoma-like or
centralclearing • Fewor manyin number
dermatofibroma
• Elevatedborders
papules
• Leoninefacies
• Eyebrowalopecia
• Nervepalsies
variable
• Sometimesenlarged
• Nervetrunkpalsies • Acral,distal,
Nerve lesions • Abscessesmost
• At timessymmetric symmetric
commonin BT
anesthesia
common
Single/
# oflesions Single Several Many Verymany
few
Anesthetic(absent) Hypoesthetic/anesthetic Hypoesthesia
Sensation
early lesions (latesign)
AFB 0-1+ 3-5+ 4-6 (plusglobi}
Lymphocytes 2 0-1
Macrophage Undifferentiated
Epitheloid Foamychanges
differentiation or foamy
Langerhans
1-3 0
giant cells
Lepromin Strong Weak
Negative
test (+++) (++)
Lymphocyte
Positive 1-10% 1-2%
transformation
TT(tuberculoid):
strongcell-mediated
immunity,stablebutdoesnotdowngrade, mayundergo spontaneous curein3 yrs
BT(borderline
tuberculoid):immunitystrongenoughto containinfection,unstable
BB(borderline):
immunologic midzone
BL(borderline
lepromatous): lowimmunityto containinfectionresultingin destructive
inflammation
LL(lepromatous):
lackof cell-mediated
immunity permitsbacillarydisseminationresultingin multiorgan
involvement
Source:RidleyOS,et al. Classification
of leprosyaccording
to immunity.IntJ LeprOtherMycobactDis.1966

B. WHO Classification
PAUCIBACILLARY I MULTIBACILLARY
(all of the following) (any one of the following)

• 1-5skin lesions • >5skin lesions


• Only I nerve trunk involved • >I nerve trunk involved
•(-)AFB in skin smears • (+) AFB in skin smears
Source:DofitasBL,et al.A PracticalGuideto LeprosyCare:Philippine
LeprosyMission,Inc.,2018
575
• No anesthesia, deformity or • No eye problems or evidence of visual
0 damage impairment
• With eye problems
• With anesthesia
I • Vision 6/60 or better, can count 6 fingers
• No deformity or damage
at6 meters
• With visible deformity or • Severe visual impairment, iridocyclitis,
2 damage lagophthalmos, corneal opacities
Source:WHOExpertCommittee
on Leprosy.WHOTechnicalReportSeries1998

D. Deformities

Facial nerve • Lagophthalmos, mask-face


• Claw hand (of 4th and 5th fingers)
Ulnar nerve
• Most commonly affected nerve trunk: ulnar nerve at the elbow
Radial nerve , Wrist-drop
• Clawing ofindex and middle finger
Median nerve
• Impairs thumb opposition and grasp
Lateral popliteal • Foot-drop
Posterior tibial • Clawing of toes and collapse of arches

2. Secondary Deformities from damage to anesthetic parts of the body & associated findings:
INVOLVED PART I EFFECT

Ocular • Corneal erosions, exposure keratitis, ulcerations


• Mask face, leonine facies, madarosis (loss of eyebrows), saddle
Face
nose, ear lobe deformities
Hands/feet • Ulcers, callosities, absorption of digits, hammer toes
Others • Gynecomastia, orchitis, palatal perforation, acquired ichthyosis
Source:S. J. Yawalker.
Leprosyfor MedicalPractitioners
andParamedical
Workers.8th Ed.2009

III. DIAGNOSIS
A. Clinical Diagnosis (WHO)
0An individual who has not completed a course of treatment & has I or more cardinal signs:
• Hypopigmented or reddish skin lesion/s with definite sensory loss
• Damage to peripheral nerves: nerve thickening, loss of sensation, weakness of
muscles of hands/feet, face
• Positive slit skin smear for AFB
B D"a f
DIAGNOSTIC I REMARKS

• Take from the most active lesion (raised and red, usually the edge)
Slit skin
smears • If none, take from a site that had active lesions or previously positive
smears

• Not required for diagnosis but can support a diagnosis ofleprosy


Skin biopsy
and rule out other diseases
576
IV. MANAGEMENT: RECOMMENDED TREATMENT REGIMEN IN ADULTS
DOSAGE AND
DRUG

Rifampicin
I FREQUENCY

600 mg once a month

300 mg once a month and


Clofazimine 12 months 6 months
50 mg once a day

Dapsone 100 mg once a day


Source: Operational Manual, Global Leprosy Strategy 2016-2020

V. COMPLICATIONS: LEPRA REACTIONS

LEPRA
REACTIONS
I TYPE
I MANIFESTATIONS
I MANAGEMENT*

• Change in delayed type


hypersensitivity to M.
• Best for type 1: glucocorticoids
leprae
(as inflammation subsides,
• Lesions become
Type1 the glucocorticoid dose can be
PB and edematous &
Reversal tapered, but steroid therapy
MB erythematous
Reaction must be continued for at least
• Nerve enlargement and
3-6 months lest recurrence
acute neuritis
supervenes)
• Skin ulceration,
paralysis, anesthesia

• Immune complex
disease
• New crops of painful
• Best for type 2 characterized
Type2 papules developing in
by occasional crops of only a
Erythema a few hours and may
MB few skin papules and absence
Nodosum last months/years
of fever or other organ
Leprosum • Fever and pain
involvement: analgesics
• Neuritis, iritis, orchitis,
lymphadenopathy,
arthritis, proteinuria
PB: paucibacilliary
MB:multibacillary

*General management:
•Continue multidrugtherapy and do nerve function tests every 2 weeks
•Analgesics and bed rest
•For severe reactions (ulceration, nerve trunk involvement,fever): give prednisone &/orclofazimine
Source:NationalleprosyControlProgramManualof Procedures, 2018
DofitasBL,et al.A PracticalGuideto LeprosyCare:Philippine LeprosyMission,Inc.,2018
WorldHealthOrganization.Guidelinesfor the Diagnosis,Treatment,and Preventionof Leprosy:WHO, 2018

577
STEVENS-JOHNSON SYNDROME & TOXIC EPIDERMAL NECROLYSIS
I. ETIOPATHOGENESIS
Epidermal necrolysis (SJS and TEN) is an acute life-threatening mucocutaneous
reaction characterized by blisters and mucosal/epidermal detachment resulting from
full-thickness epidermal necrosis in the absence of substantial dermal inflammation
Most commonly due to drugs (some due co infection or idiopathic)

A. Classification
CLASSIFICATION I EXTENT OF SKIN DETACHMENT

SJS • <IO% body surface area (less severe condition)

SJS/TEN Overlap • 10-30% body surface area

TEN • >30% body surface area

SJS: Stevens-Johnson Syndrome


TEN:ToxicEpidermalNecrolysis
'
B. Commonly Implicated Medications:
0Allopurinol
0Antibacterial sulfonamides (e.g., sulfamethoxazole, sulfadiazine, sulfapyridine,
sulfadoxine, sulfasalazine)
0Antiseizure medications (e.g., carbamazepine, lamotrigine, phenobarbital, phenytoin,
phenylbutazone)
0Nevirapine
0Oxicam NSA!Ds
0Thiacetazone

II. CLINICALFEATURES
• Begins 8 weeks (usually 4-30 days) after exposure to a drug for the 1st time
Prodrome • Fever, headache, rhinitis, cough, malaise, odynophagia, burning or
stinging of eyes, heralding mucous membrane involvement
• Appear first on the trunk, spreading to the neck, face and proximal upper
extremities with distal extremities relatively spared
• Stages:
Begin as morbilliform, target lesion-like, multiple erythematous dusky
0

irregular macules
Cutaneous ° Confluence of individual lesions leads to extensive and diffuse
lesions erythema
Necrotic epidermis detaches from the dermis giving rise to flaccid
0

blisters (sheets of necrotic epidermis resembling wet cigarette paper)


Large denuded areas of red, oozing dermis similar to a second-
0

degree thermal burn progress to a plateau phase (but life-threatening


complications may occur)
• Mucous membrane involvement in 90% & can precede or follow skin
Mucosa! eruption
lesions • May involve mucosa! surfaces: oral, ocular, urogenital, pharyngeal,
tracheal/bronchial, esophageal, and intestinal (rare)
• Nikolsky sign: epidermal dislodgement by lateral pressure on
erythematous zones
Signs
• Asboe-Hansen sign (bulla-spread sign): the extension of the blisters
laterally by slight pressure of the thumb

578
lll. DIAGNOSIS
Clues that this is SJS/TEN: rapid progression, severe pain, constitutional symptoms
Frozen-section skin biopsy may aid in rapid diagnosis: hallmark histologic finding is
keratinocyte necrosis, ranging from partial to full-thickness necrosis of the epidermis

SCORTEN Score for SJS/TEN• (r point for each factor)


FACTOR I INDICATOR FOR MORE SEVERE DISEASE
Age • ?:40 years old
Malignancy • Yes (cancer & hematologic malignancy)
BSA detached • ~10%

Tachycardia • ?:120bpm

Serum urea • >IO mmol/L

Serum glucose • >I4mmol/L


Serum
• <20 mmol/L
bicarbonate
*Patients with more severe disease (detachment >30% BSA)or SCORTEN Score ,!:2should be
referred to an ICU, burn unit, or specialized dermatology units
Source: Guegan S, et al. J Invest Dermatol;2006

IV. MANAGEMENT
• Early recognition and withdrawal of offending drug
• Fluid and electrolyte replacement, early nutritional support, cultures
(blood, skin, urine), optimal environmental temperature (28-30°C)
Supportive
• Daily eye exam and disruption of early synechiae by an ophthalmologist
• Antiseptic mouth rinse
• Admission in intensive care or burn unit

• Corticosteroids (still controversial): dexamethasone IV 1.5mg/kg/day for


Specific 3 days; however, long-term or late systemic glucocorticoid use has been
treatment associated with increased mortality
in the acute • Cyclosporine 3-4 mg/kg/day
phase • Use of intravenous immunoglobulin (!VIG) in SJS/TEN remains
controversial, and more recent data question whether it is beneficial

V. COMPLICATIONS
Most common: sepsis from superimposed bacterial infection (S. aureus, Pseudomonas,
Enterobacteriaceae)
Dehydration
Multiple organ system failure
Late complications of mucosa! membrane involvement

Source: Kang S, et al. Fitzpatrick'sDermatology,9th Edition.McGraw-HillEducation; 2019

I
579
OTHER COMMON INPATIENT CASES
CONDITION I DEFINITION I FEATURES I MANAGEMENT
• Fluid and electrolyte
• Severe, potentially • Erythematous replacement, warm
life-threatening patches that humid environment
Erythroderma/ condition increase in size (30-32°C), wet
Exfoliative characterized by and coalesce dressings on
dermatitis diffuse erythema into generalized weeping lesions,
and scaling erythema with a antihistamines and
involving ~90% BSA shiny appearance low-potency topical
corticosteroids

• Prodrome followed • Immediate


• Severe adverse by a diffuse withdrawal of
Drug-induced multi-organ morbilliform suspected culprit
hypersensitivity drug reaction eruption usually drug; systemic
syndrome (most common: involving the face, glucocorticoids (1.5-2
(formerly known allopurinol) 2-8 weeks after mg/kg/d prednisone
as DRESS) characterized by an starting the drug equivalent) tapered
extensive skin rash and persisting slowly over 8-12
after cessation weeks

• Pinpoint pustules
• Rare, acute eruption
overlying an area
characterized by
of erythema, • Withdrawal of the
the development
Acute generalized classically offending drug
of numerous
exanthematous beginning within (self-limiting disease
non-follicular
pustulosis (AGEP) 24-48 hours of with a favorable
sterile pustules on
drug exposure prognosis)
a background of
(may occur 1-2
edematous erythema
weeks later)
Source:Goldsmith
AL, et al. Fitzpatrick's
Dermatologyin GeneralMedicine,8th Ed1t1on.
2012
Davis,M. Erythrodermain Adults.Upto Date.Mar2021
Jamesonet al. Harrison'sPrinciplesof InternalMedicine,20thEdition.2018

REFERENCES
1. Bolognia JL,JorizzoJJ,Schaffer JY,et al., editors. Dem1atology,3rd Edition. Elsevier;2012.
2. Davis, M. Erythrodenna in Adults. Uptodate. Available online lmps://www.uptodate.com/contents/erythrodenna-in-adults.
Accessed March 2021.
3. Depanment of Health, Philippines. National Leprosy Control Program Manual of Procedures, 2018.AvailableonJine hnps:/Jdoh.gov.
ph/leprosy-control-program Accessed October 2021.
4. Dofitas BL A Practical Guide to Leprosy Care: Philippine Leprosy Mission, Inc., 2018.
5. Feldman SR. Psoriasis:epidemiology, clinical manifestations, and diagnosis. Uptoclate.Availableonline hnps://www.uptodate.com/
contems/psoriasis-epidemiology-dinical-manifestations-and-diagnosis Accessed September 2021.
6. Fowler JF.Zirwas MJ, eds. Fisher's Comae! Denmuitis. 7th ed. Contact Dem1atitis Institute; 2019
7. Fransway AF, Reeder MJ. lnitant contact dennatitis in adults. Uptodate. Available online hups://www.uptodate.com/contems/
initant-con1act-dem1atitis-in-adults.Accessed Sept 19,2021
8.Gollnick H, Cinliffe W, Berson D, et al. Management of acne: a report from a Global AlUance 10 improve outcomes in acne. J Am
Acad Dcm1atol;2003~9
9.Guegan S, Basruji-GarinS, Poszepczynska-GuigneE, et al.Perfonnance of the SCORTEN during the first fivedays of hospitalization
to predict the prognosis of epidcnnal necrolysis. J Invest Dennatol; 2oo6;126:272
10. Kang S, Amagai M, Bruckner AL,et al, editors. Fitzpatrick's Dennatology, 9th Edition. McGraw-Hill Education; 2019.
11. Lawley LP, McCall CO, l.awle)' TJ. Eczema, Psoriasis, Cuianeous Infections. Acne, and Other Common Skin Disorders. In:
Jameson JL. Fauci AS, Kasper DL, Hauser SL, Longo DL, Loscalzo J, eds. Harrison's Principles of Internal Medicine. 20th ed.
McGraw Hill;2018
12. Michelleti RG, Rosenbach M, Winrroub Bu, Shinkai K. Curaneous Drug Reactions. In: Jameson JL. Fauci AS, Kasper DL, Hauser
SL Longo DL Loscalzo J,eds. Hanison's Principles oflmemal Medicine. 20th ed. McGraw Hill; 20!8
13, Ridley OS and Jopling WH. Classification ofleprosy according to immunity. Int J Lepr Other Mycobact Dis. Jul-Sept 1966;34(3):255
14. WolffK, Johnson RA, Saavedra AP and Roh EK Fitzpa1rick'sColor Adas and Synopsis of Clinical Dem1a1ology,6th Edition; 2013.
15. World Health Organization. Operational Manual, Global Leprosy Strategy 2016-2020.Available online hups://apps.who.int/iris/
bitstream/handle/io665'250119/9789290225256-Eng.pdf Accessed October 2021.
16. World Health Organization. Guidelines for the Diagnosis, Treatment, and Prevention of Leprosy:WHO, 2018
17. World Health Organi1.ationExpert Committee on Leprosy.Seventh Report.World Health Organisation Technical Report Series 1998;
18. YanceyKB,L,wley TJ. Approach 10the Patient with a Skin Disorder. In: Jameson JL. Fauci AS, Kasper DL, Hauser SL, Longo DL,
LoscalzoJ,eds. Hanison's Principles oflntemal Medicine. 2oth ed. McGraw Hill;2018
580
NEUROL
SECTION ONE
APPROACH TO PATIENTS IN NEUROL:OGY

DIAGNOSTIC CATECHISM

Ql: Does the patient have a neurologk: problem?


Focalneurologicdeficits Meflingeal
illilation
pressure
Increasedintracranial Seizures

I
If YES:

+
Q2: WHEREIs the leslon?

I
levellze l.ateralize localize
Central(SupratentcrialYS. Rightvs.Left Cerebrum
lrlralemrial) Midline Brainstem
Peripheral Diffuse Cerebellum
SpinalCont
RootDisease
ner,e
Peripheral
NMJ
Musde

I
CU: WHAT Is the lesion?
"IIITAMINc&D"

kf,opathic/latrogenic
~fectious/lnflammal:)ry Neoplastic
Traima Coogeoilal
Au1oimmune Degenerative
Metabolic
I
Consider-.s:
I I I
Onset Course Distribution
Progressive Focal
Non-progressive Multi-focal
-Static DifflJse
-Resoving
_,.
- Remissions
&

I I I

*
Q4: What TESTSshould be done to establish the dlaenc,sls?

QS: What ls the TREATMENT?

• The general approach to patients with a neurologic complaint can be simplified by


answering the following questions:
0 Question 1) Does the patient have a neurologic problem?

0 Question 2) Where is the lesion?


0
QQuestion3) WWhhat is the lhesiond?b d bl' h fi h d. _7
0 uestion 4) at tests s ou 1 e one to esta 1s & con rm t e 1agnos1s.
• It is important to consider the onset, course, and distribution of the patient's neurologic
deficits in order to arrive at a logical diagnosis
583
I. DOES THE PATIENT HAVE A NEUROLOGIC PROBLEM?
• There are many points in the patient's history & physical examination that can indicate
a neurologic problem but these are the basic ones that you should be able to pick up
NEUROLOGIC CLUES
PROBLEM I
• Numbness or weakness on one side of the body
Focal neurologic • Visual field problems like hemianopia
deficits • Trouble speaking or understanding
• Walking, balance, and coordination problems
Increased • Triad: headache, papilledema, vomiting
intracranial • Other signs: deteriorating sensorium, Cushing triad (hypertension,
pressure (ICP) bradycardia, bradypnea), anisocoria
• Nuchal rigidity, Brudzinski and Kernig signs
Meningeal • Most common infectious cause of meningeal irritation: meningitis
irritation • Most common non-infectious cause of meningeal irritation:
subarachnoid hemorrhage
Seizures • Discussedin detail in Section III

II. WHERE IS THE LESION? (Anatomic Diagnosis: Remember the 3Ls)


A. Levelize
° Central or peripheral?
0 If central, is it supratentorial or infratentorial?
• Supratentorial: cortex and subcortical structures
• Infratentorial: brainstem, cerebellum

B. Lateralize
0Right 0 Midline
0Left 0 Diffuse

C. Localize
LOCATION I COMMON MANIFESTATIONS

• Discrete deficits
Cerebrum • Language disorders, cognitive impairment
• Seizures
• Crossed signs: contralateral long tract signs + ipsilateral cranial nerve deficits
• Long tract signs: hemiparesis, hemisensory deficits
• Cranial nerve signs and symptoms:
III, IV,VI Diplopia, gaze deviations
V Decreased facial sensation, weakness of masticatory muscles
Brainstern Weakness of facial muscles, decreased taste sensation
VII
disease (anterior 2/3)
VIII Deafness, dizziness, balance problems
IX,X Dysarthria, dysphagia, decreased taste sensation (posterior 1/3)
XI Weakness of sternocleidomastoid and trapezius muscles
XII Tongue deviation, atrophy, fasciculations

• Incoordination, clumsiness, ataxia


Cerebellum • Tremors on voluntary movements
• Dysdiadochokinesia, hypotonia, nystagmus
584
LOCATION I COMMON MANIFESTATIONS
• Normal neurologic function above the level of the lesion
• Bilateral and often symmetrical deficits
• Presence of sensory level differences
Spinal cord
• Autonomic function disturbances, bowel and bladder problems
• Upper motor neuron lesion signs: extensor toe sign, spasticity,
hyperreflexia, clonus
• Pain described as sharp, stabbing, shooting, or radiating down the
limb (aggravated by maneuvers that stretch the root)
Root disease • Weakness and sensory loss follows a spinal root distribution
(myotomal/dermatomal)
• Decreased or absent reflexes
• Distal sensorimotor deficit (length-dependent pattern except for
porphyric neuropathy)
• Paresthesias or asymmetric weakness (symmetrical in GBS and some
hereditary neuropathies)
Peripheral • Decreased muscle tone and reflexes
nerve • May have fasciculations and atrophy over time
• Autonomic disturbances (e.g., anhidrosis, orthostatic hypotension,
temperature dysregulation)
• Patterns: polyneuropathy, neuronopathy, mononeuropathy, multiple
mononeuropathies, plexopathy
• Purely motor (may involve both cranially & spinally innervated muscles)
• Fluctuating weakness (fatigable weakness is the hallmark of
Neuromuscular
myasthenia gravis)
junction
• Normal muscle size ,ind tone, intact reflexes, no fasciculations, no
sensory deficits
• Proximal and symmetrical weakness (except for inclusion body
myositis and distal muscular dystrophies)
Muscle disease • Absent sensory signs except for pain and tenderness
• Atrophy with normal or mildly decreased muscle tone and reflexes
• Increased muscle enzymes

Yascular • Stroke (e.g., infarct, hemorrhage)


Infectious/
• Meningitis, encephalitis, brain abscess
Inflammatory

Trauma • Epidural or subdural hematoma, cerebral contusion

• Myasthenia gravis, demyelinating diseases (e.g., multiple sclerosis,


Autoimmune
ADEM,GBS)
• Nutritional (e.g., Vitamin B12,B6 deficiency), electrolyte problems,
Metabolic
uremia, hyperglycemia, toxic (e.g.,lead, mercury, arsenic), drug overdose

Idiopathic/
• Iatrogenic: post-procedural complications
Iatrogenic

_N_e_o_p;..l_a_s_ti_c
___ +-•-P-ri_m_a_ry_(_e_.g_._•
_m_e_n_i_n_g-io_m_a_,_g-li_o_m_a_)_,_m_e_t_a_s_ta-t-ic
_________ .:
1-Congenital • Arteriovenous malformation, muscle dystrophies
D.egenerative • Parkinson disease, Alzheimer disease, amyotrophic lateral sclerosis
585
IV. WHAT TESTS SHOULD BE DONE TO ESTABLISH THE DIAGNOSIS?
AN
IMAGING I REMARKS
Cranial MRI • Procedure of choice for imaging the brain & spinal cord
• Wider availability, shorter scanning time, & lower cost
• Shows intracranial bleeds & calcifications better than MRI
• Substitute CT for MRI when:
Cranial CT scan
0 Time is critical (e.g., acute intracranial bleeding)
0 Patient has magnetic metal in the body (e.g., older
pacemaker devices)

B. Lumbar Puncture (LP) for Cerebrospinal Fluid (CSF) Analysis


° For diagnostic procurement ofCSF for analysis and/or therapeutic reduction ofCSF
pressure or administration of spinal antibiotics or anti tumor agents
° Contraindications: increased ICP in the absence of imaging, coagulopathies, and
overlying infection over puncture site

• Electrophysiological monitoring method co record


Electro- electrical activity on the scalp
encephalography
(EEG) • Represents the macroscopic activity of the surface layer of
the brain underneath
Electromyography • Measures muscle response or electrical activity in response
(EMG) to a nerve's stimulation of the muscle
Nerve conduction • Measures how fast an electrical impulse moves through
studies (NCS) nerves
• Electrical potential recorded from the nervous system
Evoked responses following presentation of a stimulus
• Visual, auditory, and somatosensory (YEP, BAER, SSEP)
Transcranial • Non-invasive method of brain stimulation that uses
magnetic stimulation electromagnetic induction using an insulated coil placed
(TMS) over the scalp

APPROACH TO COMMON COMPLAINTS IN NEUROLOGY


l. APPROACH TO HEADACHE
A. History
0 Remember O-L-D-C-A-R-S (onset, location, duration, character/quality, aggravating
factors, relieving factors, other accompanying symptoms)
0 Quality of pain (most important information: is it pulsatile or not?)
0 Severity of pain (better index is the degree to which the pain has incapacitated the
patient - school/work performance, awakens the patient from sleep or prevents sleep)
ETIOLOGY I LOCATION/DESCRIPTION
Migraine • Usually unilateral
Posterior fossa mass • Ipsilateral occipitonuchal region
Paranasal sinuses, teeth, eyes, and
upper cervical vertebrae • Forehead or maxilla or around the eyes

Supratentorial lesion • Frontotemporal area (site of the lesion)


Dissection of!CA cervical portion • Periorbital or supraorbital pain
586
B. Physical Examination
0 Palpation: tender, hardened, elevated temporal artery; inflamed paranasal sinuses or
occipital nerve
° Check for focal neurologic deficits and nuchal rigidity

C. Common Types of Headache

I MIGRAINE I MIGRAINE I TENSION CLUSTER

I
WITHOUT AURA WITH AURA
HEADACHE HEADACHE
(Common Migrame) (C/ass,cMigraine)

• Orbito-
• Frontotemporal
Site • Generalized temporal
• Unilateral or bilateral
• Unilateral
• Mainly adults • Adolescent
Age& • Young to middle-aged adults
• More common and adult
sex • More common in women
in women males (90%)
• Non-
• Throbbing (pulsatile), worse behind • Intense
throbbing
Quality one eye or ear • Non-
• Tightness
• Becomes dull ache and generalized throbbing
• Aching
• Continuous
• Variable • Usually
Diurnal • Upon awakening or later in day intensity nocturnal, 1-2
pattern • Lasts 4-24 hours or longer • For days, hours after
weeks, or falling asleep
months
• Fatigue &
Provoking • Bright light, noise, tension, alcohol • Alcohol in
nervous
factors • Relieved by darkness and sleep some
strain
• Lacrimation
• Stuffed
• Scintillating
• Depression nostrils
Associated • Nausea and lights
• Worry • Rhinorrhea
features vomiting • Visual loss
• Anxiety • Injected
• Scotomas
conjunctiva
• Ptosis
• Oxygen,
sumatriptan,
ergotamine
• Abortive: triptans, ergotamine, • Antianxiety before attack
NSA!Ds drugs • Corticosteroid,
Therapy
• Preventive: propranolol, • Anti- verapamil,
amitriptyline depressants valproate, &
lithium in
recalcitrant
cases
Source:RapperA, et.al.AdamsandVictor'sPrinciplesof Neurology,10thed1t1on

587
II. APPROACH TO SYNCOPE VERSUS SEIZURE
Syncope is a transient loss of consciousness caused by transient global cerebral
hypoperfusion, characterized by rapid onset, short duration, and spontaneous recovery

SEIZURES

• Male > female


• Female > male
• Older age (>54 yrs)
• Younger age (<55yrs) • Younger age (<45yrs)
Demographics & • During exertion or
• Standing, warn1 • Any selling/posture
clinical setting when supine
roon1 1 emotional at onset
• Family history of
upset
sudden death

• Longer duration
(>5 sec)
• Shorter duration • Sudden onset or
• Palpitations
Premonitory (<6 sec) brief aura (deja vu,
• Blurred vision,
symptoms • Palpitations less olfactory, gustatory,
nausea, warmth,
common visual)
diaphoresis,
lightheadedness
• Blue face (cyanosis)
• No pallor
• Pallor • Frothing at the
• Diaphoresis mouth
• Blue, not pale
Observations • Dilated pupils • Tongue biting
• Incontinence can
during event • Slow pulse, low BP • Horizontal eye
occur
• Incontinence may deviation
occur • Elevated pulse & BP
• Incontinence more
likely

• Gradual over
seconds
Transition to • Gradual over
• May be sudden • Often immediate
unconsciousness seconds
with some
arrhythmias
Duration of • Prolonged
• Seconds
unconsciousness (>5 minutes)
Duration of
• 30-60 seconds tonic-
tonic/clonic • Brief (never >15seconds)
clonic (if grand ma!)
movements

• Residual symptoms
common: aching
• Residual symptoms • Residual symptoms
muscles
Residual common uncommon (unless
• Disoriented
symptoms • Prolonged fatigue with prolonged
• Fatigue
common (>90%) unconsciousness)
• Headache
• Slow recovery

Disorientation • Disoriented for


& sleepiness • Oriented (disorientation <5 minutes only) many minutes to
after event hours
Source:JamesonJL,et al. Harrison's
Principles
of InternalMedicine
20thedition,2018
ZipesDP,et al. Braunwald'sHeartDisease.11thedition.Elsevier/Saunders,
2019
588
THE NEUROLOGIC EXAMINATION
I. MENTAL STATUS EXAMINATION
A. Appearance
0Attitude: cooperative, attentive, hostile, evasive, guarded
0 Behavior/psychomotor activity: gait, mannerisms, gestures, tics, hyperactive, agitated, waxy
0 General description: posture, bearing, clothing, grooming
B. Stream of Talk
0 Rate: rapid, slow, pressured
0 Volume: loud, soft, monotonous, histrionic
0 Quality: fluent, neologisms, word salad, lacks inflection and spontaneity
0 Tangential, discursive, or unable to reach the conversational goal
° Check for the 4 D's of speech:

Dysphonia • Difficulty in producing voice sounds (phonating)


Dysarthria • Difficulty in articulating the individual sounds or the units of speech
• Difficulty with melody & rhythm of speech, accent of syllables, inflections,
Dysprosody
intonations, and voice pitch

Dysphasia • Difficulty expressing/understanding words as the symbols of communication

C. Mood and Affective Responses


0Mood: pervasive and sustained emotion that colors the person's perception of the world
0Affect: patient's present emotional responsiveness; broad, restricted, blunted, or flat,
may or may not be congruent with mood

D. Thinking and Perception


ASPECT I DESCRIPTION
Thought • Disorganized, illogical, loose associations, tangential, circumstantial,
process flight of ideas, incoherent
Thought • Preoccupations, obsessions, ideas of reference, delusions, suicidal or
content homicidal ideation
• Delusion: false belief that reason cannot dispel
Thought
• Ideas of reference and ideas of influence: belief that the radio is speaking
disturbances
to or about him/her, or that another force is controlling one's behavior
• Illusion: false sensory perception based on natural stimulation of a
sensory receptor
Perceptual • Hallucination: false sensory perception not based on natural
disturbances stimulation of a sensory receptor
• Depersonalization and derealization: extreme feelings of detachment
from self or environment

E. Sensorium
0 Alertness: awareness of environment, attention span, clouding of consciousness
0 Orientation to time, person, and place
° Concentration and calculation: serial 7s subtraction, calculation tasks
0 Memory: immediate retention/recall, recent memory (past few days), recent past
memory (past few months), and remote (e.g., childhood data)
° Fund of knowledge: should have relevance to educational & cultural background
0 Abstract thinking: interpretation of proverbs, describing similarities and differences
ofobjects
0 Levels of consciousness:
• Awake: full awareness of one's self and relationship to the environment
• Drowsy: can be aroused easily but promptly falls asleep when left alone
• Stupor: can be aroused only with vigorous and continuous stimulation
• Coma: cannot be aroused even with vigorous/painful stimulation
589
F. Glasgow Coma Scale (used mainly for trauma patients)
EYE OPENING I VERBAL RESPONSE I MOTOR RESPONSE
• 4 = opens spontaneously • 5 = normalconversation • 6 = normal
• 3 = opens to voice • 4 = disorientedconversation • 5 = localizesto pain
• 2 = opens to pain • 3 = incoherentspeech • 4 = withdrawsto pain
• 1 = no eye opening • 2 = incomprehensiblespeech • 3 = decorticateposturing
• 1 = no verbal response • 2 = decerebrateposturing
• 1 = no motor response

G. Insight & Judgement


0Insight: degree of personal awareness and understanding of illness (e.g., complete
denial, slight awareness of being sick & needing help, awareness of being sick but
blaming it on others, intellectual insight, true emotional insight
0Social judgment: understanding the likely outcome of personal behavior and its
influence
0Test judgment: patient's prediction of what he/she would do in imaginary situations
(e.g., smelling smoke in a crowded room)

II. HIGHER CORTICAL FUNCTIONS


Test if the history or mental status examination suggests a cerebral lesion
• Include test for the 3As: Agnosia, Apraxia, Aphasia

• Inability to understand the meaning, import, or


symbolic significance of ordinary sensory stimuli
Agnosia "Not knowing"
even though the sensory pathways and sensorium are
relatively intact
• Inability to perform voluntary acts even though motor
Apraxia "Inability to act"
and sensory systems and sensorium are intact

• Inability to understand or express words as symbols for


Aphasia "Lack of speech" communication, even though the primary sensorimotor
pathways & sensorium are relatively intact

Astereognosia • Inability to recognize common objects by touch


Agraphognosia
• Inability to recognize letters or numbers written on the skin
(agraphesthesia)
• Inability to recognize faces in person or in photos (can see the
Prosopagnosia face and describe the parts in detail but fails to recognize who
the person is)

Asomatognosia • Inability to locate, identify, and orient one's body parts


(autotopagnosia) • 2 types: tactile finger agnosia and right-left disorientation

Anosognosia • Lack of awareness of any bodily defect

Left-side • Inability to attend to the entire left-half of space (ignores


hemispatial persons, objects, or any stimuli from affected side, fails to
inattention dress that side, & fails to eat food from that half of the plate)
Inattention
to double • Inability to attend to one side of the body (usually the left)
simultaneous after simultaneous bilateral stimulation
cutaneous stimuli

590
B. Types of Aphasias
.
TYPES OF APHASIA
.
I FLUENCY COMPREHENSION REPETITTON

Anomic aphasia (poor naming) Good


Good
Conduction aphasia Poor
Good
Transcortical sensory aphasia Good
Poor
Wernicke's aphasia Poor
Transcortical motor aphasia Good
Good
Broca's aphasia Poor
Poor
Mixed transcortical aphasia Good
Poor
Global aphasia Poor
Example:AGO-year-Old malehad suddenonsetofright-sided weakness&facialasymmetry.Younotedthathe can follow
yourinstructions
(goodcomprehension) buthas difficulty
expressinghimself(poorfluency)andhas poorrepetition
ofwords.
Based on the table above. this patienthas Broca·saphasia (poorfluency,good comprehension,poor repetition).

III. CRANIAL NERVES (CN)


CN I NAME I FUNCTION I TESTS TO DO

• Use aron1atic, non·irritating substance, and test each


I Olfactory Sensory
nostril separately with eyes closed

• Visual acuity, visual field cuts by confrontation,


II Optic Sensory
pupillary light reflex (together with CN III), fundoscopy

• Primary gaze, extraocular muscles (all are innervated


III Oculomotor Motor by CN Ill except for superior oblique and lateral rectus
which are innervated by CN IV & VI, respectively)
• Check for ptosis & accommodation for CN Ill
IV Trochiear Motor • In those with altered sensorium, use the doll's eye
maneuver to check for eye movements

• Corneal reflex, somatic sensation on VI to V3 facial


V Trigeminal Mixed
distribution, masticatory muscles

VI Abducens Motor • Test together with CN Ill and IV (ocular motility)

• Test forehead wrinkling (differentiates central


and peripheral facial palsy), eyelid closure, mouth
retraction, whistling or puffing out of cheeks, and
VII Facial Mixed
wrinkling of skin over the neck
• Listen to labial articulations, test caste on anterior 2/3
of the tongue using salt and sugar

• Otoscopy and hearing tests (Weber, Rinne, Schwabach)


• Vestibular tests (e.g., Barany chair test, tilt tests for
Vestibuio-
VIII cochlear
Sensory postural vertigo and nystagmus)
• Vestibulo-ocular reflex tested with doll's eye
maneuver or caloric irrigation
Glosso-
IX pharyngeal
Mixed
• Listen for phonation and articulation
• Check swallowing, palatal elevation, gag reflex
X Vagus Mixed

Spinal • Inspect sternocleidomastoid & trapezius muscles


XI accessory
Motor
• Test strength of head movements & shoulder shrugging

• Check for lingual articulation, midline tongue


XII Hypoglossal Motor protrusion, lateral movement
• Inspect for tongue atrophy and fasciculations

591
IV. SOMATIC MOTOR SYSTEM
Inspection: body gestalt, posture, involuntary movements, fasciculations, muscle bulk,
joint malalignments, asymmetries
• Palpation: atrophy, hypertrophy, tenderness, muscle spasms, muscle tone (spasticity,
clonus, rigidity, or hypotonia)
Percussion of thenar eminence for percussion myotonia

AM IT ;

GRADE I BRITISH MEDICAL RESEARCH COUNCIL


(BMRC) GRADING OF MUSCLE STRENGTH
I GRADING OF MUSCLE
STRETCH REFLEXES

0 • Complete paralysis • Areflexia

• Only a flicker of muscle contraction but cannot


I • Hyporeflexia
move joint

2 • Moves part only when gravity eliminated • Normal


• Moves part full range against gravity but not
3 • Hyperreflexia
against any resistance
• Movesjoint through full range against resistancegreater
4 • Clonus present
than gravity but examiner can overcome the action

5 • Normal strength ---

B. Reflexes
0Superficial (skin-muscle) reflexes: elicited by stimulating receptors in skin & mucus
membranes
0Deep (muscle stretch) reflexes: elicited by stimulating receptors deep to skin (e.g.,
muscle spindle)
0The table below summarizes the different reflexes and their corresponding innervations

, Babinski: stimulate along lateral aspect


of foot bonom
Plantar reflex • Chaddock: stimulate lateral side of foot
(Babinski and Afferent: S1 • Oppenheim: knuckles over the shin and
Babinski-like Efferent: L5-S1-S2 move them down
maneuvers) • Gordon: grip the gastrocnemius
• Schaeffer: squeeze the Achilles tendon
• Gonda: grasp the 4th digit
Abdominal • Stroke the skin tangential to or toward
Upper quadrants: T8-9
skin and muscle the umbilicus using the blunt end of the
reflexes Lower quadrants: Tu-12
reflex hammer

Cremasteric Afferent: L1 • Elicited by scratching the skin of the


reflex Efferent: L2 medial thigh

• Tactile or noxious stimulus applied on


the skin around the anus
Anal Wink•
• Response: twitch-like contraction of the
anal sphincter

• Elicited by squeezing the glans penis,


causing a reflex contraction of the
Bulbocavernosus
Reflex• bulbocavernosus muscle detected by
pressing a gloved finger against the patient's
perineum; this may also elicit an anal wink
592
REFLEX I INNERVATION I INNERVATION & DESCRIPTION

Deep (Muscle Stretch) Reflexes


• Place your finger on the chin with patient's jaw
Masseter reflex Afferent: CN V
slightly open and strike the percussion hammer
(jaw jerk) Efferent: CN V
over your finger
• Place your thumb over the biceps tendon and
Biceps reflex
strike it with a percussion hammer
Brachioradialis • Place your thumb over brachioradialis tendon &
reflex strike your thumbnail with a percussion hammer
• Dangle or cradle the patient's arm in your hand
Triceps reflex
then strike the triceps tendon
• Tromner's method: support the patient's
completely relaxed hand and briskly flip distal
Finger flexion
phalanx of middle finger upward
reflex
• Hoffman's method: hold the middle finger & flick
the distal phalanx downward
Quadriceps reflex • With the patient sitting or supine with the knee
(knee jerk) bent, strike the patellar tendon with a crisp blow
• With the patient's knee bent and relaxed, slightly
Triceps surae
dorsiflex the patient's foot and tap the Achilles
reflex (ankle jerk)
tendon
Toe flexion reflex • Identical with the Tromner method for finger
(Rossolimo's sign) flexion reflex, but done on the middle toe
'The analwinkand bulbocavernous reflexare not partofthe routineneurologicexam(testonlyifwith
incontinenceor impotencesuggestiveof a lumbosacralcord/plexusor cauda equinalesion)

Base of toes
2 2 (shadedzone)
0 0
2 2
2 2

Lineof plantar
sUmulaUon

2 2
Start of plantar
stimulation

2 2
Proper documentationof muscle stretch reflexes Howto elicitthe Babinskireflex.Usingthe butt end
(MSRs), abdominal, cremasteric, and plantar of the reflexhammer,strokethe lateralside of the
reflexes.Thestickfigureshowsa subjectwithnormal sole as shown in the figure.Take note that it stops

I:
(grade2) triceps,biceps,brachioradialis,
quadriceps, beforethe base of the toes (shaded area). Normal
and tricepssurae reflexes.The fingerand toe flexion subjectswillwithdrawtheirfeetfromthe stimulusand
MSRsmay not be obtainable(grade 0) in normal flextheirbigtoe (flfexor
toe sfign).However,e(xtension
11
subjects. The arrows pointingdown represent of the bigtoe with anningo the sma toes extensor
normalflexortoe responsesto Babinskimaneuver. toe sign)is abnormaland suggests an uppermotor
neuronlesion.

593
V. SOMATIC SENSORY SYSTEM
A. Pattern of Distribution of Any Sensory Loss
(e.g.,dermatomal,peripheralnerve(s),plexus, centralpathway, or nonorganic)
SENSORY
MODALITIES
I TESTS

• Light touch over hands, trunk, & feet


Superficial
• Temperature discrimination & pain perception over hands, trunk, & feet
• Vibration perception at fingers and toes
Deep • Position sense of fingers and toes by using the fourth digits
• Test for stereognosis, directional scratch test, Romberg (swaying) test

B. Sensory Innervation Areas by Dermatomes


0 The numbers correspond to the spinal cord level of the dermatomes
° C: Cervical; T: Thoracic; L: Lumbar; S: Sacral

VI. CEREBELLAR SYSTEM


Check for nystagmus, hypotonia, incoordination, and ataxia
Do finger-to-nose test, rapid alternating movements, heel tapping and heel-to-shin
movements, and assess tandem gait

VII. MENINGEALS
• True nuchal rigidity is when the neck resists only flexion and moves freely through
rotation and extension
Brudzinski sign: leg adduction and flexion upon neck flexion
• Kernig sign (bent-knee and straight-knee leg-raising tests ofKernig and Lasegue):
passively flex one hip and knee 90 degrees, meningeal irritation causes the patient to
resist movement when you straighten the bent knee

Sources:BillerJ, et.al.DeMyer's
TheNeurologic Examination,6thedition
SadockBJ,et.al.Synopsisof Psychiatry:Behavioral
Science/ClinicalPsychiatry,
11thedition
594
OVERVIEW OF CEREBROVASCULAR DISEASE
I. ETIOPATHOGENESIS
A. Definition ofTerms
TERM I DEFINITION
Cerebrovascular • Umbrella term for any brain abnormality from a vascular pathologic
disease (CVD) process (e.g., occlusion, alteration in blood flow, rupture of a vessel)

Stroke or "brain • Sudden onset of focal (or global) neurologic deficit due to an
attack" underlying vascular pathology

Transient • Transient neurological dysfunction caused by a focal brain, spinal,


ischem ic attack or retinal ischemia, without evidence of infarction (normal cranial
(TIA) imaging) in which symptoms typically last less than an hour

B. Risk Factors for Stroke


Non-modifiable • Older age, male sex, ethnicity, family history of stroke

• Hypertension, diabetes, dyslipidemia, cardiac/vascular disease


Modifiable • Smoking, excessive alcohol intake, physical inactivity, obesity, stress
• Extracranial and intracranial stenosis

C. Pathophysiology & General Manifestations•


ISCHEMIC STROKE I HEMORRHAGIC STROKE
• Acute occlusion of intracranial vessel • Most commonly due to hypertension causing
leads to reduction in blood flow to spontaneous rupture of small penetrating
supplied brain region arteries of the Circle of Willis
• Due to atherothrombotic strokes • Summary of causes include (SMASH-U):
0 Structural lesions (e.g., cavernomas, AVMs)
(mostly), cardioembolism (commonly
from atrial fibrillation), artery-to-artery 'Medications (e.g., anticoagulant-induced)
embolism, and lacunar infarcts 'Amyloid angiopathy
0 Systemic (e.g.coagulopathy, thrombocytopenia)
• Deficit is maximal at onset
• Cardioembolic stroke: sudden onset • Hypertension
0 .!.[ndetermined cause
of maximal deficits (<5 min) with
rapid improvement of initially massive • Most common sites: basal ganglia (especially
symptoms ("spectacular shrinking of putamen & internal capsule), thalamus,
deficits") cerebellum, pons, and lobar areas
• Treatment focuses on saving the • Presents with headache, SBP,220 mm Hg,
ischemic penumbra vomiting, impaired consciousness & evolution
of focal deficits over minutes to hours
• Hypertensive ICH: develops over 30-90 mins
• Anticoagulant-induced ICH: may evolve for
as long as 24-48 hours
•imagingis importantbecausethereare no reliableclinicalfindingsto conclusively
distinguish
ischemiavs hemorrhage

D. Classification of Stroke, based on:

ETIOLOGY I (time fro~W!ke onset) I SEVERITY*

• Ischemic (80-85%): • Hyperacute (o-6 hours) • Mild (NIHSS 0-5)


atherothrombotic, • Acute (6-72 hours) • Moderate (NIHSS 6-21)

l-
_e_m_bo_l-ic_,_o_r_la_c_u_n_a_r
___
• Hemorrhagic (10-15%) ._·_S_u_b_a_c_u_te_(_3_d_ay_s_t_o_3_"_'e_e_ks_)_.__•_S_e_v_e_re_(_N_I
• Chronic (>3 weeks) ___
>2_2_)
__ --t I'.
*Severity
basedontheNationalInstituteof HealthStrokeScale(NIHSS)score
595
II. MANIFESTATIONS
A. Anterior vs. Posterior Circulation Strokes
I
PARAMETERS ANTERIOR CIRCULATION I POSTERIOR CIRCULATION
Incidence • More frequent (~80%) • Less frequent

• Internal carotid, middle cerebral,


Arteries • Vertebrobasilar artery & its
anterior cerebral, & anterior
involved branches
communicating arteries
Site ofischemia • Cerebral hemispheres • Brainstem and cerebellum
• Usually unilateral (contralateral • Bilateral signs frequently present
Laterality
to the hemispheric lesion) • Cerebellarlesions manifest ipsilaterally
• Monocular blindness
• Bulbar signs (dysarthria, dysphagia,
(amaurosis fugax)
dysphonia, dizziness, diplopia)
• Cortical dysfunction
• Facial paralysis involving all
• Facial weakness (sparing the
muscles of facial expression
Features frontalis & corrugator muscles)
ipsilateral to the lesion
contralateral to the lesion
• Eye deviation looking toward the
• Eye deviation looking away
hemiparetic side (or away from the
from the side of hemiparesis
side of the lesion)
(or toward side of the lesion)
• More complete neurologic
• Extensive ischemia of the brain
recovery for small infarcts
Prognosis may result in less complete
• Large infarcts often fatal because
neurologic recovery
vital centers are located in brainstem

Internal carotid
• Sudden onset transient monocular blindness
artery (I CA)
• Upper extremHy > lower extremity weakness
Middle cerebral • Dominant hemisphere: aphasia (global, Wernicke's, Broca's, etc.)
artery (MCA) • Non-dominant hemisphere: neglect syndrome, topographical
difficulty, constructional and dressing apraxia
Anterior cerebral • Lower extremity> upper extremity weakness
artery (ACA) • Abulia, muteness, perseveration, disinhibition

• Peripheral branches: memory deficits, cortical blindness, ocular


Posterior cerebral apraxia, and other visual deficits
artery (PCA) • Central branches: thalamic syndrome, cranial nerve deficits and
other manifestations of brainstem & diencephalic dysfunction
• Brainstem stroke syndromes (crossed signs: contralateral long
Vertebrobasilar
tract signs+ ipsilateral cranial nerve deficits)
artery
• Cerebellar manifestations

C. Lacunar Syndromes (<I.5 cm in size)


0 Infarct involving deep brain structures: cerebral subcortical white matter, basal
ganglia, thalamus, pons, and cerebellum
0 Intact higher cortical functions
0 Risk factors: diabetes, hypertension
0 Syndromes: pure motor, pure sensory, sensorimotor, dysarthria/clumsy hand, ataxic
hemiparesis
596
III. DIAGNOSIS
A. Plain Cranial CT Scan
0 Initial neuroimaging of choice to differentiate ischemic & hemorrhagic stroke
° CT findings in the hyperacute phase (look for early signs of infarction - GIRLS):
• Gray-white matter differentiation loss
• Insular ribbon sign
• aE,tery: dense middle cerebral artery sign
• Lentiform nucleus obscuration
• S.ulcal effacement
B. Cranial MRI
0 Better imaging for posterior circulation ischemic strokes because CT poorly visualizes
posterior fossa lesions (dense petrous bone degrades the image)
0 Acute infarcts are seen as bright signals on diffusion weighted imaging (DWI)
sequence with corresponding low-intensity signals on apparent diffusion coefficient
(ADC) map (observed as early as 30 minutes after onset ofischemia)
0 Disadvantages of MRI:
• Not sensitive in detecting acute hemorrhages
• More expensive & less widely available
• Longer acquisition time compared to CT
• Contraindicated in patients with metallic implants

Source:SSPHandbook
of Stroke:Guidelinesfor Prevention,
Treatment,
andRehabilitation,
6thedition
RapperA, et.al.AdamsandVictor'sPrinciples
of Neurology,
10thedition

EARLY SPECIFIC MANAGEMENT OF ISCHEMIC STROKE

I. THROMBOLYSIS: IV RECOMBINANT TISSUE PLASMINOGEN ACTIVATOR (rt-PA)


Thrombolytics restore cerebral blood flow in some patients with acute ischemic stroke
and may lead to improvement or resolution of neurologic deficits
Benefit is time-dependent so treatment should be initiated as quickly as possible
Thrombolysis within the first 3 hours from stroke onset offers substantial benefit (but
may be extended up to 4.5 hours)
Extension into the 3- to 4.5-h time window is recommended for:
0Patients s8o years of age
0Without history of both DM and prior stroke
0National Institutes of Health Stroke Scale (NIHSS) score s25
0Not taking any oral anticoagulants
0Without imaging evidence of ischemic injury involving >I/3of the MCA territory

A. Administration of Alteplase (rt-PA):


Dose: 0.9 mg/kg (max of 90 mg), 10% of total dose given as IV bolus over I minute then
0

the rest as infusion over 60 minutes


0Risk of symptomatic !CH from rt-PA: 6.4%
Guide to rt-PA Administration
• Admit the patient to an intensive care or stroke unit for monitoring
• Monitor BP every 15mins for 2 hours from the start of infusion, then every 30 mins
for 6 hours, then hourly thereafter for 16 hours (maintain <185/no mmHg prior to
thrombolysis and <180/io5 mmHg for at least 24 hours after)
• Monitor for headache, nausea and vomiting, any form of bleeding, decrease in
sensorium, or progression of neurologic deficits
• Check blood glucose (the only test required before starting IV thrombolysis): IV alteplase
is recommended in otherwise eligible patients with initial glucose levels ~50 mg/dL
• No need to wait for hematologic/coagulation testing if not suspecting abnormal results
(but reasonable in patients who have history of warfarin use if INR s1.7 or PT <15secs)
• No anti platelets or anticoagulants should be given within 24 hours of rt-PA treatment
• Insertion of nasogastric tubes, indwelling catheters, or intra-arterial pressure catheters
should be delayed if the patient can be managed without them
• Repeat CT or MRI scan at 24 hours after IV rt-PAbefore starting antiplatelets or anticoagulants

597
B. Management of Symptomatic lntracranial Bleeding within 24 hours after IV rt-PA
0 Discontinue IV rt-PA infusion
° Facilitate STAT plain cranial CT
° Check CBC, PT, INR, aPTT, fibrinogen level, blood typing, and cross-match
° Cryoprecipitate: IO units infused over 10-30 mins (additional dose if fibrinogen level
<200 mg/dL)
0 Tranexamic acid 1000 mg IV infused over 10 mins
° Continue supportive therapy (e.g., BP and glucose control, ICP management)
° Consider referring to a hematologist and/or neurosurgeon

• Acute intracranial hemorrhage on • Mild ischemic stroke symptoms


pretreatment cranial CT scan • Intracranial vascular malformation
• Acute head trauma • lntracranial arterial dissection
• Severe head trauma or prior ischemic • Small number of cerebral microbleeds on
stroke within 3 months MRl(1-10)
• History of prior intracerebral • Extra-axial intracranial neoplasm
hemorrhage • Procedural stroke
• Presentation suggestive of • Seizure at the onset of stroke symptoms
subarachnoid hemorrhage (SAH), even • Unruptured & unsecured intracranial
with normal CT aneurysm <IOmm
• Intra-axial intracranial neoplasm • Use ofwarfarin (unless INR <I.7 and/or
• Recent intracranial/intraspinal surgery PT <15seconds)
within 3 months • Use of direct oral anticoagulants (unless
• Coagulopathy (platelet <IO0,ooo mm', aPTT, INR, platelet count, clotting time,
INR>I.7, aPTT >40 secs, or PT>15 secs) thrombin time, or direct factor Xa assays
• Active internal bleeding are normal or has not received these for
• Infective endocarditis >48 hours & renal function is normal)
• Aortic arch dissection • Blood glucose <50 mg/dL or >400 mg/dL
• GI malignancy or recent GI bleed (unless it normalizes)
within 21days • Major surgery/serious trauma (excluding
• Use offull-dose (not prophylactic) head trauma) within previous 14 days
LMWH within the previous 24 hours • Arterial puncture at a non-compressible
• Sustained pretreatment SBP>185or site within 7 days
DBP>IJO mmHg despite aggressive • Lumbar puncture in preceding 7 days
treatment • Recent major surgery in preceding 14 days
• Urinary tract hemorrhage within the past
21days
• MI within the past 3 months
• Acute pericarditis
• Systemic malignancy
• Menstruation (but no history of menorrhagia)
• Pregnancy
• Illicit drug use
Source:2019Guidelines
fortheEarlyManagement
of Patients
WithAcutelschemic
Stroke.AHNASA;2019

II. ENDOVASCULAR THERAPY (MECHANICAL THROMBECTOMY)


Aims to remove the obstructing clots by introducing a clot retrieval device delivered via
an intravascular catheter, thereby restoring blood flow and minimizing tissue damage
• Recommended for selected patients with acute ischemic stroke within 6-16 hours
Beneficial if added to IV rt-PA among patients with intracranial ICA, M1, or M2
occlusion

598
III. ANTITHROMBOTIC THERAPY
A Non-Cardioembolic Ischemic Stroke
, Antiplatelet agents (rather than oral anticoagulation) is recommended
to reduce the risk of recurrent stroke and other cardiovascular events
• Aspirin within 24-48 hours after stroke onset is recommended (but
generally delayed until 24 hours later from those given IV alteplase)
• Acceptable options for initial therapy (choiceshould be individualized):
° Clopidogrel 75 mg OD
Anti platelet
° Cilostazol 100 mg BID
therapy
Triflusal 300 mg BID
0

• Dual antiplatelet therapy (clopidogrel + aspirin) may be initiated


within 24 hours and continued up to 21days of a minor ischemic stroke
(NIHSS ~3) or high-risk TIA (ABCD <!4)
• Ticagrelor is not recommended over aspirin in acute treatment of
minor ischemic stroke
• Urgent anticoagulant to prevent early stroke recurrence, halt
neurologic worsening, or improving outcomes is not recommended for
Anticoagulant
treatment of patients with acute ischemic stroke
therapy
• Anticoagulation might be considered in those who are found to have
abnormal findings on coagulation testing after an initial ischemic stroke
• Intermittent pneumatic compression is recommended
DVT
• Benefit of prophylactic-dose subcutaneous heparin (UFH or LMWH) is
prophylaxis
not well established

B. Cardioembolic Ischemic Stroke


0Benefit of anticoagulation within 4-14 days of ischemic stroke in patients with atrial
fibrillation (AF) should be weighed carefully against the risk of hemorrhagic conversion
(e.g., large infarctions, severe strokes or neurologic deficits, uncontrolled hypertension)
° Compute for the CHA2DS2-VASc and HAS BLED score of patients with AF
° For patients with history of ischemic stroke, AF, and coronary artery disease, the
usefulness of adding antiplatelets to oral anticoagulants is uncertain for purposes of
reducing risk ofischemic cardiovascular and cerebrovascular events

IV NEUROPROTECTION* (Th e5 "H'' prmc1p


. 1e )
• Target mean arterial pressure (MAP): uo-130 mmHg
• Permissive hypertension during the first 7 days, except in cases of:
0 Acute myocardial infarction
Avoid ° Congestive heart failure or acute pulmonary edema
Hypotension 0 Aortic dissection
0 Acute renal failure
0 Hypertensive encephalopathy
• See treatment of hypertension below
Avoid
• Target 02 sat: >94% (monitor via pulse oximeter and/or ABG)
liypoxemia
Avoid • Target CBG: 140-180 mg/dL
Hypoglycemia • Hyperglycemia during the first 24 hours after acute ischemic stroke is
or associated with worse outcomes
Hyperglycemia • Use isotonic saline (0.9% NaCl) & avoid glucose-containing (D5) fluids
• Target normothermia
• Relative risk of death or disability increases twofold for every 1°C
Avoid
increase in body temperature
Hyperthermia
• Treat fever with antipyretics/cooling blankets & work-up for source
• Benefit of induced hypothermia is not well established
'Role of neuroprotectiveand neurorestorativedrugs (e.g.,citicoline,cerebrolysin,neuroAiD)remainsa
matterof preferenceof the attendingphysician
599
EARLY SPECIFIC MANAGEMENT OF HEMORRHAGIC STROKE
I. MEDICAL TREATMENT
A. Blood Pressure Management
° For SBP 150-220 mmHg without contraindication to acute BP treatment: acute lowering
of SBP to 140 mm Hg is safe and can be effective for improving functional outcome
° For SBP >220 mmHg: it is reasonable to consider aggressive reduction of BP with
continuous IV drugs

B. Management oflncreased lntracerebral Pressure


Head elevation • Elevate head by 30-45°
• Mannitol 20% IV infusion (0.5-1.5g/kg every 3-6 hours)
Osmotic therapy
• Hypertonic saline (target Na: 145-155mmol/L)
Serum osmolality • Maintain at 300-320 mosmol/kg
• Target pCO2 of 30-34 mmHg (effect lasts only -6hrs)
• Only for acute severe neurological decline from brain
Hyperventilation
swelling as a bridge to more definitive therapy (not used as
prophylaxis)
'ICP monitoringmay be considered for patients with GCS :SB,those withclinicalevidence of
transtentorialherniation,or those with significantintraventricularhemorrhage or hydrocephalus
(may be reasonable to maintaincerebral perfusion pressure of 50-70mmHg)

"Corticosteroids should not be administeredfor treatment of elevated ICP in ICH

C. Other Aspects of Management


0 Avoidance of hyper- and hypoglycemia
° Control seizures
0 Ensure neuroprotection
0 Maintain adequate nutrition and fluid & electrolyte balance
0 Stool softeners
0 DVT prophylaxis, early rehabilitation once stable, and bedsore precautions

II. SURGICAL TREATMENT


IMMEDIATE SURGICAL I OTHER PATIENTS WHO MAY
CANDIDATES BENEFIT FROM SURGERY
• Cerebellar hemorrhage >3 cm with • Basal ganglia or thalamic hemorrhage
neurologic deterioration or brainstem • ~GCS 5
compression and hydrocephalus from • Supratentorial hematoma with volume >30cc
ventricular obstruction
• Bleed from structural lesions (e.g.,
aneurysms, AVM)
• Clinically deteriorating young
patients with moderate or large lobar
hemorrhage
• Ventricular drainage for intraventricular
hemorrhage with moderate to severe
hydrocephalus, especially in patients
with decreased level of consciousness

600
BLOODPRESSUREMANAGEMENTFORACUTESTROKE
The relationship between hypertension and stroke is very dynamic
Management during the acute onset of stroke (whether ischemic or hemorrhagic) poses
a challenge due to the intricacies of how elevated BP must be handled

00 ss

ACUTE
INTRACEREBRAL
HEMORRHAGE

BP threshold
• Severe HPN (SBP >220
for initiating • >185/uo mmHg • SBP ,!!80 mmHg
therapy or DBP >120 mmHg)

• If with severe HPN, • If SBP ,!!80, careful


• <185/110mmHg reduce BP by 15% lowering to 140-160
prior to & during during first 24 hours mmHg is considered
BP targets thrombolysis after onset of stroke • Avoid reductions
• <!80/105mmHg in • Maintain target mean ,!60 mmHg in I hour
the next 24 hours arterial pressure (MAP) • Do not lower BP
ofuo-130 mmHg acutely to <140mmHg

Acutestroke

No Yes
If 5220 mmHg,
carefulBP
loweringto
KeepBPstable. Maintain MAP between
140-160mmHg.
Avoidvariability. No Yes 110-130mmHg. If with
Avoidreductions
of ~60 mmHgin MaintainBP concomitantmedicalissues,
1 hr. <185/110 mmHg initiateBPlowering
priorto and
Forthe next 24 during
hrsafter thrombolysis
treatment,
maintain BP at
<180/105 mmHg

If neededfor BPlowering, use nicardipine5 mg/hr IV,titrate up by 2.5 mg/hr every 5-15 minutes,with
maximumof 15 mg/hr.
If available,labetalol 10 mg IV over 1-2 min followed by continuousIV infusion of 2-8 mg/min mayalso be used.

*Concomitantseriousmedicalissues:acutecoronarysyndrome(ACS),acuteheartfailure,aortic
dissection,
post-fibrinolysis,
symptomatic
intracerebral
hemorrhage,
or preeclampsia/eclampsia
J ClinHypertens
Source:OnaDID,et al. 2020CPGin thePhilippines. 2021
601
PRIMARY AND SECONDARY PREVENTION OF STROKE

RISKOR I REMARKS
COMORBID
• Most important modifiable risk factor (3-4Xhigher risk of stroke)
• Primary prevention: degree of BP reduction is more imponant than the specific agent
Hypertension • Secondary prevention: anti hypertensives (e.g., thiazide diuretics, ACEi,
ARBs) recommended for all types of ischemic stroke and TIA patients
beyond the hyperacute period (office BP goal is ,;130/80)
• For most (especially those <65 years of age and without life-limiting comorbid
illness): target HbA1c ~7%
• Primary prevention: aspirin is not recommended if with no evidence of
Diabetes
atherosclerotic disease
mellitus
• Secondary prevention: glucose control; all ischemic stroke & TIA patients should
be screened for DM (HbA1c more accurate in the immediate post-event period)
• Patients with prediabetes & ischemic stroke/TIA: metformin may be beneficial
• Primary prevention: stat in therapy reduces all stroke types
Dyslipidemia
• Secondary prevention: see dyslipidemiaguidelinesin Cardiologychapter
• 4-5x higher risk of stroke (regardless of whether paroxysmal or sustained)
Atrial • Oral anticoagulation for patients with CHA2DS2-VASc~I (provided that the
fibrillation score of 1is not due to female gender)
(AF) • In those with high risk of hemorrhagic conversion in the setting of AF, it is
reasonable to delay initiation of oral anticoagulation beyond 14 days
• Carotid duplex ultrasonography to screen for carotid stenosis in symptomatic
patients or asymptomatic patients with risk factors for significant carotid disease
• Highest risk of recurrent stroke from symptomatic carotid stenos is during the
Carotid
first few days after index event
Stenosis
• Carotid revascularization must be done between 48 hours to 7 days of index
event for secondary prevention in patients with minor, nondisabling stroke
(Modified Rankin Scale score 0-2)
• Risk of recurrent ipsilateral stroke highest if?o-99% stenosis and those with
recent symptoms within 2 weeks
• No data on primary prevention
• Secondary prevention:
Intracranial 0 Vascular studies for screening of intracranial artery stenos is is recommended
stenosis
for ischemic stroke or TIA
0 Aspirin is recommended and clopidogrel or cilostazol may be added to
aspirin therapy (usefulness of clopidogrel alone, aspirin+dipyridamole
combination, ticagrelor alone, or cilostazol alone is not well established)
• Smoking cessation for all current smokers & avoidance of passive smoke
Smoking
• Consider drug therapy (e.g., nicotine replacement, bu pro pion, or varenicline)
• Eliminate or reduce alcohol consumption
Excessive • Light-to-moderate intake of alcohol:
alcohol 0 1 drink per day for non-pregnant women
0 2 drinks per day for men

• Mediterranean-type diet (monounsaturated fat, plant-based foods, fish


consumption, with either high extra virgin olive oil or nut supplementation)
Nutrition in preference to a low-fat diet to reduce risk of recurrent stroke
• Reduce sodium intake by at least r g/day (2.5 g/day salt)
• Routine supplementation of single- or multi-vitamins is not recommended
• Goals:
• BM! 18.5-24.9kg/m'
Obesity
0 Waist-hip-ratio~ I (men) or 0.85 (women)
0 Waist circumference 35 inches (men) or 31 inches (women)
602
SUBARACHNOID HEMORRHAGE (SAH)
I. ETIOPATHOGENESIS
Most common cause: trauma
Most common cause of non-traumatic SAH: ruptured aneurysm
Most common site of aneurysm: anterior communicating artery- anterior cerebral
artery (ACom-ACA) junction
Other causes: bleeding from a vascular malformation (arteriovenous malformation or dural
arteriovenous fistula) and subarachnoid extension of a primary intracerebral hemorrhage

II. CLINICAL MANIFESTATIONS


Sudden severe headache ("the worst headache of my life") in -80% of cases
May be accompanied by nausea/vomiting, loss of consciousness, neck stiffness,
photophobia, focal neurologic deficits, and seizures
Delayed neurologic deficits may be due to re-rupture, acute hydrocephalus, or vasospasm
Clinical findings of subarachnoid hemorrhages depend on aneurysm site

III. CLASSIFICATION OF SAH


A. Clinical Classifications (Hunt & Hess, 1968;Drake, 1988)
0 To assess severity of injury & predict the prognosis & outcome in patients with SAH
0 A higher grade means poorer prognosis & outcome

GRADE* I HUNT AND HESS**

• Asymptomatic or mild headache, slight


I 15
nuchal rigidity

• Moderate-severe headache, nuchal None


II rigidity, no neurologic deficit other than 13-14
cranial nerve palsy

• Mild focal neurologic deficit or alteration


III 13-14 +
in mental status

IV • Stupor, moderate to severe hemiparesis 7-12 + or-

V • Comatose, decerebrate posturing 3-6 +or-


'Followthe highergrade. In a patientwho is awake but withsignificanthemiplegia,the grade is IV.
"In Hunt& Hess, grades 1-111have favorableoutcomes, & these patients are candidates for early surgery
Sources:Hunt& Hess,Journalof Neurosurgery; 1968
Drake.Journalof Neurosurgery; 1988

B. Fisher Scale
0 Based on cranial CT scan findings
0Used to predict the risk of cerebral vasospasm based on the amount and location of
blood in the initial scan within 5 days of SAH
SCORE I CT SCAN FINDINGS

I • No blood detected

• Diffuse deposition of subarachnoid blood


2 • No clots & no layers of blood 2'1 mm (look at the interhemispheric fissure,
insular cistern, or ambient cisterns)

3 • Localized clots and/or vertical layers of blood 2'! mm in thickness

• Diffuse or no subarachnoid blood but intracerebral or intraventricular clots


4 present
Source:FisherCM.Neurosurgery;
1980
603
IV. DIAGNOSIS
DIAGNOSTIC I REMARKS/FINDINGS
• Remains the cornerstone of diagnosis
Plain cranial
• Location of blood within the subarachnoid space correlates with the
CT scan
location of the aneurysm in most cases
• Performed as a follow-up test when CT scan has shown no SAH
and has excluded possible contraindications to LP (e.g., intracranial
Lumbar mass effect, elevated ICP, obstructive hydrocephalus)
puncture (LP) • Done to evaluate CSF for the presence ofRBCs and xanthochromia
• Xanthochromia after 5-7 days when rate of negative CT increases
sharply
• Gold standard in determining the cause of the SAH (i.e., for
Cerebral detection of cerebral aneurysm)
angiography • Can provide information on vascular anatomy and aneurysm
location & source of bleeding
CT • Acceptable options to catheter angiography in the following
angiography situations:
(CTA)orMR 0 Poor grade patients
angiography ° Catheter angiography cannot be d~ne in a timely fashion
(MRA) 0 As a follow-up diagnostic if the initial angiogram is negative

V. MANAGEMENT
ASPECT I MANAGEMENT
• Bed rest v.,ithout bathroom privileges until aneurysm is secured
• May start feeding if with no planned immediate surgical intervention
• Analgesics for headache (avoid NSA!Ds and aspirin)
General • Proton pump inhibitors or H2 blockers for stress gastritis prophylaxis
symptomatic • Anti-emetics for nausea and vomiting, stool softeners
treatment • Sedatives for restlessness and agitation
• Antipyretics and/or cooling blankets for fever
• DVT prophylaxis with pneumatic compression devices with or
without thigh-high anti-embolic stockings
• Nimodipine 30 mg/tab 2 tabs PO q4h x 3 weeks
• Short-term anticonvulsants for patients with documented seizures;
prophylactic anticonvulsants may be given for patients with poor
Early specific grade in the immediate post-hemorrhagic period
treatment • Manage increased ICP (as discussed in !CH)
• Optimal BP management (IV nicardipine to maintain SBP<r50
mmHg in unsecured aneurysm)
• Correction of hyponatremia & maintain euglycemia & euvolemia
• Early surgery (ideally within 72 hours from ictus) for good to
moderate grade SAH (Hunt and Hess or WFNS grades I-III)
• Poor grade SAH may warrant early surgery if with hematoma and
Timing of
hydrocephalus
surgery
• Clipping: MCA aneurysms and large parenchymal clots
• Coiling: Poor clinical grades, those with vasospasm, elderly,
posterior circulation aneurysms

604
SECTION THREE
SEIZURE AND EPILEPSY

OVERVIEW OF SEIZURES AND EPILEPSY


I. DEFINITION OF TERMS
TERM I DEFINITION
Seizure • Paroxysmal electrical discharges of the brain
• Disease of the brain defined by any of the following conditions:
1. A least two unprovoked (or reflex) seizures occurring >24 h apart
2. One unprovoked (or reflex) seizure and a probability of further
seizures similar to the general recurrence risk (at least 60%) after
Epilepsy
two unprovoked seizures, occurring over the next IO years
3. Diagnosis of an epilepsy syndrome
• Seizure is the event while epilepsy is the disease associated with
spontaneously recurring seizures

Reflex • Seizures provoked habitually by an external stimulus: visual (most


epilepsy common), auditory, somatosensory, etc.
• Reactive, provoked, or situation-related seizures which may be
metabolic, toxic, structural, infectious, or inflammatory
Acute
• Interval between the insult and the seizure may vary according
symptomatic
to the underlying clinical condition but the usual defining time is
seizure
within I week (except for alcohol withdrawal seizures which occur
within 7-48 hours of the last drink)
Refractory
or drug- • Failure to achieve seizure freedom after adequate trials of 2 tolerated
resistant and appropriately chosen antiepileptic drug (AED) schedules
seizure
Seizure • Freedom from all seizure types for 12 months or 3x the pre-
freedom intervention inter-seizure interval, whichever is longer

II. ETIOPATHOGENESIS
Conditions which cause seizure:
0 Population of pathologically excitable neurons (epileptogenic focus)
0 Increase in excitatory activity (mainly glutamatergic)
0 Reduction in inhibitory GABAergic projections
Kindling: creation of a secondary seizure focus due to repeated stimulation with
subconvulsive electrical pulses from another established focus

III. MANIFESTATIONS
A. Acute Symptomatic Seizures
0 Most common seizure type: generalized tonic-clonic
• Difference from epilepsy:
• Proximate cause of seizure is clearly identifiable (usually within I week from insult)
• Unlikely to recur unless the underlying acute causal condition recurs
• Most individuals need not be treated with long-term antiepileptic drugs (may be
warranted only until the acute condition is resolved)

B. Focal Seizures
• Originating within networks limited to one hemisphere
0 Patient may be aware or has impaired awareness, & onset may be motor or non-motor

° Can evolve to a bilateral convulsive seizure


605
C. Generalized Seizures
TYPE I DESCRIPTION
• Begins abruptly without warning; some with vague premonitory symptoms
• Tonic Phase:
0 Eyes open and roll up, elbows flex, arms pronate

0 Associated incontinence, moaning, cyanosis, and apnea


0 Increased HR, BP and pupillary size
• Clonic Phase:
Generalized 0 Tonic phase evolves into the clonic phase after 10-20 sec
tonic-clonic 0 Generalized clonic movements, atonic between jerks, tongue biting
seizures 0 Periods of relaxation progressively increase until the end of ictal phase
(Grand Mal) 0 Lasts <I min
• Postictal state:
0 Regular respiration resumes
0 May have headache and muscle soreness
0 Gradually regains consciousness over minutes to hours
0 May have postictal confusion

Myoclonic • Sudden brief muscle contraction


seizures • May involve one part or the entire body (shock-like jerks)
• Usually occur in children with normal intelligence
Absence • EEG: Generalized 3-Hz spike-and-wave electroencephalogram
seizures
• Brief duration (usually a few seconds)
(Petit Mal or
• Abrupt recovery (consciousness returns as suddenly as it was lost)
Pyknoepilepsy)
• No aura and no postictal phase

V. DIAGNOSIS
DIAGNOSTIC I REMARKS
• CBC, random sugar, creatinine, electrolytes, 12-LECG
Basic tests
• Septic work-up if warranted (e.g., chest x-ray, urinalysis)
• Most important diagnostic procedure for patients with epilepsy
Electro- • A normal EEG does not totally rule out epilepsy and an abnormal
encephalogram EEG does not always mean epilepsy
(EEG) • Region with the earliest spike activity corresponds best to the
epileptogenic focus (guide for epilepsy surgery)
• Most useful tool for detection of underlying structural abnormalities
Cranial MRI with
• Indications: focal seizures, intractable seizures, progressive neurologic
contrast (indicate
disease, or structural lesions that may warrant surgical intervention
seizure protocol)
• Cranial CT with contrast if MRI is not available
Lumbar puncture • Done if CNS infection is suspected

MANAGEMENT OF SEIZURES
I. GENERAL PRINCIPLES IN INITIATING ANTIEPILEPTIC DRUGS (AED)
Goal: seizure freedom without adverse drug reactions
Monotherapy is the mainstay of treatment
Stan at a low dose & gradually increase until seizures are controlled or adverse effects appear
Indications for prescribing AEDs in a single unprovoked seizure:
° Focal seizures
0 Signs of a focal lesion on neurologic evaluation
0 Abnormal neuroimaging or abnormal EEG (e.g., focal slowing, epileptiform activity)

II. CRITERIAFOR CONSIDERINGDISCONTINUANCEOF ANTIEPILEPTICDRUGS


Seizure-free 2-5 years on antiepileptic drugs (mean: 3.5 years)
Single type of partial or generalized seizure
Normal neurologic examination or normal IQ
Normal EEG
606
III. CHOICE OF ANTIEPILEPTIC DRUGS (AED)
TYPE I AED I LOE
Adults with generalized onset tonic- • CBZ, LTG, OXC, PB, PHT, TPM, VPA B
clonic seizures • GBP, LEV, VGB D

• CBZ, LEV, PHT, ZNS A


•VPA B
Adults with focal-onset seizures
• GBP, LTG, OXC, PB, TPM, VGB C
• CZP,PRM D
• GBP, LTG A
Elderly adults with focal-onset
• CBZ C
seizures
• TPM, VPA D

Juvenile myoclonic epilepsy (JME) •TPM,VPA D


CBZ- cartiarnazepine.LTG- lamotrigine.
OXC-oxcarbazepine,PB- phenobarbital,
PHT- phenytoin,
TPM- topiramate,VPA-
valproicadd,GBP- gabapentin,LEV- levetiraoetam,
VGB-vigabatrin,ZNS- zonisamide,CZP- donazepam,PRM- primidooe

Level of Evidence (LOE)


• A: Antiepileptic drug established as efficacious or effective as initial monotherapy
• B: Antiepileptic drug probably efficacious or effective as initialmonotherapy
• C: Antiepileptic drug possibly efficacious or effective as initial monotherapy
• D:Antiepileptic drug potentially efficacious or effective as initial monotherapy

STATUS EPILEPTICUS
I. DEFINITION
Condition resulting either from the failure of seizure termination or from the initiation of
abnormally prolonged seizures after 5 minutes
Condition which can have long-term consequences after 30 minutes

II. TREATMENT PROTOCOL


TIME I MANAGEMENT
• Short history and focused PE, check ABC, insert IV line, and extract blood for
0-5 tests (e.g., chemistry, toxic screen)
mins • Diazepam 5-10 mg at 2-5 mg/min until seizure stops or a total of 20 mg has been given
• Early referral to a neurologist may be warranted

• If seizure persists, choose one from the following 2nd line AEDs:
• Phenobarbital 15-20 mg/kg JV loading dose (LD) at <l00 mg/min
• Phenytoin 18-20 mg/kg IV LD at a rate not exceeding 50 mg/min
5-30
• Valproic acid 20-30 mg/kg IV LD in at least 50 cc pNSS over 15min or aqo mg/min
mins
• Levetiracetam 20 mg/kg IV LD in 100 cc of pNSS over 15 minutes
• Intubate the patient before giving loading dose of phenobarbital
• Monitor BP & hook to a cardiac monitor when giving loading dose of phenytoin

30-40
• Additional 5-10 mg/kg IV of the 2nd line AED started, same infusion rate
mins

40-60 • Use one of the 2nd line AEDs that was not selected initially
mins • Intubate patient (if not yet done) & do continuous EEG monitoring (if available)

• May use any of the following anesthetic agents:


• Midazolam 0.2 mg/kg IV bolus followed by infusion at l mcg/kg/min; may
increase by 1 mcg/kg/min every 15 min until seizures controlled (mean infusion
>60 rate is 8 mcg/kg/min and range is 3-21 mcg/kg/min)

I
mins • Propofol 1-2 mg/kg IV bolus to terminate seizure, then infuse at 2-15 mg/kg/hr
titrated by I mg/kg/hr (may be used for a maximum of 48 hours)
• Thiopental 5 mg/kg IV bolus followed by infusion of 0.5-6 mg/kg/hr
• Referral to anesthesiologist may be warranted for propofol and thiopental
Source: PhilippineNeurologicAssociation EpilepsyCouncil& Glauser T, et.al. Updated lLAE
607
SECTION FOUR
INFECTIONS OF THE CENTRAL NERVOUS SYSTEM (CNS}
BACTERIAL MENINGITIS
I. ETIOPATHOGENESIS
Most common meningeal pathogens are normal inhabitants of the nasopharynx
Routes of infection include hematogenous spread, extension from cranial structures adjacent
to the brain, & iatrogenic (post-neurosurgical procedure or cranial appliance insertion)

• S. pneumoniaeand N. meningitides
• L. monocytogenes
Adults (in order of • Staphylococci
frequency) • Gram-negative bacilli (E.coli,Klebsiella,Enterobacter,
P.aeruginosa)
• H. influenzae
Trauma or • Staphylococci
neurosurgical • Gram-negative bacilli
procedures • S. pneumoniae
• S. pneumoniae
Immunocompromised • L. monocytogenes
• Gram-negative bacilli

II. MANIFESTATIONS
Subacute onset, but with rapid progression of symptoms within hours to days
Triad offever, headache, and nuchal rigidity
Other cerebral symptoms: seizures, confusion, cranial nerve palsies, possible focal deficits

III. DIAGNOSIS
DIAGNOSTIC I REMARKS
• Positive blood culture in 40-60% (H. influenzae,meningococcal, and
pneumococcal meningitis)
Basic
• Peripheral leukocytosis
laboratory
• Hyponatremia (usually from SIADH)
findings
• Procalcitonin >0.2 ng/mL in patients with severe bacterial meningitis
(versus viral infection)
• Indispensable part of the examination
Lumbar
• Low CSF glucose, elevated CSF protein, PMNs as predominant WBC
puncture
type, positive gram stain & culture
• MRI with contrast: displays meningeal exudates & cortical reactions
• CT scan with contrast: shows meningeal enhancements and lesions
Imaging that erode the skull and provide routes for bacterial invasion
• CXR, sinus, and skull films: check for pneumonia, cranial
osteomyelitis, paranasal sinusitis, mastoiditis

608
IV. MANAGEMENT
Bacterial meningitis is a medical emergency and treatment should begin while awaiting
the results of diagnostic tests
, Repeat lumbar puncture is not routinely recommended to document CSF sterilization
and improvement ofCSF parameters except in patients who have not responded
clinically after 48h of appropriate antimicrobial therapy

-=:- Suspicionfor bacteriaI


meningitis?
_:=

! Yes

History:
lmmunocompromised,
history
of CNSdisease,
newonsetseizure
PE:Papilledema,altered consciousness,focal neurologicdeficit
I
I Others: Delay in performance of diagnostic lumbar puncture
I
No 1 1 Yes
STAT:BloodcultureandCSFstudies
STAT:Bloodculture
I (lumbarpuncture) I I
l I
I Dexamethasone+ empirical antimicrobialtherapy

l I
I CSFfindingsconsistentwith bacterial meningitis
I NegativeCTscanof the head

l I
I PositiveCSFgram stain
I
I CSFstudies(lumbarpuncture)

No I I Yes

therapy I I Dexamethasone
I Dexamethasone
+ empiricalantimicrobial + targetedantimicrobial
therapy

Dexamethasone: 10 mgIV 10-20minutesbeforefirstdoseof antibiotics, thenq6hfor 4 days


Shownto reducein-hospitalmortalityand improveoveralloutcome
No effectson neurologicsequelaelike hearingloss
Givento HIV-negativeadultswith suspectedor provenpneumococcalmeningitis
Shouldonly be continuedif the CSFgramstainrevealsgram-positivediplococci,or if bloodor CSF
culturesare positivefor S. pneumoniae
Shouldnot be givenif antimicrobialtherapywas alreadystartedbecauseadministrationof
dexamethasone is unlikelyto improvepatientoutcome
Source:IDSAManagementAlgonthm forAdultswithSuspectedBacterial
Mening1t1s
Tunkel,AR.,et al. PracticeGuidelines
fortheManagement
of Bacterial
Meningitis.
IDSAGuidelines,
2004

A. Empiric Therapy (while waiting for diagnostics)


PATIENT I ANTIMICROBIAL THERAPY
• Third-generation cephalosporin, plus
3 months-50 years old • Vancomycin, with or without
• Ampicillin
• Third-generation cephalosporin, plus
>50 years old • Vancomycin,plus
• Ampicillin
• Vancomycin, plus
lmmunocompromised state
, Ampicillin and ceftazidime
, Third-generation cephalosporin, plus

I
Basilar skull fracture
• Vancomycin
Head trauma, neurosurgery • Vancomycin, plus
CSF shunt • Ceftazidime
Source:RapperA, et.al.AdamsandVictorsPnnc1ples
of Neurology,
10thEd1t1on
609
B. Specific Antimicrobial Therapy
MICROORGANISM
I STANDARD THERAPY
Haemophilus influe11Zae
Beta-lactamase-negative
. Beta-lactamase-positive
• Ampicillin
• 3rd-generation cephalosporin
Neisseria meningitides • Penicillin G or 3rd-generation cephalosporin
Streptococcus pneumoniae
Penicillin sensitive • Penicillin G or ampicillin
Penicillin intermediate sensitivity • 3rd generation cephalosporin
Penicillin highly resistant • Vancomycin + 3rd-generation cephalosporin
Enterobacteriaceae • 3rd-generation cephalosporin
Pseudomonas aeruginosa • Ceftazidime or cefepime
Listeria monocytogenes • Ampicillin or penicillin G
Streptococcus agalactiae • Ampicillin or penicillin G

.. Methicillin-sensitive
Staphylococcus aureus
• Nafcillin or oxacillin + 3rd-generation cephalosporin
Methicillin-resistant • Vancomycin + 3rd-generation cephalosporin
Staphylococcus epidermidis • Vancomycin
Durationof Treatment:
Neisseriameningitides,
Haemophi/us influenzae:
7 days
Streptococcuspneumoniae: 10-14days
Streptococcusagalactiae:
14-21days
Aerobicgram-negativebacilli:21days
Listeriamonocytogenes:~21days
Source:RapperA, et.al.AdamsandVictor'sPnnc,plesof Neurology,10thEd1t1on

TUBERCULOUS MENINGITIS
I. ETIOPATHOGENESIS
Bacterial seeding of meninges & subpial regions of the brain causes formation of tubercles
Rupture of one or more tubercles and discharge of bacteria into the subarachnoid space
• Exudate is not confined to subarachnoid space but can invade brain tissue (meningoencephalitis)

II. MANIFESTATIONS
Same as bacterial meningitis, but with slower evolution of symptoms
Prodrome of2-4 weeks: nonspecific symptoms of fatigue, malaise, and possibly fever
Fever, meningismus, headache, vomiting, seizures, & CN palsies (CN VI commonly affected)
2/3 have active TB elsewhere
Complications: hydrocephalus and cerebral infarctions

III. DIAGNOSIS
DIAGNOSTIC I REMARKS

• Lymphocytic pleocytosis (50-500/mm')


• Reduced glucose and increased protein
CSF studies
• AFB smear is seldom positive so send CSF for direct culture
• CSF TB PCR rapidly permits detection of small amounts of tubercle bacilli

• Cranial CT or MRI with contrast: triad of basal enhancement,


Imaging
communicating hydrocephalus, multiple vasculitic infarcts
studies
• Chest x-ray to identify lymphadenopathy, Ghon's complex, or miliai;y lesions
• Serum Na (low from SIADH or Addisonian state if with adrenal TB)
Other tests
• Work up for possible disseminated TB (e.g., pulmonary, GI, GU)
610
IV. GRADING AND STAGING OF TUBERCULOUS

G~I MODIFIED
MENINGITIS
Med;oal
Research I
I
VELLORE GRADING Council FINDINGS
VELLORE
SYSTEM
GRADING SYSTEM ~RC)
tage

• Headache, vomiting, • GCS15


fever ± neck stiffness • Headache, vomiting,
I • Fully
• Normal sensorium fever± neck stiffness
• No neurologic deficit • No neurologic deficit I conscious
• No paresis
• Normal sensorium • GCS15
II
• With neurologic deficit • With neurologic deficit
• Altered sensorium but • GCS9-14 • ! sensorium
easily arousable
III , With/without 2 • Localizing
• With/without dense
neurologic deficit neurologic deficit pain

• Deeply comatose • GCS 3-8 • Deeply


IV • Decerebrate or • With/without 3 comatose±
decorticate posturing neurologic deficit gross paresis
Joseph,&Abraham,1991
Chandy,
Source:Palur,RaJshekhar,
2009
& Chandy,1998;Rajshekhar,
Mathew,Rajshekhar,
V. MANAGEMENT
2 months HRZE then IO months HR for a total of12 months
• Role of corticosteroids: reduction of mortality but no effect on residual disability

BRAIN ABSCESS
I. ETIOPATHOGENESIS
Almost always from bacteremia and bacterial focus elsewhere in the body (only 10%
comes from infections outside - e.g., skull fractures, surgical operations)
Most common etiologic agents: anaerobic or microaerophilic virulent streptococci in
combination with other anaerobes (Bacteroides,Fusibacterium,and Prevotella)
Routes of infection:
0 Direct extension (e.g., otogenic or rhinogenic abscess): high frequency in cerebellum
via venous route because of close anatomic relationship with transverse sinus
0 Hematogenous (majority of brain abscesses): a septic focus leads to metastatic
spread to deep cerebral artery territories (most common site is the distal territory of
middle cerebral arteries)
0 Unknown source in -20% of cases (cryptogenic)

II. MANIFESTATIONS
Most frequent initial symptom: headache
Other early symptoms: drowsiness and confusion, focal or generalized seizures, focal
motor, sensory, or speech disorder (focal deficits depend on location of abscess)
Signs of systemic infection may be absent (fever & leukocytosis not consistently present)
Sudden deterioration may be seen if the abscess ruptures into the subarachnoid or
ventricular CSF

III. DIAGNOSIS
DIAGNOSTIC I REMARKS

• All abscesses >I cm are seen on imaging


Cranial CT • CT: rim-enhancing lesion (capsule enhances while center and surrounding
edema are hypodense)

• Tr-WI: rim-enhancement of capsule, hypointense center, restriction of cliffusion


MRI with
• T2-WI: hypointense capsule, hyperintense surrounding edema
contrast
• Cerebritis: dot-sized areas of decreased intensity that enhance with contrast
611
IV. MANAGEMENT
ASPECT I MANAGEMENT
• Otitis, mastoiditis, & sinusitis: metronidazole or ampicillin-sulbactam +
3rd-generation cephalosporin
• Dental infection: penicillin+ metronidazole or amoxicillin-sulbactam
Antibiotics•
• Intravenous drug abuse: vancomycin + cefepime or ceftazidime
• Penetrating trauma: vancomycin + 3rd-generation cephalosporin
• Postsurgical: vancomycin + cefepime or ceftazidime or meropenem
For elevated
• Manni to!, hypertonic saline, or dexamethasone
ICP
• Abscesses >Z.5 cm are aspirated or excised
Surgery • Stereotactic aspiration preferred for deep-seated lesions
• Open removal attempted only for solitary, superficial, & encapsulated abscess
*Choiceof antibioticshouldbe basedon commoncausativeorganisms& predisposing
conditions

REFERENCES
1. Albers, G. W., Marks, M. P.. Kemp, S., Christensen, S.,Tsai, J.P.,Onega-Gutierrez.,S.,... Lansberg, M. G. (2018).Thrombectom}' for Stroke
at 6 ro 16Hours with Selection byPerfusionImaging.Ne-.vEngland Journal of Medicine,NEJMoa17139i3-
2.American Academy of Neurology Quality Standard Subcomminee. (1996). Practice Parameter: A guideline for discontinuing
amiepileptic drugs in seizure-free patients--Summary Statement Neurology,47(2),6oo-6o2.
3-Beghi, E., Carpio.A, Forsgren,L, Hesdorffer, D. C., Malmgren, K., Sander, J.W., Hauser, W. A (2010). Recommendation for a definition
of acute symptomatic seizure. Epilepsia,51(4),671-675.
4. Biller,J.,Grucncr, G.,& Brazis,P.(2011).DeMyerSThe NeurologicExamination (6th ed.).USA:The McGraw-HillCompanies, Inc.
5. Drake,C. (1988).Report ofWorld Federationof NeurologicalSurgeons Committee on a UniversalSubarachnoid Hemorrhage Grading
Scale.Journal of Neurosurgery,68(6),985-0.
6. Fisher,C. M.,l(jstler,J.P.,& Davis,J.M. (198o).Relationof cerebral vasospasm10 subarachnoid hemorrhage visualizedby computerized
tomographic scanning. Neurosurgery,6(1),1--9.
7. Fisher, R. S., Acevedo,C., Arzimanoglou,A., Bogacz,A., Cross, J, H., Elger,C. E, ... Wiebe, S. (204). IIAE OfficialRepor<A practical
clinjcaJdefinitionof epilepsy.Epilepsia,55(4),475-482.
8. Fisher,RS., Cross,J.H., O'Souza,C., French,J.A., Haut, S. R., Higurashi,N.,... Zuberi, S. M. (2017).Instruction manual for the ILA£2017
operational classificationof seizure types. Epilepsia,58(4),531--542.
9.Glauser,T., Ben-Menachem, E., Bourgeois,B.,Cnaan, A., Guerre~ C., Kalviainen,R., ... Tomson, T. (2013).Upda1edHAE evidence
reviewof anticpilepticdnig efficacyand effectivenessas initial monotherapy for epileptic seizuresand syndromes. Epilepsia,54(3)
10. Hemphill Ill JC, Greenberg SM, Anderson CS, el al. Guidelines for the Management of Spontaneous lmracerebral Hemorrhage:A
Guideline for Heallhcare Professionalsfrom the American Heart Association/AmericanStroke Association.2015~6:2032-2o6o.
11. Hunt, WE., & Hess, RM. (1968).Surgical risk as related to time of intervention in the repair of intracranial aneurysms. Journal of
Neurosurgery,28(1),4-20.
12. Jameson JI...,FauciAS, Kasper DL, Hauser SL, Longo DL,and LoscalzoJ (editors}.HarrisonSPrinciplesof Internal Medicine,20h ed.
McGrawHill Education, 2018.
13· G=~~~t:~e=s:~~~=ti~n~~~~~::-lh~.troke and Transient
4 Kothari RU,BronT, BroderickJP,Barsan WG, Sauerbeck LR,ZuccareUoM, et al. The ABCsof measuring intracerebral hemorrhage
volumes.Stroke.1996;27(8):1305-5
15. Kwan,P.,& Brodie,M. J.(2004).Drug treatment of epilepsy:when does i1failand how cooptimize its use?CNS Spectrums, 9(2),llo-<J.
16. Mathe\\\ J. M., Rajshekhar, V.,& Chandy, M. J.(1998).Shunt surgery in poor grade patients wi1h rubercuJous meningitis and
hydrocephalus:effectsof response to e.xtemalventriculardrainage and olher variableson long term oUlcome.Journal of Neurology,
Neurosurgery& Psychiatry,65(1),115-118.
17. Nogueira,R G.,Jadhav,A P.,Haus.sen,D. C.,Bonafe,A, Budzik.R F.,Bhuva,P.,... Jovin,T.G. (2017).
Thrombec1omy6 to 24Hours after
Stroke with a Mismatch between Deficitand Infarct New England Journal of Medicine,NEJMoa1700442.
18. Ona DID, Jimeno CA, Jasul GV Jr, et al. Executive summary of the 2020 clinical praaice guidelines for 1he managemem of
19. ~'.t
~rut;_~j~~~:~~~ith~~eJ;!
long-tem1follow-upstudy.Journal of Neurosurgery,74(1),64-9.
surgery for hydrocephalus in tuberculous meningitis:a

20. Philippine NeurologicAssociationEpilepsyCouncil. (2010).Recommendations for the Treatment of Status Epilepticusin Children
More Than 2 Months Old and in Adults.
21. Prasad, K, Singh, M. B., & Ryan, H. (2016).Corticosteroids for managing tuberculous meningitis. In K Prasad (Ed.), Cochrane
Database of SystematicRC\'lcws.Chichester, UK;John Wiley& Sons, Ltd.
22. Powers,W.J.,Rabinstein,A A.,Ackerson,T.,Adeoye,0. M.,Bambalcidis,N.C.,Becker,K, ... TirschweU,0. L (2019).2019Guidelinesfor
the EarlyManagement of PatientsWith Acute lschemic Stroke:A GuideHnefor Heallhcare ProfessionalsFrom the American Hean
Association/AmericanStroke Association.Stroke.2019;50:e344-e,i18.
23. Rajshekhar,V.(2009).Management of hydrocephalusin patienlSwith tuberculous meningitis.NeurologyIndia, 57(4),368.
24 Ropper,A.,Samuels, M.,& Klein,J.(204). Adams and Vidor's Principlesof Neurology(101hed.).McGrawHill Education.
25. Sadock, B.).,Sadock, V.A., & Ruiz,P.(2015).Kaplan & Sadock'sSynopsis of Psychiatty:BehavioralSciences/ClinicalPsychiatry(111h
ed.), WoltersKluwer.
26. The National lnstinue of NeurologicalDisordersand Stroke rt-PAStroke Study Group. (199;).Tissue plasminogen activatorfor acute
ischemicstroke.The New England Journal of Medicine,333(24).
27. The Stroke Societyof the Philippines.(204). SSP Handbook of Stroke:Guidelines for Prevention,Treatment, and Rehabilitation(6lh
ed.).Golden PagesPublishingCompany.
28. Tunkel,A. R.,Hartman, B.J, Kaplan,S. L, Kau6nan, B.A.,Roos,KL, Scheid, W.M, & Whidey.R.J.(2004).PracticeGuidelines fonhe
Management of BacterialMeningitis.lDSA Guidelines,19129(Augus1), 1267-1284-
29. 1 ~
~ann DL, Tomaselli GF. Braumva.ld's Heart Disease:A Te.xtbookof Cardiovascular Medicine. ulh

612
· ONCOLOG
INTRODUCTION TO ONCOLOGY
1. Basic Concepts in Oncology
2. Epidemiology and Etiology of Cancer

0 CANCER SCREENING
1. Cancer Screening Guidelines
2. Commonly Used Tumor Markers

0 OVERVIEW OF THE MANAGEMENT OF CANCER


1. Detection of Cancer
2. Establishing a Cancer Diagnosis
3. Defining Extent and Prognosis of the Disease
4. Cancer Treatment
5. Common Side Effects of Cancer Treatment

0 ONCOLOGIC EMERGENCIES
1. Superior Vena Cava Syndrome
2. Venous Thromboembolism
3. Other Oncologic Emergencies
SECTION ONE
INTRODUCTION TO ONCOLOG¥
BASIC ONCOLOGY CONCEPTS
Oncology is a branch of science that focuses on prevention, diagnosis, & treatment of cancer
• Over the past few years, cancer survival has improved due to:
Improved identification and prevention to reduce exposure to risk factors (e.g.,
0

smoking, infectious agents, various substances)


Improved screening of some cancers
0

Improved treatment strategies


0

I. DEFINITION OF TERMS
TERM I DEFINITION
Oncogene • Genes that positively influence tumor formation
Tumor
• Genes that negatively impact tumor growth
suppressor gene
Gatekeeper gene • A subset of tumor suppressor genes that directly regulate tumor growth
• A subset of tumor suppressor genes that do not affect cell growth directly,
Caretaker gene
but rather control the ability of the cell to maintain genome integrity
Two-hit • Both copies of a tumor suppressor gene must be inactivated in order to
hypothesis cause cancer

Oncogene • Mechanisms include point mutations, DNA amplifications, and


activation chromosomal rearrangement
• A short repeated sequence of profound genetic instability that
Microsatellite
increases the rate of mutations in other genes

P53 • A tumor suppressor gene that serves as the "guardian of the genome"
• Repetitive nucleotide sequences at each end of a chromosome that
Telomere
protect it from deterioration
• A specialized DNA polymerase that adds telomere repeat segments
Telomerase to the ends of telomeric DNA; expressed in ~90% of spontaneously
immortalized cells, including human cancer cells

II. SOME TERMINOLOGIES


A. Benign Tumors
0 Tumors that remain localized to their site of origin

0 "-OMA": suffix used to connote a neoplastic process


• On its own, it usually refers to benign tumors except lymphoma, seminoma,
hepatoma, blastoma, melanoma (these are malignant neoplasms by convention)
• If coined with CARCIN- or SARC-, it refers to malignant neoplasms
TERM I DESCRIPTION
Fibroma • Benign tumor arising in fibrous tissue
Chondroma • Benign cartilaginous tumor
Adenoma • Applied to benign tumors of glandular origin
Papilloma • Production of fingerlike or warty projections from epithelial surfaces
Cystadenoma • Formation oflarge cystic masses (such as in the ovary)

I
• Neoplasm (benign or malignant) producing a macroscopically visible
Polyp
projection above mucosal surface
615
B. Malignant Tumors
Tumors capable of invading adjacent tissues and spreading to distant sites
0

Amenability to surgical removal depends on the degree of tumor spread


0

TERM I DEFINITION I EXAMPLES


• Adenocarcinoma: adeno
Carcinoma • Malignant epithelial neoplasm
(gland-forming) + carcinoma

• Osteosarcoma: osteo (bone) +


Sarcoma • Malignant mesenchymal neoplasm
sarcoma

• Follicular lymphoma:
• Malignant lymphoid neoplasm that
Lymphoma Follicular (forming lymphoid
forms distinct tissue masses
follicle) + lymphoma
• Malignant hematolymphoid neoplasm • Acute myeloid leukemia:
that presents as proliferation of acute leukemia derived from
Leukemia
neoplastic cells in the bone marrow myeloid (bone marrow-
and/or peripheral blood derived) hematopoietic cells

III. HALLMARKS OF CANCER


HALLMARKS I DESCRIPTION
Sustaining • Fundamental trait of cancer cells which involvestheir ability to sustain
proliferative signaling chronic proliferation by deregulating growth-promoting signals
• Cancer cells avoid death by evading mechanisms that negatively
Evading growth
regulate cell proliferation (via inactivation of tumor suppressor
suppressors
genes like RB-associated and TP53 proteins)
• Cancer cells have the capability to inactivate or attenuate
Resisting cell death
apoptotic cell death programs, leading to their prolonged survival
• Like normal tissues, cancer cells get nutrients and oxygen via
Inducing angiogenesis
tumor-associated angiogenesis (new vessels are typically aberrant)
• Involves a multistep process that begins with local invasion,
Activating invasion then intravasation of cancer cells into nearby blood vessels
and metastasis and lymphatics, followed by extravasation into distant tissues,
micrometastasis, and finally formation of macroscopic tumors
• Unlike normal cells which have limited cell growth-and-division
Enabling replicative
cycles, cancer cells can enter a state of immortalization through
immortality
the action of telomerases
Deregulating cellular • Cancer cells not only control proliferation but can also reprogram
energetics energy metabolisn1 to fuel its growth and division

Avoiding immune • Cancer cells can avoid detection by the immune system, thereby
destruction evading eradication

• Acquisition of the abovementioned hallmarks are mainly due to


Genome instability
the different genomic alterations in neoplastic cells (including
and mutation•
chromosomal rearrangements)

• Involves the inflammatory state of premalignant and malignant


Tumor-promoting
lesions, which is due to a variety of cells of both the adaptive and
inflammation•
innate immune systems
'Enablingcharacteristics:
these2 characteristics
facilitateacquisition
of the previous8 hallmarksof cancer
Source:DeVita,et al. Principles
andPracticeof Oncology,
11thed.2018

616
EPIDEMIOLOGY & ETIOLOGY OF CANCER
Important information is obtained from the routine history and examination, including
duration of symptoms, past medical/family/social history, and the review of systems
Most significant risk factor for cancer overall is age

I. CANCER AROUND THE WORLD


CANCER TYPE I EPIDEMIOLOGIC DESCRIPTION
• Most common cancer and the most common cause of cancer
Lung cancer
death in the world
Breast cancer • Second most common cancer worldwide
Cancer of the lung,
breast, prostate and • Common cancers in developed countries
colorectum
Cancer of the liver,
• Common cancers in less developed countries
cervix and esophagus
Stomach cancer • Similar incidence in both more- & less-developed countries
Source:JamesonJL, et al. Harrison'sPrinciplesof InternalMedicine20thedition,2018

II. ONCOGENIC PATHOGENS


PATHOGEN I CANCER SITES/TYPES
Epstein-Barr virus • Burkitt and nasal T-cell lymphoma
(EBY, HHV-4) • Nasopharyngeal carcinoma
• Gastric carcinoma
Helicobacterpylori
• Gastric MALT lymphoma
Hepatitis B virus
(HBV) • Hepatocellular carcinoma

Hepatitis C virus • Hepatocellular carcinoma


(HCV) • Marginal zone lymphoma of the spleen
High-risk
• Carcinoma of the cervix, penis, anus, vagina, vulva, tonsils,
papillomavirus
base of the tongue, oropharynx
(e.g., HPV-16)
Human • Non-Hodgkin's lymphoma
immunodeficiency • Kaposi sarcoma
virus(HIV) • Squamous cell carcinoma of the urogenital tract
Human T-cell
leukemia virus • Adult T-cell leukemia/lymphoma
(HTLV-1)
Kaposi sarcoma
• Kaposi sarcoma
herpesvirus
• Multicentric Castleman disease
(KSHV, HHV-8)

Merkel cell
polyomavirus • Merkel cell carcinoma
(MCPyV, MCV)
Schistosoma • Squamous cell carcinoma of the bladder

I
Sources:DeVita,et al. Principles
andPracticeof Oncology,11thedition,2018
JamesonJL.et al. Harrison's Principles
of InternalMedicine
20thedition,2018

617
III. CHEMICAL CARCINOGENS
AGENTS I CANCER SITES/TYPES
Aflatoxin, vinyl chloride, tobacco • Hepatocellular carcinoma
smoke, alcoholic beverages • Hemangiosarcoma

Alcohol, tobacco smoking • Colon adenocarcinoma


Androgens, cadmium • Prostate carcinoma
Arsenic, benzo(a)pyrene, coal tar and
pitch, mineral oils, soot, cyclosporin • Squamous and basal cell carcinoma of the skin
A, azathioprine, shale oils

Asbestos, erionite • Pleural mesothelioma, lung cancer

Benzene, tobacco smoke, ethylene


oxide, antineoplastic agents, • Leukemia, lymphoma
cyclosporin A, formaldehyde

Diethylstilbestrol (prenatal} • Clear cell vaginal carcinoma

Estrogens • Endometrial, breast and liver carcinoma

Ethyl alcohol • Breast, liver, esophageal, head & neck carcinoma

Phenacetin • Bladder and renal pelvis cancer

Tobacco smoking (including


• Cancer of the upper aerodigestive tract, bladder
smokeless}

Tobacco smoke, 4-aminobiphenyl,


benzidine, 2-napthylamine, , Transitional cell carcinoma of the bladder
cyclophosphamide, phenacetin
Tobacco smoke, arsenic1 asbestos,
crystalline sicca, beryllium, coal • Squamous, large cell, small cell, and
tar and pitch, diesel exhaust, nickel adenocarcinoma of the lung
compounds, soot, mustard gas

Tobacco smoke, alcoholic beverages,


• Squamous cell carcinoma of the esophagus
betel quid

Tobacco smoke, alcoholic beverages,


• Squamous cell carcinoma of the oral cavity
nickel compounds, betel quid

Tobacco smoke, trichloroethylene • Renal cell carcinoma


Sources:DeVita,et al. Principlesand Practiceof Oncology,11thedition,2018
JamesonJL, et al. Harrison'sPrinciplesof InternalMedicine20thedition,2018

IV. PHYSICAL FACTORS

I
PHYSICAL
CANCER RISKS
FACTORS
• Leukemia
Ionizing radiation
• Carcinoma of the thyroid, breast, ovary, bladder, lung and colon

• Squamous and basal cell carcinoma of the skin


Ultraviolet sunlight
• Melanoma

Radiofrequency/
• Equivocal evidence on brain tumors (e.g., acoustic neuroma, glioma)
microwave radiation

• Carcinoma of the breast, colon, endometrium, esophagus, kidney


Obesity
(renal cell), pancreas
Source:DeVita,et al. Principles
andPracticeof Oncology,
11thedition,2018
618
SECTION TWO
CANCER SCREENING
CANCER SCREENING GUIDELINES

TESTOR
PROCEDURE

Breast Cancer
• Annually for as long as the woman is
in good health'
• Women 40-54 years old • Individualize decision to start
screening mammography prior to
Mammography age 50 years o]db

• Women ~55 years old • Every 2 years'

• Women ~75 years old • No sufficient evidenceb

• Women with -20-25% lifetime • Screen annually with MRI plus


Magnetic
resonance risk' for breast cancer mammography'
imaging • Women with 15-20% lifetime • Discuss option of MRI plus
(MRI)
risk' for breast cancer mammography annually'
Cervical Cancer
• Women 21-65 years old • Every 3 years'·b

Pap test • Women after total hysterectomy


(cytology) for noncancerous causes
• Do not screena,b
• Women >65 years old following
prior negative screening

• Screen (with Pap test) every 5 years


• Women 30-65 years old as preferred approach' if woman
desires to lengthen screening intervalb
HPVtest • Women after total hysterectomy
for noncancerous causes
• Do not screen 11·b
• Women >65 years old following
prior negative screening

Ovarian and Prostate Cancer ~:

• Women at high risk of • Offered with transvaginal ultrasound


ovarian cancer and/or who & pelvic exam for women at high
CA-125
have unexplained, persistent risk of ovarian cancer &/or have
symptoms unexplained, persistent symptoms'

• Discuss risks and benefits


• Men ~50 years old
Prostate • May be performed with or without a
• Men ~45 years old with first-
specific rectal exama
degree relative with prostate
antigen (PSA) • Frequency of determination will
cancer
depend on their PSA level
I

Skin Cance1,:

Skin exam by • Self-exam monthly


clinician or
• Clinical exam as part of routine cancer-related check-up'
patient

I
*Riskdefinedby BRCAPR0or usingtools likethe ModifiedGailModel(bcrisktool.cancer.govlcalc.ulator.html)
619
• Every 5 years'
Sigmoidoscopy • Every 5 years; improved benefit if performed every
IO years in combination with annual FITb

FOBT&FIP • Average risk • Fecal occult blood test (FOBT): annually'·b


adu Its 45 _75 years 1-•_F_e_c_al_i_m_,_n_u_n_o_c_h_e_m_i_ca_l_t_e_st_(:...F_IT.....:..):_a_
___ --1
Colonosco old** • Every 10 ears 3 •6
• Every 3 years, per manufacturer's recommendation'
Fecal DNA test
• Every I to 3 yearsb
• Every 5 years•·b

• Annual screening in adults' who:


• Adults 55-74' 0 Received evidence-based smoking cessation
(50-80)" years old, counseling, if they are current smokers; and
0 Made informed/shared decision, including
~30 pack-year
Low-dose CT (20 pack-year)•
potential benefits, limitations, & harms of
screening with low-dose CT; and
scan smoking history, 0 Have access to a high-volume, high-quality lung
still smoking or cancer screening and treatment center
have quit within • Stop screening once a person has noc smoked for
past 15years 15years or has a health problem that limits life
ex ectancy or ability to have luna surgeryb
• AmericanCancerSociety(ACS)Recommendation
b US Preventive
ServicesTaskForce(USPSTF)
Recommendation
'No evidencethatoneis betterthantheother,butFITis preferredsinceit doesnothaveanydietary
restrictions
andis betterin detectingadenomas
"USPSTFloweredagefor colorectalcancer(CRC)screeningto 45 yearsold. "Averagerisk"means:
No priordiagnosisof CRC,adenomatous polypsor inflammatory
boweldisease
No personaldiagnosisor familyhistoryof knowngeneticdisordersthatpredispose themto a highlifetime
riskof CRC(suchas LynchSyndromeor familialadenomatous polyps)
No personalhistoryof receivingradiationto theabdomenor pelvicareato treata priorcancer
Source:SmithRA,etal.Review
ofAmerican
CancerSocietyGuidelines;
2019
USPreventive
Services
TaskForceA andB Recommendations.
USPreventive
ServicesTaskForce;2021

COMMONLY USED TUMOR MARKERS


Tumor markers are found in the actual tumor themselves, typically in a sample of the
tumor removed during biopsy
At the most, these are just adjuncts to diagnosis
Potential uses of tumor markers:
0 Risk categorization
Screening for early diagnosis and treatment
0 Differential diagnosis
0 Prognosis and/or prediction
0 Monitoring of patients
TUMOR NEOPLASTIC I NON-NEOPLASTIC
MARKER
Hormones
I CONDITIONS I USE
CONDITIONS

"
Human chorionic • Gestational trophoblastic • Treatment
gonadotropin disease response &
(hCG) • Gonadal germ cell tumors recurrence
• Pregnancy
, Medullary thyroid • Diagnosis &
Calcitonin
carcinoma (CA) disease response
Catecholamines • Pheochromocytoma • Diagnosis
620
TUMOR NEOPLASTIC I NON-NEOPLASTIC
MARKER I CONDITIONS I USE
CONDITIONS

Oncofetal Antigens -
• Hepatocellular • Monitor treatment
Alpha•
carcinoma response (male • Cirrhosis
fetoprotein
• Gonadal germ cell gonadal germ cell • Hepatitis
(AFP)
tumors tumors)

• Pancreatitis
• Adenocarcinomas of • Monitor treatment
Carcinoembryonic • Hepatitis
the colon, pancreas, response (colorectal
antigen (CEA) • Inflammatory
lung, breast, ovary cancer)
bowel disease
-
Enzymes
• Monitor & assess • Prostatitis
Prostatic acid
• Prostate carcinoma progression (largely • Prostatic
phosphatase
replaced by PSA) hypertrophy
Neuron-specific • Small cell lung CA • Diagnosis & monitor ...
enolase (NSE) • Neuroblastoma treatment response

• Prognosis (lymphoma
Lactate • Lymphoma • Hepatitis
& Ewing's sarcoma)
dehydrogenase • Ewing's sarcoma • Hemolytic anemia
• Diagnosis
(LDH) • Dysgerminoma • Pleural effusions
(dysgerminoma)

Tumor-Associated Proteins
• Prostatitis
Prostate specific • Diagnosis & disease
• Prostate carcinoma • Prostatic
antigen (PSA) response
hypertrophy
• Infection
• Monoclonal
Monoclonal
gammopathy
immuno- • Myeloma • Diagnosis
of uncertain
globulins
significance
(MGUS)

• Treatment response • Menstruation


• Ovarian carcinoma
CA-125 and recurrence • Peritonitis
• Some lymphomas
(ovarian carcinoma) • Pregnancy
• Colon, pancreatic, • Treatment response • Pancreatitis
CA19-9 (pancreatic CA) • Ulcerative colitis
prostate, & breast CA
• Benign breast
disease, lactation,
CA15-3and • Monitoring response pregnancy
• Breast carcinoma
CA27-29 to systemic treatment • Pelvic
inflammatory
disease

• Hodgkin's lymphoma
• Diagnosis & choice of ...
CD30 • Anaplastic large cell
targeted treatment
lymphoma
• Hairy cell leukemia • Diagnosis
CD25 • Adult T-cell • Occasionally to assess ...
leukemia/lymphoma progression
11thedition,2018.1·
andPracticeof Oncology,
Source:DeVita,et al. Principles
621 .
SECTION THREE
OVERVIEW OF' THE MANAGEMENT OE CANCER
DETECTION OF CANCER
• Careful history and physical examination
Use of imaging modalities like: plain x-ray, CT scan, ultrasound, positron emission
tomography (PET), nuclear magnetic resonance or direct visualization via endoscopy
HISTORY & PE
I IMPORTANT INFORMATION RELATED TO CANCER

Symptom duration • Chronicity of the disease

Past medical • Presence of underlying disease that may affect the choice of therapy or
history the side effects of cancer treatment

Social history • Occupational exposure to carcinogens or habits

• Underlying familial cancer predisposition & point out the need to begin
Family history
surveillance or other preventive therapy for unaffected siblings

Review of systems • Suggest early symptoms of metastatic disease or a paraneoplastic syndrome

ESTABLISHING A CANCER DIAGNOSIS


Diagnosis of cancer relies most heavily on invasive tissue biopsy and should never be
made without obtaining tissue
Adequate tissue sample is important for light microscopy & determination of genetic
abnormalities & protein expression patterns, which provide information on prognosis &
response to treatment

CANCER BIOMARKER TESTING &


TYPE
I SPECIAL STUDIES I IMPLICATIONS ON TREATMENT

Breast cancer • If ER, PR positive: give hormonal treatment


• Hormone receptors: estrogen
(bothfor primary (Tamoxifen, Anastrozole, Letrozole, Fulvestrant)
(ER), progesterone (PR)
&suspected • IfHer2/neu positive:give anti-Her2/neu treatment
• Her2/neu oncoprotein
metastatic) (Trastuzumab, Pertuzumab, Lapatinib)

• If nonsquamous, non-small
• IfEGFR mutation positive: give tyrosine
cell pathology (advanced or
kinase inhibitors (Osimertinib, Erlotinib,
metastatic disease):
Gefitinib, Afatinib or Dacomitinib)
Lung cancer • Epidermal growth factor
• If ALK rearrangement positive: give Alectinib,
(bothfor (EGFR) mutation
Brigatinib, Lorlatinib or Crizotinib
primary and •ALK
• If PD-Lt expression positive and negative for
suspected • Programmed cell death
other actionable mutations (e.g., EGFR, ALK,
metastatic) ligand-I (PD-LI)
ROS1):give immunotherapy (Pembrolizumab)
• Others: ROSI, BRAF,
• lfROS1 rearrangement positive: give
NTRK1/2/3,METex14
Crizotinib or Entrectinib
skipping, RET

Colon cancer • If without Ki-ras mutation (wild type):


(suspected • Ki-ras mutation give monoclonal antibody (Cetuximab or
metastatic) Panitumumab)

GlstromaI • If c-kit positive: give molecular inhibitor


• c-kit oncoprotein mutation
tumor (GIST) (lmatinib)

• If CD20 positive: give monoclonal antibody


(Rituximab)
• Immunohistochemistry:
Lymphoma • If CD30 positive: give antibody-drug
CD20,CD30
conjugate in the relapsed or refractory setting
(Brentuximab vedotin)

622
DEFINING EXTENT AND PROGNOSIS OF THE DISEASE
I.STAGING
Determining the extent of disease is a primary priority in appropriate patient
management, which has implications in the type of treatment that will be given
Staging is the process of determining the extent of the disease through a variety of
diagnostic tests and procedures

A. Two Types of Staging


° Clinical staging: based on examination, radiographs, CT scans, and other imaging
modalities
0 Pathologic staging: takes into account information from a surgical procedure

B. Tumor, Node, Metastasis (TNM) Staging


0Most widely used system of staging, which is an anatomically based system
° Categorizes the tumor on the basis of:
• Size of the primary tumor lesion (T1-T4)
• Presence of nodal involvement (No and N1)
• Presence of metastatic disease (Mo and M1)

II. PHYSIOLOGIC RESERVE OF PATIENT


Second major determinant of treatment outcome
Determinant of how a patient is likely to cope with the physiologic stresses
Karnofsky performance status <70 or ECOG performance status ~3 have a poor
prognosis, unless the poor performance is a reversible consequence of the tumor

A. Karnofsky Performance
PERFORMANCE
STATUS
I FUNCTIONAL CAPABILITY OF PATIENT

100 • Normal/no complaints


• No evidence of disease

90 • Able to carry on normal activity


• Minor signs or symptoms of disease

80 • Normal activity with effort


• Some signs or symptoms of disease

70 • Cares for self


• Unable to carry on normal activity or do active work
60 • Requires occasional assistance but is able to care for most needs
50 • Requires considerable assistance and frequent medical care

40 • Disabled
• Requires special care and assistance

30 • Severely disabled
• Hospitalization is indicated, although death is not imminent
• Very sick
20 • Hospitalization is necessary
• Active supportive treatment is necessary

10 • Moribund
• Fatal processes progressing rapidly
0 , Dead
199
Source:AltillioT,et al. OxfordUniversityPress;
KarnofskyD,et al. ColumbiaUniversityPress;1949 •
623
~,
BE

0
C .
• Fully active
0 a G .(ECOG)P

PERFORMANCE
£ s

• Able to carry on all pre-disease performance without restriction

• Restricted in physically strenuous activity but ambulatory & able to carry


I
out work of a light or sedentary nature (e.g., light housework, office work)

• Ambulatory & capable of all self-care but unable to carry out any work activities
2
• Up and about >50% of waking hours

• Capable of only limited self-care


3 • Confined to bed or chair >50% of waking hours
• Completely disabled
4 • Cannot carry on any self-care
• Totally confined to bed or chair

5 • Dead
Source:MMOkenet al:AmJ ClinOncol.1982

CANCER TREATMENT
CANCER
TREATMENT I DESCRIPTION

L·oca ize dC ancer l7r~aiments


• Can cure at least 40% of cancer patients (early stage); but
unfortunately, ~60% of patients present with metastatic disease
Surgery
• Curative intent surgery: complete excision of tumor with adequate
margin of normal tissue

• Use of ionizing radiation that causes breaks in DNA of cancer cells


Radiotherapy • Bone~ among the most radioresistant organs
• Testis, ovary & bone marrow - most radiosensitive organs

• Stent placement using endoscopic techniques - for GI or biliary


obstructions
Others
• Radiofrequency ablation (RFA) for control of metastatic lesions in liver
• Cryosurgery - for very early-stage prostate and kidney cancer
... 0,,
Systemic Cancer Treq,t7!!ents ,,.
• Agents that target the DNA structure or the segregation of cancer
Chemotherapy
DNA as chromosomes in mitosis

Targeted • Molecules that interact with a distinct target that is significant in


treatment maintaining the malignant state of tumor cells or cells that they express

• Agents that manipulate the host-tumor interaction in favor of the host


Cancer biologic
• Distinguished from targeted treatment since it requires an active
therapy
response on the part of the tumor to achieve a therapeutic effect

• Agents that leverage on the host's own specific & potent immune
Immunotherapy system against cancer leading to durable clinical tumor regression
and/or response
Source:DeVita,et al. Principles
andPracticeof Oncology,
11thedition,2018

624
COMMON SIDE EFFECTS OF CANCER TREATMENT

SIDE
EFFECTS
I , MANAGEMENT

• Neutropenia:
0Granulocyte colony-stimulating factor (GCSF): 5 mg/kg per day SC
0Granulocyte-macrophage colony-stimulating factor (GM-CSF):
250 mg/kg per day SC
Myelo-
0Pegfilgastrim: one dose of 6 mg 24 hours after chemotherapy
suppression
• Anemia:
0Transfusion if hemoglobin falls to <80 g/L, compromise of end organ
function occurs, or an underlying condition (e.g., coronary artery
disease) calls for hemoglobin >90 g/L
• Neurokinin 1 (NK1)receptor antagonist (RA): aprepitant 125mg PO once
• Serotonin (5HT3) receptor antagonist (RA):
0 Granisetron 10 mg SC once or 2 mg PO once
0 Ondansetron 16-24mg PO once or 8-16 mg IV once
Nausea and
0 Palonosetron 0.25 mg IV once
vomiting
• Combination of NK1 RA and serotonin RA: Netupitant 300 mg/
Palonosetron 0.5 mg PO once
• Olanzapine 5-10 mg PO once
• Dexamethasone 12mg PO/IV once
• Maintain hydration and electrolyte repletion
• Antimotility: Loperamide 4 mg PO, then 2 mg every 2 h until 12h
Diarrhea
without loose stools (not to exceed a total daily dose of 16mg)
• If not responding to loperamide: may give Octreotide
• Topical therapies: anesthetics and barrier-creating preparations (e.g.,
"Magic Mouthwash" - combination of several drugs like anesthetics,
Mucositis anti-inflammatory, antimicrobials and antacid barriers, which can be
compounded by drug stores)
• Palifermin or keratinocyte growth factor
• Psychological support
Alopecia • Cosmetic resources
• Use of"chemo caps" to cool down the head - still controversial

625
I
SECTION FOUR
ONCOLOGIC EMERGENCIES
SUPERIOR VENA CAVA SYNDROME (SVCS)
I. ETIOPATHOGENESIS
Clinical manifestation of superior vena caval obstruction with severe reduction in
venous return from the head, neck and upper extremities
Most common etiologies are lung cancer, lymphoma and metastatic tumors
0 Lung cancer (small & squamous cell) accounts for 85% of all cases of malignant origin
0 Malignant lymphoma is the leading cause of SVCS in young adults
Other benign causes: aortic aneurysms, thyromegaly, thrombosis, fibrosing
mediastinitis, histoplasmosis or Behcet's syndrome

II. MANIFESTATIONS
Usually present with neck and facial swelling (especially around the eyes),dyspnea and cough
Others: hoarseness, tongue swelling, headache, nasal congestion, epistaxis, hemoptysis,
dysphagia, pain, dizziness, syncope, lethargy (aggravated by bending forward/lying down)
PE findings: dilated neck veins, increased number of collateral veins over the anterior
chest wall, cyanosis, and edema of the face, arms, and chest
More severe cases present with proptosis, glossal & laryngeal edema, obtundation &
signs of cerebral edema
Cardiorespiratory symptoms may occur at rest when significant airway and vascular
obstruction occur

III. DIAGNOSIS (SVCS is usually a clinical diagnosis)


DIAGNOSTIC I COMMENTS/EXPECTED FINDINGS
• Most significant finding is widening of the superior mediastinum
(more commonly on the right side)
Chest
• Pleural effusion occurs in 25% (often on the right side) majority are
radiography
exudative and occasionally chylous
• May be normal in some cases
• Provides the most reliable view of the mediastinal anatomy
Chest CT • Diminished or absent opacification of central venous structures with
prominent collateral venous circulation
Chest MRI • No advantages over CT
• Includes bronchoscopy, percutaneous core needle biopsy,
Invasive
procedures mediastinoscopy and thoracotomy
• Necessary for etiologic/histologic diagnosis

IV. MANAGEMENT
Upper airway obstruction demands emergent therapy
0 Diuretics with low-salt diet
0 Head elevation
0 Oxygen support
' Glucocorticoids for lymphoma masses (no benefit in lung cancer)
Radiation therapy is the primary treatment for SVCS caused by NSCLC and other
metastatic solid tumors
• Chemotherapy is effective when the underlying cancer is small cell lung ·cancer (SCLC),
lymphoma or germ cell tumor
Recurrent SVC may be palliated with use of intravascular self-expanding stents
(however, may precipitate heart failure and pulmonary edema)
Mortality does not relate to caval obstruction but rather to the underlying cause

626
VENOUS THROMBOEMBOLISM
I. ETIOPATHOGENESIS
Second leading cause of death in cancer patients
Risk of developing VTE in cancer patients is 20%
Pathogenesis of prothrombotic state in cancer involves:
0 Production of procoagulants by tumor cells
0 Suppression offibrinolytic activiry
0 Platelet activation

II. MANIFESTATIONS AND DIAGNOSIS


Highest risk among patients initially diagnosed with metastatic disease
Patients with advanced cancer of the stomach, pancreas, colon, kidney, brain and
lymphoma have the highest incidence
• Most common events are pulmonary embolism (PE) and deep venous thrombosis (DVT)

III. KHORANA RISK SCORE


• Validated in large cohorts of a variery of malignancies who are undergoing chemotherapy
RISK FACTOR I SCORE
Site of Primary Tumor:
• Very high risk (stomach, pancreas) 2
• High risk (lung, lymphoma, gynecologic, bladder, testicular) 1
• All other sites 0
Prechemotherapy platelet count ~350,000/uL 1
Hemoglobin level <IO g/dL or use of erythropoiesis-stimulating agents 1
Prechemotherapy WBC >n,ooo/uL 1
BMI ~35 kg/m' 1
Interpretation:
• Score 0: lowrisk for VTE
• Score 1-2: intermediaterisk for VTE
• Score 3-6: high riskfor VTE
Source:KhoranaAA.et al. Blood;2008

IV. THROMBOEMBOLISM PROPHYLAXIS & TREATMENT


Unfractionated heparin
Enoxaparin t preferred for patients without gastric or gastroesophageal lesions
Fondaparinux
Apixabant
Rivaroxabant

OTHER ONCOLOGIC EMERGENCIES


DIAGNOSIS I ASSOCIATED CANCERS I MANAGEMENT

I) Spinal Cord Compression (SCC)


• Most common site is the • Lung (most common) • Radiation therapy+
thoracic spine • Breast and prostate glucocorticoids: initial
• Most common initial (commonly involve treatment of choice
symptom is back pain multiple sites) • Surgery: resection of
• Minimum radiologic • Renal cell and anterior vertebral body
evidence is indentation genitourinary cancers along with tumor, followed
of the theca at the level of • Lymphoma by spinal stabilization
clinical features • Melanoma
• Earliest radiographic
evidence of vertebral tumor

I
is erosion of the pedicles
("winking owl")
627
DIAGNOSIS I ASSOCIATED CANCERS I MANAGEMENT
2) umor Lys1s syn tirome
11
• Syndrome consisting • Most commonly seen • Allopurinol or febuxostat
ofhyperuricemia, after treatment for • Urinary alkalinization
hyperkalemia, Burkitt's lymphoma, • Aggressive hydration
hypocalcemia, & acute lymphoblastic
hyperphosphatemia leukemia and other
• Caused by destruction rapidly proliferating
of rapidly proliferating lymphomas
neoplastic cells
• Usually occurs 1-5days
after chemotherapy

3) Febrile Neurr,ope~ia
• Fever: defined as single • -10-50% of solid tumors & • Determine patient risk
oral temperature >80% with hematological using the MASCC Scoring
measurement of~38-3°C or malignancies will develop Index
a temperature of~38-3°C fever associated with • Low risk patients:
sustained over a I-hour neutropenia ciprofloxacin + amoxicillin/
period clavulanate recommended
• Neutropenia: defined as for oral empirical treatment
an ANC of <500 cells/mm' (others: levofloxacin or
or an ANC expected to ciprofloxacin monotherapy,
decrease to <500 cells/mm' ciprofloxacin +
during the next 48 hours clindamycin)
• High risk patients:
monotherapy with an anti-
pseudomonal ~-lactam
agent (e.g., cefepime),
a carbapenem (e.g.,
meropenem or imipenem-
cilastatin), or piperacillin-
tazobactam
Sources:DeV1ta,
et al. Principlesand Practiceof Oncology,11thed1t1on, 2018
Freifeld,et al. ClinicalInfectiousDiseases.2011
Pulla,MP(editor).ESMOHandbookof OncologicalEmergencies, 2nded. ESMOPress,2016

REFERENCES
1. AhillioT, Otis-GreenS. Oxfordte.xtbookof palliativemedicine.Oxford UniversityPress: 1993
2. DeVitaVT, LawrenceTS, RosenbergSA (editors).DeVita, Hellman,and Rosenberg'sCancer:Principlesand Practiceof Oncology,
11thed.WoltersKluwer Health, 2018.
>FreifeldAG.BowEJ,SepkowiczKA, BoeckhMJ,ItoJI,et al Cliniatlpracticeguidelineforthe useof antimiaobiala,,•entsin neutropcnicF"cients
";th cancer.2010updatebythe InfectiousDiseasesSocietyofAmericaClinkalInfectious 20u:Volume52, Issue4, 15-Pageses6-E<J>
Diseases.
4. JamesonJL. KasperDL, Longo DL. Fauci AS, Hauser SL Loscalzo J.Harrison's Principles of Internal Medicine. 20th Edition. New
York:McGrawHill Education,2018.
5. Kam.orskyD, Burchenal J,The clinical evaluation or chemotherapeutic agents in cancer. In: Macleod C, ed. Evaluationor
ChemotherapeuticAgents.New York.NY:Columbia UniversityPress;1949:191-20;.
6. KhoranaAA, KudererNM, CulakovaE, et al. Development and validationof a predictivemodel for chemotherapy-associated
thrombosis.Blood.2008;111:4902.
7-NationalCompreheru.ivcCancer Netv.urk.
(Version 1.2021~Antiemesis.A¼lilableacww\\Q'lCD1.0rg/profossionalsJphysician_gls/p:Ulantiemesi
8.National ComprehensiveCancer Network.(Version1.2021).Cancer-AssociatedVenous Thromboembolic Disease. Availableat:
\w1w.nccn.org/professionals/physician_gls/pdf/vte.pdf
9. National Comprehensive Cancer Network. (Version 2.2021).Hematopoietic Growth Factors. Available at www.nccn.org/
professionals/physician__glsipdUgrowthfactors.pdf
10. NationalComprehen.siveC.ancer Net\\Ork.
(Version 3,2021~Breast:Cancer:Availableat:wwwncmoig/professionals/p~cian_g)s/pd.£lbrea.st.pdf
IL National Comprehensive CancerNet\\Ork.
(VeThion 42021). Non.SmallCellLungC.ancer:
Available
at \vw,\:Jlccnorglprofossionalslphysk:ian_gl.v
t2 Oken M,CreechR Tanney D, etal Toxicity and responseai1eriaofthe EasternCooperali\~OncologyGroup.Am J ClinOncol 1')8~5-"64~5-
13. Pulla, MP (editor).ESMO Handbook of Oncological Emergencies,2nd ed. ESMO Press,2016.
14 SmithRA.Andre"' KS,BrooksD, Fedes,~SA,Manassaram-Baptiste D, DoroshenkM,SaslowD,etal Cancerscreeningin the UnitedStates,
currentAmericanCancerSociety guidelines
201'1,A revie-,.vof CA:A CancerJournal
andcurrentisruesincancerscreening. forClinicians.
2019;
1;. US PreventiveServicesTaskForceA and B Recommendations.US PreventiveServicesTaskForce.2021March.Availableal:W\'"'~
uspreventiveservicestaskforce.org/espstflrecommendarion•ropicsluspstf-a-and-b-recommendations/
16. Van EsN, VentrescaM, Di Nisio M, e1 al. The Khoranascore for predictionor venous thromboembolismin cancer patiems:An
individualpatientdata meta-analysis.JThromb HaemosL2020;18:1940-1951.
628
" .

CORREL
- BOARD
BOARD CORRELATES
IM PLATINUM 4rn EDITION

PART 11CARDIOLOGY
Key Points About the Cardiac Cycle

PHASE EVENTS
1. Atrial Contraction • Preceded by P-wave, atrial pressure increases, a wave seen in venous pulse curve.
2. lsovolumetric • Begins during the QRS complex, c wave seen in atrial pressure curve
ventricular
contraction • Period between aortic valve opening and mitral valve closing

• Ventricular pressure reaches its maximum value during this phase. C wave on venous
3. Rapid Ventricular
pulse curve occurs because of bulging oftricuspid valve into right atrium during right
Ejection
ventricular contraction.
4. Reduced • Ejection of blood from the ventricle continues, but is slower. Ventricular pressure begins
Ventricular Ejection to decrease.
• Repolarization of the ventricles is now complete (end of the T wave). The aortic valve
5. lsovolumetric
closes, followed by closure of the pulmonic valve. Closure of the semilunar valves
ventricular
corresponds to the second heart sound. Period between aortic valve closing and mitral
relaxation
valve opening
6. Rapid Ventricular
• When ventricular pressure becomes less than atrial pressure, the mitral valve opens.
Filling
7. Reduced
• Is the longest phase ofthe cardiac cycle. Ventricular filling continues, but at a slower rate.
Ventricular Filling
Occurs before mitral valve opening
(diastasis)

Important Equations in Cardiac Physiology


Blood Pressure= Cardiac Output x Total Peripheral Resistance (TPR) = (Heart Rate x
Stroke Volume) x TPR
Blood Pressure
TPR is synonymous with Systemic Vascular Resistance and is determined by functional
and anatomic chanaes in small arteries (lumen diameter 100·400 um) and arterioles
Pulse Pressure= Systolic BP minus diastolic BP - a marker that correlates with stroke
Pulse Pressure volume
Mean Arterial Pressure MAP = 2h (Diastole J + 1/, [Systole J = Diastole + ½ Pulse Pressure
fMAPl
Capacitance Capacitance= Volume/Pressure
R = Bnljnr<
R = Resistance
Poiseuille Law n = viscosity of blood (determined by hematocrit}
I= length of blood vessel
r = radius of blood vessel
Jv = Kf[(Pc-Pi) - (Uc-ni)]
Jv = Fluid movement
Kf = Hydraulic Conductance
Pc= Capillary Hydrostatic Pressure
Starling Equation Pi = interstitial Hydrostatic Pressure
nc = Capillary Oncotic Pressure
ni = Interstitial Oncotic Pressure
Jfjv is positive, there is net fluid movement out of the capillary (filtration). Jfjv is
negative, there is net fluid movement into the capillary (absorption).

High-Yield Terms in the Physical Examination of the Cardiovascular System


• Firm and consistent pressure over the upper portion of the abdomen over the
right upper quadrant, for >15 s.
• Positive response: sustained rise of> 3 cm in JVPduring the application of firm
Abdominojugular Reflex abdominal pressure
• Assess response after 10 s of continuous pressure to allow for respiratory
artifacts and tensing of the abdominal muscles to subside.
• Blowing holosystolic murmur of Tri cuspid Regurgitation along the lower left
Carvallo Sign sternal margin which may be intensified during inspiration
• Heard along the left sternal border
• High-pitched, decrescendo diastolic murmur that can be difficult to distinguish
from the more frequently appreciated murmur of AR.
Graham Steell Murmur
• May become louder with inspiration
• Associated with a loud and sometimes palpable P2 and an RVlift, as would be
expected in patients with significant PA hypertension of any cause.
• Condition where the murmur of aortic stenosis may be transmitted downward
Gallavardin Effect and to the apex and may be confused with the systolic murmur of mitral
regurgitation
• A rapidly rising "water-hammer" pulse that collapses suddenly as arterial
Corrigan Pulse pressure falls rapidly during late systole and diastole, seen in aortic regurgitation
• Capillary pulsations manifest as alternate flushing and paling of the skin while
Quincke Pulse pressure is applied to the tip of the nail, seen in aortic regurgitation
• A booming "pistol-shot" sound heard over the femoral arteries, seen in aortic
Traube Sign regurgitation
• Normally there are only venous pulsations visible on the ocular fundus. In aortic
Becker Sign regurgitation, retinal arterial pulsations are visible
Muller Sign • Systolic pulsations of the uvula in aortic regurgitation
• To-and-fro murmur audible if the femoral artery is lightly compressed with a
Duroziez Sign stethoscope, seen in aortic regurgitation

Broadbent Sign • Apical pulse is reduced and may retract in systole in constrictive pericarditis
Diagnostic Procedures Used in Cardiology
Major noninvasive marker of increased CV morbidity/ mortality
Left Ventricular Hypertrophy (LVH) in ECG
risk
12-lead ECGbefore, during. and after
Most widely used test for the diagnosis of IHD
exercise, usually on a treadmill
The hallmark of myocardial ischemia during stress New regional wall motion abnormalities and
echocardiography reduced systolic wall thickening
Diagnostic test of choice for assessment of small lesions in the
Transesophageal echocardiography
heart such as valvular vegetations
Main method for clinical assessment of diastolic function Echocardiography
Left atrial size (because left atrial
Hemoglobin Ale of diastolic function enlargement reflects long-standing increase
in left-sided filling pressures)
Gold standard for assessing LV mass & volumes MRI
Assessment of ejection fraction (subtract
Main method to assess systolic function end-systolic volume from end-diastolic
volume and divide by end-diastolic volume)
Gold standard in assessing the anatomy & physiology of the heart Diagnostic cardiac catheterization and
& associated vasculature coronary angiography
Independent risk factor for IHD
Elevated level of high-sensitivity C-reactive
protein (CRP) (specifically, between Oand 3
May be useful in therapeutic decision-making about the initiation
mg/dL)
of hypolipidemic treatment.

High-Yield Triads in Cardiology


• Sinus tachycardia
Triad specific for pericardia) • Low QRSvoltages
effusion
• Electrical alternans (best seen in leads V3 and V4)
• Pro4lnged QRSduration (produced by slow conduction through direct
Diagnostic Triad of activation of ventricular myocardium over the accessory pathway)
Wolff-Parkinson-White
• Short P-R interval (<0.12 s)
(WPW) ECGPattern
• Slurring of the initial part of the QRScomplex (delta wave)
• Hypotension
Three principal features of
tamponade • Soft/ absent heart sounds
(Beck Triad) • Jugular venous distention with a prominent x-descent but an absent y-descent
• Heart rate
Major determinants of
myocardial 02 demand • Myocardial contractility
(MV02) • Myocardial wall tension (stress)
• Claudication of the affected extremity
Triad of Buerger disease • Raynaud phenomenon
• Migratory superficial vein thrombophlebitis
• Venous stasis
Virchow's Triad • Endothelial injury
• Hypercoagulability
CARDIACPHARMACOLOGY
Pharmacologic therapy for HFrEF
GOALS CAN BE ACHIEVED BY

• Diuretics
• Beta blockers (carvedilol, bisoprolol, and metoprolol succinate-agents
tested and proven to improve survival in clinical trials)
• RAASinhibitors (ACEis or ARBs)
• ARNI (LCZ696, an ARB (valsartan) with an endopeptidase inhibitor
Improvement in symptoms
(sacubitril, referred to as angiotensin receptor-neprilysin inhibitor and
denoted Entrezto)
• Hydralazine plus nitrate
• Digoxin
• Mineralocorticoid antagonists

• Beta blockers (carvedilol, bisoprolol, and metoprolol succinate-agents


tested and proven to improve survival in clinical trials)
• RAASinhibitors (ACEis or ARBs)
• ARNI (LCZ696, an ARB (valsartan) with an endopeptidase inhibitor
Prolongation of patient survival (sacubitril, referred to as angiotensin receptor-neprilysin inhibitor and
denoted Entrezto J
• Hydralazine plus nitrate
• Mineralocorticoid antagonists

Pharmacologic therapy for HFpEF


Treatment of HFpEF mainly consists of management of symptoms and associated conditions because of limited evidence
to support a specific pharmacologic regimen.
•· ':
Control of congestion Diuretics
Treat hypertension ARBs, ACE inhibitors, calcium channel blockers, and beta blockers
Anti-Arrhythmic Drugs
Mechanism of Actrnn Climcal Use
• Binds to activated sodium channels • Atrial fibrillation • Quinidine
and blocks the flow of sodium ions • Atrial flutter • Procainamide
IA into the cardiac myocyte • Ventricular tachycardia • Disopyramide
• (Prolongs action potential) • Mnemonic: Quiapo
Police De artment
• Binds to both activated and inactivated • Post•ischemic arrhythmia or • Lidocaine
sodium channels and blocks the flow post-Ml • Tocainide
of sodium ions in the cardiac myocyte • Ventricular fibrillation • Mexiletine
18 • (Shortens action potential) • Ventricular tachycardia • Mnemonic: Libby's
• Preferentially affects ischemic or Mexican Tacos
depolarized Purkinje and ventricular
tissues.
• Binds to activated sodium channels • Treatment of severe • Flecainide
and blocks the flow of sodium ions refractory ventricular • Encainide
IC
into the cardiac myocyte arrhythmia • Propafenone
• No effect on action otential
• Blocks beta-adrenergic receptors • SVT,ventricular rate control • Propranolol
II for atrial fibrillation and atrial • Metoprolol
flutter
• Binds to potassium channels and • Atrial and ventricular • Sotalol
blocks the flow of Kin the myocyte arrhythmias • lbutilide
Ill • (Prolongs action potential) • Dofetilide
• Amiodaronet
• Mnemonic: BIAS
• Blocks voltage-gated calcium channels • Supraventricular tachycardia • Verapamil
IV thereby blocking the flow of calcium • Rate reduction in patients • Diltiazem
into the cell with atrial fibrillation
1 Remember to check PFTs, LFTs,and TFTs when using amiodarone. Amiodarone is lipophilic and has class I, II, III,and IVeffects.

Drugs in Hypertension
Causes Na excretion and reduction in blood volume Diuretics

Calcium Channel Blocker that exerts more effect on the vessels Dihydropyridines
than the heart (Nifedipine, Felodipine, Amlodipine)

Nondihydropyridines
Calcium Channel Blocker that exerts more effect on the heart (Verapamil, Diltiazem)
than the vessels *Remember V for Verapamil is shaped like the
heart and has more effect on the heart

Decreases the work load of the heart Beta-blockers

Blocks the ATl receptor of angiotensin II ARBs

Notorious for drug-induced cough by increasing bradykinin ACEinhibitors

Blocks aldosterone action in the collecting tubules Spironolactone, Eplerenone

Hypertension with Benign Prostatic Hyperplasia (BPH) Alpha-1 antagonists (Prazosin)

Most commonly used to acutely manage severe hypertension in


IV labetalol or hydralazine
preeclampsia
CARDIOVASCULARDISEASES
High-Yield Concepts in Cardiac Dysrhythmias
P-Wave: atrial depolarization
Physiologic basis for normal ECGtracing QRS complex: ventricular depolarization
T Wave: ventricular renolarization
Phase 0: Depolarization (due to rapid Na• inUux)
Phase 1: Partial Repolarization (due to K• efflux)
Phases of the Cardiac Action Potential Phase 2: Plateau Phase (K• efflux being balanced by Ca2 • inUux)
Phase 3: Complete Repolarization (due to K• efflux)
Phase 4: Resting Membrane Potential (RMP)

Phase 4: Unstable RMP with gradual rise to threshold [due to


slow Na+ influx using h channels)
Phases of the SA Node Action Potential Phase 0: Depolarization (due to caz+ influx)
Phase 3: Repolarization [due to K• efflux) high K+ permeability
through K+ channels.

Master pacemaker of the heart Sinoatrial (SA) Node


Failure to increase heart rate during exercise, alternatively
defined as:
0 Unable to achieve 85% of predicted maximal heart rate at
peak exercise
Chronotropic Incompetence 0 Unable to achieve a heart rate >100 beats/min with
exercise
0 Maximal HR with exercise <2 standard deviations below
that of an age-matched control population
The only electrical connection between the
AV node
atria and ventricles
Most common arrhythmia mechanism Reentry
Most reliable treatment for patients with
symptomatic AVconduction system disease
Temporary or permanent artificial pacing
in the absence of extrinsic and reversible
etiolo2:ies
Most expeditious technique in the
Transcutaneous pacing
management of AVconduction block

Most common sustained arrhythmia Atrial Fibrillation

Has prolongation of PR interval before dropped QRS complex

'
Mobitz Type I .Mnl:.mlmil;: Think of the Roman Numeral I that gets taller-+
PR
prolongation in Mobitz I

V' Has no prolongation of PR interval before dropped QRS complex


Mobitz Type II .Mnl:.mlmil;: Think of the Roman Numeral II with equal heights
between the two letter no PR prolongation in Mobitz II
Non-sustained VT

Ventricular tachycardia that terminates Sustained VT persists for >30 s or is terminated by an active
spontaneously within 30 s intervention, such as administration of an intravenous
medication, external cardioversion, or pacing or a shock from an
implanted cardioverter defibrillator
Underlies the majority of sudden cardiac
Coronary heart disease
death
Abolishes ventricular ectopic activity in
patients with STEM!and in the prevention of Beta-adrenoceptor blocking agents
ventricular fibrillation
High-Yield Concepts in Heart Failure (HF)
Coronary Artery Disease (CAD) and
Cause of HF in men and women in industrialized countries responsible
Hypertension (contributes to the
for 60-75% of cases
development of HF in 75% ofpatientsl
Most useful index of LV function Ejection fraction
JVP (internal jugular vein is preferred
because the external jugular vein is valved
Single most important bedside measurement to estimate volume status
and not directly in line with the superior vena
cava and rie:ht atriuml
Cardinal symptoms of HF Fatigue and shortness of breath
Pulmonary congestion with accumulation of
Most important mechanism of dyspnea in HF interstitial or intra-alveolar fluid, which
activates iuxtacanillarv I receotors
Dyspnea occurring in the recumbent position

Results from redistribution of fluid from the splanchnic circulation and


lower extremities into the central circulation during recumbency, with a
Orthopnea
resultant increase in pulmonary capillary pressure

Relieved by sitting upright at the side of the bed with the legs in a
deoendent nosition
Acute and severe shortness of breath and coughing that generally occur
at night; awakens the patient from sleep (1-3 h after the patient retires)

Caused by increased pressure in the bronchial arteries leading to airway


compression, along with interstitial pulmonary edema that leads to Paroxysmal nocturnal dyspnea
increased airway resistance

Often have persistent coughing and wheezing even after they have
assumed the uorie:ht oosition
Cornerstone ofpharmacotherapy for HF with reduced EF RAAS blockers and Beta Blockers
LCZ696, an ARB (valsartan) with an
endopeptidase inhibitor (sacubitril)
HF drug that has shown a survival benefit in a large trial versus ARB
alone
Classified as an angiotensin receptor-
neprilysin inhibitor (ARNI) (Entresto)
Most common symptom of cor pulmonale Dyspnea

High-Yield Concepts in Valvular Heart Diseases


Murmurs that always signify structural heart disease Diastolic murmurs
Opening Snap followed by a low-pitched, rumbling, diastolic murmur,
heard best at the apex with the patient in the left lateral recumbent Mitral stenosis (MS)
oosition
Leadine: cause of MS Rheumatic (ever
Pansystolic murmur; may be due to Mitra! Valve Prolapse fMVP) Mitra! Re2ur2itation (MR)
Papillary muscle involved more frequently in acute MR (with acute Ml)
Posteromedial papillary muscle
because of sine:le blood suoolv
Most prominent complaints in chronic severe MR Fatigue, exertional dyspnea, and orthopnea
Freauent findine: on auscultation in MVP Mid- or late rnon-eiectionl svstolic click
Most common ECGfindine: in MVP Normal
Most common congenital heart valve defect Bicuspid Aortic Valve Disease
Syncope, Angina pectoris, Exertional D,yspnea
Three cardinal symptoms of AS
ISADl
IE In IV Drug Abusers, marked hepatomegaly with systolic pulsations,
ascites, pleural effusions, edema, and a positive hepatojugular reflux sign Tricuspid Regurgitation
, giant C-V Wave in )ue:ular Venous Pulses
Most common valvular heart disease in patients with Carcinoid heart
Tricuspid insufficiency
disease
High-Yield Concepts Cardiomyopathies and Myocarditis
Clinical Characteristics of Cardiomyopathies
DILATED RESTRICTIVE HYPERTROPHIC
CARDIOMYOPATHY CARDIOMYOPATHY CARDIOMYOPATHY
• LVenlargement resulting • Endomyocardial scarring • Disproportionate
in impaired systolic or myocardial infiltration hypertrophy, typically
Pathophysiology
function, heart failure, resulting in restriction of involving the IV septum
arrhvthmia, emboli ventricular filling more than the free wall
LV ejection fraction* • Usually <30% • 25-50% • >60%
LVdiastolic • <60 mm (may be
• Dilated ~60 mm • Often decreased
diameter** decreased)
LV wall thickness • Decreased • Normal or increased • Markedly increased
Atrial size • Increased • Increased; mav be massive • Increased
Valvular • Related to endocardial • Related to valve-septum
• Related to annular dilation
re1mrei.tation involvement interaction
Common first • Exertional intolerance, • Exertional intolerance;
• Exertional intolerance
svmotoms fluid retention earlv mav have chest oain
Congestive • Left-sided congestion may
• Left before right • Right often predominates
symptoms develoo late
• Most common abnormality
• Viral
• Amyloidosis found at autopsy in young
• Peripartum
Common examples • Loeffler's endocarditis competitive athletes who
• Alcohol, MAP, cocaine
• Endomyocardial fibrosis experience sudden cardiac
• Chemotherapy arrest (SCAl
'"Normal LVEF: >50%
0 Normal LV dimension: <SS mm

Young to middle-aged adult who develops


progressive dyspnea and weakness within a few days
Typical clinical picture ofmyocarditis
to weeks after a viral syndrome that was
accomoanied bv fever and mval2ias.
Third most common parasitic infection in the world and the
most common infective cause of cardiomyopathy; Chagas Disease
a cause of dilated cardiomyopathy
Bite of the reduviid bug
*Chagas disease, American trypanosomiasis, does not
How is Trypanosoma cruzi transmitted? cause sleeping sickness. African sleeping sickness
caused by Trypanosoma brucel
America never sleevs!
Last trimester or within the first 6 months after
Time frame of Peripartum Cardiomyopathy
pree;nancy
Most common toxin in chronic dilated cardiomvooathv Alcohol
Most common drugs implicated in toxic cardiomyopathy Chemotherapy
Apical ballooning syndrome, or stress-induced cardiomyopathy
Tako-tsubo Cardiomyopathy
Typically in older women after sudden intense emotional or a.k.a "Broken-Heart" Syndrome
ohvsical stress.
Most common cause of thyroid abnormalities in the
Treatment of tachyarrhythmias with amiodarone
cardiac oooulation
Main cause of sudden death in the young and important cause of
Hypertrophic Cardiomyopathy (HCM)
heart failure
Misaligned and disarrayed enlarged myofibrils and
Histologic changes associated with hypertrophic
myocytes; fibrosis and microvascular disease; and
cardiomyopathy
Interstitial fibrosis
Common first symptoms of HCM Exertional intolerance; may have chest pain
Classic finding on the echocardiogram of HCM Systolic anterior motion (SAM) of the mitral valve
First-line agents that reduce the severity of obstruction by
P-Adrenergic blocking agents and L-type calcium
slowing heart rate, enhancing diastolic filling. and decreasing
channel blockers ( e.g., verapamil)
contractilitv in HCM
High-Yield Concepts in Pericardia) Diseases
Differentials for Cardiac Tamponade
CARDIAC CONSTRICTIVE
CHARACTERISTIC RESTRICTIVE CMP RVMI
TAMPONADE PERICARDITIS

Clinical Features
Pulsus Paradoxus +++ + + +
Jugular Veins
Prominenty-descent - ++ + +
Prominentx-descent +++ ++ +++ +
Kussmaul's Sien• - +++ + +++
Third Heart Sound - + +
Pericardia) Knock - ++ - -

Electrocardio.Qram
Low ECG Voltai,:e ++ ++ + -
Electrical Alternans I ++ - - I -
Echocardioaraohv
Thickened Pericardium - +++ - -
Pericardia) Effusion +++ - - -
RV Size Usuallv small Usuallv normal Usuallv normal Enlare:ed
Exaggerated respiratory
variation of mitral and tricuspid +++ +++ - +++
inflow

CT /MRI
Thickened / Calcific Pericardium I - +++ - -

Cardiac Catheterization
Equalization of diastolic
+++ +++ - ++
pressure

CMP: cardiomyopathy: RVMI:right ventricular myocardial Infarction

•Kussmaul's sign: rise or a lack of fall of the JVP with inspiration, classically associated with constrictive pericarditis
(but also seen in restrictive cardiomyopathy, pulmonary embolism, RVinfarction, and advanced systolic heart failure)

Most common pathologic process involving the


Acute Pericarditis
oericardium
Worsened by lying supine, relieved by sitting up and
Characteristic pain in pericarditis
leanin2 forward
Pericardia) friction rub in acute pericarditis is heard
End-expiration with patient upright and leaning forward
most frequently at
• Stage 1: widespread elevation of the ST segments,
often with upward concavity, involving two or three
standard limb leads and V2-V6, with reciprocal
depressions only in aVR and sometimes Vt.
0 Depression of the PR segment below the TP seg-
ment, reflecting atrial involvement.
Four stages of evolution of the ECGin acute 0 No significant changes in QRS complexes, unless a
pericarditis large pericardia( effusion develops
• Stage 2: ST segments return to normal afer several
days
• Stage 3: T waves become inverted
• Stage 4: Weeks or months after the onset of acute
pericarditis, the ECGreturns to normal
• 1. Idiopathic pericarditis
• 2. Pericarditis secondary to neoplastic disease,
tuberculosis, or bleeding into the pericardial space
Most common causes of cardiac tamponade
after leakage from an aortic dissection, cardiac
operation, trauma, and treatment with
anticoagulants
Important clue to the presence of cardiac tamponade
consisting of a greater than normal (10 mmHg) Paradoxical Pulse (Pulsus Paradoxus)
insniratorv decline in svstolic arterial nressure
• Neoplasm
Most common causes of bloody pericardia) fluid • Renal failure
• After cardiac injury
• Extension or invasion of metastatic tumors
Most common causes of pericarditis due to 0 Carcinoma of Lung and Breast
neoplastic disease 0 Malignant Melanoma
0 Hematologic (Lymphoma, Leukemia)
• Neoplasm
Grossly sanguineous pericardia! fluid in chronic • Tuberculosis
pericarditis results most commonly from • Renal failure
• Slow leakage from an aortic aneurysm
Basic physiologic abnormality in chronic constrictive Inability of ventricles to fill because of limitations
pericarditis imposed by the rigid, thickened pericardium
Most prominent deflection in constrictive
y descent
nericarditis (absent/ diminished in tamnonade)
The only definitive treatment of constrictive
Pericardial Resection
oericarditis
High-Yield Concepts in Cardiac Tumors and Trauma
Most common primary sites from which cardiac metastases originate Carcinoma of the breast and lung

Malignant melanoma and to a lesser extent,


Cardiac metastases are especially high in
leukemia and lymphoma

Most common primary cardiac tumor in adults Myxomas (90% are sporadic)

Most common tumors of the cardiac valves Papillary Fibroelastomas

Most common cardiac tumors in infants and children Rhabdomyomas and fibromas

Sarcomas (commonly involving right side


Almost all primary cardiac malignancies are
of the heart)

Pericardium > Myocardium >


Most often involved in metastasis to the heart
Endocardium or Cardiac Valves

Central role in the diagnostic evaluation of cardiac metastases and


Cardiac MRI
cardiac tumors

High-Yield Concepts in Congenital Heart Diseases (CHD)


Ventricular Septal Defect (VSD)
Most common congenital anomaly recognized at birth
Most common location - membranous septum

Most common type or ASD i Secundum ASD

Mnemonic: All CHDs that start with "T":


Tetra logy of Fallo! (TOF),
0 Tricuspid Atresia,
Cyanotic CHDs (R-to-L shunt) 0 Truncus Arteriosus,
0 Total Anomalous Pulmonary Venous
Connection (TAPVC)
0 Transposition of the Great Arteries (TGA)

Most common form of cyanotic CHO Tetralogy of Fallo! (TOF)

Acyanotic CHDs (L to R shunt) ASD, VSD, PDA

Conversion of an initial L to R shunt into a R to L shunt Eisenmengerization

CHO associated with Congenital Rubella Syndrome;"continuous


machine-like murmur"; needs indomethacin to close and PGEl Patent Ductus Arteriosus (PDA)
to remain open

CHO associated with Turner Syndrome Coarctation of the Aorta (CoA)

Most common CHO associated with Trisomy 21 Defects in the atrioventicular canal

CHO associated with offspring of diabetic mother TGA

CXRshows boot-shaped heart (Coeur en Sabot); Components:


subpulmonic stenos is ( main determinant of severity), RVH, VSO, TOF
overriding of the aorta

CXRshows egg-shaped silhouette or egg-on-its-side appearance TGA


High-Yield Concepts in lschemic Heart Disease (IHD}
Risk factors for !HO Obesity, insulin resistance, and type 2 diabetes mellitus

The most common cause of myocardial ischemia Atherosclerotic disease of an epicardial coronary artery

Most common cause of nontraumatic chest discomfort Gastrointestinal disorders

Blood flow through the coronary arteries occur during Diastole

Major site of atherosclerotic disease Epicardial coronary arteries

Blood flow at rest may be reduced when a stenosis


~80%
reduces the diameter of an epicardial artery by

• Central obesity
• Hyperglycemia
Major features of metabolic syndrome • Hypertriglyceridemia
• Hypertension
• Low HDL cholesterol

First choice medication intervention to lower LDL


Statins
cholesterol in patients with metabolic syndrome

Key feature of the metabolic syndrome Central adiposity

Most accepted & unifying hypothesis to describe


Insulin resistance
pathophysiology of metabolic syndrome

Driving force behind the metabolic syndromee Obesity

Weight reduction (caloric restriction: most important


Primary approach to metabolic syndrome component, whereas increases in physical activity are
....;: important for maintenance of weight loss)
...., .,..,.,
Drug of choice to lower fasting TG Fibrates

Only currently available drug with predictable HDL-


Nicotinic acid
raising properties

Among patients with NSTE-ACSstudied at angiography,


Three-vessel disease
most have

Sites of predilection for atherosclerotic plaques to


Branch points in the epicardial arteries
develop due to increased turbulence

Time frame for reversible damage in myocardium .s,20 minutes for total occlusion in the absence of collaterals

Route of administration where absorption of nitrates is


Sublingual / through mucous membranes
most rapid and complete

Most common route in administration of nitroglycerin Sublingual

Most common etiology of coronary thrombosis Plaque rupture


Hypo tension or the recent use of a phosphodiesterase type 5
Only absolute contraindications to nitrate use (PDE·S) inhibitor, sildenafil or vardenafil (within 24 h), or
tadalafil (within 48 h).

Mostimportant adverse effectof all antithrombotic agents Excessive bleeding

Diagnostic hallmark of Prinzmetal variant angina Transient coronary spasm

Main therapeutic agents for Prinzmetal angina Nitrates & Calcium Channel Blockers

In Prinzmetal variant angina, this drug may increase


the severity of ischemic episodes, possibly as a result of
Aspirin
the sensitivity of coronary tone to modest changes in
the synthesis of prostacyclin

Pivotal diagnostic and triage tool for patients with


12-lead ECG
prolonged ischemic discomfort

Distinguishes UAfrom NSTEMI Serum cardiac biomarkers

Most common presenting complaint in STEM!patients Chest pain

May radiate as high as the occipital area but not below the
The pain of STEMI
umbilicus

Distinguishing feature that suggests pericarditis rather


Radiation of discomfort to the trapezius
than STEMI

The proportion of painless STEMls is greater in what


Patients with diabetes mellitus and in the elderly
patient population

Within the first hour of STEM!,about one•fourth of Manifestations of sympathetic nervous system hyperactivity
patients with anterior infarction have (tachycardia and/or hypertension)

Within the first hour of STEMI, up to one•half of Parasympathetic hyperactivity (bradycardia and/or
patients with inferior infarction have hypotension)

Fibrinous Pericarditis (bread & butter pericarditis)


Dressler Syndrome
post•MI

t 1 t time Ml patients (cardiac scar in those with previous Ml


Myocardial rupture post•Ml occurs in patients who are
prevents rupture)

Free wall > IVS > Papillary muscle


Sites of myocardial rupture in Ml
(decreasing order of frequency)

Preferred biochemical markers for Ml Cardiac-Specific Troponin T & Cardiac•Specific Troponin I

When the ECGis not diagnostic of STEM!,what Echocardiography showing early detection of the presence or
diagnostic test can aid in the management decision? absence of wall motion abnormalities

Atherosclerotic plaque disruption (exposing its contents to


Pathologic consequence of atherosclerotic plaque that
the blood) with conditions (local or systemic) that favor
leads to STEM!
thrombogenesis.
Primary cause of out-of-hospital deaths from STEM! Ventricular Fibrillation

Primary cause of in-hospital deaths from STEM! Pump Failure

The most common clinical signs of pump failure Pulmonary rales and S3 and S4 gallop sounds

Minimize the time from first medical contact to initiation of


reperf'usion therapy.
Overarching goal of STEM!management This may involve transfer from a non-PCI hospital to one that
is PCI capable, with a goal of initiating PCI within 120 min of
first medical contact (FMC-device time)

FMC-devicetime ifa STEMI patient who is a candidate


for reperfusion was initally seen at a PCl-capable FMC-device time less than or equal to 90 min
hospital

Drugs that should be avoided in patients with STEM!


because they can impair infarct healing and increase
the risk of myocardial rupture
Glucocorticoids and nonsteroidal anti-inflammatory agents
(with the exception of aspirin)
Their use may result in a larger infarct scar. In addition,
they can increase coronary vascular resistance, thereby
potentially reducing flow to ischemic myocardium.

Greatest delay usually occurs between Onset of pain and the patient's decision to call for help

Principal goal of fibrinolysis Prompt restoration of full coronary arterial patency

S30 min; fibrinolytic therapy should ideally be initiated


Door-to- needle time
within 30 min of presentation

Most frequent and potentially the most serious Hemorrhage (Hemorrhagic stroke: Most serious
complication offibrinolysis complication)

Standard antiplateletagent for STEM! y Aspirin

Standard anticoagulant agent for STEMI Unfractionated Heparin

Extent of LVinvolvement that usually results in


Infarction~ 40%
cardiogenic shock

Severe multivessel coronary artery disease with evidence of


Typically, patients who develop cardiogenic shock have Npiecemeal" necrosis extending outward from the original
infarct zone

3-5 days
• During the first 1-2 weeks the patient should be encouraged to
increase activity by walking about the house and outdoors in
Usual duration of hospitalization for an uncomplicated good weather
STEMI
• After 2 weeks, the physician must regulate the patient's activity
on the basis of exercise tolerance
• Most patients will be able to return to work within 2-4 weeks

Most common complication of angioplasty Restenosis, or re narrowing of the dilated coronary stenosis
High-Yield Concepts in Hypertension
Most common cause of death in hypertensive patients Heart disease
Second most frequent cause of death in the world Stroke
Primary mechanism for rapid buffering of acute fluctuations
Arterial baroreDex mediated by stretch-sensitive
of arterial pressure that may occur during postural changes,
sensory nerve endings in the carotid sinuses and
behavioral or physiologic stress, and changes in blood
the aortic arch.
volume
Most common cause of secondary hypertension Primary renal disease
Classic symptom of Peripheral Artery Disease (PAD) Intermittent Claudication
AB! cut off diagnostic of PAD and associated with >50%
ABI <0.90
stenosis in at least one major lower limb vessel
AB! cut off associated with elevated BP, particularly systolic
ABI <0.80
BP
Time of the day where myocardial infarction and stroke are
Early morning hours
more freauent
Gold standard for evaluation and identification of renal
Contrast Arteriography
artery lesions
Most common congenital cardiovascular cause of
CoA
hvoertension
• Attain and maintain BMI <25kg/m 2

• <6gNaCl/d
• Diet rich in fruits and vegetable and low-fat
dairy products with reduced content of
Lifestyle modifications to manage Hypertension saturated and total fat
• For those who drink alcohol, consume
s 2 drinks/din men ands 1 drink/din women
• Regular aerobic activity, e.g., brisk walking for
30min/d
Single most effective intervention for slowing the rate of
Hypertension control
oroe:ression of hvoertension-related CKD

High-Yield Concepts in Rheumatic Fever


Group A Beta-Hemolytic Strep (GABHS)
Other name for Streptococcuspyogenes
Group A is based on Lancefield classification

Infection that precedes RF Streptococcal pharyngitis

Tests to document history of antecedent GABHSinfection in Anti-streptolysin O (ASO) and anti-DNase B (ADB)
RF titers

Mechanism of damage in RF Type II hypersensitivity

Polyarthritis, Carditis, Subcutaneous Nodules,


Signs and symptoms of Rheumatic Fever
Erythema Marginatum, Syndenham Chorea
Most common clinical features Polyarthritis (60-75%) and carditis (50-60%)
Commonly occurs in the absence of other manifestations of
Sydenham Chorea
RF and is found mainlv in females
Hallmark of rheumatic carditis Valvular damage
Characteristic manifestation of carditis in previously
Mitra( regurgitation
unaffected individuals
Aschoff Bodies: granuloma with
Pathologic lesion in Rheumatic fever giant cells (Anitschkow cells): enlarged
macrophages with ovoid, wavy, rod-like nucleus
The classic rash of ARF Erythema marginatum
High-Yield Concepts in Aortic and Vascular Pathologies
Most common pathologic condition associated with
Atherosclerosis
degenerative aortic aneurysms

Location of 90% of syphilitic aneurysms Ascending aorta or aortic arch

Typical location of Tuberculous Aneurysms Thoracic Aorta

Aneurysms of the aortic arch and descending


Aneurysms associated with Takayasu's Arteritis
thoracic aorta
Most common pathology associated with ascending aortic
Medial degeneration
aneurysms
Most frequently associated with aneurysms of the
descending thoracic aorta.
Atherosclerosis
ChestX-Ray
First test that suggests the diagnosis of a thoracic aortic (findings: Widened Mediastinum and
aneurysm displacement or compression of the trachea or
left main stem bronchus)
Harbinger of rupture and represents a medical emergency Aneurysmal pain

Sudden onset of pain, very severe and tearing and


Description of pain of aortic dissection
is associated with diaphoresis

Usual location of aortic dissection Right lateral wall of the ascending aorta

Pathology ofTakayasu's Arteritis Panarteritis

Focal granulomatous lesions involving the entire


Pathology of Giant Cell Arteritis
arterial wall
Obliterative endarteritis of the vasa vasorum,
Initial lesion of Syphilitic Aortitis
especially in the adventitia
Buerger's Disease (Thromboangiitis Obliterans) has a
Cigarette smoking
definite relationship with
20-30 mmHg · suitable for most patients with
simple varicose veins
In chronic venous disease, graduated compression stockings
30-40 mmHg • may be required for patients with
are recommended with pressures of
manifestations of venous insufficiency such as
edema and ulcers
Most common cause of secondary lymphedema Filariasis

Most important initial screening test for pulmonary HPN Echocardiogram with bubble study

Gold standard for diagnosis and assessement of disease


Invasive hemodynamic monitoring
severity of Pulmonary Hypertension
PART 2 I PULMONOLOGY
High-Yield Concepts in Pulmonary Physiology
1. Inspiratory Reserve Volume (IRV)
2. Tidal Volume (TV): amount ofair inhaled/exhaled during
the relaxed state (approximately SOOmL)
4 lung volumes 3. Expiratory Reserve Volume (ERV)
4. Residual Volume (RV): remaining air in the lungs after
maximal exhalation; maintains oxygenation of the blood
during breath-holding
1. Vital capacity (VC]: IRV+ TV +ERV;maximum amount of air
that can be inhaled/exhaled
2. lnspiratory Capacity (IC): IRV+ TV
4 lung capacities 3. Functional Residual Capacity (FRC): ERV+ RV; volume of gas
in lungs after normal expiration
4. Total Lung Capacity (TLC): IRV+ TV+ ERV+ RV; Volume of
gas present in lungs after a maximal aspiration
• Zone 1 (no blood flow since Pulmonary Capillary Pressure<
Alveolar Pressure)
• Zone 2 (intermittent blood flow since Pulmonary Capillary
Lung zones Pressure > Alveolar Pressure only during systole but not
diastole)
• Zone 3 (continuous blood flow since Pulmonary Capillary
Pressure> Alveolar Pressure)
V/Q at apex of lung
3 (wasted ventilation)
V for ventilation; 0 for oerfusion

V/Q at base of lung 0.6 (wasted perfusion)


Airway obstruction leading to a shunt
V/Q= 0
100% OYV•endoes not improve Pa02
Blood flow obstruction (physiologic
V/Q=oo dead space]
100% oxygen improves Pao2 (eg, pulmonary embolus)

~y 98%: bound to HgB (oxyhemoglobin)


How Oz is transported in blood
2%: freely-dissolved in plasma

70%: converted to HCO,·
How CO2is transported in blood 23%: bound to HgB (carbaminohemoglobin)
7%: freely-dissolved in plasma

Increase in the following factors would cause Mnemonic· CADET face RIGHT:
shift to the right of the 02-Hgb dissociation curve Increased CO2,Acidosis, 2,3-BPG, Exercise, increased
(unloading of02 from Hgb) Temperature

Increase in the following factors would cause Increased Carbon monoxide, Methemoglobin, fetal
shift to the left of the 02-Hgb dissociation curve hemoglobin, decreased temperature, decreased 2,3-BPG,
(increased binding of 02 to Hgb) decreased CO2, alkalosis
Main respiratory center in the medulla; sends
Dorsal respiratory group (DRG) of the medulla
inspiratorv ramp signal to diaphragm
Central control of both inspiration and
expiration (suoolements DRG) during exercise
Ventral respiratory group (VRG) of the medulla
Decreases duration of inspiration and increases
Pneumotaxic center of the pons
resoiratorv rate
Increases duration of inspiration and decreases
respiratory rate Apneustic center of the pons
Pulmonary diseases
High-Yield Concepts in Bronchial Asthma
>12% AND 200 mL increase in FEVi:
15 minutes after an inhaled short-acting 82-agonist;
Reversibility in asthma (spirometry) is demonstrated by or
After a 2 to 4 week trial of oral corticosteroids
[prednisone or prednisolone 30-40 mg daily)
Physiologic abnormality of asthma Airway hyperresponsiveness
Majpr risk factor for asthma Atopy
Imbalance favoring TH2 production over TH1
Pathogenesis behind asthma
IL-5 increased eosinophils
Putative mediators of asthma SRS-A [made up ofleukotrienes C4, D4, E4)
Whorls of shed epithelium in mucus plugs in asthma Curschmann's spirals
Eosinophilic, hexagonal, double-pointed crystals formed
Charcot-Leyden Crystals
from breakdown of eosinophils in sputum
Thickening of the basement membrane due to
Characteristic finding in asthamtic airways
subepithelial collagen deposition
None
Key predominant cell in asthma Many inflammatory cells are involved in asthma
with no key cell that is predominant
Most common allergens that trigger asthma Dermatophagoides (house dust mites)
Most common triggers of acute severe asthma URTI: rhinovirus, respiratory syncytial virus (RSV),
exacerbations coronavirus
Mechanism of exercise-induced asthma (EIA) Hyperventilation
Begins after exercise has ended, and recovers
spontaneously within about 30 min.
Typical presentation of EIA
Worse in cold, dry climates than in hot, humid
conditions.
EIA is best prevented by regular treatment with Inhaled corticosteroids (ICS)
Confirms airflow limitation with a reduced FEY,, FEV,/FVC
Spirometry
ratio, and PEF
Confirms diurnal variations in airflow obstruction Measurements of PEF twice daily
• Relax smooth-muscle cells of all airways, where
they act as functional antagonists
Primary action of 82-agonists
• Has little or no effect on the underlying
inflammatory process

Most common side effects of 82-agonists Muscle tremor and palpitations


• Dry mouth
Most common side effect of anticholinergics • In elderly patients, urinary retention and
glaucoma may also be observed.
Most common side effects of theophylline Nausea, vomiting, headaches
Most effective controllers for asthma ICS
Indicates the need for regular controller therapy Use of a reliever medication >2x a week
Most common reason for poor control of asthma Noncompliance with medications, usuallv ICS
• Short-acting 82-agonists
Drugs that are safe for asthma in pregnancy • ICS
• Theophylline
High-Yield Concepts in COPD
Asthma and COPDare variations of the same basic disease Dutch hypothesis
Asthma (allergic phenomenon) and COPD (smoking-related
British hypothesis
inflammation and damaP-el are fundamentallv different diseases
Imbalance between Protease (Elastase) and Anti·
Pathogenesis behind emphysema
Protease (Alpha• 1 •Anti• Trypsin)

First symptom of emphysema Progressive dyspnea

Ratio of mucus gland layer thickness to the thickness of the wall Reid's Index
between the epithelium and the cartilage orthe trachea and bronchi (>0.4 in Chronic Bronchitis)

Most highly significant predictor ofFEV1 Pack-years of cigarette smoking


Important causes of COPD exacerbations Respiratory infections

Most common form of severe o: 1.AT deficiency PiZ: two Z alleles or one Zand one null allele

Most typical finding in COPD Persistent reduction in forced expiratory flow rates
Accounts for essentially all or the reduction in Pa02 that occurs in
Ventilation-perfusion mismatching
COPD
Major site or increased resistance in COPD Small airways< 2 mm diameter

Centrilobular emphysema: prominent in the upper


Type or emphysema frequently associated with cigarette smoking,
lobes and superior segments of lower lobes and often
characterized by enlarged air spaces found (initially) in association
focal; involves the respiratory bronchiole
with respiratory bronchioles
(Mnemonic: SENTROacinar, Smoking)

Type or emphysema usually observed in patients with al-AT


deficiency, characterized by abnormally large air spaces evenly Panlobular emphysema: predilection for lower lobes
distributed within and across acinar units
Type or emphysema distributed along the pleural margins with
Paraseptal emphysema
relative snarinl!' or the lunl!' core or central rel!'ions
Most typical finding in COPD Persistent reduction in forced expiratory flow rates
'
Major physiologic change in COPD Airflow limitation

Characteristic of COPD,reOecting the heterogeneous nature or the Non-uniform ventilation and ventilation-perfusion
disease process within the airways and lung parenchyma mismatching

Accounts for essentially all of the reduction in Pao2 that occurs in Ventilation-perfusion mismatching
COPD (shunting is minimal)

Most highly significant predictor ofFEVl Pack-years of cigarette smoking

Newly-developed clubbing of-the digits (not a sign ofCOPD) should


Lung Cancer
alert an investigati9n for

Hallmark or COPD Airflow obstruction

The only pharmacologic therapy demonstrated to unequivocally


Supplemental 02
decrease mortality rates

Strong predictor or future COPD exacerbations History of prior exacerbations


• Streptococcus pneumoniae
• Haemophilus injluenzae
Bacteria frequently implicated in COPDexacerbations • Moraxe/la catarrhalis
In addition, Mycoplasma pneumoniae or Chlamydia
pneumoniae are found in 5-10% of exacerbations
• Smoking cessation
The only three interventions shown to inOuence the natural history • Oxygen therapy in chronically hypoxemic patients
ofCOPD • Lung volume reduction surgery in selected patients
with emphysema
High-Yield Concepts in Pneumonia and Other Pulmonary Infections
Most common way microorganisms gain access to the lower
Aspiration from the oropharynx
respiratory tract in pneumonia

Most common etiology of community-acquired pneumonia Streptococcus pneumoniae

Mycoplasma pneumonlae, Chlamydia


pneumoniae, and Legionella species

Common etiologies of atypical pneumonia Respiratory viruses such as influenza


viruses, adenoviruses, human
metapneumovirus, and respiratory syncytial
viruses

Combination or an unprotected airway ( e.g.,


Major risk factor for anaerobic pneumonia in patients with alcohol or drug overdose or a
seizure disorder) and significant gingivitis

Organism well known to complicate influenza infection and has


S. aureus
potentially serious consequences, such as necrotizing pneumonia

Main purpose of the sputum gram stain Ensure suitability of sample for culture

>25 neutrophils; and


To be adequate for culture, a sputum sample must have
<10 squamous cells per low power field

Most frequently isolated pathogen in blood cultures or


Streptococcus pneumoniae
community·acquired pneumonia

High-Yield Concepts in Bronchiectasis


Irreversible airway dilation that involves the lung in either a focal
Bronchiectasis
or a diffuse manner

Most common form of bronchiectasis Cylindrical or tubular

Vicious Cycle Hypothesis - susceptibility to


infection and poor mucociliary clearance
Most widely cited mechanism of infectious bronchiectasis
result in microbial colonization of the bron·
chial tree.

Persistent productive cough with ongoing


Most common clinical presentation of bronchiectasis
production of thick, tenacious sputum

Imaging modality of choice for confirming bronchiectasis Chest CT


High-Yield Concepts in Pleural Effusion and Pneumothorax
Determine whether effusion is a
First step in the diagnostic approach to pleural effusion
transudate or exudate

Leading causes of transudative pleural effusion LV failure and cirrhosis

Pleural fluid N-terminal pro-brain


Virtually diagnostic that the effusion is secondary to congestive heart
natriuretic peptide (NT-proBNP) >1500
failure.
pg/mL

Bacterial pneumonia, malignancy, viral


Leading causes of exudative pleural effusion
inrection, pulmonary embolism

Trauma (most frequently thoracic


Most common cause of chylothorax surgery), but it also may result from
tumors in the mediastinum

• Lung carcinoma
Three tumors that cause ~75% of all malignant pleural effusions • Breast carcinoma
• Lymphoma

Benign ovarian tumors producing ascites and pleural effusion Meigs syndrome

Condition most commonly overlooked in the differential diagnosis of a


Pulmonary embolism
patient with an undiagnosed effusion

Insertion of a chest tube plus


Treatment of choice for most cases of chylothorax
administration of octreotide

Primary spont.aneous pneumothoraxes occur almost exclusively in Smokers

Tracheal deviation in tension pneumothorax Contralateral tracheal deviation

High-Yield Concepts in Obstructive Sleep Apnea/Hypopnea syndrome


(1) Either symptoms of nocturnal breathing disturbances [snoring, snorting, gasping, or
breathing pauses during sleep) or daytime sleepiness or fatigue that occurs despite
Diagnosis of
sufficient opportunities to sleep and is unexplained by other medical problems; and
Obstructive Sleep
Apnea/Hypopnea
(2) Five or more episodes of obstructive apnea or hypopnea per hour of sleep (the
syndrome
apnea-hypopnea index [AHi], calculated as the number of episodes divided by the
number of hours of sleep) documented during a sleep study.

Cessation of airflow for HO s during sleep, accompanied by:


Apnea 0 Persistent respiratory effort (obstructive apneas), or
0 Absence of respiratory effor ( central apneas)

A ~30% reduction in airflow for at least 10 s during sleep that is accompanied by either
Hypopnea
a ~3% desaturation or an arousal
High-Yield Concepts in Mediastinal Masses
First step in evaluating a mediastinal mass Place it in one of the three mediastinal comparbnents
Mnemonic· Remember T!
• Thymomas
Most common lesions in anterior mediastinum • Teratomatous neoplasms
• Thyroid masses
• Terrible Lvmohomas
• Bronchogenic cysts
• Pericardial cysts
Most common masses in the middle mediastinurn • Lymphoma
• Metastatic lymph node enlargement
• Masses of vascular origin
• Neurogenic tumors, meningocele, meningomyelocele,
gastroenteric cysts, esophageal diverticula, hernia
Most common masses in the posterior rnediastinum through foramen of Bochdalek, extramedullary
hematopoiesis

H"1gh -y·1eId Concep ts. m DVT an dP u Imonary Emb o rism


Most common preventable cause of death among
hospitalized patients.
Pulmonary embolism
"
Arterial hypoxemia and an increased alveolar-arterial
Most common gas exchange abnormalities in PE
02 tension gradient,

Hallmarks of massive PE Dyspnea, syncope, hypotension, and cyanosis

Cramp of"'charley horse" in the lower calf that persists


Most common symptom of DVT
and intensifies over several days

Most common symptom of PE Unexplained breathlessness


Useful rule out test: > 95% of patients with normal
Quantitative plasma D-dimer ELISA
levels ( <S00ng/mL) do not have PE
Most frequently cited ECGabnormality in PE St Q3 T3 sign
(in addition to sinus tachycardia) (specific but insensitive)

Most common ECGabnormality in PE T-wave inversion in leads Vt to V4

Principal imaging test for the diagnosis of PE Chest CT Scan with IV contrast

Second-line diagnostic test for PE, used mostly for


Lung Scanning
patients who cannot tolerate IV contrast
McConnell's sign: hypokinesls of the RV free wall with
Best known indirect sign of PE on transthoracic echo
normal motion of the RV apex
Definite diagnostic test for PE which visualizes an
Pulmonary Angiography
intra1uminal filling defect in more than one projection

Foundation for successful treatment of DVTand PE Anticoagulation

Systemic arterial hypotension with usually


Massive pulmonary embolism
anatomically widespread thromboembolism

Moderate to large pulmonary embolism RV hypokinesis with normal systemic arterial pressure

Normal RV function and normal systemic arterial


Small to moderate pulmonary embolism pressure
( excellent prognosis with adequate anticoagulation)
High-Yield Concepts in ARDS
• Severity ofhypoxemia:
• Mild: 200 mmHg < Pao2/Fio2,; 300 mmHg
• Moderate: to0_mmHg < Pao2/Fio2,; 200 mmHg
• Severe: Pao2/Fio2 ,; 100 mmHg

• Acute: Within 1 week of a clinical insult or new or worsening


Diagnostic criteria for ARDS
respiratory symptoms.
• Bilateral opacities consistent with pulmonary edema not fully
explained by effusions, lobar /lung collapse, or nodules
• Absence of left atrial hypertension - Hydrostatic edema is not
the primary cause of respiratory failure. If no ARDSrisk factor
is present. then some objective evaluation is required ( e.g.,
echocardiography) to rule out hydrostatic edema

Pneumonia and sepsis (-40-60%), followed in incidence by


Most cases of ARDSare caused by aspiration of gastric contents, trauma, multiple transfusions,
and drug overdose

3 phases of ARDS Exudative Proliferative Fibrotic phase

Evidence-based recommendation for tidal


Low VT ventilation (6 mL/kg of predicted body weight)
volume to minimize ventilator-inducedlung
The only Grade A recommendation in treatment of ARDS
injury

Mortality in ARDSis Nonpulmonary causes, with sepsis and nonpulmonary organ


largely attributable to failure accounting for >80% of deaths.

High-Yield Concepts in Acute Respiratory Failure


Acute hypoxemic respiratory failure - occurs with alveolar flooding and subsequent
intrapulmonary shunt physiology
• Pulmonary edema
Type 1 • Pneumonia
• Alveolar hemorrhage
• Lung injury

Alveolar hypoventilation and results from the inability to eliminate carbon dioxide
effectively
Type2 • Impaired central nervous system (CNS) drive to breathe
• Impaired strength with failure of neuromuscular function in the respiratory system
• Increased respiratory load

Type3 Respiratory failure due to atelectasis (aka perioperative respiratory failure)

Type4 Hypoperfusion of respiratory muscles usually secondary to shock


PART 31GASTROENTEROLOGY
High-Yield Physiology Concepts in Gastroenterology
Causes contraction of muscularis mucosa; for
Meissner Plexus/Submucosal Plexus
secretion
Causes contraction of the inner circular and
Auerbach Plexus/Myenteric Plexus
outer longitudinal muscles; for motility

Mucus Neck Cells Secretes mucus in the stomach


Secretes HCIand Intrinsic Factor (IF) in the stomach (HCL=
Parietal Cells
parietal)

Chief Cells Secretes pepsinogen in the stomach

Enterochromaffin Cells Secretes serotonin in the stomach

Enterochromaffin-Like Cells Secretes histamine in the stomach

Interstitial cells ofCajal Pacemaker cells of the GI tract (generate slow waves)

Ito Cells Store vitamin A in the liver

Carbohydrates Mouth (salivary amylase/ptyalin)

Start of Digestion Fats Stomach (lingual lipase)

Proteins Stomach (pepsin and denaturation by HCI)

Duodenum Iron, vitamin C


Absorption of
Jejunum Carbohydrates, fats, proteins, water
Nutrients
Ileum Vitamin 812, IF, Bile salts, vitamins ADEK

GI Hormones
DESCRIPTION
TRIGGER: CHON and aa (esp F, W, M), Gastric Distention
, ..
SOURCE: G cells of the antrum Gastrin
ACTIONS:Stimulates Parietal cells in fundus for HCISecretion, growth of gastric mucosa

TRIGGER: All Types of Food [main: Fatty Acids)


SOURCE: I cells in the duodenum Cholecystokinin
ACTIONS: Bile Secretion (gallbladder contraction, Sphincter of Oddi relaxation), Increases (CCK)
gastric emptying time ( decreases GE), Increases pancreatic enzyme secretion

TRIGGER: H' in the duodenum, FA in duodenum


SOURCE: S cells in the duodenum
Secretin
ACTIONS: Inhibits HCIsecretion, increases biliary and pancreatic HCO3·
*this hormone does NOT affect pancreatic ENZYMEsecretion!

TRIGGER: Oral Glucose


Glucose-dependent
SOURCE: K cells in the duodenum
lnsulinotropic
ACTIONS:Stimulates insulin secretion; inhibits gastric emptying (above normal physio
Peptide (GIP)
levels)

TRIGGER: Fasting
SOURCE:M cells in the duodenum and Jejunum Motilin
ACTIONS:activates interdigestive / migrating myoelectric complex
High-Yield Classic Disease Patterns in the Gastrointestinal System
• 2% of the population
• 2 years old (most are symptomatic)
• 2:1 male: female ratio
Rule of 2s in Meckel's Diverticulum
• 2 types of epithelia (gastric/pancreatic)
• 2 feet from ileocecal valve
• 2 inches long

Recurrent abdominal pain, on average, at least 1 day per week in the


last 3 months, associated with ~2 of the following criteria
Diagnostic Criteria for Irritable Bowel
Syndrome (JBS)
1. Related to defecation
Rome IV
2. Associated with a change in frequency of stool
3. Associated with a change in form (appearance) of stool

Charcot's Triad for Ascending FPJ


Cholangitis Fever, RUQ Pain, Jaundice

Reynolds' Pentad Charcot's Cholangitis Triad + Shock and Altered mental status

• Liver Disease
Triad of Hepatopulrnonary Syndrome • Hypoxemia
• Pulmonary Arteriovenous Shunting

Clinical manifestation suggestive of Platypnea - shortness of breath that occur paradoxically upon the
hepatopulmonary syndrome assumption of an upright position

• Sudden RUQtenderness
Triad of Acute Cholecystitis • Fever
• Leukocytosis
V
• Abdominal Pain
Triad of Choledochal Cyst • Jaundice
• Abdominal Mass

~---

• Biliary Pain
Triad of Hemobilia • Obstructive Jaundice
• Melena

Typical abdominal pain in the epigastrium that may radiate to the


Diagnosis of Acute Pancreatitis back
(Requires at least 2 of the 3) 3x or greater elevation in serum amylase and/or lipase levels
Confirmatory findings on cross-sectional abdominal imaging
High-Yield Concepts in Esophageal Disorders
Typical symptoms ofGERD Heartburn and regurgitation

Most sensitive test for diagnosis of GERO 24-hour ambulatory pH monitoring

Endoscopic hallmark ofGERD Erosive esophagitis at the esophagogastric junction


Perception of a lump or fullness in the throat that is
Globus sensation, also known as globus pharyngeus
felt irresnective of swallowing
Characteristic symptom of Infectious Esophagi tis Odynophagia

Schatzki ring in the lower esophagus (meat usually instigates


Common cause of Steakhouse Syndrome
intermittent food impaction)

Bird's beak appearance Radiographic sign in achalasia


Seen radiographically in diffuse esophageal spasm (DES) or spastic
Corkscrew or rosary bead esophagus
achalasia
Esophageal Manometry Detects impaired LES relaxation and absent peristalsis in achalasia

Test for evaluation of the proximal GIT Endoscopy/esophagogastroduodenoscopy (EGO)

Cobblestone appearance of esophagus Crohn's disease (on endoscopy or barium radiography)


Barrett's metaplasia with the associated risk of esophageal
Most severe histologic consequence of GERO
adenocarcinoma
Progressive dysphagia (initially to solids, and then to liquids) and
Typical presentation
weie:htloss
Proximal esophagus, associated with smoking alcohol consumption,
Esophageal Squamous Cell CA
caustic iniurv, and human oaoilloma virus infection
Cancer
Distal esophagus, associated with GERO & Barrett's Esophagus
Adenocarcinoma
(metaplasia from squamous to columnar epithelium)

High-Yield Concepts in Gastrointestinal Bleeding


Most common cause of UGIB Peptic ulcers

Most common cause of LGIBoverall


~' Hemorrhoids

Most common cause of LGIBin adults if hemorrhoids and anal fissures are excluded Diverticulosis

Most common cause of Rectal Bleeding in infancy Anal fissure

Most common cause of significant small-intestinal GIB in children Meckel's diverticulum

Most common colonic causes of significant GIB in children and adolescents 180 and juvenile polyps

Bleeding vascular ectasias and aortic stenosis Heyde's syndrome

Boerhaave Syndrome Full-thickness esophageal tear (rupture)

Mallory-Weiss Tear Partial-thickness esophageal tear

Vomiting, retching, coughing preceding


Classic history of Mallory-Weiss Tear
hematemesis in an alcoholic patient

Most important cause of gastric and duodenal erosions NSAID

Best way to initially assess a patient with GIB Heart rate and BP

Should be performed within 24 h in most patients with UGIB Upper endoscopy

Procedure of choice in LGIB Colonoscopy after an oral lavage solution

Initial test for patients with massive bleeding suspected to be from the small intestine Angiography
High-Yield Concepts in Peptic Ulcer Disease (PUD)
Key enzyme in rate·limiting step ofprostaglandin
Cyclooxygenase (COX)
svnthesis
Most common causes of gastric/duodenal ulcers
Helicobacter pylori and NSAIDs
(GU/DU)
Mechanism of survival of ff. pylori in the UGItract Urease production
tst portion of the duodenum, with ~90% located within
Most common location of DUs
3 cm of the pylorus
Pain that awakens the patient from sleep (between
Most discriminating symptom of DUs
midni2ht and 3 AMl
Occurs 90 min to 3 h after a meal and is frequently
Typical pain pattern in DU
relieved by antacids or food.
Most frequent finding in patients with GU or DU Epigastric tenderness

GI bleeding> perforation> gastric outlet obstruction


PUD·related complications
(in order of decreasing frequency)
Monoclonal stool antigen test or a urea breath test
Test of choice for documenting eradication
(UBT).
Most sensitive and specific approach for examining the
Endoscopy
upper GI tract
• Rapid urease test
Invasive tests for H. pylori • Histology
• Culture
Noninvasive test that is not useful for early follow-up Serology

Most potent acid inhibitory agents Proton Pump Inhibitors (PPls)

Most common toxicity with sucralfate Constipation

Most common toxicity with prostaglandin analogues Diarrhea

GU: failure to heal after 12 weeks of therapy


Refractory Peptic Ulcers
DU: failure to heal after 8 weeks of therapy

Most common cause of treatment failure in compliant


Antibiotic-resistant ff. pylori strains
oatients
• Amoxicillin
• Clarithromycin
Antibiotics used to eradicate H. pylori • Metronidazole
• Tetracycline
• Levofloxacin
• Vagotomy and drainage
Most commonly performed operations for DUs • Highly selective vagotomy
• Vagotomy with antrectomy

High ulcer recurrence rate, but lowest complication rate Highly Selective Vagotomy
Lowest ulcer recurrence rate, but highest complication
Vagotomy with Antrectomy
rate
Antrectomy (including the ulcer) with a Billroth 1
Surgery of choice for an Antral Ulcer
anastomosis
Cornerstone of therapy for Dumping Syndrome (DS) Dietary modification
Severe peptic ulcer diathesis secondary to gastric acid
hypersecretion due to unregulated gastrin release from Zollinger-Ellison Syndrome (ZES)
gastrinomas
Superior border: cystic and common bile ducts
Gastrinoma Triangle Inferior border: junction of the 2nd and Jrd portions of
(contains over 80% of these tumors) duodenum
Medial border: iunction of neck and bodv of nancreas
Most common clinical manifestations ofGastrinoma Peptic ulcer, followed by diarrhea

First step in the evaluation ofGastrinoma Obtain a fasting gastrin level

Most sensitive/specific Gastrin Provocative Test Secretin study

Treatment of choice for Gastrinoma PPis


Most common presentation of
GI bleeding
Stress-Related Mucosa! Injury (SRMI]
Treatment of choice for Stress Prophylaxis PPis (preferably oral if tolerated)

Most common causes of Acute Gastritis Infectious

Important predisposing factor for Gastric Cancer Intestinal metaplasia


Involves primarily the fundus and body, with antral
Type A Gastritis
sparing
("Autoimmune": anti-parietal cell antibodies)
Less common tvne
Type B Gastritis Antral-predominant
("Bacteria": H. pylori-associated) More common type
Large. tortuous gastric mucosa) folds (not a form of
Menetrier's Disease
eastritisl

High-Yield Concepts in Inflammatory Bowel Disease (IBD)


Mucosal disease that usually involves the rectum &
Ulcerative Colitis (UC)
extends proximally to involve all or part of the colon
Can affect any part of the GIT from mouth to anus, but
Crohn's Disease (CD) ,( rectum is often spared (in contrast to UC)
,.....__, Transverse or right colon with diameter of >6 cm and
Toxic Megacolon
loss ofhaustrations in severe attacks of UC
pANCAPositivity (Perinuclear Anti-neutrophil
UC» CD
Cytoplasmic Antibodies)
ASCAPositivity (Anti-Saccharomyces cerevisiae
CD» UC
Antibodiesl
Markers of Intestinal Inflammation
Fecal lactoferrin and calprotectin (leukocyte-derived
Used frequently to rule out active inflammation versus proteins)
symptoms of irritable bowel or bacterial overgrowth.
Earliest macroscopic findings of colonic CD Aphthoid ulcers

Pathognomonicfeature of CD Noncaseating granulomas

Most common site of inflammation in CD Terminal ileum

Most dangerous local complication of UC Perforation

Most common ocular complications of IBD Conjunctivitis, anterior uveitis/iritis, and episcleritis

Most common genitourinary complications of IBD Calculi, ureteral obstruction, and fistulas

Most frequent late complication of IPAA Pouchitis


Comparison of Ulcerative Colitis vs. Crohn's Disease
ULCERATIVECOLITIS (UC} CROHN'S DISEASE (CD)

Mani estations
• Major symptoms: diarrhea, rectal bleeding. • Gross bleeding not as common as in UC
tenesmus, passage of mucus • Significant perinea! and perianal disease
Signs and • Signs: tender anal canal, blood on rectal occur more frequently
symptoms examination, and tenderness to palpation
directly over the colon with more extensive
disease
• Continuous, symmetric, and diffuse • Most common site of intlammation is the
Involvement involvement of colon only terminal ileum (involves the colon also)
• Rectum is icall involved • With rectal s arin
• Appendectomy is protective • Smoking is a risk factor
Risks
• Smokin is rotective • NSAIDs ma exacerbate disease activi

Dia nasties
Laboratory • Elevated acute phase reactants, elevated fecal lactoferrin and calprotectin, hypoalbuminemia,
findin s anemia, leukoc osis
Serolo ANCA> ASCA • ASCA> ANCA
• Earliest radiologic finding: fine mucosa! • Earliest radiographic finding: thickened folds
granularity and aphthous ulcerations
Barium enema
• Loss of haustral folds • String sign: narrowed intestinal lumen on
findings
radiographic tests due to edema, bowel wall
thickenin and bowel wall fibrosis
• Symmetric ulceration • Skip lesions with normal intervening mucosa
• Pseudopolyps may form • Rectal sparing is specific
Endoscopic
• Terminal ileum not often involved (except in • Aphthous ulcerations (earliest macroscopic
findings
severe cases, as in backwash ileitis) finding), erythema, edema, cobblestoning
ser i inous ulcers
• Defining lesion: crypt abscesses and ulcers • Granuloma formation more common
Histopathology • Depth of inflammation: mucosa! • Depth of inflammation: mucosa!, submucosal,
transmural
• Toxic colitis: severe pain and bleeding • Fibrostenotic obstructive disease or a
Complications • Megacolon: abdominal distention, hepatic penetrating fistulous disease
m an

Mana ement
Mild to • Oral or rectal 5-aminosalicylic acid (5-ASA) • Sulfasalazine or controlled•release oral
moderate agents budesonide
disease
Moderate to • Oral corticosteroids, methotrexate, anti•TNF • Oral (IV if severe) corticosteroids,
severe disease agents thiopurines (azathioprine, 6·
mercaptopurine), methotrexate, anti·TNF
a ents
Role of • No role in active or quiescent UC • Metronidazole or ciprofloxacin
Antibiotics
High-Yield Concepts in Irritable Bowel Syndrome (JBS)
Prerequisite clinical feature of 18S Abdominal pain

Altered bowel habits (most common pattern


Most consistent clinical feature in IBS
is constipation alternating with diarrhea)

• Evidence of anemia
Laboratory features that argue against IBS • Elevated sedimentation rate
• Presence of leukocytes or blood in stool
• Stool volume> 200-300 ml/day

Best management for postprandial pain Antispasmodics 30 minutes before meals

Initial therapy of choice for IBS-D(Diarrhea Predominant) Peripherally acting opiate-based agents

Only antibiotic for !BSwith sustained benefit beyond therapy


Rifaximin
cessation
'- ..,
High-Yield Concepts in Diverticular Diseases
True Diverticulum Saclike herniation of entire bowel wall

False Diverticulum Only a protrusion of the mucosa and submucosa through the
(Pseudodiverticulum) muscularis propria of the colon (where the vasa recti penetrates)

Most common site of diverticular disease Sigmoid


Most common cause of hematochezia in
Hemorrhage from a colonic diverticulum
oatients >60 vears
Diverticulitis Inflammation of a diverticulum
Air-fluid level in the LLQon plain abdominal
Giant diverticulum of the sigmoid colon
film

Hinchey Classification System


-~ Staging system for predicting outcomes after surgery for
perforated diverticulitis

• Sigmoid diverticula
Diagnosis of Diverticulitis is best made with • Thickened colonic wall >4 mm
these CT findings • Inflammation within the pericolic fat± the collection of contrast
material or fluid
"-">'
6 weeks after an attack of diverticular disease
Safety window for colonoscopy (should not be performed in acute setting due to higher risk of
perforation)

Best management for massive Diverticular Angiography± coiling fif patient unstable or has had a 6-unit
Bleeding in a stable patient bleed within 24 hours, emergent surgery should be performed)

Best management for asymptomatic


Lifestyle changes
Diverticular Disease

Initial treatment for Symptomatic


Uncomplicated Diverticular Disease (SUDD)
Bowel rest
with confirmation of inflammation and
infection within the colon
High-Yield Concepts in Anorectal Disorders
Circumferential, full-thickness protrusion of the rectal wall through
Procidentia (Rectal Prolapse)
the anal orifice
Involuntary passage of recaJ material for at least 1 month in an
Fecal Incontinence
individual with a develoomental a2:e of at least 4 vears
The result of attempting to defecate against a closed pelvic floor
Anismus
(a.k.a. nonrelaxing puborectalis)
Isolated Mucosa! prolapse vs. Full-thickness
Radial grooves vs. circumferential grooves around anus, respectively
Rectal Prolaose
Mainstay of Therapy for Rectal Prolapse Surgical correction

3 Hemorrhoidal Complexes in the Anal Canal Left lateral, right anterior, and right posterior

Most common presentation of Hemorrhoids Bleeding and protrusion

Hallmarks of an Anorectal Abscess Perianal pain and fever

Most common location of Anorectal Abscess Perianal, followed by ischiorectal

Posterior position, followed by anterior (lateral fissure is worrisome,


Most common location of Anal Fissures
and systemic disorders should be ruled out)
Most common location of Internal Opening of
Dentate line
Fistula In Ano (FIA]
Most common type of FIA lntersphincteric, followed by transsphincteric

Anterior fistula: straight tract to nearest crypt


Posterior fistula: curved tract to enter anal canal at posterior midline
Goodsall's Rule for FIA
Exception: fistulas exiting >3 cm from the anal verge may not obey
Goodsall's rule

Best Management for Newly Diagnosed FIA Seton (vessel loop or silk tie placed through the tract)

High-Yield Concepts in Mesenteric Vascular Disease


Most prevalent gastrointestinal disease complicating
Nonocclusive mesenteric ischemia
cardiovascular surgery

The most significant indicator of survival in intestinal ischemia


Timeliness of diagnosis and treatment
patients

Clinical presentation of patients with acute mesenteric ischemia Severe acute, nonremitting abdominal pain strikingly
resulting from arterial embolus or thrombosis out of proportion to the physical findings
Griffith's point: splenic nexure
Most common locations for colonic lschemia
Sudeck's point: descendin2/si2moid colon
Gold standard for diagnosis of Acute Arterial Occlusive Disease Angiography

Management of Acute Arterial Occlusive Disease Laparotomy

Intervention of choice to maintain hemodynamics in


Fluid resuscitation
Nonocclusive/Vasospastic Mesenteric lschemia
Resection ofischemic bowel & formation of a
proximal stoma
Optimal treatment for lschemic Colitis
Primary anastomosis should not be performed in
oatients with acute intestinal ischemia
Best prognosis of all Acute Intestinal lschemic Disorders Mesenteric venous insufficiency

Area ofnonfluorescence >5 mm in diameter under


Marker of Intestinal Nonviability
UV illumination with Woods lamp
High-Yield Concepts in Intestinal Obstruction
Most commonly identified form of functional bowel
lieus that occurs after intraabdominal surgery
obstruction,
Responsible for the majority of cases of early
Adhesions
postoperative obstruction that require intervention

Most common cause of Colonic Obstruction Colon cancer

Most common precursor for strangulation Closed-loop obstruction


Colicky abdominal pain, abdominal distention, emesis,
Cardinal signs of acute intestinal obstruction
and obstipation

Classical findings seen in patients with small-bowel A "staircasing'' pattern of dilated air and fluid-filled
obstruction on abdominal radiography (which must small-bowel loops >2.5 cm in diameter with little or no
include upright or cross-table lateral views) air seen in the colon

Free air seen in radiography suggests Perforation

Most commonly used modality to evaluate patients for


Abdominal CT
Intestinal Obstruction
Ileum
Most common site of intestinal obstruction in patients
with gallstone ileus Enters the intestinal tract most often via a
cholecvstoduodenal fistula
Cecal diameter that increases likelihood of perforation >10-12 cm

High-Yield Concepts in Appendicitis and Peritonitis


Most common emergency general surgical disease
Appendicitis
affectine the abdomen,
Nausea followed the development of abdominal pain
Sequence of symptoms in AA that helps distinguish it
from gastroenteritis
In e:astroenteritis. nausea occurs first
Symptom so common that the diagnosis of
Anorexia
aooendicitis should be auestioned in its absence
Dilatation >6 mm with wall thickening
A lumen that does not fill with enteric contrast
Fatty tissue stranding or air surrounding the appendix,
Suggestive CT imaging findings in appendicitis which suggests inflammation

The presence of luminal air or contrast is not consistent


with a diae:nosis of appendicitis
Indicated to help exclude genitourinary conditions
Urinalysis
that mav mimic acute aooendicitis
Most common extrauterine condition requiring
Appendicitis
abdominal operation during pregnancy

Best diagnostic exam for AAduring pregnancy Ultrasound

Cardinal manifestations of Peritonitis Acute abdominal pain and tenderness. usually with fever
High-Yield Concepts in Evaluation of Liver Disease
Liver injury, inflammation and necrosis
Hepatocellular pattern of liver disease
predominate

Cholestatic pattern of liver disease Inhibition of bile flow predominates

• Assessment of the severity or activity of


Grading of liver disease
disease
• Active or inactive
• Mild, moderate or severe

• Estimation of the point in the course of the


Staging of liver disease
natural history of the disease
• Early or late
• Precirrhotic, cirrhotic, end-stage

Criterion standard in evaluation of liver disease and most


Liver biopsy
accurate means of assessing grade and stage

Prognostication for cirrhosis and provides standard criteria for


listing for liver transplantation (Class 8 & CJ; utilizes serum
Child-Pugh Score
bilirubin, serum albumin, PT-INR and severity of ascites and
hepatic encephalopathy

More objective means of assessing disease severity; utilizes


Model for End-Stage Liver Disease (MELD}Score
serum bilirubin, serum creatinine, and PT-INR

Occurrence of signs or symptoms of hepatic encephalopathy in


Hepatic Failure
a person with severe acute or chronic liver disease

Hepatic inflammation and necrosis that continue for at least 6


Chronic Hepatitis
months

Most common and most characteristic symptom of liver disease Fatigue

Hallmark of liver disease and most reliable marker of severity Jaundice

Most reliable physical finding in examining the liver Hepatic tenderness

Best physical exam maneuver to appreciate ascites Shifting dullness on percussion

Hepatic encephalopathy during acute hepatitis


Major criterion for diagnosis of Fulminant Hepatitis
(indicates poor prognosis)

Screening test for Hepatopulrnonary Syndrome Oxygen saturation by pulse oximetry

Serum bilirubin, serum albumin, prothrombin


Most commonly used Liver Function Tests
time(PT}

Transport of conjugated bilirubin into the bile


Rate-limiting step in bilirubin metabolism
canaliculi (not conjugation itself)

Any bilirubin found in urine Conjugated/direct bilirubin (water-soluble)


Exclusive site for synthesis of serum albumin Hepatocytes

Only clotting factor not produced in the liver Factor VIII

Protime (PT)
(PT prolongation >5 secs not corrected by
Single best acute measure of hepatic synthetic function
parenteral vitamin K administration is a poor
prognostic sign in acute viral hepatitis)

Most helpful in recognizing Acute Hepatocellular Diseases Elevated arninotransferases/transarninases

• Viral hepatitis
Differentials for striking elevations in aminotransferases • Ischernic liver injury
• Toxin- or drug-induced liver injury
(>1000 U/L)
• Acute phase of biliary obstruction caused by
passage or gallstone into CBD

-AST>-ALT Alcoholic liver disease (AST for San Miguel Beer)

Viral hepatitis (ALT is a more specific indicator


-ALT>-AST
ortiver injury]

Key events in hepatic fibrogenesis Stellate cell activation and collagen production

Most commonly employed imaging tests for the liver Ultrasound, CT, MRI

Ultrasonography
First diagnostic test to use in patients whose liver tests suggest to look for the presence or a dilated intrahepatic
cholestasis or extrahepatic biliary tree or to identify
gallstones

First test for suspected Budd Chiari Syndrome (Hepatic Vein


Ultrasound with Doppler imaging
Thrombosis]

Extravasation or RBCs in BCS [but not in cardiac


Budd-Chiari Syndrome (BCS) vs. Cardiac Cirrhosis
cirrhosis)

Special Tests for Liver Disease


DISEASE DIAGNOSTICTESTS AND EXPECTEDFINDINGS
• ANAorSMA
Autoimmune hepatitis • Anti-LKMl
• Elevated I G levels
• AMA(anti-mitochondrial antibody)
Primary biliary cirrhosis
• Elevated I M levels
Primary sclerosing • p-ANCA
cholan itis • Chalan io ra h
• Decreased serum ceruloplasmin and increased urinary copper
• Increased hepatic copper level
Wilson's disease
• Kayser-Fleischer rings (golden-brown copper pigment deposited in the periphery
of the cornea
• Elevated iron saturation and serum ferritin
Hemochromatosis • Genetic testing for HFE gene mutations
• Slate- ra i mentation of skin
• Elevated alpha-fetoprotein level >500 ng/mL
Hepatocellular cancer
• CT or MRI ima e of mass
High-Yield Concepts in Complications of Liver Cirrhosis
Elevation of hepatic venous pressure gradient (HVPG)to >5 mm
Portal Hypertension
Hg
Most common cause of Portal HPN in the US Cirrhosis

Gastroesophageal varices with hemorrhage


3 primary complications of Portal HPN Ascites
Hypersplenism

First indication of Portal HPN in Liver


Hypersplenism with thrombocytopenia
Cirrhosis
Offers an alternative to surgery for acute
Transjugular lntrahepatic Portosystemic Shunt (TIPS)
decompression of portal hypertension
First-line treatment to control Acute Variceal
Endoscopic intervention
Bleeding
Most common cause of ascites Portal HPN related to cirrhosis

Laterality of Hepatic Hydrothorax More common on the right side


Recommended Sodium Restriction for Small
< Z g of sodium per day
Amounts of Ascites

Most common organisms causing Spontaneous


Escherichia coli and other gut bacteria
Bacterial Peritonitis (SBP)

Presumed mechanism for development of SBP Bacterial translocation

Treatment for SBP Third-generation cephalosporin


Hepatic Encephalopathy (Portosystemic Alteration in mental status and cognitive function occurring in
Encephalopathy) the presence of liver failure
Sudden forward movement of the wrist after it is bent back on
Asterixis or Liver Flap
an extended arm; cannot be elicited if patient already comatose
-
Mainstay of treatment for Hepatic
Encephalopathy s::
.
Lactulose, to promote 2-3 soft stools per day

Functional renal failure without renal pathology in patients


Hepatorenal Syndrome (HRS)
-1 with advanced cirrhosis or acute liver failure
Progressive impairment in renal function and significant
Type 1 HRS
~'\...'-0
.
reduction in creatinine clearance within 1-2 weeks
Reduction in GFR with an elevation of serum creatinine level,
Type 2 HRS
but fairly stable (better outcome than Type 1 HRS)
Best Therapy for HRS Liver transplantation
Most common indications for liver
Hepatitis C infection and alcoholic liver disease
transplantation
Phenotype of AlAT Deficiency with Greatest
ZZphenotype
Risk for Developing Chronic Liver Disease
High-Yield Concepts in Viral Hepatitis
Hepatitis B (others are RNAviruses)
It belongs to the HepaDNAviridae family.
Only human Hepatitis Virus that is a DNA Virus
It is the only DNA virus that uses reverse
transcriptase enzyme.

First detectable marker HBs Antigen (HBsAg)

Qualitative marker for high


HBe Antigen (HBeAg)
i nfectivity / replication

Quantitative marker for high


HBV DNA (also correlates with level ofliver injury)
i nfectivity / replication

HBV
Serology First antibody to rise Anti•HBcantibody (1·2 weeks after HBsAg)

Positive during window period lgM anti-HBC antibody

Protective antibody and the only marker to


Anti·HBs antibody
appear after immunization

Criteria for chronic HBV infection HBsAg remains detectable beyond 6 months

Nonpercutaneous routes of HBVtransmission with the Intimate (especially sexual) contact


greatest impact Perinatal transmission (vertical transmission)

Most important mode of HBVperpetuation in the Far East Perinatal transmission (particularly at time of
and Developing countries delivery; not related to breastfeeding)

Risks of Cirrhosis and HCCAin Hepatitis 8 increase with the Level of HBV replication

Reverse transcriptase inhibitor used in managing HBVand


Lamivudine
HIV

Vasculitic syndrome associated with HBVinfection Polyarteritis nodosa

Most common symptom in Hepatitis C Fatigue (jaundice is rare)

Most common risk factor for Hepatitis C Injection Drug Use

The most common genotype of Hepatitis C worldwide Genotype 1

Gold standard for establishing a diagnosis of Hepatitis C


HCVRNA
(most sensitive indicator)
Best prognostic indicator in chronic Hepatitis C Liver Histology

Defective RNAVirus that coinfects with and requires helper


Hepatitis D (HDV)
function of HBV

Presence of antibodies to liver-kidney microsomes


Autoantibody present in chronic Hepatitis D
(anti-LKM3)

Most common cause of acute Hepatitis in India, Asia, Africa, Hepatitis E


and Central America *High mortality among pregnant women

Most feared complication of Viral Hepatitis Fulminant hepatitis (massive hepatic necrosis)

The striking postmortem finding in massive hepatic


Acute Yellow Atrophy
necrosis, referring to a small, shrunken, soft liver

First approved therapy for chronic Hepatitis B IFN-alpha (although no longer used for treatment)

First nucleoside analogue to be approved for Hepatitis B Lamivudine

Most potent of the HBVantivirals Entecavir

• PEG-IFN
First•line drugs for Hepatitis B • Entecavir
_rf_ • Tenofovir

Eradicate HCVRNA during therapy and to document


Goal of treatment in Hepatitis C that the virus remains undetectable for at least 12
weeks after completion of therapy (SVR12).
J
... HCV
Viral indications for ribavirin use
RSV

Most pronounced side effect of ribavirin Hemolysis

Most important variable in progression of liver disease in


Duration of infection
patients with chronic hepatitis C

Hepatitis infection associated with essential mixed


HCV
cryoglobulinemia

• Hepatitis A - PicoRNAviridae, small (pico) RNA


virus
Viral taxonomy (family) • Hepatitis B - HepaDNAviridae
• Hepatitis C - Flaviviridae
• Hepatitis E - Hepevirus
High-Yield Concepts in Alcoholic Liver Disease (ALD)
3 major lesions of ALO Fatty liver, alcoholic hepatitis, cirrhosis
Initial and most common histologic response to
Fatty liver
hepatotoxic stimuli

Most important risk factors for ALO Quantity and duration of alcohol intake

Men: >14 drinks per week


Threshold for developing ALO
Women: >7 drinks per week

Major enzyme responsible for alcohol metabolism Alcohol dehydrogenase


1. Steatosis
2. Steatohepatitis: Ballooning degeneration orhepatocytes,
Morphologic forms of ALO Mallory-Denk bodies, PMN infiltrates
3. Steatofibrosis: chicken wire appearance of fibrosis around
central vein
Hemolytic anemia with spur cells and acanthocytes in patients
Zieve's Syndrome
with severe alcoholic hepatitis
Value of DF where there is improved survival at 28
days with the use of glucocorticoids in patients Discriminant function > 32
with severe alcoholic
Complete abstinence from alcohol (liver biopsy should not be
Cornerstone of ALO treatment
performed until abstinence maintained for at least 6 months)

High-Yield Concepts in Non-Viral Hepatitis


Cause of most Drug Hepatotoxicity Phase I toxic metabolite (cytochrome P450)

Carbon tetrachloride, acetaminophen, Amanita phalloides


Examples of Direct Hepatotoxins
(deathcap mushroom)

Examples of Idiosyncratic Hepatotoxins Amoxicillin-clavulanate, isoniazid, ciprofloxacin

Most common drug causing acute liver failure Acetaminophen

Drugs wherein hepatotoxicity is more frequentJn


Aspirin, methotrexate, isoniazid, antiretrovirals
patients with underlying CLO
Acetaminophen dose producing clinical evidence of
Single dose of 10-15 grams
liver iniurv
Acetaminophen dose usually associated with fatal
25 grams
fulminant disease

Liver transplantation (if with progressive hepatic failure despite


Last resort for Acetaminophen Hepatotoxicity
NAC therapy)

Most important adverse effect of Erythromycin Cholestatic reaction (infrequent)

Hepatotoxic component of trimethoprim-


Sulfamethoxazole
sulfamethoxazole (TMP-SMX)

Chronic disorder characterized by continuing hepatocellular


Autoimmune Hepatitis (AIH) necrosis and inflammation, usually with fibrosis, which can
progress to cirrhosis and liver failure

Classic syndrome in young women


Type I AIH
ANA antibodies (and p-ANCA)
Often seen in children, common in Mediterranean
Type II AIH
Anti-LKM antibodies
Anti-LKMl Type II AIH & hepatitis C
Anti-LKM2 Drue:-induced heoatitis
Anti-LKM3 Chronic hepatitis D
Mainstay of treatment for AIH Glucocorticoid therapy
High-Yield Concepts in Gallbladder and Biliary Diseases
Cholesterol stones (>80%)
2 major types of Gallstones
Pigment stones (<20%)

Most important mechanism in the formation of


Increased biliary secretion of cholesterol
lithogenic bile

A marked increase in cholesterol saturation of bile during


2 key changes during pregnancy that contribute the 3 rd trimester
to a cholelithogenic state Sluggish gallbladder contraction in response to a standard
meal a impaired gallbladder emptying
Most frequently isolated organisms in Escherichia coli, Klebsiella spp., Streptococcus spp., Clostridium
eallbladder bile SOD,

Anaerobes, such as Clostridium welchii or C/ostridium


Most frequently isolated bacteria in
per{rinnens
emphysematous cholecystitis
Aerobes, such as E. coli (a facultative anaerobe)
Most frequent demographic for emphysematous
Elderly men and diabetics
cholecystitis

Gas within the gallbladder lumen on plain abdominal film,


Radiographic diagnosis of Emphysematous
dissecting within the gallbladder wall to form a gaseous ring.
Cholecystitis
or in the pericholecystic tissues

Deep inspiration or cough during subcostal palpation or the


Murphy's Sign RUQ produces increased pain and inspiratory arrest,
suggestive of acute cholecystitis or cholangitis
Gallstone becomes impacted in the cystic duct or neck of the
Mirizzi's Syndrome gallbladder causing compression of the CBD,resulting in
obstruction and jaundice

Presence of a palpably enlarged gallbladder suggests that the


Courvoisier's Law biliary obstruction is secondary to an underlying malignancy
rather than to calculous disease

Acoustic "shadowing" of opacities that are within the


Sonographic criteria for identifying gallstones gallbladder lumen
Change with the patient's position (by gravity)
Most common site of fistula formation in
Fistula into the duodenum
Cholecvstitis
Usual site of obstruction in Gallstone lieus lleocecal Valve

Calcium salt deposition within the wall of a chronically


Porcelain Gallbladder inflamed gallbladder; associated with gallbladder carcinoma,
so cholecystectomy is advised

Meperidine or NSAIDs(produce less spasm of sphincter of


Usual analgesics for Acute Cholecystitis
Oddi than morphine)
Gold standard for treating symptomatic
Laparoscopic cholecystectomy
Cholelithiasis
Treatment of choice for uncomplicated Acute
Early cholecystectomy (within 72 hours)
Cholecvstitis
Overall medical condition imposes an unacceptable risk for
Delayed surgical intervention in Cholecystitis early surgery
Diagnosis of acute cholecyslilis in doubt
Overlooked symptomatic nonbiliary disorder (renux
Most common cause of persistent
esophagitis, peptic ulceration, pancreatitis, or most often,
Postcholecystectomy Symptoms
irritable bowel svndrome)
Atretic and hypoplastic lesions of the extra hepatic and large
Most common biliary anomalies in infancy
intrahepatic bile ducts
Congenital biliary ectasia involving the major intrahepatic
Caroli's Disease
radicles
Most common type of Cholangitis Nonsuppurative acute cholangitis (vs. suppurative)
Preferred initial procedure for both establishing
a definitive diagnosis and providing effective ERCPwith endoscopic sphincterotomy
theraov
Most common associated entity in patients with
Biliary tract disease
Nonalcoholic Acute Pancreatitis
History of jaundice or pancreatitis
Risk factors for concomitant CBDstones in
Abnormal tests of liver function
patients with Gallstones
Ultrasonographic or MRCP evidence of a dilated CBD
Preferred approach ifCBD stones are suspected Preoperative ERCPwith endoscopic papiUotomy and stone
prior to Laparoscopic Cholecystectomy extraction

Treatment of choice in the management of Endoscopic biliary sphincterotomy (EBS) followed by


patients with common duct stones spontaneous passage or stone extraction

Most common cause of benign strictures of the


Surgical trauma
extrahepatic bile ducts
Most common cause of extrinsic bile duct
Carcinoma of the pancreatic head
comoression
Organisms most commonly involved in Trematodes or nukes, including Clonorchis sinensis,
hepatobiliary parasitism Opisthorchis viverrini or O.felineus, Fascia/a hepatica
Clonorchis sinensis
Parasite associated with cholangiocarcinoma

Earliest lesion in Primary Biliary Cirrhosis (~BC) Chronic nonsuppurative destructive cholangitis

Antibodies associated with PBC Antimitochondrial antibodies (AMA)

Main symptoms of PBC ....,v Fatigue and pruritus

Ursodeoxycholic acid (UDCA)can slow the rate of progression


Only approved treatment for PBC
of disease (but cannot reverse or cure)
Treatment of choice for decompensated
Liver transplantation
cirrhosis due to PBC
Imaging technique of choice for initial evaluation Magnetic resonance imaging (MRI) with magnetic resonance
of Primary Sclerosing Cholangitis (PSC) cholangiopancreatography (MRCP)
Multifocal stricturing and beading involving both the intra-
Typical cholangiographic findings in PSC
and extrahepatic biliary tree

Ultimate treatment for PSC Liver transplant

Dreaded complication of PSC Development of cholangiocarcinoma


High-Yield Concepts in Pancreatic Disorders
Most common cause of Acute Pancreatitis Gallstones, followed by alcohol

Currently accepted pathogenic theory for Pancreatitis Autodigestion

Abdominal pain

Major symptom of Acute Pancreatitis Steady and boring in character, is located in the
epigastrium and periumbilical region, and may
radiate to the back, chest, flanks, and lower abdomen

Blue discoloration around the umbilicus from


Cullen Sign
hemoperitoneum

Blue-red-purple or green brown discoloration of the


Turner Sign
flanks from tissue catabolism of hemoglobin
Laterality of pleural effusion in Pancreatitis Most frequent on the left

Recommended screening tests Serum lipase and amylase levels

Single best enzyme to measure for the diagnosis Lipase (more specific)

Pancreatitis from Hypertriglyceridernia Serum triglyceride levels (>1000 mg/dL

Hematocrit > 44%


Associated with more severe acute pancreatitis
Admission BUN>22 mg/dL

Initial diagnostic imaging modality and is most useful to


Abdominal ultrasound
evaluate for gallstone disease and the pancreatic head
3-5 days into hospitalization when patients are not
CT imaging in acute pancreatitis is best evaluated responding to supportive care to look for local
complications such as necrosis.

Use of a prophylactic pancreatic duct stent


Shown to reduce pancreatitis after ERCP Rectal nonsteroidal anti-inflammatory drugs
(NSAIDs,indomethicin)
Most important treatment intervention for acute
Safe, aggressive intravenous fluid resuscitation.
oancreatitis
Enteral nutrition should be considered 2-3 days after
admission in subjects with more severe pancreatitis
Nutritional therapy in Acute Pancreatitis instead of total parenteral nutrition (TPNJ.
The choice of gastric versus nasojejunal enteral
feedim! is currentlv under investie:ation.
Diagnosis of Pancreatic Infection Percutaneous aspiration with Gram stain and culture

Most common cause of Chronic Pancreatitis in US adults Alcoholism


Most common cause of Chronic Pancreatitis in US
Cystic fibrosis
children
Predominant symptoms of Chronic Pancreatitis Abdominal pain or maldigestion and weight loss
Diagnostic test with best sensitivity /specificity for Secretin stimulation test (abnormal once >60% of
Chronic Pancreatitis pancreatic exocrine function has been lost)

Most reproducible measurement in the Secretin Test Maximal HC01 concentration

Most common cause of pancreatic calcification Alcohol


Most common congenital anatomic variant of the
Pancreas divisum
oancreas
PART 4 I NEPHROLOGY
High-Yield Physiology Concepts in Nephrology
Site of erythropoietin (EPO)
Interstitial cells of the peritubular capillaries
production

Absorb Na• and H20 and secrete K• Principal cells of the Late Distal Tubule

Absorb K• and secrete H• Intercalated cells of the Late Distal Tubule

Macula Densa in the distal tubule detects decreased BP or decreased


GFR Juxtaglomerular Cells (JG Cells) release Renin converts
angiotensinogen to Angiotensin I Angiotensin I converted to
Renin-Angiontensin-Aldosterone
Angiotensin II in the lungs using Angiotensin-Converting Enzyme (ACE)
System
stimulates Aldosterone release from the Zona Glomerulosa of the
Adrenal increases Na+reabsorption, K+secretion H• secretion
increases intravascular volume increased VR, CO and BP
Via Vt receptors: vasoconstriction of systemic arterioles increased
TPR -+ increased BP
Via V2 receptors: Insertion ofaquaporins (AQP-2) in the collecting ducts
ADH actions
-+ increased water reabsorption leading to:
1. Increased intravascular volume -+ increased VR, CO, BP
2. Decreased urine volume increased urine osmolaritv
• Increased plasma osmolarity [main trigger]
Triggers for ADHsecretion • Decreased blood volume
• Decreased blood pressure

Free Water Clearance (CH20)


Estimates ability to concentrate or
If(-) ADH: Free water excreted and CH,O is positive
dilute urine
If(+) ADH: Free water is reabsorbed and CH20 is negative

• Afferent arteriolar vasodilation


Increases GFR
• Moderate efferent arteriolar vasoconstriction

-~ • Afferent arteriolar vasoconstriction

Decreases GFR • Efferent arteriolar vasodilation


• Severe efferent arteriolar vasoconstriction
I \)

• Filtration Fraction = GFR/Renal Plasma Flow


• Normal Value = 20%
Filtration Fraction [FF) • Vasoconstriction of Efferent Arteriole will decrease FF.
• Increased Plasma Protein, Ureteral Stone will increase FF.
• Vasoconstriction of Afferent Arteriole will not chani?e FF.
Ascending limb of the Loop of Henle is Solutes fMnemonjc:asinding limb is permeable to solutes)
permeable to Impermeable to water

Descending limb of the Loop of Henle Water


is permeable to Impermeable to solutes

Acid-base abnormalities caused by Metabolic acidosis: acetazolamide fMnemonil;: acidazolamide)


diuretics Metabolic alkalosis: loop diuretics, thiazide diuretics
CLINICALNEPHROLOGY
Diagnostic Procedures in Nephrology
Test for measurement of albuminuria Urinary albumin to creatinine ratio

Most useful renal imaging study Renal ultrasound


It is technically difficult and has a greater likelihood
of causing bleeding and other adverse consequences,

Renal biopsy is not advised in a patient with bilaterally There is usually so much scarring that the underlying
small kidneys because disease may not be apparent, and

The window of opportunity to render disease-specific


therapy has passed.
Helical computed tomography (CT) scanning without
Imaging test for diagnosis ofnephrolithiasis
radiocontrast enhancement

u·1gh -y·1eId Concepts m Rena IR ep acement Th erapy A"-.•


Most common form of renal replacement therapy for AKI Hemodialysis
1.Uremic pericarditis
2.Encephalopathy
3.Intractable muscle cramping, anorexia, and nausea
Clear indications for initiation of renal replacement not attributable to reversible causes
therapy in patients with CKD 4.Evidence of malnutrition
5.Fluid & electrolyte abnormalities (principally
hyperkalemia or ECF volume overload) refractory
to other measures
Number of hours of hemodialysis in majority ofESRD
9 - 12 hours each week divided into 3 equal sessions
patients
Best potential for complete renal rehabilitation Kidney transplantation

Educational programs should be commenced No later than stage 4 CKD

Most common therapeutic modality for end-stage renal


Hemodialysis
disease (ESRD)

Leading cause of ESRD Diabetes mellitus

Dialysis access with highest long-term patency rate Fistula

Most important complication of arteriovenous grafts Thrombosis of the graft and graft failure

Most common acute complication ofhemodialysis,


Hypo tension
particularly among DM patients
Heparin
Most common additives to peritoneal dialysis solutions
Antibiotics
Insulin
Most common organisms in peritoneal dialysis-related Gram-positive cocci, including Staphylococcus
peritonitis (reflecting the origin from the skin)
Absolute indication for the urgent initiation of or
Uremic pericarditis
intensification of dialysis prescription
High-Yield Concepts in Acute Kidney Injury (AKI)

A rise ofat least 0.3 mg/dL within 48 h or 50% higher


Definition of AKI than baseline within 1 week; or reduction in urine
output to < 0.5 mL/kg/h for >6 hours

Definition of oliguria <400 mL/24 h

Light chain cast nephropathy commonly known as


Associated with multiple myeloma
myeloma kidney

Hallmark of AKI Elevated BUN

Most common form of AKI Prerenal Azotemia

Renal autoregulation usually fails once the systolic blood


80mmHg
pressure falls below

Extensive necrosis, PCT and DT affected, relatively longer Toxic-type ATN [e.g. in use of aminoglycosides,
lengths of tubules radiocontrast dyes)

• Prerenal Azotemia
Three broad categories of AKI
• Intrinsic AKJ
• Post•renal AKI

• Hypovolemia

Most common clinical conditions associated with


• Decreased cardiac output
prerenal azotemia • Medications that interfere with renal autoregulatory
responses such as NSAIDs and inhibitors of
-~ angiotensin II

Most common causes of intrinsic AKI Sepsis, ischemia, and nephrotoxins

A rise in SCr beginning 24-48 hours following


exposure
Most common clinical course of contrast nephropathy
Peak within 3-5 days
Resolution within 1 week

Most common protein in urine and produced in the thick


Uromodulin/Tamm-Horsfall Protein
ascending limb of the loop of Henle
High-Yield Concepts in Chronic Kidney Disease
• Diabetic nephropathy
Causes of large kidneys observed in CKD • HIV-associated nephropathy
• Infiltrative diseases
In the absence of a clinical diagnosis, the only recourse to
Kidney biopsy
establish an etiology in early-sta~e CKDis
Definitive treatment of the hepatorenal syndrome Liver transplantation
• Severe hemodynamic instability
Continuous Renal Replacement Therapy is often preferred in
patients with • Cerebral edema
• Significant volume overload

ESRDrefers to Stage 5 CKD(<15% GFR)

In early nephropathy, such as in diabetic nephropathy, Microalbuminuria and represents the presence of
proteinuria increases to 30-300 mg/24 hand is called renal disease
Total-body calcium accumulation and
Major side effect of calcium-based phosphate binders
hvnercalcemia
Leading cause of morbidity and mortality in patients at every
Cardiovascular disease (CVD)
stage ofCKD
Among the strongest risk factors for cardiovascular Left ventricular hypertrophy and dilated
morbidity and mortality in CKD cardiomyopathy

Poor left ventricular function


Absence of hypertension in CKDmay signify
Low blood pressure actually carries a worse
prognosis than does high blood pressure

Stage of CKDwhere normocytic, normochromic anemia As early as Stage 3 CKD


appears Almost universal by Stage 4 CKD

Primary cause of anemia ~~.., Insufficient production of EPOby the diseased


kidneys

Target hemoglobin concentration in CKD 100-115 g/L

Stage of CKDwhere peripheral neuropathy usually becomes


Stage4CKD
clinicallv evident
Protein excretion >300 mg
Indication for therapy with ACE inhibitors or ARBs
(especially in OM nephropathy)
Derives from the breakdown of urea to ammonia in saliva and
is often associated with an unpleasant metallic taste Uremic fetor
(dysgeusial
Most important initial diagnostic step in the evaluation of a To distinguish newly diagnosed CKDfrom acute or
patient presenting with elevated serum creatinine subacute renal failure

Classic lesion of secondary hyperparathyroidism; high bone


Osteitis fibrosa cystica
turnover with increased PTH levels

Low bone turnover with low or normal PTH levels Adynamic bone disease
Devastating condition seen almost exclusively in patients
Calciphylaxis
with advanced CKD
Seen in patients with CKDwho have been exposed to
Nephrogenic fibrosing dermopathy
gadolinium
High-Yield Concepts in Glomerular Diseases
RBC casts or dysmorphic RBCs seen in the sediment GN
Most common causes of glomerulonephritis throughout the
Malaria and schistosomiasis (closely followed by: HIV,
world [save for subacute bacterial endocarditis in the
chronic hepatitis Band C)
Western hemisnhere)
Poststreptococcal GN (PSGN)
Prototypical for acute endocapillary proliferative GN
PSGN due to impetigo develops 2-6 weeks afer skin
infection and 1-3 weeks after streptococcal pharyngitis

M types 47, 49, 55, 2, 60, and 57


Streptococcus pyogenes strains associated with impetigo *Nephritogenicity is mainly restricted to certain M protein
serotypes

M types 1, 2, 4, 3, 25, 49, and 12


Streptococcus pyogenes strains associated with pharyngitis *Nephritogenicity is mainly restricted to certain M protein
serotypes
Kidneys have subcapsular hemorrhages with a "flea-bitten"
Endocarditis-associated GN
annearance
Primary treatment for endocarditis-associated GN Eradication of the infection with 4-6 weeks of antibiotics

Immune-mediated glomerulonephritis characterized by


thickening of the GBMwith mesangioproliferative changes; Membranoproliferative GN (MPGN)
70% of patients have hypocomplementemia

Presence of subendothelial deposits; low C3


Type I MPGNcharacteristics
Has double contour sometimes called tram-trackin~
lntramembranous deposits, ribbon-like pattern, lgG
Type II MPGNcharacteristics autoantibody; low C3
Dense deposit disease

Most proliferative of the three types of MPGN Type I MPGN

Glomeruli located at the corticomedullary junction (if the


Pathologic changes of FSGSare most prominent in renal biopsy specimen is from superficial tissue, the lesions
can be missed)
Has the highest reported incidences of renal vein
thrombosis, pulmonary embolism, and deep vein Membranous Nephropathy (MGN)
thrombosis
Sensitive indicator for the presence of diabetes but
correlates poorly with the presence or absence of clinically Thickening of the GBM
significant neohrooathv
Earliest manifestation in ~40% of patients with diabetes Increase in albuminuria detected by sensitive
who develop diabetic nephropathy radioimmunoassay

Potent risk factor for cardiovascular events and death in


Microalbuminuria
patients with type 2 diabetes
Fibrillar deposits ofimmunoglobulin light chains known
Most renal amyloidosis is the result of as amyloid L (AL), or secondary to fibrillar deposits of
serum amyloid A {AAl protein fra2:ments
Lesion in HIV-associated nephropathy FSGS

MGN: more common in children


GN caused by Hepatitis B
MPGN: more common in adults

Schistosoma species most commonly associated with Schistosoma mansoni


clinical renal disease Note: Schistosoma haematobium causes bladder disease
Nephrotic versus Nephritic Syndrome
Mnemonic·EPAL
Edema
Signs and symptoms of nephrotic syndrome Proteinuria >3.Sg/day(patho) or >3.0g/day(IM)
HypoAlbuminemia
HyperLipidemia (hypercholesterolemia)

• MCD
• MGN
Diseases presenting with nephrotic syndrome • DM Nephropathy
• Renal Amyloidosis
• FSGS

Most common cause of nephrotic syndrome in Children MCD

Most common cause of nephrotic syndrome in the elderly. MGN

Mnemonic: OHHA
• Oliguria
Signs and symptoms of nephritic syndrome • Hematuria
• HPN
• Azotemia

• PSGN
• Rapidly-Progressive GN (RPGN)
Diseases presenting with nephritic syndrome • Goodpasture syndrome
• Alport syndrome
• Lupus nephropathy

Summary of Nephrotic Glomerular Diseases

Minimal Change • Effacement of foot processes


Disease (MCD) • Highly•selective proteinuria, good response to steroids, excellent prognosis

• Idiopathic but may be caused by SLE, hepatitis 8, syphilis, gold, penicillamine, malignancy
Membranous
• Diffuse capillary and BM thickening. "spike & dome appearance" with subepithelial lgG and
Glomerulonephritis C3 deposits
(MGN)
• Non-selective proteinuria, poor response to steroids, indolent course

• Microangiopathy leading to BM thickening


DM Nephropathy • Some patients also develop eosinophilic, PAS+ nodules called nodular glomerulosclerosis or
Kimmelstiel-Wilson nodules

Focal segmental
glomerulosclerosis • Deposition of hyaline masses (hyalinosis) leading to obliteration of capillary lumina
(FSGS)

• Subendothelialfmesangial amyloid deposits


Renal Amyloidosis
• Apple-green birefringence on Congo red stain
Summary of Nephritic Glomerular Diseases
• Caused by GABHS;marked by high ASO titer
Post-Streptococcus • Most common pediatric cause of nephritic syndrome
Glomerulonephritis • Hypercellular glomeruli, "lumpy-bumpy" deposits of lgG and C3, subepithelial humps on
(PSGN) electron microscopy
• Self-resolving

• Poststreptococcal etiology in 50%


Rapidly- • Renal failure within weeks or months
Progressive • Type I RPGN:anti-GBM antibody-induced disease
Glomerulonephritis • Type II RPGN: immune complex-mediated disease
(RPGN) • Type Ill RPGN: pauci-immune type
• Rupture of the basement membrane seen in the EM

• Anti-GBM antibodies and anti-alveolar BM antibodies


Goodpasture
• Linear pattern oflgG on IF
Syndrome
• Hematuria and hemoptysis

• Appears before age 20


• Hereditary structural defect in collagen IV
Alport Syndrome
• Leaky BM (BM splitting on electron microscope)
• Hematuria, hearing loss, blindness

High-Yield Concepts in Lupus Nephritis


Correlate best with the presence of renal disease in lupus
Anti-dsDNA
nephritis

The only reliable method of identifying the morphologic


Renal biopsy
variants of lupus nephritis

Has the most varied course of lupus nephritis Class Ill Nephritis

Describes global, diffuse proliferative lesions involving the


vast majority of glomeruli; Most common and most severe Class IV Nephritis
type; wire-loop lesions

Has the worst renal prognosis without treatment Patients with crescents on biopsy

Predisposed to renal-vein thrombosis and other thrombotic


Class V Nephritis
complications
Diseases with Prominent Renal Vascular Injury
• Prominent systemic symptoms
Henoch-SchOnlein purpura is distinguished clinically • Younger age (<20 years old)
from lgA nephropathy by • Preceding infection
• Abdominal complaints

• Berger's Disease
• lgA deposits in the mesangium
lgA nephropathy • Usually follows infection
• Mesangial cell proliferation
• Most common type of glomerulonephritis worldwide

• Recurrent episodes of macroscopic hematuria often


accompanied by proteinuria during or immediately
Two most common presentations of lgA nephropathy following an upper respiratory infection, OR
• Persistent asymptomatic microscopic hematuria
More common in granulornatosis with polyangiitis
Anti-PR3 antibodies
[Weeener'sl
More common in microscopic polyangiitis or Churg-
Anti-MPO antibodies
Strauss
• Necrotizing granulomas of the respiratory tract
Triad ofWegener's granulomatosis • Necrotizing or granulomatous vasculitis
• Focal necrotizing, or often crescentic glomerulonephritis
Classically present with fever, purulent rhinorrhea, nasal
ulcers, sinus pain, polyarthralgias/arthritis, cough,
Characteristics ofWegener's granulomatosis
hemoptysis, shortness of breath, microscopic hematuria,
and 0.5-1 e oroteinuria oer 24 h
Following an endovascular procedure with manipulation of
Common cause of cholesterol emboli
the aorta
Prototypes of Microangiopathic Hemolytic Anemia Hemolytic Uremic Syndrome (HUS) and
(MAHA) Thrombotic Thrombocytopenic Purpura (TTP)
Shiga toxin-producing Escherichia coli (STEC) HUS,which is
Most common variant of HUS also known as D+ (diarrhea-associated) HUS or
enterohemorrhagic E.coli (EHEC) HUS
Most common Shiga-toxigenic £. coli (STEC)strain E.coli 0157:H7

Most severe manifestation of scleroderma characterized by


Characteristics of scleroderma renal crisis (SRC) accelerated hypertension, rapid decline in renal function,
nephrotic proteinuria, and hematuria
Anti RNA polymerase I and Ill
Autoantibody associated with development of SRC
Anti-U3-RNP may identify young patients at risk for
scleroderma renal crisis
Renal lesion in SRC Onion-skinning

First-line therapy in SRCunless contraindicated Treatment with ACE inhibitor

Reduce SBP by 20 mmHg and DBP by 10 mmHg every 24


Goal in SRC
hours until BP normalized

• Anticardiolipin (aCL): lgG, lgM, or lgA


Antiphospholipid antibodies are mainly • Lupus anticoagulant (LA)
• Anti-B-2 glycoprotein I (antlB2GPI)

HELLPSYNDROME(Hemolysis, Elevated Liver Enzymes,


Third trimester
Low Platelets) commonly occurs in

More commonly involved in renal vein thrombosis (RVT) Left renal vein
High-Yield Concepts in Polycystic Kidney and Tubulointerstitial Diseases
Most common renal abnormality in
Angiomyolipomas
tuberous sclerosis
Gitelman's syndrome is distinguished
Severe hypomagnesemia
from most forms of Bartter's syndrome
Hypocalciuria
bv the oresence of
Acute TIN most often presents with Acute renal failure

Predominant pathology in chronic TIN Interstitial fibrosis

Hailmark feature of TIN with uveitis Painful anterior uveitis

• Due to mutations affecting any of five ion transportproteins in the TAL


Bartter's syndrome
• Clinical syndrome mimics the effects of chronic ingestion of a loop diuretic
• Due to mutations in the thiazide-sensitive Na-Cl co-transporter (NCCT) in
Gitelman's syndrome the OCT
• Mimics the effects ofthiazide diuretics
Mimics a state ofaldosterone excess with early and severe hypertension,
Liddle's syndrome hypokalemia and metabolic alkalosis, but plasma aldosterone and renin
levels are low
• Saturnine gout
Triad of heavy metal (lead)
• Hypertension
nephropathy
• Renal insufficiency
Results from the long-term use of compound analgesic preparations
Analgesic nephropathy
containing phenacetin, aspirin, and caffeine

Interstitial fibrosis & tubular atrophy out of proportion to degree of


Renal biopsy of chronic TIN
glomerulosclerosis or vascular disease

High-Yield Concepts in Nephrolithiasis


Normal points of narrowing in the urinary • Ureteropelvic and ureterovesical junctions
tract and common sites of stone • Bladder neck
obstruction: • Urethral meatus
Most common type of urolithiasis Calcium oxalate stones
Associated with Proteus mirabilis, forming
Struvite stones (magnesium ammonium phosphate stones)
stae:horn calculi
Pathogenesis of struvite stones Bacterial ureaase production
• UTI
Urologic intervention should be postponed • A low probability of spontaneous stone passage ( e.g., a stone measuring
unless there is evidence of i?:6mm or an anatomic abnormality),
• Intractable oain
RadiopaquE:_~toneson standard x-rays Calcium, cystine, and struvite stones

Radiolucent stones on standard x-rays Uric acid stones


The two main risk factors for uric acid
Persistently low urine pH and higher uric acid excretion
stones
Recommended target urine pH in uric acid
pH6.5
stones
For all stone types, target urine
Alleasl 2 L/day(H4]
volume/dav
Increasing cystine solubility
Achieved by treatment with medication that covalently binds to cystine
Focus for cystine stone prevention
(tiopronin or penicillamine) and a medication that raises urine pH.
Tiooronin is the ureferred choice due to its better adverse event urofile.
Treatment for struvite stones Complete removal by a urologist
High-Yield Concepts in Urinary Tract Infections (UTI)
• Recent use of a diaphragm with spermicide
Independent risk factors for acute cystitis • Frequent sexual intercourse
• History ofUTI

• Frequent sexual intercourse


• A new sexual partner
Factors independently associated with pyelonephritis • UTI in the previous 12 months
in young healthy women • Maternal history of UTI
• Diabetes
• lincontinence

E. coli accounts for 75-90% of isolates (Mnl:ml!nil;:


Common etiologic agents in acute uncomplicated
KEPS- Klebsie/la, E.coli, Proteus, Staphylococcus
cystitis
saprophyticusJ, Enterococcus, and Citrobacter species

• Dysuria
Typical symptoms of cystitis • Urinary frequency
• Urgency

Generally an indication that the upper urinary tract is


Unilateral back or flank pain
involved

Indication of invasive infection of either the kidney or


Fever
the prostate

Fever (fever of pyelonephritis exhibits a high, spiking,


Main feature distinguishing cystitis and pyelonephritis "picket•fence" pattern & resolves over 72 h of
treatment)

• Nitrofurantoin
Drugs considered relatively safe for UTI in early
• Penicillin
pregnancy
• Cephalosporins

The standard of care for pregnant women with overt Parenteral beta-lactam with or without
pyelonephritis aminoglycosides

Treatment of asymptomatic bacteriuria (ABU)does not • Pregnant women


decrease the frequency of symptomatic infections or • Persons undergoing urologic surgery
complications, EXCEPTin • Neutropenic patients and renal transplant recipients

Most common cause of bilateral hydronephrosis in


Posterior urethral valves
boys

Most common cause of urinary tract obstruction in


Acquired defects
adults

Pathognomonic ofvesicoureteral reflux Flank pain that occurs only with micturition
PART 51ENDOCRINOLOGY
ENDOCRINEPHYSIOLOGY
KEYPOINTSABOUT HORMONES

HORMONE KEY POINTS

• GHRH GH IGF-1
GH
• Increases blood glucose, has direct and indirect effects (via IGF-1)
CYJ~
• Hypothalamic Dopamine (PIH)~ inhibits Prolactin
Prolactin
• Milk production, inhibits GnRH A'I~~
Oxytocin • Milk secretion, pregnant uterine contraction

Vasopressin • Insertion of Aquaporin at the renal collecting ducts, vasoconstriction ( increases TPR)
/ADH • Greater effect on plasma osmolarity than aldosterone

• TRH TSH --> T4(more common), T3 (more active)


Thyroid • increases BMR(via stimulation of NA-K-ATPase pump), increases cerebration, causes maturity of
Hormone CNS in the perinatal period, stimulates Bl receptors in the heart, needed for GH to work properly,
increases Glu absorption, glycogenolysis, gluconeogenesis, lipolysis

• Trigger: HypoCa
• Source: Chief Cells, Parathyroid Gland
• Counter-regulatory Hormone: Calcitonin (C Cells/ Parafollicular Cells, Thyroid Gland)
PTH
• increases Bone Calcium and Phosphate resorption, increases renal calcium reabsorption in the DT,
decreases renal phosphate reabsorption in the PCT, increases active Vit D by increasing 1 alpha
hydroxylase

• Renin ---+Angiotensin I ---+Angiotensin II ---+Aldosterone (ZG,AC)


• Renal: Increases Na+ reabsorption (increases ENaCs), K+ Secretion, H+ secretion
Aldosterone
• Others: increases Na reabsorption in the sweat glands and salivary gland, increases K+ influx in
other cells

• CRH ACTH Cortisol (ZF [largest area in the AC],AC)


Cortisol • ·Anti-inflammatory effects (e.g. inhibits phospholipase A2), increases blood sugar, increases
lipolysis, has multiple other organ effects

Weak • DHEA,Androstenedione (ZR, AC): significant only in females


Androgens

• Source: Beta Cells, Islets of Langerhans, Endocrine Pancreas


• 2nd Messenger: Tyrosine Kinase
• Precursor: proinsulin (splits into insulin and C peptide)
Insulin • Increases well-fed state pathways: glycolysis, lipid and protein synthesis
• Increases K+ influx in other cells
• GLUT-4:only one that is insulin-mediated
• Counter-regulatory Hormone: Glucagon (Alpha Cells)
High-Yield Physiology Concepts in Endocrinology
Hormones derived from proopiomelanocortin (POMC) MSH, ACTH, 8-Lipotropin, 8-Endorphin

Main site of ADH/vasopressin synthesis Supraoptic nuclei of the anterior hypothalamus

Para ventricular nuclei of the anterior hypothalamus


Main site of oxytocin synthesis
(Mnemonic: PARA sa Voovs!)

Site of oxytocin & ADH/vasopressin storage and


Posterior pituitary
secretion

From outer to inner: G-F-R


Zona .G.lomerulosa {Aldosterone secretion}
3 parts of the adrenal cortex
Zona fasciculata (Cortisol secretion)
Zona .Beticularis (Weak androgen secretion)

2 products of the adrenal medulla Epinephrine (80%), norepinephrine (20%)


Decreases calcium and phosphate excretion but
increases urinary calcium; increases intestinal calcium
VitaminD
absorption; calcium deposition at RDAlevels and calcium
resorption at high toxic levels
Decreases calcium excretion and increases phosphate
excretion; increases 1-alpha hydroxylase; increases bone PTH
resorption

Stimulated by LH,releases the "libido" hormone LeydigCells


testosterone (Mnemonic: LLL:LH, Leydig, Libido Hormone)
Sertoli Cells
Stimulated by FSH, nurse cell for sperm
(Mnemonic: SSS: FSH, Sertoli Cells, Sperm)
Causes ovulation Estrogen-induced LHsurge

Promotes growth of the fetus and insulin resistance and Human chorionic somatomammotropin (HCS)
lipolysis in the mom formerly known as HPL (human placental lactogen)

Prevents pregnancy during breastfeeding Inhibition of GnRH by prolactin

ENDOCRINEPATHOLOGY
High-Yield Concepts in Pituitary Pathology
Most common pituitary hormone hypersecretion
Hyperprolactinemia
svndrome in both men and women
Most common cause of pituitary hormone hypersecretion
Pituitary adenomas
and hyposecretion syndromes in adults
Most common type of pituitary adenoma and are usually
macroadenomas at the time of diagnosis because clinical Nonfunctioning pituitary adenomas
features are not aooarent until tumor mass effects occur
The most common cause of GHRH-mediated acromegaly Chest or abdominal carcinoid tumor

Postpartum necrosis of anterior pituitary gland Sheehan syndrome


presenting as sudden cessation of lactation
Headache, diplopia and hypopituitarism, caused by
hemorrhage into a pituitary adenoma
Pituitary apoplexy
Endocrine emergency that may result in severe
hypoglycemia, hypotension and shock, central nervous
system (CNS)hemorrha~e. and death
High-Yield Concepts in Thyroid Pathology
Th rmd Malignancies
Papillary Thyroid CA Most common type; good prognosis; Orphan Annie nuclei

Follicular Thyroid CA Capsular and/or vascular invasion


Derived from C cells, MEN-associated
Medullary Thyroid CA
Elevated serum calcitonin rovides a marker of residual or recurrent disease
Anaplastic Thyroid CA Giant cells and highly pleomorphic cells poor prognosis
Other Thyroid Gland Disorders
Autoimmune hypothyroidism that may be associated with a goiter; with
germinal centers, fibrosis, atrophy of the thyroid follicles accompanied by Hashimoto's thyroiditis
ox hil meta lasia, absence of colloid
Common cause of hypothyroidism worldwide Iodine deficiency
Presents with an insidious, painless goiter with local symptoms due to
compression of the esophagus, trachea, neck veins, or recurrent laryngeal
nerves
Dense fibrosis disrupts normal gland architecture and can extend outside the Riedel thyroiditis
thyroid capsule
Goiter is hard, nontender, often asymmetric, and fixed, leading to suspicion of
a mali nanc
Presents with hyperthyroidism, ophthalmopathy, dermopathy Graves' disease

Histopathologic finding of Graves' disease

Most common primary thyroid cancer


fapillary carcinoma of thyroid
Somatostatinorna
M.eningioma
Associated disease with PSaMMOMa Bodies Malignant Mesothelioma
.Qvarian serous papillary
cystadenocarcinoma
Prolactinoma Milk

High-Yield Concepts in Parathyroid Pathology


Most common cause of primary hyperparathyroidism Parathyroid adenorna

PTH hypersecretion no longer responsive to medical therapy and


Tertiary hyperparathyroidism
a state of severe HPT in patients with CKDthat requires surgery

Elevated PTH occurring in CKDpatients Secondary Hyperparathyroidism

High-Yield Concepts in Adrenal Pathology


Difficult-to-treat hypertension associated with hypokalemia Hyperaldosteronism
More common than unilateral adrenal adenomas as a cause of
Bilateral micronodular hyperplasia
mineralocorticoid excess
Acute primary adrenal insufficiency due to adrenal hemorrhage
Waterhouse-Friderichsen Syndrome
associated with septicemia

Common cause ofWaterhouse-Friderichsen syndrome Neisseria meningitidis

Neoplasms composed of chromaffin cells, which synthesize and


Pheochromocytoma
release catecholamines and in some instances peptide hormones
DISEASESOF THE ENDOCRINESYSTEM
High-Yield Concepts Related to the Hypothalamus-Pituitary Axis
Tropic hormone failure associated with pituitary
compression or destruction usually occurs in this GH > FSH > LH > TSH >ACTH
sequence
Most common presentation of tropic hormone failure
Growth retardation
in childhood
Earliest symptom of tropic hormone failure in the adult Hypogonadism
Cut-off size for microadenoma < 10mm diameter
Most common mechanism where suprasellar extension
Compression of the optic chiasm
can lead to visual loss

Surgical approach for most pituitary tumors Transsphenoidal surgery

Treatment of choice for prolactinomas Dopamine agonists (Cabergoline and Bromocriptine)

Hallmarks of hyperprolactinemia Amenorrhea, galactorrhea, infertility


Most abundant anterior pituitary hormone and major
GH
determinant of hepatic IGF-1synthesis
Major source of circulating IGF-1 Liver
Most validated test to distinguish pituitary-sufficient
Insulin-induced hypoglycemia
patients from AGHD
Confirmed by demonstrating the failure ofGH
suppression to <0.4 g/L within 1-2 h of a 75g oral Acromegaly
glucose load
Most significant cause of mortality in patients with
Cardiovascular
acromegaly
Provides a useful screening measure when clinical
IGF-IIevel
features raise the possibility of acromegaly
Initial treatment for most patients with acromegaly Surgical resection of GA-secreting adenomas
Side effects associated with somatostatin analogues
Suppression of gastrointestinal motility and secretion
used in acromegaly

Drug that acts by competitive inhibition of GH Pegvisomant


Central redistribution of fat
Features that make pathologic causes of
Thin skin with striae and bruising
hypercortisolism more likely
Proximal muscle weakness
Primary cause of death in Cushing's syndrome Cardiovascular disease
Precise and cost-effective screening test for Cushing's
Measurement of24-h urine free cortisol
syndrome
Distinguish patients with ACTH-independent (adrenal
or exogenous glucocorticoid) from ACTH-dependent Basal plasma ACTHlevels
(pituitary, ectopic ACTH) Cushing's syndrome
Size of most ACTH-secreting pituitary tumors <5 mm in diameter (half undetectable by MRI)

May be required to distinguish pituitary tumors from Bilateral inferior petrosal sinus ACTHsampling before
ectopic ACTH-secreting tumors and after CRHadministration
Harbinger of hypothalamic or pituitary lesions that
impair GnRH production or delivery through the Hypogonadism
pituitary stalk
Origin of most nonfunctioning adenomas Gonadotrope cells
Most important, if not the only, physiologic action of
Reabsorb water and promote concentration of urine
AVP
Syndrome characterized by the production of
abnormally large volumes of dilute urine (24-hour
Diabetes insipidus (DI)
urine volume >40 mL/kg body weight and osmolarity
<300 mosmol/L)
Excessive urination secondary to insensitivity of the
Nephrogenic DI
renal tubules to ADH
Excessive urination secondary to lack of ADH Central DI
Excess ADH causing excessive resorption of water and
SIADH
hyponatremia
Simpler, and less stressful, but equally reliable way to
Measuring basal plasma AVP and urine osmolarity
differentiate between pituitary DI, nephrogenic DI, and
under conditions of unrestricted fluid intake
primary polydipsia

High-Yield Concepts Related to the Thyroid Gland


'
Start of thyroid hormone synthesis About 11 weeks AOG

jlsubunit
Subunit unique to TSH
(a subunit common to LH, FSH, hCG)

Transporter involved in iodide trapping Na-Iodide Symporter (NIS)

Transporter involved in moving iodide from follicular cell


Pendrin
to follicular lumen

Hypothyrmdism

Most common cause of preventable mental retardation Iodine deficiency

Hypothyroidism in infancy or early childhood Cretinism

Importance of diagnosing thyroid hormone resistance To avoid inappropriate treatment of mistaken


hyperthyroidism and to provide genetic counseling

Method of choice in determining thyroid size accurately Ultrasound

First test in the approach to thyroid testing TSH Levels

Detected most easily by measuring circulating antibodies


Autoimmune thyroid disease
against TPO and Tg
Used to predict both fetal and neonatal thyrotoxicosis
caused by transplacental passage of high maternal levels
TRAbassay
ofTRAb or TSI (>3x upper limit of normal) in the last
trimester of ore1mancv.
Most common symptom of hypothyroidism Tiredness, weakness

Most common sign of hypothyroidism Dry coarse skin, cool peripheral extremities

Least common symptom of hypothyroidism Impaired hearing

Least common sign of hypothyroidism Serous cavity effusions


Best documented risk factors for autoimmune HLA-DR polymorphisms (especially HLA-DR3, -DR4
hvoothvroidism and-DRSl
Thyroid cell destruction in autoimmune thyroiditis is CDO+cytotoxic T cells
primarily mediated by

Major role in pathogenesis in myxedema coma, leading to


Hypoventilation
hypoxia and hypercapnia
Indication for external warming in hypothermia of
Temperature is <302C
ede
Hyperthyrod1s1m
Graves disease, toxic multinodular goiter, toxic
Major etiologies of hyperthyroidism
adenoma
Most common symptom of hyperthyroidism Hyperactivity, irritability, dysphoria

Most common sign of hyperthyroidism Tachycardia, atrial fibrillation in the elderly

Least common symptom of hyperthyroidism Oligomenorrhea, loss of libido

Least common sign of hyperthyroidism Gynecomastia


Minor risk factor for Graves disease and major risk factor
Smoking
for Graves ophthalmopathy
Apathetic thyrotoxicosis in elderly present as Fatigue and weight loss
Most common cardiovascular manifestation of
Sinus tachycardia (associated with palpitations)
hyperthyroidism
Sensation of grittiness, eye discomfort, and excess
Earliest manifestations of Graves ophthalmopathy
tearine:
In Graves ophthalmopathy, muscle swelling is so severe
that diplopia results, typically, but not exclusively, when Up and laterally
patient looks
Most serious manifestation of Graves ophthalmopathy Compression of optic nerve at apex of the orbit
Anterior and lateral aspects of the lower leg (Pretibial
Most frequent site of thyroid dermopathy
mvxedemal
Time of major risk for relapse in Graves disease in
Postpartum period
oree:nancv
Duration of Carbimazole or methirnazole - free period
2-3 days before
prior to radioiodine therapy
Longer period before radioiodine is given (no
Duration of PTU-free period prior to radioiodine therapy
mention of how lon~l
Absolute contraindications to radioiodine Pregnancy and breastfeeding
Most common cause of acute thyroiditis in children and Presence of a piriform sinus (predominantly left-
young adults sided)
Most common clinically apparent cause of chronic
Hashimoto thyroiditis
thvroiditis
Major cause of sick euthyroid syndrome Release of cytokines

Decrease in total and unbound T3 levels [low T3


Most common pattern of sick euthyroid syndrome
syndrome) with normal levels ofT4 and TSH

Clinical manifestations of most goiters Asymptomatic


Venous distention over the neck and difficulty breathing
especially when the arms are raised (in large retrosternal Pemberton sign
goiters)
Most frequent cause of acquired hypoparathyroidism in
Surgery for hyperthyroidism
the oast
Hypoparathyroidism now usually occurs after Surgery for hyperparathyroidism
Multiple Endocrine Neoplasia (MEN) Syndromes
MEN2
MEN l MENZA MEN2B MEN4
• Parathyroid • Medullary thyroid carcinoma Also known as MEN3 • Parathyroid
hyperplasia/adenoma • Pheochromocytoma • Medullary thyroid adenoma
• Pancreatic islet cell carcinoma • Pituitary adenoma
• Parathyroid hyperplasia/adenoma
hyperplasia/adenoma/carcinoma • Pheochromocytoma • Reproductive
MEN 2A with cutaneous lichen
• Pituitary hyperplasia/adenoma amvloidosis • Mucosa! and organ tumors (e.g.,
MEN 2A with Hirschorum(s disease gastrointestinal testicular cancer,
Other less common manifestations: neuromas neuroendocrine
fore9ut carcinoid, • Marfanoid features cervical
pheochromocytoma, subcutaneous Familial medullary thyroid carcinoma)
or visceral lipomas carcinoma • Adrenal + renal
tumors

High-Yield Concepts Related to Thyroid Malignancies


Sonographic characteristics of thyroid nodules suggestive of Hypoechoic solid nodules with infiltrative borders
malignancy and microcalcifications, >90% cancer risk

Most common malignancy of the endocrine system Thyroid CA

Most common type of thyroid cancer Papillary Thyroid Carcinoma

More common in iodine-deficient regions Follicular Thyroid Carcinoma

Surgical treatment in almost all patients with well-


Near-Total Thyroidectomy
differentiated cancer

Most common type of thyroid lymphoma Diffuse large-cell lymphoma

Primary management of medullary thyroid carcinoma Surgery

Identify in a cost-effective manner the small


Main goal in the approach to thyroid nodule
subgroup of malignant lesions

Size of most palpable thyroid nodules >tern

High-Yield Concepts Related to the Adrenal Cortex


Cushmg's Syndrome

Constellation of clinical features that result from chronic


Cushing's syndrome
exposure to excess glucocorticoids of any etiology

In the overwhelming majority of patients, Cushing


ACTH-producing pituitary adenoma (Cushing disease]
syndrome is caused by

Medical use of glucocorticoids for inflammatory or


Most common cause of Cushing syndrome overall
immunosuppressive treatment

Ectopic ACTHproduction is predominantly caused by Occult carcinoid tumors (usually lung]

Majority of patients with ACTH-independent cortisol


Cortisol-producing adrenal adenoma
excess caused by
Most prominent features in Cushing's syndrome are Upregulation of gluconeogenesis, lipolysis, and
caused by protein catabolism

Trying to stand up from a chair without the use of


Signs of proximal myopathy become most obvious when
hands or when climbing stairs

Majority of patients experience psychiatric symptoms


Anxiety or depression
mostly in the form of

Most important first step in the management of suspected


Establish the correct diagnosis
Cushing's syndrome

Investigation of choice in ACTH-dependent cortisol excess MRIof the pituitary

Oral agents with established efficacy in Cushing's


Metyrapone and Ketoconazole
syndrome

Hyperaldosteromsm and Conn Syndrome

Primary hyperaldosteronism secondary to aldosterone-


Conn syndrome
secreting adrenal adenoma

Clinical hallmark of mineralocorticoid excess Hypokalemic hypertension

Concurrent measurement of plasma renin and


Accepted screening test for primary hyperaldosteronism aldosterone with subsequent calculation of the
aldosterone-renin ratio (ARR)

Most straightforward test for primary


Saline infusion test
hyperaldosteronism

Imaging of choice for hyperaldosteronism Fine-cut CT scanning of the adrenal region

Preferred approach for unilateral lesions Laparoscopic adrenalectomy

Medical treatment for hyperaldosteronism Spironolactone

Majority of adrenal nodules Inactive adrenocortical adenomas

Metastasis in ACCmost frequently occurs to Liver and lung

Characteristics of benign adrenal lesions Rounded and homogenous

Characteristics of malignant adrenal lesions Lobulated and inhomogeneous

Most common histopathologic classification for


Weiss Score
adrenocortical carcinoma

Highly sensitive for the detection of malignancy and can


be used to detect small metastases or local recurrence 18-FDG PET
that may not be obvious on CT
Site of metastasis in adrenocortical carcinoma Liver and lung

• Capsule violation during primary surgery


Major determinants of poor survival in adrenal carcinoma • Metastasis at diagnosis
• Primary treatment in a nonspecialist center
Adrenal Insufficiency
Characterized by the loss of both glucocorticoid and
Primary adrenal insufficiency (Al)
mineralocorticoid secretion
Only glucocorticoid deficiency is present Secondary Al
Most frequent origin of Al Hypothalamic-pituitary
Most common cause of primary Al Autoimmune adrenalitis
Excluding iatrogenic suppression, majority of secondary Pituitary or hypothalamic tumors, or their treatment
Al is caused b b su er or irradiation
Distinguishing feature of primary Al Hyperpigmentation
Characteristic feature in primary Al Hyponatremia (80%)
Diagnosis of Al is established by Short cosyntropin test
Monitoring of glucocorticoid replacement History and PE for signs and symptoms
• 1 mg hydrocortisone
• 1.6 mg cortisone acetate
Equipotency of steroids can be assumed for • 0.2 mg prednisolone
• 0.25 mg prednisone
• 0.025 m dexamethasone

Classic triad in pheochromocytoma

Dominant sign in pheochromocytoma


• 10% are bilateral
"Rule of 10s" in pheochromocytoma
• 10% are extraadrenal
• 10% are mali nant
First step in diagnosis ofpheochromocytoma Measurement of catecholamines
Elevated plasma & urinary catecholamines and
Cornerstone for the diagnosis
metane brines
Most tumors continuously leak this metabolite, which are
0-methylated metabolites
detected b measurements of metane hrines
Most sensitive test which is less susceptible to false-
Measurements of plasma metanephrine
positive elevations from stress, including venipuncture

Ultimate therapeutic goal for pheochromocytoma Complete tumor removal

Before surgery, blood pressure should be Consistently <160/90 mmHg

Minimally invasive techniques (laparoscopy or


Standard approaches in pheochromocytoma surgery
retroperitoneoscopy)

First-described pheochromocytoma-associated syndrome Neurofibromatosis Type 1 (NF 1)


Multiple endocrine neoplasia (MEN) type 2A and type
Best-known pheochromocytoma-associated syndrome
2B
Summary of Hypercortisolism
.,
" '
Serum cortisol i i
''i
Urine free cortisol i i i
Low-dose
Cortisol not suppressed Cortisol not suppressed Cortisol not suppressed
dexamethasone
High-dose
Cortisol suppressed Cortisol not suppressed Cortisol not suppressed
dexamethasone
Plasma ACTH N to i i Markedlv i

High-Yield Concepts Related to the Parathyroid Gland and Calcium Homeostasis


Hyperparathyro1d1sm and Hypercalcem1a
Parathyroid hormone (PTH) from chief cells of
Primary regulator of calcium physiology
oarathvroid 2land
Maintain the extracellular Huid (ECF) calcium
Primary function of PTH
concentration within a narrow normal range
Major source of calcitonin Thyroid (C cells/parafollicular cells)

Most common cause of hypercalcemia Hyperparathyroidism

Second most common cause of hypercalcemia Malignancy


Inadvertent hemoconcentration during blood
False-positive hypercalcemia is usually the result of collection or elevation in serum proteins such as
albumin
Serve as the principal laboratory test in establishing the
lmmunometric PTH assays
diagnosis of hypercalcemia
Bones (bone pain), Groans (abdominal pain),
Symptoms of hypercalcemia Psychiatric Overtones (decreased sensorium,
nsvchosisl
Severe hypercalcemia and medical emergency V 3.7-4.5 mmol/L or 14.8-18 mg/dL
Isolated adenomas without other
Most often presentation of parathyroid tumors
endocrinopathv
Most common location of parathyroid adenomas Inferior parathyroid glands

Primary manifestations of hyperparathyroidism involve the Kidneys and the skeletal system

Most prevalent form of hyperparathyroidism Asymptomatic

Definitive therapy for hyperparathyroidism Surgical excision of abnormal parathyroid

Responsible humoral agent in most solid tumors that cause


PTH-related protein (PTHrP)
hypercalcemia

Treatment of hypercalcemia of malignancy Control of tumor

Striking feature of malignancy-associated hypercalcemia Rapidity of the course

First principle of treatment of hypercalcemia Restore normal hydration

Treatment of choice for severe hypercalcemia complicated by


Dialysis
renal failure
Tapping along facial nerve induces contractions of eye, mouth,
Chvostek sign
or nose muscles
Carpal spasms produced by occlusion of the circulation to the
Trousseau sign
forearm
Osteoporosis
Bone density that falls 2.5 standard deviations (SD) below the
mean for young healthy adults of the same sex and race (T- WHO definition of osteoporosis
score)
Compare individual results to those in a young population that
T-scores
is matched for race and sex
Compare individual results to those of an an age and gender-
Z-scores
matched reference population

Chief clinical manifestations of osteoporosis Vertebral and hip fractures

Final common pathway in osteoclast development & activation Activation of RANKby RANKL
Time when resorption and formation processes become
After age 30-45
imbalanced, wherein resorption exceeds formation
Cessation of ovarian function at the time of
Most common estrogen-deficient state
menopause (average at age 51)
Sites where trabecular bone contributes most to
Fractures occur earliest at these sites
bone strenl(th
Most common early consequence of estrogen deficiency Vertebral fractures

Most common cause of medication-induced osteoporosis Glucocorticoids

Highly accurate X-ray technique that is the standard for


Dual Energy X-Ray Absorptiometry (DXA)
measuring bone density

Sites of DXAdeterminations Lumbar spine and hip

Tend to falsely increase bone density of the spine and are a


Bone spurs
particular problem in measuring the spine in older individuals

Amenable for use as a screening procedure for osteoporosis Ultrasound

Guidelines further recommend that bone mass measurement


All women by age 65
be considered in
Indication for radiography or vertebral fracture assessment by
DXAto rule out asymptomatic vertebral fractures, as is the
Height loss >2.5-3.8 cm (>1-1.5 in.)
presence of significant kyphosis or back pain, particularly if it
be~an after menopause
Primary use of biochemical markers Monitoring response to treatment

Preferred source of calcium Dairy products and other foods

Calcium supplement best taken with food (since they require


Calcium carbonate
acid for solubility)

Calcium supplement taken anytime Calcium citrate


SERMapproved for the prevention and treatment of
Raloxifene
osteoporosis
SERMapproved for the prevention and treatment of breast Tamoxifen (but increases risk of uterine cancer
cancer in postmenopausal women)
Osteonecrosis of the jaw is found mostly in cancer patients
Bisphosphonates
given high doses of
Preferred site for bone mineral density (BMD) scan due to
Hip
lareer surface area and e:reater reoroducibilitv
Fully human monoclonal antibody to RANKL Denosumab

An exogenous PTH analogue Teriparatide (1-34hPTH)


By far the most common form of glucocorticoid-induced
Therapeutic use of glucocorticoids
osteooorosis
Affected more severely in glucocorticoid-induced osteoporosis Trabecular bone
Should have measurement of bone mass at both the spine and Patients on long-term (>3montbs)
hip using DXA glucocorticoids

Spine in individuals < 60 years


If only one skeletal site can be measured, it is best to assess the
Hip in individuals> 60 years

Demonstrated in large clinical trials to reduce the risk of


Bisphosphonates
fractures in patients being treated with glucocorticoids

" \[
Summary of Bone Disorders
Disease (al• PO, ALP PTH Notes
Osteoporosis
-- -- -- -- ! Bone mass, normal mineral
content
Osteopetrosis -- -- -- -- Thickened, dense bones

Osteomalacia/rickets
l l -- t Decreased mineral content of bone,
soft bones
Osteitis fibrosa cystica
t l t t "Brown tumors"

Paget's Disease -- -- t -- Abnormal bone architecture

High-Yield Concepts in Diabetes


Leading cause of ESRD,nontraumatic lower
DM
extremitv amoutations, and adult blindness
Most reliable and convenient tests for identifying OM
HbAlC orFPG
in asymptomatic individuals
Key regulator of insulin secretion Glucose

Glucose level that stimulates insulin synthesis > 70mg/dl (3.9mmol/L)


• Glucokinase (liver; higher Km, lower affinity, higher
Rate-limiting step that controls glucose-regulated Vmax)
insulin secretion • Hexokinase (everywhere else; lower Km, higher affinity,
lowerVmaxl
lncretin hormones that bind specific receptors on the
beta cell to stimulate insulin secretion through cyclic Glucagon-like peptide 1 (GLP-1) and glucose-dependent
AMP production, but have this effect only when the insulinotropic peptide (GIP)
blood elucose is above the fastine level
Major portion of postprandial glucose utilized by Skeletal muscle
Features of diabetes do not become evident until
70-80%
how much beta cells are destroyed
Major susceptibility gene in TlDM HLAregion on chromosome 6

Central to the development of T2DM Insulin resistance and abnormal insulin secretion

Predominantly accounts for increased FPG levels Increased hepatic glucose output

Results in postprandial hyperglycemia Decreased peripheral glucose utilization


All individuals >45 years old every 3 years; screening at
Screening for OM an earlier age if overweight (BMI >25) and have one
additional risk factor for DM
Time when glycemic control is achieved with modest
Honeymoon phase
doses of insulin
Kussmaul respirations and fruity odor (secondary to
Classic signs of diabetic ketoacidosis (OKA)
metabolic acidosis and increased acetone)
• Acetoacetate (excreted in the urine)
3 ketone bodies • fl-hydroxybutyrate
• Acetone (not used as source of energy)
Extremely serious complication of DKAseen most
Cerebral edema
frequently in children
Necessary for DKAto develop Both insulin deficiency and glucagon excess

Preferred method for detecting ketones that more


Serum or plasma assays for fl-hydroxybutyrate
accurately reflect the true ketone level
Synthesized at a 3-fold greater rate than acetoacetate
fl-hydroxybutyrate
in DKA
Preferentially detected by a commonly used ketosis
Acetoacetate
detection reae:ent (nitroorussidel
Consistent finding in DKAand distinguishes it from
Ketonemia
simple hyper~lvcemia
False-positive reaction with nitroprusside tablet or
Captopril or penicillamine
stick (test for DKAl
Careful monitoring and frequent reassessment to ensure
Central to successful treatment of DKA
that patient and metabolic derangements are improving
Acceptable potassium level wherein insulin drip can
Initial serum K•?. 3.3 meq/L
be started
Underlying cause of hyperglycemic hyperosmotic
Relative insulin deficiency and inadequate Ouid intake
state (HHS)
Elderly individual with type 2 DM, with a several-week
history of polyurla, weight loss, and diminished oral
Prototypical patient of HHS
intake that culminates in mental confusion, lethargy, or
coma.
Prominent features of both HHS and DKA Volume depletion and hyperglycemia

Confirms a patient's need for insulin Low C-peptide level


Symptoms of diabetes usually resolve when glucose
< 200 mg/dL (11.1 mmol/L)
is
Primary goal in treatment of adults with DM HbA1C<7.0%

150 min/week ( distributed over at least 3 days) of


Recommendations on exercise
moderate aerobic physical activity

Standard of care in diabetes management Self-monitoring of blood glucose

Standard method for long-term glycemic control Measurement of HbAlc

Primary predictor of long-term DMcomplications HbAlc


Can be used as an alternative indicator of glycemic
Albumin, or measurement of 1,5-anhydroglucitol
control when the HbAlc is inaccurate
Measurement of glycated albumin that reflects
Fructosamine assay
glycemic status over the prior 2 weeks
Glucose-lowering agents other than insulin are
ineffective in type 1 DM and should not be used for
With the exception amylin analogue
glucose management of severely ill individuals with
type 2 DM
Major toxicity of metformin Lactic acidosis

Major side effects of GLP-1 agonists Nausea, vomiting, and diarrhea


Major side effects of alpha-glucosidase inhibitors Diarrhea, natulence, abdominal distention

Most common side effects of bile acid-binding resins Gastrointestinal

Should not be used if hypertriglyceridemia is present Bile acid-binding resins

Most serious complication of therapy for DM Hypoglycemia

Predictor of poor outcome in hospitalized patients Hyperglycemia


Preferred in ICUor in a clinically unstable setting
Insulin infusions
(absorption of SC insulin is variable)
Preferred over insulin analogue for IV insulin
Regular insulin
infusion (less expensive and equally effective)
Preferred method for managing patients with type 1
DMin the perioperative period or when serious Insulin infusion
concurrent illness is oresent
Most crucial period of glycemic control in pregnancy Soon after fertilization

Type 1 vs Type 2 Diabetes Mellitus


Character1st1cs Type 1 TypeZ

Usually occurs in those <30 y.o. +

Postulated to occur due to autoimmune causes +

DKA is the most common complication +

Strong polygenic genetic predisposition +

HLA-DR3, -DR4 +

Depleted beta cells +

Islet leukocytic infiltrate +

Islet amyloid deposit +

High-Yield Concepts in DM Complications


Microvascular manifestations of DM Retinopathy, neuropathy, nephropathy

Coronary heart disease, peripheral arterial Disease,


Macrovascular manifestations of DM
cerebrovascular disease

Leading cause of blindness between ages 20 and 74 OM retinopathy

Progressive diabetic retinopathy and clinically


Severe vision loss is primarily the result of significant macular edema and new blood vessel
formation

Retinal vascular microaneurysms, blot hemorrhages


Non-proliferative DMretinopathy
and cotton wool spots
Hallmark of proliferative DMretinopathy Neovascularization in response to retinal hypoxia

Best predictors of development of retinopathy Duration of OM and degree of glycemic control

Duration ofDM in patients with non-proliferative


DM >ZOyears
retinopathy

Most effective therapy for DMretinopathy Prevention

Laser photocoagulation and/or anti-VEGF therapy


Treatment of proliferative retinopathy or macular edema
(ocular injection)

Albuminuria (The American Diabetes Association (ADA)


Increased urinary protein excretion (spot urinary
no longer uses the terms microalbuminuria or
albumin-to-creatinine ratio >30 mg/g Cr).
macroalbuminuria)

Nodular glomerulosclerosis or Kimmelstiel-Wilson


Eosinophilic, PAS+nodules in diabetic nephropathy
nodules

Optimal therapy of DMnephropathy Prevention by control of glycemia

Preferred therapy for DMnephropathy when the GFR


Renal transplantation from a living related donor
approaches 20 mL/min per 1.743 m2.

Pigmented pretibial papules or diabetic skin spots DM dermopathy

Most common site of foot ulcers Great toe or metatarsophalangeal (MTP) areas

Optimal therapy for foot ulcers and amputation Prevention

Most common site of ulceration Plantar surface of the foot

A wound that probes to the bone represents clinical


Osteomyelitis
evidence of

Most specific modality for osteomyelitis MRI of the foot

Prolonged antibiotics (IV then oral) and possible


Osteomyelitis best treated by
debridement of infected bone

Distal symmetric polyneuropathy (frequently


Most common form of diabetic neuropathy
presents with distal sensory loss and pain)

Most commonly involved nerve in mononeuropathy CN III (heralded by diplopia)

Most prominent GI symptoms in DM Delayed gastric emptying and altered small- and
large-bowel motility

Primary goal in gastrointestinal and genitourinary


Improved glycemic control
dysfunction in DM

Most common pattern of DMdyslipidemia Hypertriglyceridemia and reduced HDL


High-Yield Concepts in Hypoglycemia
Limiting factor in the glycemic management of
Hypoglycemia
diabetes
Drugs used to treat DM or by exposure to other drugs,
Most common cause of hypoglycemia
including alcohol
Serious illnesses such as renal, hepatic, or cardiac failure,
Second only to drugs as causes of hypoglycemia
sepsis, and inanition
1. Symptoms consistent with hypoglycemia
2. Low plasma glucose measured with a precise method
Whipple's Triad (in insulinoma)
(not a glucose monitor)
3. Relief of symptoms after the plasma glucose level is raised
• Plasma insulin concentration >3 uU/mL (>18 pmol/L),
Critical diagnostic findings when plasma glucose is • Plasma C-peptide concentration >0.6 ng/mL (>0.2
<55 mg/dL (with symptoms of hypoglycemia) nmol/L), and
• Plasma proinsulin concentration >5.0 pmol/L
Lower limit of the fasting plasma glucose
Approximately 70 mg/dL (3.9 mmol/L)
concentration
Obligate metabolic fuel for the brain under Glucose (may also use ketones in prolonged fasting after two
physiologic conditions weeks)
Major site of endogenous glucose production Liver
Hepatic glycogen stores are able to sustaine
Approximately -8 h
glucose needs for
Glycemic maintenance goals in hospitalized
140-180 mg/dL
patients

High-Yield Concepts Related to Hypoglycemia


First defense against hypoglycemia Decreased insulin
Second defense against hypoglycemia
- - Increased glucagon
Third defense against hypoglycemia Increased epinephrine
Loss of the warning adrenergic and cholinergic symptoms
that previously allowed the patient to recognize
Hypoglycemia unawareness
developing hypoglycemia and therefore to abort the
episode by ingesting carbohydrates
Overproduction of insulin-like growth factor II
Hypoglycemia in non-beta cell tumors is due to
('"biglGF-11"')
Prototypical cause of endogenous hyperinsulinism Insulinorna
Ingestion of an insulin secretagogue Causes hypoglycemia with increased C-peptide levels
Exogenous insulin Causes hypoglycemia with low C-peptide levels

High-Yield Concepts Related to the Gonads


Defined as the inability to conceive after 12 months of
unprotected sexual intercourse or after 6 months in Infertility
women>/ 35
Probability of achieving pregnancy in one menstrual cycle Fecundability
Emerging as the method of choice for testosterone Liquid chromatography tandem mass spectrometry
measurement (LC-MS/MS)
Most important step in the evaluation of male infertility Semen analysis
• Semen volume, 1.5 mL
• Total sperm number, 39 million per ejaculate
• Sperm concentration, 15 million per mL
WHO·generated reference limits for semen parameters • Vitality, 58% live
• Progressive motility, 32%
• Total (progressive+ non•progressive) motility, 40%
Morphologically normal forms, 4.0%.
Most common cause of androgen deficiency in acute
Hypogonadotropic hypogonadism
illness
Most common chromosomal disorder associated with
Klinefelter Syndrome (47 XXY)
testicular dysfunction and male infertility
Characterized by bilateral streak gonads, primary
Turner Syndrome ( 45 XO)
amenorrhea, short stature, and other phenotypic features
Glandular breast tissue that is >4 cm in diameter and
True gynecomastia
often tender
Most effective therapy if gynecomastia is of long duration Surgery

Most common cause of female infertility Abnormalities in menstrual function

Most widely used form of hormonal contraception Oral contraceptives


Midcycle pelvic discomfort that is thought to be caused by
the rapid expansion of the dominant follicle at the time of Mittelschmerz
ovulation
Precocious Puberty In Boys Before age 9

Delayed Puberty In Boys Absence of secondary sexual characteristics by age 14

Precocious Puberty In Girls ... Before age 8

Delayed Puberty In Girls ,<-....""-y Absence of secondary sexual characteristics by age 13

ENDOCRINOLOGY AND BIOCHEMISTRY


CORRELATION
Basic Biochemistry of Lipids
Long chain of carboxylic acid with no double bond Saturated fatty acid

Long chain of carboxylic acid with one double bond Monounsaturated fatty acid

Long chain of carboxylic acid with two or more double bonds Polyunsaturated fatty acid

Fatty acids associated with increased risk of atherosclerosis Trans-fatty acids and Saturated fatty acids

Essential fatty acids Linoleic acid and Linolenic acid

Immediate precursor of prostaglandins Arachidonic acid

End product of fatty acid synthesis Palmitic acid

2 primary bile acids


Cholic acid and Chenodeoxycholic acid
Starts with a letter "C": Primary)
(M.11.!m!.l!nil;:
2 secondary bile acids Oeoxycholic acid and Lithocholic acid

2 molecules conjugated to bile acids to convert them to bile salts Taurine and Glycine

Clinical manifestation of lipid malabsorption Steatorrhea

Spherical macromolecular complexes composed of a neutral lipid


core surrounded by a shell of amphipathic Jipoproteins, phospholipid, Lipoproteins
and non-esterified cholesterol

Protein moiety oflipoproteins Apoproteins

Transport dietary triglycerides and cholesterol from intestine to


Chylomicrons
tissues

Transports triglycerides from liver to tissues VLDL

Delivers cholesterol into cells LDL

Reverse cholesterol transport HDL

Shuttles Apo C-11and Apo E in the blood HDL

Mediates chylomicron secretion ,... ApoB-48

Activates lipoprotein lipase ApoC-11

Mediates uptake of chylomicron remnant ApoE

Binds to LDLreceptor and mediates VLDLsecretion Apo 8-100

Activates LCATto produce cholesteryl esters in HDL ApoA-1

Degradation of TAG stored in adipocytes Hormone sensitive lipase

Degradation of TAG from ingested lipids Pancreatic lipase

Degradation of TAG contained in chylomicrons in the capillaries of


Lipoprotein lipase
skeletal and cardiac muscle and adipose tissues

Dipalmitoylphosphatidylcholine
Major component of lung surfactant
(Lecithin)

Only glycerophospholipid that is antigenic cardiolipin

Reservoir for arachidonic acid in the membranes and precursor for


Phosphatidylinositol
1P3 and DAG

Important constituent of myelin Sphingomyelin


Diseases Involved in Lipid Metabolism
Alcohol leads to fat accumulation in the liver Fatty Liver

Cerebrohepatorenal syndrome Zellweger Syndrome

Accumulation of phytanic acid due to deficiency of alpha-hydroxylase Refsum Disease

Hypoglycin from unripe fruit of the akee tree inactivates medium- and short-
Jamaican Vomiting Sickness
chain acyl CoA dehydrogenase

Can present in childhood or adulthood with recurrent episodes of severe


abdominal pain due to acute pancreatitis
Familial chylomicronemia syndrome
Eruptive xanthomas, which are small, yellowish-white papules, may appear in
clusters on the back, buttocks, and extensor surfaces of the arms and legs

Generally a clinical diagnosis based on substantial hypercholesterolemia with


LDL-C>190 mg/dL in the absence of a secondary etiology, and a family history of Familial Hypercholesterolemia
hypercholesterolemia and/or premature coronary disease.

Accumulation of fat in intestinal enterocytes and hepatocytes, with deficiency in


Abetalipoproteinemia
fat-soluble vitamins and essential fatty acids

Mental retardation from accumulation ofGM2 ganglioside Tay-Sachs Disease

Mental retardation from accumulation ofsphingomyelin - Niemann-Pick Disease

Mental retardation with enlarged liver and spleen from accumulation of


Gaucher Disease
glucocerebroside

Diseases involved in Carbohydrate Metabolism


Flatulence, cramps, and diarrhea after ingestion of dairy products Lactose intolerance

Severe fasting hypoglycemia, hepatomegaly, elevated glycogen in Von Gierke Disease (Glucose 6-phosphatase
the liver deficiency)

Cardiomyopathy, hypotonia, exercise intolerance, and systemic Pompe ("Pump") Disease (Lysosomal acid maltase
findings lead to early death deficiency)

Hepatomegaly, milder form of Von Gierke Disease Cori Disease (Debranching enzyme deficiency)

McArdle Syndrome (Skeletal muscle glycogen


Myoglobinuria with strenuous exercise
phosphorylase deficiency)

Decreased NAOPH in RBCsleads to hemolytic anemia due to poor G6PD deficiency


RBCdefense against oxidizing agents (Rate limiting enzyme o/pentose phosphate pathway)

Recurrent pyogenic infections due to impairment of respiratory Chronic Granulomatous Disease


burst of neutrophils and monocytes (NADPH oxidase deficiency)

Cataracts within a few days of birth, vomiting and diarrhea after Classic Galactosemia (Galactose lp-uridyltransferase
milk ingestion, lethargy, hypotonia, mental retardation deficiency)

Galactosemia, galactosuria, cataracts in early childhood Galactokinase deficiency

Fructokinase deficiency Essential fructosuria

Hereditary Fructose intolerance (Aldolase B


Fructosuria, severe hypoglycemia, cirrhosis, vomiting, jaundice
deficiency)
PART 6 I INFECTIOUSDISEASES
COMMONINFECTIOUSDISEASE CONDITIONS
Important Etiologic Organisms for Diarrhea
I • • I
MECHANISM ETIOLOGY DIARRHEA ASS
Pain Vomitin
• Bacilluscereus
Preformed toxins* • Staphylococcus aureus Watery + ++ ++++
• Clostridium perfringens

• Vibrio cholerae
• Enterotoxigenic E.coli
Enterotoxin* • Klebsiella pneumoniae Watery + ++ ++++
• Aeromonas spp

• Enteropathogenic E.coli
• Enteroadherent E.coli
Enteroadherenrt' • Giardia spp Watery, mushy ++ +++ +
• Cryptosporidiosis
•Helminths

• Clostridium difficile
Cytotoxin
• Hemorrhagic E.coli Watery, occasional bloody ++ ++++ +
producers**

Invasive Or anisms 0

Minimal
•Rotavirus
• Norovirus Watery ++++ +++ +++
inflammation
•Salmonella
• Campylobacter spp
Variable •Aeromonas spp Watery or bloody ++++ ++++ +++
inflammation • Vibrioparahaemolyticus
• Yersinia

• Shigella spp
Severe • Enteroinvasive E.coli Bloody ++++ ++++ +
inflammation • Entamoeba histo/ytica
*Causes profuse, watery diarrhea from small-bowel hypersecretion
••causes hi h fever and abdominal ain

Important Mosquito Borne Infections


Dengue viruses
The most clinically important flaviviruses that cause the
fever and myalgia syndrome are
(breakbone fever)

Most important of the parasitic diseases of humans, Malaria

Wuchereria bancrofti
Causes of filariasis
Brugia malayi
Comparison of the Different Malarial Species
P.falc,parum P. v1vax P. malarme P. ovule

Asexual Cycle 48 hours 48 hours 72 hours 50 hours

Periodicity Malignant Tertian Benign Tertian Benign Quartan Benign Tertian

Young RBCs Old RBCs Young RBCs


All ages
RBC Preference (Mnemonic: (Mnemonic: (Mnemonic:
[Mnemonic: Fokfok)
VOta •young) Matanda) VOta • young)

Merozoites released per


30,000 10,000 15,000 15,000
infected hepatocyte

Irregularly shaped
Band or rectangular Enlarged and oval
Banana-shaped large rings and
Morphology forms of with tufted ends
gametocytes trophozoites;
trophozoites Schuffner dots
Schuffner dots

Cerebral Malaria Yes No No No

Relapse No Yes No Yes

Drug Resistance Many Few Few Few

High-Yield Concepts in Malaria


Return of disease after its apparent cessation due to
Relapse (seen in P. ovale and P. vivax)
reactivation of hypnozoites
When a patient in or from a malarious area presents with
fever, what should be done to confirm diagnosis and Thick and thin blood smears
identifv snecies of infectinP" ornanism?
Acute hemolytic anemia with massive hemoglobinuria in
Blackwater fever
malaria
Highest endemicity of malaria in (MIMAROPA region) Palawan

High Yield Concepts in Enteric (Typhoid) Fever


Fever and abdominal pain
Hallmark features of enteric fever Enteric fever is a misnomer in that hallmark features
are variable

Prolonged fever
Most prominent symptom (38.8°-4O.S"C; 101.8°-104.9°F), which can continue
for up to 4 weeks if untreated

Faint, salmon-colored, blanching, maculopapular rash


Rose spots
located primarily on the trunk and chest
Neuropsychiatric symptoms with picking at bedclothes
Muttering delirium or coma vigil
or imaginary objects in patients with enteric fever
Women, infants, and persons who have biliary
Chronic carriage is more common among abnormalities or concurrent bladder infection with
Schistosoma haematobium
Most effective class of agents in treatment Fluoroquinolones
High-Yield Concepts Related to Leptospirosis
Weil's Syndrome ( characterized by jaundice, renal
Most severe form of leptospirosis
dysfunction, and hemorrhagic diathesis)

Septic shock with multiorgan failure


Severe bleeding complications that most commonly
involve the lungs (pulmonary hemorrhage),
Causes of death in severe leptospirosis
gastrointestinal tract (melena, hemoptysis), urogenital
tract (hematuria), and skin (petechiae, ecchymosis, and
bleeding from venipuncture sites)

Severe leptospirosis should be treated with IV Penicillin

Prophylaxis for leptospirosis exposure Doxycycline

Comparison of Trematode Infections


Trematode Transmission Sites Affected lntermedrate Hosts Treatment

Schistosoma
Penetrates skin Liver Snail Praziquantel
japonicum

Ingested with raw crab


Paragonimus
(Sundathelphusa Lung Snail and Crab Praziquantel
westermani
philippina)

Clonorchis sinensis Ingested with raw fish Liver Snail and Fish Praziquantel

High-Yield Concepts Related to Schistosomiasis


Intermediate host of S.japonicum Oncomelania quadrasi (snail)
' r
V
Intermediate host of P. westermani Sundathelphusa philippina (mountain crab)

Symptomatic acute schistosomiasis usually seen in travelers who


Katayama fever
have contracted the infection for the first time

Sorsogon, Samar, Leyte. Oriental Mindoro


Areas of endemicity of schistosomiasis (most prevalent), Bohol & all of Mindanao
EXCEPTMisamis Oriental

Mainstay in tbe control program of schistosomiasis in the Philippines Mass treatment with praziquantel

High-Yield Concepts Related to Tetanus & Rabies


Anaerobic, gram-positive, spore-forming rods Clostridium tetani

Negri bodies
- eosinophilic cytoplasmic inclusions in brain
Most characteristic pathologic finding in rabies
neurons that are composed of rabies virus proteins
and viral RNA.
ANTI-INFECTIVES
Antibiotic Classification Based on Action
Very Finely Proficient At Cell Murder!
Vancomycin
Fluoroquinolones
Bactericidal Antibiotics Penicillins
Aminoglycosides
Cephalosporins
Metronidazole

We're ECSTaTiC about bacteriostatics!


Erythromycin
Clindamycin
Bacteriostatic Antibiotics Sulfamethoxazole
Trimethoprim
Tetracycline
Chloramphenicol

Penicillins
Drug Characteristic Side Effect

Penicillin G Narrow spectrum penicillins Hypersensitivity

Dicloxacillin, nafcillin, oxacillin Penicillinase-resistant penicillins Hypersensitivity, Interstitial Nephritis

Ampicillin Extended-spectrum penicillins Pseudomembranous colitis

Ticarcillin/Piperacillin
Antipseudomonal penicillins Hypersensitivity
Carbenicillin

Remembering the Penicillins


Pen V is Oral, ipinapasok sa Vunganga
Pen G is IV: You inGect Pen G
Use naf (nafcillin) for Staph
AMPicillin = AMPed up penicillin
amOxicillin = greater Oral bioavailability

Organisms Susceptible to Amoxicillin


Amoxicillin HELPSkill Enterococci

Haemophilus influenzae
Helicobacterpylori
Escherichia coli
Enterococci
Listeria monocytogenes
Proteus mirabilis
Salmonella spp.
Shigella
Cephalosporins
Drug Description Side Effect

1st generation cephalosporin, surgical prophylaxis, greatest


Cefazolin Hypersensitivity reaction
gram positive coverage

Cefaclor, cefoxitin, 2nd generation cephalosporin, added gram negative


Disulfiram-like reaction
cefuroxime, cefotetan coverage

Ceftriaxone, cefotaxime,
3 rdgeneration cephalosporin, Pseudomonas coverage for
cefpodoxime, Disulfiram-like reaction
Ceftazidime, Ceftriaxone has NO pseudomonal coverage
ceftazidime, cefixime

4th generation cephalosporin, broad spectrum activity


Cefepime
(both gram positive and gram negative)

5th generation. ceftaroline )-broad gram positive and


Gram negative organism coverage;
Ceftaroline unlike 1st -4th generation cephalosporins, ceftaroline covers
MRSA,and Enterococcus faecalis-does NOT
cover Pseudomonas.

Ceftriaxone Best CNSpenetrance

Some Important Points about Remembering


Cephalosphorins First Generation Cephalosporins
Microbes covered by 1st gen cephalosporins FADer, help me FAZ my PHarmacology boards!
KEPs CeFADroxil
Klebsiella spp. CeFAZolin
E.coli CePHalothin
Proteus spp. CePHapirin
CePHradine
CePHalexin
Organisms Susceptible to Remembering
Second Generation Cephalosporins Second Generation Cephalosphorins
HEN has KEPS In a FAMily gathering. you see your FOXy cousin wearing a
Haemophilusinfluenzae FUR coat and drinking TEa.
Enterobacter aerogenes CeFAMandole
Neisseriaspp. CeFOXitin
Klebsiellapneumoniae CeFURoxime
Escherichiacoli CefoTEtan
Proteus mirabilis or
Serratia marcescens FAC! LORA the PROfessional AZhOLE is still on the FONe.
CeFAClor
LORAcarbef
CeFPROzil
CefmetAZOLE
CeFONicid
Cephalosporins with Remembering
PseudomonalCoverage Third Generation Cephalosphorins
CeITAZidime FEnge PO ng PERA to FIX my TTTTTV!
CeFEPime CeFEtam et Ceffazidime
CeFOPerazone CefPOdoxin CefoTaxime
CefoPERAzone CeITizoxime
CeFlXime CeITibuten
Ceffriaxone
Protein Synthesis Inhibitors
Drug Remarks I Side Effect

Aplastic Anemia (dose independent]


Chloramphenicol Binds to SOSsubunit Anemia (dose dependent)
Gray Baby Syndrome

Tooth Enamel Discoloration


Tetracycline Binds to 30S subunit
Photosensitivity

Erythromycin Binds to SOSsubunit Diarrhea, cholestatic jaundice

Binds to SOSsubunit, highest volume of distribution,


Azithromycin
single dose administration for certain indications

Clindamycin Binds to SOSsubunit, anaerobic coverage Pseudomembranous colitis

Linezolid Binds to SOSsubunit, for vancomycin and methicillin-resistant organisms

Aminoglycosides
Drug Remarks Side Effect

Prototype aminoglycoside Nephrotoxicity


Bactericidal Ototoxicity
Gentamicin Binds to 30S subunit Neuromuscular blockade
(absolute contraindication with
myasthenia gravis)

Tobramycin Treatment of ocular infections

Streptomycin Tuberculosis

Spectinomycin Treatment of drug-resistant gonorrhea

Has pseudomonal coverage


Amikacin
Narrow therapeutic window

Neomycin Treatment of hepatic encephalopathy (second line)

Remembering Aminoglycosides
Mean GiANTS canNOT kill anaerobes.
Gentamicin Nephrotoxicity
Amikacin Ototoxicity
Neomycin Teratogen
Tobramycin Streptomycin
AminOglycosides require Ozfor transport
They won't work under anaerobic conditions.
Sulfonamides
Drug Remarks Side Effect

Sulfamethoxazole Blocks Dihydropteroate Synthase

Hypersensitivity (SJS,TEN),
Sequential blockade in folate synthesis
TMP-SMX kernicterus, hemolysis in patients
Commonly used for UTI
with G6PD deficiency

Fluoroquinolones
Drug Remarks Side Effect

Tendonitis or tendon rupture in


2ndgeneration quinolone
Ciprofloxacin people > 60 years old and in patients
Used for UT! and GIT infections
taking prednisone

3rd generation quinolone


Levofloxacin
Used for pulmonary infections

generation quinolone
4th

Moxifloxacin Broad spectrum of activity, anaerobic coverage


Treatment of ocular infections

Gatifloxacin 4th generation quinolone Diabetes mellitus

Anti-Mycobacterial Agents
Drug Remarks Side Effect

Neurotoxicity, hepatotoxicity,
Bactericidal
lsoniazid sideroblastic anemia, drug-induced
Inhibits mycolic acid synthesis
lupus, potent CYP450 inhibitor

Bacteriostatic,
Rifampicin Red orange urine, hepatotoxicity
Inhibits DNA-dependent RNA polymerase

Bacteriostatic Visual dysfunction (retrobulbar


Ethambutol Blocks arabinosyltransferase to decrease carbohydrate neuritis, color blindness, red-green,
polymerization of mycobacterium cell wall reversible)

Pyrazinamide Bacteriostatic but bactericidal on actively dividing MTB Hyperuricemia, most hepatotoxic

Streptomycin Bactericidal, binds to 30S Nephrotoxicity, ototoxicity

Other Important Things to Remember About Anti-Mycobacterials


INH Injures Neurons and Hepatocytes

R = Rifampicin (RNA polymerase inhibitor, Red-orange body fluids, Rapid development of resistance, Revs up
cytochrome P450 (inducer)
Hepatotoxicity: Iso a Red Pyre! [lsoniazid < Rifampin < Pyrazinamide]
Drugs Used for Leprosy
Drug Remarks Side Effect

Methemoglobinemia, Hemolysis if
Dapsone Inhibits folate synthesis
G6PD deficient

Rifampicin Inhibits DNA-dependent RNA polymerase Red-orange urine

Causes a red-black skin discoloration


Phenazine dye
Clofazirnine that accumulates, particularly in
Binds to guanine bases
lesional areas

Other Antimicrobials
Drug Remarks Side Effect

No gram positive or anaerobic activity


Aztreonam
Gram negative rods only

Meropenem Broad spectrum of activity CNS toxicity at high plasma levels

Disulfiram-like reaction (when taken


Anaerobic and antiprotozoal coverage
Metronidazole with alcohol), metallic taste,
Treatment of pseudomembranous colitis
neurotoxicity

Nitrofurantoin Treatment of urinary tract infections Pulmonary fibrosis

Antibiotic Drugs of Last Resort


I AM your Last Shot at Victory!
Imipenem ,kinezolid
Amikacin Streptogramins
Meropenem Y,ancomycin

Antifungals
Drug Remarks Side Effect

Fever/chills ("shake and bake"),


Forms artificial pores
Amphotericin B hypotension,nephrotoxicity, arrhythmias,
Used for serious, systemic mycoses
anemia, IV phlebitis ("amphoterrible"J.

Topical treatment of dermatophytosis and Gynecomastia,


Ketoconazole
candidiasis CYP450 inhibitor

Fluconazole Prophylaxis and treatment of candidiasis and cryptococcosis

Griseofulvin Interferes with fungal microtubules Potent CYP450 inducer

Treatment of candidiasis ( oropharyngeal, esophageal, vaginal)


Nystatin
Swish and swallow or suppository preparations
Antivirals
Drug Remarks Side Effect

Treatment of HSV and VZV Obstructive crystalline nephropathy


Acyclovir Requires activation by viral thymidine kinase (hence and acute kidney injury if not
only activated when infection is present) adequately hydrated

Treatment of CMV
Ganciclovir
Requires activation by viral thymidine kinase (hence only activated when infection is present)

Treatment of HSV,VZVand CMV


Foscarnet
Does NOT require viral thymidine kinase activation

Prevents viral uncoating Cerebellar dysfunction,


Amantadine
Influenza A coverage livedo reticularis

Neuraminidase inhibitor
Oseltamivir
Treatment and prevention of influenza A and 8

Lamivudine Treatment of hepatitis B infection and HIV

Ribavirin Treatment of hepatitis C and RSVinfection

Some Important Points About FOScarnet and AMANTADINE


FOScarnet
pyroFOSphate analog

AMANTADINE
A man to dine takes off his coat.
Amantadine prevents uncoating.
Blocks influenza A and rubellA
Causes problems with the cerebe!IA
Anti-Retroviral Drugs
Drug Remarks Side Effect

NRTls • Nucleoside reverse transcriptase inhibitor (NRTI) • Lactic acidosis


• Requires phosphorylation • Bone marrow
Abacavir (ABC) suppression
• Red ragged fibers are a histologic hallmark of zidovudine-
Emtricitabine (FTC) induced myopathy • Anemia
Lamivudine (3TC)
• ZDV can be used for general prophylaxis and during • Peripheral neuropathy
Tenofovir (TDF) pregnancy to decrease risk of fetal transmission.
Zidovudine (ZDV,
formerly AZT)

NNRTls • Non-nucleoside reverse transcriptase inhibitor (NNRTI) • Rash and


• No phosphorylation required hepatotoxicity are
Delavirdine common to all NNRTls.
Efavirenz • Vivid dreams and CNS
Nevirapine symptoms are common
with efavirenz.

lntegrase inhibitors • Inhibits HIV genome integration into host cell • Increased creatine
• chromosome by reversibly inhibiting HIV integrase kinase
Bictegravir
Dolutegravir
Elvitegravir
Raltegravir

Protease inhibitors • Prevent maturation of new viruses. • Hyperglycemia, GI


intolerance (nausea,
Atazanavir diarrhea),
lipodystrophy
Darunavir
(Cushing-like
Lopinavir
syndrome).
Ritonavir

Entry inhibitors • Enfuvirtide: Binds gp41, inhibiting viral entry (inhibits • Enfuvirtide: Skin
FUSION) reaction at injection
Enfuvirtide • Maraviroc: Binds CCR-5on surface ofT cells/monocytes, sites
Maraviroc inhibiting interaction with gp120 (inhibits DOCKING)

Remembering NNRTis
Never Ever Deliver Nucleosides
Nevirapine
Efavirenz
Delavirdine

Remembering Protease Inhibitors


All protease inhibitors end with -navir

NAVIR (never) TEASE a proTEASE


Anti-Malarial Drugs
Drug Remarks Side Effect

Blocks detoxification of heme into hemozoin


Choloroquine Retinopathy; pruritus
Heme accumulates and is toxic to plasmodia

Quinine Used as second-line treatment Hypoglycemia, cinchonism

DOCfor eradication of hypnozoites of P. vivax and ovale to eradicate persistent liver stages and
Primaquine prevent relapse
Should not be given to pregnant women

Only drug advised for pregnant women traveling to areas with drug-resistant malaria
Mefloquine
Generally considered safe in the second and third trimesters of pregnancy

Doxycycline Prophylaxis in areas with chloroquine- or mefloquine-resistant P.falciparum

Drug of choice for all patients with severe malaria everywhere


Artesunate
Should be used in pregnant women with severe malaria.

Anti-Protozoa) Drugs
Drug Remarks

Asymptomatic cyst carriers of E. histolytica


Diloxanide furoate
Luminally active agent used to eradicate the cysts of£. histolytica

Amoebic dysentery
Metronidazole Trichomoniasis
Giardiasis

Nitazoxanide Cryptosporidium parvum infection

TMP-SMX (Cotrimoxazole) Pneumocystis jirovecii pneumonia

Pyrimethamine-Sulfadiazine Toxoplasmosis

Suramin + Melarsoprol African sleeping sickness

Nifurtimox Chagas disease

Stihogluconate Leishmaniasis
Anti-Helminthic Drugs
Drug Remarks

Mebendazole Inhibits helminthic microtubules

Inhibits helminthic microtubules


Albendazole
Treatment for echinococcosis, intestinal parasitic nematodes

Diethylcarbamazine Drug of choice for filarial disease and Loa loa

Considered more effective than albendazole for Strongyloides


lvermectin First line agent for onchocerciasis
SE: Mazzetti reaction r.. ,\
~.,
y
Pyrantel pamoate Used in Enterobius infection

Drug of choice for trematodes and cestodes


Praziquantel
EXCEPTechinococcosis (treated with albendazole)

Niclosamide Back-up drug to Praziquantel

BASIC BACTERIOLOGY

Steps in Gram Staining (remember V-1-A-S)

Step Procedure Reagent

1 Primary Stain Crystal Violet

2 Mordant Iodine

3 Decolorizing Agent Acetone

4 Counterstain Safranin

Atypical Bacteria (Based on Gram staining)

Name Reason Alternative

Mycobacteria Too much lipid in cell wall so dye cannot penetrate Acid-fast stain

Spirochetes Too thin to see Dark field microscopy

Mycoplasma No cell wall None


Use serologic studies

Legionella Poor uptake of counterstain Silver stain

Chlamydiae Intracellular Locate for inclusion bodies

Rickettsiae Intracellular Giemsa/Tissue stains


Generalities in Bacteria
Mycoplasmataceae
All bacteria have cell walls except
2 pathogenic species of Mycoplasmataceae:
Mycoplasma pneumoniae and Ureaplasma urealyticum

All bacterial capsules are composed of polysaccharide


Bacillus anthracis (made up of poly D•glutamate)
except

Bacterial Oxygen Metabolism


Nocardia, Bacillus cereus, Neisseria, Pseudomonas, Bordetella, Legionella, Brucella,
Obligate Aerobes
Mycobacterium,

Facultative Anaerobes Staphylococcus, Bacillus anthracis, Corynebacterium, Listeria, Actinomyces, Mycoplasma

Enterococcus,Streptococcus, spirochetes (Borrelia, Treponema,, Leptospira),


Microaerophiles
Campylobacter

Obligate Anaerobes Bacteroides, Clastridium

~....y
Bacterial Culture Media
Clostridium spp McClung•Toabe (Egg yolk)

Corynebacterium diphtheria Black colonies on cystine•tellurite agar

Enterococci Can grow in 6.5% NaCl and bile (lab test)

Staphylococcus spp. Ferments mannitol

Thayer•Martin agar
N. meningitides, Selectively favors growth of Neisseria by inhibiting growth of gram positive
N.gonorrhoeae organisms with vancomycin, gram negative organisms except Neisseria with
trimethoprim and colistin, and fungi with nystatin

Haemophilusinfluenzae Chocolate agar and Factors V (NAD+) and X (hematin)

LOwenstein•Jensen; Middlebrook
Mycobacteriumtuberculosis
medium, rapid automated broth cultures

Vibriocholerae Thiosulfate citrate bile salts sucrose (TCBS)

Bordet·Gengou
Bordetellapertussis
Regan•Lowe medium

Charcoal·yeast extract agar buffered with


Legionellapneumophila
cysteine and iron

Mycoplasmapneumoniae Eaton

Grows on blood and MacConkey agars


Pseudomonasaeruginosa Colonies have a shiny "gun·metal" appearance and a characteristic fruity odor

Salmonella, Shigella Xylose•Lysine•Deoxycholate (XLD)

Leptospirainterrogans Ellinghausen•McCullough•Johnson•Harris (EMJH)


PART 71ALLERGYAND RHEUMATOLOGY
High Yield Concepts in Basic Immunology
CD21 (other B cell markers:
B cell surface marker and EBY receptor
CDl 9, 20, CD79a
Highly phagocytic APCs [antigen presenting cells) in the body
Dendritic cells
Function as link between innate and adaptive immune systems

Major receptor for antigen in B cells lgMandlgD

Majority of total serum immunoglobulins. Only isotype able to cross the placenta lgG

Antibody secreted in mucosa! surfaces as dimer, most produced antibody overall lgA
Produced in the primary (IMmediate) response to an antigen
Pentamer enables avid binding to antigen while humeral response evolves. lgM
Associated with cold autoimmune hemolytic anemia.
Protein molecules capable of activating up to 20% of T-cell pool resulting in
Superantigen
widespread immune response
Binding of an opsonized target cell to a FC receptor bearing effector cell resulting ADCC(antibody dependent
in lysis of the target by the effector cell cellular cytotoxicity)
Rapid, first-line immunity involving neutrophils, macrophages, dendritic, natural
Innate immunity
killer cells, complement, physical epithelial barriers, secreted enzymes
Learned, highly-specific immunity involving T and B cells and antibodies and
Adaptive immunity
utilizing memory cells
Expressed in all nucleated cells, MHC(Major Histocompatibility
Present endogenous intracellular antigens to CDS cytotoxic T-cells Complex) la

Expressed only on APCs and present exogenous or extracellularly engulfed


MHC2
antigens to CD4 T-helper cells
Secrete IFN-y, which enhances the ability ofmonocytes and macrophages to kill
THl (T-Helper 1)
microbes thev ine:est

Secretes IL4, 5, 6, 13 recruiting eosinophils stimulating antibody production TH2 (T-Helper 2)

Release cytotoxic granules (perforin, granzyme B) and activates apoptosis Cytotoxic T cells

Facilitates phagocytosis by coating antigen Opsonins (lgG and C3b)


Serum Amyloid A, CRP,Ferritin,
Acute phase reactants that are increased during inflammation Fibrinogen, Hepcidin,
Haotmtlobin, Procacitonin
Proteins that are decreased during inflammation Albumin, Transferrin

Classical - lgG or lgM mediated


Complement Activation
Alternative - Microbial surface proteins
Pathways
Lectin - Mannose or other sugars on microbe surface

Complement proteins involved in anaphylaxis (anaphylatoxins) C3a, 4a, Sa

Induces neutrophil chemotaxis CSa


The key effector cell in the biologic response in allergic rhinitis, asthma,
Mast cells
anaphylaxis and urticaria
Neutrophil chemotaxis Leukotriene LTB4

Most cost-effective means of managing allergic rhinitis Allergen avoidance


Common Cytokines and Their Function
ILl • Fever, activates osteoclasts

IL2 • Stimulates all types ofT cells

IL3 • Stimulates bone marrow stem cells

• Induces 8 cell growth


IL4
• Enhances class switching to IgE and lgG ,.....
ILS
• Enhances class switching to IgA
• Stimulates growth and differentiation of Eosinophils
.
~--"
IL6 • Fever, stimulates acute phase proteins

ILB • Major neutrophil chemotactic factor

• Modulates inflammatory response (together with TGF-B)


ILlO

IL12
• Inhibits activated macrophages and dendritic cells
• Also secreted by regulatory T cells.

• Differentiates T cells into THl; activates NK cells


1'.
.~•
.._,,

TNFalpha • Activates endothelium. Causes WBC recruitment, vascular leak.

• Secreted by NK cells and T cells in response to antigen or IL·12 from macrophages; stimulates
macrophages to kill phagocytosed pathogens.
IFNgamma
• Inhibits differentiation ofTh2 cells.
• Induces IgG isotype switching in 8 cells

Common Autoantibodies and Associated l)iseases


Anti-Ach Receptor Myasthenia Gravis (MG)

Anti•glomerular basement membrane Goodpasture's Syndrome


Anti-glutamic acid decarboxylase, islet cell cytoplasmic
Type 1 Diabetes Mellitus (DM)
antibodies
Antisynthetase (eg, anti-Jo-1), anti-SRP, anti helicase (anti-Mi-2) Dermatomyositis, Polymyositis

Anti-Thyroglobulin, Anti-TPO (Thyroid Peroxidase) Hashimoto's Thyroiditis

Anti-Mitochondrial Primary Biliary Cirrhosis (PBC)

Anti-smooth muscle, anti-liver /kidney microsomal-1 Autoimmune Hepatitis


Anti-TSH receptor Graves' Disease
Anti-Ul RNP (Ribonucleoprotein] Mixed Connective Tissue Disease (MCTD)

Anti-Centromere Limited Scleroderma, CRESTSyndrome

Anti-Scl-70 (anti-DNA topoisomerase I) Diffuse Scleroderma

Anti Ro (SS-A), Anti-LA (SS-B) Sjogren syndrome


High-Yield Concepts in Systemic Lupus Erythematosus (SLE)
Female of reproductive age, joint pains, pleural or pericardia! effusions, photosensitive rash over
the nose, cheek areas, microscopic hematuria or proteinuria, oral ulcers, anemia, leukopenia or SLE
thrornbocytopenia, seizures or psychosis, headache, fever, rnyalgias, presence of autoantibodies

Best screening test for SLE (most sensitive) ANA

SLE-specific antibodies that correlate with level of disease activity, nephritis and vasculitis Anti-dsDNA

SLE-specific antibodies with no clinical correlations Anti-Sm

Antibodies associated with Sicca syndrome, subacute cutaneous lupus, neonatal lupus with
Anti-Ro (SS·A)
congenital heart block and decreased risk for nephritis
_, ..
Antibodies in drug-induced lupus 4~ Anti-Histone

Positive test in serum correlates with depression or psychosis due to CNS lupus Anti-Ribosomal P

Antibodies predisposing to recurrent fetal loss, thrombosis, detected by ELISAfor Cardiolipin Anti-Phospholipid
and B2Gl and DRVVTfor lupus anticoagulant Antibody

Most serious manifestation of SLE


& y
Nephritis

Pleuritis with or
Most common pulmonary manifestation of SLE
without effusion

Most frequent cardiac manifestation of SLE Pericarditis

Libman-Sacks
Fibrinous vegetations and endocarditis in SLE Endocarditis
(LSE)

Normocytic
Most frequent hematologic manifestation of SLE Normochromic
Anemia

Cognitive
Most common manifestation of diffuse CNSlupus
Dysfunction

Discoid lupus
Most common chronic dermatitis in SLE
erythematosus
High-Yield Concepts in Rheumatoid Arthritis (RA)
Most common form of chronic inflammatory arthritis RA

Most common cardiac and valvular manifestation in RA Pericarditis and mitral regurgitation

Most common cause of death in patients with RA Cardiovascular disease

Environmental factor most implicated in RA Smoking

Triad of neutropenia, splenomegaly and nodular RA Felty's syndrome

Length of time for joint symptoms to be suggestive of RA 6 weeks or more

Synovial inflammation and proliferation, focal bone


Pathologic hallmarks of RA
erosions, thinning of the articular cartilage

lgM against the Fe portion of lgG Rheumatoid ractor

Serum marker with higher specificity for RA than RF Anti-CCP

Initial radiological finding in RA Periarticular osteopenia

Test with greatest sensitivity for detecting synovitis, joint


MRI
effusions and early bone and marrow changes in RA

DMARDof choice for RA Methotrexate

High-Yield Concepts in Osteoarthritis (OA)


Obese, elderly, female complaining of unilateral knee pain
exacerbated by exertion and relieved by rest and NSAIDs; OA
No warmth, no swelling, nor redness; (+) crepitus

Most common type of arthritis OA

2 major factors contributing to the development of OA Joint loading and joint vulnerability

Most potent risk factor for QA Age

Nodes found on the PIP joint in QA Bouchard's nodes (Mnemonic·B of Bouchard comes
first in the alphabet before Hof Heherden's)

Nodes found on the DIP joint in OA Heberden's nodes

Fulcrum of the longest lever arm in the body Knee

Radiographic hallmark of QA Osteophytes

Initial analgesic of choice for QA Acetaminophen or Paracetamol


High-Yield Concepts in Gouty Arthritis and Pseudogout
50 year old alcoholic male patient complaining of severe joint pain
starting last night with noted swelling and redness on his right first Gouty arthritis
metatarsophalangeal (MTP) joint
Inflammation of the first MTP joint in gout Podagra
Needle-shaped crystals that are negatively birefringent [yellow
Gouty arthritis
under parallel light and blue under perpendicular light]

Rhomboid-shaped crystals that are weakly positively birefringent Pseudogout

NSAIDs{lndomethacin] colchicine, or
Mainstay of treatment during acute attack of gout
e:lucocorticoids
• The number of acute attacks (urate
lowering may be cost-effective after two
attacks)
• Serum uric acid levels (progression is
more rapid in patients with serum uric
acid >535 µmol/L (>9.0 mg/dL])
Decision to initiate hypouricemic therapy is made taking into
• The patient's willingness to commit to
consideration:
lifelong therapy
• The presence of uric acid stones

• Urate-lowering therapy should be initiated


in any patient who already has tophi or
chronic goutv arthritis.
Under excretors of uric acid
Indications for uricosuric agents
r<600me in 24 hour urine samole l
Most commonly used hypouricemic and best drug to use in urate
Allopurinol
overoroducers, urate stone formers and renal disease
Target theraoeutic blood uric acid level for gout Less than or eaual to 6mg/dl
Joint most freauentlv affected in CPPDor oseudogout Knee
Radiograph findings of punctate or linear radiodense deposits in
fibrocartilaginous joint menisci or articular hyaline cartilage Chondrocalcinosis
suggestive of CPPD

High-Yield Concepts in Psoriatic Arthritis


Arthritis presenting with predominant DIP
involvement, asymmetric or symmetric,
involving one or more joints, dactylitis, Psoriatic arthritis
shortening of digits and nail changes with
or without silverv scalv skin lesions
Six patterns of nail involvement are identified: pitting, horizontal
Psoriatic nail dystrophy ridging, onycholysis, yellowish discoloration of the nail margins,
dvstroohic hvoerkeratosis, and combinations of these findings
Uveitis in osoriatic arthritis Bilateral chronic posterior uveitis
(1) Arthritis of the DIP joints
(2) Asymmetric oligoarthritis
5 patterns of psoriatic arthritis (from most
(3) Symmetric polyarthritis similar to RA
to least common)
(4) Axial involvement (spine and sacroiliac joints)
(51 Arthritis mutilans a hiehlv destructive form of the disease
• Classic "pencil•in•cup" deformity
• Marginal erosions with adjacent bony proliferation ("whiskering1
• Smal(.joint ankylosis
Radiographic characteristics of psoriatic
• Osteolysis of phalangeal and metacarpal bone, with telescoping
arthritis
of digits
• Periostitis and proliferative new bone at sites of enthesitis "Ray''
distribution oflesions
Ideal treatment for psoriatic arthritis Anti·TNF·alpha aeents
High-Yield Concepts in Reactive Arthritis (ReA)
Acute non purulent arthritis complicating an infection elsewhere
Reactive arthritis
in the body

Triad of arthritis, urethritis, and conjunctivitis Reactive arthritis

Chlamydia, Shigella, Salmonella, Yersinia,


Common organisms implicated in ReA
Campylobacter, E coli, Yersinia

Initial treatment of choice for ReA NSAIDs

High-Yield Concepts in Ankylosing Spondylitis (AS)


Young adult male, insidious onset of dull pain in the lower lumbar or gluteal
region, lumbar morning stiffness lasting a few hours that improves with AS
activity and returns after inactivity with nocturnal exacerbation

Plays a direct role in AS pathogenesis HLA-B27

Synovitis and myxoid marrow


Earliest changes in AS
Sacroiliitis is often an early
manifestation of AS and nr-ax-SpA

lnnammation in the fibrocartilaginous regions where a tendon, ligament or


joint capsule attaches to bone characteristic of SpA
Enthesitis
Associated with prominent edema of the adjacent bone marrow and is often
characterized by erosive lesions that eventually undergo ossification.

Most common extra-articular manifestation of AS Acute anterior uveitis

First line pharmacologic management for AS NSAIDs

High-Yield Concepts in Infectious Arthritis


Patient with pneumonia presenting with sudden onset moderate to severe pain on the right
Infectious or septic
knee, with muscle spasm, decreased range of motion, swelling and redness, high fever,
arthritis
leukocytosis

Most common site of infectious arthritis Knee

Most common route of infection for infectious arthritis in all age groups Hematogenous

Most common etiologic agent for infectious arthritis among young aduuts and adolescents Neisseria
(sexually active] gonorrhoeae

Staphylococcus
Most common nongonococcal cause of infectious arthritis in adults of all age groups
aureus

Staphylococcus
Most common etiologic agent for infectious arthritis after surgery or penetrating injuries
aureus
Subset of patients with highest incidence of infectious arthritis RA patients

Most common presentation of infectious arthritis Monoarthritis

Spine, sacroiliac and


Sites of infectious arthritis common among IVdrug abusers sternoclavicular
joints

Timely drainage of
Other than antibiotics, essential treatment needed for a favorable outcome on joint function in
pus and necrotic
infectious arthritis
debris

Late manifestation of congenital syphilis manifesting as chronic painless synovitis with


Clutton's joints
effusion of large joints, particularly the knees and elbows

Reactive symmetric form of polyarthritis that affects persons with visceral or disseminated
Poncet's disease
tuberculosis

High -Yield Concepts m Vascuht1des


\.}
Antibodies directed against proteinase-3 detected as diffuse cANCA( cytoplasmic anti-neutropbil cytoplasmic
granular cytoplasmic staining pattern in neutrophils antibodies)

Antibodies directed against myeloperoxidase with pANCA(perinuclear antineutropbil cytoplasmic


perinuclear or nuclear staining in neutrophils antibodies)

First step in the workup of a patient with suspected vasculitis Exclude other diseases

Palpable purpura, pulmonary infiltrates,


microscopic hematuria, chronic inHammatory
Symptoms suggestive ofvasculitis
sinusitis, mononeuritis multiplex, unexplained
ischemic events and glomerulonephritis

Granulomatous necrotizing vasculitis of the triad of upper


and lower respiratory tract and kidney (sinusitis, lung Wegener's granulomatosis
involvement, glomerulonephritis), [+J cANCA

Microscopic polyangiitis
Necroitzing inflammation of small arteries and veins
including venules, glomerulonephritis, usually with no upper The absence of granulomatous inHammation in
airway involvement and no pulmonary nodules, (+) pANCA microscopic polyangiitis is said to differentiate it
from GPA (Wegener's)

Asthma, peripheral and tissue eosinophilia, extravascular


granuloma and vasculitis of multiple organ systems Cburg-Strauss syndrome
(predominant pulmonary findings)

Necrotizing vasculitis, renal and visceral artery involvement


with aneurysmal dilatations, no pulmonary aassociated with Polyarteritis nodosa
Hepatitis B infection
Elderly female presenting with fever, anemia, headaches,
temporal tenderness, jaw claudication, high ESR,and
Giant cell/temporal arteritis
accompanying stiffness and muscle pains of the neck,
shoulders, hip and thighs

Syndrome characterized by stiffness aching and pain in the


muscles of the neck, shoulders, lower back, hips and thighs Polymyalgia rheumatica
associated with giant cell arteritis

Dreaded complication of giant cell arteritis lschemic optic neuropathy

Young female, systemic symptoms, arm claudication,


Takayasu arteritis (also known as Aortic Arch
diminished pulses on one arm, vasculitis of medium to large
Syndrome or Pulseless Disease)
arteries involving the aortic arch and branches

Child with glomerulonephritis, palpable purpura over the


buttocks and lower extremities, gastrointestinal symptoms, Henoch Schonlein Purpura (HSP)
arthralgias, and history of recent respiratory infection (lmmunoglobulin A vasculitis)

Most commonly encountered vasculitis in clinical practice Cutaneous vasculitis

The most effective therapy for the systemic vasculitides is Cyclophosphamide

Cutaneous vasculitis, arthritis, peripheral neuropathy,


membranoproliferative glomerulonephritis, Hepatitis C, cold- Cryglobulinemic vasculitis
precipitated agglutinins or immunoglobulins

,V" Allopurinol, Thiazides, Gold, Sulfonamides,


Drugs implicated in vasculitis syndromes
Phenytoin, Penicillin

Other High-Yield Concepts in Rheumatology


Malar rash, Gottron's papules, erythematous periorbital rash (heliotrope rash), shawl
and face rash, mechanic's hands, high creatine kinase, [+)ANA, [+) anti-MDAS,anti-TI Fl, Dermatomyositis
anti-Mi-2, anti-NXP2

Calcinosis
Raynaud's phenomenon
CRESTsyndrome
Esophageal dysmotility
(limited scleroderma)
Sclerodactyly
Telangiectasia

Chronic, slowly progressing autoimmune disease characterized by lymphocytic


infiltration of the exocrine glands resulting in xerostomia and dry eyes Sjogren's syndrome
(keratoconjunctivitis sicca)
PART 8 I HEMATOLOGY
High-Yield Physiology Concepts in Hematology
Yolk sac (3-8 weeks)
Liver (6 weeks-birth)
Fetal erythropoiesis
Spleen (10-28 weeks)
Bone marrow (18 weeks to adult)

Last RBCstage capable of mitosis Polychromatic erythroblast

Last stage ofRBC with a nucleus Orthochromatic erythroblast

Immature RBC released by the bone marrow into the blood Reticulocytes

Transfers iron in the blood Transferrin

Primary storage protein for Iron Ferritin

Supplemental storage protein for Iron Hemosiderin

Release histamine & heparin, involved in allergies Basophils

Increased in allergies and parasitic infections Eosinophils

Involved in bacterial infection and acute inflammation


\' Neutrophils

Osteoclasts (bones)
Kupffer cells (liver)
Tissue macrophages Histiocytes/Langerhans cells (skin)
Microglia (brain)
Alveolar macrophages (lungs)

Derived from megakaryocytes; last 8-10 days Platelets

Requires glycoprotein lb and van Willebrand Factor Platelet adhesion

Requires glycoprotein Ilb-Illa and fibrinogen Platelet aggregation

Secrete immunoglobulins (lg) Plasma cells (derived from B cells)

lg involved in the primary response; largest; pentameric lgM

lg involved in the secondary response; smallest (can penetrate


lgG
placental barrier)

lg in secretions (e.g. saliva, ?eyer's patches) lgA

Unclear function. Found in serum and on surface of B cells lgD

MHCl,CDB T-KillerCell

MHC11,CD4 T-Helper Cell


Histopathologic Findings in Systemic and Hematologic Diseases
DESCRIPTION MARKER DISEASE

Abnormal azurophilic
Toxic granules Severe infection
(primary) granules

Patches of dilated endoplasmic reticulum that Severe infection


Diihle bodies
appear as sky blue cytoplasmic puddles

Distinctive needle-like azurophilic granules found Acute Myelogenous


Auer rods
in myeloblasts Leukemia

Scattered macrophages with abundant wrinkled


Sea-blue histiocytes Chronic Myeloid Leukemia
green blue cytoplasm

Small lymphocytes disrupted in the process of Chronic Lymphoid


Smudge cells
making smears Leukemia

Giant 8 cells with bilobed nuclei with prominent


Reed-Sternberg (RS) cells Hodgkin's lymphoma
inclusions ("owl's eye")

Cells found in adultT-cell lymphoma which appear


to have multi-lobulated nuclei Cloverleaf or flower cells Adult T-cell lymphoma

Destructive plasma cell tumors involving axial


Plasmacytoma
skeleton

PAS(+) globular cytoplasmic inclusions


Russell bodies
(made oflg) Multiple myeloma

PAS (+)globular nuclear inclusions (made of lg) Dutcher bodies

M proteins causing RBCs to stick in linear arrays Rouleaux conformation

Erythroblasts with iron-laden mitochondria visible


Ringed sideroblasts Sideroblastic anemia
as perinuclear granules with Prussian Blue stain

Neutrophils with only two nuclear lobes Pseudo-Pelger-Hiiet cells


Myelodysplastic syndrome
Megakaryocytes with single nuclear lobes or
Pawn ball megakaryocytes
multiple separate nuclei

Premature release of nucleated erythroid and early


Leukoerythroblastosis
granulocyte progenitors
Primary myelofibrosis
Cells that were probably damaged during the
birthing process in the fibrotic marrow Teardrop cells or dacrocytes

Pentalaminar tubules, often with a dilated terminal Langerhans cell


end (tennis racket-like appearance) in EM Birbeck granules
histiocytosis

Small yellow•brown, brown or rust•colored foci in


Gandy·Gamna nodules Congestion of the spleen
the spleen
Acute and chronic leukemias
Most common cancer in children Acute lymphoblastic leukemia

Primarily a disease of older adults


Chronic lymphocytic leukemia
With smudge cells

AMLwith genetic abnormalities (Acute promyelocytic


Type of acute myeloid leukemia with the best prognosis leukemia with t(15;17)) (treatment is all trans-
retinoic acid)

AMLwith myelodysplasia•related changes and


Types of acute myeloid leukemia with worse prognosis
Therapy-related AML

Most important independent prognostic information Chromosome findings at diagnosis

Cut off blast percentage in peripheral blood and bone


20%
marrow for a diagnosis of AMLto be made

Background: iflymphocytic background, probably


Presumptive identity of blasts can be discerned through
lymphoblast; if granulocytic, probably myeloblast

Chronic myeloproliferative disorder associated with


t(9;22) (Philadelphia chromosome); CBC:WBCusually Chronic myeloid leukemia (CML)
> 100,000/mm' of immature granulocytic cells

An elderly with refractory anemia not attributed to any


other cause, or any abnormality in one or more cell Myelodysplastic syndrome
lineages should raise suspicion of

Most patients with myelodysplastic syndrome die as a Complications of pancytopenia and not due to
result of leukemic transformation

Hodgkin's (HL} versus Non-Hodgkin's Lymphoma (NHL}


HL NHL
A.

Reed-Sternberg cells + """

Associated with autoimmune diseases


and viral infections (eg, HIV,EBY,HTLV
""" +

Multiple peripheral nodes; extranodal involvement common,


non-contiguous spread
""" +

Low-grade fever, night sweats, weight loss + +

EBVassociation + +
Types of Hodgkin's Lymphoma
Most common type Nodular Sclerosis

Lymphocytes make up the vast majority of cellular infiltrate;


Lymphocyte-rich
mononuclear variant RScells; best prognosis

Relatively good prognosis; contains lymphohistiocytic popcorn" cells Lymphocyte predominant

Worst prognosis; associated with HIV, and presents as an advanced


Lymphocyte-depleted
disease

Types of Non-Hodgkin's Lymphoma ~v


Most common type Diffuse large 8-cell

Translocation between chromosomes 8 and 14; presents with starry-


sky pattern
Sheets of lymphocytes with interspersed "tingible body" Burkitt's lymphoma
macrophages
Associated with EBV

Biopsy reveals homogenous population of small lymphocytes, does


Mantle cell lymphoma
not have centroblasts and proliferation centers

Lymphomas that usually arise from sites of chronic inflammation Marginal zone lymphoma

Plasma Cell, Thymus and Spleen Disorders


Excess light or heavy chains along with complete lg synthesized by
neoplastic plasma cells Bence-Jones Proteins
Negative in urine dipstick

Tumor of the thymus associated with myasthenia gravis and pure


Thymoma

,
red cell aplasia
.,
CRAB
Hyper!;alcemia
Clinical features of Multiple myeloma Renal involvement
Anemia
~, y .!!_onelytic lesions ("punched out" on X-ray)
y
Histopathologic Findings in RBC-Related Diseases
DESCRIPTION MARKER ASSOCIATEDDISEASE

Small hyperchromic RBClacking


Spherocytes Hereditary spherocytosis
central pallor

Small dark nuclear remnants in


Howell-Jolly bodies Asplenia
RBCs of asplenic patients
.
Membrane-bound precipitates on
......
Heinz bodies
denatured globin chains

G6PD deficiency
RBCs with damaged membranes
due to removal of Heinz bodies by Bite cells
splenic macrophages

RBCs shaped like curved blades Sickled cells Sickle cell anemia

Dehydrated RBCs with bull's eye HbC disease, Asplenia,


appearance
Target cells ( codocytes J Liver disease, Thalassemia

MAHAs (eg, DIC,TTP/HUS, HELLP


Fragmented RBCs; also called
Schistocytes syndrome), mechanical hemolysis
helmet cells if cut in half
( eg, heart valve prosthesis]

Liver disease, abetalipoproteinemia,


RBCs with spikes Burr cells or echinocytes
vitamin E deficiency

High-Yield Concepts in Hemolytic Anemias


Triad of hemolytic anemia Anemia, jaundice and splenomegaly
,-.Y'
Hemoglobinuria (often associated with
Telltale sign of intravascular hemolysis
hemosiderinuria)

Autosornal dominant disorder caused by intrinsic defects


in the red cell membrane; Hereditary Spherocytosis
Increased MCHC

X-linked recessive disorder that reduces protection of


G6PD Deficiency
RBCs from oxidative injuries, leading to hemolysis

lntravascular hemolysis due to increased complement-


Paroxysmal Nocturnal Hemoglobinuria
mediated RBC Iysis

Hemolytic anemia seen in DIC,TTP-HUS, SLE, and


Microangiopathic Hemolytic Anemia
malignant hypertension
Classification of Hemolytic Anemias
lntracorpuscular defects Extracorpuscular defects

Hemoglobinopathies
Inherited Enzymopathies Familial [atypical) HUS
Membrane cytoskeletal defects

Mechanical destruction (microangiopathic)


Toxic agents
Acquired PNH Drugs
Infectious
Autoimmune

Summary of Bleeding Disorders


Platelet Bleeding Time PT PTT

Immune Thrombocytopenic
Decreased Prolonged Normal Normal
Purpura (ITP)

Bernard-Soulier Syndrome Decreased Prolonged Normal Normal

Glanzmann's Thrombasthenia Normal Prolonged Normal Normal

Von Willebrand Disease\e Normal Prolonged Normal Prolonged

Hemophilia A, B, or C Normal Normal Normal Prolonged

Vitamin K Deficiency Normal Normal Prolonged Prolonged

Disseminated lntravascular
Decreased Prolonged Prolonged Prolonged
Coagulation (DIC)

Anemias of Decreased Erythropoeisis


Impairment of DNA synthesis that leads to distinctive morphologic
Megaloblastic Anemia
changes

Most common hypoproliferative anemia


Iron Deficiency Anemia
Microcytic hypochromic anemia

Most distinguishing feature between true iron-deficiency anemia and the


Serum ferritin value
iron-restricted erythropoiesis associated with inflammation

Aplastic Anemia Pancytopenia with bone marrow hypocellularity


HEMATOLOGYAND BIOCHEMISTRYCORRELATION
Difference between Hemoglobin and Myoglobin
DESCRIPTION HEMOGLOBIN MYOGLOBIN

Heme-containing + +

Contains fibrous components --- ---


Level of structure exhibited Quaternary Tertiary

Tissues in the body where it is mostly found Blood Heart and muscle

Number of maximum bound oxygen molecules 4 1


\
Oxygen binding affected by pH and CO2 Yes No

Function in relationship with oxygen Oz transporter 02 reservoir

Has taut and relaxed forms Yes No

Curve exhibited in terms of 02 dissociation Sigmoidal Hyperbolic

High-Yield Concepts about Hemoglobin .


Most abundant form in adults Hemoglobin A

Used to determine levels of glucose by non-enzymatic addition of


HbA1C1
glucose to hemoglobin

Oxidation of the heme component of hemoglobin to iron which cannot


Methemoglobin
bind oxygen

Form of hemoglobin where CO binds tightly but reversibly Carboxyhemoglobin

Tetramer consisting of two alpha and gamma chains Fetal Hemoglobin

Gamma-tetramers in the newborns Hemoglobin Barts

1 Can be used to diagnose T2DM. Cut off value is >6.5%.


PART 9 I ONCOLOGY
High-Yield Concepts in Oncology

Most significant risk factor for cancer overall Age

Most common cancer worldwide and most common cause of cancer death Lung cancer

Second most common cancer worldwide Breast cancer

Genes involved in restraining cell growth; loss of function mutations lead to


Tumor suppressor genes
tumorigenesis

Mutated forms of genes involved in normal cellular growth, which are


Oncogenes
characterized by uncontrolled proliferation

Most effective means of treating cancer Surgery

Delivery of radiation therapy from a distance Teletherapy

Encapsulated sealed sources of radiation implanted directly or adjacent to tumor Brachytherapy

Radionuclides targeted to the site of the tumor Systemic radiation therapy

Most significant risk factor for head and neck cancer Alcohol and smoking

Most commonly used treatment for head and neck cancers Chemoradiotherapy

Serotonin receptor antagonists


Most effective drugs against highly emetogenic agents
(i.e. ondansetron)

~v
General Cancer Screening Recommendations for Asymptomatic Average-Risk
Patients
Screemng Recommended Frequency
Procedure

Sigmoidoscopy • Adults .2:SOyears old: screen every 5 years

FOBT (fecal
occult blood • Adults .2:SOyears old: screen every year
testing)
RI
.
Colonoscopy • Adults 2.SOyears old: screen every 10 years IIA

• All women who have reached the age of 21 (before this age, even in individuals that have begun
Pap smear sexual activity, screening may cause more harm than benefit)
• Recommended interval - 3 years

• Women 40-44 years: Provide the opportunity to begin annual screening


• Women 45-54 years: Screen annually
Mammography • Women 2:55 years: Transition to biennial screening or have the opportunity to continue annual
screening
• Women 2:40 should continue screening mammography as long as their overall health is good and
they have a life expectancy of 10 years or longer

• Age SO - men should talk to a doctor about the pros and cons of testing so they can decide if
testing is the right choice for them.
• If African American or have a father or brother who had prostate cancer before age 65, men
DREand PSA
should have this talk starting at age 45.
• For PSA,if men decide to be tested, they should have the PSAblood test with or without a rectal
exam

Most Commonly Used Tumor Markers and Immunohistochemical stains


cKIT (CD117) Gastrointestinal stromal tumor(GIST), Mastocytosis

hCG(human chorionic Hydatidiform moles and Choriocarcinomas (Gestational trophoblastic


gonadotropin) disease), testicular cancer, mixed germ cell tumor

Calcitonin Medullary thyroid cancer

Catecholamines Pheochromocytoma

Hepatocellular carcinoma, endodermal sinus tumor, mixed germ cell tumor,


AFP (alpha fetoprotein)
ataxia-telangiectasia, neural tube defects

CEA(carcinoembryonic antigen) Colon, pancreatic, breast, lung and ovarian cancer

Prostatic acid phosphatase, prostate


Prostate cancer
specific antigen (PSA)
Neuroendocrine tumors ( eg. small cell lung cancer, carcinoid tumor,
Neuron-specific enolase
neuroblastoma

Monoclonal immunoglobulins Myeloma

CD15andCD30 Hodgkin lymphoma

CD20+, CD23+, CDS CLL/SLL

CD138 Plasma cells (used to document plasmacytoid differentiation in lymphomas)

CA-125, HE-4 Ovarian cancer


II r,.

CA 19-9 Colon, pancreatic, and breast cancer

CA 15-3 Breast

Cancers of neural crest origin (melanomas, schwannomas, Langerhans cell


S-100
histiocytosis)

Oncogenes and Associated Malignancies


Melanoma, non-Hodgkin lymphoma, colorectal
BRAF carcinoma, papillary thyroid carcinoma, hairy
cell leukemia
BCR-ABL CML,ALL

BCL-2 Follicular lymphoma

C-myc Burkitt's lymphoma

MYCLl Lung

N-myc Neuroblastoma

RAS Colon, lung, pancreatic cancer

RET Multiple endocrine neoplasia (MEN) ZA and 2B

Tumor Suppressor Genes and Associated Malignancies


APC Colon cancer (FAP)

BRCAl, BRCA2 Breast and ovarian cancer

MLHl, MSH2 Hereditary nonpolyposis colon cancer

MENl MEN 1

NFl, NF2 Neurofibromatosis 1 and 2

p53 (Guardian of the genome] Li-Fraumeni Syndrome

Rb (Governor of the cell cyle) Retinoblastoma, osteosarcoma

VHL Von Hippe! Lindau Syndrome

WT1,WT2 Wilm'sTumor
Suspected Carcinogens and Associated Malignancies
Alkylating agents and benzene AML

Aromatic dyes and Schistosoma Bladder cancer (Aromatic dyes: urothelial; Schistosoma haematobium:
hematobium squamous)

Asbestos Lung cancer, mesothelioma

Burkitt's lymphoma, nasal T cell lymphoma, some types of Hodgkin


Epstein-Barr virus
lymphoma

Vaginal clear cell cancer (in daughters exposed to it during fetal


Diethylstilbestrol (DES)
development)

Lymphoma (usually 8-cell), Kaposi's sarcoma (HHV-8 Virus), Sqamous


HIV
cell carcinomas especially of the urogenital tract

Human Papilloma Virus (HPV) Cancers of the cervix and anus, head and neck cancer

HTLV•l Adult T-cell leukemia ..__...,


.
UV radiation (sunlight) Skin
~'\),
,
Vinyl chloride Liver angiosarcoma

Bladder, lung, esophageal, kidney, head and neck and pancreatic


Smoking
cancers

H.pylori Gastric adenocarcinoma and MALTlymphoma

HBV, HCV,aflatoxin-1, ethanol Hepatocellular carcinoma


High-Yield Concepts in Lung Cancer and Mesothelioma
Majority of lung tumors are Carcinomas

Primary cause of lung cancer worldwide Smoking

Types of lung cancer implicated with smoking which tend to present


Small cell and squamous cell lung cancer
centrally

Most common histology of lung cancer overall; most prevalent type


of lung cancer among women, young adults and non-smokers Adenocarcinoma of the lung
presenting peripherally

Results from local extension of tumor growing in the apex involving


Pancoast syndrome or superior sulcus
CS and T1-T2 nerve roots with shoulder pain radiating to the ulnar
tumor
distribution and Horner's syndrome

Most common life threatening metabolic complication of malignancy Hypercalcemia from ectopic PTH/PTH-
associated with squamous cell cancer of the lung related protein production

SIADH,Cushing's Syndrome, Lambert Eaton


Paraneoplastic syndromes associated with small cell lung cancer
Syndrome

Treatment of choice for small cell lung cancer Chemotherapy

Treatment of choice for early (stage 1 or 2) non-small cell lung cancer Surgical resection

lmmunostains for squamous cell carcinoma ,.... P63,P40

lmmunostains for adenocarcinoma ~"v' TIF-1, napsin A

lmmunostains for small cell carcinoma CD56, chromogranin, synaptophysin, NCAM

CKS/6, calretinin, and Wilms tumor gene-1


lmmunostains for mesothelioma
(WT-1)
'-'7

High-Yield Concepts in Breast Cancer


Early menarche
Breast cancer risk is increased in women with Late first full-term pregnancy
Late menopause

Best time for breast examination Days 5-7 of the menstrual cycle

Most important prognostic variable in breast cancer Tumor stage

Hormonal treatment for breast cancer which increases the risk of Selective estrogen receptor modulators
endometrial cancer (SERM)

Monoclonal antibody directed against the HER2/neu receptor used


Trastuzumab
for breast cancers

Method used to assess hormonal and HER-2 status of breast lmmunohistochemistry (IHC) with ER, PR,
carcinomas and HER-2/Neu
High-Yield Concepts in Gastric and Esophageal Cancers
Esophageal cancer related to smoking and alcohol,
Squamous cell carcinoma
arising in the upper 2/3

Esophageal cancer related to acid reflux and Barrett's


Adenocarcinoma
esophagus arising in the distal esophagus

Initial symptoms of esophageal cancer in majority of


Progressive dysphagia and weight Joss
patients

Threshold of dysphagia >60% of esophageal circumference is infiltrated

Type of gastric adenocarcinoma with loss of cell


cohesion developing throughout the stomach resulting
Diffuse type
to Joss of distensibility [linitis plastica or leather bottle
appearance)

Type of gastric adenocarcinoma frequently ulcerative


Intestinal type
and involving the antrum and lesser curvature

Low socio-economic class, H. pylori infection, ingestion


of high concentrations of nitrates in preserved foods,
Implicated risk factors for gastric cancer
Menetrier's disease (hypertrophy ofrugal folds), blood
group A

Gastric cancer metastatic to the ovary Krukenberg tumor

Gastric cancer metastatic to the periumbilical region Sister Mary Joseph nodes

Gastric cancer metastatic to the peritoneal cul-de-sac Blumer shelf

Gastric cancer metastatic to the supraclavicular lymph


Virchow nodes
nodes

Most common site for hematogenous spread of gastric


Liver
cancer

Complete surgical removal of the tumor with resection


Only chance of cure for gastric cancer
of adjacent lymph nodes

Most frequent site of extra-nodal lymphoma Stomach

Eradication of H. pylori (as long it has not transformed


Primary treatment of gastric MALTlymphoma
into DLBCL)
High-Yield Concepts in Colorectal Neoplasms
Sessile, serrated,
Characteristics of colonic polyps most associated with malignancy Villous architecture
Size of the polyp

FAP, osseous and soft tissue tumors, congenit.al hypertrophy of retinal


FAP (familial adenomatous polyposis)
pigment epithelium

Multiple polyps in the small and large intestines with osteomas, fibromas,
Gardner syndrome
and congenital hypertrophy of the retinal pigment epithelium

FAP or Lynch syndrome, malignant brain tumor (eg, medulloblastoma,


Turcot syndrome
glioma)

Multiple small and large intestinal polyps (hamartornatous/juvenile),


Peutz-Jeghers syndrome
mucocutaneous pigmentation, tumors of the ovary, breast, and pancreas

Hereditary autosomal dominant predisposition to colon, ovarian and Hereditary nonpolyposis colon cancer
endometrial cancers caused by defects in DNAmismatch repair (Lynch syndrome)

Most effective class of agents to reduce the risk of colon adenomas and
Aspirin and NSAIDs
carcinomas

Usually non•obstructive, discovered late, with iron•deficiency anemia Right•sided colon cancers

Usually with obstructive symptoms and apple·core or napkin ring


Left.sided colon cancers
deformity on barium studies

Hematochezia, tenesmus, narrowing of stool caliber


. Rectosigmoid cancers

Depth of tumor penetration into the


bowel wall
Prognosis for individuals having colorectal cancer is related to
Presence of both regional lymph node
involvement and distant metastases

Period of time when most recurrences after surgical resection of large


Within the first 4 years
bowel cancer occur

Number of sampled lymph nodes necessary to accurately define tumor


Minimum of 12 lymph nodes
stage during surgery

Most frequent visceral site of metastasis for colon cancer Liver

Backbone chemotherapeutic agent for colon cancer 5-fluorouracil (5-FU)

Major side effect of irinotecan used in FOLFIRIregimen for colon cancer Diarrhea

Common side effect of oxaliplatin used in FOLFOXregimen for colon


Dose•dependent sensory neuropathy
cancer
High-Yield Concepts in Hepatic, Pancreatic and Biliary Malignancies
Leading risk factor for pancreatic cancer for pancreatic cancer Smoking

Non-cirrhotic or Child-Pugh A cirrhosis


Candidates for resection in HCC Single lesion
No metastasis

Cirrhotic patients with single tumors S5


cm and portal hypertension (including
Liver transplantation is the first treatment choice for Child Pugh Band C) or with small
multinodular tumors (S3 nodules, each
S3cm)

Rare form of primary liver cancer that typically affects children and
Fibrolamellar HCC
young adults (10-30 years of age) without background liver disease

Most common benign liver tumor Hemangioma

Benign liver mass associated with contraceptive use and use of


Adenoma
anabolic androgenic steroids in male body builders
-
Benign liver mass
......."I'

Hyperplastic reaction of hepatocytes to an aberrant dystrophic artery Focal nodular hyperplasia


Marked by central stellate scar
Usually asymptomatic and detected incidentally

Vascular uptake orthe nodule in the arte-


Using contrast-enhanced imaging technique, the typical hallmark of
rial phase with washout in the portal
HCC
venous or delayed phases
-

Second most common liver cancer following HCC Cholangiocarcinoma

Clonorchis sinensis
Parasitic infection associated with cholangiocarcinoma
Opisthorchis viverrini

Nodular tumors arising at the bifurcation of the common bile duct Klatskin tumors

Palpable gallbladder associated with obstructive biliary malignancy Courvoisier's sign

Pylorus preserving
Standard surgical procedure for pancreatic head and uncinate tumors pancreaticoduodenectomy (modified
Whipple's procedure)
High-Yield Concepts in Genitourinary Malignancies
Painless hematuria ( either gross or
Initial manifestation of an underlying urinary tract cancer
microscopic)

Urothelial (formerly rererred to as


Most common histology of bladder cancer
transitional cell carcinoma)

Vaccine component used as intravesicular therapy in bladder cancer BCG

Leading risk factor for development of bladder cancer Smoking

Chronic infection of the bladder with this parasite can lead to squamous
Schistosoma hoematobium
cell carcinoma of the bladder

Most common histopathologic type of renal carcinoma Clear cell carcinoma

• Hematuria
Classic triad of renal cell carcinoma • Abdominal pain
• Palpable abdominal mass

Site where most prostate cancers develop Peripheral zone

5-alpha reductase inhibitors


Predominant therapy to reduce future risk of prostate cancer diagnosis
(finasteride/dutasteride)

Test to establish prostate cancer diagnosis Image-guided needle biopsy

Scoring used to measure histologic aggressiveness of the dominant and Gleason scoring (and WHO group
secondary glandular histology of prostate cancers grading)

High-Yield Concepts in Soft Tissue Malignancies

Most common presentation of soft tissue sarcomas Asymptomatic mass

Mainstay of treatment for Ewing's sarcoma, PNET,and rhabdomyosarcoma Chemotherapy

Most common malignant tumor of bone Plasma cell tumors

Malignant primary bone tumor with common location at the metaphysis of long bones
[often in knee region)
Osteosarcoma
Characteristic Codman triangle (from elevation ofperiosteum) or sunburst pattern on
x-ray

Most important prognostic factor for long-term survival in osteosarcoma Response to chemotherapy

Site most commonly involved in bone metastasis Vertebrae


High-Yield Concepts on Skin Malignancies
Inability to repair DNA pyrimidine dimers caused by UV exposure
Xeroderma pigmentosum
Presents with dry skin, photosensitivity, skin cancer

One or few small waxy, semitranslucent nodules forming around a


central depression that may be ulcerated, crusted or bleeding, edge is
Basal cell carcinoma
rolled or pearly with rodent ulcer, rarely metastasizes.
Most common cancer in humans

Most common site of basal cell carcinoma Face/head and neck area

Most common type of basal cell carcinoma Classical or nodular type

Dome-shaped, elevated, hard infiltrating lesion (deeply nodular),


Squamous cell carcinoma
may eventually develop an ulcer, occurs on sun•exposed areas

In situ form of squamous cell carcinoma Bowen's disease

Major risk factor for squamous cell carcinoma of the skin Chronic long term sun exposure

Known precursor lesion of squamous cell carcinoma of the skin Actinic keratosis

Most common type of melanoma Superficial spreading

• Asymmetry,
Characteristics of a malignant lesion melanoma (versus benign • Border irregularity
• .Color variegation,
nevus)
• Diameter >6mm,
• Chan~ in the lesion

Single greatest determinant of metastasis of melanoma Depth of invasion (Breslow thickness)

Presence of multiple benign or atypical nevi


Strongest risk factors for melanoma
Family or personal history of melanoma

Most common type of melanoma in dark-skinned individuals and


Acral-lentiginous Melanoma
Asians

Primary lesion erythematous edematous evanescent rash Wheal

Thickening of skin with accentuation of skin fold markings Lichenification

Loss of epidermis without loss of dermis Erosion

Loss of both epidermis and dermis Ulcer


Common Paraneoplastic Syndromes
Paraneoplastic Syndrome Associated Malignancy

PTHrP: squamous cell carcinoma (lung, head & neck, skin, breast, GU,GI)
Hypercalcemia
Increased vitamin D: lymphomas

Small cell carcinoma of the lung, carcinoid tumors, GI, GU,ovarian cancer,
SIADH
intracranial neoplasms

Ectopic ACTH:small cell lung cancer, carcinoid, pancreatic islet cell


Cushing's Syndrome
tumors, neural tumors

Hypoglycemia from IGF-2 excess Mesenchymal tumors, hepatocellular and adrenal carcinomas

Polycythemia Renal and hepatocellular cancer, cerebellar hernangiomas

Trousseau's Syndrome
Pancreatic cancer
(migratory thrombophlebitis)

""
Myasthenia gravis, pure red cell aplasia Thymoma

Lambert-Eaton myasthenic syndrome


Small cell carcinoma of the lung
(LEMS)

Common Chemotherapy Drugs: Actions and Adverse Effects


Alkylatmg Agents (Intercalate DNA; non-cell-cycle specific)

Alopecia, bone marrow toxicity, gonadal failure;


Cyclophosphamide Metabolized to acrolein
Can cause hemorrhagic give MESNA(antidote)

Cisplatin Ototoxicity, nephrotoxicity, neuropathy give amifostine (antidote)

Busulfan Pulmonary fibrosis

Nitrosoureas Neurotoxicity

Oxaliplatin Neurotoxicity

Procarbazine Disulfiram•like reaction

Ant1tumor antibiotics (Intercalate DNA; non•cell•cycle specific)

Bleomycin Pulmonary fibrosis

Dactinomycin/ Actinomycin D Myelosuppression

Doxorubicin/ Adriamycin Cardiotoxicity: give dexrazoxane (antidote)


Ant1metabohtes (Interfere with DNA synthesis; S-phase specific)

Myelosuppression give leucovorin


Methotrexate Inhibits dihydrofolate reductase
(antidote)

Myelosuppression give uridine


5-FU Inhibits thymidylate synthase
(antidote)

Azathioprine and Myelosuppression, increased toxicity with


6-Mercaptopurine Purine analogue intake of allopurinol (because these drugs
(6-MP) are metabolized by xanthine oxidase)

Cytarabine Pyrimidine analogue Pancytopenia

Hydroxyurea Inhibits ribonucleotide reductase Myelosuppresion, GI upset

M1crotubule lnh1b1tors (M-phase specific)

Hyperstabilize polymerized microtubules


Alopecia, hypersensitivity,
Taxanes preventing their break down during
rnyelosuppression
anaphase

Vinblastine: myelosuppression [Mnemonic:


Vinblastine and Attach to B tubulin and inhibit
"blast the bone marrow")
Vincristine polymerization
Vincristine: neurotoxicity
,'\,.

Topmsomerase lnh1b1tors

Etoposide, Teniposide Inhibit topoisornerase II Alopecia, GI irritation, myelosuppression

Irinotecan, Topotecan Inhibit topoisornerase I Diarrhea, severe myelosuppression


PART10IDERMATOLOGY
Erythematous Non-Scaly Papules
Erythematous papules, pustules, cysts, nodules, open and closed comedones on the face
Acne vulgaris
chest and upper back

Comedones (closed:
whiteheads, open:
Hallmark lesion of acne
blackheads a black due to
oxidation)

Rule out pregnancy


Most important consideration before initiating isotretinoin (vitamin A) therapy for
(isotretinoin is extremely
severe acne
teratogenic)

Erythematous pruritic or painful papules with central punctum


.. Insect bites

Erythematous macules papules more pruritic at night located at the groin, axilla, webs of
Scabies
fingers toes, elbows and wrists, other family members with similar lesions

Imaginary circle intersecting sites of involvement in scabies Circle of Hebra

Slightly elevated tortuous lines in the skin with a vesicle or pustule at the end containing
Burrows
the mite

Etiologic organism of scabies ~v Sarcoptes scabiei

Intense pruritus of the scalp, posterior cerv:ical lymphadenopathy, excoriations and


erythematous papules at the nape of the neck and the retroauricular area secondary Pediculosis capitis
impetigo, nits more common in retroauricular area, common in children

Discrete extremely pruritic erythematous papulovesicles accompanied by prickling


Miliaria rubra or prickly
burning or tingling, frequently on the antecubital, popliteal, trunk and inframammary
heat ("bungang araw")
areas, common in hot humid climates

Erythematous Non-Scaly Nodules


Acute, round, tender, circumscribed, perifollicular, erythematous lesion that ends in
Furuncle
central suppuration

Refers to two or more confluent furuncles Carbuncle

Refers to inflammation of the follicles resulting to erythematous papules that may


Folliculitis
eventually develop pustules

Most common causative agent Staphylococcus aureus


Erythematous Non-Scaly Plaques
Dark-red to purple skin discoloration, dusky with borders not clearly delineated, deeper tissue Necrotizing
involvement, pain out of proportion to the physical findings, rapid progression of lesion, may have crepitus fasciitis

History of wound or blister, erythematous area with non-distinct borders, warm, edematous, painful.
may have fever, central portion of lesion may become fluctuant and may rupture and discharge Cellulitis
purulent material

Most common portal of entry in the leg for cellulitis Tineapedis

Erythematous plaque, heat, swelling, highly characteristic raised indurated border, fever, systemic
Erysipelas
symptoms

Ill-defined hypopigmented macules and or plaque, with minimal sensory loss to light touch and Tuberculoid
temperature, low AFB bacterial counts on skin biopsy. (+) Lepromin skin test Granulomatous leprosy,
inflammation. mild/early

Macules papules plaques and nodules, nerves are enlarged and tender, progressive loss of hair, high Lepromatous
AFB bacterial counts, leonine facies. (-) Lepromin skin test Macrophages filled with AFB. leprosy

Loss of eyebrows in leprosy


-......v Madarosis

High-Yield Concepts on Eczematous Dermatitis


Erythematous greasy yellow brown scaling on scalp eyebrows, ears and perinasal
Seborrheic dermatitis
areas, dandruff, can spread beyond the hairline to the forehead

Most common location of seborrheic dermatitis Scalp presenting as dandruff

In infants yellow brown scaling on the scalp or seborrheic dermatitis of the scalp Cradle cap

Pityrosporum ovule - lipophilic


Organism implicated in seborrheic dermatitis
yeast of the Malassezia genus

Pruritic erythematous patches and plaques, scaling, lichenification, on the flexural


antecubital popliteal areas in adults and in the face and extensors in infants and Atopic dermatitis
children

Linear transverse fold below edge of the lower eyelids Dennie-Morgan folds

Discrete coin-shaped erythematous, edematous vesicular and crusted patches on


Nummular eczema
the lower extremities and extensor surfaces of the arms

Dermatitis sudden in onset, no previous history or exposure, symptoms of pain and


Irritant contact dermatitis
burning usually from acidic or alkali substances

Most common site of involvement of Irritant Contact Dermatitis Hands

Eczematous eruption following exposure to a known or unknown allergen, usually


appearing first at the site of contact, associated with plants, nickel and other Allergic contact dermatitis
compounds, can form patterns on the skin (i.e. linear lesions with plant exposure)

Most common cause of allergic contact dermatitis Exposure to plants


Papulosquamous Scaly Diseases
Erythematous papule and plaques covered with silvery scales on elbows, knees,
Psoriasis
scalp, with nail pits and other nail changes

Pinpoint bleeding spots from exposure of dermal papillae when scales are
Auspitz sign
scraped off in psoriasis

Test tubes on a rack


appearance of rete pegs, Munro
Major histopathologic findings in Psoriasis
microabscesses, Spongiform
pustule of Kogoj

Psoriasis involving the folds, recesses, and flexor areas such as axillae, groin,
Inverse psoriasis
inframammary folds

Abrupt eruption of psoriasis lesions following acute infection such as


Guttate psoriasis
streptococcal pharyngitis

Circular sharply circumscribed slightly erythematous dry scaly hypopigmented


patches with advancing scaly border and central clearing producing annular Tinea
outlines

Most common fungal disease ~~"? Tinea pedis

VY
Causes majority of tinea pedis Trichophyton rubrum

Infection of the nail plate ~v Onychomycosis

Multiple scaly hyper• or hypo•pigmented macules over the chest back abdomen
Tinea versicolor
and proximal extremities

Etiologic cause of Tinea versicolor Ma/assezia furfur

Classical microscopic finding in Tinea versicolorof short thick fungal hyphae and
Spaghetti and meatballs
large numbers of variously sized spores

Salmon colored macules and papules, collarette of scaling, scales tend to fold
along the long axis of line of stretch follows skin lines (hanging curtain or Pityriasis rosea
"christmas tree" sign), herald patch
Sexually-Transmitted and Vesiculobullous Diseases
Symmetrical, generalized, maculopapular eruptions, polymorphous, usually over the
face, shoulders, flanks and palms and soles with scaling, with suggestive sexual Secondary syphilis
history, painless genital ulcer

Papular lesions located on folds of moist skin usually around genitals and anus, may
become hypertrophic, forming soft red mushroom-like mass, moist weeping gray Condylomata lata
surface

Most frequent manifestation of orolabial herpes Fever blister or cold sore

Dew drop on rose petal, teardrop on an erythematous base, starting with macules
progressing to vesicles pustules and crusting, examination of lesions show different Varicella
ages of healing usually starting on the trunk spreading centripetally outward

Secondary bacterial
Most common complication of varicella
infection

Erythema, papules and plaques initially, mild pain a few days before, subsequently
Herpes zoster
developing vesicles and blisters following a dermatomal distribution, painful

Vesicles on the side and tip of nose indicative of ophthalmic zoster Hutchinson's sign
'
Involvement of the facial and auditory nerves by varicella zoster virus Ramsay-Hunt syndrome
Large tense blisters on flexor surfaces, groin axillae, and trunk, subepidermal blister,
Bullous pemphigoid
anti-hemidesmosome antibodies (Bullous pemphigoid antigens), linear IF pattern

Suprabasal blisters, anti-desmosome (desmogleins) antibodies, lace-like IF pattern Pemphigus vulgaris

Papules, vesicles and pustules with honey-colored crusts Impetigo contagiosa

Variant of impetigo, inadequately treated leading to punched out ulcerative lesions Ecthyma

High-Yield Concepts on Drug-Induced or -Related Reactions


Blisters, epidermal detachment resulting from epidermal necrosis, target
lesions, dusky purpuric macules with mucosa! involvement, <10% body surface Steven Johnson's Syndrome
area involved

>30% involvement of body surface area Toxic Epidermal Necrolysis (TEN)

10-30% involvement of body surface area SJS-TENOverlap

Sulfa drugs, anticonvulsants,


Drugs commonly associated with $JS-TEN nevirapine, allopurinol,
lamotrigine, oxicam NSAIDs

Multiple erythematous plaques with target or iris lesion morphology, usually


precipitated by recent new drug ingestion, often triggered by mycoplasma Erythema multiforme
pneumonia and HSV

Manual pressure to the skin may elicit separation of the epidermis (found in
Nikolsky's sign
staphylococcal scalded skin syndrome, SJS,TEN and pemphigus vulgaris)

Most common pattern of drug-induced reaction Morbilliform or maculopapular


Other Important Skin Diseases and Terminologies
Young children with individual lesions of smooth surfaced, firm, dome-shaped, pearly,
Molluscum contagiosum
fleshy papules with central umbilication

Numerous small eosinophilic and basophilic inclusion bodies found in histology of Henderson-Paterson
molluscum contagiosum bodies

Brown-black plaques with adherent greasy scales, stuck on appearance Seborrheic keratosis

Sudden appearance of multiple seborrheic keratoses suggestive of visceral and


Sign of Leser Trelat
hematologic malignancy

Erythematous macules and papules, macerated skin areas and satellite lesions, white
Candida! in(ection
friable patches on mucosa! surfaces, immunocompromised states

Increased thickness of the stratum corneum Hyperkeratosis

Hyperkeratosis with retention of nuclei in stratum corneum Parakeratosis

Epidermal accumulation of edematous fluid in intracellular space Spongiosis

Process referring to loss of cohesion between epidermal cells "-V Acantholysis

Violaceous flat-topped pa pules and plaques with gray lines (Wickham's striae) Lichen planus

Condition in dermatological diseases wherein traumatized areas tend to develop new


Koebner phenomenon
lesions (found in verruca and psoriasis)

Classes of Topical Corticosteroids According to Potency


., EXAMPLES
• Clobetasol propionate gel, ointment, cream, lotion, foam, spray and shampoo 0.05%
• Betamethasone di propionate gel and ointment 0.05%
Class 1 Superpotent
• Fluocinonide cream 0.1 %
• Halobetasol ro ionate ointment and cream 0.05%
• Betamethasone di propionate cream, lotion, gel and ointment 0.05%
• Clobetasol propionate solution ("scalp application") 0.05%
Class 2 • Fluocinonide gel, ointment, cream and solution 0.05%
• Mometasone furoate ointment 0.1 o/o
High Potency • Triamcinolone acetonide ointment 0.5%
• Betamethasone di propionate cream and lotion 0.05%
• Betamethasone valerate ointment 0.1 o/o
Class 3
• Fluticasone propionate ointment 0.005%
• Triamcinolone acetonide ointment 0.1% and cream 0.5%
• Fluocinolone acetonide ointment 0.025%
Class 4
• Mometasone furoate cream and lotion 0.1 %
• Betamethasone di propionate lotion 0.05%
Medium
• Betamethasone valerate cream and lotion 0.1 o/o
Potency
Class 5 • Fluocinolone acetonide cream 0.025%
• Fluticasone propionate cream and lotion 0.05%
• Triamcinolone acetonide ointment 0.025% and lotion 0.1%
• Betamethasone valerate lotion 0.1 o/o
• Desonide gel, ointment, cream, lotion and foam 0.05%
Class 6
Low Potency • Fluocinolone acetonide cream and solution 0.01 o/o
• Triamcinolone acetonide cream and lotion 0.025%
Class 7 • To icals with h drocortisone
PART11INEUROLOGY
ANS RECEPTORS:ADRENORECEPTORSfSYMPAl
ADRENO-RECEPTOR MOA EFFECT

• Gq protein
Alpha-1 Receptors (Al) • Causes smooth muscle contraction
• increased IP3/Ca2+

• Seen in sympathetic postganglionic presynaptic


nerve terminals . Also seen in platelet, fat cells,
• Gi protein
Alpha-2 Receptors (AZ) walls of the GIT
• decreased cAMP
• Causes reuptake of NE or smooth muscle
relaxation

• Seen in the heart (SA Node, AV Node, ventricles)


• Gs protein
Beta-1 Receptors (Bl) and kidneys
• increased cAMP
• Causes excitation

• Gs protein
Beta-2 Receptors (B2) • Causes smooth muscle relaxation
• increased cAMP

ANS RECEPTORS:CHOLINORECEPTORS
(BOTH SYMPA& PARA)
RECEPTOR LOCATION MOA

Nicotinic Receptors

• Binds with ACh,


NM(Nt) • Skeletal Muscle MEP
• Opens Na-K Channel

• Binds with Ach


NN (Nz) • Autonomic Ganglia (dendrites of Postganglionic Neurons)
• Opens Na-K Channel

Muscarinic Receptors

• Binds with Ach, Gq,


Ml • CNS
• Increased IP3/Ca2+

• Binds with Ach, Gi,


M2 • Heart
• Decreased cAMP

• Glands • Binds with Ach,


M3
• Smooth muscles • Gq, increased IP3/Ca2+
Neurotransmitters
CHARACTERISTICS NEUROTRANSMITTER (NT]

Used in the neuromuscular junction, and both sympathetic and parasympathetic


Acetylcholine
nervous system; triggers REMsleep

NT secreted by postganglionic sympathetic neurons Norepinephrine (NE)

NT mainly secreted by the adrenal medulla; has greater fl-2effect than NE Epinephrine

Secreted by substantia nigra to fine tune movement and by the hypothalamus to


Dopamine
decrease prolactin

Comes from tryptophan; low levels are associated with depression Serotonin

Permeant gas, inhibitory NT, vasodilator Nitric Oxide (NO)

Main inhibitory NT of the spinal cord; increases Cl· influx Glycine

Main inhibitory NT of the brain; increases Cl· influx or K• efflux GABA

Main excitatory NT of the brain; formed from reactive amination of alpha-


Glutamate
ketoglutarate
'-
Involved in fast pain
"-'t'\..~ Glutamate

Involved in slow pain ~\..Y Substance P

High-Yield Physiology Concepts Relating to Cranial Nerves


Cranial nerve for opening of eyelids, contraction of most EOMs, accommodation
CN Ill (Oculomotor nerve)
and pupillary constriction (miosis)

Cranial nerve for special sensation (taste) of the anterior 2/3 of the tongue CN VII (Facial nerve)

Cranial nerve for general sensation (e.g. pain) of the anterior 2/3 of the tongue CN V {Trigeminal nerve)

Cranial nerve for facial muscles CN VII (Facial nerve)

Cranial nerve for facial sensation and muscles of mastication CN V {Trigeminal nerve)

High-Yield Physiology Concepts Relating to the Autonomic Nervous System


Mydriasis, sweating, increased heart rate, bronchodilation, GU and GI contraction, uterine
Sympathetic
relaxation and contraction, vasodilation of the skeletal muscles, vasoconstriction of the skin and
effects
GI tract, ejaculation

Mnemonic: "PLASMA":
Parasympathetic
Parasympathetic, Long Pre-Ganglionic Tract, Ach used, Short Post-Ganglionic Tract, Muscarinic
Nervous System
Receptors, Ach used.

Alpha-1 (for smooth muscle contraction in sphincters, radial muscle of the iris, vasoconstriction)
Adrenergic Alpha-2 [in pre-synaptic terminals),
Receptors Beta-1 (heart and kidney),
Beta-2 (for smooth muscle relaxation in bronchiolar muscles, uterus, vasodilation)
DISEASESOF THE NERVOUSSYSTEM
High-Yield Concepts Related to Headaches
Highly characteristic of posterior fossa brain tumors Vomiting that precedes headache

Dominant symptom in temporal (giant cell) arteritis Headache

Most common primary headache syndromes Migraine, tension-type headache, and cluster headache

Key pathway for pain in migraine Trigeminovascular input from the meningeal vessels

Most disabling headache Migraine

Judicious use of one or more drugs that are effective in


Mainstays of pharmacologic therapy in migraine
migraine

Most important factor in selection of the optimal


Severity of the attack
regimen for a migraine patient

Most effective drug classes in the treatment of Anti-inflammatory agents, 5-HT 1e/ 10 receptor agonists
migraine (triptans), and dopamine receptor antagonists

Most efficacious of the triptans Rizatriptan and eletriptan

Core feature of cluster headache Periodicity

Administration of drugs to prevent cluster attacks until


Most satisfactory treatment in cluster headache
the bout is over

Most serious cause of secondary headache Subarachnoid hemorrhage

Classic headache associated with a brain tumor Most evident in the morning and improves during the day

High-Yield Concepts Related to CNSTumors


Preferred diagnostic test for any patient suspected of having a
brain tumor, and should be performed with gadolinium Cranial MRI
contrast administration

The only test necessary to diagnose a brain tumor Neuroirnaging


Glucocorticoid of choice for brain tumors because of its
Dexamethasone
relatively low mineralocorticoid activity
Exposure to ionizing radiation: meningiomas,
The only established risk factors for primary brain tumors gliomas, schwannomas
lmmunosuppression: primary CNSlymphoma

Most are primary brain tumors


Important general concepts about brain tumors In adults: usually supratentorial
In children: usually infratentorial

Grade I astrocytomas: pilocytic astrocytomas


Most common primary brain tumor of childhood
(WHO grade I}
Most common orimarv brain tumor overall Menine:iomas
Frequently plays an important role in the pathogenesis of HIV-
Epstein-Barr Virus (EBY)
related primary CNSlymphoma

Most common malignant brain tumor of childhood Medulloblastomas

Most common malignant brain tumor overall Grade IVastrocytoma (Glioblastoma)

Meningiomas are most commonly located over the Cerebral convexities

Main differential diagnosis for meningioma Dural metastasis

Most common schwannomas Vestibular schwannomas or acoustic neuromas

Most common site of brain metastases Gray matter-white matter junction

85% of all brain metastases are Supratentorial

Most common sources of brain metastases Lung and breast cancer

Malignancy with greatest propensity for brain metastasis,


Melanoma
found in 80% of patients at autopsy

Malignancies with propensity to metastasize to the dura and


Prostate and breast cancer
can mimic meningioma

Arises in prostate and breast cancer, which have strong


Spinal cord compression
propensity to metastasize to axial skeleton

Bram metastases are best visualized on -


MRI
where they appear as well-circumscribed lesions

Most common sources of brain metastases Lung and breast carcinoma


c....
Greatest propensity to metastasize to the brain ' Melanoma

Solid tumors that most frequently cause leptomeningeal Breast and lung carcinomas and hematologic
metastases malignancies
Definitive method and often considered the gold standard to
Demonstration of tumor cells in CSF
dia1mose lentomeninf!eal metastases
CSF cytologic examination is most useful in Hematologic malignancies

Part of the spine affected most commonly in epidural Thoracic spine, followed by the lumbar and then
metastasis cervical spine
Presenting symptom of epidural metastasis in virtually all
Back pain
patients
Best test for epidural metastasis MRIof the complete spine

Complete removal of the mass, typically anterior


Surgical procedure of choice for epidural metastasis
to the spinal canal

Primary radiation oncology approach to brain metastases Stereotactic radiosurgery

Most serious toxicity from radiotherapy as they are often


Late delayed toxicity
irreversible

Second only to myelosuppression as dose-limiting toxicity of


Neurotoxicity
chemotherapeutic agents
High-Yield Concepts in CNSInfections
Identify whether an infection predominantly involves
the subarachnoid space (meningitis) or whether there
First task in the approach to CNS infection is evidence of either generalized or focal involvement
of brain tissue in the cerebral hemispheres,
cerebellum, or brainstem

Encephalitis Brain tissue is directly injured by a viral infection

Classic clinical triad of meningitis Fever, headache, and nuchal rigidity

Headache, fever, focal neurologic deficit (present in <


Classic clinical triad of brain abscess
50%)

Cerebritis Non-encapsulated brain abscess

Pathognomonic sign of meningeal irritation and is


Nuchal rigidity ("stiff neck")
present when neck resists passive flexion

Most common form of suppurative CNS infection Bacterial meningitis

Most common cause of meningitis in adults >20 years Streptococcus pneumoniae

Important clue to diagnosis of meningococcal infection Petechial or purpuric skin lesions

Most disastrous complication of increased ICP Cerebral herniation

Most common etiologic organisms of community-


Streptococcus pneumoniae and Neisseria meningitidis
acquired bacterial meningitis

Most common agents in acute viral meningitis Enteroviruses

Temporal encephalitis HSV

Thalarnic encephalitis Japanese B encephalitis virus

Subacute sclerosing panencephalitis Measles virus

Most common symptom of brain abscess Headache

Combination of high dose parenteral antibiotics and


Optimal therapy for brain abscess
neurosurgical drainage
High Yield Concepts Related to Seizures
Paroxysmal event due to abnormal excessive or
Seizure
synchronous neuronal activity in the brain

Condition where a person has recurrent seizures


due to underlying causes not associated with Epilepsy
structural brain damage

1) Multiple seizure types (usually generalized tonic-clonic,


atonic, and atypical absence seizures)
Triad of Lennox-Gastaut Syndrome 2) EEG showing slow (<3 Hz) spike-and-wave discharges
and a variety of other abnormalities
3) Impaired cognitive function in most but not all cases

Highest incidence of seizures Early childhood and late adulthood

Daydreaming and a decline in school performance


First clue of typical absence seizures
recognized by the teacher

Generalized, symmetric, 3-Hz spike-and-wave discharge


Electrophysiologic hallmark of typical absence
that begins and ends abruptly, superimposed on a normal
seizures
EEGbackground

Main seizure type in 10% of all persons with


Generalized tonic-clonic seizures
epilepsy

Most common seizure type resulting from metabolic


Generalized tonic-clonic seizures
derangements

• Metabolic disorders
Pathologic myoclonus is most commonly seen in
• Degenerative CNSdiseases
association with
• Anoxic brain injury

Most common syndrome associated with focal


Mesia) Temporal Lobe Epilepsy Syndrome
seizures with dyscognitive features

Most common seizures arising in late infancy and


Febrile seizures
early childhood

First goal in the approach to seizure Determine if event was truly a seizure

• Valproic acid
Best initial choice for the treatment of primary
• Lamotrigine
generalized tonic-clonic seizures
• Levetiracetam

Drugs of choice approved for the initial treatment of Carbamazepine (or a related drug, oxcarbazepine),
focal seizures lamotrigine, phenytoin, and levetiracetam

Key determinants in initiation and monitoring of Clinical measures of seizure frequency and presence of
therapy side effects, not the laboratory values

Most recurrences of seizure occur in the First 3 months after discontinuing therapy

Resection of the anteromedial temporal lobe (temporal


Most common surgical procedure for patients with
lobectomy) or a more limited removal of the underlying
temporal lobe epilepsy
hippocampus and amygdala (amygdalohippocampectomy).
High-Yield Concepts Related to Cerebrovascular Diseases
Requires that all neurologic signs and symptoms resolve within
24 hours (most last <lhr) regardless of whether there is an Transient lschemic Response (TIA}
imaging evidence of new permanent brain injury

Occurred if the neurologic signs last for more than 24 hours Stroke

Tissue surrounding the core region of infarction is ischemic but


Ischemic Penumbra
reversibly dysfunctional

Refers to infarction following atherothrombotic or


Lacunar Infarction
lipohyalinotic occlusion of a small artery
Decrease in cerebral blood flow to zero causes death of brain
4-10 minutes
tissue within

Most common cause of cerebral embolism overall Nonrheumatic atrial fibrillation

Most common source of artery to artery embolism Carotid bifurcation atherosclerosis

Atherosclerosis within the carotid artery occurs most Common carotid bifurcation and proximal
frequently within internal carotid artery

Herald a small-vessel infarct Transient symptoms (small vessel TIA}

Most significant risk factor of stroke and TIA Hypertension

Only antiplatelet agent that has proven effective for the acute
Aspirin
treatment of ischemic stroke

Principal side effect of dipyridamole Headache

Endarterectomy for carotid atherosclerosis is most beneficial Within 2 weeks of symptom onset (benefit is
when performed more pronounced in men>75 years)

Sudden onset of bilateral signs, including ptosis,


Hallmark of top of the basilar artery occlusion papillary asymmetry or lack of reaction to light
and somnolence
Imaging modality of choice in patients with acute stroke to rule
Plain cranial CT
out bleed
Gold standard for identifying and quantifying atherosclerotic
Conventional x-ray cerebral angiography
stenoses of arteries

Bleeding into subdural and epidural spaces is principally


Trauma
produced by

Most common sites of hypertensive intraparenchymal Basal ganglia (especially putamen}, thalamus,
hemorrhage cerebellum, pons

Putamen (Sentinel) sign: contralateral


Most common site of hypertensive hemorrhage
hemiparesis)

Most common cause of lobar hemorrhage in the elderly Cerebral amyloid angiopathy

• Choriocarcinoma
Most common metastatic tumors associated with intracerebral
• Mali~nant melanoma
hemorrhage
• Renal cell carcinoma
Most cerebellar hematomas of this diameter will require
>3 cm in diameter
surgical evacuation
High Yield Concepts Related to Nerve Disorders
Most common cause of peripheral neuropathy Diabetes mellitus

Median neuropatby at tbe wrist and ulnar


Most common diabetic mononeuropathies
neuropathy at the elbow

Seventh nerve palsy, followed by third nerve,


Most common cranial mononeuropathies in DM
sixth nerve, and, less frequently, fourth nerve
palsies

Most common cranial mononeuropathies in scleroderma Trigeminal nerve

Most common cranial nerve involved in sarcoidosis Seventh nerve

Most common mononeuropathies in uremia Carpal tunnel syndrome

Most common form of peripheral neuropathy associated with


Distal symmetric polyneuropatby
HIV infection & usually seen in patients with AIDS

Most common associated malignancy with neuropathies Lung cancer

Most common causes of acute generalized weakness leading to


GBSand myasthenia gravis
admission to ICU

Manifests as a rapidly evolving symmetric, ascending. areflexic


motor paralysis with or without sensory disturbance,
Guillain Barre Syndrome (GBS)
associated with Campylobacter jejuni found in undercooked
chicken

Neuromuscular disorder characterized by weakness and


fatigability of skeletal muscles, caused by a decrease in the
number of available acetylcholine receptors (AChRs) at Myastbenia Gravis (MG)
neuromuscular junctions due to an antibody-mediated
autoimmune attack, associated with diplopia
High-Yield Concepts Related to Movement Disorders
Most common form of Parkinsonism Parkinson's Disease (PD)

Most common cause of Familial PD Mutations of the LRRK2 gene

Most common cause of secondary Parkinsonism Dopamine blocking agents

Protein misfolding and accumulation and


Most significant pathogenic mechanism in Parkinsonisrn
mitochondrial dysfunction

Mainstay of therapy for PD Levodopa-carbidopa

Major clinical effect of central-acting anticholinergic drugs Tremors

Most widely used antidyskinesia agent in patients with


advanced PD and the only oral agent that has been Amantadine
demonstrated in controlled studies to reduce dyskinesia

Most common cause of nursing home placement in PD patients Dementia

Most common movement disorder Essential tremor

Major differentials for tremors Dystonic tremor or PD


t::-....
Standard drug therapies for essential tremors
S$>-' jl-blockers or primidone

Most common forms of dystonia Focal dystonias

Most commonly seen with neuroleptic drugs or after chronic


Drug-induced dystonia
levodopa treatment in PD patients

Most common systemic disorder that causes chorea SLE

Most common acute hyperkinetic drug reaction Dystonia

Most common subacute drug reaction Akathisia

Gold standard for diagnosis of Wilson's disease Liver Biopsy

Most common psychogenic movement disorder Tremor affecting tbe upper limbs
NERVOUSSYSTEM:PHARMACOLOGY
Sedative Hypnotics
Midazolam Used in acute anxiety attacks, anesthesia induction, preoperative sedation

Diazepam Used in seizure disorders (status epilepticus), alcohol withdrawal, tranquilizer

Flunitrazepam Date-rape drug

Flumazenil Antidote to benzodiazepine overdose


,..._,
Thiopental Used in anesthesia induction, Lethal injection, Truth serum

Phenobarbital Used in seizure disorders in children, can precipitate porphyria, potent inducer of CYP450

GABAreceptor effects Sedative-hypnotic poisoning


FREnzodiazepines: FREquency Hot Hot Hot DeCisioN!
BarbiDURATes: DURATion Hypothermia
zzZZzzZZzzZZzz (sleep) Hypotension
Zolpidem, Zaleplon = Zleep disorders Hypoactive BS
Disinhibition
Buspirone for Busy People Coma
(Always Anxious) Nystagmus
BuSPirone like your BenzodiaSePine

Alcohols
Most frequently abused drug, causes Wernicke-Korsakoff syndrome in overdose and
Ethanol
delirium tremens in withdrawal

Thiamine Used for prevention of Wernicke-Korsakoff syndrome

Diazeparn Used for treatment of alcohol withdrawal

Methanol Wood alcohol, causes visual dysfunction due to formaldehyde accumulation

Ethylene glycol Found in antifreeze, causes nephrotoxicity due to oxalic acid accumulation

Fornepizole Alcohol dehydrogenase inhibitor

Disulfirarn Acetaldehyde dehydrogenase inhibitor


y
DrugsCausingDisulftram-likeReactions
Clara took the Pre-Medical Test in the PM
Chlorpropamide
CefoPerazone
CefaMandole
CefoTetan
Procarbazine
Metronidazole
Anti-Seizure Medications
Seizure Type Drugs FIRST LINEChoice AlterALTERNATIVESDrugs

Lamotrigine Zonisarnide
Valproic acid Phenytoin
Levetiracetam
Carbamazepine
Generalized Tonic-Clonic Seizures Oxcarbazepine
Topiramate
Phenobarbital
Primidone
Felbamate

Lamotrigine Zonisarnide
Carbamazepine Brivaracetam
Oxcarbazepine Topiramate
Phenytoin Valproic acid
Levetiracetam Tiagabine
Focal Seizures
Gabapentin
Lacosamide
Phenobarbital
Primidone
Felbamate

Valproic acid Larnotrigine


Typical Absence Seizures Ethosuximide Clonazepam
Lamotrigine

Valproic acid Clonazepam


Myoclonic and Atypical Absence Lamotrigine Felbamate
Syndromes Topiramate Clobazam
Rufinamide
EPONYMS
Eponym Descnpt1on
• Characterized by Confusion, Ophthalmoplegia, Ataxia
Wernicke Encephalopathy
• COA
• Caused by chronic alcohol overuse
Korsakoff Syndrome
• Confabulation, personality changes, memory loss (permanent) A.
• Deficiency ofThl 7 cells leading to defective neutrophils
Autosomal Dominant Hyper-lgE
• Presents with recurrent cold (non inflamed) abscesses, eczema, increased
Syndrome Ooh Syndrome]
eosinophils and serum lgE
• Mutation in WAS gene
Wiskott-Aldrich Syndrome
• Thrombocytopenia, infections, eczema
• Presents with fever, chills, headache, myalgia after antibiotics treatment for
Jarisch-Herxheimer Reaction syphilis are started
• Due to host response to sudden release of bacterial antigens
• Dilated cardiomyopathy
• Megacolon, megaesophagus
Chagas Disease
• Unilateral periorbital swelling (Romafia sign)
• Transmitted by kissing bug
• Sporadic (some familial forms]
• Rapidly progressive dementia
Creutzfeldt-Jakob Disease
• Transmitted by contaminated materials such as corneal transplant,
neurosurgical equipment
Bovine Spongiform • Mad cow disease
Encephalopathy
• Inflammation of liver capsule and "violin string" adhesions of peritoneum to
Fitz-Hugh-Curtis Syndrome liver caused by gonococcal infection
• Multiple seborrheic keratoses associated with GI and other visceral
Sign of Leser-Trelat malignancies
Good Syndrome • Hypogammaglobulinemia
Trousseau Syndrome • Migratory superficial thrombophlebitis
Lambert-Eaton Myasthenic • Antibodies against presynaptic calcium channels at NMJassociated with SCLC
Svndrome
• Antibodies against postsynaptic Ach receptors at NMJassociated with
Myasthenia Gravis thymoma
• Pulmonary infundibular stenosis (most important for prognosis)
• Right ventricular hypertrophy [RVH]- boot-shaped heart on chest xray
Tetralogy of Fallot
• Overriding aorta
• VSD
• Displacement of tricuspid valve downward into RV
Ebstein Anomaly
• Associated with lithium exposure in utero
Eisenmenger Syndrome • Conversion of an initial L to R shunt into a R to L shunt
• Autosomal dominant
• Due to loss of function mutation of sodium channels.
Brugada Syndrome
• ECGshows pseudo-right bundle branch block and ST-segment elevations in
leads V1-V2
• Prolonged QRS duration (produced by slow conduction through direct
activation of ventricular myocardium over the accessory pathway)
Wolff-Parkinson-White
Syndrome • Short P-R interval (<0.12 s)
• Slurring of the initial part of the QRS complex (delta wave)
Dressler Syndrome • Fibrinous Pericarditis (bread & butter pericarditis) post-Ml
• Paradoxical increase in JVP on inspiration
Kussmaul Sign • Seen in constrictive pericarditis, restrictive cardiomyopathy, right heart
failure, massive pulmonary embolism, right atrial or ventricular tumors.
Osler•Weber-Rendu syndrome • Autosomal dominant
Hereditary Hemorrhagic • Telangiectasias on skin and mucous membranes, epistaxis, skin
Telangiectasia discolorations, arteriovenous malformations (AVMs),GI bleeding, hematuria.
• Autoimmune hypothyroidism that may be associated with a goiter; with
Hashimoto Thyroiditis germinal centers, fibrosis, atrophy of the thyroid follicles accompanied by
oxyphil metaplasia, absence of colloid.
• Insidious, painless goiter with local symptoms due to compression of the
Riedel Thyroditis esophagus, trachea, neck veins, or recurrent laryngeal nerves
• Hard, non tender, often asymmetric, and fixed goiter
• Most common cause of hyperthyroidism.
Graves Disease
• Presents with hyperthyroidism, ophthalmopathy, dermopathy
• Iodine-induced hyperthyroidism
Jod-Basedow Phenomenon • Occurs after iodine IV contrast or amiodarone use.
• Opposite to Wolff-Chai4off effect
• Constellation of clinical features that result from chronic exposure to excess
Cushing Syndrome glucocorticoids of any etiology.
• Enlargement of pre-existing ACTH-secreting pituitary adenoma after
Nelson Syndrome bilateral adrenalectomy for refractory Cushing disease
• Primary adrenal insufficiency
Addison Disease
• Most commonly due to autoimmune adrenalitis
Waterhouse-Friderichsen • Acute primary adrenal insufficiency due to adrenal hemorrhage associated
Syndrome with septicemia
• Severe peptic ulcer diathesis secondary to gastric acid hypersecretion due to
Zollinger-Ellison Syndrome unregulated gastrin release from gastrinomas
Menetrier Disease • Large, tortuous gastric mucosa! folds (not a form of gastritis)
Virchow Node • Left supraclavicular node by metastasis from stomach.
• Bilateral (usually) metastasis to ovaries
Krukenberg Tumor
• Abundant mucin-secreting, signet ring cells
Sister Mary Joseph Nodule • Subcutaneous periumbilical metastasis.
Blumer Shelf • Palpable mass on DRE suggesting metastasis to pouch of Douglas
• aka gluten-sensitive enteropathy or celiac sprue
Celiac Disease • Autoimmune-mediated intolerance of gliadin (gluten protein found in wheat,
barley, rye J
• Infection with Tropheryma whipplei (intracellular gram positive)
• Symptoms include: Cardiac symptoms, Arthralgias, and Neurologic
Whipple Disease symptoms. Diarrhea/steatorrhea occur later in disease course
• Most common in older males.
• Affects any portion of the GI tract, usually the terminal ileum and colon
Crohn Disease • Has skip lesions and spares the rectum
• Transmural inflammation leading to fistulas
Zenker Diverticulum • Pharyngoesophageal false diverticulum
• True diverticulum
Meckel Diverticulum • Persistence of the vitelline (omphalomesenteric) duct
• May harbor ectopic acid-secreting gastric mucosa and/or pancreatic tissue
• Lack of ganglion cells/enteric nervous plexuses (Auerbach and Meissner
Hirschsprung Disease plexuses) in distal segment of colon
• Congenital megacolon
• Multiple polyps in the small and large intestines with osteomas, fibromas, and
Gardner Syndrome congenital hypertrophy of the retinal pigment epithelium
• FAP or Lynch syndrome, malignant brain tumor (eg, medulloblastoma,
Turcot Syndrome glioma)
• Multiple small and large intestinal polyps (hamartomatous/juvenile),
Peutz-Jeghers Syndrome mucocutaneous pigmentation, tumors of the ovary, breast, and pancreas
• aka hereditary nonpolyposis colorectal cancer (HNPCC
• Autosomal dominant mutation of mismatch repair genes with subsequent
Lynch Syndrome microsatellite instability.
• Associated with endometrial, ovarian, and skin cancers.
• Hepatic venous outflow tract obstruction
Budd-Chiari syndrome • Associated with hypercoagulable states, polycythemia vera, postpartum state,
HCC.May cause nutmeg liver (mottled appearance).
• Mildly decreased UDP-glucuronosyltransferase conjugation
Gilbert syndrome
• Asymptomatic or mild jaundice usually with stress, illness, or fasting.
• Absent UDP-glucuronosyltransferase
Crigler-Najjar
• Presents early in life but some patients may not have neurologic signs until
syndrome, type I
later in life.
Dubin-Johnson • Conjugated hyperbilirubinemia due to defective liver excretion.
syndrome • Black liver due to impaired excretion of epinephrine metabolites
• Phenotypically similar to Dubin-Johnson, but milder in presentation
Rotor syndrome
• No black liver
• Aka hepatolenticular degeneration
• Copper accumulation in liver, brain, cornea, kidneys
Wilson disease • Increased urine copper
• May present with Kayser-Fleischer rings (deposits in Descemet membrane of
cornea)
• Basophilic nuclear remnants removed by splenic macrophages
Howell-Jolly bodies
• Associated with functional hyposplenia (eg, sickle cell disease), asplenia
• Denatured and precipitated hemoglobin (contain iron)
Heinz bodies • Phagocytic removal of Heinz bodies result to bite cells
• Associated with G6PD deficiency
Bernard-Soulier • Autosomal recessive defect in adhesion.
syndrome • Defective Gplb lead to decreased platelet-to-vWF adhesion.
• Autosomal recessive defect in aggregation.
Glanzmann
thrombasthenia
• Defective Gpllb/llla lead to decreased platelet-to-platelet aggregation and
defective platelet plug formation
• Intrinsic pathway coagulation defect:
von Willebrand • Defect in platelet-to-vWF adhesion
disease • Mostly autosomal dominant
• Most common inherited bleeding disorder
• (+)Reed-Sternberg cells - CDlS+ and CD30+ B-cell origin
Hodgkin lymphoma
• Binucleate or bilobed, "owl eyes"
• "Starry sky" appearance
Burkitt lymphoma • Sheets of lymphocytes with interspersed "tingible body" macrophages.
• Associated with EBY
• Overproduction of lgM
Waldenstrom
• Bone marrow analysis shows > 10% small lymphocytes with intranuclear
macroglobulinemia
pseudoinclusions containing lgM (lymphoplasmacytic lymphoma)
• Group of proliferative disorders of Langerhans cells
Langerhans cell • Presents as lytic bone lesions and skin rash or recurrent otitis media
histiocytosis
• Birbeck granules ("tennis rackets" or rod shaped on EM) are seen
Erb palsy ["waiter's tip") • Injury to upper trunk ofbrachial plexus: CS-C6 roots
Klumpke palsy • Injury to lower trunk ofbrachial plexus: CS-Tl roots
• Aka traction apophysitis
Osgood-Schlatter • Overuse injury caused by repetitive strain and chronic avulsion of the
disease secondary ossification center of proximal tibial tubercle.
• Common in athletes
• Idiopathic avascular necrosis of femoral head
Legg-Calve-Perthes • Commonly presents between 5-7 years with insidious onset of hip pain that
disease may cause child to limp.
• More common in males
• Commonly affects diaphysis of long bones (especially femur), pelvic flat
bones.
• Anaplastic small blue cells of neuroectodermal origin (resemble
lymphocytes).
Ewing sarcoma
• Has t(11;22) (fusion protein EWS-FLll)
• "Onion skin" periosteal reaction in bone
• Aggressive with early metastases, but responsive to chemotherapy
• Chronic, slowly progressing autoimmune disease characterized by
lymphocytic infiltration of the exocrine glands resulting in xerostomia and
Sjogren syndrome dry eyes (keratoconjunctivitis sicca)
• (+) Anti-SSAand anti-SSB
• Affects Asian females < 40 years old.
• Aka "Pulseless disease" (weak upper extremity pulses), fever, night sweats,
Takayasu arteritis arthritis, myalgias, skin nodules, ocular disturbances.
• Granulomatous thickening and narrowing of aortic arch and proximal great
vessels.
• Risk factors: Heavy tobacco smoking history, males < 40 years old.
Buerger disease
[thromboangiitis • Symptoms include intermittent claudication
obliterans) • May lead to gangrene, autoamputation of digits, superficial nodular phlebitis.
Kawasaki disease • Usually Asian children < 4 years old.
(mucocutaneous
• Conjunctiva! injection, Rash, Cervical adenopathy, Strawberry tongue (oral
lymph node
mucositis), Hand foot changes (edema, erythema), fever
svndromel
• Recurrent aphthous ulcers, genital ulcerations, uveitis, erythema nodosum.
Beh~et syndrome Can be precipitated by HSVor parvovirus.
• Flares last 1-4 weeks.
• Presents with asthma, sinusitis, skin nodules or purpura, peripheral
Eosinophilic neuropathy
granu.Jomatosis with • Can also involve heart, GI,kidneys (pauciimmune glomerulonephritis).
polyangiitis (formerly Churg-
• (+) MPO-ANCA/p-ANCA
Strauss syndrome)
• Increased lgE levels
• Most common childhood systemic vasculitis.
lmmunoglobulin A • Often follows URI
Vasculitis (Formerly called • Triad of Arthralgias, Stomach pain (abdominal pain associated with
Henoch-Schonlein purpura) intussusception), Palpable purpura on buttocks/legs
• Vasculitis secondary to lgA immune complex deposition
• Decreased blood flow to skin due to arteriolar (small vessel) vasospasm in
response to cold or stress
Raynaud phenomenon • Sequence of color change from white (ischemia) to blue (hypoxia) to red
(reperfusion)
• Most often in the fingers and toes
• Fever, bullae formation and necrosis, sloughing of skin at dermal-epidermal
junction
• (+) Nikolsky sign, high mortality rate.
Stevens-Johnson • Usually associated with adverse drug reaction
syndrome
• Toxic epidermal necrolysis (TEN) is more severe form of SJSinvolving> 30%
body surface area.
0 10-30% involvement SJS-TEN
• Broca area found at inferior frontal gyrus of frontal lobe. Defective language
Broca's aphasia (expressive) production
• Wernicke area is found at the superior temporal gyrus of temporal lobe
Wernicke's aphasia (receptive)
• Impaired language comprehension. Patients do not have insight
• Associated with chronic hypertension
Charcot-Bouchard
• Affects small vessels (eg, lenticulostriate arteries in basal ganglia, thalamus)
microaneurysm
and can cause hemorrhagic intraparenchymal hemorrhage
• Characterized by
0 Tremor (pill-rolling tremor at rest)
0 Rigidity (cogwheel)
0 Akinesia (or bradykinesia)
Parkinson disease 0 Postural instability
0 Shuffling gait
0 Small handwriting (micrographia)
• Results from Loss of dopaminergic neurons (ie, depigmentation) of
substantia nigra pars compacta
• Autosomal dominant trinucleotide (CAG)repeat expansion in the huntingtin
Huntington disease (HTT) gene on chromosome 4
• Symptoms include: chorea, athetosis, aggression, depression, dementia
• Most common cause of dementia in elderly
• Widespread cortical atrophy especially hippocampus. Narrowing of gyri and
widening of sulci
Alzheimer disease • (+) Neurofibrillary tangles: intracellular, hyperphosphorylated tau protein =
insoluble cytoskeletal elements
• (+)Hirano bodies-intracellular eosinophilic proteinaceous rods in
hippocampus.
• Presents with visual hallucinations and dementia with fluctuating cognition/
Lewy body dementia alertness, REMsleep behavior disorder, and parkinsonism
• Histologic/ Gross findings: Intracellular Lewy bodies in cortex -
• Aka Hereditary motor and sensory neuropathy.
Charcot-Marie-Tooth • Group of progressive hereditary nerve disorders related to the defective
disease production of proteins involved in the structure and function of peripheral
nerves or the myelin sheath
• Autosomal dominant
von Hippel-Lindau • Deletion ofVHL gene on chromosome 3p.
disease • Associated with Hemangioblastomas in retina, brain stem, cerebellum, spine;
Angiomatosis; bilateral Renal cell carcinomas; Pheochromocytomas
• Most common cause of peripheral facial palsy
Bell palsy
• Usually after HSVreactivation
• Aka relative afferent pupillary defect (RAPD)
• When the light shines into a normal eye, constriction of the ipsilateral (direct
reflex) and contralateral eye (consensual reflex) is observed. When the light
Marcus Gunn pupil is then swung to the affected eye, both pupils dilate instead of constrict due to
impaired conduction of light signal along the injured optic nerve
• Associated with optic neuritis (eg, multiple sclerosis), optic neuropathies (eg,
giant cell arteritis).
• Ptosis (slight drooping of eyelid: superior tarsal muscle)
Horner syndrome • Anhidrosis (absence of sweating) and flushing of affected side of face
• Miosis (pupil constriction)
• Reabsorption defect in PCT which leads to excretion of amino acids, glucose,
Fanconi syndrome HCO,·, and P0, 3 ·, and all substances reabsorbed by the PCT
• Reabsorption defect in thick ascending loop of Henle (affects Na+/K+/2CI-
Bartter syndrome cotransporter)
• Reabsorption defect of NaCl in OCT
Gitelman syndrome • Presents similarly to lifelong thiazide diuretic use
• Less severe than Bartter syndrome
• Gain of function mutation leading to decreased Na+ channel degradation
Liddle syndrome leading to increased Na+ reabsorption in collecting tubules
lgA nephropathy • Episodic hematuria occurring with respiratory or GI tract infections
(Berger disease) • On light microscopy, shows mesangial proliferation
• Mutation in type IVcollagen leads to irregular thinning and thickening and
splitting of glomerular basement membrane
Alport syndrome • Most commonly X-linked dominant
• Presents with eye problems (eg, retinopathy, anterior lenticonus),
glornerulonephritis, sensorineural deafness
Miillerian agenesis (Mayer • May present as primary amenorrhea (due to a lack of uterine development)
Rokitansky- Kiister-Hauser in females with fully developed 2° sexual characteristics (functional ovaries)
svndromel
• Male, 4 7,XXY
• Characteristic findings include: testicular atrophy (small, firm testes), tall
Klinefelter syndrome stature with eunuchoid proportions, gynecomastia, female hair distribution
• May present with developmental delay
• Female, 45,XO
• Characteristic findings include: short stature, ovarian dysgenesis (streak
ovary), shield chest, bicuspid aortic valve, coarctation of the aorta, lymphatic
Turner syndrome defects (result in webbed neck or cystic hygroma; lymphederna in feet,
hands), horseshoe kidney, high-arched palate, shortened 4th metacarpals
• Most common cause of primary amenorrhea
• A form of hypogonadotropic hypogonadism
• Defective migration of neurons and subsequent failure of olfactory bulbs to
Kallmann syndrome
develop lead to decreased synthesis of GnRH in the hypothalamus;
hyposrnia/anosmia; infertility
• lntraepithelial adenocarcinoma
• Carcinoma in situ
Extramammary Paget
• Has low risk of underlying carcinoma versus Paget disease of the breast,
disease
which is always associated with underlying carcinoma)
• Symptoms include pruritus, erythema, crusting, ulcers
• Adhesions and/or fibrosis of the endometrium often associated with dilation
and curettage of intrauterine pregnancy
Asherman syndrome
• Symptoms and signs include decreased fertility, recurrent pregnancy loss,
AUB,pelvic pain
• HGPRTdeficiency, X-linked recessive
Lesch-Nyhan syndrome
• Present with gout, intellectual disability, self-mutilating behavior in a boy
• Dynein arm defect affecting cilia
Kartagener syndrome
• Situs inversus, chronic ear infections, sinusitis, bronchiectasis, infertility
• Type V collagen defect
Ehlers-Danlos syndrome • Type Ill collagen defect seen in vascular subtype of ED
• Elastic skin, hypermobility of joints, increased bleeding tendency
• Fibrillin defect
Marfan syndrome • Arachnodactyly, lens dislocation (upward and temporal), aortic dissection,
hyperflexible joints
• Gs-protein activating mutation
McCune-Albright syndrome • CafE!-au-laitspots (unilateral), polyostotic fibrous dysplasia, precocious
puberty, multiple endocrine abnormalities
• X-linked recessive frameshift mutation of dystrophin gene
Duchenne muscular dystrophy
• Presents with calf pseudohypertrophy
Gowers sign • Child uses arms to stand up from squat
• X-linked non-frameshift deletions in dystrophin
Becker muscular dystrophy • Less severe than Duchenne
• Slow, progressive muscle weakness in boys

You might also like